Sunteți pe pagina 1din 266

G.R. No.

L-23145 November 29, 1968 It is its view, therefore, that under the circumstances, the stock certificates cannot be declared or
considered as lost. Moreover, it would allege that there was a failure to observe certain
requirements of its by-laws before new stock certificates could be issued. Hence, its appeal.
TESTATE ESTATE OF IDONAH SLADE PERKINS, deceased. RENATO D. TAYAG, ancillary
administrator-appellee,
vs. As was made clear at the outset of this opinion, the appeal lacks merit. The challenged order
BENGUET CONSOLIDATED, INC., oppositor-appellant. constitutes an emphatic affirmation of judicial authority sought to be emasculated by the wilful
conduct of the domiciliary administrator in refusing to accord obedience to a court decree. How,
then, can this order be stigmatized as illegal?
FERNANDO, J.:

As is true of many problems confronting the judiciary, such a response was called for by the
Confronted by an obstinate and adamant refusal of the domiciliary administrator, the County
realities of the situation. What cannot be ignored is that conduct bordering on wilful defiance, if it
Trust Company of New York, United States of America, of the estate of the deceased Idonah
had not actually reached it, cannot without undue loss of judicial prestige, be condoned or
Slade Perkins, who died in New York City on March 27, 1960, to surrender to the ancillary
tolerated. For the law is not so lacking in flexibility and resourcefulness as to preclude such a
administrator in the Philippines the stock certificates owned by her in a Philippine corporation,
solution, the more so as deeper reflection would make clear its being buttressed by indisputable
Benguet Consolidated, Inc., to satisfy the legitimate claims of local creditors, the lower court,
principles and supported by the strongest policy considerations.
then presided by the Honorable Arsenio Santos, now retired, issued on May 18, 1964, an order
of this tenor: "After considering the motion of the ancillary administrator, dated February 11,
1964, as well as the opposition filed by the Benguet Consolidated, Inc., the Court hereby (1) It can truly be said then that the result arrived at upheld and vindicated the honor of the judiciary
considers as lost for all purposes in connection with the administration and liquidation of the no less than that of the country. Through this challenged order, there is thus dispelled the
Philippine estate of Idonah Slade Perkins the stock certificates covering the 33,002 shares of atmosphere of contingent frustration brought about by the persistence of the domiciliary
stock standing in her name in the books of the Benguet Consolidated, Inc., (2) orders said administrator to hold on to the stock certificates after it had, as admitted, voluntarily submitted
certificates cancelled, and (3) directs said corporation to issue new certificates in lieu thereof, the itself to the jurisdiction of the lower court by entering its appearance through counsel on June 27,
same to be delivered by said corporation to either the incumbent ancillary administrator or to the 1963, and filing a petition for relief from a previous order of March 15, 1963.
Probate Division of this Court."1
Thus did the lower court, in the order now on appeal, impart vitality and effectiveness to what
From such an order, an appeal was taken to this Court not by the domiciliary administrator, the was decreed. For without it, what it had been decided would be set at naught and nullified.
County Trust Company of New York, but by the Philippine corporation, the Benguet Unless such a blatant disregard by the domiciliary administrator, with residence abroad, of what
Consolidated, Inc. The appeal cannot possibly prosper. The challenged order represents a was previously ordained by a court order could be thus remedied, it would have entailed, insofar
response and expresses a policy, to paraphrase Frankfurter, arising out of a specific problem, as this matter was concerned, not a partial but a well-nigh complete paralysis of judicial
addressed to the attainment of specific ends by the use of specific remedies, with full and ample authority.
support from legal doctrines of weight and significance.
1. Appellant Benguet Consolidated, Inc. did not dispute the power of the appellee ancillary
The facts will explain why. As set forth in the brief of appellant Benguet Consolidated, Inc., administrator to gain control and possession of all assets of the decedent within the jurisdiction
Idonah Slade Perkins, who died on March 27, 1960 in New York City, left among others, two of the Philippines. Nor could it. Such a power is inherent in his duty to settle her estate and
stock certificates covering 33,002 shares of appellant, the certificates being in the possession of satisfy the claims of local creditors.5 As Justice Tuason speaking for this Court made clear, it is a
the County Trust Company of New York, which as noted, is the domiciliary administrator of the "general rule universally recognized" that administration, whether principal or ancillary, certainly
estate of the deceased.2 Then came this portion of the appellant's brief: "On August 12, 1960, "extends to the assets of a decedent found within the state or country where it was granted," the
Prospero Sanidad instituted ancillary administration proceedings in the Court of First Instance of corollary being "that an administrator appointed in one state or country has no power over
Manila; Lazaro A. Marquez was appointed ancillary administrator, and on January 22, 1963, he property in another state or country."6
was substituted by the appellee Renato D. Tayag. A dispute arose between the domiciary
administrator in New York and the ancillary administrator in the Philippines as to which of them
It is to be noted that the scope of the power of the ancillary administrator was, in an earlier case,
was entitled to the possession of the stock certificates in question. On January 27, 1964, the
set forth by Justice Malcolm. Thus: "It is often necessary to have more than one administration
Court of First Instance of Manila ordered the domiciliary administrator, County Trust Company,
of an estate. When a person dies intestate owning property in the country of his domicile as well
to "produce and deposit" them with the ancillary administrator or with the Clerk of Court. The
as in a foreign country, administration is had in both countries. That which is granted in the
domiciliary administrator did not comply with the order, and on February 11, 1964, the ancillary
jurisdiction of decedent's last domicile is termed the principal administration, while any other
administrator petitioned the court to "issue an order declaring the certificate or certificates of
administration is termed the ancillary administration. The reason for the latter is because a grant
stocks covering the 33,002 shares issued in the name of Idonah Slade Perkins by Benguet
of administration does not ex proprio vigore have any effect beyond the limits of the country in
Consolidated, Inc., be declared [or] considered as lost."3
which it is granted. Hence, an administrator appointed in a foreign state has no authority in the
[Philippines]. The ancillary administration is proper, whenever a person dies, leaving in a country
It is to be noted further that appellant Benguet Consolidated, Inc. admits that "it is immaterial" as other than that of his last domicile, property to be administered in the nature of assets of the
far as it is concerned as to "who is entitled to the possession of the stock certificates in question; deceased liable for his individual debts or to be distributed among his heirs." 7
appellant opposed the petition of the ancillary administrator because the said stock certificates
are in existence, they are today in the possession of the domiciliary administrator, the County
It would follow then that the authority of the probate court to require that ancillary administrator's
Trust Company, in New York, U.S.A...."4
right to "the stock certificates covering the 33,002 shares ... standing in her name in the books of
[appellant] Benguet Consolidated, Inc...." be respected is equally beyond question. For appellant
is a Philippine corporation owing full allegiance and subject to the unrestricted jurisdiction of 3. Appellant Benguet Consolidated, Inc. would seek to bolster the above contention by its
local courts. Its shares of stock cannot therefore be considered in any wise as immune from invoking one of the provisions of its by-laws which would set forth the procedure to be followed
lawful court orders. in case of a lost, stolen or destroyed stock certificate; it would stress that in the event of a
contest or the pendency of an action regarding ownership of such certificate or certificates of
stock allegedly lost, stolen or destroyed, the issuance of a new certificate or certificates would
Our holding in Wells Fargo Bank and Union v. Collector of Internal Revenue8 finds application.
await the "final decision by [a] court regarding the ownership [thereof]."15
"In the instant case, the actual situs of the shares of stock is in the Philippines, the corporation
being domiciled [here]." To the force of the above undeniable proposition, not even appellant is
insensible. It does not dispute it. Nor could it successfully do so even if it were so minded. Such reliance is misplaced. In the first place, there is no such occasion to apply such by-law. It is
admitted that the foreign domiciliary administrator did not appeal from the order now in question.
Moreover, there is likewise the express admission of appellant that as far as it is concerned, "it is
2. In the face of such incontrovertible doctrines that argue in a rather conclusive fashion for the
immaterial ... who is entitled to the possession of the stock certificates ..." Even if such were not
legality of the challenged order, how does appellant, Benguet Consolidated, Inc. propose to
the case, it would be a legal absurdity to impart to such a provision conclusiveness and finality.
carry the extremely heavy burden of persuasion of precisely demonstrating the contrary? It
Assuming that a contrariety exists between the above by-law and the command of a court
would assign as the basic error allegedly committed by the lower court its "considering as lost
decree, the latter is to be followed.
the stock certificates covering 33,002 shares of Benguet belonging to the deceased Idonah
Slade Perkins, ..."9 More specifically, appellant would stress that the "lower court could not
"consider as lost" the stock certificates in question when, as a matter of fact, his Honor the trial It is understandable, as Cardozo pointed out, that the Constitution overrides a statute, to which,
Judge knew, and does know, and it is admitted by the appellee, that the said stock certificates however, the judiciary must yield deference, when appropriately invoked and deemed
are in existence and are today in the possession of the domiciliary administrator in New York." 10 applicable. It would be most highly unorthodox, however, if a corporate by-law would be
accorded such a high estate in the jural order that a court must not only take note of it but yield
to its alleged controlling force.
There may be an element of fiction in the above view of the lower court. That certainly does not
suffice to call for the reversal of the appealed order. Since there is a refusal, persistently
adhered to by the domiciliary administrator in New York, to deliver the shares of stocks of The fear of appellant of a contingent liability with which it could be saddled unless the appealed
appellant corporation owned by the decedent to the ancillary administrator in the Philippines, order be set aside for its inconsistency with one of its by-laws does not impress us. Its
there was nothing unreasonable or arbitrary in considering them as lost and requiring the obedience to a lawful court order certainly constitutes a valid defense, assuming that such
appellant to issue new certificates in lieu thereof. Thereby, the task incumbent under the law on apprehension of a possible court action against it could possibly materialize. Thus far, nothing in
the ancillary administrator could be discharged and his responsibility fulfilled. the circumstances as they have developed gives substance to such a fear. Gossamer
possibilities of a future prejudice to appellant do not suffice to nullify the lawful exercise of
judicial authority.
Any other view would result in the compliance to a valid judicial order being made to depend on
the uncontrolled discretion of the party or entity, in this case domiciled abroad, which thus far
has shown the utmost persistence in refusing to yield obedience. Certainly, appellant would not 4. What is more the view adopted by appellant Benguet Consolidated, Inc. is fraught with
be heard to contend in all seriousness that a judicial decree could be treated as a mere scrap of implications at war with the basic postulates of corporate theory.
paper, the court issuing it being powerless to remedy its flagrant disregard.
We start with the undeniable premise that, "a corporation is an artificial being created by
It may be admitted of course that such alleged loss as found by the lower court did not operation of law...."16 It owes its life to the state, its birth being purely dependent on its will. As
correspond exactly with the facts. To be more blunt, the quality of truth may be lacking in such a Berle so aptly stated: "Classically, a corporation was conceived as an artificial person, owing its
conclusion arrived at. It is to be remembered however, again to borrow from Frankfurter, "that existence through creation by a sovereign power."17 As a matter of fact, the statutory language
fictions which the law may rely upon in the pursuit of legitimate ends have played an important employed owes much to Chief Justice Marshall, who in the Dartmouth College decision defined
part in its development."11 a corporation precisely as "an artificial being, invisible, intangible, and existing only in
contemplation of law."18
Speaking of the common law in its earlier period, Cardozo could state fictions "were devices to
advance the ends of justice, [even if] clumsy and at times offensive."12 Some of them have The well-known authority Fletcher could summarize the matter thus: "A corporation is not in fact
persisted even to the present, that eminent jurist, noting "the quasi contract, the adopted child, and in reality a person, but the law treats it as though it were a person by process of fiction, or by
the constructive trust, all of flourishing vitality, to attest the empire of "as if" today."13 He likewise regarding it as an artificial person distinct and separate from its individual stockholders.... It owes
noted "a class of fictions of another order, the fiction which is a working tool of thought, but its existence to law. It is an artificial person created by law for certain specific purposes, the
which at times hides itself from view till reflection and analysis have brought it to the light."14 extent of whose existence, powers and liberties is fixed by its charter." 19Dean Pound's terse
summary, a juristic person, resulting from an association of human beings granted legal
personality by the state, puts the matter neatly.20
What cannot be disputed, therefore, is the at times indispensable role that fictions as such
played in the law. There should be then on the part of the appellant a further refinement in the
catholicity of its condemnation of such judicial technique. If ever an occasion did call for the There is thus a rejection of Gierke's genossenchaft theory, the basic theme of which to quote
employment of a legal fiction to put an end to the anomalous situation of a valid judicial order from Friedmann, "is the reality of the group as a social and legal entity, independent of state
being disregarded with apparent impunity, this is it. What is thus most obvious is that this recognition and concession."21 A corporation as known to Philippine jurisprudence is a creature
particular alleged error does not carry persuasion. without any existence until it has received the imprimatur of the state according to law. It is
logically inconceivable therefore that it will have rights and privileges of a higher priority than that
of its creator. More than that, it cannot legitimately refuse to yield obedience to acts of its state That is all then that this case presents. It is obvious why the appeal cannot succeed. It is always
organs, certainly not excluding the judiciary, whenever called upon to do so. easy to conjure extreme and even oppressive possibilities. That is not decisive. It does not settle
the issue. What carries weight and conviction is the result arrived at, the just solution obtained,
grounded in the soundest of legal doctrines and distinguished by its correspondence with what a
As a matter of fact, a corporation once it comes into being, following American law still of
sense of realism requires. For through the appealed order, the imperative requirement of justice
persuasive authority in our jurisdiction, comes more often within the ken of the judiciary than the
according to law is satisfied and national dignity and honor maintained.
other two coordinate branches. It institutes the appropriate court action to enforce its right.
Correlatively, it is not immune from judicial control in those instances, where a duty under the
law as ascertained in an appropriate legal proceeding is cast upon it. WHEREFORE, the appealed order of the Honorable Arsenio Santos, the Judge of the Court of
First Instance, dated May 18, 1964, is affirmed. With costs against oppositor-appelant Benguet
Consolidated, Inc.
To assert that it can choose which court order to follow and which to disregard is to confer upon
it not autonomy which may be conceded but license which cannot be tolerated. It is to argue that
it may, when so minded, overrule the state, the source of its very existence; it is to contend that
what any of its governmental organs may lawfully require could be ignored at will. So
extravagant a claim cannot possibly merit approval.

5. One last point. In Viloria v. Administrator of Veterans Affairs,22 it was shown that in a
guardianship proceedings then pending in a lower court, the United States Veterans
Administration filed a motion for the refund of a certain sum of money paid to the minor under
guardianship, alleging that the lower court had previously granted its petition to consider the
deceased father as not entitled to guerilla benefits according to a determination arrived at by its
main office in the United States. The motion was denied. In seeking a reconsideration of such
order, the Administrator relied on an American federal statute making his decisions "final and
conclusive on all questions of law or fact" precluding any other American official to examine the
matter anew, "except a judge or judges of the United States court." 23 Reconsideration was
denied, and the Administrator appealed.

In an opinion by Justice J.B.L. Reyes, we sustained the lower court. Thus: "We are of the
opinion that the appeal should be rejected. The provisions of the U.S. Code, invoked by the
appellant, make the decisions of the U.S. Veterans' Administrator final and conclusive when
made on claims property submitted to him for resolution; but they are not applicable to the
present case, where the Administrator is not acting as a judge but as a litigant. There is a great
difference between actions against the Administrator (which must be filed strictly in accordance
with the conditions that are imposed by the Veterans' Act, including the exclusive review by
United States courts), and those actions where the Veterans' Administrator seeks a remedy from
our courts and submits to their jurisdiction by filing actions therein. Our attention has not been
called to any law or treaty that would make the findings of the Veterans' Administrator, in actions
where he is a party, conclusive on our courts. That, in effect, would deprive our tribunals of
judicial discretion and render them mere subordinate instrumentalities of the Veterans'
Administrator."

It is bad enough as the Viloria decision made patent for our judiciary to accept as final and
conclusive, determinations made by foreign governmental agencies. It is infinitely worse if
through the absence of any coercive power by our courts over juridical persons within our
jurisdiction, the force and effectivity of their orders could be made to depend on the whim or
caprice of alien entities. It is difficult to imagine of a situation more offensive to the dignity of the
bench or the honor of the country.

Yet that would be the effect, even if unintended, of the proposition to which appellant Benguet
Consolidated seems to be firmly committed as shown by its failure to accept the validity of the
order complained of; it seeks its reversal. Certainly we must at all pains see to it that it does not
succeed. The deplorable consequences attendant on appellant prevailing attest to the necessity
of negative response from us. That is what appellant will get.
GR No. 46997 January 11, 1940 (Arts. 18, 60, 69, Law No. 1956; Dharmdas v. Buenaflor, 57 Jur. Fil., 505; Cu Unjeng e
Hijos v Mitchell, 58 Jur. Fil., 515; Bastida contra Penalosa, 30 Jur. Fil. , 156; De
Amuzategui v Macleod, 33 Jur. Fil., 85).
WISE & COMPANY, INC., Plaintiff-appellant,
vs.
MAN SUN LUNG, defendant. As for the second alleged error, which is alleged to have been committed by the Trubunal a quo,
THE SHERIFF PROVINCIAL DE CAMARINES SUR, defendant-appealed. having been illegal the seizure of the assets here treated as the property of the defendant Man
Sun Lung, not always, the lower court did not incur in error when ordering the lifting of the
embargo on them and their delivery to the insolvency syndicate of Man Sun Lung & Co., Inc.
VILLA-REAL, J .:

Due to the above considerations, we are of opinion and thus we declare (1) that the articles or
This is an appeal filed by the plaintiff, Wise & Company, Inc., against the order of the Manila
movable goods that are found in the store of a corporation, are presumed to be the property of
Court of First Instance denying the motion, filed by said plaintiff on February 15, 1937, in which
the corporation and not of any member of the corporation to unless proven otherwise; (2) that
requested that the Provincial Sheriff of Camarines Sur be ordered to sell the different properties
the seizure locked by the Sheriff on such movable property, by virtue of an injunction
seized by him and the product of the sale be delivered to said claimant.
issued against a member thereof, not having proven that they belong exclusively to said
member, as illegal; and (3) that it has issued the embargo order, it is not beyond its jurisdiction
In support of its appeal, the repeated complainant entity, Wise & Company, Inc., notes the when ordering the lifting of the same and the delivery of the seized assets to the insolvency
following errors of law as committed by the Court to quo , namely: trustee of said corporation.

1. The lower court erred in failing to distinguish between the natural person, Man Sun By virtue of it, finding no error in the instant appeal, we confirm it in all its parts with the costs to
Lung from the corporation, Man Sun Lung & Co., Inc. the appellant.

2. The lower court erred in gratuitously relinguishing its custody of the goods put under
preliminary attachment to the prejudice of Wise & Company, Inc.

The necessary and pertinent facts for the resolution of the questions of law raised in this appeal
and the only ones that we can take into account are those that are reported in the appealed car,
namely:

It appears in the Sheriff's report on the back of the order of execution Exhibit B that he
delivered said goods to the Insolvency Syndicate of Man Sun Lung & Co., Inc.,
because he had seized such effects in the Man Sun store Lung & Co., Inc. It also
appears in the Sheriff's reply that he made said delivery under the order of the Court
of Camarines Sur and that said goods have been sold by the trustee by order of the
Court in said Insolvency Matter and its amount of P656.64 is held by the notary, by
order of the Court of Camarines Sur dated January 20, 1937.

Undoubtedly, Man Sun Lung, the defendant in this case, is legally different and distinct from Man
Sun Lung & Co., Inc., declared insolvent in the corresponding insolvency file: the first is a natural
person and the last is a Legal person with a distinct and independent personality from that of the
person, Not realizing that the assets seized by the Sheriff of Camarines Sur, for believing that
they were the property of the defendant Man Sun Lung, were really of this, the presumption is
that they belonged to Man Sun Lung & Co., Inc., as part of the assets of that business
entity. Having appointed a trustee to possess and insure the assets of the insolvent, subject to
the orders of the Court that he knew of the insolvency, only this had jurisdiction to order the
disposition that should be made of such goods. Having seized the Provincial Sheriff of
Camarines Sur of the assets under discussion as the property of the defendant Man Un Lung
and not even includingprima facie , that they were of this, but of the mercantile entity, Man Sun
Lung & Co., Inc., in doing so its seizure was illegal, and the Court quo work within its jurisdiction
when ordering the delivery of said goods to the trustee of the aforementioned insolvency, who
then sold them by order of the Court of repeated insolvency, depositing the proceeds of the sale,
which amounted to P656.64, by order of the same Court of Camarines Sur dated January 20,
1937, held of the scribe of the same court.
G.R. No. 90580 April 8, 1991 The well-known rule that shareholders cannot ordinarily sue in equity to redress
wrongs done to the corporation, but that the action must be brought by the Board of
Directors, . . . has its exceptions. (If the corporation [were] under the complete control
RUBEN SAW, DIONISIO SAW, LINA S. CHUA, LUCILA S. RUSTE AND EVELYN
of the principal defendants, . . . it is obvious that a demand upon the Board of
SAW, petitioners,
Directors to institute action and prosecute the same effectively would have been
vs.
useless, and the law does not require litigants to perform useless acts.
HON. COURT OF APPEALS, HON. BERNARDO P. PARDO, Presiding Judge of Branch 43,
(Regional Trial Court of Manila), FREEMAN MANAGEMENT AND DEVELOPMENT
CORPORATION, EQUITABLE BANKING CORPORATION, FREEMAN INCORPORATED, Equitable demurs, contending that the collection suit against Freeman, Inc, and Saw Chiao Lian
SAW CHIAO LIAN, THE REGISTER OF DEEDS OF CALOOCAN CITY, and DEPUTY is essentially in personam and, as an action against defendants in their personal capacities, will
SHERIFF ROSALIO G. SIGUA, respondents. not prejudice the petitioners as stockholders of the corporation. The Everett case is not
applicable because it involved an action filed by the minority stockholders where the board of
directors refused to bring an action in behalf of the corporation. In the case at bar, it was
CRUZ, J.:
Freeman, Inc. that was being sued by the creditor bank.

A collection suit with preliminary attachment was filed by Equitable Banking Corporation against
Equitable also argues that the subject matter of the intervention falls properly within the original
Freeman, Inc. and Saw Chiao Lian, its President and General Manager. The petitioners moved
and exclusive jurisdiction of the Securities and Exchange Commission under P.D. No. 902-A. In
to intervene, alleging that (1) the loan transactions between Saw Chiao Lian and Equitable
fact, at the time the motion for intervention was filed, there was pending between Freeman, Inc.
Banking Corp. were not approved by the stockholders representing at least 2/3 of corporate
and the petitioners SEC Case No. 03577 entitled "Dissolution, Accounting, Cancellation of
capital; (2) Saw Chiao Lian had no authority to contract such loans; and (3) there was collusion
Certificate of Registration with Restraining Order or Preliminary Injunction and Appointment of
between the officials of Freeman, Inc. and Equitable Banking Corp. in securing the loans. The
Receiver." It also avers in its Comment that the intervention of the petitioners could have only
motion to intervene was denied, and the petitioners appealed to the Court of Appeals.
caused delay and prejudice to the principal parties.

Meanwhile, Equitable and Saw Chiao Lian entered into a compromise agreement which they
On the second assignment of error, Equitable maintains that the petitioners' appeal could only
submitted to and was approved by the lower court. But because it was not complied with,
apply to the denial of their motion for intervention and not to the main case because their
Equitable secured a writ of execution, and two lots owned by Freeman, Inc. were levied upon
personality as party litigants had not been recognized by the trial court.
and sold at public auction to Freeman Management and Development Corp.

After examining the issues and arguments of the parties, the Court finds that the respondent
The Court of Appeals1 sustained the denial of the petitioners' motion for intervention, holding that
court committed no reversible error in sustaining the denial by the trial court of the petitioners'
"the compromise agreement between Freeman, Inc., through its President, and Equitable
motion for intervention.
Banking Corp. will not necessarily prejudice petitioners whose rights to corporate assets are at
most inchoate, prior to the dissolution of Freeman, Inc. . . . And intervention under Sec. 2, Rule
12 of the Revised Rules of Court is proper only when one's right is actual, material, direct and In the case of Magsaysay-Labrador v. Court of Appeals,3 we ruled as follows:
immediate and not simply contingent or expectant."
Viewed in the light of Section 2, Rule 12 of the Revised Rules of Court, this Court
It also ruled against the petitioners' argument that because they had already filed a notice of affirms the respondent court's holding that petitioners herein have no legal interest in
appeal, the trial judge had lost jurisdiction over the case and could no longer issue the writ of the subject matter in litigation so as to entitle them to intervene in the proceedings
execution. below. In the case of Batama Farmers' Cooperative Marketing Association, Inc. v.
Rosal, we held: "As clearly stated in Section 2 of Rule 12 of the Rules of Court, to be
permitted to intervene in a pending action, the party must have a legal interest in the
The petitioners are now before this Court, contending that:
matter in litigation, or in the success of either of the parties or an interest against both,
or he must be so situated as to be adversely affected by a distribution or other
1. The Honorable Court of Appeals erred in holding that the petitioners cannot disposition of the property in the custody of the court or an officer thereof."
intervene in Civil Case No. 88-44404 because their rights as stockholders of Freeman
are merely inchoate and not actual, material, direct and immediate prior to the
To allow intervention, [a] it must be shown that the movant has legal interest in the
dissolution of the corporation;
matter in litigation, or otherwise qualified; and [b] consideration must be given as to
whether the adjudication of the rights of the original parties may be delayed or
2. The Honorable Court of Appeals erred in holding that the appeal of the petitioners in prejudiced, or whether the intervenor's rights may be protected in a separate
said Civil Case No. 88-44404 was confined only to the order denying their motion to proceeding or not. Both requirements must concur as the first is not more important
intervene and did not divest the trial court of its jurisdiction over the whole case. than the second.

The petitioners base their right to intervene for the protection of their interests as stockholders The interest which entitles a person to intervene in a suit between other parties must
on Everett v. Asia Banking Corp.2 where it was held: be in the matter in litigation and of such direct and immediate character that the
intervenor will either gain or lose by the direct legal operation and effect of the
judgment. Otherwise, if persons not parties of the action could be allowed to intervene,
proceedings will become unnecessarily complicated, expensive and interminable. And That right of the intervenor should merely be in aid of the right of the original
this is not the policy of the law. party, like the plaintiffs in this case. As this right of the plaintiffs had ceased
to exist, there is nothing to aid or fight for. So the right of intervention has
ceased to exist.
The words "an interest in the subject" mean a direct interest in the cause of action as
pleaded, and which would put the intervenor in a legal position to litigate a fact alleged
in the complaint, without the establishment of which plaintiff could not recover. Consequently, it will be illogical and of no useful purpose to grant or even consider
further herein petitioner's prayer for the issuance of a writ of mandamus to compel the
lower court to allow and admit the petitioner's complaint in intervention. The dismissal
Here, the interest, if it exists at all, of petitioners-movants is indirect, contingent,
of the expropriation case has no less the inherent effect of also dismissing the motion
remote, conjectural, consequential and collateral. At the very least, their interest is
for intervention which is but the unavoidable consequence.
purely inchoate, or in sheer expectancy of a right in the management of the
corporation and to share in the profits thereof and in the properties and assets thereof
on dissolution, after payment of the corporate debts and obligations. The Court observes that even with the denial of the petitioners' motion to intervene, nothing is
really lost to them.The denial did not necessarily prejudice them as their rights are being litigated
in the case now before the Securities and Exchange Commission and may be fully asserted and
While a share of stock represents a proportionate or aliquot interest in the property of
protected in that separate proceeding.
the corporation, it does not vest the owner thereof with any legal right or title to any of
the property, his interest in the corporate property being equitable or beneficial in
nature. Shareholders are in no legal sense the owners of corporate property, which is WHEREFORE, the petition is DENIED, with costs against the petitioners. It is so ordered.
owned by the corporation as a distinct legal person.

On the second assignment of error, the respondent court correctly noted that the notice of
appeal was filed by the petitioners on October 24, 1988, upon the denial of their motion to
intervene, and the writ of execution was issued by the lower court on January 30, 1989. The
petitioners' appeal could not have concerned the "whole" case (referring to the decision)
because the petitioners "did not appeal the decision as indeed they cannot because they are not
parties to the case despite their being stockholders of respondent Freeman, Inc." They could
only appeal the denial of their motion for intervention as they were never recognized by the trial
court as party litigants in the main case.

Intervention is "an act or proceeding by which a third person is permitted to become a party to
an action or proceeding between other persons, and which results merely in the addition of a
new party or parties to an original action, for the purpose of hearing and determining at the same
time all conflicting claims which may be made to the subject matter in litigation.4

It is not an independent proceeding, but an ancillary and supplemental one which, in the nature
of things, unless otherwise provided for by the statute or Rules of Court, must be in
subordination to the main proceeding.5 It may be laid down as a general rule that an intervenor
is limited to the field of litigation open to the original parties.6

In the case at bar, there is no more principal action to be resolved as a writ of execution had
already been issued by the lower court and the claim of Equitable had already been satisfied.
The decision of the lower court had already become final and in fact had already been enforced.
There is therefore no more principal proceeding in which the petitioners may intervene.

As we held in the case of Barangay Matictic v. Elbinias:7

An intervention has been regarded, as merely "collateral or accessory or ancillary to


the principal action and not an independent proceedings; and interlocutory proceeding
dependent on and subsidiary to, the case between the original parties." (Fransisco,
Rules of Court, Vol. 1, p. 721). With the final dismissal of the original action, the
complaint in intervention can no longer be acted upon. In the case of Clareza v.
Resales, 2 SCRA 455, 457-458, it was stated that:
G.R. No. L-48930 February 23, 1944 agreeable to that result, filed the present petition for certiorari (G.R. No. 48930) to review and
reverse the judgment of the Court of Appeals; and the plaintiff Francisco de Borja, excepting to
the resolution of the Court of Appeals whereby its original judgment was set aside and the case
ANTONIO VAZQUEZ, petitioner,
was ordered remanded to the court of origin for further proceedings, filed a cross-petition for
vs.
certiorari (G.R. No. 48931) to maintain the original judgment of the Court of Appeals.
FRANCISCO DE BORJA, respondent.

The original decision of the Court of Appeals and its subsequent resolutions on reconsideration
x---------------------------------------------------------x
read as follows:

G.R. No. L-48931 February 23, 1944


Es hecho no controvertido que el 25 de Febrero de 1932, el demandado-apelante
vendio al demandante 4,000 cavanes de palay al precio de P2.10 el cavan, de los
FRANCISCO DE BORJA, petitioner, cuales, dicho demandante solamente recibio 2,583 cavanes; y que asimismo recibio
vs. para su envase 4,000 sacos vacios. Esta provbado que de dichos 4,000 sacos vacios
ANTONIO VAZQUEZ, respondent. solamente se entregaron, 2,583 quedando en poder del demandado el resto, y cuyo
valor es el de P0.24 cada uno. Presentada la demanda contra los demandados
Antonio Vazquez y Fernando Busuego para el pago de la cantidad de P4,702.70, con
OZAETA, J.:
sus intereses legales desde el 1.o de marzo de 1932 hasta su completo pago y las
costas, el Juzgado de Primera Instancia de Manila el asunto condenando a Antonio
This action was commenced in the Court of First Instance of Manila by Francisco de Borja Vazquez a pagar al demandante la cantidad de P3,175.20, mas la cantidad de
against Antonio Vazquez and Fernando Busuego to recover from them jointly and severally the P377.50, con sus intereses legales, absolviendo al demandado Fernando Busuego de
total sum of P4,702.70 upon three alleged causes of action, to wit: First, that in or about the la demanda y al demandante de la reconvencion de los demandados, sin especial
month of January, 1932, the defendants jointly and severally obligated themselves to sell to the pronunciamiento en cuanto a las costas. De dicha decision apelo el demandado
plaintiff 4,000 cavans of palay at P2.10 per cavan, to be delivered during the month of February, Antonio Vazquez, apuntado como principal error el de que el habia sido condenado
1932, the said defendants having subsequently received from the plaintiff in virtue of said personalmente, y no la corporacion por el representada.
agreement the sum of P8,400; that the defendants delivered to the plaintiff during the months of
February, March, and April, 1932, only 2,488 cavans of palay of the value of P5,224.80 and
Segun la preponderancia de las pruebas, la venta hecha por Antonio Vazquez a favor
refused to deliver the balance of 1,512 cavans of the value of P3,175.20 notwithstanding de Francisco de Borja de los 4,000 cavanes de palay fue en su capacidad de
repeated demands. Second, that because of defendants' refusal to deliver to the plaintiff the said Presidente interino y Manager de la corporacion Natividad-Vazquez Sabani
1,512 cavans of palay within the period above mentioned, the plaintiff suffered damages in the
Development Co., Inc. Asi resulta del Exh. 1, que es la copia al carbon del recibo
sum of P1,000. And, third, that on account of the agreement above mentioned the plaintiff otorgado por el demandado Vazquez, y cuyo original lo habia perdido el demandante,
delivered to the defendants 4,000 empty sacks, of which they returned to the plaintiff only 2,490 segun el. Asi tambien consta en los libros de la corporacion arriba mencionada,
and refused to deliver to the plaintiff the balance of 1,510 sacks or to pay their value amounting puesto que en los mismos se ha asentado tanto la entrada de los P8,400, precio del
to P377.50; and that on account of such refusal the plaintiff suffered damages in the sum of palay, como su envio al gobierno en pago de los alquileres de la Hacienda Sabani.
P150. Asi mismo lo admitio Francisco de Borja al abogado Sr. Jacinto Tomacruz, posterior
presidente de la corporacion sucesora en el arrendamiento de la Sabani Estate,
The defendant Antonio Vazquez answered the complaint, denying having entered into the cuando el solicito sus buenos oficios para el cobro del precio del palay no entregado.
contract mentioned in the first cause of action in his own individual and personal capacity, either Asi igualmente lo declaro el que hizo entrega de parte del palay a Borja, Felipe
solely or together with his codefendant Fernando Busuego, and alleging that the agreement for Veneracion, cuyo testimonio no ha sido refutado. Y asi se deduce de la misma
the purchase of 4,000 cavans of palay and the payment of the price of P8,400 were made by the demanda, cuando se incluyo en ella a Fernando Busuego, tesorero de la Natividad-
plaintiff with and to the Natividad-Vasquez Sabani Development Co., Inc., a corporation Vazquez Sabani Development Co., Inc.
organized and existing under the laws of the Philippines, of which the defendant Antonio
Vazquez was the acting manager at the time the transaction took place. By way of counterclaim, Siendo esto asi, la principal responsable debe ser la Natividad-Vazquez Sabani
the said defendant alleged that he suffered damages in the sum of P1,000 on account of the
Development Co., Inc., que quedo insolvente y dejo de existir. El Juez sentenciador
filing of this action against him by the plaintiff with full knowledge that the said defendant had declaro, sin embargo, al demandado Vazquez responsable del pago de la cantidad
nothing to do whatever with any and all of the transactions mentioned in the complaint in his own reclamada por su negligencia al vender los referidos 4,000 cavanes de palay sin
individual and personal capacity.
averiguar antes si o no dicha cantidad existia en las bodegas de la corporacion.

The trial court rendered judgment ordering the defendant Antonio Vazquez to pay to the plaintiff Resulta del Exh. 8 que despues de la venta de los 4,000 cavanes de palay a
the sum of P3,175.20 plus the sum of P377.50, with legal interest on both sums, and absolving
Francisco de Borja, el mismo demandado vendio a Kwong Ah Phoy 1,500 cavanes al
the defendant Fernando Busuego (treasurer of the corporation) from the complaint and the precio de P2.00 el cavan, y decimos 'despues' porque esta ultima venta aparece
plaintiff from the defendant Antonio Vazquez' counterclaim. Upon appeal to the Court of asentada despues de la primera. Segun esto, el apelante no solamente obro con
Appeals, the latter modified that judgment by reducing it to the total sum of P3,314.78, with legal
negligencia, sino interviniendo culpa de su parte, por lo que de acuerdo con los arts.
interest thereon and the costs. But by a subsequent resolution upon the defendant's motion for 1102, 1103 y 1902 del Codigo Civil, el debe ser responsable subsidiariamente del
reconsideration, the Court of Appeals set aside its judgment and ordered that the case be pago de la cantidad objecto de la demanda.
remanded to the court of origin for further proceedings. The defendant Vazquez, not being
En meritos de todo lo expuesto, se confirma la decision apelada con la modificacion It is well known that a corporation is an artificial being invested by law with a personality of its
de que el apelante debe pagar al apelado la suma de P2,295.70 como valor de los own, separate and distinct from that of its stockholders and from that of its officers who manage
1,417 cavanes de palay que dejo de entregar al demandante, mas la suma de and run its affairs. The mere fact that its personality is owing to a legal fiction and that it
P339.08 como importe de los 1,417 sacos vacios, que dejo de devolver, a razon de necessarily has to act thru its agents, does not make the latter personally liable on a contract
P0.24 el saco, total P3,314.78, con sus intereses legales desde la interposicion de la duly entered into, or for an act lawfully performed, by them for an in its behalf. The legal fiction
demanda y las costas de ambas instancias. by which the personality of a corporation is created is a practical reality and necessity. Without it
no corporate entities may exists and no corporate business may be transacted. Such legal fiction
may be disregarded only when an attempt is made to use it as a cloak to hide an unlawful or
Vista la mocion de reconsideracion de nuestra decision de fecha 13 de Octubre de
fraudulent purpose. No such thing has been alleged or proven in this case. It has not been
1942, y alegandose en la misma que cuando el apelante vendio los 1,500 cavanes de
alleged nor even intimated that Vazquez personally benefited by the contract of sale in question
palay a Ah Phoy, la corporacion todavia tenia bastante existencia de dicho grano, y no
and that he is merely invoking the legal fiction to avoid personal liability. Neither is it contended
estando dicho extremo suficientemente discutido y probado, y pudiendo variar el
that he entered into said contract for the corporation in bad faith and with intent to defraud the
resultado del asunto, dejamos sin efecto nuestra citada decision, y ordenamos la
plaintiff. We find no legal and factual basis upon which to hold him liable on the contract either
devolucion de la causa al Juzgado de origen para que reciba pruebas al efecto y dicte
principally or subsidiarily.
despues la decision correspondiente.

The trial court found him guilty of negligence in the performance of the contract and held him
Upon consideration of the motion of the attorney for the plaintiff-appellee in case CA-
personally liable on that account. On the other hand, the Court of Appeals found that he "no
G.R. No. 8676, Francisco de Borja vs. Antonio Vasquez et al., praying, for the reasons
solamente obro con negligencia, sino interveniendo culpa de su parte, por lo que de acuerdo
therein given, that the resolution of December 22, 1942, be reconsidered: Considering
con los arts. 1102, 1103 y 1902 del Codigo Civil, el debe ser responsable subsidiariamente del
that said resolution remanding the case to the lower court is for the benefit of the
pago de la cantidad objeto de la demanda." We think both the trial court and the Court of
plaintiff-appellee to afford him opportunity to refute the contention of the defendant-
Appeals erred in law in so holding. They have manifestly failed to distinguish a contractual from
appellant Antonio Vazquez, motion denied.
an extracontractual obligation, or an obligation arising from contract from an obligation arising
from culpa aquiliana. The fault and negligence referred to in articles 1101-1104 of the Civil Code
The action is on a contract, and the only issue pleaded and tried is whether the plaintiff entered are those incidental to the fulfillment or nonfullfillment of a contractual obligation; while the fault
into the contract with the defendant Antonio Vazquez in his personal capacity or as manager of or negligence referred to in article 1902 is the culpa aquiliana of the civil law, homologous but
the Natividad-Vazquez Sabani Development Co., Inc. The Court of Appeals found that according not identical to tort of the common law, which gives rise to an obligation independently of any
to the preponderance of the evidence "the sale made by Antonio Vazquez in favor of Francisco contract. (Cf. Manila R.R. Co. vs. Cia. Trasatlantica, 38 Phil., 875, 887-890; Cangco vs. Manila
de Borja of 4,000 cavans of palay was in his capacity as acting president and manager of the R.R. Co., 38 Phil. 768.) The fact that the corporation, acting thru Vazquez as its manager, was
corporation Natividad-Vazquez Sabani Development Co., Inc." That finding of fact is final and, it guilty of negligence in the fulfillment of the contract, did not make Vazquez principally or even
resolving the only issue involved, should be determinative of the result. subsidiarily liable for such negligence. Since it was the corporation's contract, its nonfulfillment,
whether due to negligence or fault or to any other cause, made the corporation and not its agent
liable.
The Court of Appeals doubly erred in ordering that the cause be remanded to the court of origin
for further trial to determine whether the corporation had sufficient stock of palay at the time
appellant sold, 1500 cavans of palay to Kwong Ah Phoy. First, if that point was material to the On the other hand if independently of the contract Vazquez by his fault or negligence cause
issue, it should have been proven during the trial; and the statement of the court that it had not damaged to the plaintiff, he would be liable to the latter under article 1902 of the Civil Code. But
been sufficiently discussed and proven was no justification for ordering a new trial, which, by the then the plaintiff's cause of action should be based on culpa aquiliana and not on the contract
way, neither party had solicited but against which, on the contrary, both parties now vehemently alleged in his complaint herein; and Vazquez' liability would be principal and not merely
protest. Second, the point is, in any event, beside the issue, and this we shall now discuss in subsidiary, as the Court of Appeals has erroneously held. No such cause of action was alleged
connection with the original judgment of the Court of Appeals which the plaintiff cross-petitioner in the complaint or tried by express or implied consent of the parties by virtue of section 4 of
seeks to maintain. Rule 17. Hence the trial court had no jurisdiction over the issue and could not adjudicate upon it
(Reyes vs. Diaz, G.R. No. 48754.) Consequently it was error for the Court of Appeals to remand
the case to the trial court to try and decide such issue.
The action being on a contract, and it appearing from the preponderance of the evidence that
the party liable on the contract is the Natividad-Vazquez Sabani Development Co., Inc. which is
not a party herein, the complaint should have been dismissed. Counsel for the plaintiff, in his It only remains for us to consider petitioner's second assignment of error referring to the lower
brief as respondent, argues that altho by the preponderance of the evidence the trial court and courts' refusal to entertain his counterclaim for damages against the respondent Borja arising
the Court of Appeals found that Vazquez celebrated the contract in his capacity as acting from the bringing of this action. The lower courts having sustained plaintiff's action. The finding
president of the corporation and altho it was the latter, thru Vazquez, with which the plaintiff had of the Court of Appeals that according to the preponderance of the evidence the defendant
contracted and which, thru Vazquez, had received the sum of P8,400 from Borja, and altho that Vazquez celebrated the contract not in his personal capacity but as acting president and
was true from the point of view of a legal fiction, "ello no impede que tambien sea verdad lo manager of the corporation, does not warrant his contention that the suit against him is malicious
alegado en la demanda de que la misma persona de Vasquez fue la que contrato con Borja y and tortious; and since we have to decide defendant's counterclaim upon the facts found by the
que la misma persona de Vasquez fue quien recibio la suma de P8,400." But such argument is Court of Appeals, we find no sufficient basis upon which to sustain said counterclaim. Indeed,
invalid and insufficient to show that the president of the corporation is personally liable on the we feel that a a matter of moral justice we ought to state here that the indignant attitude adopted
contract duly and lawfully entered into by him in its behalf. by the defendant towards the plaintiff for having brought this action against him is in our
estimation not wholly right. Altho from the legal point of view he was not personally liable for the
fulfillment of the contract entered into by him on behalf of the corporation of which he was the
acting president and manager, we think it was his moral duty towards the party with whom he
contracted in said capacity to see to it that the corporation represented by him fulfilled the
contract by delivering the palay it had sold, the price of which it had already received. Recreant
to such duty as a moral person, he has no legitimate cause for indignation. We feel that under
the circumstances he not only has no cause of action against the plaintiff for damages but is not
even entitled to costs.

The judgment of the Court of Appeals is reversed, and the complaint is hereby dismissed,
without any finding as to costs.
G.R. No. L-19550 June 19, 1967 On March 22, 1962, this Court issued the writ of preliminary injunction prayed for in the petition.
However, by resolution dated June 29, 1962, the writ was partially lifted or dissolved, insofar as
the papers, documents and things seized from the offices of the corporations above mentioned
HARRY S. STONEHILL, ROBERT P. BROOKS, JOHN J. BROOKS and KARL
are concerned; but, the injunction was maintained as regards the papers, documents and things
BECK, petitioners,
found and seized in the residences of petitioners herein.7
vs.
HON. JOSE W. DIOKNO, in his capacity as SECRETARY OF JUSTICE; JOSE LUKBAN, in
his capacity as Acting Director, National Bureau of Investigation; SPECIAL Thus, the documents, papers, and things seized under the alleged authority of the warrants in
PROSECUTORS PEDRO D. CENZON, EFREN I. PLANA and MANUEL VILLAREAL, JR. and question may be split into two (2) major groups, namely: (a) those found and seized in the
ASST. FISCAL MANASES G. REYES; JUDGE AMADO ROAN, Municipal Court of Manila; offices of the aforementioned corporations, and (b) those found and seized in the residences of
JUDGE ROMAN CANSINO, Municipal Court of Manila; JUDGE HERMOGENES CALUAG, petitioners herein.
Court of First Instance of Rizal-Quezon City Branch, and JUDGE DAMIAN JIMENEZ,
Municipal Court of Quezon City, respondents.
As regards the first group, we hold that petitioners herein have no cause of action to assail the
legality of the contested warrants and of the seizures made in pursuance thereof, for the simple
CONCEPCION, C.J.: reason that said corporations have their respective personalities, separate and distinct from the
personality of herein petitioners, regardless of the amount of shares of stock or of the interest of
each of them in said corporations, and whatever the offices they hold therein may be. 8 Indeed, it
Upon application of the officers of the government named on the margin1 — hereinafter referred
is well settled that the legality of a seizure can be contested only by the party whose rights have
to as Respondents-Prosecutors — several judges2 — hereinafter referred to as Respondents-
been impaired thereby,9 and that the objection to an unlawful search and seizure is purely
Judges — issued, on different dates,3 a total of 42 search warrants against petitioners
personal and cannot be availed of by third parties. 10 Consequently, petitioners herein may not
herein4 and/or the corporations of which they were officers,5 directed to the any peace officer, to
validly object to the use in evidence against them of the documents, papers and things seized
search the persons above-named and/or the premises of their offices, warehouses and/or
from the offices and premises of the corporations adverted to above, since the right to object to
residences, and to seize and take possession of the following personal property to wit:
the admission of said papers in evidence belongs exclusively to the corporations, to whom the
seized effects belong, and may not be invoked by the corporate officers in proceedings against
Books of accounts, financial records, vouchers, correspondence, receipts, ledgers, them in their individual capacity. 11 Indeed, it has been held:
journals, portfolios, credit journals, typewriters, and other documents and/or papers
showing all business transactions including disbursements receipts, balance sheets
. . . that the Government's action in gaining possession of papers belonging to
and profit and loss statements and Bobbins (cigarette wrappers).
the corporation did not relate to nor did it affect the personal defendants. If these
papers were unlawfully seized and thereby the constitutional rights of or any one were
as "the subject of the offense; stolen or embezzled and proceeds or fruits of the offense," or invaded, they were the rights of the corporation and not the rights of the other
"used or intended to be used as the means of committing the offense," which is described in the defendants. Next, it is clear that a question of the lawfulness of a seizure can be
applications adverted to above as "violation of Central Bank Laws, Tariff and Customs Laws, raised only by one whose rights have been invaded. Certainly, such a seizure, if
Internal Revenue (Code) and the Revised Penal Code." unlawful, could not affect the constitutional rights of defendants whose property had
not been seized or the privacy of whose homes had not been disturbed; nor could they
claim for themselves the benefits of the Fourth Amendment, when its violation, if any,
Alleging that the aforementioned search warrants are null and void, as contravening the was with reference to the rights of another. Remus vs. United States (C.C.A.)291 F.
Constitution and the Rules of Court — because, inter alia: (1) they do not describe with 501, 511. It follows, therefore, that the question of the admissibility of the evidence
particularity the documents, books and things to be seized; (2) cash money, not mentioned in
based on an alleged unlawful search and seizure does not extend to the personal
the warrants, were actually seized; (3) the warrants were issued to fish evidence against the defendants but embraces only the corporation whose property was taken. . . . (A
aforementioned petitioners in deportation cases filed against them; (4) the searches and Guckenheimer & Bros. Co. vs. United States, [1925] 3 F. 2d. 786, 789, Emphasis
seizures were made in an illegal manner; and (5) the documents, papers and cash money
supplied.)
seized were not delivered to the courts that issued the warrants, to be disposed of in accordance
with law — on March 20, 1962, said petitioners filed with the Supreme Court this original action
for certiorari, prohibition, mandamus and injunction, and prayed that, pending final disposition of With respect to the documents, papers and things seized in the residences of petitioners herein,
the present case, a writ of preliminary injunction be issued restraining Respondents-Prosecutors, the aforementioned resolution of June 29, 1962, lifted the writ of preliminary injunction previously
their agents and /or representatives from using the effects seized as aforementioned or any issued by this Court, 12 thereby, in effect, restraining herein Respondents-Prosecutors from
copies thereof, in the deportation cases already adverted to, and that, in due course, thereafter, using them in evidence against petitioners herein.
decision be rendered quashing the contested search warrants and declaring the same null and
void, and commanding the respondents, their agents or representatives to return to petitioners
In connection with said documents, papers and things, two (2) important questions need be
herein, in accordance with Section 3, Rule 67, of the Rules of Court, the documents, papers,
settled, namely: (1) whether the search warrants in question, and the searches and seizures
things and cash moneys seized or confiscated under the search warrants in question.
made under the authority thereof, are valid or not, and (2) if the answer to the preceding
question is in the negative, whether said documents, papers and things may be used in
In their answer, respondents-prosecutors alleged, 6 (1) that the contested search warrants are evidence against petitioners herein.
valid and have been issued in accordance with law; (2) that the defects of said warrants, if any,
were cured by petitioners' consent; and (3) that, in any event, the effects seized are admissible
in evidence against herein petitioners, regardless of the alleged illegality of the aforementioned
searches and seizures.
Petitioners maintain that the aforementioned search warrants are in the nature of general Thus, the warrants authorized the search for and seizure of records pertaining to all business
warrants and that accordingly, the seizures effected upon the authority there of are null and void. transactions of petitioners herein, regardless of whether the transactions were legal or illegal.
In this connection, the Constitution 13provides: The warrants sanctioned the seizure of all records of the petitioners and the aforementioned
corporations, whatever their nature, thus openly contravening the explicit command of our Bill of
Rights — that the things to be seized be particularly described — as well as tending to defeat its
The right of the people to be secure in their persons, houses, papers, and effects
major objective: the elimination of general warrants.
against unreasonable searches and seizures shall not be violated, and no warrants
shall issue but upon probable cause, to be determined by the judge after examination
under oath or affirmation of the complainant and the witnesses he may produce, and Relying upon Moncado vs. People's Court (80 Phil. 1), Respondents-Prosecutors maintain that,
particularly describing the place to be searched, and the persons or things to be even if the searches and seizures under consideration were unconstitutional, the documents,
seized. papers and things thus seized are admissible in evidence against petitioners herein. Upon
mature deliberation, however, we are unanimously of the opinion that the position taken in the
Moncado case must be abandoned. Said position was in line with the American common law
Two points must be stressed in connection with this constitutional mandate, namely: (1) that no
rule, that the criminal should not be allowed to go free merely "because the constable has
warrant shall issue but upon probable cause, to be determined by the judge in the manner set
blundered," 16 upon the theory that the constitutional prohibition against unreasonable searches
forth in said provision; and (2) that the warrant shall particularly describe the things to be seized.
and seizures is protected by means other than the exclusion of evidence unlawfully
obtained, 17 such as the common-law action for damages against the searching officer, against
None of these requirements has been complied with in the contested warrants. Indeed, the the party who procured the issuance of the search warrant and against those assisting in the
same were issued upon applications stating that the natural and juridical person therein named execution of an illegal search, their criminal punishment, resistance, without liability to an
had committed a "violation of Central Ban Laws, Tariff and Customs Laws, Internal Revenue unlawful seizure, and such other legal remedies as may be provided by other laws.
(Code) and Revised Penal Code." In other words, no specific offense had been alleged in said
applications. The averments thereof with respect to the offense committed were abstract. As a
However, most common law jurisdictions have already given up this approach and eventually
consequence, it was impossible for the judges who issued the warrants to have found the
adopted the exclusionary rule, realizing that this is the only practical means of enforcing the
existence of probable cause, for the same presupposes the introduction of competent proof that
constitutional injunction against unreasonable searches and seizures. In the language of Judge
the party against whom it is sought has performed particular acts, or
Learned Hand:
committed specific omissions, violating a given provision of our criminal laws. As a matter of fact,
the applications involved in this case do not allege any specific acts performed by herein
petitioners. It would be the legal heresy, of the highest order, to convict anybody of a "violation of As we understand it, the reason for the exclusion of evidence competent as such,
Central Bank Laws, Tariff and Customs Laws, Internal Revenue (Code) and Revised Penal which has been unlawfully acquired, is that exclusion is the only practical way of
Code," — as alleged in the aforementioned applications — without reference to any determinate enforcing the constitutional privilege. In earlier times the action of trespass against the
provision of said laws or offending official may have been protection enough; but that is true no longer. Only in
case the prosecution which itself controls the seizing officials, knows that it cannot
profit by their wrong will that wrong be repressed.18
To uphold the validity of the warrants in question would be to wipe out completely one of the
most fundamental rights guaranteed in our Constitution, for it would place the sanctity of the
domicile and the privacy of communication and correspondence at the mercy of the whims In fact, over thirty (30) years before, the Federal Supreme Court had already declared:
caprice or passion of peace officers. This is precisely the evil sought to be remedied by the
constitutional provision above quoted — to outlaw the so-called general warrants. It is not
If letters and private documents can thus be seized and held and used in evidence
difficult to imagine what would happen, in times of keen political strife, when the party in power
against a citizen accused of an offense, the protection of the 4th Amendment,
feels that the minority is likely to wrest it, even though by legal means.
declaring his rights to be secure against such searches and seizures, is of no value,
and, so far as those thus placed are concerned, might as well be stricken from the
Such is the seriousness of the irregularities committed in connection with the disputed search Constitution. The efforts of the courts and their officials to bring the guilty to
warrants, that this Court deemed it fit to amend Section 3 of Rule 122 of the former Rules of punishment, praiseworthy as they are, are not to be aided by the sacrifice of those
Court 14 by providing in its counterpart, under the Revised Rules of Court 15 that "a search great principles established by years of endeavor and suffering which have resulted in
warrant shall not issue but upon probable cause in connection with one specific offense." Not their embodiment in the fundamental law of the land.19
satisfied with this qualification, the Court added thereto a paragraph, directing that "no search
warrant shall issue for more than one specific offense."
This view was, not only reiterated, but, also, broadened in subsequent decisions on the same
Federal Court. 20After reviewing previous decisions thereon, said Court held, in Mapp vs.
The grave violation of the Constitution made in the application for the contested search warrants Ohio (supra.):
was compounded by the description therein made of the effects to be searched for and seized,
to wit:
. . . Today we once again examine the Wolf's constitutional documentation of the right
of privacy free from unreasonable state intrusion, and after its dozen years on our
Books of accounts, financial records, vouchers, journals, correspondence, receipts, books, are led by it to close the only courtroom door remaining open to evidence
ledgers, portfolios, credit journals, typewriters, and other documents and/or papers secured by official lawlessness in flagrant abuse of that basic right, reserved to all
showing all business transactions including disbursement receipts, balance sheets persons as a specific guarantee against that very same unlawful conduct. We hold
and related profit and loss statements. that all evidence obtained by searches and seizures in violation of the Constitution is,
by that same authority, inadmissible in a State.
Since the Fourth Amendment's right of privacy has been declared enforceable against guarantee under consideration, overlooks the fact that violations thereof are, in general,
the States through the Due Process Clause of the Fourteenth, it is enforceable against committed By agents of the party in power, for, certainly, those belonging to the minority could
them by the same sanction of exclusion as it used against the Federal Government. not possibly abuse a power they do not have. Regardless of the handicap under which the
Were it otherwise, then just as without the Weeks rule the assurance against minority usually — but, understandably — finds itself in prosecuting agents of the majority, one
unreasonable federal searches and seizures would be "a form of words," valueless must not lose sight of the fact that the psychological and moral effect of the possibility 21 of
and underserving of mention in a perpetual charter of inestimable human liberties, so securing their conviction, is watered down by the pardoning power of the party for whose benefit
too, without that rule the freedom from state invasions of privacy would be so the illegality had been committed.
ephemeral and so neatly severed from its conceptual nexus with the freedom from all
brutish means of coercing evidence as not to permit this Court's high regard as a
In their Motion for Reconsideration and Amendment of the Resolution of this Court dated June
freedom "implicit in the concept of ordered liberty." At the time that the Court held in
29, 1962, petitioners allege that Rooms Nos. 81 and 91 of Carmen Apartments, House No.
Wolf that the amendment was applicable to the States through the Due Process
2008, Dewey Boulevard, House No. 1436, Colorado Street, and Room No. 304 of the Army-
Clause, the cases of this Court as we have seen, had steadfastly held that as to
Navy Club, should be included among the premises considered in said Resolution as residences
federal officers the Fourth Amendment included the exclusion of the evidence seized
of herein petitioners, Harry S. Stonehill, Robert P. Brook, John J. Brooks and Karl Beck,
in violation of its provisions. Even Wolf "stoutly adhered" to that proposition. The right
respectively, and that, furthermore, the records, papers and other effects seized in the offices of
to when conceded operatively enforceable against the States, was not susceptible of
the corporations above referred to include personal belongings of said petitioners and other
destruction by avulsion of the sanction upon which its protection and enjoyment had
effects under their exclusive possession and control, for the exclusion of which they have a
always been deemed dependent under the Boyd, Weeks and Silverthorne Cases.
standing under the latest rulings of the federal courts of federal courts of the United States. 22
Therefore, in extending the substantive protections of due process to all
constitutionally unreasonable searches — state or federal — it was logically and
constitutionally necessarily that the exclusion doctrine — an essential part of the right We note, however, that petitioners' theory, regarding their alleged possession of and control
to privacy — be also insisted upon as an essential ingredient of the right newly over the aforementioned records, papers and effects, and the alleged "personal" nature thereof,
recognized by the Wolf Case. In short, the admission of the new constitutional Right has Been Advanced, not in their petition or amended petition herein, but in the Motion for
by Wolf could not tolerate denial of its most important constitutional privilege, namely, Reconsideration and Amendment of the Resolution of June 29, 1962. In other words, said theory
the exclusion of the evidence which an accused had been forced to give by reason of would appear to be readjustment of that followed in said petitions, to suit the approach intimated
the unlawful seizure. To hold otherwise is to grant the right but in reality to withhold its in the Resolution sought to be reconsidered and amended. Then, too, some of the affidavits or
privilege and enjoyment. Only last year the Court itself recognized that the purpose of copies of alleged affidavits attached to said motion for reconsideration, or submitted in support
the exclusionary rule to "is to deter — to compel respect for the constitutional guaranty thereof, contain either inconsistent allegations, or allegations inconsistent with the theory now
in the only effectively available way — by removing the incentive to disregard it" . . . . advanced by petitioners herein.

The ignoble shortcut to conviction left open to the State tends to destroy the entire Upon the other hand, we are not satisfied that the allegations of said petitions said motion for
system of constitutional restraints on which the liberties of the people rest. Having reconsideration, and the contents of the aforementioned affidavits and other papers submitted in
once recognized that the right to privacy embodied in the Fourth Amendment is support of said motion, have sufficiently established the facts or conditions contemplated in the
enforceable against the States, and that the right to be secure against rude invasions cases relied upon by the petitioners; to warrant application of the views therein expressed,
of privacy by state officers is, therefore constitutional in origin, we can no longer permit should we agree thereto. At any rate, we do not deem it necessary to express our opinion
that right to remain an empty promise. Because it is enforceable in the same manner thereon, it being best to leave the matter open for determination in appropriate cases in the
and to like effect as other basic rights secured by its Due Process Clause, we can no future.
longer permit it to be revocable at the whim of any police officer who, in the name of
law enforcement itself, chooses to suspend its enjoyment. Our decision, founded on
reason and truth, gives to the individual no more than that which the Constitution We hold, therefore, that the doctrine adopted in the Moncado case must be, as it is hereby,
abandoned; that the warrants for the search of three (3) residences of herein petitioners, as
guarantees him to the police officer no less than that to which honest law enforcement
is entitled, and, to the courts, that judicial integrity so necessary in the true specified in the Resolution of June 29, 1962, are null and void; that the searches and seizures
administration of justice. (emphasis ours.) therein made are illegal; that the writ of preliminary injunction heretofore issued, in connection
with the documents, papers and other effects thus seized in said residences of herein petitioners
is hereby made permanent; that the writs prayed for are granted, insofar as the documents,
Indeed, the non-exclusionary rule is contrary, not only to the letter, but also, to the spirit of the papers and other effects so seized in the aforementioned residences are concerned; that the
constitutional injunction against unreasonable searches and seizures. To be sure, if the aforementioned motion for Reconsideration and Amendment should be, as it is hereby, denied;
applicant for a search warrant has competent evidence to establish probable cause of the and that the petition herein is dismissed and the writs prayed for denied, as regards the
commission of a given crime by the party against whom the warrant is intended, then there is no documents, papers and other effects seized in the twenty-nine (29) places, offices and other
reason why the applicant should not comply with the requirements of the fundamental law. Upon premises enumerated in the same Resolution, without special pronouncement as to costs.
the other hand, if he has no such competent evidence, then it is not possible for the Judge to
find that there is probable cause, and, hence, no justification for the issuance of the warrant. The
only possible explanation (not justification) for its issuance is the necessity of fishing evidence of It is so ordered.
the commission of a crime. But, then, this fishing expedition is indicative of the absence of
evidence to establish a probable cause.

Moreover, the theory that the criminal prosecution of those who secure an illegal search warrant
and/or make unreasonable searches or seizures would suffice to protect the constitutional
G.R. No. L-42780 January 17, 1936 1. Appellant first contends that the dividends paid by it to its stockholders, the Islands
Gas and Electric Company , were not subject to tax because to impose a tax thereon
would be to do so on the plaintiff corporation, in violation of the terms of its franchise
MANILA GAS CORPORATION, plaintiff-appellant,
and would, moreover, be oppressive and inequitable. This argument is predicated on
vs.
the constitutional provision that no law impairing the obligation of contracts shall be
THE COLLECTOR OF INTERNAL REVENUE, defendant-appellee.
enacted. The particular portion of the franchise which is invoked provides:

MALCOLM, J.:
The grantee shall annually on the fifth day of January of each year pay to
the City of Manila and the municipalities in the Province of Rizal in which
This is an action brought by the Manila Gas Corporation against the Collector of Internal gas is sold, two and one half per centum of the gross receipts within said
Revenue for the recovery of P56,757.37, which the plaintiff was required by the defendant to city and municipalities, respectively, during the preceding year. Said
deduct and withhold from the various sums paid it to foreign corporations as dividends and payment shall be in lieu of all taxes, Insular, provincial and municipal,
interest on bonds and other indebtedness and which the plaintiff paid under protest. On the trial except taxes on the real estate, buildings, plant, machinery, and other
court dismissing the complaint, with costs, the plaintiff appealed assigning as the principal errors personal property belonging to the grantee.
alleged to have been committed the following:
The trial judge was of the opinion that the instant case was governed by our previous
1. The trial court erred in holding that the dividends paid by the plaintiff corporation decision in the case of Philippine Telephone and Telegraph Co., vs. Collector of
were subject to income tax in the hands of its stockholders, because to impose the tax Internal Revenue ([1933], 58 Phil. 639). In this view we concur. It is true that the tax
thereon would be to impose a tax on the plaintiff, in violation of the terms of its exemption provision relating to the Manila Gas Corporation hereinbefore quoted
franchise, and would, moreover, be oppressive and inequitable. differs in phraseology from the tax exemption provision to be found in the franchise of
the Telephone and Telegraph Company, but the ratio decidendi of the two cases is
substantially the same. As there held and as now confirmed, a corporation has a
2. The trial court erred in not holding that the interest on bonds and other
personality distinct from that of its stockholders, enabling the taxing power to reach the
indebtedness of the plaintiff corporation, paid by it outside of the Philippine Islands to latter when they receive dividends from the corporation. It must be considered as
corporations not residing therein, were not, on the part of the recipients thereof, settled in this jurisdiction that dividends of a domestic corporation, which are paid and
income from Philippine sources, and hence not subject to Philippine income tax.
delivered in cash to foreign corporations as stockholders, are subject to the payment
in the income tax, the exemption clause in the charter of the corporation
The facts, as stated by the appellant and as accepted by the appellee, may be summarized as notwithstanding.
follows: The plaintiff is a corporation organized under the laws of the Philippine Islands. It
operates a gas plant in the City of Manila and furnishes gas service to the people of the For the foreign reasons, we are led to sustain the decision of the trial court and to
metropolis and surrounding municipalities by virtue of a franchise granted to it by the Philippine overrule appellant's first assigned error.
Government. Associated with the plaintiff are the Islands Gas and Electric Company domiciled in
New York, United States, and the General Finance Company domiciled in Zurich, Switzerland.
Neither of these last mentioned corporations is resident in the Philippines. 2. In support of its second assignment of error, appellant contends that, as the Islands
Gas and Electric Company and the General Finance Company are domiciled in the
United States and Switzerland respectively, and as the interest on the bonds and
For the years 1930, 1931, and 1932, dividends in the sum of P1,348,847.50 were paid by the other indebtedness earned by said corporations has been paid in their respective
plaintiff to the Islands Gas and Electric Company in the capacity of stockholders upon which
domiciles, this is not income from Philippine sources within the meaning of the
withholding income taxes were paid to the defendant totalling P40,460.03 For the same years Philippine Income Tax Law. Citing sections 10 (a) and 13 (e) of Act No. 2833, the
interest on bonds in the sum of P411,600 was paid by the plaintiff to the Islands Gas and Electric Income Tax Law, appellant asserts that their applicability has been squarely
Company upon which withholding income taxes were paid to the defendant totalling P12,348.
determined by decisions of this court in the cases of Manila Railroad Co. vs. Collector
Finally for the stated time period, interest on other indebtedness in the sum of P131,644,90 was of Internal Revenue (No. 31196, promulgated December 2, 1929, nor reported),
paid by the plaintiff to the Islands Gas and Electric Company and the General Finance Company and Philippine Railway Co. vs. Posadas (No. 38766, promulgated October 30, 1933
respectively upon which withholding income taxes were paid to the defendant totalling
[58 Phil., 968]) wherein it was held that interest paid to non-resident individuals or
P3,949.34. corporations is not income from Philippine sources, and hence not subject to the
Philippine Income Tax. The Solicitor-General answers with the observation that the
Some uncertainty existing regarding the place of payment, we will not go into this factor of the cited decisions interpreted the Income Tax Law before it was amended by Act No.
case at this point, except to remark that the bonds and other tokens of indebtedness are not to 3761 to cover the interest on bonds and other obligations or securities paid "within or
be found in the record. However, Exhibits E, F, and G, certified correct by the Treasurer of the without the Philippine Islands." Appellant rebuts this argument by "assuming, for the
Manila Gas Corporation, purport to prove that the place of payment was the United States and sake of the argument, that by the amendment introduced to section 13 of Act No. 2833
Switzerland. by Act No. 3761 the Legislature intended the interest from Philippine sources and so is
subject to tax," but with the necessary sequel that the amendatory statute is invalid
and unconstitutional as being the power of the Legislature to enact.
The appeal naturally divides into two subjects, one covered by the first assigned error, and the
other by the second assigned error. We shall discuss these subjects and errors in order.
Taking first under observation that last point, it is to be observed that neither in the pleadings,
the decision of the trial court, nor the assignment of errors, was the question of the validity of Act
No. 3761 raised. Under such circumstances, and no jurisdictional issue being involved, we do Before concluding, it is but fair to state that the writer's opinion on the first subject and the first
not feel that it is the duty of the court to pass on the constitutional question, and accordingly will assigned error herein discussed is accurately set forth, but that his opinion on the second
refrain from doing so. (Cadwaller-Gibson Lumber Co. vs. Del Rosario [1913], 26 Phil., 192; subject and the second assigned error is not accurately reflected, because on this last division
Macondray and Co. vs. Benito and Ocampo, P. 137, ante; State vs. Burke [1912], 175 Ala., his views coincide with those of the appellant. However, in the interest of the prompt disposition
561.) of this case, the decision has been written up in accordance with instructions received from the
court.
As to the applicability of the local cases cited and of the Porto Rican case of
Domenech vs. United Porto Rican Sugar co. ([1932], 62 F. [2d], 552), we need only observe that Judgment affirmed, with the cost of this instance assessed against the appellant.
these cases announced good law, but that each he must be decided on its particular facts. In
other words, in the opinion of the majority of the court, the facts at bar and the facts in those
cases can be clearly differentiated. Also, in the case at bar there is some uncertainty concerning
the place of payment, which under one view could be considered the Philippines and under
another view the United States and Switzerland, but which cannot be definitely determined
without the necessary documentary evidence before, us.

The approved doctrine is that no state may tax anything not within its jurisdiction without
violating the due process clause of the constitution. The taxing power of a state does not extend
beyond its territorial limits, but within such it may tax persons, property, income, or business. If
an interest in property is taxed, the situs of either the property or interest must be found within
the state. If an income is taxed, the recipient thereof must have a domicile within the state or the
property or business out of which the income issues must be situated within the state so that the
income may be said to have a situs therein. Personal property may be separated from its owner,
and he may be taxed on its account at the place where the property is although it is not the place
of his own domicile and even though he is not a citizen or resident of the state which imposes
the tax. But debts owing by corporations are obligations of the debtors, and only possess value
in the hands of the creditors. (Farmers Loan Co. vs. Minnesota [1930], 280 U.S., 204; Union
Refrigerator Transit Co. vs. Kentucky [1905], 199 U.S., 194 State Tax on Foreign held Bonds
[1873, 15 Wall., 300; Bick vs. Beach [1907], 206 U. S., 392; State ex rel. Manitowoc Gas
Co. vs. Wig. Tax Comm. [1915], 161 Wis., 111; United States Revenue Act of 1932, sec. 143.)

These views concerning situs for taxation purposes apply as well to an organized,
unincorporated territory or to a Commonwealth having the status of the Philippines.

Pushing to one side that portion of Act No. 3761 which permits taxation of interest on bonds and
other indebtedness paid without the Philippine Islands, the question is if the income was derived
from sources within the Philippine Islands.

In the judgment of the majority of the court, the question should be answered in the affirmative.
The Manila Gas Corporation operates its business entirely within the Philippines. Its earnings,
therefore come from local sources. The place of material delivery of the interest to the foreign
corporations paid out of the revenue of the domestic corporation is of no particular moment. The
place of payment even if conceded to be outside of tho country cannot alter the fact that the
income was derived from the Philippines. The word "source" conveys only one idea, that of
origin, and the origin of the income was the Philippines.

In synthesis, therefore, we hold that conditions have not been provided which justify the court in
passing on the constitutional question suggested; that the facts while somewhat obscure differ
from the facts to be found in the cases relied upon, and that the Collector of Internal Revenue
was justified in withholding income taxes on interest on bonds and other indebtedness paid to
non-resident corporations because this income was received from sources within the Philippine
Islands as authorized by the Income Tax Law. For the foregoing reasons, the second assigned
error will be overruled.
G.R. No. L-27155 May 18, 1978 debt to the Bank, that she told the Plaintiff that she did not consider herself
to be indebted to the Bank at all because she had an agreement with one
Jacobo-Nazon whereby she had leased to the latter her unused export
PHILIPPINE NATIONAL BANK, petitioner,
sugar quota for the 1956-1957 agricultural year, consisting of 1,000 piculs at
vs.
the rate of P2.80 per picul, or for a total of P2,800.00, which was already in
THE COURT OF APPEALS, RITA GUECO TAPNIO, CECILIO GUECO and THE PHILIPPINE
excess of her obligation guaranteed by plaintiff's bond, Exh. A. This lease
AMERICAN GENERAL INSURANCE COMPANY, INC., respondents.
agreement, according to her, was with the knowledge of the bank. But the
Bank has placed obstacles to the consummation of the lease, and the delay
ANTONIO, J.: caused by said obstacles forced 'Nazon to rescind the lease contract. Thus,
Rita Gueco Tapnio filed her third-party complaint against the Bank to
recover from the latter any and all sums of money which may be adjudged
Certiorari to review the decision of the Court of Appeals which affirmed the judgment of the against her and in favor of the plaitiff plus moral damages, attorney's fees
Court of First Instance of Manila in Civil Case No. 34185, ordering petitioner, as third-party and costs.
defendant, to pay respondent Rita Gueco Tapnio, as third-party plaintiff, the sum of P2,379.71,
plus 12% interest per annum from September 19, 1957 until the same is fully paid, P200.00
attorney's fees and costs, the same amounts which Rita Gueco Tapnio was ordered to pay the Insofar as the contentions of the parties herein are concerned, we quote
Philippine American General Insurance Co., Inc., to be paid directly to the Philippine American with approval the following findings of the lower court based on the evidence
General Insurance Co., Inc. in full satisfaction of the judgment rendered against Rita Gueco presented at the trial of the case:
Tapnio in favor of the former; plus P500.00 attorney's fees for Rita Gueco Tapnio and costs. The
basic action is the complaint filed by Philamgen (Philippine American General Insurance Co.,
It has been established during the trial that Mrs. Tapnio
Inc.) as surety against Rita Gueco Tapnio and Cecilio Gueco, for the recovery of the sum of
had an export sugar quota of 1,000 piculs for the
P2,379.71 paid by Philamgen to the Philippine National Bank on behalf of respondents Tapnio
agricultural year 1956-1957 which she did not need.
and Gueco, pursuant to an indemnity agreement. Petitioner Bank was made third-party
She agreed to allow Mr. Jacobo C. Tuazon to use said
defendant by Tapnio and Gueco on the theory that their failure to pay the debt was due to the
quota for the consideration of P2,500.00 (Exh. "4"-
fault or negligence of petitioner.
Gueco). This agreement was called a contract of lease
of sugar allotment.
The facts as found by the respondent Court of Appeals, in affirming the decision of the Court of
First Instance of Manila, are quoted hereunder:
At the time of the agreement, Mrs. Tapnio was indebted
to the Philippine National Bank at San Fernando,
Plaintiff executed its Bond, Exh. A, with defendant Rita Gueco Tapnio as Pampanga. Her indebtedness was known as a crop
principal, in favor of the Philippine National Bank Branch at San Fernando, loan and was secured by a mortgage on her standing
Pampanga, to guarantee the payment of defendant Rita Gueco Tapnio's crop including her sugar quota allocation for the
account with said Bank. In turn, to guarantee the payment of whatever agricultural year corresponding to said standing crop.
amount the bonding company would pay to the Philippine National Bank, This arrangement was necessary in order that when
both defendants executed the indemnity agreement, Exh. B. Under the Mrs. Tapnio harvests, the P.N.B., having a lien on the
terms and conditions of this indemnity agreement, whatever amount the crop, may effectively enforce collection against her. Her
plaintiff would pay would earn interest at the rate of 12% per annum, plus sugar cannot be exported without sugar quota allotment
attorney's fees in the amount of 15 % of the whole amount due in case of Sometimes, however, a planter harvest less sugar than
court litigation. her quota, so her excess quota is utilized by another
who pays her for its use. This is the arrangement
entered into between Mrs. Tapnio and Mr. Tuazon
The original amount of the bond was for P4,000.00; but the amount was regarding the former's excess quota for 1956-1957
later reduced to P2,000.00. (Exh. "4"-Gueco).

It is not disputed that defendant Rita Gueco Tapnio was indebted to the Since the quota was mortgaged to the P.N.B., the
bank in the sum of P2,000.00, plus accumulated interests unpaid, which she contract of lease had to be approved by said Bank, The
failed to pay despite demands. The Bank wrote a letter of demand to
same was submitted to the branch manager at San
plaintiff, as per Exh. C; whereupon, plaintiff paid the bank on September 18, Fernando, Pampanga. The latter required the parties to
1957, the full amount due and owing in the sum of P2,379.91, for and on raise the consideration of P2.80 per picul or a total of
account of defendant Rita Gueco's obligation (Exhs. D and D-1).
P2,800.00 (Exh. "2-Gueco") informing them that "the
minimum lease rental acceptable to the Bank, is P2.80
Plaintiff, in turn, made several demands, both verbal and written, upon per picul." In a letter addressed to the branch manager
defendants (Exhs. E and F), but to no avail. on August 10, 1956, Mr. Tuazon informed the manager
that he was agreeable to raising the consideration to
P2.80 per picul. He further informed the manager that
Defendant Rita Gueco Tapnio admitted all the foregoing facts. She claims,
however, when demand was made upon her by plaintiff for her to pay her
he was ready to pay said amount as the funds were in interests on her properties, and surety bonds, aside from the fact that from
his folder which was kept in the bank. Exh. 8-Bank, it appears that she was offering to execute a real estate
mortgage in favor of the Bank to replace the surety bond This statement is
further bolstered by the fact that Rita Gueco Tapnio apparently had the
Explaining the meaning of Tuazon's statement as to the
means to pay her obligation fact that she has been granted several value of
funds, it was stated by him that he had an approved
almost P80,000.00 for the agricultural years from 1952 to 56. 1
loan from the bank but he had not yet utilized it as he
was intending to use it to pay for the quota. Hence,
when he said the amount needed to pay Mrs. Tapnio Its motion for the reconsideration of the decision of the Court of Appeals having been denied,
was in his folder which was in the bank, he meant and petitioner filed the present petition.
the manager understood and knew he had an approved
loan available to be used in payment of the quota. In
The petitioner contends that the Court of Appeals erred:
said Exh. "6-Gueco", Tuazon also informed the
manager that he would want for a notice from the
manager as to the time when the bank needed the (1) In finding that the rescission of the lease contract of the 1,000 piculs of sugar quota allocation
money so that Tuazon could sign the corresponding of respondent Rita Gueco Tapnio by Jacobo C. Tuazon was due to the unjustified refusal of
promissory note. petitioner to approve said lease contract, and its unreasonable insistence on the rental price of
P3.00 instead of P2.80 per picul; and
Further Consideration of the evidence discloses that when the branch
manager of the Philippine National Bank at San Fernando recommended (2) In not holding that based on the statistics of sugar price and prices of sugar quota in the
the approval of the contract of lease at the price of P2.80 per picul (Exh. 1 possession of the petitioner, the latter's Board of Directors correctly fixed the rental of price per
1-Bank), whose recommendation was concurred in by the Vice-president of picul of 1,000 piculs of sugar quota leased by respondent Rita Gueco Tapnio to Jacobo C.
said Bank, J. V. Buenaventura, the board of directors required that the Tuazon at P3.00 per picul.
amount be raised to 13.00 per picul. This act of the board of directors was
communicated to Tuazon, who in turn asked for a reconsideration thereof.
On November 19, 1956, the branch manager submitted Tuazon's request Petitioner argued that as an assignee of the sugar quota of Tapnio, it has the right, both under
its own Charter and under the Corporation Law, to safeguard and protect its rights and interests
for reconsideration to the board of directors with another recommendation
for the approval of the lease at P2.80 per picul, but the board returned the under the deed of assignment, which include the right to approve or disapprove the said lease of
recommendation unacted upon, considering that the current price prevailing sugar quota and in the exercise of that authority, its
at the time was P3.00 per picul (Exh. 9-Bank).
Board of Directors necessarily had authority to determine and fix the rental price per picul of the
The parties were notified of the refusal on the part of the board of directors sugar quota subject of the lease between private respondents and Jacobo C. Tuazon. It argued
of the Bank to grant the motion for reconsideration. The matter stood as it further that both under its Charter and the Corporation Law, petitioner, acting thru its Board of
was until February 22, 1957, when Tuazon wrote a letter (Exh. 10-Bank Directors, has the perfect right to adopt a policy with respect to fixing of rental prices of export
informing the Bank that he was no longer interested to continue the deal, sugar quota allocations, and in fixing the rentals at P3.00 per picul, it did not act arbitrarily since
referring to the lease of sugar quota allotment in favor of defendant Rita the said Board was guided by statistics of sugar price and prices of sugar quotas prevailing at
Gueco Tapnio. The result is that the latter lost the sum of P2,800.00 which the time. Since the fixing of the rental of the sugar quota is a function lodged with petitioner's
Board of Directors and is a matter of policy, the respondent Court of Appeals could not substitute
she should have received from Tuazon and which she could have paid the
Bank to cancel off her indebtedness, its own judgment for that of said Board of Directors, which acted in good faith, making as its
basis therefore the prevailing market price as shown by statistics which were then in their
possession.
The court below held, and in this holding we concur that failure of the
negotiation for the lease of the sugar quota allocation of Rita Gueco Tapnio
to Tuazon was due to the fault of the directors of the Philippine National Finally, petitioner emphasized that under the appealed judgment, it shall suffer a great injustice
because as a creditor, it shall be deprived of a just claim against its debtor (respondent Rita
Bank, The refusal on the part of the bank to approve the lease at the rate of
P2.80 per picul which, as stated above, would have enabled Rita Gueco Gueco Tapnio) as it would be required to return to respondent Philamgen the sum of P2,379.71,
Tapnio to realize the amount of P2,800.00 which was more than sufficient to plus interest, which amount had been previously paid to petitioner by said insurance company in
behalf of the principal debtor, herein respondent Rita Gueco Tapnio, and without recourse
pay off her indebtedness to the Bank, and its insistence on the rental price
of P3.00 per picul thus unnecessarily increasing the value by only a against respondent Rita Gueco Tapnio.
difference of P200.00. inevitably brought about the rescission of the lease
contract to the damage and prejudice of Rita Gueco Tapnio in the aforesaid We must advert to the rule that this Court's appellate jurisdiction in proceedings of this nature is
sum of P2,800.00. The unreasonableness of the position adopted by the limited to reviewing only errors of law, accepting as conclusive the factual fin dings of the Court
board of directors of the Philippine National Bank in refusing to approve the of Appeals upon its own assessment of the evidence. 2
lease at the rate of P2.80 per picul and insisting on the rate of P3.00 per
picul, if only to increase the retail value by only P200.00 is shown by the fact
that all the accounts of Rita Gueco Tapnio with the Bank were secured by The contract of lease of sugar quota allotment at P2.50 per picul between Rita Gueco Tapnio
chattel mortgage on standing crops, assignment of leasehold rights and and Jacobo C. Tuazon was executed on April 17, 1956. This contract was submitted to the
Branch Manager of the Philippine National Bank at San Fernando, Pampanga. This
arrangement was necessary because Tapnio's indebtedness to petitioner was secured by a 1952 to 1956", there was no reasonable basis for the Board of Directors of petitioner to have
mortgage on her standing crop including her sugar quota allocation for the agricultural year rejected the lease agreement because of a measly sum of P200.00.
corresponding to said standing crop. The latter required the parties to raise the consideration to
P2.80 per picul, the minimum lease rental acceptable to the Bank, or a total of P2,800.00.
While petitioner had the ultimate authority of approving or disapproving the proposed lease since
Tuazon informed the Branch Manager, thru a letter dated August 10, 1956, that he was
the quota was mortgaged to the Bank, the latter certainly cannot escape its responsibility of
agreeable to raising the consideration to P2.80 per picul. He further informed the manager that
observing, for the protection of the interest of private respondents, that degree of care,
he was ready to pay the said sum of P2,800.00 as the funds were in his folder which was kept in
precaution and vigilance which the circumstances justly demand in approving or disapproving
the said Bank. This referred to the approved loan of Tuazon from the Bank which he intended to
the lease of said sugar quota. The law makes it imperative that every person "must in the
use in paying for the use of the sugar quota. The Branch Manager submitted the contract of
exercise of his rights and in the performance of his duties, act with justice, give everyone his
lease of sugar quota allocation to the Head Office on September 7, 1956, with a
due, and observe honesty and good faith, 4 This petitioner failed to do. Certainly, it knew that the
recommendation for approval, which recommendation was concurred in by the Vice-President of
agricultural year was about to expire, that by its disapproval of the lease private respondents
the Bank, Mr. J. V. Buenaventura. This notwithstanding, the Board of Directors of petitioner
would be unable to utilize the sugar quota in question. In failing to observe the reasonable
required that the consideration be raised to P3.00 per picul.
degree of care and vigilance which the surrounding circumstances reasonably impose, petitioner
is consequently liable for the damages caused on private respondents. Under Article 21 of the
Tuazon, after being informed of the action of the Board of Directors, asked for a reconsideration New Civil Code, "any person who wilfully causes loss or injury to another in a manner that is
thereof. On November 19, 1956, the Branch Manager submitted the request for reconsideration contrary to morals, good customs or public policy shall compensate the latter for the damage."
and again recommended the approval of the lease at P2.80 per picul, but the Board returned the The afore-cited provisions on human relations were intended to expand the concept of torts in
recommendation unacted, stating that the current price prevailing at that time was P3.00 per this jurisdiction by granting adequate legal remedy for the untold number of moral wrongs which
picul. is impossible for human foresight to specifically provide in the statutes. 5

On February 22, 1957, Tuazon wrote a letter, informing the Bank that he was no longer A corporation is civilly liable in the same manner as natural persons for torts, because "generally
interested in continuing the lease of sugar quota allotment. The crop year 1956-1957 ended and speaking, the rules governing the liability of a principal or master for a tort committed by an
Mrs. Tapnio failed to utilize her sugar quota, resulting in her loss in the sum of P2,800.00 which agent or servant are the same whether the principal or master be a natural person or a
she should have received had the lease in favor of Tuazon been implemented. corporation, and whether the servant or agent be a natural or artificial person. All of the
authorities agree that a principal or master is liable for every tort which he expressly directs or
authorizes, and this is just as true of a corporation as of a natural person, A corporation is liable,
It has been clearly shown that when the Branch Manager of petitioner required the parties to
therefore, whenever a tortious act is committed by an officer or agent under express direction or
raise the consideration of the lease from P2.50 to P2.80 per picul, or a total of P2,800-00, they
authority from the stockholders or members acting as a body, or, generally, from the directors as
readily agreed. Hence, in his letter to the Branch Manager of the Bank on August 10, 1956,
the governing body." 6
Tuazon informed him that the minimum lease rental of P2.80 per picul was acceptable to him
and that he even offered to use the loan secured by him from petitioner to pay in full the sum of
P2,800.00 which was the total consideration of the lease. This arrangement was not only WHEREFORE, in view of the foregoing, the decision of the Court of Appeals is hereby
satisfactory to the Branch Manager but it was also approves by Vice-President J. V. AFFIRMED.
Buenaventura of the PNB. Under that arrangement, Rita Gueco Tapnio could have realized the
amount of P2,800.00, which was more than enough to pay the balance of her indebtedness to
the Bank which was secured by the bond of Philamgen.

There is no question that Tapnio's failure to utilize her sugar quota for the crop year 1956-1957
was due to the disapproval of the lease by the Board of Directors of petitioner. The issue,
therefore, is whether or not petitioner is liable for the damage caused.

As observed by the trial court, time is of the essence in the approval of the lease of sugar quota
allotments, since the same must be utilized during the milling season, because any allotment
which is not filled during such milling season may be reallocated by the Sugar Quota
Administration to other holders of allotments. 3 There was no proof that there was any other
person at that time willing to lease the sugar quota allotment of private respondents for a price
higher than P2.80 per picul. "The fact that there were isolated transactions wherein the
consideration for the lease was P3.00 a picul", according to the trial court, "does not necessarily
mean that there are always ready takers of said price. " The unreasonableness of the position
adopted by the petitioner's Board of Directors is shown by the fact that the difference between
the amount of P2.80 per picul offered by Tuazon and the P3.00 per picul demanded by the
Board amounted only to a total sum of P200.00. Considering that all the accounts of Rita Gueco
Tapnio with the Bank were secured by chattel mortgage on standing crops, assignment of
leasehold rights and interests on her properties, and surety bonds and that she had apparently
"the means to pay her obligation to the Bank, as shown by the fact that she has been granted
several sugar crop loans of the total value of almost P80,000.00 for the agricultural years from
G.R. No. 195580 April 21, 2014 In the petitions, Redmont alleged that at least 60% of the capital stock of McArthur, Tesoro and
Narra are owned and controlled by MBMI Resources, Inc. (MBMI), a 100% Canadian
corporation. Redmont reasoned that since MBMI is a considerable stockholder of petitioners, it
NARRA NICKEL MINING AND DEVELOPMENT CORP., TESORO MINING AND
was the driving force behind petitioners’ filing of the MPSAs over the areas covered by
DEVELOPMENT, INC., and MCARTHUR MINING, INC., Petitioners,
applications since it knows that it can only participate in mining activities through corporations
vs.
which are deemed Filipino citizens. Redmont argued that given that petitioners’ capital stocks
REDMONT CONSOLIDATED MINES CORP., Respondent.
were mostly owned by MBMI, they were likewise disqualified from engaging in mining activities
through MPSAs, which are reserved only for Filipino citizens.
DECISION
In their Answers, petitioners averred that they were qualified persons under Section 3(aq) of
VELASCO, JR., J.: Republic Act No. (RA) 7942 or the Philippine Mining Act of 1995 which provided:

Before this Court is a Petition for Review on Certiorari under Rule 45 filed by Narra Nickel and Sec. 3 Definition of Terms. As used in and for purposes of this Act, the following terms, whether
Mining Development Corp. (Narra), Tesoro Mining and Development, Inc. (Tesoro), and in singular or plural, shall mean:
McArthur Mining Inc. (McArthur), which seeks to reverse the October 1, 2010 Decision1 and the
February 15, 2011 Resolution of the Court of Appeals (CA).
xxxx

The Facts
(aq) "Qualified person" means any citizen of the Philippines with capacity to contract, or a
corporation, partnership, association, or cooperative organized or authorized for the purpose of
Sometime in December 2006, respondent Redmont Consolidated Mines Corp. (Redmont), a engaging in mining, with technical and financial capability to undertake mineral resources
domestic corporation organized and existing under Philippine laws, took interest in mining and development and duly registered in accordance with law at least sixty per cent (60%) of the
exploring certain areas of the province of Palawan. After inquiring with the Department of capital of which is owned by citizens of the Philippines: Provided, That a legally organized
Environment and Natural Resources (DENR), it learned that the areas where it wanted to foreign-owned corporation shall be deemed a qualified person for purposes of granting an
undertake exploration and mining activities where already covered by Mineral Production exploration permit, financial or technical assistance agreement or mineral processing permit.
Sharing Agreement (MPSA) applications of petitioners Narra, Tesoro and McArthur.
Additionally, they stated that their nationality as applicants is immaterial because they also
Petitioner McArthur, through its predecessor-in-interest Sara Marie Mining, Inc. (SMMI), filed an applied for Financial or Technical Assistance Agreements (FTAA) denominated as AFTA-IVB-09
application for an MPSA and Exploration Permit (EP) with the Mines and Geo-Sciences Bureau for McArthur, AFTA-IVB-08 for Tesoro and AFTA-IVB-07 for Narra, which are granted to foreign-
(MGB), Region IV-B, Office of the Department of Environment and Natural Resources (DENR). owned corporations. Nevertheless, they claimed that the issue on nationality should not be
raised since McArthur, Tesoro and Narra are in fact Philippine Nationals as 60% of their capital
is owned by citizens of the Philippines. They asserted that though MBMI owns 40% of the
Subsequently, SMMI was issued MPSA-AMA-IVB-153 covering an area of over 1,782 hectares shares of PLMC (which owns 5,997 shares of Narra),3 40% of the shares of MMC (which owns
in Barangay Sumbiling, Municipality of Bataraza, Province of Palawan and EPA-IVB-44 which 5,997 shares of McArthur)4 and 40% of the shares of SLMC (which, in turn, owns 5,997 shares
includes an area of 3,720 hectares in Barangay Malatagao, Bataraza, Palawan. The MPSA and of Tesoro),5 the shares of MBMI will not make it the owner of at least 60% of the capital stock of
EP were then transferred to Madridejos Mining Corporation (MMC) and, on November 6, 2006, each of petitioners. They added that the best tool used in determining the nationality of a
assigned to petitioner McArthur.2 corporation is the "control test," embodied in Sec. 3 of RA 7042 or the Foreign Investments Act
of 1991. They also claimed that the POA of DENR did not have jurisdiction over the issues in
Petitioner Narra acquired its MPSA from Alpha Resources and Development Corporation and Redmont’s petition since they are not enumerated in Sec. 77 of RA 7942. Finally, they stressed
Patricia Louise Mining & Development Corporation (PLMDC) which previously filed an that Redmont has no personality to sue them because it has no pending claim or application
application for an MPSA with the MGB, Region IV-B, DENR on January 6, 1992. Through the over the areas applied for by petitioners.
said application, the DENR issued MPSA-IV-1-12 covering an area of 3.277 hectares in
barangays Calategas and San Isidro, Municipality of Narra, Palawan. Subsequently, PLMDC On December 14, 2007, the POA issued a Resolution disqualifying petitioners from gaining
conveyed, transferred and/or assigned its rights and interests over the MPSA application in favor
MPSAs. It held:
of Narra.

[I]t is clearly established that respondents are not qualified applicants to engage in mining
Another MPSA application of SMMI was filed with the DENR Region IV-B, labeled as MPSA-
activities. On the other hand, [Redmont] having filed its own applications for an EPA over the
AMA-IVB-154 (formerly EPA-IVB-47) over 3,402 hectares in Barangays Malinao and Princesa areas earlier covered by the MPSA application of respondents may be considered if and when
Urduja, Municipality of Narra, Province of Palawan. SMMI subsequently conveyed, transferred they are qualified under the law. The violation of the requirements for the issuance and/or grant
and assigned its rights and interest over the said MPSA application to Tesoro.
of permits over mining areas is clearly established thus, there is reason to believe that the
cancellation and/or revocation of permits already issued under the premises is in order and open
On January 2, 2007, Redmont filed before the Panel of Arbitrators (POA) of the DENR three (3) the areas covered to other qualified applicants.
separate petitions for the denial of petitioners’ applications for MPSA designated as AMA-IVB-
153, AMA-IVB-154 and MPSA IV-1-12. xxxx
WHEREFORE, the Panel of Arbitrators finds the Respondents, McArthur Mining Inc., Tesoro Before the MAB could resolve Redmont’s Motion for Reconsideration and Supplemental Motion
Mining and Development, Inc., and Narra Nickel Mining and Development Corp. as, for Reconsideration, Redmont filed before the RTC a Supplemental Complaint 21 in Civil Case
DISQUALIFIED for being considered as Foreign Corporations. Their Mineral Production Sharing No. 08-63379.
Agreement (MPSA) are hereby x x x DECLARED NULL AND VOID. 6
On October 6, 2008, the RTC issued an Order22 granting the issuance of a writ of preliminary
The POA considered petitioners as foreign corporations being "effectively controlled" by MBMI, a injunction enjoining the MAB from finally disposing of the appeals of petitioners and from
100% Canadian company and declared their MPSAs null and void. In the same Resolution, it resolving Redmont’s Motion for Reconsideration and Supplement Motion for Reconsideration of
gave due course to Redmont’s EPAs. Thereafter, on February 7, 2008, the POA issued an the MAB’s September 10, 2008 Resolution.
Order7 denying the Motion for Reconsideration filed by petitioners.
On July 1, 2009, however, the MAB issued a second Order denying Redmont’s Motion for
Aggrieved by the Resolution and Order of the POA, McArthur and Tesoro filed a joint Notice of Reconsideration and Supplemental Motion for Reconsideration and resolving the appeals filed
Appeal8 and Memorandum of Appeal9 with the Mines Adjudication Board (MAB) while Narra by petitioners.
separately filed its Notice of Appeal10and Memorandum of Appeal.11
Hence, the petition for review filed by Redmont before the CA, assailing the Orders issued by
In their respective memorandum, petitioners emphasized that they are qualified persons under the MAB. On October 1, 2010, the CA rendered a Decision, the dispositive of which reads:
the law. Also, through a letter, they informed the MAB that they had their individual MPSA
applications converted to FTAAs. McArthur’s FTAA was denominated as AFTA-IVB-0912 on May
WHEREFORE, the Petition is PARTIALLY GRANTED. The assailed Orders, dated September
2007, while Tesoro’s MPSA application was converted to AFTA-IVB-0813 on May 28, 2007, and
10, 2008 and July 1, 2009 of the Mining Adjudication Board are reversed and set aside. The
Narra’s FTAA was converted to AFTA-IVB-0714 on March 30, 2006.
findings of the Panel of Arbitrators of the Department of Environment and Natural Resources
that respondents McArthur, Tesoro and Narra are foreign corporations is upheld and, therefore,
Pending the resolution of the appeal filed by petitioners with the MAB, Redmont filed a the rejection of their applications for Mineral Product Sharing Agreement should be
Complaint15 with the Securities and Exchange Commission (SEC), seeking the revocation of the recommended to the Secretary of the DENR.
certificates for registration of petitioners on the ground that they are foreign-owned or controlled
corporations engaged in mining in violation of Philippine laws. Thereafter, Redmont filed on
With respect to the applications of respondents McArthur, Tesoro and Narra for Financial or
September 1, 2008 a Manifestation and Motion to Suspend Proceeding before the MAB praying
Technical Assistance Agreement (FTAA) or conversion of their MPSA applications to FTAA, the
for the suspension of the proceedings on the appeals filed by McArthur, Tesoro and Narra.
matter for its rejection or approval is left for determination by the Secretary of the DENR and the
President of the Republic of the Philippines.
Subsequently, on September 8, 2008, Redmont filed before the Regional Trial Court of Quezon
City, Branch 92 (RTC) a Complaint16 for injunction with application for issuance of a temporary
SO ORDERED.23
restraining order (TRO) and/or writ of preliminary injunction, docketed as Civil Case No. 08-
63379. Redmont prayed for the deferral of the MAB proceedings pending the resolution of the
Complaint before the SEC. In a Resolution dated February 15, 2011, the CA denied the Motion for Reconsideration filed by
petitioners.
But before the RTC can resolve Redmont’s Complaint and applications for injunctive reliefs, the
MAB issued an Order on September 10, 2008, finding the appeal meritorious. It held: After a careful review of the records, the CA found that there was doubt as to the nationality of
petitioners when it realized that petitioners had a common major investor, MBMI, a corporation
composed of 100% Canadians. Pursuant to the first sentence of paragraph 7 of Department of
WHEREFORE, in view of the foregoing, the Mines Adjudication Board hereby REVERSES and
Justice (DOJ) Opinion No. 020, Series of 2005, adopting the 1967 SEC Rules which
SETS ASIDE the Resolution dated 14 December 2007 of the Panel of Arbitrators of Region IV-B
implemented the requirement of the Constitution and other laws pertaining to the exploitation of
(MIMAROPA) in POA-DENR Case Nos. 2001-01, 2007-02 and 2007-03, and its Order dated 07
natural resources, the CA used the "grandfather rule" to determine the nationality of petitioners.
February 2008 denying the Motions for Reconsideration of the Appellants. The Petition filed by
It provided:
Redmont Consolidated Mines Corporation on 02 January 2007 is hereby ordered DISMISSED. 17

Shares belonging to corporations or partnerships at least 60% of the capital of which is owned
Belatedly, on September 16, 2008, the RTC issued an Order18 granting Redmont’s application
by Filipino citizens shall be considered as of Philippine nationality, but if the percentage of
for a TRO and setting the case for hearing the prayer for the issuance of a writ of preliminary
Filipino ownership in the corporation or partnership is less than 60%, only the number of shares
injunction on September 19, 2008.
corresponding to such percentage shall be counted as of Philippine nationality. Thus, if 100,000
shares are registered in the name of a corporation or partnership at least 60% of the capital
Meanwhile, on September 22, 2008, Redmont filed a Motion for Reconsideration19 of the stock or capital, respectively, of which belong to Filipino citizens, all of the shares shall be
September 10, 2008 Order of the MAB. Subsequently, it filed a Supplemental Motion for recorded as owned by Filipinos. But if less than 60%, or say, 50% of the capital stock or capital
Reconsideration20 on September 29, 2008. of the corporation or partnership, respectively, belongs to Filipino citizens, only 50,000 shares
shall be recorded as belonging to aliens.24(emphasis supplied)
In determining the nationality of petitioners, the CA looked into their corporate structures and Thereafter, petitioners appealed the same CA decision to this Court which is now pending with a
their corresponding common shareholders. Using the grandfather rule, the CA discovered that different division.
MBMI in effect owned majority of the common stocks of the petitioners as well as at least 60%
equity interest of other majority shareholders of petitioners through joint venture agreements.
Thus, the instant petition for review against the October 1, 2010 Decision of the CA. Petitioners
The CA found that through a "web of corporate layering, it is clear that one common controlling
put forth the following errors of the CA:
investor in all mining corporations involved x x x is MBMI."25 Thus, it concluded that petitioners
McArthur, Tesoro and Narra are also in partnership with, or privies-in-interest of, MBMI.
I.
Furthermore, the CA viewed the conversion of the MPSA applications of petitioners into FTAA
applications suspicious in nature and, as a consequence, it recommended the rejection of The Court of Appeals erred when it did not dismiss the case for mootness despite the
petitioners’ MPSA applications by the Secretary of the DENR. fact that the subject matter of the controversy, the MPSA Applications, have already
been converted into FTAA applications and that the same have already been granted.
With regard to the settlement of disputes over rights to mining areas, the CA pointed out that the
POA has jurisdiction over them and that it also has the power to determine the of nationality of II.
petitioners as a prerequisite of the Constitution prior the conferring of rights to "co-production,
joint venture or production-sharing agreements" of the state to mining rights. However, it also
The Court of Appeals erred when it did not dismiss the case for lack of jurisdiction
stated that the POA’s jurisdiction is limited only to the resolution of the dispute and not on the
approval or rejection of the MPSAs. It stipulated that only the Secretary of the DENR is vested considering that the Panel of Arbitrators has no jurisdiction to determine the nationality
with the power to approve or reject applications for MPSA. of Narra, Tesoro and McArthur.

Finally, the CA upheld the findings of the POA in its December 14, 2007 Resolution which III.
considered petitioners McArthur, Tesoro and Narra as foreign corporations. Nevertheless, the
CA determined that the POA’s declaration that the MPSAs of McArthur, Tesoro and Narra are The Court of Appeals erred when it did not dismiss the case on account of Redmont’s
void is highly improper. willful forum shopping.

While the petition was pending with the CA, Redmont filed with the Office of the President (OP) IV.
a petition dated May 7, 2010 seeking the cancellation of petitioners’ FTAAs. The OP rendered a
Decision26 on April 6, 2011, wherein it canceled and revoked petitioners’ FTAAs for violating and
circumventing the "Constitution x x x[,] the Small Scale Mining Law and Environmental The Court of Appeals’ ruling that Narra, Tesoro and McArthur are foreign corporations
Compliance Certificate as well as Sections 3 and 8 of the Foreign Investment Act and E.O. based on the "Grandfather Rule" is contrary to law, particularly the express mandate
584."27 The OP, in affirming the cancellation of the issued FTAAs, agreed with Redmont stating of the Foreign Investments Act of 1991, as amended, and the FIA Rules.
that petitioners committed violations against the abovementioned laws and failed to submit
evidence to negate them. The Decision further quoted the December 14, 2007 Order of the POA V.
focusing on the alleged misrepresentation and claims made by petitioners of being domestic or
Filipino corporations and the admitted continued mining operation of PMDC using their locally
secured Small Scale Mining Permit inside the area earlier applied for an MPSA application which The Court of Appeals erred when it applied the exceptions to the res inter alios acta
was eventually transferred to Narra. It also agreed with the POA’s estimation that the filing of the rule.
FTAA applications by petitioners is a clear admission that they are "not capable of conducting a
large scale mining operation and that they need the financial and technical assistance of a VI.
foreign entity in their operation, that is why they sought the participation of MBMI Resources,
Inc."28 The Decision further quoted:
The Court of Appeals erred when it concluded that the conversion of the MPSA
Applications into FTAA Applications were of "suspicious nature" as the same is based
The filing of the FTAA application on June 15, 2007, during the pendency of the case only on mere conjectures and surmises without any shred of evidence to show the same. 31
demonstrate the violations and lack of qualification of the respondent corporations to engage in
mining. The filing of the FTAA application conversion which is allowed foreign corporation of the
earlier MPSA is an admission that indeed the respondent is not Filipino but rather of foreign We find the petition to be without merit.
nationality who is disqualified under the laws. Corporate documents of MBMI Resources, Inc.
furnished its stockholders in their head office in Canada suggest that they are conducting This case not moot and academic
operation only through their local counterparts.29
The claim of petitioners that the CA erred in not rendering the instant case as moot is without
The Motion for Reconsideration of the Decision was further denied by the OP in a merit.
Resolution30 dated July 6, 2011. Petitioners then filed a Petition for Review on Certiorari of the
OP’s Decision and Resolution with the CA, docketed as CA-G.R. SP No. 120409. In the CA
Decision dated February 29, 2012, the CA affirmed the Decision and Resolution of the OP. Basically, a case is said to be moot and/or academic when it "ceases to present a justiciable
controversy by virtue of supervening events, so that a declaration thereon would be of no
practical use or value."32 Thus, the courts "generally decline jurisdiction over the case or dismiss Consider the history of this case and how petitioners responded to every action done by the
it on the ground of mootness."33 court or appropriate government agency: on January 2, 2007, Redmont filed three separate
petitions for denial of the MPSA applications of petitioners before the POA. On June 15, 2007,
petitioners filed a conversion of their MPSA applications to FTAAs. The POA, in its December
The "mootness" principle, however, does accept certain exceptions and the mere raising of an
14, 2007 Resolution, observed this suspect change of applications while the case was pending
issue of "mootness" will not deter the courts from trying a case when there is a valid reason to
before it and held:
do so. In David v. Macapagal-Arroyo (David), the Court provided four instances where courts
can decide an otherwise moot case, thus:
The filing of the Financial or Technical Assistance Agreement application is a clear admission
that the respondents are not capable of conducting a large scale mining operation and that they
1.) There is a grave violation of the Constitution;
need the financial and technical assistance of a foreign entity in their operation that is why they
sought the participation of MBMI Resources, Inc. The participation of MBMI in the corporation
2.) The exceptional character of the situation and paramount public interest is only proves the fact that it is the Canadian company that will provide the finances and the
involved; resources to operate the mining areas for the greater benefit and interest of the same and not
the Filipino stockholders who only have a less substantial financial stake in the corporation.
3.) When constitutional issue raised requires formulation of controlling principles to
guide the bench, the bar, and the public; and xxxx

4.) The case is capable of repetition yet evading review.34 x x x The filing of the FTAA application on June 15, 2007, during the pendency of the case only
demonstrate the violations and lack of qualification of the respondent corporations to engage in
mining. The filing of the FTAA application conversion which is allowed foreign corporation of the
All of the exceptions stated above are present in the instant case. We of this Court note that a earlier MPSA is an admission that indeed the respondent is not Filipino but rather of foreign
grave violation of the Constitution, specifically Section 2 of Article XII, is being committed by a nationality who is disqualified under the laws. Corporate documents of MBMI Resources, Inc.
foreign corporation right under our country’s nose through a myriad of corporate layering under
furnished its stockholders in their head office in Canada suggest that they are conducting
different, allegedly, Filipino corporations. The intricate corporate layering utilized by the operation only through their local counterparts.36
Canadian company, MBMI, is of exceptional character and involves paramount public interest
since it undeniably affects the exploitation of our Country’s natural resources. The corresponding
actions of petitioners during the lifetime and existence of the instant case raise questions as On October 1, 2010, the CA rendered a Decision which partially granted the petition, reversing
what principle is to be applied to cases with similar issues. No definite ruling on such principle and setting aside the September 10, 2008 and July 1, 2009 Orders of the MAB. In the said
has been pronounced by the Court; hence, the disposition of the issues or errors in the instant Decision, the CA upheld the findings of the POA of the DENR that the herein petitioners are in
case will serve as a guide "to the bench, the bar and the public." 35 Finally, the instant case is fact foreign corporations thus a recommendation of the rejection of their MPSA applications were
capable of repetition yet evading review, since the Canadian company, MBMI, can keep on recommended to the Secretary of the DENR. With respect to the FTAA applications or
utilizing dummy Filipino corporations through various schemes of corporate layering and conversion of the MPSA applications to FTAAs, the CA deferred the matter for the determination
conversion of applications to skirt the constitutional prohibition against foreign mining in of the Secretary of the DENR and the President of the Republic of the Philippines.37
Philippine soil.
In their Motion for Reconsideration dated October 26, 2010, petitioners prayed for the dismissal
Conversion of MPSA applications to FTAA applications of the petition asserting that on April 5, 2010, then President Gloria Macapagal-Arroyo signed
and issued in their favor FTAA No. 05-2010-IVB, which rendered the petition moot and
academic. However, the CA, in a Resolution dated February 15, 2011 denied their motion for
We shall discuss the first error in conjunction with the sixth error presented by petitioners since being a mere "rehash of their claims and defenses."38 Standing firm on its Decision, the CA
both involve the conversion of MPSA applications to FTAA applications. Petitioners propound affirmed the ruling that petitioners are, in fact, foreign corporations. On April 5, 2011, petitioners
that the CA erred in ruling against them since the questioned MPSA applications were already
elevated the case to us via a Petition for Review on Certiorari under Rule 45, questioning the
converted into FTAA applications; thus, the issue on the prohibition relating to MPSA Decision of the CA. Interestingly, the OP rendered a Decision dated April 6, 2011, a day after
applications of foreign mining corporations is academic. Also, petitioners would want us to this petition for review was filed, cancelling and revoking the FTAAs, quoting the Order of the
correct the CA’s finding which deemed the aforementioned conversions of applications as
POA and stating that petitioners are foreign corporations since they needed the financial
suspicious in nature, since it is based on mere conjectures and surmises and not supported with strength of MBMI, Inc. in order to conduct large scale mining operations. The OP Decision also
evidence. based the cancellation on the misrepresentation of facts and the violation of the "Small Scale
Mining Law and Environmental Compliance Certificate as well as Sections 3 and 8 of the
We disagree. Foreign Investment Act and E.O. 584."39 On July 6, 2011, the OP issued a Resolution, denying
the Motion for Reconsideration filed by the petitioners.
The CA’s analysis of the actions of petitioners after the case was filed against them by
respondent is on point. The changing of applications by petitioners from one type to another just Respondent Redmont, in its Comment dated October 10, 2011, made known to the Court the
because a case was filed against them, in truth, would raise not a few sceptics’ eyebrows. What fact of the OP’s Decision and Resolution. In their Reply, petitioners chose to ignore the OP
is the reason for such conversion? Did the said conversion not stem from the case challenging Decision and continued to reuse their old arguments claiming that they were granted FTAAs
their citizenship and to have the case dismissed against them for being "moot"? It is quite and, thus, the case was moot. Petitioners filed a Manifestation and Submission dated October
obvious that it is petitioners’ strategy to have the case dismissed against them for being "moot." 19, 2012,40 wherein they asserted that the present petition is moot since, in a remarkable turn of
events, MBMI was able to sell/assign all its shares/interest in the "holding companies" to DMCI Prior to this recent change of events, petitioners were constant in advocating the application of
Mining Corporation (DMCI), a Filipino corporation and, in effect, making their respective the "control test" under RA 7042, as amended by RA 8179, otherwise known as the Foreign
corporations fully-Filipino owned. Investments Act (FIA), rather than using the stricter grandfather rule. The pertinent provision
under Sec. 3 of the FIA provides:
Again, it is quite evident that petitioners have been trying to have this case dismissed for being
"moot." Their final act, wherein MBMI was able to allegedly sell/assign all its shares and interest SECTION 3. Definitions. - As used in this Act:
in the petitioner "holding companies" to DMCI, only proves that they were in fact not Filipino
corporations from the start. The recent divesting of interest by MBMI will not change the stand of
a.) The term Philippine national shall mean a citizen of the Philippines; or a domestic partnership
this Court with respect to the nationality of petitioners prior the suspicious change in their
or association wholly owned by the citizens of the Philippines; a corporation organized under the
corporate structures. The new documents filed by petitioners are factual evidence that this Court
laws of the Philippines of which at least sixty percent (60%) of the capital stock outstanding and
has no power to verify.
entitled to vote is wholly owned by Filipinos or a trustee of funds for pension or other employee
retirement or separation benefits, where the trustee is a Philippine national and at least sixty
The only thing clear and proved in this Court is the fact that the OP declared that petitioner percent (60%) of the fund will accrue to the benefit of Philippine nationals: Provided, That were a
corporations have violated several mining laws and made misrepresentations and falsehood in corporation and its non-Filipino stockholders own stocks in a Securities and Exchange
their applications for FTAA which lead to the revocation of the said FTAAs, demonstrating that Commission (SEC) registered enterprise, at least sixty percent (60%) of the capital stock
petitioners are not beyond going against or around the law using shifty actions and strategies. outstanding and entitled to vote of each of both corporations must be owned and held by citizens
Thus, in this instance, we can say that their claim of mootness is moot in itself because their of the Philippines and at least sixty percent (60%) of the members of the Board of Directors, in
defense of conversion of MPSAs to FTAAs has been discredited by the OP Decision. order that the corporation shall be considered a Philippine national. (emphasis supplied)

Grandfather test The grandfather rule, petitioners reasoned, has no leg to stand on in the instant case since the
definition of a "Philippine National" under Sec. 3 of the FIA does not provide for it. They further
claim that the grandfather rule "has been abandoned and is no longer the applicable
The main issue in this case is centered on the issue of petitioners’ nationality, whether Filipino or
rule."41 They also opined that the last portion of Sec. 3 of the FIA admits the application of a
foreign. In their previous petitions, they had been adamant in insisting that they were Filipino
"corporate layering" scheme of corporations. Petitioners claim that the clear and unambiguous
corporations, until they submitted their Manifestation and Submission dated October 19, 2012
wordings of the statute preclude the court from construing it and prevent the court’s use of
where they stated the alleged change of corporate ownership to reflect their Filipino ownership.
discretion in applying the law. They said that the plain, literal meaning of the statute meant the
Thus, there is a need to determine the nationality of petitioner corporations.
application of the control test is obligatory.

Basically, there are two acknowledged tests in determining the nationality of a corporation: the
We disagree. "Corporate layering" is admittedly allowed by the FIA; but if it is used to circumvent
control test and the grandfather rule. Paragraph 7 of DOJ Opinion No. 020, Series of 2005,
the Constitution and pertinent laws, then it becomes illegal. Further, the pronouncement of
adopting the 1967 SEC Rules which implemented the requirement of the Constitution and other
petitioners that the grandfather rule has already been abandoned must be discredited for lack of
laws pertaining to the controlling interests in enterprises engaged in the exploitation of natural
basis.
resources owned by Filipino citizens, provides:

Art. XII, Sec. 2 of the Constitution provides:


Shares belonging to corporations or partnerships at least 60% of the capital of which is owned
by Filipino citizens shall be considered as of Philippine nationality, but if the percentage of
Filipino ownership in the corporation or partnership is less than 60%, only the number of shares Sec. 2. All lands of the public domain, waters, minerals, coal, petroleum and other mineral oils,
corresponding to such percentage shall be counted as of Philippine nationality. Thus, if 100,000 all forces of potential energy, fisheries, forests or timber, wildlife, flora and fauna, and other
shares are registered in the name of a corporation or partnership at least 60% of the capital natural resources are owned by the State. With the exception of agricultural lands, all other
stock or capital, respectively, of which belong to Filipino citizens, all of the shares shall be natural resources shall not be alienated. The exploration, development, and utilization of natural
recorded as owned by Filipinos. But if less than 60%, or say, 50% of the capital stock or capital resources shall be under the full control and supervision of the State. The State may directly
of the corporation or partnership, respectively, belongs to Filipino citizens, only 50,000 shares undertake such activities, or it may enter into co-production, joint venture or production-sharing
shall be counted as owned by Filipinos and the other 50,000 shall be recorded as belonging to agreements with Filipino citizens, or corporations or associations at least sixty per centum of
aliens. whose capital is owned by such citizens. Such agreements may be for a period not exceeding
twenty-five years, renewable for not more than twenty-five years, and under such terms and
conditions as may be provided by law.
The first part of paragraph 7, DOJ Opinion No. 020, stating "shares belonging to corporations or
partnerships at least 60% of the capital of which is owned by Filipino citizens shall be considered
as of Philippine nationality," pertains to the control test or the liberal rule. On the other hand, the xxxx
second part of the DOJ Opinion which provides, "if the percentage of the Filipino ownership in
the corporation or partnership is less than 60%, only the number of shares corresponding to
The President may enter into agreements with Foreign-owned corporations involving either
such percentage shall be counted as Philippine nationality," pertains to the stricter, more
technical or financial assistance for large-scale exploration, development, and utilization of
stringent grandfather rule.
minerals, petroleum, and other mineral oils according to the general terms and conditions
provided by law, based on real contributions to the economic growth and general welfare of the
country. In such agreements, the State shall promote the development and use of local scientific MR. NOLLEDO: That must be based on the subscribed capital stock, because unless declared
and technical resources. (emphasis supplied) delinquent, unpaid capital stock shall be entitled to vote.

The emphasized portion of Sec. 2 which focuses on the State entering into different types of MR. VILLEGAS: That is right.
agreements for the exploration, development, and utilization of natural resources with entities
who are deemed Filipino due to 60 percent ownership of capital is pertinent to this case, since
MR. NOLLEDO: Thank you.
the issues are centered on the utilization of our country’s natural resources or specifically,
mining. Thus, there is a need to ascertain the nationality of petitioners since, as the Constitution
so provides, such agreements are only allowed corporations or associations "at least 60 percent With respect to an investment by one corporation in another corporation, say, a corporation with
of such capital is owned by such citizens." The deliberations in the Records of the 1986 60-40 percent equity invests in another corporation which is permitted by the Corporation Code,
Constitutional Commission shed light on how a citizenship of a corporation will be determined: does the Committee adopt the grandfather rule?

Mr. BENNAGEN: Did I hear right that the Chairman’s interpretation of an independent national MR. VILLEGAS: Yes, that is the understanding of the Committee.
economy is freedom from undue foreign control? What is the meaning of undue foreign control?
MR. NOLLEDO: Therefore, we need additional Filipino capital?
MR. VILLEGAS: Undue foreign control is foreign control which sacrifices national sovereignty
and the welfare of the Filipino in the economic sphere.
MR. VILLEGAS: Yes.42 (emphasis supplied)

MR. BENNAGEN: Why does it have to be qualified still with the word "undue"? Why not simply
It is apparent that it is the intention of the framers of the Constitution to apply the grandfather
freedom from foreign control? I think that is the meaning of independence, because as phrased,
it still allows for foreign control. rule in cases where corporate layering is present.

Elementary in statutory construction is when there is conflict between the Constitution and a
MR. VILLEGAS: It will now depend on the interpretation because if, for example, we retain the
60/40 possibility in the cultivation of natural resources, 40 percent involves some control; not statute, the Constitution will prevail. In this instance, specifically pertaining to the provisions
total control, but some control. under Art. XII of the Constitution on National Economy and Patrimony, Sec. 3 of the FIA will
have no place of application. As decreed by the honorable framers of our Constitution, the
grandfather rule prevails and must be applied.
MR. BENNAGEN: In any case, I think in due time we will propose some amendments.
Likewise, paragraph 7, DOJ Opinion No. 020, Series of 2005 provides:
MR. VILLEGAS: Yes. But we will be open to improvement of the phraseology.
The above-quoted SEC Rules provide for the manner of calculating the Filipino interest in a
Mr. BENNAGEN: Yes. corporation for purposes, among others, of determining compliance with nationality requirements
(the ‘Investee Corporation’). Such manner of computation is necessary since the shares in the
Investee Corporation may be owned both by individual stockholders (‘Investing Individuals’) and
Thank you, Mr. Vice-President.
by corporations and partnerships (‘Investing Corporation’). The said rules thus provide for the
determination of nationality depending on the ownership of the Investee Corporation and, in
xxxx certain instances, the Investing Corporation.

MR. NOLLEDO: In Sections 3, 9 and 15, the Committee stated local or Filipino equity and Under the above-quoted SEC Rules, there are two cases in determining the nationality of the
foreign equity; namely, 60-40 in Section 3, 60-40 in Section 9, and 2/3-1/3 in Section 15. Investee Corporation. The first case is the ‘liberal rule’, later coined by the SEC as the Control
Test in its 30 May 1990 Opinion, and pertains to the portion in said Paragraph 7 of the 1967
SEC Rules which states, ‘(s)hares belonging to corporations or partnerships at least 60% of the
MR. VILLEGAS: That is right.
capital of which is owned by Filipino citizens shall be considered as of Philippine nationality.’
Under the liberal Control Test, there is no need to further trace the ownership of the 60% (or
MR. NOLLEDO: In teaching law, we are always faced with the question: ‘Where do we base the more) Filipino stockholdings of the Investing Corporation since a corporation which is at least
equity requirement, is it on the authorized capital stock, on the subscribed capital stock, or on 60% Filipino-owned is considered as Filipino.
the paid-up capital stock of a corporation’? Will the Committee please enlighten me on this?
The second case is the Strict Rule or the Grandfather Rule Proper and pertains to the portion in
MR. VILLEGAS: We have just had a long discussion with the members of the team from the UP said Paragraph 7 of the 1967 SEC Rules which states, "but if the percentage of Filipino
Law Center who provided us with a draft. The phrase that is contained here which we adopted ownership in the corporation or partnership is less than 60%, only the number of shares
from the UP draft is ‘60 percent of the voting stock.’ corresponding to such percentage shall be counted as of Philippine nationality." Under the Strict
Rule or Grandfather Rule Proper, the combined totals in the Investing Corporation and the
Investee Corporation must be traced (i.e., "grandfathered") to determine the total percentage of Madridejos Mining Filipino 5,997 PhP 5,997,000.00 PhP
Filipino ownership. Corporation 825,000.00

Moreover, the ultimate Filipino ownership of the shares must first be traced to the level of the MBMI Resources, Canadian 3,998 PhP 3,998,000.0 PhP
Investing Corporation and added to the shares directly owned in the Investee Corporation x x x. Inc. 1,878,174.6
0

xxxx Lauro L. Salazar Filipino 1 PhP 1,000.00 PhP


1,000.00
In other words, based on the said SEC Rule and DOJ Opinion, the Grandfather Rule or the Fernando B. Filipino 1 PhP 1,000.00 PhP
second part of the SEC Rule applies only when the 60-40 Filipino-foreign equity ownership is in Esguerra 1,000.00
doubt (i.e., in cases where the joint venture corporation with Filipino and foreign stockholders
with less than 60% Filipino stockholdings [or 59%] invests in other joint venture corporation Manuel A. Agcaoili Filipino 1 PhP 1,000.00 PhP
which is either 60-40% Filipino-alien or the 59% less Filipino). Stated differently, where the 60-40 1,000.00
Filipino- foreign equity ownership is not in doubt, the Grandfather Rule will not apply. (emphasis
supplied) Michael T. Mason American 1 PhP 1,000.00 PhP
1,000.00
After a scrutiny of the evidence extant on record, the Court finds that this case calls for the Kenneth Cawkell Canadian 1 PhP 1,000.00 PhP
application of the grandfather rule since, as ruled by the POA and affirmed by the OP, doubt 1,000.00
prevails and persists in the corporate ownership of petitioners. Also, as found by the CA, doubt
is present in the 60-40 Filipino equity ownership of petitioners Narra, McArthur and Tesoro, since Total 10,000 PhP 10,000,000.00 PhP
their common investor, the 100% Canadian corporation––MBMI, funded them. However, 2,708,174.6
petitioners also claim that there is "doubt" only when the stockholdings of Filipinos are less than 0
60%.43 (emphasis
supplied)
The assertion of petitioners that "doubt" only exists when the stockholdings are less than 60%
fails to convince this Court. DOJ Opinion No. 20, which petitioners quoted in their petition, only Interestingly, looking at the corporate structure of MMC, we take note that it has a similar
made an example of an instance where "doubt" as to the ownership of the corporation exists. It
structure and composition as McArthur. In fact, it would seem that MBMI is also a major investor
would be ludicrous to limit the application of the said word only to the instances where the and "controls"45 MBMI and also, similar nominal shareholders were present, i.e. Fernando B.
stockholdings of non-Filipino stockholders are more than 40% of the total stockholdings in a Esguerra (Esguerra), Lauro L. Salazar (Salazar), Michael T. Mason (Mason) and Kenneth
corporation. The corporations interested in circumventing our laws would clearly strive to have
Cawkell (Cawkell):
"60% Filipino Ownership" at face value. It would be senseless for these applying corporations to
state in their respective articles of incorporation that they have less than 60% Filipino
stockholders since the applications will be denied instantly. Thus, various corporate schemes Madridejos Mining Corporation
and layerings are utilized to circumvent the application of the Constitution.

Name Nationality Number of Amount Amount


Obviously, the instant case presents a situation which exhibits a scheme employed by Shares Subscribed Paid
stockholders to circumvent the law, creating a cloud of doubt in the Court’s mind. To determine,
therefore, the actual participation, direct or indirect, of MBMI, the grandfather rule must be used. Olympic Mines Filipino 6,663 PhP 6,663,000.00 PhP 0
&DevelopmentCor
McArthur Mining, Inc. p.

MBMI Canadian 3,331 PhP 3,331,000.00 PhP


To establish the actual ownership, interest or participation of MBMI in each of petitioners’ Resources,Inc. 2,803,900.00
corporate structure, they have to be "grandfathered."
Amanti Limson Filipino 1 PhP 1,000.00 PhP
1,000.00
As previously discussed, McArthur acquired its MPSA application from MMC, which acquired its
application from SMMI. McArthur has a capital stock of ten million pesos (PhP 10,000,000) Fernando Filipino 1 PhP 1,000.00 PhP
divided into 10,000 common shares at one thousand pesos (PhP 1,000) per share, subscribed B.Esguerra 1,000.00
to by the following:44
Lauro Salazar Filipino 1 PhP 1,000.00 PhP
1,000.00
Name Nationality Number of Amount Amount
Shares Subscribed Paid Emmanuel Filipino 1 PhP 1,000.00 PhP
G.Hernando 1,000.00
Michael T. Mason American 1 PhP 1,000.00 PhP B.Esguerra
1,000.00

Kenneth Cawkell Canadian 1 PhP 1,000.00 PhP Manuel Filipino 1 PhP 1,000.00 PhP 1,000.00
1,000.00 A.Agcaoili

Total 10,000 PhP PhP Michael T. American 1 PhP 1,000.00 PhP 1,000.00
10,000,000.00 2,809,900.00 Mason
(emphasis
supplied) Kenneth Canadian 1 PhP 1,000.00 PhP 1,000.00
Cawkell
Noticeably, Olympic Mines & Development Corporation (Olympic) did not pay any amount with
Total 10,000 PhP 10,000,000.00 PhP
respect to the number of shares they subscribed to in the corporation, which is quite absurd
2,708,174.60(
since Olympic is the major stockholder in MMC. MBMI’s 2006 Annual Report sheds light on why
emphasis
Olympic failed to pay any amount with respect to the number of shares it subscribed to. It states
supplied)
that Olympic entered into joint venture agreements with several Philippine companies, wherein it
holds directly and indirectly a 60% effective equity interest in the Olympic Properties.46 Quoting
the said Annual report:
Except for the name "Sara Marie Mining, Inc.," the table above shows exactly the same figures
as the corporate structure of petitioner McArthur, down to the last centavo. All the other
On September 9, 2004, the Company and Olympic Mines & Development Corporation shareholders are the same: MBMI, Salazar, Esguerra, Agcaoili, Mason and Cawkell. The figures
("Olympic") entered into a series of agreements including a Property Purchase and Development under "Nationality,""Number of Shares,""Amount Subscribed," and "Amount Paid" are exactly
Agreement (the Transaction Documents) with respect to three nickel laterite properties in the same. Delving deeper, we scrutinize SMMI’s corporate structure:
Palawan, Philippines (the "Olympic Properties"). The Transaction Documents effectively
establish a joint venture between the Company and Olympic for purposes of developing the
Olympic Properties. The Company holds directly and indirectly an initial 60% interest in the joint Sara Marie Mining, Inc.
venture. Under certain circumstances and upon achieving certain milestones, the Company may
earn up to a 100% interest, subject to a 2.5% net revenue royalty.47 (emphasis supplied)
Name Nationality Number AmountSubscrib Amount
ofShares ed Paid
Thus, as demonstrated in this first corporation, McArthur, when it is "grandfathered," company
layering was utilized by MBMI to gain control over McArthur. It is apparent that MBMI has more
Olympic Mines Filipino 6,663 PhP 6,663,000.00 PhP 0
than 60% or more equity interest in McArthur, making the latter a foreign corporation.
&DevelopmentCorp
.
Tesoro Mining and Development, Inc.
MBMI Canadian 3,331 PhP 3,331,000.00 PhP
Tesoro, which acquired its MPSA application from SMMI, has a capital stock of ten million pesos Resources,Inc. 2,794,000.00
(PhP 10,000,000) divided into ten thousand (10,000) common shares at PhP 1,000 per share,
as demonstrated below: Amanti Limson Filipino 1 PhP 1,000.00 PhP
1,000.00

Name Nationality Number AmountSubscribed Amount Paid Fernando Filipino 1 PhP 1,000.00 PhP
ofShares B.Esguerra 1,000.00

Sara Filipino 5,997 PhP 5,997,000.00 PhP Lauro Salazar Filipino 1 PhP 1,000.00 PhP
MarieMining, 825,000.00 1,000.00
Inc.
Emmanuel Filipino 1 PhP 1,000.00 PhP
MBMIResour Canadian 3,998 PhP 3,998,000.00 PhP G.Hernando 1,000.00
ces, Inc. 1,878,174.60
Michael T. Mason American 1 PhP 1,000.00 PhP
Lauro L. Filipino 1 PhP 1,000.00 PhP 1,000.00 1,000.00
Salazar
Kenneth Cawkell Canadian 1 PhP 1,000.00 PhP
Fernando Filipino 1 PhP 1,000.00 PhP 1,000.00 1,000.00
Total 10,000 PhP PhP Bayani H. Agabin Filipino 1 PhP 1,000.00 PhP 1,000.00
10,000,000.00 2,809,900.00
(emphasis Robert L. American 1 PhP 1,000.00 PhP 1,000.00
supplied)
McCurdy
After subsequently studying SMMI’s corporate structure, it is not farfetched for us to spot the
glaring similarity between SMMI and MMC’s corporate structure. Again, the presence of identical Kenneth Cawkell Canadian 1 PhP 1,000.00 PhP 1,000.00
stockholders, namely: Olympic, MBMI, Amanti Limson (Limson), Esguerra, Salazar, Hernando,
Mason and Cawkell. The figures under the headings "Nationality,""Number of Shares,""Amount Total 10,000 PhP PhP 2,800,000.00
Subscribed," and "Amount Paid" are exactly the same except for the amount paid by MBMI 10,000,000.00 (emphasis supplied)
which now reflects the amount of two million seven hundred ninety four thousand pesos (PhP
2,794,000). Oddly, the total value of the amount paid is two million eight hundred nine thousand
nine hundred pesos (PhP 2,809,900). Again, MBMI, along with other nominal stockholders, i.e., Mason, Agcaoili and Esguerra, is
present in this corporate structure.
Accordingly, after "grandfathering" petitioner Tesoro and factoring in Olympic’s participation in
SMMI’s corporate structure, it is clear that MBMI is in control of Tesoro and owns 60% or more Patricia Louise Mining & Development Corporation
equity interest in Tesoro. This makes petitioner Tesoro a non-Filipino corporation and, thus,
disqualifies it to participate in the exploitation, utilization and development of our natural
resources. Using the grandfather method, we further look and examine PLMDC’s corporate structure:

Narra Nickel Mining and Development Corporation Name Nationality Number of Amount Amount Paid
Shares Subscribed
Moving on to the last petitioner, Narra, which is the transferee and assignee of PLMDC’s MPSA Palawan Alpha Filipino 6,596 PhP 6,596,000.00 PhP 0
application, whose corporate structure’s arrangement is similar to that of the first two petitioners South Resources
discussed. The capital stock of Narra is ten million pesos (PhP 10,000,000), which is divided into Development
ten thousand common shares (10,000) at one thousand pesos (PhP 1,000) per share, shown as Corporation
follows:
MBMI Canadian 3,396 PhP 3,396,000.00 PhP 2,796,000.00
Resources,Inc.
Name Nationality Number AmountSubscrib Amount Paid
ofShares ed Higinio C. Filipino 1 PhP 1,000.00 PhP 1,000.00
Mendoza, Jr.
Patricia Filipino 5,997 PhP 5,997,000.00 PhP 1,677,000.00
Fernando B. Filipino 1 PhP 1,000.00 PhP 1,000.00
LouiseMining
Esguerra
&DevelopmentC
orp. Henry E. Filipino 1 PhP 1,000.00 PhP 1,000.00
Fernandez
MBMIResources Canadian 3,998 PhP 3,996,000.00 PhP 1,116,000.00
, Inc. Lauro L. Salazar Filipino 1 PhP 1,000.00 PhP 1,000.00

Manuel A. Agcaoili Filipino 1 PhP 1,000.00 PhP 1,000.00


Higinio Filipino 1 PhP 1,000.00 PhP 1,000.00
C.Mendoza, Jr. Bayani H. Agabin Filipino 1 PhP 1,000.00 PhP 1,000.00

Henry Filipino 1 PhP 1,000.00 PhP 1,000.00 Michael T. Mason American 1 PhP 1,000.00 PhP 1,000.00
E.Fernandez
Kenneth Cawkell Canadian 1 PhP 1,000.00 PhP 1,000.00
Manuel Filipino 1 PhP 1,000.00 PhP 1,000.00 Total 10,000 PhP 10,000,000.00 PhP 2,708,174.60
A.Agcaoili (emphasis supplied)

Ma. Elena Filipino 1 PhP 1,000.00 PhP 1,000.00


A.Bocalan Yet again, the usual players in petitioners’ corporate structures are present. Similarly, the
amount of money paid by the 2nd tier majority stock holder, in this case, Palawan Alpha South
Resources and Development Corp. (PASRDC), is zero.
Studying MBMI’s Summary of Significant Accounting Policies dated October 31, 2005 explains Application of the res inter alios acta rule
the reason behind the intricate corporate layering that MBMI immersed itself in:
Petitioners question the CA’s use of the exception of the res inter alios acta or the "admission by
JOINT VENTURES The Company’s ownership interests in various mining ventures engaged in co-partner or agent" rule and "admission by privies" under the Rules of Court in the instant case,
the acquisition, exploration and development of mineral properties in the Philippines is described by pointing out that statements made by MBMI should not be admitted in this case since it is not
as follows: a party to the case and that it is not a "partner" of petitioners.

(a) Olympic Group Secs. 29 and 31, Rule 130 of the Revised Rules of Court provide:

The Philippine companies holding the Olympic Property, and the ownership and interests Sec. 29. Admission by co-partner or agent.- The act or declaration of a partner or agent of the
therein, are as follows: party within the scope of his authority and during the existence of the partnership or agency,
may be given in evidence against such party after the partnership or agency is shown by
evidence other than such act or declaration itself. The same rule applies to the act or declaration
Olympic- Philippines (the "Olympic Group")
of a joint owner, joint debtor, or other person jointly interested with the party.

Sara Marie Mining Properties Ltd. ("Sara Marie") 33.3%


Sec. 31. Admission by privies.- Where one derives title to property from another, the act,
declaration, or omission of the latter, while holding the title, in relation to the property, is
Tesoro Mining & Development, Inc. (Tesoro) 60.0% evidence against the former.

Pursuant to the Olympic joint venture agreement the Company holds directly and indirectly an Petitioners claim that before the above-mentioned Rule can be applied to a case, "the
effective equity interest in the Olympic Property of 60.0%. Pursuant to a shareholders’ partnership relation must be shown, and that proof of the fact must be made by evidence other
agreement, the Company exercises joint control over the companies in the Olympic Group. than the admission itself."49 Thus, petitioners assert that the CA erred in finding that a
partnership relationship exists between them and MBMI because, in fact, no such partnership
exists.
(b) Alpha Group

Partnerships vs. joint venture agreements


The Philippine companies holding the Alpha Property, and the ownership interests therein, are
as follows:
Petitioners claim that the CA erred in applying Sec. 29, Rule 130 of the Rules by stating that "by
entering into a joint venture, MBMI have a joint interest" with Narra, Tesoro and McArthur. They
Alpha- Philippines (the "Alpha Group") challenged the conclusion of the CA which pertains to the close characteristics of

Patricia Louise Mining Development Inc. ("Patricia") 34.0%


"partnerships" and "joint venture agreements." Further, they asserted that before this particular
partnership can be formed, it should have been formally reduced into writing since the capital
Narra Nickel Mining & Development Corporation (Narra) 60.4% involved is more than three thousand pesos (PhP 3,000). Being that there is no evidence of
written agreement to form a partnership between petitioners and MBMI, no partnership was
created.
Under a joint venture agreement the Company holds directly and indirectly an effective equity
interest in the Alpha Property of 60.4%. Pursuant to a shareholders’ agreement, the Company
exercises joint control over the companies in the Alpha Group. 48 (emphasis supplied) We disagree.

Concluding from the above-stated facts, it is quite safe to say that petitioners McArthur, Tesoro A partnership is defined as two or more persons who bind themselves to contribute money,
and Narra are not Filipino since MBMI, a 100% Canadian corporation, owns 60% or more of property, or industry to a common fund with the intention of dividing the profits among
their equity interests. Such conclusion is derived from grandfathering petitioners’ corporate themselves.50 On the other hand, joint ventures have been deemed to be "akin" to partnerships
owners, namely: MMI, SMMI and PLMDC. Going further and adding to the picture, MBMI’s since it is difficult to distinguish between joint ventures and partnerships. Thus:
Summary of Significant Accounting Policies statement– –regarding the "joint venture"
agreements that it entered into with the "Olympic" and "Alpha" groups––involves SMMI, Tesoro, [T]he relations of the parties to a joint venture and the nature of their association are so similar
PLMDC and Narra. Noticeably, the ownership of the "layered" corporations boils down to MBMI,
and closely akin to a partnership that it is ordinarily held that their rights, duties, and liabilities are
Olympic or corporations under the "Alpha" group wherein MBMI has joint venture agreements to be tested by rules which are closely analogous to and substantially the same, if not exactly
with, practically exercising majority control over the corporations mentioned. In effect, whether the same, as those which govern partnership. In fact, it has been said that the trend in the law
looking at the capital structure or the underlying relationships between and among the
has been to blur the distinctions between a partnership and a joint venture, very little law being
corporations, petitioners are NOT Filipino nationals and must be considered foreign since 60% found applicable to one that does not apply to the other.51
or more of their capital stocks or equity interests are owned by MBMI.
Though some claim that partnerships and joint ventures are totally different animals, there are publication/posting/radio announcement have been complied with. Any adverse claim, protest,
very few rules that differentiate one from the other; thus, joint ventures are deemed "akin" or opposition shall be filed directly, within thirty (30) calendar days from the last date of
similar to a partnership. In fact, in joint venture agreements, rules and legal incidents governing publication/posting/radio announcement, with the concerned Regional Office or through any
partnerships are applied.52 concerned PENRO or CENRO for filing in the concerned Regional Office for purposes of its
resolution by the Panel of Arbitrators pursuant to the provisions of this Act and these
implementing rules and regulations. Upon final resolution of any adverse claim, protest or
Accordingly, culled from the incidents and records of this case, it can be assumed that the
opposition, the Panel of Arbitrators shall likewise issue a certification to that effect within five (5)
relationships entered between and among petitioners and MBMI are no simple "joint venture
working days from the date of finality of resolution thereof. Where there is no adverse claim,
agreements." As a rule, corporations are prohibited from entering into partnership agreements;
protest or opposition, the Panel of Arbitrators shall likewise issue a Certification to that effect
consequently, corporations enter into joint venture agreements with other corporations or
within five working days therefrom.
partnerships for certain transactions in order to form "pseudo partnerships."

xxxx
Obviously, as the intricate web of "ventures" entered into by and among petitioners and MBMI
was executed to circumvent the legal prohibition against corporations entering into partnerships,
then the relationship created should be deemed as "partnerships," and the laws on partnership No Mineral Agreement shall be approved unless the requirements under this Section are fully
should be applied. Thus, a joint venture agreement between and among corporations may be complied with and any adverse claim/protest/opposition is finally resolved by the Panel of
seen as similar to partnerships since the elements of partnership are present. Arbitrators.

Considering that the relationships found between petitioners and MBMI are considered to be Sec. 41.
partnerships, then the CA is justified in applying Sec. 29, Rule 130 of the Rules by stating that
"by entering into a joint venture, MBMI have a joint interest" with Narra, Tesoro and McArthur.
xxxx

Panel of Arbitrators’ jurisdiction


Within fifteen (15) working days form the receipt of the Certification issued by the Panel of
Arbitrators as provided in Section 38 hereof, the concerned Regional Director shall initially
We affirm the ruling of the CA in declaring that the POA has jurisdiction over the instant case. evaluate the Mineral Agreement applications in areas outside Mineral reservations. He/She shall
The POA has jurisdiction to settle disputes over rights to mining areas which definitely involve thereafter endorse his/her findings to the Bureau for further evaluation by the Director within
the petitions filed by Redmont against petitioners Narra, McArthur and Tesoro. Redmont, by fifteen (15) working days from receipt of forwarded documents. Thereafter, the Director shall
filing its petition against petitioners, is asserting the right of Filipinos over mining areas in the endorse the same to the secretary for consideration/approval within fifteen working days from
Philippines against alleged foreign-owned mining corporations. Such claim constitutes a receipt of such endorsement.
"dispute" found in Sec. 77 of RA 7942:
In case of Mineral Agreement applications in areas with Mineral Reservations, within fifteen (15)
Within thirty (30) days, after the submission of the case by the parties for the decision, the panel working days from receipt of the Certification issued by the Panel of Arbitrators as provided for in
shall have exclusive and original jurisdiction to hear and decide the following: Section 38 hereof, the same shall be evaluated and endorsed by the Director to the Secretary
for consideration/approval within fifteen days from receipt of such endorsement. (emphasis
supplied)
(a) Disputes involving rights to mining areas

It has been made clear from the aforecited provisions that the "disputes involving rights to
(b) Disputes involving mineral agreements or permits
mining areas" under Sec. 77(a) specifically refer only to those disputes relative to the
applications for a mineral agreement or conferment of mining rights.
We held in Celestial Nickel Mining Exploration Corporation v. Macroasia Corp.: 53
The jurisdiction of the POA over adverse claims, protest, or oppositions to a mining right
The phrase "disputes involving rights to mining areas" refers to any adverse claim, protest, or application is further elucidated by Secs. 219 and 43 of DENR AO 95-936, which read:
opposition to an application for mineral agreement. The POA therefore has the jurisdiction to
resolve any adverse claim, protest, or opposition to a pending application for a mineral
Sec. 219. Filing of Adverse Claims/Conflicts/Oppositions.- Notwithstanding the provisions of
agreement filed with the concerned Regional Office of the MGB. This is clear from Secs. 38 and
Sections 28, 43 and 57 above, any adverse claim, protest or opposition specified in said
41 of the DENR AO 96-40, which provide:
sections may also be filed directly with the Panel of Arbitrators within the concerned periods for
filing such claim, protest or opposition as specified in said Sections.
Sec. 38.
Sec. 43. Publication/Posting of Mineral Agreement.-
xxxx
xxxx
Within thirty (30) calendar days from the last date of publication/posting/radio announcements,
the authorized officer(s) of the concerned office(s) shall issue a certification(s) that the
The Regional Director or concerned Regional Director shall also cause the posting of the These provisions lead us to conclude that the power of the POA to resolve any adverse claim,
application on the bulletin boards of the Bureau, concerned Regional office(s) and in the opposition, or protest relative to mining rights under Sec. 77(a) of RA 7942 is confined only to
concerned province(s) and municipality(ies), copy furnished the barangays where the proposed adverse claims, conflicts and oppositions relating to applications for the grant of mineral rights.
contract area is located once a week for two (2) consecutive weeks in a language generally
understood in the locality. After forty-five (45) days from the last date of publication/posting has
POA’s jurisdiction is confined only to resolutions of such adverse claims, conflicts and
been made and no adverse claim, protest or opposition was filed within the said forty-five (45)
oppositions and it has no authority to approve or reject said applications. Such power is vested
days, the concerned offices shall issue a certification that publication/posting has been made
in the DENR Secretary upon recommendation of the MGB Director. Clearly, POA’s jurisdiction
and that no adverse claim, protest or opposition of whatever nature has been filed. On the other
over "disputes involving rights to mining areas" has nothing to do with the cancellation of existing
hand, if there be any adverse claim, protest or opposition, the same shall be filed within forty-five
mineral agreements. (emphasis ours)
(45) days from the last date of publication/posting, with the Regional Offices concerned, or
through the Department’s Community Environment and Natural Resources Officers (CENRO) or
Provincial Environment and Natural Resources Officers (PENRO), to be filed at the Regional Accordingly, as we enunciated in Celestial, the POA unquestionably has jurisdiction to resolve
Office for resolution of the Panel of Arbitrators. However previously published valid and disputes over MPSA applications subject of Redmont’s petitions. However, said jurisdiction does
subsisting mining claims are exempted from posted/posting required under this Section. not include either the approval or rejection of the MPSA applications, which is vested only upon
the Secretary of the DENR. Thus, the finding of the POA, with respect to the rejection of
petitioners’ MPSA applications being that they are foreign corporation, is valid.
No mineral agreement shall be approved unless the requirements under this section are fully
complied with and any opposition/adverse claim is dealt with in writing by the Director and
resolved by the Panel of Arbitrators. (Emphasis supplied.) Justice Marvic Mario Victor F. Leonen, in his Dissent, asserts that it is the regular courts, not the
POA, that has jurisdiction over the MPSA applications of petitioners.
It has been made clear from the aforecited provisions that the "disputes involving rights to
mining areas" under Sec. 77(a) specifically refer only to those disputes relative to the This postulation is incorrect.
applications for a mineral agreement or conferment of mining rights.
It is basic that the jurisdiction of the court is determined by the statute in force at the time of the
The jurisdiction of the POA over adverse claims, protest, or oppositions to a mining right commencement of the action.54
application is further elucidated by Secs. 219 and 43 of DENRO AO 95-936, which reads:
Sec. 19, Batas Pambansa Blg. 129 or "The Judiciary Reorganization
Sec. 219. Filing of Adverse Claims/Conflicts/Oppositions.- Notwithstanding the provisions of
Sections 28, 43 and 57 above, any adverse claim, protest or opposition specified in said
Act of 1980" reads:
sections may also be filed directly with the Panel of Arbitrators within the concerned periods for
filing such claim, protest or opposition as specified in said Sections.
Sec. 19. Jurisdiction in Civil Cases.—Regional Trial Courts shall exercise exclusive original
Sec. 43. Publication/Posting of Mineral Agreement Application.- jurisdiction:

xxxx 1. In all civil actions in which the subject of the litigation is incapable of pecuniary estimation.

The Regional Director or concerned Regional Director shall also cause the posting of the On the other hand, the jurisdiction of POA is unequivocal from Sec. 77 of RA 7942:
application on the bulletin boards of the Bureau, concerned Regional office(s) and in the
concerned province(s) and municipality(ies), copy furnished the barangays where the proposed Section 77. Panel of Arbitrators.—
contract area is located once a week for two (2) consecutive weeks in a language generally
understood in the locality. After forty-five (45) days from the last date of publication/posting has
been made and no adverse claim, protest or opposition was filed within the said forty-five (45) x x x Within thirty (30) days, after the submission of the case by the parties for the
days, the concerned offices shall issue a certification that publication/posting has been made decision, the panel shall have exclusive and original jurisdiction to hear and decide the
and that no adverse claim, protest or opposition of whatever nature has been filed. On the other following:
hand, if there be any adverse claim, protest or opposition, the same shall be filed within forty-five
(45) days from the last date of publication/posting, with the Regional offices concerned, or (c) Disputes involving rights to mining areas
through the Department’s Community Environment and Natural Resources Officers (CENRO) or
Provincial Environment and Natural Resources Officers (PENRO), to be filed at the Regional
Office for resolution of the Panel of Arbitrators. However, previously published valid and (d) Disputes involving mineral agreements or permits
subsisting mining claims are exempted from posted/posting required under this Section.
It is clear that POA has exclusive and original jurisdiction over any and all disputes involving
No mineral agreement shall be approved unless the requirements under this section are fully rights to mining areas. One such dispute is an MPSA application to which an adverse claim,
complied with and any opposition/adverse claim is dealt with in writing by the Director and protest or opposition is filed by another interested applicant. In the case at bar, the dispute arose
resolved by the Panel of Arbitrators. (Emphasis supplied.) or originated from MPSA applications where petitioners are asserting their rights to mining areas
subject of their respective MPSA applications. Since respondent filed 3 separate petitions for the
denial of said applications, then a controversy has developed between the parties and it is
POA’s jurisdiction to resolve said disputes.

Moreover, the jurisdiction of the RTC involves civil actions while what petitioners filed with the
DENR Regional Office or any concerned DENRE or CENRO are MPSA applications. Thus POA
has jurisdiction.

Furthermore, the POA has jurisdiction over the MPSA applications under the doctrine of primary
jurisdiction. Euro-med Laboratories v. Province of Batangas55 elucidates:

The doctrine of primary jurisdiction holds that if a case is such that its determination requires the
expertise, specialized training and knowledge of an administrative body, relief must first be
obtained in an administrative proceeding before resort to the courts is had even if the matter
may well be within their proper jurisdiction.

Whatever may be the decision of the POA will eventually reach the court system via a resort to
the CA and to this Court as a last recourse.

Selling of MBMI’s shares to DMCI

As stated before, petitioners’ Manifestation and Submission dated October 19, 2012 would want
us to declare the instant petition moot and academic due to the transfer and conveyance of all
the shareholdings and interests of MBMI to DMCI, a corporation duly organized and existing
under Philippine laws and is at least 60% Philippine-owned.56 Petitioners reasoned that they now
cannot be considered as foreign-owned; the transfer of their shares supposedly cured the
"defect" of their previous nationality. They claimed that their current FTAA contract with the State
should stand since "even wholly-owned foreign corporations can enter into an FTAA with the
State."57Petitioners stress that there should no longer be any issue left as regards their
qualification to enter into FTAA contracts since they are qualified to engage in mining activities in
the Philippines. Thus, whether the "grandfather rule" or the "control test" is used, the
nationalities of petitioners cannot be doubted since it would pass both tests.

The sale of the MBMI shareholdings to DMCI does not have any bearing in the instant case and
said fact should be disregarded. The manifestation can no longer be considered by us since it is
being tackled in G.R. No. 202877 pending before this Court. Thus, the question of whether
petitioners, allegedly a Philippine-owned corporation due to the sale of MBMI's shareholdings to
DMCI, are allowed to enter into FTAAs with the State is a non-issue in this case.

In ending, the "control test" is still the prevailing mode of determining whether or not a
corporation is a Filipino corporation, within the ambit of Sec. 2, Art. II of the 1987 Constitution,
entitled to undertake the exploration, development and utilization of the natural resources of the
Philippines. When in the mind of the Court there is doubt, based on the attendant facts and
circumstances of the case, in the 60-40 Filipino-equity ownership in the corporation, then it may
apply the "grandfather rule."

WHEREFORE, premises considered, the instant petition is DENIED. The assailed Court of
Appeals Decision dated October 1, 2010 and Resolution dated February 15, 2011 are hereby
AFFIRMED.

SO ORDERED.
G. R. No. 164317 February 6, 2006
1857 12-09-80 03-09-81 P197,843.61 3,000 pcs. (15 bundles
calorized lance pipes [)]
ALFREDO CHING, Petitioner,
vs. 1895 12-17-80 03-17-81 P67,652.04 Spare parts for
THE SECRETARY OF JUSTICE, ASST. CITY PROSECUTOR ECILYN BURGOS- Spectrophotometer
VILLAVERT, JUDGE EDGARDO SUDIAM of the Regional Trial Court, Manila, Branch 52;
RIZAL COMMERCIAL BANKING CORP. and THE PEOPLE OF THE 1911 12-22-80 03-20-81 P91,497.85 50 pcs. Ingot moulds
PHILIPPINES, Respondents.
2041 01-30-81 04-30-81 P91,456.97 50 pcs. Ingot moulds
DECISION
2099 02-10-81 05-11-81 P66,162.26 8 pcs. Kubota Rolls for
rolling mills
CALLEJO, SR., J.:
2100 02-10-81 05-12-81 P210,748.00 Spare parts for
Before the Court is a petition for review on certiorari of the Decision1 of the Court of Appeals Lacolaboratory
(CA) in CA-G.R. SP No. 57169 dismissing the petition for certiorari, prohibition and mandamus Equipment5
filed by petitioner Alfredo Ching, and its Resolution2 dated June 28, 2004 denying the motion for
reconsideration thereof.
Under the receipts, petitioner agreed to hold the goods in trust for the said bank, with authority to
Petitioner was the Senior Vice-President of Philippine Blooming Mills, Inc. (PBMI). Sometime in sell but not by way of conditional sale, pledge or otherwise; and in case such goods were sold,
September to October 1980, PBMI, through petitioner, applied with the Rizal Commercial to turn over the proceeds thereof as soon as received, to apply against the relative acceptances
Banking Corporation (respondent bank) for the issuance of commercial letters of credit to and payment of other indebtedness to respondent bank. In case the goods remained unsold
finance its importation of assorted goods.3 within the specified period, the goods were to be returned to respondent bank without any need
of demand. Thus, said "goods, manufactured products or proceeds thereof, whether in the form
of money or bills, receivables, or accounts separate and capable of identification" were
Respondent bank approved the application, and irrevocable letters of credit were issued in favor respondent bank’s property.
of petitioner. The goods were purchased and delivered in trust to PBMI. Petitioner signed 13
trust receipts4 as surety, acknowledging delivery of the following goods:
When the trust receipts matured, petitioner failed to return the goods to respondent bank, or to
return their value amounting to ₱6,940,280.66 despite demands. Thus, the bank filed a criminal
complaint for estafa6 against petitioner in the Office of the City Prosecutor of Manila.
T/R Date Maturity Principal Description of Goods
Nos. Granted Date
After the requisite preliminary investigation, the City Prosecutor found probable cause estafa
1845 12-05-80 03-05-81 P1,596,470.05 79.9425 M/T "SDK" under Article 315, paragraph 1(b) of the Revised Penal Code, in relation to Presidential Decree
Brand Synthetic Graphite (P.D.) No. 115, otherwise known as the Trust Receipts Law. Thirteen (13) Informations were
Electrode filed against the petitioner before the Regional Trial Court (RTC) of Manila. The cases were
docketed as Criminal Cases No. 86-42169 to 86-42181, raffled to Branch 31 of said court.
1853 12-08-80 03-06-81 P198,150.67 3,000 pcs. (15 bundles)
Calorized Lance Pipes Petitioner appealed the resolution of the City Prosecutor to the then Minister of Justice. The
appeal was dismissed in a Resolution7 dated March 17, 1987, and petitioner moved for its
1824 11-28-80 02-26-81 P707,879.71 One Lot High Fired reconsideration. On December 23, 1987, the Minister of Justice granted the motion, thus
Refractory Tundish reversing the previous resolution finding probable cause against petitioner.8 The City Prosecutor
Bricks was ordered to move for the withdrawal of the Informations.

1798 11-21-80 02-19-81 P835,526.25 5 cases spare parts for


CCM This time, respondent bank filed a motion for reconsideration, which, however, was denied on
February 24, 1988.9The RTC, for its part, granted the Motion to Quash the Informations filed by
petitioner on the ground that the material allegations therein did not amount to estafa.10
1808 11-21-80 02-19-81 P370,332.52 200 pcs. ingot moulds

2042 01-30-81 04-30-81 P469,669.29 High Fired Refractory In the meantime, the Court rendered judgment in Allied Banking Corporation v.
Nozzle Bricks Ordoñez,11 holding that the penal provision of P.D. No. 115 encompasses any act violative of an
obligation covered by the trust receipt; it is not limited to transactions involving goods which are
1801 11-21-80 02-19-81 P2,001,715.17 Synthetic Graphite to be sold (retailed), reshipped, stored or processed as a component of a product ultimately sold.
Electrode [with] tapered The Court also ruled that "the non-payment of the amount covered by a trust receipt is an act
pitch filed nipples violative of the obligation of the entrustee to pay."12
On February 27, 1995, respondent bank re-filed the criminal complaint for estafa against 2. THE RESPONDENT SECRETARY OF JUSTICE COMMITTED AN ACT IN GRAVE
petitioner before the Office of the City Prosecutor of Manila. The case was docketed as I.S. No. ABUSE OF DISCRETION AND IN EXCESS OF HIS JURISDICTION WHEN THEY
95B-07614. CONTINUED PROSECUTION OF THE PETITIONER DESPITE THE LENGTH OF
TIME INCURRED IN THE TERMINATION OF THE PRELIMINARY INVESTIGATION
THAT SHOULD JUSTIFY THE DISMISSAL OF THE INSTANT CASE.
Preliminary investigation ensued. On December 8, 1995, the City Prosecutor ruled that there
was no probable cause to charge petitioner with violating P.D. No. 115, as petitioner’s liability
was only civil, not criminal, having signed the trust receipts as surety.13 Respondent bank 3. THE RESPONDENT SECRETARY OF JUSTICE AND ASSISTANT CITY
appealed the resolution to the Department of Justice (DOJ) via petition for review, alleging that PROSECUTOR ACTED IN GRAVE ABUSE OF DISCRETION AMOUNTING TO AN
the City Prosecutor erred in ruling: EXCESS OF JURISDICTION WHEN THEY CONTINUED THE PROSECUTION OF
THE PETITIONER DESPITE LACK OF SUFFICIENT BASIS.19
1. That there is no evidence to show that respondent participated in the
misappropriation of the goods subject of the trust receipts; In his petition, petitioner incorporated a certification stating that "as far as this Petition is
concerned, no action or proceeding in the Supreme Court, the Court of Appeals or different
divisions thereof, or any tribunal or agency. It is finally certified that if the affiant should learn that
2. That the respondent is a mere surety of the trust receipts; and
a similar action or proceeding has been filed or is pending before the Supreme Court, the Court
of Appeals, or different divisions thereof, of any other tribunal or agency, it hereby undertakes to
3. That the liability of the respondent is only civil in nature. 14 notify this Honorable Court within five (5) days from such notice." 20

On July 13, 1999, the Secretary of Justice issued Resolution No. 25015 granting the petition and In its Comment on the petition, the Office of the Solicitor General alleged that -
reversing the assailed resolution of the City Prosecutor. According to the Justice Secretary, the
petitioner, as Senior Vice-President of PBMI, executed the 13 trust receipts and as such, was
A.
the one responsible for the offense. Thus, the execution of said receipts is enough to indict the
petitioner as the official responsible for violation of P.D. No. 115. The Justice Secretary also
declared that petitioner could not contend that P.D. No. 115 covers only goods ultimately THE HONORABLE SECRETARY OF JUSTICE CORRECTLY RULED THAT
destined for sale, as this issue had already been settled in Allied Banking Corporation v. PETITIONER ALFREDO CHING IS THE OFFICER RESPONSIBLE FOR THE
Ordoñez,16 where the Court ruled that P.D. No. 115 is "not limited to transactions in goods which OFFENSE CHARGED AND THAT THE ACTS OF PETITIONER FALL WITHIN THE
are to be sold (retailed), reshipped, stored or processed as a component of a product ultimately AMBIT OF VIOLATION OF P.D. [No.] 115 IN RELATION TO ARTICLE 315, PAR.
sold but covers failure to turn over the proceeds of the sale of entrusted goods, or to return said 1(B) OF THE REVISED PENAL CODE.
goods if unsold or not otherwise disposed of in accordance with the terms of the trust receipts."
B.
The Justice Secretary further stated that the respondent bound himself under the terms of the
trust receipts not only as a corporate official of PBMI but also as its surety; hence, he could be
THERE IS NO MERIT IN PETITIONER’S CONTENTION THAT EXCESSIVE DELAY
proceeded against in two (2) ways: first, as surety as determined by the Supreme Court in its
HAS MARRED THE CONDUCT OF THE PRELIMINARY INVESTIGATION OF THE
decision in Rizal Commercial Banking Corporation v. Court of Appeals; 17 and second, as the
CASE, JUSTIFYING ITS DISMISSAL.
corporate official responsible for the offense under P.D. No. 115, via criminal prosecution.
Moreover, P.D. No. 115 explicitly allows the prosecution of corporate officers "without prejudice
to the civil liabilities arising from the criminal offense." Thus, according to the Justice Secretary, C.
following Rizal Commercial Banking Corporation, the civil liability imposed is clearly separate
and distinct from the criminal liability of the accused under P.D. No. 115.
THE PRESENT SPECIAL CIVIL ACTION FOR CERTIORARI, PROHIBITION AND
MANDAMUS IS NOT THE PROPER MODE OF REVIEW FROM THE RESOLUTION
Conformably with the Resolution of the Secretary of Justice, the City Prosecutor filed 13 OF THE DEPARTMENT OF JUSTICE. THE PRESENT PETITION MUST
Informations against petitioner for violation of P.D. No. 115 before the RTC of Manila. The cases THEREFORE BE DISMISSED.21
were docketed as Criminal Cases No. 99-178596 to 99-178608 and consolidated for trial before
Branch 52 of said court. Petitioner filed a motion for reconsideration, which the Secretary of
Justice denied in a Resolution18 dated January 17, 2000. On April 22, 2004, the CA rendered judgment dismissing the petition for lack of merit, and on
procedural grounds. On the procedural issue, it ruled that (a) the certification of non-forum
shopping executed by petitioner and incorporated in the petition was defective for failure to
Petitioner then filed a petition for certiorari, prohibition and mandamus with the CA, assailing the comply with the first two of the three-fold undertakings prescribed in Rule 7, Section 5 of the
resolutions of the Secretary of Justice on the following grounds: Revised Rules of Civil Procedure; and (b) the petition for certiorari, prohibition and mandamus
was not the proper remedy of the petitioner.
1. THE RESPONDENTS ARE ACTING WITH AN UNEVEN HAND AND IN FACT,
ARE ACTING OPPRESSIVELY AGAINST ALFREDO CHING WHEN THEY On the merits of the petition, the CA ruled that the assailed resolutions of the Secretary of
ALLOWED HIS PROSECUTION DESPITE THE FACT THAT NO EVIDENCE HAD Justice were correctly issued for the following reasons: (a) petitioner, being the Senior Vice-
BEEN PRESENTED TO PROVE HIS PARTICIPATION IN THE ALLEGED President of PBMI and the signatory to the trust receipts, is criminally liable for violation of P.D.
TRANSACTIONS. No. 115; (b) the issue raised by the petitioner, on whether he violated P.D. No. 115 by his
actuations, had already been resolved and laid to rest in Allied Bank Corporation v. SEC. 3. Contents and filing of petition; effect of non-compliance with requirements. — The
Ordoñez;22 and (c) petitioner was estopped from raising the petition shall contain the full names and actual addresses of all the petitioners and respondents,
a concise statement of the matters involved, the factual background of the case and the grounds
relied upon for the relief prayed for.
City Prosecutor’s delay in the final disposition of the preliminary investigation because he failed
to do so in the DOJ.
xxx
Thus, petitioner filed the instant petition, alleging that:
The petitioner shall also submit together with the petition a sworn certification that he has not
theretofore commenced any other action involving the same issues in the Supreme Court, the
I
Court of Appeals or different divisions thereof, or any other tribunal or agency; if there is such
other action or proceeding, he must state the status of the same; and if he should thereafter
THE COURT OF APPEALS ERRED WHEN IT DISMISSED THE PETITION ON THE learn that a similar action or proceeding has been filed or is pending before the Supreme Court,
GROUND THAT THE CERTIFICATION OF NON-FORUM SHOPPING the Court of Appeals, or different divisions thereof, or any other tribunal or agency, he
INCORPORATED THEREIN WAS DEFECTIVE. undertakes to promptly inform the aforesaid courts and other tribunal or agency thereof within
five (5) days therefrom. xxx
II
Compliance with the certification against forum shopping is separate from and independent of
the avoidance of forum shopping itself. The requirement is mandatory. The failure of the
THE COURT OF APPEALS ERRED WHEN IT RULED THAT NO GRAVE ABUSE OF petitioner to comply with the foregoing requirement shall be sufficient ground for the dismissal of
DISCRETION AMOUNTING TO LACK OR EXCESS OF JURISDICTION WAS
the petition without prejudice, unless otherwise provided.26
COMMITTED BY THE SECRETARY OF JUSTICE IN COMING OUT WITH THE
ASSAILED RESOLUTIONS.23
Indubitably, the first paragraph of petitioner’s certification is incomplete and unintelligible.
Petitioner failed to certify that he "had not heretofore commenced any other action involving the
The Court will delve into and resolve the issues seriatim. same issues in the Supreme Court, the Court of Appeals or the different divisions thereof or any
other tribunal or agency" as required by paragraph 4, Section 3, Rule 46 of the Revised Rules of
The petitioner avers that the CA erred in dismissing his petition on a mere technicality. He claims Court.
that the rules of procedure should be used to promote, not frustrate, substantial justice. He
insists that the Rules of Court should be construed liberally especially when, as in this case, his We agree with petitioner’s contention that the certification is designed to promote and facilitate
substantial rights are adversely affected; hence, the deficiency in his certification of non-forum
the orderly administration of justice, and therefore, should not be interpreted with absolute
shopping should not result in the dismissal of his petition. literalness. In his works on the Revised Rules of Civil Procedure, former Supreme Court Justice
Florenz Regalado states that, with respect to the contents of the certification which the pleader
The Office of the Solicitor General (OSG) takes the opposite view, and asserts that indubitably, may prepare, the rule of substantial compliance may be availed of. 27However, there must be a
the certificate of non-forum shopping incorporated in the petition before the CA is defective special circumstance or compelling reason which makes the strict application of the requirement
because it failed to disclose essential facts about pending actions concerning similar issues and clearly unjustified. The instant petition has not alleged any such extraneous circumstance.
parties. It asserts that petitioner’s failure to comply with the Rules of Court is fatal to his petition. Moreover, as worded, the certification cannot even be regarded as substantial compliance with
The OSG cited Section 2, Rule 42, as well as the ruling of this Court in Melo v. Court of the procedural requirement. Thus, the CA was not informed whether, aside from the petition
Appeals.24 before it, petitioner had commenced any other action involving the same issues in other
tribunals.
We agree with the ruling of the CA that the certification of non-forum shopping petitioner
incorporated in his petition before the appellate court is defective. The certification reads: On the merits of the petition, the CA ruled that the petitioner failed to establish that the Secretary
of Justice committed grave abuse of discretion in finding probable cause against the petitioner
for violation of estafa under Article 315, paragraph 1(b) of the Revised Penal Code, in relation to
It is further certified that as far as this Petition is concerned, no action or proceeding in the P.D. No. 115. Thus, the appellate court ratiocinated:
Supreme Court, the Court of Appeals or different divisions thereof, or any tribunal or agency.

Be that as it may, even on the merits, the arguments advanced in support of the petition are not
It is finally certified that if the affiant should learn that a similar action or proceeding has been persuasive enough to justify the desired conclusion that respondent Secretary of Justice gravely
filed or is pending before the Supreme Court, the Court of Appeals, or different divisions thereof, abused its discretion in coming out with his assailed Resolutions. Petitioner posits that, except
of any other tribunal or agency, it hereby undertakes to notify this Honorable Court within five (5) for his being the Senior Vice-President of the PBMI, there is no iota of evidence that he was a
days from such notice.25 participes crimines in violating the trust receipts sued upon; and that his liability, if at all, is purely
civil because he signed the said trust receipts merely as a xxx surety and not as the entrustee.
Under Section 1, second paragraph of Rule 65 of the Revised Rules of Court, the petition should These assertions are, however, too dull that they cannot even just dent the findings of the
be accompanied by a sworn certification of non-forum shopping, as provided in the third respondent Secretary, viz:
paragraph of Section 3, Rule 46 of said Rules. The latter provision reads in part:
"x x x it is apropos to quote section 13 of PD 115 which states in part, viz: 35. Petitioner’s being a Senior Vice-President of the Philippine Blooming Mills does not
exculpate him from any liability. Petitioner’s responsibility as the corporate official of PBM who
received the goods in trust is premised on Section 13 of P.D. No. 115, which provides:
‘xxx If the violation or offense is committed by a corporation, partnership, association or other
judicial entities, the penalty provided for in this Decree shall be imposed upon the directors,
officers, employees or other officials or persons therein responsible for the offense, without Section 13. Penalty Clause. The failure of an entrustee to turn over the proceeds of the sale of
prejudice to the civil liabilities arising from the criminal offense.’ the goods, documents or instruments covered by a trust receipt to the extent of the amount
owing to the entruster or as appears in the trust receipt or to return said goods, documents or
instruments if they were not sold or disposed of in accordance with the terms of the trust receipt
"There is no dispute that it was the respondent, who as senior vice-president of PBM, executed
shall constitute the crime of estafa, punishable under the provisions of Article Three hundred and
the thirteen (13) trust receipts. As such, the law points to him as the official responsible for the
fifteen, paragraph one (b) of Act Numbered Three thousand eight hundred and fifteen, as
offense. Since a corporation cannot be proceeded against criminally because it cannot commit
amended, otherwise known as the Revised Penal Code. If the violation or offense is committed
crime in which personal violence or malicious intent is required, criminal action is limited to the
by a corporation, partnership, association or other juridical entities, the penalty provided for in
corporate agents guilty of an act amounting to a crime and never against the corporation itself
this Decree shall be imposed upon the directors, officers, employees or other officials or persons
(West Coast Life Ins. Co. vs. Hurd, 27 Phil. 401; Times, [I]nc. v. Reyes, 39 SCRA 303). Thus,
therein responsible for the offense, without prejudice to the civil liabilities arising from the
the execution by respondent of said receipts is enough to indict him as the official responsible for
criminal offense. (Emphasis supplied)
violation of PD 115.

36. Petitioner having participated in the negotiations for the trust receipts and having received
"Parenthetically, respondent is estopped to still contend that PD 115 covers only goods which
the goods for PBM, it was inevitable that the petitioner is the proper corporate officer to be
are ultimately destined for sale and not goods, like those imported by PBM, for use in
proceeded against by virtue of the PBM’s violation of P.D. No. 115.29
manufacture. This issue has already been settled in the Allied Banking Corporation case, supra,
where he was also a party, when the Supreme Court ruled that PD 115 is ‘not limited to
transactions in goods which are to be sold (retailed), reshipped, stored or processed as a The ruling of the CA is correct.
component or a product ultimately sold’ but ‘covers failure to turn over the proceeds of the sale
of entrusted goods, or to return said goods if unsold or disposed of in accordance with the terms
In Mendoza-Arce v. Office of the Ombudsman (Visayas),30 this Court held that the acts of a
of the trust receipts.’
quasi-judicial officer may be assailed by the aggrieved party via a petition for certiorari and
enjoined (a) when necessary to afford adequate protection to the constitutional rights of the
"In regard to the other assigned errors, we note that the respondent bound himself under the accused; (b) when necessary for the orderly administration of justice; (c) when the acts of the
terms of the trust receipts not only as a corporate official of PBM but also as its surety. It is officer are without or in excess of authority; (d) where the charges are manifestly false and
evident that these are two (2) capacities which do not exclude the other. Logically, he can be motivated by the lust for vengeance; and (e) when there is clearly no prima facie case against
proceeded against in two (2) ways: first, as surety as determined by the Supreme Court in its the accused.31 The Court also declared that, if the officer conducting a preliminary investigation
decision in RCBC vs. Court of Appeals, 178 SCRA 739; and, secondly, as the corporate official (in that case, the Office of the Ombudsman) acts without or in excess of his authority and
responsible for the offense under PD 115, the present case is an appropriate remedy under our resolves to file an Information despite the absence of probable cause, such act may be nullified
penal law. by a writ of certiorari.32

"Moreover, PD 115 explicitly allows the prosecution of corporate officers ‘without prejudice to the Indeed, under Section 4, Rule 112 of the 2000 Rules of Criminal Procedure,33 the Information
civil liabilities arising from the criminal offense’ thus, the civil liability imposed on respondent in shall be prepared by the Investigating Prosecutor against the respondent only if he or she finds
RCBC vs. Court of Appeals case is clearly separate and distinct from his criminal liability under probable cause to hold such respondent for trial. The Investigating Prosecutor acts without or in
PD 115.’"28 excess of his authority under the Rule if the Information is filed against the respondent despite
absence of evidence showing probable cause therefor. 34 If the Secretary of Justice reverses the
Resolution of the Investigating Prosecutor who found no probable cause to hold the respondent
Petitioner asserts that the appellate court’s ruling is erroneous because (a) the transaction
for trial, and orders such prosecutor to file the Information despite the absence of probable
between PBMI and respondent bank is not a trust receipt transaction; (b) he entered into the
cause, the Secretary of Justice acts contrary to law, without authority and/or in excess of
transaction and was sued in his capacity as PBMI Senior Vice-President; (c) he never received
authority. Such resolution may likewise be nullified in a petition for certiorari under Rule 65 of the
the goods as an entrustee for PBMI, hence, could not have committed any dishonesty or abused
Revised Rules of Civil Procedure.35
the confidence of respondent bank; and (d) PBMI acquired the goods and used the same in
operating its machineries and equipment and not for resale.
A preliminary investigation, designed to secure the respondent against hasty, malicious and
oppressive prosecution, is an inquiry to determine whether (a) a crime has been committed; and
The OSG, for its part, submits a contrary view, to wit:
(b) whether there is probable cause to believe that the accused is guilty thereof. It is a means of
discovering the person or persons who may be reasonably charged with a crime. Probable
34. Petitioner further claims that he is not a person responsible for the offense allegedly because cause need not be based on clear and convincing evidence of guilt, as the investigating officer
"[b]eing charged as the Senior Vice-President of Philippine Blooming Mills (PBM), petitioner acts upon probable cause of reasonable belief. Probable cause implies probability of guilt and
cannot be held criminally liable as the transactions sued upon were clearly entered into in his requires more than bare suspicion but less than evidence which would justify a conviction. A
capacity as an officer of the corporation" and that [h]e never received the goods as an entrustee finding of probable cause needs only to rest on evidence showing that more likely than not, a
for PBM as he never had or took possession of the goods nor did he commit dishonesty nor crime has been committed by the suspect.36
"abuse of confidence in transacting with RCBC." Such argument is bereft of merit.
However, while probable cause should be determined in a summary manner, there is a need to return the goods, documents or instruments in the event of non-sale or upon demand of the
examine the evidence with care to prevent material damage to a potential accused’s entruster; and (6) observe all other terms and conditions of the trust receipt not contrary to the
constitutional right to liberty and the guarantees of freedom and fair play37 and to protect the provisions of the decree.40
State from the burden of unnecessary expenses in prosecuting alleged offenses and holding
trials arising from false, fraudulent or groundless charges.38
The entruster shall be entitled to the proceeds from the sale of the goods, documents or
instruments released under a trust receipt to the entrustee to the extent of the amount owing to
In this case, petitioner failed to establish that the Secretary of Justice committed grave abuse of the entruster or as appears in the trust receipt, or to the return of the goods, documents or
discretion in issuing the assailed resolutions. Indeed, he acted in accord with law and the instruments in case of non-sale, and to the enforcement of all other rights conferred on him in
evidence. the trust receipt; provided, such are not contrary to the provisions of the document.41

Section 4 of P.D. No. 115 defines a trust receipt transaction, thus: In the case at bar, the transaction between petitioner and respondent bank falls under the trust
receipt transactions envisaged in P.D. No. 115. Respondent bank imported the goods and
entrusted the same to PBMI under the trust receipts signed by petitioner, as entrustee, with the
Section 4. What constitutes a trust receipt transaction. A trust receipt transaction, within the
bank as entruster. The agreement was as follows:
meaning of this Decree, is any transaction by and between a person referred to in this Decree as
the entruster, and another person referred to in this Decree as entrustee, whereby the entruster,
who owns or holds absolute title or security interests over certain specified goods, documents or And in consideration thereof, I/we hereby agree to hold said goods in trust for the said BANK as
instruments, releases the same to the possession of the entrustee upon the latter’s execution its property with liberty to sell the same within ____days from the date of the execution of this
and delivery to the entruster of a signed document called a "trust receipt" wherein the entrustee Trust Receipt and for the Bank’s account, but without authority to make any other disposition
binds himself to hold the designated goods, documents or instruments in trust for the entruster whatsoever of the said goods or any part thereof (or the proceeds) either by way of conditional
and to sell or otherwise dispose of the goods, documents or instruments with the obligation to sale, pledge or otherwise.
turn over to the entruster the proceeds thereof to the extent of the amount owing to the entruster
or as appears in the trust receipt or the goods, documents or instruments themselves if they are
I/we agree to keep the said goods insured to their full value against loss from fire, theft, pilferage
unsold or not otherwise disposed of, in accordance with the terms and conditions specified in the
or other casualties as directed by the BANK, the sum insured to be payable in case of loss to the
trust receipt, or for other purposes substantially equivalent to any of the following:
BANK, with the understanding that the BANK is, not to be chargeable with the storage premium
or insurance or any other expenses incurred on said goods.
1. In case of goods or documents, (a) to sell the goods or procure their sale; or (b) to
manufacture or process the goods with the purpose of ultimate sale; Provided, That, in
In case of sale, I/we further agree to turn over the proceeds thereof as soon as received to the
the case of goods delivered under trust receipt for the purpose of manufacturing or
BANK, to apply against the relative acceptances (as described above) and for the payment of
processing before its ultimate sale, the entruster shall retain its title over the goods
any other indebtedness of mine/ours to the BANK. In case of non-sale within the period specified
whether in its original or processed form until the entrustee has complied fully with his
herein, I/we agree to return the goods under this Trust Receipt to the BANK without any need of
obligation under the trust receipt; or (c) to load, unload, ship or otherwise deal with
demand.
them in a manner preliminary or necessary to their sale; or

I/we agree to keep the said goods, manufactured products or proceeds thereof, whether in the
2. In the case of instruments a) to sell or procure their sale or exchange; or b) to
form of money or bills, receivables, or accounts separate and capable of identification as
deliver them to a principal; or c) to effect the consummation of some transactions
property of the BANK.42
involving delivery to a depository or register; or d) to effect their presentation,
collection or renewal.
It must be stressed that P.D. No. 115 is a declaration by legislative authority that, as a matter of
public policy, the failure of person to turn over the proceeds of the sale of the goods covered by
The sale of goods, documents or instruments by a person in the business of selling goods,
a trust receipt or to return said goods, if not sold, is a public nuisance to be abated by the
documents or instruments for profit who, at the outset of the transaction, has, as against the
imposition of penal sanctions.43
buyer, general property rights in such goods, documents or instruments, or who sells the same
to the buyer on credit, retaining title or other interest as security for the payment of the purchase
price, does not constitute a trust receipt transaction and is outside the purview and coverage of The Court likewise rules that the issue of whether P.D. No. 115 encompasses transactions
this Decree. involving goods procured as a component of a product ultimately sold has been resolved in the
affirmative in Allied Banking Corporation v. Ordoñez.44 The law applies to goods used by the
entrustee in the operation of its machineries and equipment. The non-payment of the amount
An entrustee is one having or taking possession of goods, documents or instruments under a
covered by the trust receipts or the non-return of the goods covered by the receipts, if not sold or
trust receipt transaction, and any successor in interest of such person for the purpose of
otherwise not disposed of, violate the entrustee’s obligation to pay the amount or to return the
payment specified in the trust receipt agreement.39 The entrustee is obliged to: (1) hold the
goods to the entruster.
goods, documents or instruments in trust for the entruster and shall dispose of them strictly in
accordance with the terms and conditions of the trust receipt; (2) receive the proceeds in trust for
the entruster and turn over the same to the entruster to the extent of the amount owing to the In Colinares v. Court of Appeals,45 the Court declared that there are two possible situations in a
entruster or as appears on the trust receipt; (3) insure the goods for their total value against loss trust receipt transaction. The first is covered by the provision which refers to money received
from fire, theft, pilferage or other casualties; (4) keep said goods or proceeds thereof whether in under the obligation involving the duty to deliver it (entregarla) to the owner of the merchandise
money or whatever form, separate and capable of identification as property of the entruster; (5) sold. The second is covered by the provision which refers to merchandise received under the
obligation to return it (devolvera) to the owner.46 Thus, failure of the entrustee to turn over the 4th. By arresto mayor in its medium and maximum periods, if such amount does not exceed 200
proceeds of the sale of the goods covered by the trust receipts to the entruster or to return said pesos, provided that in the four cases mentioned, the fraud be committed by any of the following
goods if they were not disposed of in accordance with the terms of the trust receipt is a crime means; xxx
under P.D. No. 115, without need of proving intent to defraud. The law punishes dishonesty and
abuse of confidence in the handling of money or goods to the prejudice of the entruster,
Though the entrustee is a corporation, nevertheless, the law specifically makes the officers,
regardless of whether the latter is the owner or not. A mere failure to deliver the proceeds of the
employees or other officers or persons responsible for the offense, without prejudice to the civil
sale of the goods, if not sold, constitutes a criminal offense that causes prejudice, not only to
liabilities of such corporation and/or board of directors, officers, or other officials or employees
another, but more to the public interest.47
responsible for the offense. The rationale is that such officers or employees are vested with the
authority and responsibility to devise means necessary to ensure compliance with the law and, if
The Court rules that although petitioner signed the trust receipts merely as Senior Vice- they fail to do so, are held criminally accountable; thus, they have a responsible share in the
President of PBMI and had no physical possession of the goods, he cannot avoid prosecution violations of the law.48
for violation of P.D. No. 115.
If the crime is committed by a corporation or other juridical entity, the directors, officers,
The penalty clause of the law, Section 13 of P.D. No. 115 reads: employees or other officers thereof responsible for the offense shall be charged and penalized
for the crime, precisely because of the nature of the crime and the penalty therefor. A
corporation cannot be arrested and imprisoned; hence, cannot be penalized for a crime
Section 13. Penalty Clause. The failure of an entrustee to turn over the proceeds of the sale of
punishable by imprisonment.49 However, a corporation may be charged and prosecuted for a
the goods, documents or instruments covered by a trust receipt to the extent of the amount
crime if the imposable penalty is fine. Even if the statute prescribes both fine and imprisonment
owing to the entruster or as appears in the trust receipt or to return said goods, documents or
as penalty, a corporation may be prosecuted and, if found guilty, may be fined. 50
instruments if they were not sold or disposed of in accordance with the terms of the trust receipt
shall constitute the crime of estafa, punishable under the provisions of Article Three hundred and
fifteen, paragraph one (b) of Act Numbered Three thousand eight hundred and fifteen, as A crime is the doing of that which the penal code forbids to be done, or omitting to do what it
amended, otherwise known as the Revised Penal Code. If the violation or offense is committed commands. A necessary part of the definition of every crime is the designation of the author of
by a corporation, partnership, association or other juridical entities, the penalty provided for in the crime upon whom the penalty is to be inflicted. When a criminal statute designates an act of
this Decree shall be imposed upon the directors, officers, employees or other officials or persons a corporation or a crime and prescribes punishment therefor, it creates a criminal offense which,
therein responsible for the offense, without prejudice to the civil liabilities arising from the otherwise, would not exist and such can be committed only by the corporation. But when a penal
criminal offense. statute does not expressly apply to corporations, it does not create an offense for which a
corporation may be punished. On the other hand, if the State, by statute, defines a crime that
may be committed by a corporation but prescribes the penalty therefor to be suffered by the
The crime defined in P.D. No. 115 is malum prohibitum but is classified as estafa under
officers, directors, or employees of such corporation or other persons responsible for the
paragraph 1(b), Article 315 of the Revised Penal Code, or estafa with abuse of confidence. It
offense, only such individuals will suffer such penalty. 51Corporate officers or employees, through
may be committed by a corporation or other juridical entity or by natural persons. However, the
whose act, default or omission the corporation commits a crime, are themselves individually
penalty for the crime is imprisonment for the periods provided in said Article 315, which reads:
guilty of the crime.52

ARTICLE 315. Swindling (estafa). – Any person who shall defraud another by any of the means
The principle applies whether or not the crime requires the consciousness of wrongdoing. It
mentioned hereinbelow shall be punished by:
applies to those corporate agents who themselves commit the crime and to those, who, by virtue
of their managerial positions or other similar relation to the corporation, could be deemed
1st. The penalty of prision correccional in its maximum period to prision mayor in its responsible for its commission, if by virtue of their relationship to the corporation, they had the
minimum period, if the amount of the fraud is over 12,000 pesos but does not exceed power to prevent the act.53 Moreover, all parties active in promoting a crime, whether agents or
22,000 pesos; and if such amount exceeds the latter sum, the penalty provided in this not, are principals.54 Whether such officers or employees are benefited by their delictual acts is
paragraph shall be imposed in its maximum period, adding one year for each not a touchstone of their criminal liability. Benefit is not an operative fact.
additional 10,000 pesos; but the total penalty which may be imposed shall not exceed
twenty years. In such cases, and in connection with the accessory penalties which
In this case, petitioner signed the trust receipts in question. He cannot, thus, hide behind the
may be imposed and for the purpose of the other provisions of this Code, the penalty
cloak of the separate corporate personality of PBMI. In the words of Chief Justice Earl Warren, a
shall be termed prision mayor or reclusion temporal, as the case may be;
corporate officer cannot protect himself behind a corporation where he is the actual, present and
efficient actor.55
2nd. The penalty of prision correccional in its minimum and medium periods, if the
amount of the fraud is over 6,000 pesos but does not exceed 12,000 pesos;
IN LIGHT OF ALL THE FOREGOING, the petition is DENIED for lack of merit. Costs against the
petitioner.
3rd. The penalty of arresto mayor in its maximum period to prision correccional in its
minimum period, if such amount is over 200 pesos but does not exceed 6,000 pesos;
SO ORDERED.
and
G.R. No. 141994 January 17, 2005 school. However there would be no instructor for such subject. Students would be informed that
course would be moved to a later date because the school is still searching for the appropriate
instructor.
FILIPINAS BROADCASTING NETWORK, INC., petitioner,
vs.
AGO MEDICAL AND EDUCATIONAL CENTER-BICOL CHRISTIAN COLLEGE OF xxx
MEDICINE, (AMEC-BCCM) and ANGELITA F. AGO, respondents.
It is a public knowledge that the Ago Medical and Educational Center has survived and has been
DECISION surviving for the past few years since its inception because of funds support from foreign
foundations. If you will take a look at the AMEC premises you’ll find out that the names of the
buildings there are foreign soundings. There is a McDonald Hall. Why not Jose Rizal or
CARPIO, J.:
Bonifacio Hall? That is a very concrete and undeniable evidence that the support of foreign
foundations for AMEC is substantial, isn’t it? With the report which is the basis of the expose in
The Case DZRC today, it would be very easy for detractors and enemies of the Ago family to stop the flow
of support of foreign foundations who assist the medical school on the basis of the latter’s
purpose. But if the purpose of the institution (AMEC) is to deceive students at cross purpose with
This petition for review1 assails the 4 January 1999 Decision2 and 26 January 2000 Resolution its reason for being it is possible for these foreign foundations to lift or suspend their donations
of the Court of Appeals in CA-G.R. CV No. 40151. The Court of Appeals affirmed with
temporarily.8
modification the 14 December 1992 Decision3 of the Regional Trial Court of Legazpi City,
Branch 10, in Civil Case No. 8236. The Court of Appeals held Filipinas Broadcasting Network,
Inc. and its broadcasters Hermogenes Alegre and Carmelo Rima liable for libel and ordered xxx
them to solidarily pay Ago Medical and Educational Center-Bicol Christian College of Medicine
moral damages, attorney’s fees and costs of suit.
On the other hand, the administrators of AMEC-BCCM, AMEC Science High School and
the AMEC-Institute of Mass Communication in their effort to minimize expenses in terms
The Antecedents of salary are absorbing or continues to accept "rejects". For example how many teachers in
AMEC are former teachers of Aquinas University but were removed because of immorality?
Does it mean that the present administration of AMEC have the total definite moral foundation
"Exposé" is a radio documentary4 program hosted by Carmelo ‘Mel’ Rima ("Rima") and from catholic administrator of Aquinas University. I will prove to you my friends, that AMEC is a
Hermogenes ‘Jun’ Alegre ("Alegre").5 Exposé is aired every morning over DZRC-AM which is dumping ground, garbage, not merely of moral and physical misfits. Probably they only
owned by Filipinas Broadcasting Network, Inc. ("FBNI"). "Exposé" is heard over Legazpi City, qualify in terms of intellect. The Dean of Student Affairs of AMEC is Justita Lola, as the family
the Albay municipalities and other Bicol areas.6
name implies. She is too old to work, being an old woman. Is the AMEC administration exploiting
the very [e]nterprising or compromising and undemanding Lola? Could it be that AMEC is just
In the morning of 14 and 15 December 1989, Rima and Alegre exposed various alleged patiently making use of Dean Justita Lola were if she is very old. As in atmospheric situation –
complaints from students, teachers and parents against Ago Medical and Educational Center- zero visibility – the plane cannot land, meaning she is very old, low pay follows. By the way,
Bicol Christian College of Medicine ("AMEC") and its administrators. Claiming that the Dean Justita Lola is also the chairman of the committee on scholarship in AMEC. She had
broadcasts were defamatory, AMEC and Angelita Ago ("Ago"), as Dean of AMEC’s College of retired from Bicol University a long time ago but AMEC has patiently made use of her.
Medicine, filed a complaint for damages7 against FBNI, Rima and Alegre on 27 February 1990.
Quoted are portions of the allegedly libelous broadcasts: xxx

JUN ALEGRE: MEL RIMA:

Let us begin with the less burdensome: if you have children taking medical course at AMEC-
xxx My friends based on the expose, AMEC is a dumping ground for moral and physically misfit
BCCM, advise them to pass all subjects because if they fail in any subject they will repeat
people. What does this mean? Immoral and physically misfits as teachers.
their year level, taking up all subjects including those they have passed already. Several
students had approached me stating that they had consulted with the DECS which told them that
there is no such regulation. If [there] is no such regulation why is AMEC doing the same? May I say I’m sorry to Dean Justita Lola. But this is the truth. The truth is this, that your are no
longer fit to teach. You are too old. As an aviation, your case is zero visibility. Don’t insist.
xxx
xxx Why did AMEC still absorb her as a teacher, a dean, and chairman of the scholarship
committee at that. The reason is practical cost saving in salaries, because an old person is not
Second: Earlier AMEC students in Physical Therapy had complained that the course is not
fastidious, so long as she has money to buy the ingredient of beetle juice. The elderly can get by
recognized by DECS. xxx
– that’s why she (Lola) was taken in as Dean.

Third: Students are required to take and pay for the subject even if the subject does not
xxx
have an instructor - such greed for money on the part of AMEC’s administration. Take the
subject Anatomy: students would pay for the subject upon enrolment because it is offered by the
xxx On our end our task is to attend to the interests of students. It is likely that the students Both parties, namely, FBNI, Rima and Alegre, on one hand, and AMEC and Ago, on the other,
would be influenced by evil. When they become members of society outside of campus will appealed the decision to the Court of Appeals. The Court of Appeals affirmed the trial court’s
be liabilities rather than assets. What do you expect from a doctor who while studying at judgment with modification. The appellate court made Rima solidarily liable with FBNI and
AMEC is so much burdened with unreasonable imposition? What do you expect from a student Alegre. The appellate court denied Ago’s claim for damages and attorney’s fees because the
who aside from peculiar problems – because not all students are rich – in their struggle to broadcasts were directed against AMEC, and not against her. The dispositive portion of the
improve their social status are even more burdened with false regulations. xxx9 (Emphasis Court of Appeals’ decision reads:
supplied)
WHEREFORE, the decision appealed from is hereby AFFIRMED, subject to the modification
The complaint further alleged that AMEC is a reputable learning institution. With the supposed that broadcaster Mel Rima is SOLIDARILY ADJUDGED liable with FBN[I] and Hermo[g]enes
exposés, FBNI, Rima and Alegre "transmitted malicious imputations, and as such, destroyed Alegre.
plaintiffs’ (AMEC and Ago) reputation." AMEC and Ago included FBNI as defendant for allegedly
failing to exercise due diligence in the selection and supervision of its employees, particularly
SO ORDERED.14
Rima and Alegre.

FBNI, Rima and Alegre filed a motion for reconsideration which the Court of Appeals denied in
On 18 June 1990, FBNI, Rima and Alegre, through Atty. Rozil Lozares, filed an
its 26 January 2000 Resolution.
Answer10 alleging that the broadcasts against AMEC were fair and true. FBNI, Rima and Alegre
claimed that they were plainly impelled by a sense of public duty to report the "goings-on in
AMEC, [which is] an institution imbued with public interest." Hence, FBNI filed this petition.15

Thereafter, trial ensued. During the presentation of the evidence for the defense, Atty. Edmundo The Ruling of the Court of Appeals
Cea, collaborating counsel of Atty. Lozares, filed a Motion to Dismiss11 on FBNI’s behalf. The
trial court denied the motion to dismiss. Consequently, FBNI filed a separate Answer claiming
The Court of Appeals upheld the trial court’s ruling that the questioned broadcasts are
that it exercised due diligence in the selection and supervision of Rima and Alegre. FBNI
claimed that before hiring a broadcaster, the broadcaster should (1) file an application; (2) be libelous per se and that FBNI, Rima and Alegre failed to overcome the legal presumption of
interviewed; and (3) undergo an apprenticeship and training program after passing the interview. malice. The Court of Appeals found Rima and Alegre’s claim that they were actuated by their
moral and social duty to inform the public of the students’ gripes as insufficient to justify the
FBNI likewise claimed that it always reminds its broadcasters to "observe truth, fairness and
objectivity in their broadcasts and to refrain from using libelous and indecent language." utterance of the defamatory remarks.
Moreover, FBNI requires all broadcasters to pass the Kapisanan ng mga Brodkaster sa
Pilipinas ("KBP") accreditation test and to secure a KBP permit. Finding no factual basis for the imputations against AMEC’s administrators, the Court of Appeals
ruled that the broadcasts were made "with reckless disregard as to whether they were true or
On 14 December 1992, the trial court rendered a Decision12 finding FBNI and Alegre liable for false." The appellate court pointed out that FBNI, Rima and Alegre failed to present in court any
libel except Rima. The trial court held that the broadcasts are libelous per se. The trial court of the students who allegedly complained against AMEC. Rima and Alegre merely gave a single
rejected the broadcasters’ claim that their utterances were the result of straight reporting name when asked to identify the students. According to the Court of Appeals, these
because it had no factual basis. The broadcasters did not even verify their reports before airing circumstances cast doubt on the veracity of the broadcasters’ claim that they were "impelled by
them to show good faith. In holding FBNI liable for libel, the trial court found that FBNI failed to their moral and social duty to inform the public about the students’ gripes."
exercise diligence in the selection and supervision of its employees.
The Court of Appeals found Rima also liable for libel since he remarked that "(1) AMEC-BCCM
In absolving Rima from the charge, the trial court ruled that Rima’s only participation was when is a dumping ground for morally and physically misfit teachers; (2) AMEC obtained the services
he agreed with Alegre’s exposé. The trial court found Rima’s statement within the "bounds of of Dean Justita Lola to minimize expenses on its employees’ salaries; and (3) AMEC burdened
the students with unreasonable imposition and false regulations."16
freedom of speech, expression, and of the press." The dispositive portion of the decision reads:

WHEREFORE, premises considered, this court finds for the plaintiff. Considering the degree The Court of Appeals held that FBNI failed to exercise due diligence in the selection and
of damages caused by the controversial utterances, which are not found by this court to supervision of its employees for allowing Rima and Alegre to make the radio broadcasts without
be really very serious and damaging, and there being no showing that indeed the the proper KBP accreditation. The Court of Appeals denied Ago’s claim for damages and
enrollment of plaintiff school dropped, defendants Hermogenes "Jun" Alegre, Jr. and attorney’s fees because the libelous remarks were directed against AMEC, and not against her.
The Court of Appeals adjudged FBNI, Rima and Alegre solidarily liable to pay AMEC moral
Filipinas Broadcasting Network (owner of the radio station DZRC), are hereby jointly and
severally ordered to pay plaintiff Ago Medical and Educational Center-Bicol Christian College of damages, attorney’s fees and costs of suit.1awphi1.nét
Medicine (AMEC-BCCM) the amount of ₱300,000.00 moral damages, plus ₱30,000.00
reimbursement of attorney’s fees, and to pay the costs of suit. Issues

SO ORDERED. 13 (Emphasis supplied) FBNI raises the following issues for resolution:

I. WHETHER THE BROADCASTS ARE LIBELOUS;


II. WHETHER AMEC IS ENTITLED TO MORAL DAMAGES; students’ alleged gripes. Neither did they inquire about nor confirm the purported irregularities in
AMEC from the Department of Education, Culture and Sports. Alegre testified that he merely
went to AMEC to verify his report from an alleged AMEC official who refused to disclose any
III. WHETHER THE AWARD OF ATTORNEY’S FEES IS PROPER; and
information. Alegre simply relied on the words of the students "because they were many and not
because there is proof that what they are saying is true."28 This plainly shows Rima and Alegre’s
IV. WHETHER FBNI IS SOLIDARILY LIABLE WITH RIMA AND ALEGRE FOR reckless disregard of whether their report was true or not.
PAYMENT OF MORAL DAMAGES, ATTORNEY’S FEES AND COSTS OF SUIT.
Contrary to FBNI’s claim, the broadcasts were not "the result of straight reporting." Significantly,
The Court’s Ruling some courts in the United States apply the privilege of "neutral reportage" in libel cases involving
matters of public interest or public figures. Under this privilege, a republisher who accurately and
disinterestedly reports certain defamatory statements made against public figures is shielded
We deny the petition. from liability, regardless of the republisher’s subjective awareness of the truth or falsity of the
accusation.29 Rima and Alegre cannot invoke the privilege of neutral reportage because
This is a civil action for damages as a result of the allegedly defamatory remarks of Rima and unfounded comments abound in the broadcasts. Moreover, there is no existing controversy
Alegre against AMEC.17 While AMEC did not point out clearly the legal basis for its complaint, a involving AMEC when the broadcasts were made. The privilege of neutral reportage applies
reading of the complaint reveals that AMEC’s cause of action is based on Articles 30 and 33 of where the defamed person is a public figure who is involved in an existing controversy, and a
the Civil Code. Article 3018 authorizes a separate civil action to recover civil liability arising from a party to that controversy makes the defamatory statement.30
criminal offense. On the other hand, Article 3319 particularly provides that the injured party may
bring a separate civil action for damages in cases of defamation, fraud, and physical injuries. However, FBNI argues vigorously that malice in law does not apply to this case. Citing Borjal v.
AMEC also invokes Article 1920 of the Civil Code to justify its claim for damages. AMEC cites
Court of Appeals,31 FBNI contends that the broadcasts "fall within the coverage of qualifiedly
Articles 217621 and 218022 of the Civil Code to hold FBNI solidarily liable with Rima and Alegre. privileged communications" for being commentaries on matters of public interest. Such being the
case, AMEC should prove malice in fact or actual malice. Since AMEC allegedly failed to prove
I. actual malice, there is no libel.

Whether the broadcasts are libelous FBNI’s reliance on Borjal is misplaced. In Borjal, the Court elucidated on the "doctrine of fair
comment," thus:
A libel23 is a public and malicious imputation of a crime, or of a vice or defect, real or imaginary,
or any act or omission, condition, status, or circumstance tending to cause the dishonor, [F]air commentaries on matters of public interest are privileged and constitute a valid defense in
discredit, or contempt of a natural or juridical person, or to blacken the memory of one who is an action for libel or slander. The doctrine of fair comment means that while in general every
dead.24 discreditable imputation publicly made is deemed false, because every man is presumed
innocent until his guilt is judicially proved, and every false imputation is deemed malicious,
nevertheless, when the discreditable imputation is directed against a public person in his public
There is no question that the broadcasts were made public and imputed to AMEC defects or capacity, it is not necessarily actionable. In order that such discreditable imputation to a
circumstances tending to cause it dishonor, discredit and contempt. Rima and Alegre’s remarks public official may be actionable, it must either be a false allegation of fact or a comment
such as "greed for money on the part of AMEC’s administrators"; "AMEC is a dumping ground, based on a false supposition. If the comment is an expression of opinion, based on
garbage of xxx moral and physical misfits"; and AMEC students who graduate "will be liabilities established facts, then it is immaterial that the opinion happens to be mistaken, as long as it
rather than assets" of the society are libelous per se. Taken as a whole, the broadcasts suggest might reasonably be inferred from the facts.32 (Emphasis supplied)
that AMEC is a money-making institution where physically and morally unfit teachers abound.

True, AMEC is a private learning institution whose business of educating students is "genuinely
However, FBNI contends that the broadcasts are not malicious. FBNI claims that Rima and imbued with public interest." The welfare of the youth in general and AMEC’s students in
Alegre were plainly impelled by their civic duty to air the students’ gripes. FBNI alleges that there particular is a matter which the public has the right to know. Thus, similar to the newspaper
is no evidence that ill will or spite motivated Rima and Alegre in making the broadcasts. FBNI articles in Borjal, the subject broadcasts dealt with matters of public interest. However, unlike
further points out that Rima and Alegre exerted efforts to obtain AMEC’s side and gave Ago the in Borjal, the questioned broadcasts are not based on established facts. The record supports
opportunity to defend AMEC and its administrators. FBNI concludes that since there is no the following findings of the trial court:
malice, there is no libel.

xxx Although defendants claim that they were motivated by consistent reports of students and
FBNI’s contentions are untenable. parents against plaintiff, yet, defendants have not presented in court, nor even gave name of a
single student who made the complaint to them, much less present written complaint or petition
Every defamatory imputation is presumed malicious.25 Rima and Alegre failed to show to that effect. To accept this defense of defendants is too dangerous because it could easily give
adequately their good intention and justifiable motive in airing the supposed gripes of the license to the media to malign people and establishments based on flimsy excuses that there
students. As hosts of a documentary or public affairs program, Rima and Alegre should have were reports to them although they could not satisfactorily establish it. Such laxity would
presented the public issues "free from inaccurate and misleading information."26 Hearing the encourage careless and irresponsible broadcasting which is inimical to public interests.
students’ alleged complaints a month before the exposé,27 they had sufficient time to verify their
sources and information. However, Rima and Alegre hardly made a thorough investigation of the
Secondly, there is reason to believe that defendant radio broadcasters, contrary to the mandates 4. Public affairs program shall present public issues free from personal bias,
of their duties, did not verify and analyze the truth of the reports before they aired it, in order to prejudice and inaccurate and misleading information. x x x Furthermore, the station
prove that they are in good faith. shall strive to present balanced discussion of issues. x x x.

Alegre contended that plaintiff school had no permit and is not accredited to offer Physical xxx
Therapy courses. Yet, plaintiff produced a certificate coming from DECS that as of Sept. 22,
1987 or more than 2 years before the controversial broadcast, accreditation to offer Physical
7. The station shall be responsible at all times in the supervision of public affairs,
Therapy course had already been given the plaintiff, which certificate is signed by no less than
public issues and commentary programs so that they conform to the provisions and
the Secretary of Education and Culture herself, Lourdes R. Quisumbing (Exh. C-rebuttal).
standards of this code.
Defendants could have easily known this were they careful enough to verify. And yet,
defendants were very categorical and sounded too positive when they made the erroneous
report that plaintiff had no permit to offer Physical Therapy courses which they were offering. 8. It shall be the responsibility of the newscaster, commentator, host and announcer to
protect public interest, general welfare and good order in the presentation of public
affairs and public issues.36 (Emphasis supplied)
The allegation that plaintiff was getting tremendous aids from foreign foundations like Mcdonald
Foundation prove not to be true also. The truth is there is no Mcdonald Foundation existing.
Although a big building of plaintiff school was given the name Mcdonald building, that was only The broadcasts fail to meet the standards prescribed in the Radio Code, which lays down the
in order to honor the first missionary in Bicol of plaintiffs’ religion, as explained by Dr. Lita Ago. code of ethical conduct governing practitioners in the radio broadcast industry. The Radio Code
Contrary to the claim of defendants over the air, not a single centavo appears to be received by is a voluntary code of conduct imposed by the radio broadcast industry on its own members. The
plaintiff school from the aforementioned McDonald Foundation which does not exist. Radio Code is a public warranty by the radio broadcast industry that radio broadcast
practitioners are subject to a code by which their conduct are measured for lapses, liability and
sanctions.
Defendants did not even also bother to prove their claim, though denied by Dra. Ago, that when
medical students fail in one subject, they are made to repeat all the other subject[s], even those
they have already passed, nor their claim that the school charges laboratory fees even if there The public has a right to expect and demand that radio broadcast practitioners live up to the
are no laboratories in the school. No evidence was presented to prove the bases for these code of conduct of their profession, just like other professionals. A professional code of conduct
claims, at least in order to give semblance of good faith. provides the standards for determining whether a person has acted justly, honestly and with
good faith in the exercise of his rights and performance of his duties as required by Article
1937 of the Civil Code. A professional code of conduct also provides the standards for
As for the allegation that plaintiff is the dumping ground for misfits, and immoral teachers,
determining whether a person who willfully causes loss or injury to another has acted in a
defendant[s] singled out Dean Justita Lola who is said to be so old, with zero visibility already.
manner contrary to morals or good customs under Article 2138 of the Civil Code.
Dean Lola testified in court last Jan. 21, 1991, and was found to be 75 years old. xxx Even older
people prove to be effective teachers like Supreme Court Justices who are still very much in
demand as law professors in their late years. Counsel for defendants is past 75 but is found by II.
this court to be still very sharp and effective.l^vvphi1.net So is plaintiffs’ counsel.
Whether AMEC is entitled to moral damages
Dr. Lola was observed by this court not to be physically decrepit yet, nor mentally infirmed, but is
still alert and docile.
FBNI contends that AMEC is not entitled to moral damages because it is a corporation. 39

The contention that plaintiffs’ graduates become liabilities rather than assets of our society is a
mere conclusion. Being from the place himself, this court is aware that majority of the medical A juridical person is generally not entitled to moral damages because, unlike a natural person, it
graduates of plaintiffs pass the board examination easily and become prosperous and cannot experience physical suffering or such sentiments as wounded feelings, serious anxiety,
mental anguish or moral shock.40 The Court of Appeals cites Mambulao Lumber Co. v. PNB, et
responsible professionals.33
al.41 to justify the award of moral damages. However, the Court’s statement in Mambulao that "a
corporation may have a good reputation which, if besmirched, may also be a ground for the
Had the comments been an expression of opinion based on established facts, it is immaterial award of moral damages" is an obiter dictum.42
that the opinion happens to be mistaken, as long as it might reasonably be inferred from the
facts.34 However, the comments of Rima and Alegre were not backed up by facts. Therefore, the
broadcasts are not privileged and remain libelous per se. Nevertheless, AMEC’s claim for moral damages falls under item 7 of Article 221943 of the Civil
Code. This provision expressly authorizes the recovery of moral damages in cases of libel,
slander or any other form of defamation. Article 2219(7) does not qualify whether the plaintiff is a
The broadcasts also violate the Radio Code35 of the Kapisanan ng mga Brodkaster sa Pilipinas, natural or juridical person. Therefore, a juridical person such as a corporation can validly
Ink. ("Radio Code"). Item I(B) of the Radio Code provides: complain for libel or any other form of defamation and claim for moral damages.44

B. PUBLIC AFFAIRS, PUBLIC ISSUES AND COMMENTARIES Moreover, where the broadcast is libelous per se, the law implies damages.45 In such a case,
evidence of an honest mistake or the want of character or reputation of the party libeled goes
only in mitigation of damages.46 Neither in such a case is the plaintiff required to introduce
1. x x x
evidence of actual damages as a condition precedent to the recovery of some damages. 47 In this attributable to the KBP Manila Office. FBNI claims that membership in the KBP is merely
case, the broadcasts are libelous per se. Thus, AMEC is entitled to moral damages. voluntary and not required by any law or government regulation.

However, we find the award of ₱300,000 moral damages unreasonable. The record shows that FBNI’s arguments do not persuade us.
even though the broadcasts were libelous per se, AMEC has not suffered any substantial or
material damage to its reputation. Therefore, we reduce the award of moral damages from
The basis of the present action is a tort. Joint tort feasors are jointly and severally liable for the
₱300,000 to ₱150,000.
tort which they commit.52 Joint tort feasors are all the persons who command, instigate, promote,
encourage, advise, countenance, cooperate in, aid or abet the commission of a tort, or who
III. approve of it after it is done, if done for their benefit.53Thus, AMEC correctly anchored its cause
of action against FBNI on Articles 2176 and 2180 of the Civil Code.1a\^/phi1.net
Whether the award of attorney’s fees is proper
As operator of DZRC-AM and employer of Rima and Alegre, FBNI is solidarily liable to pay for
damages arising from the libelous broadcasts. As stated by the Court of Appeals, "recovery for
FBNI contends that since AMEC is not entitled to moral damages, there is no basis for the award
defamatory statements published by radio or television may be had from the owner of the
of attorney’s fees. FBNI adds that the instant case does not fall under the enumeration in Article
station, a licensee, the operator of the station, or a person who procures, or participates in,
220848 of the Civil Code.
the making of the defamatory statements."54 An employer and employee are solidarily liable for a
defamatory statement by the employee within the course and scope of his or her employment, at
The award of attorney’s fees is not proper because AMEC failed to justify satisfactorily its claim least when the employer authorizes or ratifies the defamation. 55 In this case, Rima and Alegre
for attorney’s fees. AMEC did not adduce evidence to warrant the award of attorney’s fees. were clearly performing their official duties as hosts of FBNI’s radio program Exposé when they
Moreover, both the trial and appellate courts failed to explicitly state in their respective decisions aired the broadcasts. FBNI neither alleged nor proved that Rima and Alegre went beyond the
the rationale for the award of attorney’s fees.49 In Inter-Asia Investment Industries, Inc. v. scope of their work at that time. There was likewise no showing that FBNI did not authorize and
Court of Appeals ,50 we held that: ratify the defamatory broadcasts.

[I]t is an accepted doctrine that the award thereof as an item of damages is the exception rather Moreover, there is insufficient evidence on record that FBNI exercised due diligence in
than the rule, and counsel’s fees are not to be awarded every time a party wins a suit. The the selection andsupervision of its employees, particularly Rima and Alegre. FBNI merely
power of the court to award attorney’s fees under Article 2208 of the Civil Code demands showed that it exercised diligence in the selection of its broadcasters without introducing any
factual, legal and equitable justification, without which the award is a conclusion without evidence to prove that it observed the same diligence in the supervision of Rima and Alegre.
a premise, its basis being improperly left to speculation and conjecture. In all events, the FBNI did not show how it exercised diligence in supervising its broadcasters. FBNI’s alleged
court must explicitly state in the text of the decision, and not only in the decretal portion thereof, constant reminder to its broadcasters to "observe truth, fairness and objectivity and to refrain
the legal reason for the award of attorney’s fees.51 (Emphasis supplied) from using libelous and indecent language" is not enough to prove due diligence in the
supervision of its broadcasters. Adequate training of the broadcasters on the industry’s code of
conduct, sufficient information on libel laws, and continuous evaluation of the broadcasters’
While it mentioned about the award of attorney’s fees by stating that it "lies within the discretion performance are but a few of the many ways of showing diligence in the supervision of
of the court and depends upon the circumstances of each case," the Court of Appeals failed to broadcasters.
point out any circumstance to justify the award.

FBNI claims that it "has taken all the precaution in the selection of Rima and Alegre as
IV.
broadcasters, bearing in mind their qualifications." However, no clear and convincing evidence
shows that Rima and Alegre underwent FBNI’s "regimented process" of application.
Whether FBNI is solidarily liable with Rima and Alegre for moral damages, attorney’s fees and Furthermore, FBNI admits that Rima and Alegre had deficiencies in their KBP
costs of suit accreditation,56 which is one of FBNI’s requirements before it hires a broadcaster. Significantly,
membership in the KBP, while voluntary, indicates the broadcaster’s strong commitment to
observe the broadcast industry’s rules and regulations. Clearly, these circumstances show
FBNI contends that it is not solidarily liable with Rima and Alegre for the payment of damages FBNI’s lack of diligence in selecting andsupervising Rima and Alegre. Hence, FBNI is solidarily
and attorney’s fees because it exercised due diligence in the selection and supervision of its liable to pay damages together with Rima and Alegre.
employees, particularly Rima and Alegre. FBNI maintains that its broadcasters, including Rima
and Alegre, undergo a "very regimented process" before they are allowed to go on air. "Those
who apply for broadcaster are subjected to interviews, examinations and an apprenticeship WHEREFORE, we DENY the instant petition. We AFFIRM the Decision of 4 January 1999 and
program." Resolution of 26 January 2000 of the Court of Appeals in CA-G.R. CV No. 40151 with the
MODIFICATION that the award of moral damages is reduced from ₱300,000 to ₱150,000 and
the award of attorney’s fees is deleted. Costs against petitioner.
FBNI further argues that Alegre’s age and lack of training are irrelevant to his competence as a
broadcaster. FBNI points out that the "minor deficiencies in the KBP accreditation of Rima and
Alegre do not in any way prove that FBNI did not exercise the diligence of a good father of a SO ORDERED.
family in selecting and supervising them." Rima’s accreditation lapsed due to his non-payment of
the KBP annual fees while Alegre’s accreditation card was delayed allegedly for reasons
G.R. No. 131723 December 13, 2007 Vice-President had met with petitioner's representative. Ultra further intimated that assuming
that there was tampering of the meters, petitioner's assessment was excessive. 10 For failure of
TEC to pay the differential billing, petitioner disconnected the electricity supply to the DCIM
MANILA ELECTRIC COMPANY, petitioner,
building on April 29, 1988.
vs.
T.E.A.M. ELECTRONICS CORPORATION, TECHNOLOGY ELECTRONICS ASSEMBLY and
MANAGEMENT PACIFIC CORPORATION; and ULTRA ELECTRONICS INSTRUMENTS, TEC demanded from petitioner the reconnection of electrical service, claiming that it had nothing
INC., respondents. to do with the alleged tampering but the latter refused to heed the demand. Hence, TEC filed a
complaint on May 27, 1988 before the Energy Regulatory Board (ERB) praying that electric
power be restored to the DCIM building.11 The ERB immediately ordered the reconnection of the
DECISION
service but petitioner complied with it only on October 12, 1988 after TEC paid P1,000,000.00,
under protest. The complaint before the ERB was later withdrawn as the parties deemed it best
NACHURA, J.: to have the issues threshed out in the regular courts. Prior to the reconnection, or on June 7,
1988, petitioner conducted a scheduled inspection of the questioned meters and found them to
have been tampered anew.12
This is a petition for review on certiorari under Rule 45 of the Rules of Court seeking the reversal
of the Decision1 of the Court of Appeals (CA) dated June 18, 1997 and its Resolution2 dated
December 3, 1997 in CA-G.R. CV No. 40282 denying the appeal filed by petitioner Manila Meanwhile, on April 25, 1988, petitioner conducted another inspection, this time, in TEC's NS
Electric Company. Building. The inspection allegedly revealed that the electric meters were not registering the
correct power consumption. Petitioner, thus, sent a letter dated June 18, 1988 demanding
payment of P280,813.72 representing the differential billing.13TEC denied petitioner's allegations
The facts of the case, as culled from the records, are as follows:
and claim in a letter dated June 29, 1988.14 Petitioner, thus, sent TEC another letter demanding
payment of the aforesaid amount, with a warning that the electric service would be disconnected
Respondent T.E.A.M. Electronics Corporation (TEC) was formerly known as NS Electronics in case of continued refusal to pay the differential billing.15 To avert the impending disconnection
(Philippines), Inc. before 1982 and National Semi-Conductors (Phils.) before 1988. TEC is of electrical service, TEC paid the above amount, under protest. 16
wholly owned by respondent Technology Electronics Assembly and Management Pacific
Corporation (TPC). On the other hand, petitioner Manila Electric Company (Meralco) is a utility On January 13, 1989, TEC and TPC filed a complaint for damages against petitioner and
company supplying electricity in the Metro Manila area.
Ultra17 before the Regional Trial Court (RTC) of Pasig. The case was raffled to Branch 162 and
was docketed as Civil Case No. 56851.18 Upon the filing of the parties' answer to the complaint,
Petitioner and NS Electronics (Philippines), Inc., the predecessor-in-interest of respondent TEC, pre-trial was scheduled.
were parties to two separate contracts denominated as Agreements for the Sale of Electric
Energy under the following account numbers: 09341-1322-163 and 09341-1812-13.4 Under the At the pre-trial, the parties agreed to limit the issues, as follows:
aforesaid agreements, petitioner undertook to supply TEC's building known as Dyna Craft
International Manila (DCIM) located at Electronics Avenue, Food Terminal Complex, Taguig,
Metro Manila, with electric power. Another contract was entered into for the supply of electric 1. Whether or not the defendant Meralco is liable for the plaintiffs' disconnection of
power to TEC's NS Building under Account No. 19389-0900-10. electric service at DCIM Building.

In September 1986, TEC, under its former name National Semi-Conductors (Phils.) entered into 2. Whether or not the plaintiff is liable for (sic) the defendant for the differential billings
a Contract of Lease5 with respondent Ultra Electronics Industries, Inc. (Ultra) for the use of the in the amount of P7,040,401.01.
former's DCIM building for a period of five years or until September 1991. Ultra was, however,
ejected from the premises on February 12, 1988 by virtue of a court order, for repeated violation
3. Whether or not the plaintiff is liable to defendant for exemplary damages. 19
of the terms and conditions of the lease contract.

For failure of the parties to reach an amicable settlement, trial on the merits ensued. On June
On September 28, 1987, a team of petitioner's inspectors conducted a surprise inspection of the
17, 1992, the trial court rendered a Decision in favor of respondents TEC and TPC, and against
electric meters installed at the DCIM building, witnessed by Ultra's 6 representative, Mr. Willie
respondent Ultra and petitioner. The pertinent portion of the decision reads:
Abangan. The two meters covered by account numbers 09341-1322-16 and 09341-1812-13,
were found to be allegedly tampered with and did not register the actual power consumption in
the building. The results of the inspection were reflected in the Service Inspection WHEREFORE, judgment is hereby rendered in this case in favor of the plaintiffs and
Reports7 prepared by the team. against the defendants as follows:

In a letter dated November 25, 1987, petitioner informed TEC of the results of the inspection and (1) Ordering both defendants Meralco and ULTRA Electronics Instruments,
demanded from the latter the payment of P7,040,401.01 representing its unregistered Inc. to jointly and severally reimburse plaintiff TEC actual damages in the
consumption from February 10, 1986 until September 28, 1987, as a result of the alleged amount of ONE MILLION PESOS with legal rate of interest from the date of
tampering of the meters.8 TEC received the letters on January 7, 1988. Since Ultra was in the filing of this case on January 19, 1989 until the said amount shall have
possession of the subject building during the covered period, TEC's Managing Director, Mr. been fully paid;
Bobby Tan, referred the demand letter to Ultra9 which, in turn, informed TEC that its Executive
(2) Ordering defendant Meralco to pay to plaintiff TEC the amount TEC and TPC of the results of the inspection; and in disconnecting the electric power without
of P280,813.72 as actual damages with legal rate of interest also from prior notice.
January 19, 1989;
Petitioner now comes before this Court in this petition for review on certiorari contending that:
(3) Ordering defendant Meralco to pay to plaintiff TPC the amount
of P150,000.00 as actual damages with interest at legal rate from January
The Court of Appeals committed grievous errors and decided matters of substance
19, 1989;
contrary to law and the rulings of this Honorable Court:

(4) Condemning defendant Meralco to pay both plaintiffs moral damages in


1. In finding that the issue in the case is whether there was deliberate tampering of the
the amount pf P500,000.00;
metering installations at the building owned by TEC.

(5) Condemning defendant Meralco to pay both plaintiffs corrective and/or


2. In not finding that the issue is: whether or not, based on the tampered meters,
exemplary damages in the amount of P200,000.00;
whether or not petitioner is entitled to differential billing, and if so, how much.

(6) Ordering defendant Meralco to pay attorney's fees in the amount


3. In declaring that petitioner ME RALCO had the burden of proof to show by clear and
of P200,000.00
convincing evidence that with respect to the tampered meters that TEC and/or TPC
authored their tampering.
Costs against defendant Meralco.
4. In finding that petitioner Meralco should not have held TEC and/or TPC responsible
SO ORDERED.20 for the acts of Ultra.

The trial court found the evidence of petitioner insufficient to prove that TEC was guilty of 5. In finding that TEC should not be held liable for the tampering of this electric meter
tampering the meter installations. The deformed condition of the meter seal and the existence of in its DCIM Building.
an opening in the wire duct leading to the transformer vault did not, in themselves, prove the
alleged tampering, especially since access to the transformer was given only to petitioner's
6. In finding that there was no notice of disconnection.
employees.21 The sudden drop in TEC's (or Ultra's) electric consumption did not, per se, show
meter tampering. The delay in the sending of notice of the results of the inspection was likewise
viewed by the court as evidence of inefficiency and arbitrariness on the part of petitioner. More 7. In finding that petitioner MERALCO was negligent in informing TEC of the alleged
importantly, petitioner's act of disconnecting the DCIM building's electric supply constituted bad tampering.
faith and thus makes it liable for damages.22 The court further denied petitioner's claim of
differential billing primarily on the ground of equitable negligence. 23Considering that TEC and
8. In making the finding that it is difficult to believe that when petitioner MERALCO
TPC paid P1,000,000.00 to avert the disconnection of electric power; and because Ultra
manifested to settle the claims of petitioner, the court imposed solidary liability on both Ultra and inspected on June 7, 1988 the meter installations, they were found to be tampered.
petitioner for the payment of the P1,000,000.00.
9. In declaring that petitioner MERALCO estopped from claiming any tampering of the
Ultra and petitioner appealed to the CA which affirmed the RTC decision, with a modification of meters.
the amount of actual damages and interest thereon. The dispositive portion of the CA decision
dated June 18, 1997, states: 10. In finding that "the method employed by MERALCO to as certain (sic) the 'correct'
amount of electricity consumed is questionable";
WHEREFORE, this Court renders judgment affirming in toto the Decision rendered by
the trial court with the slight modification that the interest at legal rate shall be 11. In declaring that MERALCO all throughout its dealings with TEC took on an
computed from January 13, 1989 and that Meralco shall pay plaintiff T.E.A.M. "attitude" which is oppressive, wanton and reckless.
Electronics Corporation and Technology Electronics Assembly and Management
Pacific Corporation the sum of P150,000.00 per month for five (5) months for actual
damages incurred when it was compelled to lease a generator set with interest at the 12. In declaring that MERALCO acted arbitrarily in inspecting TEC's DCIM building
legal rate from the above-stated date. and the NS building.

SO ORDERED.24 13. In declaring that respondents TEC and TPC are entitled to the damages which it
awarded.

The appellate court agreed with the RTC's conclusion. In addition, it considered petitioner
negligent for failing to discover the alleged defects in the electric meters; in belatedly notifying 14. In not declaring that petitioner is entitled to the differential bill.
15. In not declaring that respondents are liable to petitioner for exemplary damages, they would register the correct consumption of TEC. Logically, then, after the correction of the
attorney's fee and expenses for litigation.25 allegedly tampered meters, the customer's registered consumption would go up.

The petition must fail. In this case, the period claimed to have been affected by the tampered electric meters is from
February 1986 until September 1987. Based on petitioner's Billing Record31 (for the DCIM
building), TEC's monthly electric consumption on Account No. 9341-1322-16 was between 4,500
The issues for resolution can be summarized as follows: 1) whether or not TEC tampered with
and 27,000 kwh.32 Account No. 9341-1812-13 showed a monthly consumption between 9,600
the electric meters installed at its DCIM and NS buildings; 2) If so, whether or not it is liable for
and 34,200 kwh.33 It is interesting to note that, after correction of the allegedly tampered meters,
the differential billing as computed by petitioner; and 3) whether or not petitioner was justified in
TEC's monthly electric consumption from October 1987 to February 1988 (the last month that
disconnecting the electric power supply in TEC's DCIM building.
Ultra occupied the DCIM building) was between 8,700 and 24,300 kwh in its first account, and
16,200 to 46,800 kwh on the second account.
Petitioner insists that the tampering of the electric meters installed at the DCIM and NS buildings
owned by respondent TEC has been established by overwhelming evidence, as specifically
Even more revealing is the fact that TEC's meters registered 9,300 kwh and 19,200 kwh
shown by the shorting devices found during the inspection. Thus, says petitioner, tampering of
consumption on the first and second accounts, respectively, a month prior to the inspection. On
the meter is no longer an issue.
the first month after the meters were corrected, TEC's electric consumption registered at 9,300
kwh and 22,200 kwh on the respective accounts. These figures clearly show that there was no
It is obvious that petitioner wants this Court to revisit the factual findings of the lower courts. palpably drastic difference between the consumption before and after the inspection, casting a
Well-established is the doctrine that under Rule 45 of the Rules of Court, only questions of law, cloud of doubt over petitioner's claim of meter-tampering. Indeed, Ultra's explanation that the
not of fact, may be raised before the Court. We would like to stress that this Court is not a trier of corporation was losing; thus, it had lesser consumption of electric power appear to be the more
facts and may not re-examine and weigh anew the respective evidence of the parties. Factual plausible reason for the drop in electric consumption.
findings of the trial court, especially those affirmed by the Court of Appeals, are binding on this
Court.26
Petitioner likewise claimed that when the subject meters were again inspected on June 7, 1988,
they were found to have been tampered anew. The Court notes that prior to the inspection, TEC
Looking at the record, we note that petitioner claims to have discovered three incidences of was informed about it; and months before the inspection, there was an unsettled controversy
meter-tampering; twice in the DCIM building on September 28, 1987 and June 7, 1988; and between TEC and petitioner, brought about by the disconnection of electric power and the non-
once in the NS building on April 24, 1988. payment of differential billing. We are more disposed to accept the trial court's conclusion that it
is hard to believe that a customer previously apprehended for tampered meters and
assessed P7 million would further jeopardize itself in the eyes of petitioner.34 If it is true that
The first instance was supposedly discovered on September 28, 1987. The inspector allegedly
there was evidence of tampering found on September 28, 1987 and again on June 7, 1988, the
found the presence of a short circuiting device and saw that the meter seal was deformed. In better view would be that the defective meters were not actually corrected after the first
addition, petitioner, through the Supervising Engineer of its Special Billing Analysis inspection. If so, then Manila Electric Company v. Macro Textile Mills Corporation35 would apply,
Department,27 claimed that there was a sudden and unexplainable drop in TEC's electrical where we said that we cannot sanction a situation wherein the defects in the electric meter are
consumption starting February 10, 1986. On the basis of the foregoing, petitioner concluded that allowed to continue indefinitely until suddenly, the public utilities demand payment for the
the electric meters were tampered with. unrecorded electricity utilized when they could have remedied the situation immediately.
Petitioner's failure to do so may encourage neglect of public utilities to the detriment of the
However, contrary to petitioner's claim that there was a drastic and unexplainable drop in TEC's consuming public. Corollarily, it must be underscored that petitioner has the imperative duty to
electric consumption during the affected period, the Pattern of TEC's Electrical make a reasonable and proper inspection of its apparatus and equipment to ensure that they do
Consumption28 shows that the sudden drop is not peculiar to the said period. Noteworthy is the not malfunction, and the due diligence to discover and repair defects therein. Failure to perform
observation of the RTC in this wise: such duties constitutes negligence.36 By reason of said negligence, public utilities run the risk of
forfeiting amounts originally due from their customers.37
In fact, in Account No. 09341-1812-13 (heretofore referred as Account/Meter No. 2),
as evidenced by Exhibits "35" and "35-A," there was likewise a sudden drop of As to the alleged tampering of the electric meter in TEC's NS building, suffice it to state that the
electrical consumption from the year 1984 which recorded an average 141,300 allegation was not proven, considering that the meters therein were enclosed in a metal cabinet
kwh/month to 1985 which recorded an average kwh/month at 87,600 or a difference- the metal seal of which was unbroken, with petitioner having sole access to the said meters.38
drop of 53,700 kwh/month; from 1985's 87,600 recorded consumption, the same
dropped to 18,600 kwh/month or a difference-drop of 69,000 kwh/month. Surely, a
In view of the negative finding on the alleged tampering of electric meters on TEC's DCIM and
drop of 53,700 could be equally categorized as a sudden drop amounting to 69,000 NS buildings, petitioner's claim of differential billing was correctly denied by the trial and
which, incidentally, the Meralco claimed as "unexplainable. x x x.29 appellate courts. With greater reason, therefore, could petitioner not exercise the right of
immediate disconnection.
The witnesses for petitioner who testified on the alleged tampering of the electric meters,
declared that tampering is committed by consumers to prevent the meter from registering the The law in force at the time material to this controversy was Presidential Decree (P.D.) No.
correct amount of electric consumption, and result in a reduced monthly electric bill, while
40139 issued on March 1, 1974.40 The decree penalized unauthorized installation of water,
continuing to enjoy the same power supply. Only the registration of actual electric energy electrical or telephone connections and such acts as the use of tampered electrical meters. It
consumption, not the supply of electricity, is affected when a meter is tampered with. 30 The was issued in answer to the urgent need to put an end to illegal activities that prejudice the
witnesses claimed that after the inspection, the tampered electric meters were corrected, so that
economic well-being of both the companies concerned and the consuming public. 41 P.D. 401 likewise proper, pursuant to Article 220848 of the Civil Code. It is obvious that TEC needed the
granted the electric companies the right to conduct inspections of electric meters and the services of a lawyer to argue its cause through three levels of the judicial hierarchy. Thus, the
criminal prosecution42of erring consumers who were found to have tampered with their electric award of P200,000.00 is in order.49
meters. It did not expressly provide for more expedient remedies such as the charging of
differential billing and immediate disconnection against erring consumers. Thus, electric
We, however, deem it proper to delete the award of moral damages. TEC's claim was premised
companies found a creative way of availing themselves of such remedies by inserting into their
allegedly on the damage to its goodwill and reputation.50 As a rule, a corporation is not entitled to
service contracts (or agreements for the sale of electric energy) a provision for differential billing
moral damages because, not being a natural person, it cannot experience physical suffering or
with the option of disconnection upon non-payment by the erring consumer. The Court has
sentiments like wounded feelings, serious anxiety, mental anguish and moral shock. The only
recognized the validity of such stipulations.43 However, recourse to differential billing with
exception to this rule is when the corporation has a reputation that is debased, resulting in its
disconnection was subject to the prior requirement of a 48-hour written notice of disconnection.44
humiliation in the business realm.51 But in such a case, it is imperative for the claimant to present
proof to justify the award. It is essential to prove the existence of the factual basis of the damage
Petitioner, in the instant case, resorted to the remedy of disconnection without prior notice. While and its causal relation to petitioner's acts.52 In the present case, the records are bereft of any
it is true that petitioner sent a demand letter to TEC for the payment of differential billing, it did evidence that the name or reputation of TEC/TPC has been debased as a result of petitioner's
not include any notice that the electric supply would be disconnected. In fine, petitioner abused acts. Besides, the trial court simply awarded moral damages in the dispositive portion of its
the remedies granted to it under P.D. 401 and Revised General Order No. 1 by outrightly decision without stating the basis thereof.
depriving TEC of electrical services without first notifying it of the impending disconnection.
Accordingly, the CA did not err in affirming the RTC decision.
WHEREFORE, the petition is DENIED. The Decision of the Court of Appeals in CA-G.R. CV No.
40282 dated June 18, 1997 and its Resolution dated December 3, 1997 are AFFIRMED with the
As to the damages awarded by the CA, we deem it proper to modify the same. Actual damages following MODIFICATIONS: (1) the award of P150,000.00 per month for five months as
are compensation for an injury that will put the injured party in the position where it was before reimbursement for the rentals of the generator set is REDUCEDto P150,000.00; and (2) the
the injury. They pertain to such injuries or losses that are actually sustained and susceptible of award of P500,000.00 as moral damages is hereby DELETED.
measurement. Except as provided by law or by stipulation, a party is entitled to adequate
compensation only for such pecuniary loss as is duly proven. Basic is the rule that to recover
SO ORDERED.
actual damages, not only must the amount of loss be capable of proof; it must also be actually
proven with a reasonable degree of certainty, premised upon competent proof or the best
evidence obtainable.45

Respondent TEC sufficiently established, and petitioner in fact admitted, that the former
paid P1,000,000.00 and P280,813.72 under protest, the amounts representing a portion of the
latter's claim of differential billing. With the finding that no tampering was committed and, thus,
no differential billing due, the aforesaid amounts should be returned by petitioner, with interest,
as ordered by the Court of Appeals and pursuant to the guidelines set forth by the Court. 46

However, despite the appellate court's conclusion that no tampering was committed, it held Ultra
solidarily liable with petitioner for P1,000,000.00, only because the former, as occupant of the
building, promised to settle the claims of the latter. This ruling is erroneous. Ultra's promise was
conditioned upon the finding of defect or tampering of the meters. It did not acknowledge any
culpability and liability, and absent any tampered meter, it is absurd to make the lawful occupant
liable. It was petitioner who received the P1 million; thus, it alone should be held liable for the
return of the amount.

TEC also sufficiently established its claim for the reimbursement of the amount paid as rentals
for the generator set it was constrained to rent by reason of the illegal disconnection of electrical
service. The official receipts and purchase orders submitted by TEC as evidence sufficiently
show that such rentals were indeed made. However, the amount of P150,000.00 per month for
five months, awarded by the CA, is excessive. Instead, a total sum of P150,000.00, as found by
the RTC, is proper.

As to the payment of exemplary damages and attorney's fees, we find no cogent reason to
disturb the same. Exemplary damages are imposed by way of example or correction for the
public good in addition to moral, temperate, liquidated, or compensatory damages.47 In this case,
to serve as an example – that before a disconnection of electrical supply can be effected by a
public utility, the requisites of law must be complied with – we affirm the award of P200,000.00
as exemplary damages. With the award of exemplary damages, the award of attorney's fees is
G.R. No. 100812 June 25, 1999 II.

FRANCISCO MOTORS CORPORATION, petitioner, WHETHER OR NOT PLAINTIFF-APPELLANT NOT BEING A REAL
vs. PARTY IN THE ALLEGED PERMISSIVE COUNTERCLAIM SHOULD BE
COURT OF APPEALS and SPOUSES GREGORIO and LIBRADA MANUEL, respondents. HELD LIABLE TO THE CLAIM OF DEFENDANT-APPELLEES.

QUISUMBING, J.: III.

This petition for review on certiorari, under Rule 45 of the Rules of Court, seeks to annul the WHETHER OR NOT THERE IS FAILURE ON THE PART OF PLAINTIFF-
decision 1 of the Court of Appeals in C.A. G.R. CV No. 10014 affirming the decision rendered by APPELLANT TO ANSWER THE ALLEGED PERMISSIVE
Branch 135, Regional Trial Court of Makati, Metro Manila. The procedural antecedents of this COUNTERCLAIM. 8
petition are as follows:
Petitioner contended that the trial court did not acquire jurisdiction over it because no summons
On January 23, 1985, petitioner filed a complaint 2 against private respondents to recover three was validly served on it together with the copy of the answer containing the permissive
thousand four hundred twelve and six centavos (P3,412.06), representing the balance of the counterclaim. Further, petitioner questions the propriety of its being made party to the case
jeep body purchased by the Manuels from petitioner; an additional sum of twenty thousand four because it was not the real party in interest but the individual members of the Francisco family
hundred fifty-four and eighty centavos (P20,454.80) representing the unpaid balance on the cost concerned with the intestate case.
of repair of the vehicle; and six thousand pesos (P6,000.00) for cost of suit and attorney's
fees. 3 To the original balance on the price of jeep body were added the costs of repair. 4 In their
In its assailed decision now before us for review, respondent Court of Appeals held that a
answer, private respondents interposed a counterclaim for unpaid legal services by Gregorio
counterclaim must be answered in ten (10) days, pursuant to Section 4, Rule 11, of the Rules of
Manuel in the amount of fifty thousand pesos (P50,000) which was not paid by the incorporators,
Court; and nowhere does it state in the Rules that a party still needed to be summoned anew if a
directors and officers of the petitioner. The trial court decided the case on June 26, 1985, in
counterclaim was set up against him. Failure to serve summons, said respondent court, did not
favor of petitioner in regard to the petitioner's claim for money, but also allowed the counter-
effectively negate trial court's jurisdiction over petitioner in the matter of the counterclaim. It
claim of private respondents. Both parties appealed. On April 15, 1991, the Court of Appeals
likewise pointed out that there was no reason for petitioner to be excused from answering the
sustained the trial court's decision. 5 Hence, the present petition.
counterclaim. Court records showed that its former counsel, Nicanor G. Alvarez, received the
copy of the answer with counterclaim two (2) days prior to his withdrawal as counsel for
For our review in particular is the propriety of the permissive counterclaim which private petitioner. Moreover when petitioner's new counsel, Jose N. Aquino, entered his appearance,
respondents filed together with their answer to petitioner's complaint for a sum of money. Private three (3) days still remained within the period to file an answer to the counterclaim. Having failed
respondent Gregorio Manuel alleged as an affirmative defense that, while he was petitioner's to answer, petitioner was correctly considered in default by the trial
Assistant Legal Officer, he represented members of the Francisco family in the intestate estate court. 9 Even assuming that the trial court acquired no jurisdiction over petitioner, respondent
proceedings of the late Benita Trinidad. However, even after the termination of the proceedings, court also said, but having filed a motion for reconsideration seeking relief from the said order of
his services were not paid. Said family members, he said, were also incorporators, directors and default, petitioner was estopped from further questioning the trial court's jurisdiction. 10
officers of petitioner. Hence to petitioner's collection suit, he filed a counter permissive
counterclaim for the unpaid attorney's fees. 6
On the question of its liability for attorney's fees owing to private respondent Gregorio Manuel,
petitioner argued that being a corporation, it should not be held liable therefor because these
For failure of petitioner to answer the counterclaim, the trial court declared petitioner in default fees were owed by the incorporators, directors and officers of the corporation in their personal
on this score, and evidence ex-parte was presented on the counterclaim. The trial court ruled in capacity as heirs of Benita Trinidad. Petitioner stressed that the personality of the
favor of private respondents and found that Gregorio Manuel indeed rendered legal services to corporation, vis-a-vis the individual persons who hired the services of private respondent, is
the Francisco family in Special Proceedings Number 7803 — "In the Matter of Intestate Estate of separate and distinct, 11 hence, the liability of said individuals did not become an obligation
Benita Trinidad". Said court also found that his legal services were not compensated despite chargeable against petitioner.
repeated demands, and thus ordered petitioner to pay him the amount of fifty thousand
(P50,000.00) pesos. 7
Nevertheless, on the foregoing issue, the Court of Appeals ruled as follows:

Dissatisfied with the trial court's order, petitioner elevated the matter to the Court of Appeals,
However, this distinct and separate personality is merely a fiction created by
posing the following issues:
law for convenience and to promote justice. Accordingly, this separate
personality of the corporation may be disregarded, or the veil of corporate
I. fiction pierced, in cases where it is used as a cloak or cover for found (sic)
illegality, or to work an injustice, or where necessary to achieve equity or
when necessary for the protection of creditors. (Sulo ng Bayan, Inc. vs.
WHETHER OR NOT THE DECISION RENDERED BY THE LOWER
Araneta, Inc., 72 SCRA 347) Corporations are composed of natural persons
COURT IS NULL AND VOID AS IT NEVER ACQUIRED JURISDICTION
and the legal fiction of a separate corporate personality is not a shield for
OVER THE PERSON OF THE DEFENDANT.
the commission of injustice and inequity. (Chemplex Philippines, Inc. vs.
Pamatian, 57 SCRA 408).
In the instant case, evidence shows that the plaintiff-appellant Francisco assert that the members of petitioner corporation took advantage of their positions by not
Motors Corporation is composed of the heirs of the late Benita Trinidad as compensating respondent Gregorio Manuel after the termination of the estate proceedings
directors and incorporators for whom defendant Gregorio Manuel rendered despite his repeated demands for payment of his services. They cite findings of the appellate
legal services in the intestate estate case of their deceased mother. court that support piercing the veil of corporate identity in this particular case. They assert that
Considering the aforestated principles and circumstances established in this the corporate veil may be disregarded when it is used to defeat public convenience, justify
case, equity and justice demands plaintiff-appellant's veil of corporate wrong, protect fraud, and defend crime. It may also be pierced, according to them, where the
identity should be pierced and the defendant be compensated for legal corporate entity is being used as an alter ego, adjunct, or business conduit for the sole benefit of
services rendered to the heirs, who are directors of the plaintiff-appellant the stockholders or of another corporate entity. In these instances, they aver, the corporation
corporation. 12 should be treated merely as an association of individual persons. 16

Now before us, petitioner assigns the following errors: Private respondents dispute petitioner's claim that its right to due process was violated when
respondents' counterclaim was granted due course, although no summons was served upon it.
They claim that no provision in the Rules of Court requires service of summons upon a
I.
defendant in a counterclaim. Private respondents argue that when the petitioner filed its
complaint before the trial court it voluntarily submitted itself to the jurisdiction of the court. As a
THE COURT OF APPEALS ERRED IN APPLYING THE DOCTRINE OF consequence, the issuance of summons on it was no longer necessary. Private respondents say
PIERCING THE VEIL OF CORPORATE ENTITY. they served a copy of their answer with affirmative defenses and counterclaim on petitioner's
former counsel, Nicanor G. Alvarez. While petitioner would have the Court believe that
respondents served said copy upon Alvarez after he had withdrawn his appearance as counsel
II.
for the petitioner, private respondents assert that this contention is utterly baseless. Records
disclose that the answer was received two (2) days before the former counsel for petitioner
THE COURT OF APPEALS ERRED IN AFFIRMING THAT THERE WAS withdrew his appearance, according to private respondents. They maintain that the present
JURISDICTION OVER PETITIONER WITH RESPECT TO THE petition is but a form of dilatory appeal, to set off petitioner's obligations to the respondents by
COUNTERCLAIM. 13 running up more interest it could recover from them. Private respondents therefore claim
damages against petitioner. 17
Petitioner submits that respondent court should not have resorted to piercing the veil of
corporate fiction because the transaction concerned only respondent Gregorio Manuel and the To resolve the issues in this case, we must first determine the propriety of piercing the veil of
heirs of the late Benita Trinidad. According to petitioner, there was no cause of action by said corporate fiction.
respondent against petitioner; personal concerns of the heirs should be distinguished from those
involving corporate affairs. Petitioner further contends that the present case does not fall among Basic in corporation law is the principle that a corporation has a separate personality distinct
the instances wherein the courts may look beyond the distinct personality of a corporation. from its stockholders and from other corporations to which it may be connected. 18 However,
According to petitioner, the services for which respondent Gregorio Manuel seeks to collect fees under the doctrine of piercing the veil of corporate entity, the corporation's separate juridical
from petitioner are personal in nature. Hence, it avers the heirs should have been sued in their personality may be disregarded, for example, when the corporate identity is used to defeat
personal capacity, and not involve the corporation. 14
public convenience, justify wrong, protect fraud, or defend crime. Also, where the corporation is
a mere alter ego or business conduit of a person, or where the corporation is so organized and
With regard to the permissive counterclaim, petitioner also insists that there was no proper controlled and its affairs are so conducted as to make it merely an instrumentality, agency,
service of the answer containing the permissive counterclaim. It claims that the counterclaim is a conduit or adjunct of another corporation, then its distinct personality may be ignored. 19 In these
separate case which can only be properly served upon the opposing party through summons. circumstances, the courts will treat the corporation as a mere aggrupation of persons and the
Further petitioner states that by nature, a permissive counterclaim is one which does not arise liability will directly attach to them. The legal fiction of a separate corporate personality in those
out of nor is necessarily connected with the subject of the opposing party's claim. Petitioner cited instances, for reasons of public policy and in the interest of justice, will be justifiably set
avers that since there was no service of summons upon it with regard to the counterclaim, then aside.
the court did not acquire jurisdiction over petitioner. Since a counterclaim is considered an action
independent from the answer, according to petitioner, then in effect there should be two
In our view, however, given the facts and circumstances of this case, the doctrine of piercing the
simultaneous actions between the same parties: each party is at the same time both plaintiff and corporate veil has no relevant application here. Respondent court erred in permitting the trial
defendant with respect to the other, 15 requiring in each case separate summonses.
court's resort to this doctrine. The rationale behind piercing a corporation's identity in a given
case is to remove the barrier between the corporation from the persons comprising it to thwart
In their Comment, private respondents focus on the two questions raised by petitioner. They the fraudulent and illegal schemes of those who use the corporate personality as a shield for
defend the propriety of piercing the veil of corporate fiction, but deny the necessity of serving undertaking certain proscribed activities. However, in the case at bar, instead of holding certain
separate summonses on petitioner in regard to their permissive counterclaim contained in the individuals or persons responsible for an alleged corporate act, the situation has been reversed.
answer. It is the petitioner as a corporation which is being ordered to answer for the personal liability of
certain individual directors, officers and incorporators concerned. Hence, it appears to us that
the doctrine has been turned upside down because of its erroneous invocation. Note that
Private respondents maintain both trial and appellate courts found that respondent Gregorio according to private respondent Gregorio Manuel his services were solicited as counsel for
Manuel was employed as assistant legal officer of petitioner corporation, and that his services members of the Francisco family to represent them in the intestate proceedings over Benita
were solicited by the incorporators, directors and members to handle and represent them in Trinidad's estate. These estate proceedings did not involve any business of petitioner.
Special Proceedings No. 7803, concerning the Intestate Estate of the late Benita Trinidad. They
Note also that he sought to collect legal fees not just from certain Francisco family members but voluntarily submitting himself to its jurisdiction. (Tejones vs. Gironella, 159
also from petitioner corporation on the claims that its management had requested his services SCRA 100). Estoppel is a bar against any claims of lack of jurisdiction.
and he acceded thereto as an employee of petitioner from whom it could be deduced he was (Balais vs. Balais, 159 SCRA 37). 22
also receiving a salary. His move to recover unpaid legal fees through a counterclaim against
Francisco Motors Corporation, to offset the unpaid balance of the purchase and repair of a jeep
WHEREFORE, the petition is hereby GRANTED and the assailed decision is hereby
body could only result from an obvious misapprehension that petitioner's corporate assets could
REVERSED insofar only as it held Francisco Motors Corporation liable for the legal obligation
be used to answer for the liabilities of its individual directors, officers, and incorporators. Such
owing to private respondent Gregorio Manuel; but this decision is without prejudice to his filing
result if permitted could easily prejudice the corporation, its own creditors, and even other
the proper suit against the concerned members of the Francisco family in their personal
stockholders; hence, clearly inequitous to petitioner.
capacity. No pronouncement as to costs.

Furthermore, considering the nature of the legal services involved, whatever obligation said
SO ORDERED.
incorporators, directors and officers of the corporation had incurred, it was incurred in their
personal capacity. When directors and officers of a corporation are unable to compensate a
party for a personal obligation, it is far-fetched to allege that the corporation is perpetuating fraud
or promoting injustice, and be thereby held liable therefor by piercing its corporate veil. While
there are no hard and fast rules on disregarding separate corporate identity, we must always be
mindful of its function and purpose. A court should be careful in assessing the milieu where the
doctrine of piercing the corporate veil may be applied. Otherwise an injustice, although
unintended, may result from its erroneous application.

The personality of the corporation and those of its incorporators, directors and officers in their
personal capacities ought to be kept separate in this case. The claim for legal fees against the
concerned individual incorporators, officers and directors could not be properly directed against
the corporation without violating basic principles governing corporations. Moreover, every action
— including a counterclaim — must be prosecuted or defended in the name of the real party in
interest. 20 It is plainly an error to lay the claim for legal fees of private respondent Gregorio
Manuel at the door of petitioner (FMC) rather than individual members of the Francisco family.

However, with regard to the procedural issue raised by petitioner's allegation, that it needed to
be summoned anew in order for the court to acquire jurisdiction over it, we agree with
respondent court's view to the contrary. Section 4, Rule 11 of the Rules of Court provides that a
counterclaim or cross-claim must be answered within ten (10) days from service. Nothing in the
Rules of Court says that summons should first be served on the defendant before an answer to
counterclaim must be made. The purpose of a summons is to enable the court to acquire
jurisdiction over the person of the defendant. Although a counterclaim is treated as an entirely
distinct and independent action, the defendant in the counterclaim, being the plaintiff in the
original complaint, has already submitted to the jurisdiction of the court. Following Rule 9,
Section 3 of the 1997 Rules of Civil Procedure, 21 if a defendant (herein petitioner) fails to
answer the counterclaim, then upon motion of plaintiff, the defendant may be declared in default.
This is what happened to petitioner in this case, and this Court finds no procedural error in the
disposition of the appellate court on this particular issue. Moreover, as noted by the respondent
court, when petitioner filed its motion seeking to set aside the order of default, in effect it
submitted itself to the jurisdiction of the court. As well said by respondent court:

Further on the lack of jurisdiction as raised by plaintiff-appellant[,] [t]he


records show that upon its request, plaintiff-appellant was granted time to
file a motion for reconsideration of the disputed decision. Plaintiff-appellant
did file its motion for reconsideration to set aside the order of default and the
judgment rendered on the counterclaim.

Thus, even if the court acquired no jurisdiction over plaintiff-appellant on the


counterclaim, as it vigorously insists, plaintiff-appellant is considered to have
submitted to the court's jurisdiction when it filed the motion for
reconsideration seeking relief from the court. (Soriano vs. Palacio, 12 SCRA
447). A party is estopped from assailing the jurisdiction of a court after
G.R. No. L-30822 July 31, 1975 On December 14, 1964, respondent workers were accompanied by the Chief of Police of
Talisay, Negros Occidental to the compound of herein petitioner company to report for
reinstatement per order of the court. Respondent workers were, however, refused reinstatement
EDUARDO CLAPAROLS, ROMULO AGSAM and/or CLAPAROLS STEEL AND NAIL
by company accountant Francisco Cusi for he had no order from plant owner Eduardo Claparols
PLANT, petitioners,
nor from his lawyer Atty. Plaridel Katalbas, to reinstate respondent workers.
vs.
COURT OF INDUSTRIAL RELATIONS, ALLIED WORKERS' ASSOCIATION and/or
DEMETRIO GARLITOS, ALFREDO ONGSUCO, JORGE SEMILLANO, SALVADOR Again, on December 15, 1964, respondent workers were accompanied by a police officer to the
DOROTEO, ROSENDO ESPINOSA, LUDOVICO BALOPENOS, ASER AMANCIO, MAXIMO company compound, but then, they were again refused reinstatement by Cusi on the same
QUIOYO, GAUDENCIO QUIOYO, and IGNACIO QUIOYO, respondents. ground.

MAKASIAR, J.: On January 15, 1965, the CIR Chief Examiner Submitted his report containing three
computations, to wit:
A petition for certiorari to set aside the order of respondent Court of Industrial Relations dated
May 30, 1969 directing petitioners to pay back wages and bonuses to private respondents as The first computation covers the period February 1, 1957 to October 31,
well as its resolution of July 5, 1969 denying the motion for reconsideration of said order in Case 1964. The second is up to and including December 7, 1962, when the
No. 32-ULP-Iloilo entitled "Allied Workers' Association, et. al., versus Eduardo Claparols, et. al.." corporation stopped operations, while the third is only up to June 30, 1957
when the Claparols Steel and Nail Plant ceased to operate (Annex B,
Petition for Review on Certiorari, p. 14, Brief for appellees, p. 113, rec.).
It appears that on August 6, 1957, a complaint for unfair labor practice was filed by herein
private respondent Allied Workers' Association, respondent Demetrio Garlitos and ten (10)
respondent workers against herein petitioners on account of the dismissal of respondent workers with the explanation that:
from petitioner Claparols Steel and Nail Plant.
6. Since the records of the Claparols Steel Corporation show that it was
On September 16, 1963, respondent Court rendered its decision finding "Mr. Claparols guilty of established on July 1, 1957 succeeding the Claparols Steel and Nail Plant
union busting and" of having "dismissed said complainants because of their union activities," which ceased operations on June 30, 1957, and that the Claparols Steel
and ordering respondents "(1) To cease and desist from committing unfair labor practices Corporation stopped operations on December 7, 1962, three (3)
against their employees and laborers; (2) To reinstate said complainants to their former or computations are presented herein for the consideration of this Honorable
equivalent jobs, as soon as possible, with back wages from the date of their dismissal up to their Court (p. 2, Report of Examiner, p. 29, rec.).
actual reinstatement" (p. 12, Decision; p. 27, rec.).
On January 23, 1965, petitioners filed an opposition alleging that under the circumstances
A motion to reconsider the above decision was filed by herein petitioners, which respondent presently engulfing the company, petitioner Claparols could not personally reinstate respondent
Court, sitting en banc, denied in a resolution dated January 27, 1964. workers; that assuming the workers are entitled to back wages, the same should only be limited
to three months pursuant to the court ruling in the case of Sta. Cecilia Sawmills vs. CIR (L-
19273-74, February 20, 1964); and that since Claparols Steel Corporation ceased to operate on
On March 30, 1964, counsel for herein respondent workers (complainants in the ULP case) filed
December 7, 1962, re-employment of respondent workers cannot go beyond December 7, 1962.
a motion for execution of respondent Court's September 16, 1963 decision.

A reply to petitioner's opposition was filed by respondent workers, alleging among others, that
On May 14, 1964, respondent Court, in its order of September 16, 1963, granted execution and
Claparols Steel and Nail Plant and Claparols Steel and Nail Corporation are one and the same
directed herein petitioners
corporation controlled by petitioner Claparols, with the latter corporation succeeding the former.

to reinstate the above complainants to their former or equivalent jobs within


On November 28, 1966, after conducting a series of hearings on the report of the examiner,
five (5) days after receipt of a copy of this order. In order to implement the
respondent Court issued an order, the dispositive portion of which reads:
award of back wages, the Chief of the Examining Division or any of his
assistants is hereby directed to proceed to the office of the respondents at
Matab-ang, Talisay, Negros Occidental, and examine its payrolls and other WHEREFORE, the Report of the. Examiner filed on January 15, 1965, is
pertinent records and compute the back wages of the complainants in hereby approved subject to the foregoing findings and dispositions.
accordance with the decision dated September 16, 1963, and, upon Consequently, the Corporation Auditing Examiner is directed to recompute
termination, to submit his report as soon as possible for further disposition the back wages of complainants Demetrio Garlitos and Alfredo Ongsuco on
(p. 7, Brief for Respondents, p. 113, rec.). the basis of P200.00 and P270.00 a month, respectively; to compute those
of complainant Ignacio Quioyo as aforesaid; to compute the deductible
earnings of complainants Ongsuco, Jorge Semillano and Garlitos, as found
which was reiterated by respondent Court in a subsequent order dated November 10, 1964 (pp.
in the body of this order; and to compute the bonuses of each and every
7-8, Brief for Respondents, p. 113, rec.).
complainant, except Honorato Quioyo. Thereafter, as soon as possible, the
Examiner should submit a report in compliance herewith of the Court's
further disposition (p. 24, Brief for Respondents, p. 113, rec.).
On December 7, 1966, a motion for reconsideration was filed by petitioner, assailing respondent Then on March 21, 1968, the Chief Examiner came out with his report, the disputed portion of
Court's ruling that (1) the ruling in the case of Sta. Cecilia Sawmills Inc. CIR, et. al, does not which (regarding bonuses) reads:
apply in the case at bar; and (2) that bonus should be included in the recoverable wages.
xxx xxx xxx
On December 14, 1966, a counter-opposition was filed by private respondents alleging that
petitioners' motion for reconsideration was pro forma, it not making express reference to the
4. The yearly bonuses of the employees and laborers of respondent
testimony or documentary evidence or to the provision of law alleged to be contrary to such
corporation are given on the following basis:
findings or conclusions of respondent Court.

Basic Additional:
On February 8, 1967, respondent Court of Industrial Relations dismissed petitioners' motion for
reconsideration for being pro forma.
a. For every dependent 1% of monthly salary
Whereupon, petitioners filed a petition for certiorari with this COURT in G.R. No. L-27272 to set
aside the November 28, 1966 order of respondent Court, as well as its February 8, 1967 b. For every dependent in elementary grade 2% of
resolution. Petitioners assigned therein as errors of law the very same assignment of errors it monthly salary
raises in the present case, to wit:
c. For every dependent in high school 3% of monthly
I salary

THE RESPONDENT COURT ERRED AND/OR ACTED WITH GRAVE d. For every dependent in college 5% of monthly salary
ABUSE OF DISCRETION, AMOUNTING TO LACK OF JURISDICTION, IN
HOLDING IN THE ORDER UNDER REVIEW THAT BONUSES SHOULD
xxx xxx xxx
BE PAID TO THE RESPONDENT WORKERS DESPITE THE FACT THAT
THE SAME WAS NOT ADJUDICATED IN ITS ORIGINAL DECISION.
7. The computed ... bonuses after deducting the earnings elsewhere of
Messrs. Ongsuco, Garlitos and Semillano are as follows:
II

THE RESPONDENT COURT ERRED AND/OR ACTED WITH GRAVE Name x x x Bonuses x x x
ABUSE OF DISCRETION, AMOUNTING TO LACK OF JURISDICTION, IN
NOT APPLYING THE DOCTRINE LAID DOWN BY THIS HONORABLE 1. Alfredo Ongsuco P1,620.00
TRIBUNAL IN THE CASE OF "STA. CECILIA SAWMILLS, INC. VS. C.I.R., 2. Demetrio Garlitos 1,200.00
ET. AL.," G.R. No. 3. Ignacio Quioyo 455.23
L-19273-74, PROMULGATED ON FEBRUARY 29, 1964 (pp. 10-11, rec.). 4. Aser Abancio 461.00
5. Ludovico Belopeños 752.05
On April 27, 1967, the Supreme Court denied petitioners' petition for certiorari (p. 77, rec. of L- 6. Salvador Doroteo 714.70
27272), which was reiterated on May 19, 1967 (p. 27, Respondent's Brief, p. 113, rec.; p. 81, 7. Rosendo Espinosa 1,075.40
rec. of L-27272). 8. Gaudencio Quioyo 1,167.92
9. Jorge Semillano 1,212.08
10. Maximo Quioyo 449.41
On May 3, 1967, private respondents moved to have the workers' back wages properly Total P9,107.79
recomputed. A motion to the same end was reiterated by private respondents on June 14, 1967.
(Pp. 30-31, Respondent's Brief, p. 113, rec.)
On July 13, 1967, respondent Court directed a recomputation of the back wages of respondent
workers in accordance with its order dated November 28, 1966. The said order in part reads:
On April 16, 1968, petitioners filed their opposition to the report of the Examiner dated March 21,
1968 on grounds already rejected by respondent Court in its order dated November 28, 1966,
WHEREFORE, the Chief Auditing Examiner of the Court or any of his and by the Supreme Court also in its ruling in G.R. No. L-27272.
assistants, is hereby directed to recompute the back wages of the workers
involved in this case in accordance with the Order of November 28, 1966
within 20 days from receipt of a copy of this Order (p. 28, Brief for On May 4, 1968, a rejoinder to petitioners' opposition was filed by private respondents, alleging
among others "that the grounds of petitioners' opposition were the same grounds raised by them
Respondents, p. 113, rec.).
before and passed upon by respondent Court and this Honorable Tribunal; that this order of
November 28, 1966 which passed upon these issues became final and executory on June 3,
1967 from the Honorable Supreme Court. (Order of respondent Court dated July 13, 1967). [p. However, without lending a sympathetic ear to the obvious desire of herein petitioners of this
32, Brief for Respondents, p. 113, rec.]. Court to re-examine — which would be an exercise in futility — the final ruling in G.R. No. L-
27272, which as above-stated is the law of the instant case, but solely to remind herein
petitioners, We reiterate the governing principles.
On July 26, 1968, private respondents filed their motion for approval of the Report of the
Examiner submitted on March 21, 1968, alleging, among others, that petitioners, in their
opposition, did not actually dispute the data elicited by the Chief Examiner but rather harped on WE uniformly held that "a bonus is not a demandable and enforceable obligation, except when it
grounds which, as already stated, had already been turned down by the Supreme Court. is a part of the wage or salary compensation" (Philippine Education Co. vs. CIR and the Union of
Philippine Co. Employees [NLU], 92 Phil. 381; Ansay, et. al. vs. National Development Co., et.
al., 107 Phil. 998, 999; Emphasis supplied).
On October 19, 1968, herein private respondents filed their "Constancia", submitting the case for
resolution of respondent Court of Industrial Relations.
In Atok Big Wedge Mining Co. vs. Atok Big Wedge Mutual Benefit Association (92 Phil. 754), this
Court, thru Justice Labrador, held:
On May 30, 1969, respondent Court issued an order, subject of the present appeal, the
dispositive portion of which reads:
Whether or not bonus forms part of wages depends upon the condition or
circumstance for its payment. If it is an additional compensation WHICH
WHEREFORE, there being no proof offered to substantiate respondent
THE EMPLOYER PROMISED AND AGREED to give without any condition
Eduardo Claparols' opposition, the Examiner's Report should be, and it is
imposed for its payment ... then it is part of the wage. (Emphasis supplied).
hereby, APPROVED. Consequently, pursuant to the decision dated
September 16, 1963, respondent ... (petitioners herein) are hereby directed
to pay the respective back wages and bonuses of the In Altomonte vs. Philippine American Drug Co. (106 Phil. 137), the Supreme Court held that an
complainants (respondents herein) ... (p. 35, Brief for Respondents; p. 113, employee is not entitled to bonus where there is no showing that it had been granted by the
rec.; emphasis supplied). employer to its employees periodically or regularly as to become part of their wages or salaries.
The clear implication is that bonus is recoverable as part of the wage or salary where the
employer regularly or periodically gives it to employees.
On June 7, 1969, petitioners filed a motion for reconsideration on practically the same grounds
previously raised by them.
American jurisprudence equally regards bonuses as part of compensation or recoverable wages.
On June 30, 1969, respondents filed an opposition to petitioners' motion for reconsideration, with
the following allegations: Thus, it was held that "... it follows that in determining the regular rate of pay, a bonus which in
fact constitutes PART OF AN EMPLOYEE'S compensation, rather than a true gift or gratuity,
has to be taken into consideration." (48 Am. Jur. 2d, Labor and Labor Relations, No. 1555, citing
1. The issues raised, namely, whether bonuses should be included in the
the cases of Triple "AAA" Co. vs. Wirtz and Haber vs. Americana Corporation; Emphasis
award for back wages had already been resolved by respondent court in its
supplied). It was further held that "... the regular rate includes incentive bonuses paid to the
orders dated November 28, 1966, and December 7, 1966, and in the
employees in addition to the guaranteed base rates regardless of any contract provision to the
Resolution of the Honorable Supreme Court in G.R. No. L-27272 dated April
contrary and even though such bonuses could not be determined or paid until such time after the
26, 1967 and May 19, 1967, and the same is already a settled and final
pay day" (48 Am. Jur. 2d, Labor and Labor Relations, No. 1555, citing the case of Walling vs.
issue.
Harnischfeger Corp., 325 US 427, 89 L Ed 1711, 65 S Ct. 1246; Emphasis supplied).

2. Petitioners' motion for reconsideration is merely a rehash of previous


Petitioners in the present case do not dispute that as a matter of tradition, the company has
arguments, effete and unrejuvenated, pro forma, and intended merely to
been doling out bonuses to employees. In fact, the company balance sheets for the years 1956
delay the proceedings.
to 1962 contained bonus and pension computations which were never repudiated or questioned
by petitioners. As such, bonus for a given year earmarked as a matter of tradition for distribution
As correctly contended by private respondents, the present petition is barred by Our resolutions to employees has formed part of their recoverable wages from the company. Moreover, with
of April 26, 1967 and May 19, 1967 in G.R. No. L-27272 (Eduardo Claparols, et. al. vs. CIR, et. greater reason, should recovery of bonuses as part of back wages be observed in the present
al.) [pp. 77-83, rec. of L- 27272], dismissing said case, wherein said petitioners invoked the case since the company, in the light of the very admission of company accountant Francisco
applicability of the doctrine in Sta. Cecilia Sawmills, Inc. vs. CIR, et. al. (L-19273-74, Feb. 29, Cusi, distributes bonuses to its employees even if the company has suffered losses. Specifically,
1964, 10 SCRA 433) and impugned the illegality of the order of respondent Court dated petitioner company has done this in 1962 (t.s.n., p. 149, Sept. 20, 1965).
November 28, 1966 directing the computation and payment of the bonuses, aside from back
wages on the ground that these bonuses were not included in the decision of September 16,
Since bonuses are part of back wages of private respondents, the order of May 30, 1969,
1963, which had long become final.
directing the payment of their bonuses, did not amend the decision of September 16, 1963 of
respondent Court directing payment of their wages, which has long become final and executory,
The aforesaid resolutions in G.R. No. L-27272 constitute the law of the instant case, wherein in the same way that the previous order of May 14, 1964 granting execution of said decision of
herein petitioners raised again practically the same issues invoked in the abovementioned case. September 16, 1963 also directed the computation of the wages to be paid to private
The denial of the petition in G.R. No. L-27272 suffices to warrant the denial of the present respondents as decreed by the decision of September 16, 1963. All the orders of May 30, 1969,
petition; and We need not go any further.
November 28, 1966 and May 14, 1964 merely implement the already final and executory
decision of September 16, 1963.

Petitioners insist that We adopt the ruling in the Sta. Cecilia Sawmills case wherein the
recoverable back wages were limited to only three (3) months; because as in the Sta. Cecilia
Sawmills case, the Claparols Steel and Nail Plant ceased operations due to enormous business
reverses.

Respondent Court's findings that indeed the Claparols Steel and Nail Plant, which ceased
operation of June 30, 1957, was SUCCEEDED by the Claparols Steel Corporation effective the
next day, July 1, 1957 up to December 7, 1962, when the latter finally ceased to operate, were
not disputed by petitioners. It is very clear that the latter corporation was a continuation and
successor of the first entity, and its emergence was skillfully timed to avoid the financial liability
that already attached to its predecessor, the Claparols Steel and Nail Plant. Both predecessors
and successor were owned and controlled by the petitioner Eduardo Claparols and there was no
break in the succession and continuity of the same business. This "avoiding-the-liability" scheme
is very patent, considering that 90% of the subscribed shares of stocks of the Claparols Steel
Corporation (the second corporation) was owned by respondent (herein petitioner) Claparols
himself, and all the assets of the dissolved Claparols Steel and Nail Plant were turned over to
the emerging Claparols Steel Corporation.

It is very obvious that the second corporation seeks the protective shield of a corporate fiction
whose veil in the present case could, and should, be pierced as it was deliberately and
maliciously designed to evade its financial obligation to its employees.

It is well remembering that in Yutivo & Sons Hardware Company vs. Court of Tax Appeals (L-
13203, Jan. 28, 1961, 1 SCRA 160), We held that when the notion of legal entity is used to
defeat public convenience, justify wrong, protect fraud, or defend crime, the law will regard the
corporation as an association or persons, or, in the case of two corporations, will merge them
into one.

In Liddel & Company, Inc. vs. Collector of Internal Revenue (L-9687, June 30, 1961, 2 SCRA
632), this Court likewise held that where a corporation is a dummy and serves no business
purpose and is intended only as a blind, the corporate fiction may be ignored.

In Commissioner of Internal Revenue vs. Norton and Harrison Company (L-17618, Aug. 31,
1964, 11 SCRA 714), We ruled that where a corporation is merely an adjunct, business conduit
or alter ego of another corporation, the fiction of separate and distinct corporate entities should
be disregarded.

To the same uniform effect are the decisions in the cases of Republic vs. Razon (L-17462, May
29, 1967, 20 SCRA 234) and A.D. Santos, Inc. vs. Vasquez (L-23586, March 20, 1968, 22
SCRA 1156).

WE agree with respondent Court of Industrial Relations, therefore, that the amount of back
wages recoverable by respondent workers from petitioners should be the amount accruing up to
December 7, 1962 when the Claparols Steel Corporation ceased operations.

WHEREFORE, PETITION IS HEREBY DENIED WITH TREBLE COSTS AGAINST


PETITIONERS TO BE PAID BY THEIR COUNSEL.
G.R. No. 127181 September 4, 2001 On May 5, 1982, ECO submitted to LBP a "Revised Plan of Payment" deleting the
latter’s participation in the proposed financing company. The Trust Committee
deliberated on the "Revised Plan of Payment" and resolved to reject it. LBP then sent
LAND BANK OF THE PHILIPPINES, petitioner,
a letter to the PVTA for the latter’s comments. The letter stated that if LBP did not hear
vs.
from PVTA within five (5) days from the latter’s receipt of the letter, such silence would
THE COURT OF APPEALS, ECO MANAGEMENT CORPORATION and EMMANUEL C.
be construed to be an approval of LBP’s intention to file suit against ECO and its
OÑATE, respondents.
corporate officers. PVTA did not respond to the letter.

QUISUMBING, J.:
On June 28, 1982, Landbank filed a complaint for Collection of Sum of Money against
ECO and Emmanuel C. Oñate before the Regional Trial Court of Manila, Branch 50.
This petition for review on certiorari seeks to reverse and set aside the decision1 promulgated on
June 17, 1996 in CA-GR No. CV-43239 of public respondent and its resolution2 dated November
After trial on the merits, a judgment was rendered in favor of LBP; however, appellee
29, 1996 denying petitioner’s motion for reconsideration.3
Oñate was absolved from personal liability for insufficiency of evidence.

The facts of this case as found by the Court of Appeals and which we find supported by the
Dissatisfied, both parties filed their respective Motions for Reconsideration. LBP
records are as follows:
claimed that there was an error in computation in the amounts to be paid. LBP also
questioned the dismissal of the case with regard to Oñate.
On various dates in September, October, and November, 1980, appellant Land Bank
of the Philippines (LBP) extended a series of credit accommodations to appellee ECO,
On the other hand, ECO questioned its being held liable for the amount of the loan.
using the trust funds of the Philippine Virginia Tobacco Administration (PVTA) in the
Upon order of the court, both parties submitted Supplemental Motions for
aggregate amount of P26,109,000.00. The proceeds of the credit accommodations
Reconsideration and their respective Oppositions to each other’s Motions.
were received on behalf of ECO by appellee Oñate.

On February 3, 1993, the trial court rendered an Amended Decision, the dispositive
On the respective maturity dates of the loans, ECO failed to pay the same. Oral and
portion of which reads as follows:
written demands were made, but ECO was unable to pay. ECO claims that the
company was in financial difficulty for it was unable to collect its investments with
companies which were affected by the financial crisis brought about by the Dewey ACCORDINGLY, the Decision, dated December 3, 1990, is hereby modified
Dee scandal. to read as follows:

xxx WHEREFORE, judgment is rendered ordering defendant Eco Management


Corporation to pay plaintiff Land Bank of the Philippines:
On October 20, 1981, ECO proposed and submitted to LBP a "Plan of Payment"
whereby the former would set up a financing company which would absorb the loan A. The sum of P26,109,000.00 representing the total amount of the ten (10)
obligations. It was proposed that LBP would participate in the scheme through the loan accommodations plus 16% interest per annum computed from the
conversion of P9,000,000.00 which was part of the total loan, into equity. dates of their respective maturities until fully paid, broken down as follows:

On March 4, 1982, LBP informed ECO of the action taken by the former’s Trust 1. the principal amount of P4,000,000.00 with interest at 16%
Committee concerning the "Plan of Payment" which reads in part, as follows: computed from September 18, 1981;

xxx 2. the principal amount of P5,000,000.00 with interest at 16%


computed from September 21, 1981;
Please be informed that the Bank’s Trust Committee has deliberated on the
plan of payment during its meetings on November 6, 1981 and February 23, 3. the principal amount of P1,000,000.00 with interest rate at 16%
1982. The Committee arrived at a decision that you may proceed with your computed from September 28, 1981;
Plan of Payment provided Land Bank shall not participate in the undertaking
in any manner whatsoever.
4. the principal amount of P1,000,000.00 with interest at 15%
computed from October 5, 1981;
In view thereof, may we advise you to make necessary revision in the
proposed Plan of Payment and submit the same to us as soon as possible.
5. the principal amount of P2,000,000.00 with interest rate at of
(Records, p. 428)
16% computed from October 8, 1981;
6. the principal amount of P2,000,000.00 with interest rate at of THE COURT OF APPEALS GRAVELY ERRED IN AFFIRMING THE RULING OF
16% from October 23, 1981; THE LOWER COURT THE SAME NOT BEING SUPPORTED BY THE EVIDENCE
AND APPLICABLE LAWS AND JURISPRUDENCE.6
7. the principal amount of P814,000.00 with interest rate at of
16% computed from November 1, 1981; The primary issues for resolution here are (1) whether or not the corporate veil of ECO
Management Corporation should be pierced; and (2) whether or not Emmanuel C. Oñate should
be held jointly and severally liable with ECO Management Corporation for the loans incurred
8. the principal amount of P2,295,000.00 with interest rate at of
from Land Bank.
16% computed from November 6, 1981;

Petitioner contends that the personalities of Emmanuel Oñate and of ECO Management
9. the principal amount of P3,000,000.00 with interest rate at of
Corporation should be treated as one, for the particular purpose of holding respondent Oñate
16% computed from November 7, 1981;
liable for the loans incurred by corporate respondent ECO from Land Bank. According to
petitioner, the said corporation was formed ostensibly to allow Oñate to acquire loans from Land
10. the principal amount of P5,000,000.00 with interest rate at Bank which he used for his personal advantage.
16% computed from November 9, 1981;
Petitioner submits the following arguments to support its stand: (1) Respondent Oñate owns the
B. The sum of P260,000.00 as attorney’s fees; and majority of the interest holdings in respondent corporation, specifically during the crucial time
when appellees applied for and obtained the loan from LANDBANK, sometime in September to
November, 1980. (2) The acronym ECO stands for the initials of Emmanuel C. Oñate, which is
C. The costs of the suit.
the logical, sensible and concrete explanation for the name ECO, in the absence of evidence to
the contrary. (3) Respondent Oñate has always referred to himself as the debtor, not merely as
The case as against defendant Emmanuel Oñate is dismissed for an officer or a representative of respondent corporation. (4) Respondent Oñate personally paid
insufficiency of evidence. P1 Million taken from trust accounts in his name. (5) Respondent Oñate made a personal
offering to pay his personal obligation. (6) Respondent Oñate controlled respondent corporation
by simultaneously holding two (2) corporate positions, viz., as Chairman and as treasurer,
SO ORDERED. (Records, p. 608)4 beginning from the time of respondent corporation’s incorporation and continuously thereafter
without benefit of election. (7) Respondent corporation had not held any meeting of the
The Court of Appeals affirmed in toto the amended decision of the trial court.5 stockholders or of the Board of Directors, as shown by the fact that no proceeding of such
corporate activities was filed with or borne by the record of the Securities and Exchange
Commission (SEC). The only corporate records respondent corporation filed with the SEC were
On June 9, 1996, petitioner filed a motion for reconsideration, which was denied in a resolution the following: Articles of Incorporation, Treasurer’s Affidavit, Undertaking to Change Corporate
dated November 29, 1996. Hence, this present petition, assigning the following errors allegedly Name, Statement of Assets and Liabilities.7
committed by the Court of Appeals:

Private respondents, in turn, contend that Oñate’s only participation in the transaction between
A petitioner and respondent ECO was his execution of the loan agreements and promissory notes
as Chairman of the corporation’s Board of Directors. There was nothing in the loan agreement
THE COURT OF APPEALS GRAVELY ERRED IN NOT RULING THAT BASED ON nor in the promissory notes which would indicate that Oñate was binding himself jointly and
THE FACTS AS ESTABLISHED BY EVIDENCE, THERE EXISTS A SUBSTANTIAL severally with ECO. Respondents likewise deny that ECO stands for Emmanuel C. Oñate.
AND JUSTIFIABLE GROUND UPON WHICH THE LEGAL NOTION OF THE Respondents also note that Oñate is no longer a majority stockholder of ECO and that the
CORPORATE FICTION OF RESPONDENT ECO MANAGEMENT CORPORATION payment by a third person of the debt of another is allowed under the Civil Code. They also
MAY BE PIERCED. alleged that there was no fraud and/or bad faith in the transactions between them and Land
Bank. Hence, private respondents conclude, there is no legal ground to pierce the veil of
respondent corporation’s personality.8
B

At the outset, we find the matters raised by petitioner in his argumentation are mainly questions
THE COURT OF APPEALS GRAVELY ERRED IN NOT A[T]TACHING LIABILITY TO of fact which are not proper in a petition of this nature. 9 Petitioner is basically questioning the
RESPONDENT EMMANUEL C. OÑATE JOINTLY AND SEVERALLY WITH evaluation made by the Court of Appeals of the evidence submitted at the trial. The Court of
RESPONDENT ECO MANAGEMENT CORPORATION FOR THE PRINCIPAL SUM Appeals had found that petitioner’s evidence was not sufficient to justify the piercing of ECO’s
OF P26 M PLUS INTEREST THEREON. corporate personality.10 Petitioner contended otherwise. It is basic that where what is being
questioned is the sufficiency of evidence, it is a question of fact. 11 Nevertheless, even if we
C regard these matters as tendering an issue of law, we still find no reason to reverse the findings
of the Court of Appeals.
A corporation, upon coming into existence, is invested by law with a personality separate and
distinct from those persons composing it as well as from any other legal entity to which it may be
related.12 By this attribute, a stockholder may not, generally, be made to answer for acts or
liabilities of the said corporation, and vice versa.13This separate and distinct personality is,
however, merely a fiction created by law for convenience and to promote the ends of
justice.14 For this reason, it may not be used or invoked for ends subversive to the policy and
purpose behind its creation15 or which could not have been intended by law to which it owes its
being.16 This is particularly true when the fiction is used to defeat public convenience, justify
wrong, protect fraud, defend crime,17 confuse legitimate legal or judicial issues,18 perpetrate
deception or otherwise circumvent the law.19 This is likewise true where the corporate entity is
being used as an alter ego, adjunct, or business conduit for the sole benefit of the stockholders
or of another corporate entity.20 In all these cases, the notion of corporate entity will be pierced
or disregarded with reference to the particular transaction involved. 21

The burden is on petitioner to prove that the corporation and its stockholders are, in fact, using
the personality of the corporation as a means to perpetrate fraud and/or escape a liability and
responsibility demanded by law. In order to disregard the separate juridical personality of a
corporation, the wrongdoing must be clearly and convincingly established.22 In the absence of
any malice or bad faith, a stockholder or an officer of a corporation cannot be made personally
liable for corporate liabilities.23

The mere fact that Oñate owned the majority of the shares of ECO is not a ground to conclude
that Oñate and ECO is one and the same. Mere ownership by a single stockholder of all or
nearly all of the capital stock of a corporation is not by itself sufficient reason for disregarding the
fiction of separate corporate personalities.24 Neither is the fact that the name "ECO" represents
the first three letters of Oñate’s name sufficient reason to pierce the veil. Even if it did, it does not
mean that the said corporation is merely a dummy of Oñate. A corporation may assume any
name provided it is lawful. There is nothing illegal in a corporation acquiring the name or as in
this case, the initials of one of its shareholders.

That respondent corporation in this case was being used as a mere alter ego of Oñate to obtain
the loans had not been shown. Bad faith or fraud on the part of ECO and Oñate was not also
shown. As the Court of Appeals observed, if shareholders of ECO meant to defraud petitioner,
then they could have just easily absconded instead of going out of their way to propose "Plans of
Payment."25 Likewise, Oñate volunteered to pay a portion of the corporation’s debt. 26 This offer
demonstrated good faith on his part to ease the debt of the corporation of which he was a part. It
is understandable that a shareholder would want to help his corporation and in the process,
assure that his stakes in the said corporation are secured. In this case, it was established that
the P1 Million did not come solely from Oñate. It was taken from a trust account which was
owned by Oñate and other investors.27 It was likewise proved that the P1 Million was a loan
granted by Oñate and his co-depositors to alleviate the plight of ECO.28 This circumstance
should not be construed as an admission that he was really the debtor and not ECO.

In sum, we agree with the Court of Appeals’ conclusion that the evidence presented by the
petitioner does not suffice to hold respondent Oñate personally liable for the debt of co-
respondent ECO. No reversible error could be attributed to respondent court’s decision and
resolution which petitioner assails.

WHEREFORE, the petition is DENIED for lack of merit. The decision and resolution of the Court
of Appeals in CA-G.R. CV No. 43239 are AFFIRMED. Costs against petitioner.

SO ORDERED.
G.R. No. 123893 November 22, 2001 President and General Manager. And Phoenix Omega. which was not a party, to the
July 28. 1988 lease contract sought to be amended but which was a party to the
amended contract. was also represented by Padilla as Chairman of the Board of
LUISITO PADILLA and PHOENIX-OMEGA DEVELOPMENT AND MANAGEMENT
Directors of Phoenix Omega.
CORPORATION, petitioners,
vs.
THE HONORABLE COURT OF APPEALS and SUSANA REALTY, INC., respondents. PKA's building permit was later revoked due to certain violations of the National
Building Code (BP 344).
QUISUMBING, J.:
On August 24, 1989, PKA was allowed by the (Department) of Public Works and
Highway(s) to resume construction on the leased premises subject to PKA's correction
This petition for review seeks the reversal of the Court of Appeals decision1 in CA-G.R. SP No.
of the defects in the construction to conform to BP 344.
36685, refusing to set aside (1) the order dated November 29, 1994 of the Regional Trial Court
of Pasay City, Branch 113, which authorized the issuance of an alias writ of execution in
connection with Civil Case No. 7302 filed before said court; and (2) the order dated February 10, As SRI's approval of PKA's amended plans in the construction was required, PKA
1995, which denied petitioners' motion for reconsideration of the order of November 29, 1994, transmitted the same to SRI which withheld approval thereof pending PKA's correction
regarding the annulment of the alias writ of execution and cancellation of the notice of levy and of the defects in the construction.
sale dated December 16, 1994, issued pursuant to the implementation of said alias writ.
Repeated requests for approval of its amended plans not having been heeded by SRI,
The antecedent facts, as summarized by the Court of Appeals, are as follows: PKA filed at the court a quo the action at bar for rescission of contract of lease against
SRI, alleging that SRI's refusal to approve the plans without any justifiable reason
deprived it of the use of the commercial stalls, thereby incurring losses.
"On June 27, 1983, Susana Realty, Inc. (SRI), by a deed of absolute sale, sold to the
Light Rail Transit Authority (LRTA) several parcels of land located in Taft Avenue
Extension, San Rafael District, Pasay City. Under paragraph 7 of the deed of sale, SRI SRI, upon the other hand, claimed that it was PKA which violated the terms of their
reserved to itself the right of first refusal to develop and/or improve the property sold contract, alleging that PKA failed to complete within six months the construction of the
should the LRTA decide to lease and/or assign to any person the right to develop commercial stalls during which period it was not paying any rentals and that PKA
and/or improve the property. undertook the construction without first having its plans approved." 2(Underscoring in
the original.)
On November 28, 1986, the LRTA and Phoenix Omega Development and
Management Corporation (Phoenix Omega) entered into a Commercial Stall On January 7,1991, the RTC rendered its decision, as follows:
Concession Contract authorizing the latter to construct and develop commercial stalls
on a 90 sq. m. portion of the property bought from SRI. SRI opposed the agreement
"WHEREFORE, judgment is hereby rendered:
as having violated the deed of sale it entered with LRTA. A tripartite agreement was
later concluded by the parties, however, whereby SRI agreed to honor the terms of the
concession contract and to lease to Phoenix Omega its (SRI's) property (remaining 1. Declaring the rescission and termination of the Contract of Lease, as amended, and
property) adjacent to the 90 sq. m. portion subject of the concession contract. the passing in ownership of all the improvements now existing on the premises, and
ordering plaintiff to surrender possession of the leased premises to the defendant.
A contract was thus entered into on July 28, 1988 between Phoenix Omega and SRI
with LRTA whereby Phoenix Omega undertook to construct commercial stalls on the 2. Ordering plaintiff to pay to the defendant the following sums of money:
90-sq. m. property in accordance with plans and specifications prepared by the latter,
the construction to begin, however, only upon SRI's approval of such plans and
(a) P1,750,000.00 as of April 30, 1990, plus monthly rental of P200,000 per
specifications. Also on July 28, 1988, Phoenix Omega, by a deed of assignment,
assigned its right and interests over the remaining property unto its sister company, month starting in May, 1990, until plaintiff shall turn over possession of the
PKA Development and Management Corporation (PKA). Signatories to the deed of premises to the defendant, with interest at 1% per month until fully paid;
assignment were Eduardo Gatchalian in his capacity as President of Phoenix Omega,
and Luisito B. Padilla (Padilla), one of the petitioners herein, in his capacity as (b) Moral damages in the amount of P100,000.00;
President and General Manager of PKA. The development of the remaining property
having been assigned to PKA, it entered into a contract of lease with SRI likewise on
July 28, 1988: (c) Exemplary damages in the amount of P100,000.00; and

In the meantime, SRI sold part of its remaining property to a third party. An amended (d) Attorney's fees in the amount of P150,000.00; and
contract of lease was thus forged in January 1989 among SRI, PKA and Phoenix
Omega, whereby the parties agreed to substitute the already sold portion ,of SRI's (e) The cost of suit."3
remaining property with 2 parcels of land also belonging to SRI. In this amended
contract of lease, PKA was again represented by Padilla in his capacity as its
PKA appealed the RTC decision to the Court of Appeals. On October 2, 1992, the CA affirmed the same to the defendant SUSANA REALTY, INC., aside from your own fees on this
the RTC decision, decreeing as follows: execution and to likewise return this writ to this Court within 60 days from receipt
hereof with your proceeding indorsed thereon.
"WHEREFORE, with MODIFICATIONS that the award of P100,000.00 for moral
damages and P100,000.00 for exemplary damages is DELETED from the judgment But if sufficient personal properties of the plaintiff cannot be found whereof to satisfy
appealed from, the rest thereof not inconsistent herewith is AFFIRMED. No costs." 4 the amount of said judgment, you are directed to levy the real property of the plaintiff,
PKA Development and Management Corporation, Phoenix-Omega Development and
Management Corporation and Luisito B. Padilla and to sell the same or so much
PKA' S motion for reconsideration was denied by the CA in a resolution dated March 15, 1993.
thereof in the manner provided for by law for the satisfaction of the said judgment."7
PKA then filed before this Court a petition for review on certiorari, which we denied in a
resolution dated September 27, 1993. We likewise denied PKA's motion for reconsideration in a
resolution dated January 17, 1994. Alleging that the writ of execution cannot be enforced against them, herein petitioners filed with
the RTC on December 15, 1994, an omnibus motion for the reconsideration of the order of
November 29, 1994, and for annulment of the alias writ of the same date and cancellation of the
A writ of execution was issued in due course by the RTC, which reads as follows:
notice of levy and sale dated December 16, 1994. Petitioners assailed these orders as
confiscatory, since they were never parties to the case filed by PKA against SRI, and they were
"NOW THEREFORE, you are hereby commanded to cause the execution of the unable to present evidence on their behalf. The motion was denied on February 10, 1995.
aforesaid decision, ordering the plaintiff and all persons claiming under it to surrender
possession of the premises to the defendant, and that of the goods and chattels of the
Subsequently, on March 8, 1995, petitioners filed with the Court of Appeals a petition for
plaintiff you cause to be made the sum of! P1,750,000.00 plus monthly rental of
certiorari and prohibition under Rule 65 of the Rules of Court. This petition was also denied; so
P200,000.00 starting in May, 1990 until plaintiff shall turn over possession of the
was petitioners' motion for reconsideration of said denial.
premises to defendant with interest of 1% per month until fully paid, and the further
sum of PI50,000.00 as attorney's fees, and the cost of suit, together with your lawful
fees for service of this execution all in Philippine currency, and that you tender the The Court of Appeals agreed with the RTC's finding that there is evidence on record to support
same to defendant Susana Realty, Inc. aside from your own fees on this execution the RTC's conclusion that PKA and Phoenix-Omega are one and the same, or that the former is
and to likewise return this writ to this Court within sixty (60) days from receipt hereof a mere conduit of the latter. It pointed out that petitioner Padilla is both president and general
with your proceedings endorsed thereon. manager of PKA and at the same time chairman of the board of directors and controlling
stockholder of Phoenix-Omega. PKA and Phoenix-Omega also shared officers, laborers, and
offices.
But if sufficient personal property of the plaintiff cannot be found whereof to satisfy the
amount of said judgment, you are hereby directed to levy the real property of the said
plaintiff and to sell the same or so much thereof in the manner provided for by law for While aware that the dispositive portion of the RTC decision holds only PKA liable to SRI, the
the satisfaction of the said judgment."5 Court of Appeals pointed out that the intent of the RTC was clearly to hold PKA, Phoenix-
Omega, and Padilla liable, as shown in the body of the RTC decision. The rule that the
dispositive portion of a decision is the subject of execution only applies where the disposition is
Possession of the subject properties was subsequently restored to SRI, but the monetary award
clear and unequivocal, according to the CA, unlike in this case where there is uncertainty and
was left unsatisfied. Thus, on November 14, 1994, SRI filed a motion for issuance of an alias writ
ambiguity. The body of the decision may be consulted to construe the judgment in this case.
against herein petitioners, based on the trial court's observation that PKA and Phoenix-Omega
are one and the same entity. This was granted by the RTC in an order6 dated November 29,
1994, which reads: On the claim that Phoenix-Omega and Padilla were not parties to the case, the CA ruled that

"WHEREFORE, as prayed for by the defendant-judgment creditor Susana Realty, "a person not so impleaded to an action is deemed to be a party to a suit when he has
Inc., let an alias writ of execution issue against the properties, both real and the right to control the proceedings, to make defense, to adduce and cross examine
personal, of PKA Development and Management Corporation, of Phoenix-Omega witnesses, and to appeal from a decision ( 67 C.J.S. 887 cited in Albert v. University
Development Corporation, and of Luisito B. Padilla, for the enforcement of the Publishing Co., 13 SCRA 84). That petitioner Padilla is in reality the one who had and
decision dated January 7, 1991, promulgated by this Court, the same be implemented duly exercised these rights is glaringly borne by the records."8
by deputy sheriff Edilberto A. Santiago." (Underscoring by petitioners.)
Hence, this petition for review, in which petitioners allege that the CA erred:
The RTC issued an alias writ on the same day pursuant to the above order:
I. ...IN RENDERING THE DECISION AND RESOLUTION IN QUESTION... IN
"NOW THEREFORE, you are hereby commanded to cause the execution of the DEFIANCE OF LAW AND JURISPRUDENCE BY SUSTAINING THE TRIAL
aforesaid decision and that of the goods and chattels of the plaintiff, PKA COURT'S ORDER AND WRIT BOTH DATED NOVEMBER 29, 1994... FINDING
Development and Management Corporation, Phoenix- Omega, caused to be made the PETITIONERS JOINTLY AND SEVERALLY LIABLE WITH PKA, THEREBY
sum of P1,750,000.00 plus monthly rentals of P200,000.00 starting in May, 1990 with AUTHORIZING THE EXECUTION OF THE DECISION... AGAINST THEIR
interest of 1% per month, until fully paid, and the further sum of P150,000.00 as PROPERTIES, DESPITE THE ADMITTED FACT THAT --
attorney's fees; P100,000.00 moral damages and the cost of suit, together with your
lawful fees for service of this execution all in Philippine currency, and that you tender
A. PETITIONERS WERE NEVER IMPLEADED AS PARTIES IN THE CASE why, when the first writ of execution was returned unsatisfied, SRI moved for the issuance of an
BEFORE THE TRIAL COURT (CIVIL CASE NO. 7302), THEREBY alias writ of execution not only against the properties of PKA but those of petitioners as well.
CONFIRMING THE OPPRESSIVE AND CONFISCATORY NATURE OF There is no violation of petitioners' right to due process since petitioner Padilla actively
THE ORDER AND WRIT (ANNEXES N AND O); participated in the proceedings before the R TC as the responsible officer of both PKA and
Phoenix-Omega.
B. PETITIONERS COULD NOT AND DID NOT HAVE ANY OPPORTUNITY
TO ADDUCE EVIDENCE TO REFUTE THE CAUSES OF ACTIONS Private respondent also contends that the CA ruled on the necessity of construing the dispositive
ALLEGED IN RESPONDENT SRI'S COMPLAINT BEFORE THE TRIAL portion of the judgment along with its text, which petitioners allegedly accepted by not discussing
COURT (CIVIL CASE NO. 7302) THUS VIOLATING THEIR RIGHT TO the issue in their pleadings.
DUE PROCESS OF LAW.
To our mind, the main issue for our consideration is whether or not the trial court had jurisdiction
II. ...IN CONCLUSIONS REACHED IN THE DECISION AND RESOLUTION IN over petitioners, to justify the issuance of an alias writ of execution against their properties.
QUESTION... BY AFFIRMING THE ORDER AND WRIT... AS ISSUED BY THE
TRIAL COURT IN CIVIL CASE NO. 7302 WHICH EXPANDED THE SCOPE OF THE
A court acquires jurisdiction over a person through either a valid service of summons or the
WRIT HOLDING PETITIONERS SOLIDARILY LIABLE WITH PKA
person's voluntary appearance in court.11 A court must necessarily have jurisdiction over a party
NOTWITHSTANDING THAT THIS FINDING WAS NOT CONTAINED IN THE
for the latter to be bound by a court decision.
DISPOSITIVE PORTION OF THE DECISION..., IN DEFIANCE OF LAW AND
JURISPRUDENCE ON THE MATTER.
"Generally accepted is the principle that no man shall be affected by any proceeding
to which he is a stranger, and strangers to a case are not bound by judgment
III. ...IN APPLYING THE DOCTRINE OF PIERCING THE VEIL OF CORPORATE
rendered by the court. xxx",12
FICTION TO THE CASE AT BAR DESPITE THE FACT THAT THE GROUNDS FOR
ITS APPLICATION UNDER CASE LAW HAVE NOT BEEN SHOWN, THEREBY
ABROGATING PRONOUNCEMENTS OF THIS HONORABLE COURT IN In the present case, we note that the trial court never acquired jurisdiction over petitioners
NUMEROUS DECISIONS ON THE SUBJECT . through any of the modes mentioned above. Neither of the petitioners was even impleaded, as a
party to the case.13
IV. ...IN AFFIRMING THE ORDER AND WRIT... OF THE TRIAL COURT
NOTWITHSTANDING THE ABSENCE OF ANY MISTAKE, OMISSION OR Without the trial court having acquired jurisdiction over petitioners, the latter could not be bound
AMBIGUITY IN THE JANUARY 9, 1991 DECISION IN THE MAIN CASE... AS by the decision of the court. Execution can only be issued against a party and not against one
WOULD HAVE JUSTIFIED ITS MODIFICATION PURSUANT TO EXTANT who was not accorded his day in court.14 To levy upon their properties to satisfy a judgment in a
JURISPRUDENCE ON THE MATTER.9 case in which they were not even parties is not only inappropriate; it most certainly is deprivation
of property without due process of law.15 This we cannot allow.
Petitioners stress that the RTC, the CA, and this Court, in the main case (Civil Case No. 7302),
did not find them solidarily liable with PKA, and rightly so since PKA and Phoenix-Omega are The courts a quo ruled that petitioner Padilla, in particular, had his day in court. As general
two different entities. Phoenix-Omega's only participation in the properties subject of the main manager of PKA, he actively participated in the case in the trial court. He "ha(d) the right to
case was as the construction company that would develop the properties on behalf of PKA. control the proceedings, to make defense, to adduce and cross examine witnesses, and to
Phoenix-Omega was involved in the amended lease agreement between SRI and PKA only to appeal from a decision."16 Therefore, Padilla and Phoenix-Omega, of which Padilla is chairman
the extent that it had to apply the terms of the tripartite agreement (among LRTA, SRI, and of the board, could not now argue that they did not have the opportunity to present their case in
Phoenix-Omega) to the development of the LRTA-owned property situated in front of the lots court, according to private respondent.
leased to PKA by SRI.10 Petitioners argue that the amended lease contract was, in reality, only
between SRI and PKA.
To begin with, it is clear that Padilla participated in the proceedings below as general manager of
PKA and not in any other capacity. The fact that at the same time he was the chairman of the
Petitioners protest the piercing of the veil of corporate fiction between themselves and PKA. board of Phoenix-Omega cannot, by any stretch of reasoning, equate to participation by
They contend, citing Filmerco Commercial Co., Inc. v. IAC, No. L-70661, 149 SCRA 193 (1987), Phoenix- Omega in the same proceedings. We again stress that Phoenix-Omega was not a
that the court must first acquire jurisdiction over the corporation attempting to misuse the party to the case and so could not have taken part therein.
corporate vehicle to shield the commission of a fraud.
Private respondent, however, insists that the trial court had pierced the veil of corporate fiction
Petitioners contend that the finding by the trial court as regards the single personality of PKA protecting petitioners, and this justifies execution against their properties.
and Phoenix-Omega was made only to refute PKA's claim that it was not liable for constructions
made by Phoenix- Omega outside the leased areas.
The general rule is that a corporation is clothed with a personality separate and distinct from the
persons composing it. It may not be held liable for the obligations of the persons composing it,
On the other hand, private respondent argues that there is no error in the issuance of the alias and neither can its stockholders be held liable for its obligations.17
writ of execution against the properties of petitioners since the trial court, the CA, and this Court
had all ruled that petitioners and PKA are in reality one and the same entity. This is the reason
This veil of corporate fiction may only be disregarded in cases where the corporate vehicle is
being used to defeat public convenience, justify wrong, protect fraud, or defend crime. 18 PKA
and Phoenix-Omega are admittedly sister companies, and may be sharing personnel and
resources, but we find in the present case no allegation, much less positive proof, that their
separate corporate personalities are being used to defeat public convenience, justify wrong,
protect fraud, or defend crime. "For the separate juridical personality of a corporation to be
disregarded, the wrongdoing must be clearly and convincingly established. It cannot be
presumed."19 We find no reason to justify piercing the corporate veil in this instance.

We understand private respondent's frustration at not being able to have the monetary award in
their favor satisfied. But given the circumstances of this case, public respondent cannot order
the seizure of petitioners' properties without violating their constitutionally enshrined right to due
process, merely to compensate private respondent.

WHEREFORE, the instant petition is GRANTED. The assailed decision and resolution of the
Court of Appeals in CA-G.R. SP No. 36685 are SET ASIDE, and the order of the trial court
dated November 29, 1994 and the alias writ of execution issued on the same date in connection
with Civil Case No. 7302, are declared NULL and VOID.

Costs against private respondent.

SO ORDERED.
G.R. No. L-17618 August 31, 1964 Company contended otherwise — that is, the transaction subject to tax is the sale from Jackbilt
to Norton.
COMMISSIONER OF INTERNAL REVENUE, petitioner,
vs. Wherefore, the parties respectfully pray that the foregoing stipulation of facts be admitted and
NORTON and HARRISON COMPANY, respondent. approved by this Honorable Court, without prejudice to the parties adducing other evidence to
prove their case not covered by this stipulation of facts.
PAREDES, J.:
The majority of the Tax Court, in relieving Norton & Harrison of liability under the assessment,
made the following observations:
This is an appeal interposed by the Commissioner of Internal Revenue against the following
judgment of the Court of Tax Appeals:
The law applicable to the case is Section 186 of the National Internal Revenue Code
which imposes a percentage tax of 7% on every original sale of goods, wares or
IN VIEW OF THE FOREGOING, we find no legal basis to support the assessment in
merchandise, such tax to be based on the gross selling price of such goods, wares or
question against petitioner. If at all, the assessment should have been directed against
merchandise. The term "original sale" has been defined as the first sale by every
JACKBILT, the manufacturer. Accordingly, the decision appealed from is reversed,
manufacturer, producer or importer. (Sec. 5, Com. Act No. 503.) Subsequent sales by
and the surety bond filed to guarantee payment of said assessment is ordered
persons other than the manufacturer, producer or importer are not subject to the sales
cancelled. No pronouncement as to costs.
tax.

Norton and Harrison is a corporation organized in 1911, (1) to buy and sell at wholesale and
If JACKBILT actually sold concrete blocks manufactured by it to petitioner under the
retail, all kinds of goods, wares, and merchandise; (2) to act as agents of manufacturers in the
distributorship or agency agreement of July 27, 1948, such sales constituted the
United States and foreign countries; and (3) to carry on and conduct a general wholesale and
original sales which are taxable under Section 186 of the Revenue Code, while the
retail mercantile establishment in the Philippines. Jackbilt is, likewise, a corporation organized on
sales made to the public by petitioner are subsequent sales which are not taxable. But
February 16, 1948 primarily for the purpose of making, producing and manufacturing concrete
it appears to us that there was no such sale by JACKBILT to petitioner. Petitioner
blocks. Under date of July 27, 1948. Norton and Jackbilt entered into an agreement whereby
merely acted as agent for JACKBILT in the marketing of its products. This is shown by
Norton was made the sole and exclusive distributor of concrete blocks manufactured by Jackbilt.
the fact that petitioner merely accepted orders from the public for the purchase of
Pursuant to this agreement, whenever an order for concrete blocks was received by the Norton
JACKBILT blocks. The purchase orders were transmitted to JACKBILT which
& Harrison Co. from a customer, the order was transmitted to Jackbilt which delivered the
delivered the blocks to the purchaser directly. There was no instance in which the
merchandise direct to the customer. Payment for the goods is, however, made to Norton, which
blocks ordered by the purchasers were delivered to the petitioner. Petitioner never
in turn pays Jackbilt the amount charged the customer less a certain amount, as its
purchased concrete blocks from JACKBILT so that it never acquired ownership of
compensation or profit. To exemplify the sales procedures adopted by the Norton and Jackbilt,
such concrete blocks. This being so, petitioner could not have sold JACKBILT blocks
the following may be cited. In the case of the sale of 420 pieces of concrete blocks to the
for its own account. It did so merely as agent of JACKBILT. The distributorship
American Builders on April 1, 1952, the purchaser paid to Norton the sum of P189.00 the
agreement of July 27, 1948, is denominated by the parties themselves as an "agency
purchase price. Out of this amount Norton paid Jackbilt P168.00, the difference obviously being
for marketing" JACKBILT products. ... .
its compensation. As per records of Jackbilt, the transaction was considered a sale to Norton. It
was under this procedure that the sale of concrete blocks manufactured by Jackbilt was
conducted until May 1, 1953, when the agency agreement was terminated and a management xxx xxx xxx
agreement between the parties was entered into. The management agreement provided that
Norton would sell concrete blocks for Jackbilt, for a fixed monthly fee of P2,000.00, which was
Therefore, the taxable selling price of JACKBILT blocks under the aforesaid
later increased to P5,000.00.
agreement is the price charged to the public and not the amount billed by JACKBILT
to petitioner. The deficiency sales tax should have been assessed against JACKBILT
During the existence of the distribution or agency agreement, or on June 10, 1949, Norton & and not against petitioner which merely acted as the former's agent.
Harrison acquired by purchase all the outstanding shares of stock of Jackbilt. Apparently, due to
this transaction, the Commissioner of Internal Revenue, after conducting an investigation,
xxx xxx xxx
assessed the respondent Norton & Harrison for deficiency sales tax and surcharges in the
amount of P32,662.90, making as basis thereof the sales of Norton to the Public. In other words,
the Commissioner considered the sale of Norton to the public as the original sale and not the Presiding Judge Nable of the same Court expressed a partial dissent, stating:
transaction from Jackbilt. The period covered by the assessment was from July 1, 1949 to May
31, 1953. As Norton and Harrison did not conform with the assessment, the matter was brought
to the Court of Tax Appeals. Upon the aforestated circumstances, which disclose Norton's control over and
direction of Jackbilt's affairs, the corporate personality of Jackbilt should be
disregarded, and the transactions between these two corporations relative to the
The Commissioner of Internal Revenue contends that since Jackbilt was owned and controlled concrete blocks should be ignored in determining the percentage tax for which Norton
by Norton & Harrison, the corporate personality of the former (Jackbilt) should be disregarded for is liable. Consequently, the percentage tax should be computed on the basis of the
sales tax purposes, and the sale of Jackbilt blocks by petitioner to the public must be considered sales of Jackbilt blocks to the public.
as the original sales from which the sales tax should be computed. The Norton & Harrison
The majority opinion is now before Us on appeal by the Commissioner of Internal Revenue, on accounts are distinct and different; they have separate income tax returns, separate balance
four (4) assigned errors, all of which pose the following propositions: (1) whether the acquisition sheets and profit and loss statements. These explanations notwithstanding an over-all appraisal
of all the stocks of the Jackbilt by the Norton & Harrison Co., merged the two corporations into a of the circumstances presented by the facts of the case, yields to the conclusion that the Jackbilt
single corporation; (2) whether the basis of the computation of the deficiency sales tax should be is merely an adjunct, business conduit or alter ego, of Norton and Harrison and that the fiction of
the sale of the blocks to the public and not to Norton. corporate entities, separate and distinct from each, should be disregarded. This is a case where
the doctrine of piercing the veil of corporate fiction, should be made to apply. In the case
of Liddell & Co. Inc. v. Coll. of Int. Rev., supra, it was held:
It has been settled that the ownership of all the stocks of a corporation by another corporation
does not necessarily breed an identity of corporate interest between the two companies and be
considered as a sufficient ground for disregarding the distinct personalities (Liddell & Co., Inc. v. There are quite a series of conspicuous circumstances that militates against the
Coll. of Int. Rev. L-9687, June 30, 1961). However, in the case at bar, we find sufficient grounds separate and distinct personality of Liddell Motors Inc., from Liddell & Co. We notice
to support the theory that the separate identities of the two companies should be disregarded. that the bulk of the business of Liddell & Co. was channel Red through Liddell Motors,
Among these circumstances, which we find not successfully refuted by appellee Norton are: (a) Inc. On the other hand, Liddell Motors Inc. pursued no activities except to secure cars,
Norton and Harrison owned all the outstanding stocks of Jackbilt; of the 15,000 authorized trucks, and spare parts from Liddell & Co., Inc. and then sell them to the general
shares of Jackbilt on March 31, 1958, 14,993 shares belonged to Norton and Harrison and one public. These sales of vehicles by Liddell & Co, to Liddell Motors. Inc. for the most part
each to seven others; (b) Norton constituted Jackbilt's board of directors in such a way as to were shown to have taken place on the same day that Liddell Motors, Inc. sold such
enable it to actually direct and manage the other's affairs by making the same officers of the vehicles to the public. We may even say that the cars and trucks merely touched the
board for both companies. For instance, James E. Norton is the President, Treasurer, Director hands of Liddell Motors, Inc. as a matter of formality.
and Stockholder of Norton. He also occupies the same positions in Jackbilt corporation, the only
change being, in the Jackbilt, he is merely a nominal stockholder. The same is true with Mr.
xxx xxx xxx
Jordan, F. M. Domingo, Mr. Mantaring, Gilbert Golden and Gerardo Garcia, while they are
merely employees of the North they are Directors and nominal stockholders of the Jackbilt (c)
Norton financed the operations of the Jackbilt, and this is shown by the fact that the loans Accordingly, the mere fact that Liddell & Co. and Liddell Motors, Inc. are corporations
obtained from the RFC and Bank of America were used in the expansion program of Jackbilt, to owned and controlled by Frank Liddell directly or indirectly is not by itself sufficient to
pay advances for the purchase of equipment, materials rations and salaries of employees of justify the disregard of the separate corporate identity of one from the other. There is
Jackbilt and other sundry expenses. There was no limit to the advances given to Jackbilt so however, in this instant case, a peculiar sequence of the organization and activities of
much so that as of May 31, 1956, the unpaid advances amounted to P757,652.45, which were Liddell Motors, Inc.
not paid in cash by Jackbilt, but was offset by shares of stock issued to Norton, the absolute and
sole owner of Jackbilt; (d) Norton treats Jackbilt employees as its own. Evidence shows that
Norton paid the salaries of Jackbilt employees and gave the same privileges as Norton As opined in the case of Gregory v. Helvering "the legal right of a tax payer to
employees, an indication that Jackbilt employees were also Norton's employees. Furthermore decrease the amount of what otherwise would be his taxes, or altogether avoid them,
service rendered in any one of the two companies were taken into account for purposes of by means which the law permits, cannot be doubted". But as held in another case,
promotion; (e) Compensation given to board members of Jackbilt, indicate that Jackbilt is merely "where a corporation is a dummy, is unreal or a sham and serves no business
a department of Norton. The income tax return of Norton for 1954 shows that as President and purpose and is intended only as a blind, the corporate form may be ignored for the law
cannot countenance a form that is bald and a mischievous fictions".
Treasurer of Norton and Jackbilt, he received from Norton P56,929.95, but received from
Jackbilt the measly amount of P150.00, a circumstance which points out that remuneration of
purported officials of Jackbilt are deemed included in the salaries they received from Norton. The ... a taxpayer may gain advantage of doing business thru a corporation if he pleases,
same is true in the case of Eduardo Garcia, an employee of Norton but a member of the Board but the revenue officers in proper cases, may disregard the separate corporate entity
of Jackbilt. His Income tax return for 1956 reveals that he received from Norton in salaries and where it serves but as a shield for tax evasion and treat the person who actually may
bonuses P4,220.00, but received from Jackbilt, by way of entertainment, representation, take benefits of the transactions as the person accordingly taxable.
travelling and transportation allowances P3,000.00. However, in the withholding statement (Exh.
28-A), it was shown that the total of P4,200.00 and P3,000.00 (P7,220.00) was received by
Garcia from Norton, thus portraying the oneness of the two companies. The Income Tax Returns ... to allow a taxpayer to deny tax liability on the ground that the sales were made
of Albert Golden and Dioscoro Ramos both employees of Norton but board members of Jackbilt, through another and distinct corporation when it is proved that the latter is virtually
also disclose the game method of payment of compensation and allowances. The offices of owned by the former or that they are practically one and the same is to sanction a
Norton and Jackbilt are located in the same compound. Payments were effected by Norton of circumvention of our tax laws. (and cases cited therein.)
accounts for Jackbilt and vice versa. Payments were also made to Norton of accounts due or
payable to Jackbilt and vice versa. In the case of Yutivo Sons Hardware Co. v. Court of Tax Appeals, L-13203, Jan. 28, 1961, this
Court made a similar ruling where the circumstances of unity of corporate identities have been
Norton and Harrison, while not denying the presence of the set up stated above, tried to explain shown and which are identical to those obtaining in the case under consideration. Therein, this
that the control over the affairs of Jackbilt was not made in order to evade payment of taxes; that Court said:
the loans obtained by it which were given to Jackbilt, were necessary for the expansion of its
business in the manufacture of concrete blocks, which would ultimately benefit both We are, however, inclined to agree with the court below that SM was actually owned
corporations; that the transactions and practices just mentioned, are not unusual and and controlled by petitioner as to make it a mere subsidiary or branch of the latter
extraordinary, but pursued in the regular course of business and trade; that there could be no created for the purpose of selling the vehicles at retail (here concrete blocks) ... .
confusion in the present set up of the two corporations, because they have separate Boards,
their cash assets are entirely and strictly separate; cashiers and official receipts and bank
It may not be amiss to state in this connection, the advantages to Norton in maintaining a
semblance of separate entities. If the income of Norton should be considered separate from the
income of Jackbilt, then each would declare such earning separately for income tax purposes
and thus pay lesser income tax. The combined taxable Norton-Jackbilt income would subject
Norton to a higher tax. Based upon the 1954-1955 income tax return of Norton and Jackbilt
(Exhs. 7 & 8), and assuming that both of them are operating on the same fiscal basis and their
returns are accurate, we would have the following result: Jackbilt declared a taxable net income
of P161,202.31 in which the income tax due was computed at P37,137.00 (Exh. 8); whereas
Norton declared as taxable, a net income of P120,101.59, on which the income tax due was
computed at P25,628.00. The total of these liabilities is P50,764.84. On the other hand, if the net
taxable earnings of both corporations are combined, during the same taxable year, the tax due
on their total which is P281,303.90 would be P70,764.00. So that, even on the question of
income tax alone, it would be to the advantages of Norton that the corporations should be
regarded as separate entities.

WHEREFORE, the decision appealed from should be as it is hereby reversed and another
entered making the appellee Norton & Harrison liable for the deficiency sales taxes assessed
against it by the appellant Commissioner of Internal Revenue, plus 25% surcharge thereon.
Costs against appellee Norton & Harrison.
G.R. No. 115849 January 24, 1996 2. Ordering defendant Producers Bank of the Philippines, upon finality of this decision
and receipt from the plaintiffs the amount of P5.5 Million, to execute in favor of said
plaintiffs a deed of absolute sale over the aforementioned six (6) parcels of land, and
FIRST PHILIPPINE INTERNATIONAL BANK (Formerly Producers Bank of the Philippines)
to immediately deliver to the plaintiffs the owner's copies of T.C.T. Nos. T-106932 to
and MERCURIO RIVERA, petitioners,
T- 106937, inclusive, for purposes of registration of the same deed and transfer of the
vs.
six (6) titles in the names of the plaintiffs;
COURT OF APPEALS, CARLOS EJERCITO, in substitution of DEMETRIO DEMETRIA, and
JOSE JANOLO,respondents.
3. Ordering the defendants, jointly and severally, to pay plaintiffs Jose A. Janolo and
Demetrio Demetria the sums of P200,000.00 each in moral damages;
DECISION

4. Ordering the defendants, jointly and severally, to pay plaintiffs the sum of
PANGANIBAN, J.:
P100,000.00 as exemplary damages ;

In the absence of a formal deed of sale, may commitments given by bank officers in an
5. Ordering the defendants, jointly and severally, to pay the plaintiffs the amount of
exchange of letters and/or in a meeting with the buyers constitute a perfected and enforceable
P400,000.00 for and by way of attorney's fees;
contract of sale over 101 hectares of land in Sta. Rosa, Laguna? Does the doctrine of "apparent
authority" apply in this case? If so, may the Central Bank-appointed conservator of Producers
Bank (now First Philippine International Bank) repudiate such "apparent authority" after said 6. Ordering the defendants to pay the plaintiffs, jointly and severally, actual and
contract has been deemed perfected? During the pendency of a suit for specific performance, moderate damages in the amount of P20,000.00;
does the filing of a "derivative suit" by the majority shareholders and directors of the distressed
bank to prevent the enforcement or implementation of the sale violate the ban against forum-
With costs against the defendants.
shopping?

After the parties filed their comment, reply, rejoinder, sur-rejoinder and reply to sur-rejoinder, the
Simply stated, these are the major questions brought before this Court in the instant Petition for
petition was given due course in a Resolution dated January 18, 1995. Thence, the parties filed
review on certiorariunder Rule 45 of the Rules of Court, to set aside the Decision promulgated
their respective memoranda and reply memoranda. The First Division transferred this case to
January 14, 1994 of the respondent Court of Appeals1 in CA-G.R CV No. 35756 and the
the Third Division per resolution dated October 23, 1995. After carefully deliberating on the
Resolution promulgated June 14, 1994 denying the motion for reconsideration. The dispositive
aforesaid submissions, the Court assigned the case to the undersigned ponentefor the writing of
portion of the said Decision reads:
this Decision.

WHEREFORE, the decision of the lower court is MODIFIED by the elimination of the
The Parties
damages awarded under paragraphs 3, 4 and 6 of its dispositive portion and the
reduction of the award in paragraph 5 thereof to P75,000.00, to be assessed against
defendant bank. In all other aspects, said decision is hereby AFFIRMED. Petitioner First Philippine International Bank (formerly Producers Bank of the Philippines;
petitioner Bank, for brevity) is a banking institution organized and existing under the laws of the
Republic of the Philippines. Petitioner Mercurio Rivera (petitioner Rivera, for brevity) is of legal
All references to the original plaintiffs in the decision and its dispositive portion are
age and was, at all times material to this case, Head-Manager of the Property Management
deemed, herein and hereafter, to legally refer to the plaintiff-appellee Carlos C.
Department of the petitioner Bank.
Ejercito.

Respondent Carlos Ejercito (respondent Ejercito, for brevity) is of legal age and is the assignee
Costs against appellant bank.
of original plaintiffs-appellees Demetrio Demetria and Jose Janolo.

The dispositive portion of the trial court's2 decision dated July 10, 1991, on the other hand, is as
Respondent Court of Appeals is the court which issued the Decision and Resolution sought to
follows:
be set aside through this petition.

WHEREFORE, premises considered, judgment is hereby rendered in favor of the


The Facts
plaintiffs and against the defendants as follows:

The facts of this case are summarized in the respondent Court's Decision3 as follows:
1. Declaring the existence of a perfected contract to buy and sell over the six (6)
parcels of land situated at Don Jose, Sta. Rosa, Laguna with an area of 101 hectares,
more or less, covered by and embraced in Transfer Certificates of Title Nos. T-106932 (1) In the course of its banking operations, the defendant Producer Bank of the
to T-106937, inclusive, of the Land Records of Laguna, between the plaintiffs as Philippines acquired six parcels of land with a total area of 101 hectares located at
buyers and the defendant Producers Bank for an agreed price of Five and One Half Don Jose, Sta. Rose, Laguna, and covered by Transfer Certificates of Title Nos. T-
Million (P5,500,000.00) Pesos; 106932 to T-106937. The property used to be owned by BYME Investment and
Development Corporation which had them mortgaged with the bank as collateral for a
loan. The original plaintiffs, Demetrio Demetria and Jose O. Janolo, wanted to Attention: JOSE O. JANOLO
purchase the property and thus initiated negotiations for that purpose.
Dear Sir:
(2) In the early part of August 1987 said plaintiffs, upon the suggestion of BYME
investment's legal counsel, Jose Fajardo, met with defendant Mercurio Rivera,
Thank you for your letter-offer to buy our six (6) parcels of acquired lots at Sta. Rosa,
Manager of the Property Management Department of the defendant bank. The
Laguna (formerly owned by Byme Industrial Corp.). Please be informed however that
meeting was held pursuant to plaintiffs' plan to buy the property (TSN of Jan. 16,
the bank's counter-offer is at P5.5 million for more than 101 hectares on lot basis.
1990, pp. 7-10). After the meeting, plaintiff Janolo, following the advice of defendant
Rivera, made a formal purchase offer to the bank through a letter dated August 30,
1987 (Exh. "B"), as follows: We shall be very glad to hear your position on the on the matter.

August 30, 1987 Best regards.

(4) On September 17, 1987, plaintiff Janolo, responding to Rivera's aforequoted reply,
The Producers Bank of the Philippines
wrote (Exh. "D"):
Makati, Metro Manila

Attn. Mr. Mercurio Q. Rivera September 17, 1987


Manager, Property Management Dept.
Producers Bank
Gentleman: Paseo de Roxas
Makati, Metro Manila
I have the honor to submit my formal offer to purchase your properties covered by
titles listed hereunder located at Sta. Rosa, Laguna, with a total area of 101 hectares, Attention: Mr. Mercurio Rivera
more or less.
Gentlemen:
TCT NO. AREA
T-106932 113,580 sq. m. In reply to your letter regarding my proposal to purchase your 101-hectare lot located
at Sta. Rosa, Laguna, I would like to amend my previous offer and I now propose to
T-106933 70,899 sq. m. buy the said lot at P4.250 million in CASH..
T-106934 52,246 sq. m.
T-106935 96,768 sq. m. Hoping that this proposal meets your satisfaction.
T-106936 187,114 sq. m.
(5) There was no reply to Janolo's foregoing letter of September 17, 1987. What took
T-106937 481,481 sq. m. place was a meeting on September 28, 1987 between the plaintiffs and Luis Co, the
Senior Vice-President of defendant bank. Rivera as well as Fajardo, the BYME lawyer,
My offer is for PESOS: THREE MILLION FIVE HUNDRED THOUSAND attended the meeting. Two days later, or on September 30, 1987, plaintiff Janolo sent
(P3,500,000.00) PESOS, in cash. to the bank, through Rivera, the following letter (Exh. "E"):

Kindly contact me at Telephone Number 921-1344. The Producers Bank of the Philippines
Paseo de Roxas, Makati
Metro Manila
(3) On September 1, 1987, defendant Rivera made on behalf of the bank a formal
reply by letter which is hereunder quoted (Exh. "C"):
Attention: Mr. Mercurio Rivera

September 1, 1987 Re: 101 Hectares of Land


in Sta. Rosa, Laguna
JP M-P GUTIERREZ ENTERPRISES
142 Charisma St., Doña Andres II Gentlemen:
Rosario, Pasig, Metro Manila
Pursuant to our discussion last 28 September 1987, we are pleased to inform you that purchase, you now refuse to honor your commitment. Instead, you have advertised for
we are accepting your offer for us to purchase the property at Sta. Rosa, Laguna, sale the same lot to others.
formerly owned by Byme Investment, for a total price of PESOS: FIVE MILLION FIVE
HUNDRED THOUSAND (P5,500,000.00).
In behalf of our client, therefore, we are making this formal demand upon you to
consummate and execute the necessary actions/documentation within three (3) days
Thank you. from your receipt hereof. We are ready to remit the agreed amount of P5.5 million at
your advice. Otherwise, we shall be constrained to file the necessary court action to
protect the interest of our client.
(6) On October 12, 1987, the conservator of the bank (which has been placed under
conservatorship by the Central Bank since 1984) was replaced by an Acting
Conservator in the person of defendant Leonida T. Encarnacion. On November 4, We trust that you will be guided accordingly.
1987, defendant Rivera wrote plaintiff Demetria the following letter (Exh. "F"):
(8) Defendant bank, through defendant Rivera, acknowledged receipt of the foregoing
Attention: Atty. Demetrio Demetria letter and stated, in its communication of December 2, 1987 (Exh. "I"), that said letter
has been "referred . . . to the office of our Conservator for proper disposition"
However, no response came from the Acting Conservator. On December 14, 1987,
Dear Sir:
the plaintiffs made a second tender of payment (Exh. "L" and "L-1"), this time through
the Acting Conservator, defendant Encarnacion. Plaintiffs' letter reads:
Your proposal to buy the properties the bank foreclosed from Byme investment Corp.
located at Sta. Rosa, Laguna is under study yet as of this time by the newly created
PRODUCERS BANK OF
committee for submission to the newly designated Acting Conservator of the bank.
THE PHILIPPINES
Paseo de Roxas,
For your information. Makati, Metro Manila

(7) What thereafter transpired was a series of demands by the plaintiffs for compliance Attn.: Atty. NIDA ENCARNACION
by the bank with what plaintiff considered as a perfected contract of sale, which Central Bank Conservator
demands were in one form or another refused by the bank. As detailed by the trial
court in its decision, on November 17, 1987, plaintiffs through a letter to defendant
We are sending you herewith, in - behalf of our client, Mr. JOSE O. JANOLO, MBTC
Rivera (Exhibit "G") tendered payment of the amount of P5.5 million "pursuant to (our)
Check No. 258387 in the amount of P5.5 million as our agreed purchase price of the
perfected sale agreement." Defendants refused to receive both the payment and the
101-hectare lot covered by TCT Nos. 106932, 106933, 106934, 106935, 106936 and
letter. Instead, the parcels of land involved in the transaction were advertised by the
106937 and registered under Producers Bank.
bank for sale to any interested buyer (Exh, "H" and "H-1"). Plaintiffs demanded the
execution by the bank of the documents on what was considered as a "perfected
agreement." Thus: This is in connection with the perfected agreement consequent from your offer of P5.5
Million as the purchase price of the said lots. Please inform us of the date of
documentation of the sale immediately.
Mr. Mercurio Rivera
Manager, Producers Bank
Paseo de Roxas, Makati Kindly acknowledge receipt of our payment.
Metro Manila
(9) The foregoing letter drew no response for more than four months. Then, on May 3,
Dear Mr. Rivera: 1988, plaintiff, through counsel, made a final demand for compliance by the bank with
its obligations under the considered perfected contract of sale (Exhibit "N"). As
recounted by the trial court (Original Record, p. 656), in a reply letter dated May 12,
This is in connection with the offer of our client, Mr. Jose O. Janolo, to purchase your
1988 (Annex "4" of defendant's answer to amended complaint), the defendants
101-hectare lot located in Sta. Rosa, Laguna, and which are covered by TCT No. T-
through Acting Conservator Encarnacion repudiated the authority of defendant Rivera
106932 to 106937.
and claimed that his dealings with the plaintiffs, particularly his counter-offer of P5.5
Million are unauthorized or illegal. On that basis, the defendants justified the refusal of
From the documents at hand, it appears that your counter-offer dated September 1, the tenders of payment and the non-compliance with the obligations under what the
1987 of this same lot in the amount of P5.5 million was accepted by our client thru a plaintiffs considered to be a perfected contract of sale.
letter dated September 30, 1987 and was received by you on October 5, 1987.
(10) On May 16, 1988, plaintiffs filed a suit for specific performance with damages
In view of the above circumstances, we believe that an agreement has been against the bank, its Manager Rivers and Acting Conservator Encarnacion. The basis
perfected. We were also informed that despite repeated follow-up to consummate the of the suit was that the transaction had with the bank resulted in a perfected contract
of sale, The defendants took the position that there was no such perfected sale
because the defendant Rivera is not authorized to sell the property, and that there was The findings and conclusions of the Court of Appeals do not conform to the evidence
no meeting of the minds as to the price. on record.

On March 14, 1991, Henry L. Co (the brother of Luis Co), through counsel Sycip On the other hand, petitioners prayed for dismissal of the instant suit on the ground8 that:
Salazar Hernandez and Gatmaitan, filed a motion to intervene in the trial court,
alleging that as owner of 80% of the Bank's outstanding shares of stock, he had a
I.
substantial interest in resisting the complaint. On July 8, 1991, the trial court issued an
order denying the motion to intervene on the ground that it was filed after trial had
already been concluded. It also denied a motion for reconsideration filed thereafter. Petitioners have engaged in forum shopping.
From the trial court's decision, the Bank, petitioner Rivera and conservator
Encarnacion appealed to the Court of Appeals which subsequently affirmed with
modification the said judgment. Henry Co did not appeal the denial of his motion for II.
intervention.
The factual findings and conclusions of the Court of Appeals are supported by the
In the course of the proceedings in the respondent Court, Carlos Ejercito was substituted in evidence on record and may no longer be questioned in this case.
place of Demetria and Janolo, in view of the assignment of the latters' rights in the matter in
litigation to said private respondent. III.

On July 11, 1992, during the pendency of the proceedings in the Court of Appeals, Henry Co The Court of Appeals correctly held that there was a perfected contract between
and several other stockholders of the Bank, through counsel Angara Abello Concepcion Regala Demetria and Janolo (substituted by; respondent Ejercito) and the bank.
and Cruz, filed an action (hereafter, the "Second Case") — purportedly a "derivative suit" — with
the Regional Trial Court of Makati, Branch 134, docketed as Civil Case No. 92-1606, against
Encarnacion, Demetria and Janolo "to declare any perfected sale of the property as IV.
unenforceable and to stop Ejercito from enforcing or implementing the sale" 4 In his answer,
Janolo argued that the Second Case was barred by litis pendentia by virtue of the case then The Court of Appeals has correctly held that the conservator, apart from being
pending in the Court of Appeals. During the pre-trial conference in the Second Case, plaintiffs estopped from repudiating the agency and the contract, has no authority to revoke the
filed a Motion for Leave of Court to Dismiss the Case Without Prejudice. "Private respondent contract of sale.
opposed this motion on the ground, among others, that plaintiff's act of forum shopping justifies
the dismissal of both cases, with prejudice."5 Private respondent, in his memorandum, averred
that this motion is still pending in the Makati RTC. The Issues

In their Petition6 and Memorandum7 , petitioners summarized their position as follows: From the foregoing positions of the parties, the issues in this case may be summed up as
follows:

I.
1) Was there forum-shopping on the part of petitioner Bank?

The Court of Appeals erred in declaring that a contract of sale was perfected between
Ejercito (in substitution of Demetria and Janolo) and the bank. 2) Was there a perfected contract of sale between the parties?

II. 3) Assuming there was, was the said contract enforceable under the statute of frauds?

The Court of Appeals erred in declaring the existence of an enforceable contract of 4) Did the bank conservator have the unilateral power to repudiate the authority of the
sale between the parties. bank officers and/or to revoke the said contract?

III. 5) Did the respondent Court commit any reversible error in its findings of facts?

The Court of Appeals erred in declaring that the conservator does not have the power The First Issue: Was There Forum-Shopping?
to overrule or revoke acts of previous management.
In order to prevent the vexations of multiple petitions and actions, the Supreme Court
IV. promulgated Revised Circular No. 28-91 requiring that a party "must certify under oath . . . [that]
(a) he has not (t)heretofore commenced any other action or proceeding involving the same
issues in the Supreme Court, the Court of Appeals, or any other tribunal or agency; (b) to the
best of his knowledge, no such action or proceeding is pending" in said courts or agencies. A
violation of the said circular entails sanctions that include the summary dismissal of the multiple where the plaintiff or any of the plaintiffs resides, at the election of the plaintiff" (Rule 4, Sec, 2
petitions or complaints. To be sure, petitioners have included a [b]). As to remedies, aggrieved parties, for example, are given a choice of pursuing civil liabilities
VERIFICATION/CERTIFICATION in their Petition stating "for the record(,) the pendency of Civil independently of the criminal, arising from the same set of facts. A passenger of a public utility
Case No. 92-1606 before the Regional Trial Court of Makati, Branch 134, involving vehicle involved in a vehicular accident may sue on culpa contractual, culpa aquiliana or culpa
a derivative suit filed by stockholders of petitioner Bank against the conservator and other criminal — each remedy being available independently of the others — although he cannot
defendants but which is the subject of a pending Motion to Dismiss Without Prejudice. 9 recover more than once.

Private respondent Ejercito vigorously argues that in spite of this verification, petitioners are In either of these situations (choice of venue or choice of remedy), the litigant
guilty of actual forum shopping because the instant petition pending before this Court involves actually shops for a forum of his action, This was the original concept of the term
"identical parties or interests represented, rights asserted and reliefs sought (as that) currently forum shopping.
pending before the Regional Trial Court, Makati Branch 134 in the Second Case. In fact, the
issues in the two cases are so interwined that a judgement or resolution in either case will
Eventually, however, instead of actually making a choice of the forum of their actions,
constitute res judicata in the other." 10
litigants, through the encouragement of their lawyers, file their actions in all available
courts, or invoke all relevant remedies simultaneously. This practice had not only
On the other hand, petitioners explain 11 that there is no forum-shopping because: resulted to (sic) conflicting adjudications among different courts and consequent
confusion enimical (sic) to an orderly administration of justice. It had created extreme
inconvenience to some of the parties to the action.
1) In the earlier or "First Case" from which this proceeding arose, the Bank was
impleaded as a defendant, whereas in the "Second Case" (assuming the Bank is the
real party in interest in a derivative suit), it wasplaintiff; Thus, "forum shopping" had acquired a different concept — which is unethical
professional legal practice. And this necessitated or had given rise to the formulation
of rules and canons discouraging or altogether prohibiting the practice. 15
2) "The derivative suit is not properly a suit for and in behalf of the corporation under
the circumstances";
What therefore originally started both in conflicts of laws and in our domestic law as a legitimate
device for solving problems has been abused and mis-used to assure scheming litigants of
3) Although the CERTIFICATION/VERIFICATION (supra) signed by the Bank
dubious reliefs.
president and attached to the Petition identifies the action as a "derivative suit," it
"does not mean that it is one" and "(t)hat is a legal question for the courts to decide";
To avoid or minimize this unethical practice of subverting justice, the Supreme Court, as already
mentioned, promulgated Circular 28-91. And even before that, the Court had prescribed it in the
4) Petitioners did not hide the Second Case at they mentioned it in the said
Interim Rules and Guidelines issued on January 11, 1983 and had struck down in several
VERIFICATION/CERTIFICATION.
cases 16 the inveterate use of this insidious malpractice. Forum shopping as "the filing of
repetitious suits in different courts" has been condemned by Justice Andres R. Narvasa (now
We rule for private respondent. Chief Justice) in Minister of Natural Resources, et al., vs. Heirs of Orval Hughes, et al., "as a
reprehensible manipulation of court processes and proceedings . . ." 17 when does forum
shopping take place?
To begin with, forum-shopping originated as a concept in private international law.12 , where non-
resident litigants are given the option to choose the forum or place wherein to bring their suit for
various reasons or excuses, including to secure procedural advantages, to annoy and harass There is forum-shopping whenever, as a result of an adverse opinion in one forum, a
the defendant, to avoid overcrowded dockets, or to select a more friendly venue. To combat party seeks a favorable opinion (other than by appeal or certiorari) in another. The
these less than honorable excuses, the principle of forum non conveniens was developed principle applies not only with respect to suits filed in the courts but also in connection
whereby a court, in conflicts of law cases, may refuse impositions on its jurisdiction where it is with litigations commenced in the courts while an administrative proceeding is
not the most "convenient" or available forum and the parties are not precluded from seeking pending, as in this case, in order to defeat administrative processes and in anticipation
remedies elsewhere. of an unfavorable administrative ruling and a favorable court ruling. This is specially
so, as in this case, where the court in which the second suit was brought, has no
jurisdiction.18
In this light, Black's Law Dictionary 13 says that forum shopping "occurs when a party attempts to
have his action tried in a particular court or jurisdiction where he feels he will receive the most
favorable judgment or verdict." Hence, according to Words and Phrases14 , "a litigant is open to The test for determining whether a party violated the rule against forum shopping has been laid
the charge of "forum shopping" whenever he chooses a forum with slight connection to factual dawn in the 1986 case of Buan vs. Lopez 19 , also by Chief Justice Narvasa, and that is, forum
circumstances surrounding his suit, and litigants should be encouraged to attempt to settle their shopping exists where the elements of litis pendentia are present or where a final judgment in
differences without imposing undue expenses and vexatious situations on the courts". one case will amount to res judicata in the other, as follows:

In the Philippines, forum shopping has acquired a connotation encompassing not only a choice There thus exists between the action before this Court and RTC Case No. 86-36563
of venues, as it was originally understood in conflicts of laws, but also to a choice of remedies. identity of parties, or at least such parties as represent the same interests in both
As to the first (choice of venues), the Rules of Court, for example, allow a plaintiff to commence actions, as well as identity of rights asserted and relief prayed for, the relief being
personal actions "where the defendant or any of the defendants resides or may be found, or founded on the same facts, and the identity on the two preceding particulars is such
that any judgment rendered in the other action, will, regardless of which party is In an earlier case 23 but with the same logic and vigor, we held:
successful, amount to res adjudicata in the action under consideration: all the
requisites, in fine, of auter action pendant.
In other words, the filing by the petitioners of the instant special civil action
for certiorari and prohibition in this Court despite the pendency of their action in the
xxx xxx xxx Makati Regional Trial Court, is a species of forum-shopping. Both actions
unquestionably involve the same transactions, the same essential facts and
circumstances. The petitioners' claim of absence of identity simply because the PCGG
As already observed, there is between the action at bar and RTC Case No. 86-36563,
had not been impleaded in the RTC suit, and the suit did not involve certain acts which
an identity as regards parties, or interests represented, rights asserted and relief
transpired after its commencement, is specious. In the RTC action, as in the action
sought, as well as basis thereof, to a degree sufficient to give rise to the ground for
before this Court, the validity of the contract to purchase and sell of September 1,
dismissal known as auter action pendant or lis pendens. That same identity puts into
1986, i.e., whether or not it had been efficaciously rescinded, and the propriety of
operation the sanction of twin dismissals just mentioned. The application of this
implementing the same (by paying the pledgee banks the amount of their loans,
sanction will prevent any further delay in the settlement of the controversy which might
obtaining the release of the pledged shares, etc.) were the basic issues. So, too, the
ensue from attempts to seek reconsideration of or to appeal from the Order of the
relief was the same: the prevention of such implementation and/or the restoration of
Regional Trial Court in Civil Case No. 86-36563 promulgated on July 15, 1986, which
the status quo ante. When the acts sought to be restrained took place anyway despite
dismissed the petition upon grounds which appear persuasive.
the issuance by the Trial Court of a temporary restraining order, the RTC suit did not
become functus oficio. It remained an effective vehicle for obtention of relief; and
Consequently, where a litigant (or one representing the same interest or person) sues the same petitioners' remedy in the premises was plain and patent: the filing of an amended and
party against whom another action or actions for the alleged violation of the same right and the supplemental pleading in the RTC suit, so as to include the PCGG as defendant and
enforcement of the same relief is/are still pending, the defense of litis pendencia in one case is seek nullification of the acts sought to be enjoined but nonetheless done. The remedy
bar to the others; and, a final judgment in one would constitute res judicata and thus would was certainly not the institution of another action in another forum based on essentially
cause the dismissal of the rest. In either case, forum shopping could be cited by the other party the same facts, The adoption of this latter recourse renders the petitioners amenable
as a ground to ask for summary dismissal of the two 20 (or more) complaints or petitions, and for to disciplinary action and both their actions, in this Court as well as in the Court a quo,
imposition of the other sanctions, which are direct contempt of court, criminal prosecution, and dismissible.
disciplinary action against the erring lawyer.
In the instant case before us, there is also identity of parties, or at least, of interests represented.
Applying the foregoing principles in the case before us and comparing it with the Second Case, it Although the plaintiffs in the Second Case (Henry L. Co. et al.) are not name parties in the First
is obvious that there exist identity of parties or interests represented, identity of rights or causes Case, they represent the same interest and entity, namely, petitioner Bank, because:
and identity of reliefs sought.
Firstly, they are not suing in their personal capacities, for they have no direct personal interest in
Very simply stated, the original complaint in the court a quo which gave rise to the instant the matter in controversy. They are not principally or even subsidiarily liable; much less are they
petition was filed by the buyer (herein private respondent and his predecessors-in-interest) direct parties in the assailed contract of sale; and
against the seller (herein petitioners) to enforce the alleged perfected sale of real estate. On the
other hand, the complaint 21 in the Second Case seeks to declare such purported sale involving
Secondly, the allegations of the complaint in the Second Case show that the stockholders are
the same real property "as unenforceable as against the Bank", which is the petitioner herein. In
bringing a "derivative suit". In the caption itself, petitioners claim to have brought suit "for and in
other words, in the Second Case, the majority stockholders, in representation of the Bank, are
behalf of the Producers Bank of the Philippines" 24 . Indeed, this is the very essence of a
seeking to accomplish what the Bank itself failed to do in the original case in the trial court. In
derivative suit:
brief, the objective or the relief being sought, though worded differently, is the same, namely, to
enable the petitioner Bank to escape from the obligation to sell the property to respondent. In
Danville Maritime, Inc. vs. Commission on Audit. 22 , this Court ruled that the filing by a party of An individual stockholder is permitted to institute a derivative suit on behalf of the
two apparently different actions, but with the same objective, constituted forum shopping: corporation wherein he holdsstock in order to protect or vindicate corporate
rights, whenever the officials of the corporation refuse to sue, or are the ones to be
sued or hold the control of the corporation. In such actions, the suing stockholder is
In the attempt to make the two actions appear to be different, petitioner impleaded
regarded as a nominal party, with the corporation as the real party in interest.
different respondents therein — PNOC in the case before the lower court and the
(Gamboa v. Victoriano, 90 SCRA 40, 47 [1979]; emphasis supplied).
COA in the case before this Court and sought what seems to be different reliefs.
Petitioner asks this Court to set aside the questioned letter-directive of the COA dated
October 10, 1988 and to direct said body to approve the Memorandum of Agreement In the face of the damaging admissions taken from the complaint in the Second Case,
entered into by and between the PNOC and petitioner, while in the complaint before petitioners, quite strangely, sought to deny that the Second Case was a derivative suit,
the lower court petitioner seeks to enjoin the PNOC from conducting a rebidding and reasoning that it was brought, not by the minority shareholders, but by Henry Co et al., who not
from selling to other parties the vessel "T/T Andres Bonifacio", and for an extension of only own, hold or control over 80% of the outstanding capital stock, but also constitute the
time for it to comply with the paragraph 1 of the memorandum of agreement and majority in the Board of Directors of petitioner Bank. That being so, then they really represent the
damages. One can see that although the relief prayed for in the two (2) actions are Bank. So, whether they sued "derivatively" or directly, there is undeniably an identity of
ostensibly different, the ultimate objective in both actions is the same, that is, approval interests/entity represented.
of the sale of vessel in favor of petitioner and to overturn the letter-directive of the
COA of October 10, 1988 disapproving the sale. (emphasis supplied).
Petitioner also tried to seek refuge in the corporate fiction that the personality Of the Bank is The foregoing conclusion finding the existence of forum-shopping notwithstanding, the only
separate and distinct from its shareholders. But the rulings of this Court are consistent: "When sanction possible now is the dismissal of both cases with prejudice, as the other sanctions
the fiction is urged as a means of perpetrating a fraud or an illegal act or as a vehicle for the cannot be imposed because petitioners' present counsel entered their appearance only during
evasion of an existing obligation, the circumvention of statutes, the achievement or perfection of the proceedings in this Court, and the Petition's VERIFICATION/CERTIFICATION contained
a monopoly or generally the perpetration of knavery or crime, the veil with which the law covers sufficient allegations as to the pendency of the Second Case to show good faith in observing
and isolates the corporation from the members or stockholders who compose it will be lifted to Circular 28-91. The Lawyers who filed the Second Case are not before us; thus the rudiments of
allow for its consideration merely as an aggregation of individuals." 25 due process prevent us from motu propio imposing disciplinary measures against them in this
Decision. However, petitioners themselves (and particularly Henry Co, et al.) as litigants are
admonished to strictly follow the rules against forum-shopping and not to trifle with court
In addition to the many cases 26 where the corporate fiction has been disregarded, we now add
proceedings and processes They are warned that a repetition of the same will be dealt with
the instant case, and declare herewith that the corporate veil cannot be used to shield an
more severely.
otherwise blatant violation of the prohibition against forum-shopping. Shareholders, whether
suing as the majority in direct actions or as the minority in a derivative suit, cannot be allowed to
trifle with court processes, particularly where, as in this case, the corporation itself has not been Having said that, let it be emphasized that this petition should be dismissed not merely because
remiss in vigorously prosecuting or defending corporate causes and in using and applying of forum-shopping but also because of the substantive issues raised, as will be discussed
remedies available to it. To rule otherwise would be to encourage corporate litigants to use their shortly.
shareholders as fronts to circumvent the stringent rules against forum shopping.
The Second Issue: Was The Contract Perfected?
Finally, petitioner Bank argued that there cannot be any forum shopping, even
assuming arguendo that there is identity of parties, causes of action and reliefs sought,
The respondent Court correctly treated the question of whether or not there was, on the basis of
"because it (the Bank) was the defendant in the (first) case while it was the plaintiff in the other
the facts established, a perfected contract of sale as the ultimate issue. Holding that a valid
(Second Case)",citing as authority Victronics Computers, Inc., vs. Regional Trial Court, Branch
contract has been established, respondent Court stated:
63, Makati, etc. et al., 27 where Court held:

There is no dispute that the object of the transaction is that property owned by the
The rule has not been extended to a defendant who, for reasons known only to him,
defendant bank as acquired assets consisting of six (6) parcels of land specifically
commences a new action against the plaintiff — instead of filing a responsive pleading
identified under Transfer Certificates of Title Nos. T-106932 to T-106937. It is likewise
in the other case — setting forth therein, as causes of action, specific denials, special
beyond cavil that the bank intended to sell the property. As testified to by the Bank's
and affirmative defenses or even counterclaims, Thus, Velhagen's and King's motion
Deputy Conservator, Jose Entereso, the bank was looking for buyers of the property.
to dismiss Civil Case No. 91-2069 by no means negates the charge of forum-shopping
It is definite that the plaintiffs wanted to purchase the property and it was precisely for
as such did not exist in the first place. (emphasis supplied)
this purpose that they met with defendant Rivera, Manager of the Property
Management Department of the defendant bank, in early August 1987. The procedure
Petitioner pointed out that since it was merely the defendant in the original case, it could not in the sale of acquired assets as well as the nature and scope of the authority of
have chosen the forum in said case. Rivera on the matter is clearly delineated in the testimony of Rivera himself, which
testimony was relied upon by both the bank and by Rivera in their appeal briefs. Thus
(TSN of July 30, 1990. pp. 19-20):
Respondent, on the other hand, replied that there is a difference in factual setting
between Victronics and the present suit. In the former, as underscored in the above-quoted
Court ruling, the defendants did not file any responsive pleading in the first case. In other words, A: The procedure runs this way: Acquired assets was turned over to me and
they did not make any denial or raise any defense or counter-claim therein In the case before us then I published it in the form of an inter-office memorandum distributed to
however, petitioners filed a responsive pleading to the complaint — as a result of which, the all branches that these are acquired assets for sale. I was instructed to
issues were joined. advertise acquired assets for sale so on that basis, I have to entertain offer;
to accept offer, formal offer and upon having been offered, I present it to the
Committee. I provide the Committee with necessary information about the
Indeed, by praying for affirmative reliefs and interposing counter–claims in their responsive
property such as original loan of the borrower, bid price during the
pleadings, the petitioners became plaintiffs themselves in the original case, giving unto
foreclosure, total claim of the bank, the appraised value at the time the
themselves the very remedies they repeated in the Second Case.
property is being offered for sale and then the information which are relative
to the evaluation of the bank to buy which the Committee considers and it is
Ultimately, what is truly important to consider in determining whether forum-shopping exists or the Committee that evaluate as against the exposure of the bank and it is
not is the vexation caused the courts and parties-litigant by a party who asks different courts also the Committee that submit to the Conservator for final approval and
and/or administrative agencies to rule on the same or related causes and/or to grant the same or once approved, we have to execute the deed of sale and it is the
substantially the same reliefs, in the process creating the possibility of conflicting decisions being Conservator that sign the deed of sale, sir.
rendered by the different fora upon the same issue. In this case, this is exactly the problem: a
decision recognizing the perfection and directing the enforcement of the contract of sale will
The plaintiffs, therefore, at that meeting of August 1987 regarding their purpose of
directly conflict with a possible decision in the Second Case barring the parties front enforcing or
buying the property, dealt with and talked to the right person. Necessarily, the agenda
implementing the said sale. Indeed, a final decision in one would constitute res judicata in the
was the price of the property, and plaintiffs were dealing with the bank official
other 28 .
authorized to entertain offers, to accept offers and to present the offer to the
Committee before which the said official is authorized to discuss information relative to bank, claim that the offer of the plaintiff was never discussed by the Committee. In the
price determination. Necessarily, too, it being inherent in his authority, Rivera is the same vein, both Co and Entereso openly admit that they seldom attend the meetings
officer from whom official information regarding the price, as determined by the of the Committee. It is important to note that negotiations on the price had started in
Committee and approved by the Conservator, can be had. And Rivera confirmed his early August and the plaintiffs had already offered an amount as purchase price,
authority when he talked with the plaintiff in August 1987. The testimony of plaintiff having been made to understand by Rivera, the official in charge of the negotiation,
Demetria is clear on this point (TSN of May 31,1990, pp. 27-28): that the price will be submitted for approval by the bank and that the bank's decision
will be relayed to plaintiffs. From the facts, the official bank price. At any rate, the bank
placed its official, Rivera, in a position of authority to accept offers to buy and
Q: When you went to the Producers Bank and talked with Mr. Mercurio
negotiate the sale by having the offer officially acted upon by the bank. The bank
Rivera, did you ask him point-blank his authority to sell any property?
cannot turn around and later say, as it now does, that what Rivera states as the bank's
action on the matter is not in fact so. It is a familiar doctrine, the doctrine of ostensible
A: No, sir. Not point blank although it came from him, (W)hen I asked him authority, that if a corporation knowingly permits one of its officers, or any other agent,
how long it would take because he was saying that the matter of pricing will to do acts within the scope of an apparent authority, and thus holds him out to the
be passed upon by the committee. And when I asked him how long it will public as possessing power to do those acts, the corporation will, as against any one
take for the committee to decide and he said the committee meets every who has in good faith dealt with the corporation through such agent, he estopped from
week. If I am not mistaken Wednesday and in about two week's (sic) time, denying his authority (Francisco v. GSIS, 7 SCRA 577, 583-584; PNB v. Court of
in effect what he was saying he was not the one who was to decide. But he Appeals, 94 SCRA 357, 369-370; Prudential Bank v. Court of Appeals, G.R. No.
would refer it to the committee and he would relay the decision of the 103957, June 14, 1993). 29
committee to me.
Article 1318 of the Civil Code enumerates the requisites of a valid and perfected contract as
Q — Please answer the question. follows: "(1) Consent of the contracting parties; (2) Object certain which is the subject matter of
the contract; (3) Cause of the obligation which is established."
A — He did not say that he had the authority (.) But he said he would refer
the matter to the committee and he would relay the decision to me and he There is no dispute on requisite no. 2. The object of the questioned contract consists of the six
did just like that. (6) parcels of land in Sta. Rosa, Laguna with an aggregate area of about 101 hectares, more or
less, and covered by Transfer Certificates of Title Nos. T-106932 to T-106937. There is,
however, a dispute on the first and third requisites.
"Parenthetically, the Committee referred to was the Past Due Committee of which Luis
Co was the Head, with Jose Entereso as one of the members.
Petitioners allege that "there is no counter-offer made by the Bank, and any supposed counter-
offer which Rivera (or Co) may have made is unauthorized. Since there was no counter-offer by
What transpired after the meeting of early August 1987 are consistent with the the Bank, there was nothing for Ejercito (in substitution of Demetria and Janolo) to
authority and the duties of Rivera and the bank's internal procedure in the matter of accept." 30 They disputed the factual basis of the respondent Court's findings that there was an
the sale of bank's assets. As advised by Rivera, the plaintiffs made a formal offer by a offer made by Janolo for P3.5 million, to which the Bank counter-offered P5.5 million. We have
letter dated August 20, 1987 stating that they would buy at the price of P3.5 Million in perused the evidence but cannot find fault with the said Court's findings of fact. Verily, in a
cash. The letter was for the attention of Mercurio Rivera who was tasked to convey petition under Rule 45 such as this, errors of fact — if there be any - are, as a rule, not
and accept such offers. Considering an aspect of the official duty of Rivera as some reviewable. The mere fact that respondent Court (and the trial court as well) chose to believe the
sort of intermediary between the plaintiffs-buyers with their proposed buying price on
evidence presented by respondent more than that presented by petitioners is not by itself a
one hand, and the bank Committee, the Conservator and ultimately the bank itself with reversible error. In fact, such findings merit serious consideration by this Court, particularly
the set price on the other, and considering further the discussion of price at the where, as in this case, said courts carefully and meticulously discussed their findings. This is
meeting of August resulting in a formal offer of P3.5 Million in cash, there can be no
basic.
other logical conclusion than that when, on September 1, 1987, Rivera informed
plaintiffs by letter that "the bank's counter-offer is at P5.5 Million for more than 101
hectares on lot basis," such counter-offer price had been determined by the Past Due Be that as it may, and in addition to the foregoing disquisitions by the Court of Appeals, let us
Committee and approved by the Conservator after Rivera had duly presented review the question of Rivera's authority to act and petitioner's allegations that the P5.5 million
plaintiffs' offer for discussion by the Committee of such matters as original loan of counter-offer was extinguished by the P4.25 million revised offer of Janolo. Here, there are
borrower, bid price during foreclosure, total claim of the bank, and market value. questions of law which could be drawn from the factual findings of the respondent Court. They
Tersely put, under the established facts, the price of P5.5 Million was, as clearly also delve into the contractual elements of consent and cause.
worded in Rivera's letter (Exh. "E"), the official and definitive price at which the bank
was selling the property.
The authority of a corporate officer in dealing with third persons may be actual or apparent. The
doctrine of "apparent authority", with special reference to banks, was laid out in Prudential Bank
There were averments by defendants below, as well as before this Court, that the vs. Court of Appeals31 , where it was held that:
P5.5 Million price was not discussed by the Committee and that price. As correctly
characterized by the trial court, this is not credible. The testimonies of Luis Co and
Conformably, we have declared in countless decisions that the principal is liable for
Jose Entereso on this point are at best equivocal and considering the gratuitous and
obligations contracted by the agent. The agent's apparent representation yields to the
self-serving character of these declarations, the bank's submission on this point does
principal's true representation and the contract is considered as entered into between
not inspire belief. Both Co ad Entereso, as members of the Past Due Committee of the
the principal and the third person (citing National Food Authority vs. Intermediate April 26, 1990, pp. 34-35). At said meeting, Co, a major shareholder and officer of the
Appellate Court, 184 SCRA 166). Bank, confirmed Rivera's statement as to the finality of the Bank's counter-offer of
P5.5 million (TSN, January 16, 1990, p. 21; TSN, April 26, 1990, p. 35);
A bank is liable for wrongful acts of its officers done in the interests of the
bank or in the course of dealings of the officers in their representative (h) In its newspaper advertisements and announcements, the Bank referred to Rivera
capacity but not for acts outside the scape of their authority (9 C.J.S., p. as the officer acting for the Bank in relation to parties interested in buying assets
417). A bank holding out its officers and agents as worthy of confidence will owned/acquired by the Bank. In fact, Rivera was the officer mentioned in the Bank's
not be permitted to profit by the frauds they may thus be enabled to advertisements offering for sale the property in question (cf. Exhs. "S" and "S-1").
perpetrate in the apparent scope of their employment; nor will it be
permitted to shirk its responsibility for such frauds even though no benefit
In the very recent case of Limketkai Sons Milling, Inc. vs. Court of Appeals, et. al.32 , the Court,
may accrue to the bank therefrom (10 Am Jur 2d, p. 114). Accordingly, a
through Justice Jose A. R. Melo, affirmed the doctrine of apparent authority as it held that the
banking corporation is liable to innocent third persons where the
apparent authority of the officer of the Bank of P.I. in charge of acquired assets is borne out by
representation is made in the course of its business by an agent acting
similar circumstances surrounding his dealings with buyers.
within the general scope of his authority even though, in the particular case,
the agent is secretly abusing his authority and attempting to perpetrate a
fraud upon his principal or some other person, for his own ultimate benefit To be sure, petitioners attempted to repudiate Rivera's apparent authority through documents
(McIntosh v. Dakota Trust Co., 52 ND 752, 204 NW 818, 40 ALR 1021). and testimony which seek to establish Rivera's actual authority. These pieces of evidence,
however, are inherently weak as they consist of Rivera's self-serving testimony and various
inter-office memoranda that purport to show his limited actual authority, of which private
Application of these principles is especially necessary because banks have a fiduciary
respondent cannot be charged with knowledge. In any event, since the issue is apparent
relationship with the public and their stability depends on the confidence of the people
authority, the existence of which is borne out by the respondent Court's findings, the evidence of
in their honesty and efficiency. Such faith will be eroded where banks do not exercise
actual authority is immaterial insofar as the liability of a corporation is concerned 33 .
strict care in the selection and supervision of its employees, resulting in prejudice to
their depositors.
Petitioners also argued that since Demetria and Janolo were experienced lawyers and their "law
firm" had once acted for the Bank in three criminal cases, they should be charged with actual
From the evidence found by respondent Court, it is obvious that petitioner Rivera has apparent
knowledge of Rivera's limited authority. But the Court of Appeals in its Decision (p. 12) had
or implied authority to act for the Bank in the matter of selling its acquired assets. This evidence
already made a factual finding that the buyers had no notice of Rivera's actual authority prior to
includes the following:
the sale. In fact, the Bank has not shown that they acted as its counsel in respect to any
acquired assets; on the other hand, respondent has proven that Demetria and Janolo merely
(a) The petition itself in par. II-i (p. 3) states that Rivera was "at all times material to associated with a loose aggrupation of lawyers (not a professional partnership), one of whose
this case, Manager of the Property Management Department of the Bank". By his own members (Atty. Susana Parker) acted in said criminal cases.
admission, Rivera was already the person in charge of the Bank's acquired assets
(TSN, August 6, 1990, pp. 8-9);
Petitioners also alleged that Demetria's and Janolo's P4.25 million counter-offer in the letter
dated September 17, 1987 extinguished the Bank's offer of P5.5 million 34 .They disputed the
(b) As observed by respondent Court, the land was definitely being sold by the Bank. respondent Court's finding that "there was a meeting of minds when on 30 September 1987
And during the initial meeting between the buyers and Rivera, the latter suggested Demetria and Janolo through Annex "L" (letter dated September 30, 1987) "accepted" Rivera's
that the buyers' offer should be no less than P3.3 million (TSN, April 26, 1990, pp. 16- counter offer of P5.5 million under Annex "J" (letter dated September 17, 1987)", citingthe late
17); Justice Paras35 , Art. 1319 of the Civil Code 36 and related Supreme Court rulings starting with
Beaumont vs. Prieto 37 .
(c) Rivera received the buyers' letter dated August 30, 1987 offering P3.5 million
(TSN, 30 July 1990, p.11); However, the above-cited authorities and precedents cannot apply in the instant case because,
as found by the respondent Court which reviewed the testimonies on this point, what was
"accepted" by Janolo in his letter dated September 30, 1987 was the Bank's offer of P5.5 million
(d) Rivera signed the letter dated September 1, 1987 offering to sell the property for
as confirmed and reiterated to Demetria and Atty. Jose Fajardo by Rivera and Co during their
P5.5 million (TSN, July 30, p. 11);
meeting on September 28, 1987. Note that the said letter of September 30, 1987 begins
with"(p)ursuant to our discussion last 28 September 1987 . . .
(e) Rivera received the letter dated September 17, 1987 containing the buyers'
proposal to buy the property for P4.25 million (TSN, July 30, 1990, p. 12);
Petitioners insist that the respondent Court should have believed the testimonies of Rivera and
Co that the September 28, 1987 meeting "was meant to have the offerors improve on their
(f) Rivera, in a telephone conversation, confirmed that the P5.5 million was the final position of P5.5. million."38 However, both the trial court and the Court of Appeals found
price of the Bank (TSN, January 16, 1990, p. 18); petitioners' testimonial evidence "not credible", and we find no basis for changing this finding of
fact.
(g) Rivera arranged the meeting between the buyers and Luis Co on September 28,
1994, during which the Bank's offer of P5.5 million was confirmed by Rivera (TSN,
Indeed, we see no reason to disturb the lower courts' (both the RTC and the CA) common writing subscribed by the Bank to evidence such contract. (Please see article 1403[2],
finding that private respondents' evidence is more in keeping with truth and logic — that during Civil Code.)
the meeting on September 28, 1987, Luis Co and Rivera "confirmed that the P5.5 million price
has been passed upon by the Committee and could no longer be lowered (TSN of April 27,
Upon the other hand, the respondent Court in its Decision (p, 14) stated:
1990, pp. 34-35)"39 . Hence, assuming arguendo that the counter-offer of P4.25 million
extinguished the offer of P5.5 million, Luis Co's reiteration of the said P5.5 million price during
the September 28, 1987 meeting revived the said offer. And by virtue of the September 30, 1987 . . . Of course, the bank's letter of September 1, 1987 on the official price and the
letter accepting this revived offer, there was a meeting of the minds, as the acceptance in said plaintiffs' acceptance of the price on September 30, 1987, are not, in themselves,
letter was absolute and unqualified. formal contracts of sale. They are however clear embodiments of the fact that a
contract of sale was perfected between the parties, such contract being binding in
whatever form it may have been entered into (case citations omitted). Stated simply,
We note that the Bank's repudiation, through Conservator Encarnacion, of Rivera's authority and
the banks' letter of September 1, 1987, taken together with plaintiffs' letter dated
action, particularly the latter's counter-offer of P5.5 million, as being "unauthorized and illegal"
September 30, 1987, constitute in law a sufficient memorandum of a perfected
came only on May 12, 1988 or more than seven (7) months after Janolo' acceptance. Such
contract of sale.
delay, and the absence of any circumstance which might have justifiably prevented the Bank
from acting earlier, clearly characterizes the repudiation as nothing more than a last-minute
attempt on the Bank's part to get out of a binding contractual obligation. The respondent Court could have added that the written communications commenced not only
from September 1, 1987 but from Janolo's August 20, 1987 letter. We agree that, taken
together, these letters constitute sufficient memoranda — since they include the names of the
Taken together, the factual findings of the respondent Court point to an implied admission on the
parties, the terms and conditions of the contract, the price and a description of the property as
part of the petitioners that the written offer made on September 1, 1987 was carried through
the object of the contract.
during the meeting of September 28, 1987. This is the conclusion consistent with human
experience, truth and good faith.
But let it be assumed arguendo that the counter-offer during the meeting on September 28, 1987
did constitute a "new" offer which was accepted by Janolo on September 30, 1987. Still, the
It also bears noting that this issue of extinguishment of the Bank's offer of P5.5 million was
statute of frauds will not apply by reason of the failure of petitioners to object to oral testimony
raised for the first time on appeal and should thus be disregarded.
proving petitioner Bank's counter-offer of P5.5 million. Hence, petitioners — by such utter failure
to object — are deemed to have waived any defects of the contract under the statute of frauds,
This Court in several decisions has repeatedly adhered to the principle that points of pursuant to Article 1405 of the Civil Code:
law, theories, issues of fact and arguments not adequately brought to the attention of
the trial court need not be, and ordinarily will not be, considered by a reviewing court,
Art. 1405. Contracts infringing the Statute of Frauds, referred to in No. 2 of article
as they cannot be raised for the first time on appeal (Santos vs. IAC, No. 74243,
1403, are ratified by the failure to object to the presentation of oral evidence to prove
November 14, 1986, 145 SCRA 592).40
the same, or by the acceptance of benefits under them.

. . . It is settled jurisprudence that an issue which was neither averred in the complaint
As private respondent pointed out in his Memorandum, oral testimony on the reaffirmation of the
nor raised during the trial in the court below cannot be raised for the first time on
counter-offer of P5.5 million is a plenty — and the silence of petitioners all throughout the
appeal as it would be offensive to the basic rules of fair play, justice and due process
presentation makes the evidence binding on them thus;
(Dihiansan vs. CA, 153 SCRA 713 [1987]; Anchuelo vs. IAC, 147 SCRA 434 [1987];
Dulos Realty & Development Corp. vs. CA, 157 SCRA 425 [1988]; Ramos vs. IAC,
175 SCRA 70 [1989]; Gevero vs. IAC, G.R. 77029, August 30, 1990).41 A Yes, sir, I think it was September 28, 1987 and I was again present because Atty.
Demetria told me to accompany him we were able to meet Luis Co at the Bank.
Since the issue was not raised in the pleadings as an affirmative defense, private respondent
was not given an opportunity in the trial court to controvert the same through opposing evidence. xxx xxx xxx
Indeed, this is a matter of due process. But we passed upon the issue anyway, if only to avoid
deciding the case on purely procedural grounds, and we repeat that, on the basis of the
Q Now, what transpired during this meeting with Luis Co of the Producers Bank?
evidence already in the record and as appreciated by the lower courts, the inevitable conclusion
is simply that there was a perfected contract of sale.
A Atty. Demetria asked Mr. Luis Co whether the price could be reduced, sir.
The Third Issue: Is the Contract Enforceable?
Q What price?
The petition alleged42 :
A The 5.5 million pesos and Mr. Luis Co said that the amount cited by Mr. Mercurio
Even assuming that Luis Co or Rivera did relay a verbal offer to sell at P5.5 million Rivera is the final price and that is the price they intends (sic) to have, sir.
during the meeting of 28 September 1987, and it was this verbal offer that Demetria
and Janolo accepted with their letter of 30 September 1987, the contract produced Q What do you mean?.
thereby would be unenforceable by action — there being no note, memorandum or
A That is the amount they want, sir. A It was not discussed by the Committee but it was discussed initially by Luis Co and
the group of Atty. Demetrio Demetria and Atty. Pajardo (sic) in that September 28,
1987 meeting, sir.
Q What is the reaction of the plaintiff Demetria to Luis Co's statement (sic) that the
defendant Rivera's counter-offer of 5.5 million was the defendant's bank (sic) final
offer? [Direct testimony of Mercurio Rivera, TSN, 30 July 1990, pp. 14-15.]

A He said in a day or two, he will make final acceptance, sir. The Fourth Issue: May the Conservator Revoke
the Perfected and Enforceable Contract.
Q What is the response of Mr. Luis Co?.
It is not disputed that the petitioner Bank was under a conservator placed by the Central Bank of
the Philippines during the time that the negotiation and perfection of the contract of sale took
A He said he will wait for the position of Atty. Demetria, sir.
place. Petitioners energetically contended that the conservator has the power to revoke or
overrule actions of the management or the board of directors of a bank, under Section 28-A of
[Direct testimony of Atty. Jose Fajardo, TSN, January 16, 1990, at pp. 18-21.] Republic Act No. 265 (otherwise known as the Central Bank Act) as follows:

Q What transpired during that meeting between you and Mr. Luis Co of the defendant Whenever, on the basis of a report submitted by the appropriate supervising or
Bank? examining department, the Monetary Board finds that a bank or a non-bank financial
intermediary performing quasi-banking functions is in a state of continuing inability or
unwillingness to maintain a state of liquidity deemed adequate to protect the interest of
A We went straight to the point because he being a busy person, I told him if the
depositors and creditors, the Monetary Board may appoint a conservator to take
amount of P5.5 million could still be reduced and he said that was already passed charge of the assets, liabilities, and the management of that institution, collect all
upon by the committee. What the bank expects which was contrary to what Mr. Rivera monies and debts due said institution and exercise all powers necessary to preserve
stated. And he told me that is the final offer of the bank P5.5 million and we should
the assets of the institution, reorganize the management thereof, and restore its
indicate our position as soon as possible. viability. He shall have the power to overrule or revoke the actions of the previous
management and board of directors of the bank or non-bank financial intermediary
Q What was your response to the answer of Mr. Luis Co? performing quasi-banking functions, any provision of law to the contrary
notwithstanding, and such other powers as the Monetary Board shall deem necessary.
A I said that we are going to give him our answer in a few days and he said that was it.
Atty. Fajardo and I and Mr. Mercurio [Rivera] was with us at the time at his office. In the first place, this issue of the Conservator's alleged authority to revoke or repudiate the
perfected contract of sale was raised for the first time in this Petition — as this was not litigated
in the trial court or Court of Appeals. As already stated earlier, issues not raised and/or
Q For the record, your Honor please, will you tell this Court who was with Mr. Co in his ventilated in the trial court, let alone in the Court of Appeals, "cannot be raised for the first time
Office in Producers Bank Building during this meeting? on appeal as it would be offensive to the basic rules of fair play, justice and due process." 43

A Mr. Co himself, Mr. Rivera, Atty. Fajardo and I. In the second place, there is absolutely no evidence that the Conservator, at the time the
contract was perfected, actually repudiated or overruled said contract of sale. The Bank's acting
Q By Mr. Co you are referring to? conservator at the time, Rodolfo Romey, never objected to the sale of the property to Demetria
and Janolo. What petitioners are really referring to is the letter of Conservator Encarnacion, who
took over from Romey after the sale was perfected on September 30, 1987 (Annex V, petition)
A Mr. Luis Co. which unilaterally repudiated — not the contract — but the authority of Rivera to make a binding
offer — and which unarguably came months after the perfection of the contract. Said letter dated
Q After this meeting with Mr. Luis Co, did you and your partner accede on (sic) the May 12, 1988 is reproduced hereunder:
counter offer by the bank?
May 12, 1988
A Yes, sir, we did.? Two days thereafter we sent our acceptance to the bank which
offer we accepted, the offer of the bank which is P5.5 million.
Atty. Noe C. Zarate
Zarate Carandang Perlas & Ass.
[Direct testimony of Atty. Demetria, TSN, 26 April 1990, at pp. 34-36.] Suite 323 Rufino Building
Ayala Avenue, Makati, Metro-Manila
Q According to Atty. Demetrio Demetria, the amount of P5.5 million was reached by
the Committee and it is not within his power to reduce this amount. What can you say Dear Atty. Zarate:
to that statement that the amount of P5.5 million was reached by the Committee?
This pertains to your letter dated May 5, 1988 on behalf of Attys. Janolo and Demetria Obviously, therefore, Section 28-A merely gives the conservator power to revoke contracts that
regarding the six (6) parcels of land located at Sta. Rosa, Laguna. are, under existing law, deemed to be defective — i.e., void, voidable, unenforceable or
rescissible. Hence, the conservator merely takes the place of a bank's board of directors. What
the said board cannot do — such as repudiating a contract validly entered into under the
We deny that Producers Bank has ever made a legal counter-offer to any of your
doctrine of implied authority — the conservator cannot do either. Ineluctably, his power is not
clients nor perfected a "contract to sell and buy" with any of them for the following
unilateral and he cannot simply repudiate valid obligations of the Bank. His authority would be
reasons.
only to bring court actions to assail such contracts — as he has already done so in the instant
case. A contrary understanding of the law would simply not be permitted by the Constitution.
In the "Inter-Office Memorandum" dated April 25, 1986 addressed to and approved by Neither by common sense. To rule otherwise would be to enable a failing bank to become
former Acting Conservator Mr. Andres I. Rustia, Producers Bank Senior Manager solvent, at the expense of third parties, by simply getting the conservator to unilaterally revoke
Perfecto M. Pascua detailed the functions of Property Management Department all previous dealings which had one way or another or come to be considered unfavorable to the
(PMD) staff and officers (Annex A.), you will immediately read that Manager Mr. Bank, yielding nothing to perfected contractual rights nor vested interests of the third parties who
Mercurio Rivera or any of his subordinates has no authority, power or right to make had dealt with the Bank.
any alleged counter-offer. In short, your lawyer-clients did not deal with the authorized
officers of the bank.
The Fifth Issue: Were There Reversible Errors of Facts?

Moreover, under Sec. 23 and 36 of the Corporation Code of the Philippines (Bates
Basic is the doctrine that in petitions for review under Rule 45 of the Rules of Court, findings of
Pambansa Blg. 68.) and Sec. 28-A of the Central Bank Act (Rep. Act No. 265, as
fact by the Court of Appeals are not reviewable by the Supreme Court. In Andres
amended), only the Board of Directors/Conservator may authorize the sale of any
vs. Manufacturers Hanover & Trust Corporation, 45 , we held:
property of the corportion/bank..

. . . The rule regarding questions of fact being raised with this Court in a petition
Our records do not show that Mr. Rivera was authorized by the old board or by any of
for certiorari under Rule 45 of the Revised Rules of Court has been stated in
the bank conservators (starting January, 1984) to sell the aforesaid property to any of
Remalante vs. Tibe, G.R. No. 59514, February 25, 1988, 158 SCRA 138, thus:
your clients. Apparently, what took place were just preliminary
discussions/consultations between him and your clients, which everyone
knows cannot bind the Bank's Board or Conservator. The rule in this jurisdiction is that only questions of law may be raised in a petition
for certiorari under Rule 45 of the Revised Rules of Court. "The jurisdiction of the
Supreme Court in cases brought to it from the Court of Appeals is limited to reviewing
We are, therefore, constrained to refuse any tender of payment by your clients, as the
and revising the errors of law imputed to it, its findings of the fact being conclusive "
same is patently violative of corporate and banking laws. We believe that this is more
[Chan vs. Court of Appeals, G.R. No. L-27488, June 30, 1970, 33 SCRA 737,
than sufficient legal justification for refusing said alleged tender.
reiterating a long line of decisions]. This Court has emphatically declared that "it is not
the function of the Supreme Court to analyze or weigh such evidence all over again,
Rest assured that we have nothing personal against your clients. All our acts are its jurisdiction being limited to reviewing errors of law that might have been committed
official, legal and in accordance with law. We also have no personal interest in any of by the lower court" (Tiongco v. De la Merced, G. R. No. L-24426, July 25, 1974, 58
the properties of the Bank. SCRA 89; Corona vs. Court of Appeals, G.R. No. L-62482, April 28, 1983, 121 SCRA
865; Baniqued vs. Court of Appeals, G. R. No. L-47531, February 20, 1984, 127
SCRA 596). "Barring, therefore, a showing that the findings complained of are totally
Please be advised accordingly.
devoid of support in the record, or that they are so glaringly erroneous as to constitute
serious abuse of discretion, such findings must stand, for this Court is not expected or
Very truly yours, required to examine or contrast the oral and documentary evidence submitted by the
parties" [Santa Ana, Jr. vs. Hernandez, G. R. No. L-16394, December 17, 1966, 18
SCRA 973] [at pp. 144-145.]
(Sgd.) Leonida T. Encarnacion
LEONIDA T. EDCARNACION
Acting Conservator Likewise, in Bernardo vs. Court of Appeals 46 , we held:

In the third place, while admittedly, the Central Bank law gives vast and far-reaching powers to The resolution of this petition invites us to closely scrutinize the facts of the case,
the conservator of a bank, it must be pointed out that such powers must be related to the relating to the sufficiency of evidence and the credibility of witnesses presented. This
"(preservation of) the assets of the bank, (the reorganization of) the management thereof and Court so held that it is not the function of the Supreme Court to analyze or weigh such
(the restoration of) its viability." Such powers, enormous and extensive as they are, cannot evidence all over again. The Supreme Court's jurisdiction is limited to reviewing errors
extend to the post-facto repudiation of perfected transactions, otherwise they would infringe of law that may have been committed by the lower court. The Supreme Court is not a
against the non-impairment clause of the Constitution 44 . If the legislature itself cannot revoke an trier of facts. . . .
existing valid contract, how can it delegate such non-existent powers to the conservator under
Section 28-A of said law?
As held in the recent case of Chua Tiong Tay vs. Court of Appeals and Goldrock Construction
and Development Corp. 47 :
The Court has consistently held that the factual findings of the trial court, as well as To become credible and unequivocal, petitioners should have presented then Conservator
the Court of Appeals, are final and conclusive and may not be reviewed on appeal. Rodolfo Romey to testify on their behalf, as he would have been in the best position to establish
Among the exceptional circumstances where a reassessment of facts found by the their thesis. Under the rules on evidence 51 , such suppression gives rise to the presumption that
lower courts is allowed are when the conclusion is a finding grounded entirely on his testimony would have been adverse, if produced.
speculation, surmises or conjectures; when the inference made is manifestly absurd,
mistaken or impossible; when there is grave abuse of discretion in the appreciation of
The second point was squarely raised in the Court of Appeals, but petitioners' evidence was
facts; when the judgment is premised on a misapprehension of facts; when the
deemed insufficient by both the trial court and the respondent Court, and instead, it was
findings went beyond the issues of the case and the same are contrary to the
respondent's submissions that were believed and became bases of the conclusions arrived at.
admissions of both appellant and appellee. After a careful study of the case at bench,
we find none of the above grounds present to justify the re-evaluation of the findings of
fact made by the courts below. In fine, it is quite evident that the legal conclusions arrived at from the findings of fact by the
lower courts are valid and correct. But the petitioners are now asking this Court to disturb these
findings to fit the conclusion they are espousing, This we cannot do.
In the same vein, the ruling of this Court in the recent case of South Sea Surety and Insurance
Company Inc. vs. Hon. Court of Appeals, et al. 48 is equally applicable to the present case:
To be sure, there are settled exceptions where the Supreme Court may disregard findings of fact
by the Court of Appeals 52 . We have studied both the records and the CA Decision and we find
We see no valid reason to discard the factual conclusions of the appellate court, . . .
no such exceptions in this case. On the contrary, the findings of the said Court are supported by
(I)t is not the function of this Court to assess and evaluate all over again the evidence,
a preponderance of competent and credible evidence. The inferences and conclusions are
testimonial and documentary, adduced by the parties, particularly where, such as
seasonably based on evidence duly identified in the Decision. Indeed, the appellate court
here, the findings of both the trial court and the appellate court on the matter coincide.
patiently traversed and dissected the issues presented before it, lending credibility and
(emphasis supplied)
dependability to its findings. The best that can be said in favor of petitioners on this point is that
the factual findings of respondent Court did not correspond to petitioners' claims, but were closer
Petitioners, however, assailed the respondent Court's Decision as "fraught with findings and to the evidence as presented in the trial court by private respondent. But this alone is no reason
conclusions which were not only contrary to the evidence on record but have no bases at all," to reverse or ignore such factual findings, particularly where, as in this case, the trial court and
specifically the findings that (1) the "Bank's counter-offer price of P5.5 million had been the appellate court were in common agreement thereon. Indeed, conclusions of fact of a trial
determined by the past due committee and approved by conservator Romey, after Rivera judge — as affirmed by the Court of Appeals — are conclusive upon this Court, absent any
presented the same for discussion" and (2) "the meeting with Co was not to scale down the price serious abuse or evident lack of basis or capriciousness of any kind, because the trial court is in
and start negotiations anew, but a meeting on the already determined price of P5.5 million" a better position to observe the demeanor of the witnesses and their courtroom manner as well
Hence, citingPhilippine National Bank vs. Court of Appeals 49 , petitioners are asking us to review as to examine the real evidence presented.
and reverse such factual findings.
Epilogue.
50
The first point was clearly passed upon by the Court of Appeals , thus:
In summary, there are two procedural issues involved forum-shopping and the raising of issues
There can be no other logical conclusion than that when, on September 1, 1987, for the first time on appeal [viz., the extinguishment of the Bank's offer of P5.5 million and the
Rivera informed plaintiffs by letter that "the bank's counter-offer is at P5.5 Million for conservator's powers to repudiate contracts entered into by the Bank's officers] — which per
more than 101 hectares on lot basis, "such counter-offer price had been determined se could justify the dismissal of the present case. We did not limit ourselves thereto, but delved
by the Past Due Committee and approved by the Conservator after Rivera had duly as well into the substantive issues — the perfection of the contract of sale and its enforceability,
presented plaintiffs' offer for discussion by the Committee . . . Tersely put, under the which required the determination of questions of fact. While the Supreme Court is not a trier of
established fact, the price of P5.5 Million was, as clearly worded in Rivera's letter facts and as a rule we are not required to look into the factual bases of respondent Court's
(Exh. "E"), the official and definitive price at which the bank was selling the property. decisions and resolutions, we did so just the same, if only to find out whether there is reason to
(p. 11, CA Decision) disturb any of its factual findings, for we are only too aware of the depth, magnitude and vigor by
which the parties through their respective eloquent counsel, argued their positions before this
Court.
xxx xxx xxx

We are not unmindful of the tenacious plea that the petitioner Bank is operating abnormally
. . . The argument deserves scant consideration. As pointed out by plaintiff, during the
under a government-appointed conservator and "there is need to rehabilitate the Bank in order
meeting of September 28, 1987 between the plaintiffs, Rivera and Luis Co, the senior
to get it back on its feet . . . as many people depend on (it) for investments, deposits and well as
vice-president of the bank, where the topic was the possible lowering of the price, the
employment. As of June 1987, the Bank's overdraft with the Central Bank had already reached
bank official refused it and confirmed that the P5.5 Million price had been passed upon
P1.023 billion . . . and there were (other) offers to buy the subject properties for a substantial
by the Committee and could no longer be lowered (TSN of April 27, 1990, pp. 34-35)
amount of money." 53
(p. 15, CA Decision).

While we do not deny our sympathy for this distressed bank, at the same time, the Court cannot
The respondent Court did not believe the evidence of the petitioners on this point, characterizing
emotionally close its eyes to overriding considerations of substantive and procedural law, like
it as "not credible" and "at best equivocal and considering the gratuitous and self-serving
respect for perfected contracts, non-impairment of obligations and sanctions against forum-
character of these declarations, the bank's submissions on this point do not inspire belief."
shopping, which must be upheld under the rule of law and blind justice.
This Court cannot just gloss over private respondent's submission that, while the subject
properties may currently command a much higher price, it is equally true that at the time of the
transaction in 1987, the price agreed upon of P5.5 million was reasonable, considering that the
Bank acquired these properties at a foreclosure sale for no more than P3.5 million 54 . That the
Bank procrastinated and refused to honor its commitment to sell cannot now be used by it to
promote its own advantage, to enable it to escape its binding obligation and to reap the benefits
of the increase in land values. To rule in favor of the Bank simply because the property in
question has algebraically accelerated in price during the long period of litigation is to reward
lawlessness and delays in the fulfillment of binding contracts. Certainly, the Court cannot stamp
its imprimatur on such outrageous proposition.

WHEREFORE, finding no reversible error in the questioned Decision and Resolution, the Court
hereby DENIES the petition. The assailed Decision is AFFIRMED. Moreover, petitioner Bank is
REPRIMANDED for engaging in forum-shopping and WARNED that a repetition of the same or
similar acts will be dealt with more severely. Costs against petitioners.

SO ORDERED.
G.R. No. 157851 June 29, 2007 Private respondent Villanueva avers that she was a regular employee of Countrywide Bank’s
Marbel, South Cotabato branch. On December 7, 1998, she received a memorandum from the
Interim Board of Directors accepting her courtesy resignation. She, however, denies that she
ATTY. ANDREA UY and FELIX YUSAY, Petitioners,
submitted a written courtesy resignation.8
vs.
ARLENE VILLANUEVA and NATIONAL LABOR RELATIONS COMMISSION, Respondents.
On November 16, 1999, Labor Arbiter Arturo P. Gamolo of NLRC Sub-Regional Arbitration
Branch No. XI, General Santos City rendered a Decision in RAB-XI-01-50043-99, the dispositive
DECISION
portion of which reads:

NACHURA, J.:
WHEREFORE, premises considered, respondent Country Wide Rural Bank of La Carlota, Inc.
and Individual Respondents Atty. Andrea Uy and Felix Yusay are solidarily liable to pay
This appeal on certiorari under Rule 45 of the Rules of Court seeks the nullification of the complainant Arlene Villanueva the sum PESOS: ONE HUNDRED THIRTEEN THOUSAND
February 28, 2002 Resolution and the February 27, 2003 Resolution denying the motion for SIX HUNDRED FORTY (₱113,640.00) ONLY representing her monetary awards and attorney’s
reconsideration thereof of the Former Tenth Division of the Court of Appeals (CA) in CA-G.R. SP fees.9
No. 68680.
On January 21, 2000, Villanueva filed a Motion for Execution of Judgment10 to which
The antecedents of the case are as follows: Countrywide Bank, through the PDIC, filed an Opposition. 11

Countrywide Rural Bank of La Carlota, Inc. (Countrywide Bank) is a private banking corporation Thereafter, Labor Arbiter Gamolo rendered a Resolution and Order for all three cases against
engaged in rural banking and other allied services through its branches nationwide. Countrywide Bank, the dispositive portion of which reads:

Sometime in 1998, Countrywide Bank experienced liquidity problems and its treasury Wherefore, finding the PDIC’s opposition to complainants’ motion for execution meritorious,
department was unable to comply with its branches’ demands for fresh funds. Its various complainants are hereby directed to file their respective money claims as adjudged in the
branches eventually experienced bank runs.1 decisions rendered in the above-entitled cases before the liquidation court for the latter’s
approval of inclusion in the Bank’s Distribution Plan.
Several of the bank’s depositors were alarmed at the prospect of losing their deposits and
investments. A group of depositors, holding about 70% of the bank’s deposit accounts, met and SO ORDERED.12
agreed to organize themselves into a "Committee of Depositors." Petitioner Felix Yusay was
elected by the Committee as Chairman of the Interim Board of Directors, while petitioner Atty.
Petitioners then filed a Notice of Appeal with Memorandum of Appeal with the NLRC, 5th
Andrea Uy was designated Secretary. According to petitioners, the Committee was formed for
Division, Cagayan de Oro City.13 On November 27, 2000, the NLRC dismissed the appeal for
the purpose of protecting their collective interests and to increase their chances of recovering
being filed out of time.14 Petitioners filed a motion for reconsideration.15 The NLRC then recalled
their deposits.2
its November 27, 2000 Resolution and set the case for clarificatory hearing. 16

With the consent and approval of the incumbent Board of Directors, the Committee of Depositors
Petitioners, however, received the Resolution five days after the scheduled clarificatory hearing.
assumed temporary administrative control of the remaining operations of the bank.3 The
They instead filed their memorandum in lieu of the clarificatory hearing.
incumbent Board of Directors informed the Committee that some employees had tendered
courtesy resignations, while some had expressed their willingness to resign upon official
request. The Committee then accepted some of the courtesy resignations. 4 On October 10, 2001, the NLRC rendered another Resolution reinstating its November 27, 2000
Resolution.17
The Bangko Sentral ng Pilipinas (BSP) subsequently placed the bank under receivership and
appointed a liquidator. Meanwhile, the Philippine Deposit Insurance System (PDIC) commenced Petitioners filed a petition for certiorari before the CA to nullify the NLRC’s November 27, 2000
the processing of claims for return of deposits.5 and October 10, 2001 Resolutions.

Realizing that their bid to rehabilitate the bank had failed, the Committee of Depositors On February 28, 2002, the Tenth Division of the CA dismissed the petition for certiorari on
disbanded.6 technical grounds. In particular, the CA cited the following grounds for dismissal:

Eventually, three cases for illegal dismissal were filed against Countrywide Bank before the 1. Failure to attach necessary pleadings and comments which are material portion of the records
National Labor Relations Commission (NLRC). These were filed by Amalia Bueno (NLRC Case in able [sic] for this to [sic] judiciously evaluate the merit of the case such as:
No. RAB-XI-01-50037-99), Amelia Valdez and Lyn Villa (NLRC Case No. RAB-XI-01-20039-99),
and herein private respondent Arlene Villanueva (NLRC Case No. RAB-XI-01-50043-99).7
a.) memorandum of appeal filed by the petitioner on May 18, 2000;
b.) Motion for Reconsideration of the petitioners dated December 21, 2000; Secretary was recognized by the Bangko Sentral ng Pilipinas, hence, had no legal authority to
act for the bank.29 As such, the Court said:
in violation of Section 3, Rule 46 of the 1997 Rules of Civil Procedure as amended;
Lacking this evidence, the act of petitioner Uy in dismissing the respondent cannot be deemed
an act as an officer of the bank. Consequently, it cannot be held that there existed an employer-
2. Failure to attach certified photocopy copies [sic] of the assailed resolutions and decisions of
employee relationship between petitioner Uy and respondent Bueno when the former allegedly
the original documents in violation of the same rules; and
dismissed the latter. This requirement of employer-employee relationship is jurisdictional for the
provisions of the Labor Code, specifically Book VI thereof, on Post-Employment, to apply. Since
3. Failure to send copy of the resolution to the public respondent. 18 the employer-employee relationship between petitioner Uy and respondent Bueno was not
established, the labor arbiter never acquired jurisdiction over petitioner Uy. Consequently,
whether petitioner Uy was properly served summons is immaterial. Likewise, that she terminated
Petitioners filed a Motion for Reconsideration19 arguing that the failure to attach the the services of respondent Bueno in bad faith and with malice is of no moment. Her liability, if
abovementioned documents was merely a procedural lapse on their part. They, likewise, any, should be determined in another forum.30
attached the documents to the motion.

The Court noted the manifestation in a Resolution31 dated April 23, 2007.
Their motion for reconsideration having been denied,20 petitioners filed the present appeal on
certiorari.
We find the present petition meritorious.
They argue that the CA’s dismissal of their petition for certiorari on technical grounds deprived
them of substantial justice. They assail the CA’s Resolution dismissing their petition on technical At the outset, we note that Countrywide Bank did not appeal the NLRC’s rulings. As to the bank,
grounds. They cite previous decisions of this Court where it held that technicalities can be therefore, the NLRC Decision has become final and executory.
relaxed in order to uphold the substantive rights of the parties. 21
Rule 45 of the Rules of Civil Procedure provides that only questions of law shall be raised in an
They likewise allege that the Labor Arbiter ruled in favor of respondent Villanueva based only on appeal by certiorari before this Court. This rule, however, admits of certain exceptions, namely,
the pleadings filed by the latter. They allege that they were not properly served summons and (1) when the findings are grounded entirely on speculations, surmises, or conjectures; (2) when
notices which led to their failure to file their position paper. They also argue that they cannot be the inference made is manifestly mistaken, absurd, or impossible; (3) when there is a grave
held solidarily liable to private respondent because they were mere depositors of the bank and abuse of discretion; (4) when the judgment is based on misappreciation of facts; (5) when the
not stockholders. Even assuming that they were stockholders, they still cannot be held findings of fact are conflicting; (6) when in making its findings, the same are contrary to the
individually liable for the bank’s obligations. admissions of both appellant and appellee; (7) when the findings are contrary to those of the trial
court; (8) when the findings are conclusions without citation of specific evidence on which they
are based; (9) when the facts set forth in the petition as well as in the petitioner’s main and reply
On the other hand, private respondent argues that the appeal on certiorari merely reiterated briefs are not disputed by the respondent; and (10) when the findings of fact are premised on the
arguments and issues on questions of facts that have already been passed upon by competent supposed absence of evidence and contradicted by the evidence on record.32
authority.22 Having none of the circumstances that will warrant exemption from the requirement
that a petition for review on certiorari under Rule 45 shall only raise questions of law, the petition
must be dismissed. Likewise, private respondent argues that the petition has no other purpose In this case, the CA committed grave abuse of discretion in dismissing the petition without first
than to delay the final execution of the decision. examining its merits. The policy of our judicial system is to encourage full adjudication of the
merits of an appeal. In the exercise of its equity jurisdiction, this Court may reverse the dismissal
of appeals that are grounded merely on technicalities.33
While this case was pending, petitioners filed a Manifestation23 on February 20, 2007, informing
this Court that the case entitled Atty. Andrea Uy and Felix Yusay v. Amalia Bueno,24 docketed as
G.R. No. 159119 and involving the same factual antecedents as the present case, was decided In the past, the Court has held that technicalities should not be permitted to stand in the way of
by this Court’s Second Division on March 14, 2006 in this wise: equitably and completely resolving the rights and obligations of the parties. Where the ends of
substantial justice would be better served, the application of technical rules of procedure may be
relaxed.34 Rules of procedure should indeed be viewed as mere tools designed to facilitate the
IN VIEW WHEREOF, the petition is GRANTED. The Court of Appeals Decision dated January
attainment of justice.35
24, 2003 and Resolution dated May 26, 2003 in CA-G.R. SP No. 70672, which found petitioner
Atty. Andrea Uy25 solidarily liable with Countrywide Rural Bank of [La] Carlota, Inc. in Marbel,
Koronadal City, South Cotabato, are REVERSED. No costs. Section 1, Rule 65 of the Rules of Court provides:

SO ORDERED.26 SECTION 1. Petition for certiorari. – When any tribunal, board or officer exercising judicial or
quasi-judicial functions has acted without or in excess of its or his jurisdiction, or with grave
abuse of discretion amounting to lack or excess of jurisdiction, and there is no appeal, or any
In the Bueno case, the Court found that, per the records of the case, petitioner Uy was a "mere plain, speedy, and adequate remedy in the ordinary course of law, a person aggrieved thereby
depositor,"27 one of several depositors who formed themselves into a group or association may file a verified petition in the proper court, alleging the facts with certainty and praying that
indicating their intention to help rehabilitate Countrywide Rural Bank.28 It also found no evidence
judgment be rendered annulling or modifying the proceedings of such tribunal, board or officer,
that the Committee of Depositors that elected petitioner Uy as Interim President and Corporate and granting such incidental reliefs as law and justice may require.
The petition shall be accompanied by a certified true copy of the judgment, order or resolution We now proceed to rule on the merits of the case.
subject thereof, copies of all pleadings and documents relevant and pertinent thereto, and a
sworn certification of non-forum shopping as provided in the third paragraph of Section 3, Rule
In order to sustain a finding of illegal dismissal, we must first determine the relationship between
46. (emphasis supplied)
the petitioners and private respondent. Illegal dismissal presupposes that there was an
employer-employee relationship between the dismissed employee and the persons complained
Records show that in the petition for certiorari, filed before the CA, the petitioners attached of.
photocopies of the assailed October 10, 2001 NLRC Resolution,36 the NLRC Resolution dated
November 27, 2000,37 the Labor Arbiter’s Decision dated November 16, 1999,38 and the Labor
To determine whether there was an employer-employee relationship between petitioners and
Arbiter’s Resolution and Order dated April 17, 2000.39 Subsequently, when the CA dismissed the
private respondent, the Court has consistently used the "four-fold" test. The test calls for the
petition on technical grounds, petitioners filed a motion for reconsideration explaining the reason
determination of (1) whether the alleged employer has the power of selection and engagement
for the omission and attaching, in addition to the abovementioned documents, the other
of an employee; (2) whether he has control of the employee with respect to the means and
documents referred to in the CA Resolution.
methods by which work is to be accomplished; (3) whether he has the power to dismiss; and (4)
whether the employee was paid wages. Of the four, the control test is the most important
The Court’s ruling in the case of Garcia v. Philippine Airlines 40 is most instructive, to wit: element.42

It is evident, therefore, that aside from the assailed decision, order or resolution, not every In the instant case, all these elements are attributable to the bank itself and not to petitioners.
pleading or document mentioned in the petition is required to be submitted – only those that are There is no question that private respondent was an employee of the bank. As mentioned
pertinent and relevant to the judgment, order or resolution subject of the petition. The initial above, the NLRC Decision has become final and executory as to the bank. Its liability for private
determination of what pleadings, documents or orders are relevant and pertinent to the petition respondent’s dismissal is no longer in dispute.
rests on the petitioner. If, upon its initial review of the petition, the CA is of the view that
additional pleadings, documents or order should have been submitted and appended to the
The same cannot be said of petitioners. Petitioners assumed only limited administrative control
petition, the following are its options: (a) dismiss the petition under the last paragraph of Rule 46
of the bank as part of the "Committee of Depositors." However, there is no showing that they
of the Rules of Court; (b) order the petitioner to submit the required additional pleadings,
took over the management and control of the bank.
documents, or order within a specific period of time; or (c) order the petitioner to file an amended
petition appending thereto the required pleadings, documents or order within a fixed period.
Given that there is in fact no employer-employee relationship between petitioners and private
respondents, the Labor Arbiter, and consequently, the NLRC, is without jurisdiction to adjudicate
If the CA opts to dismiss the petition outright and the petitioner files a motion for the
the dispute between them. The cases a Labor Arbiter can hear and decide are "employment-
reconsideration of such dismissal, appending thereto the requisite pleadings, documents or
related."43
order/resolution with an explanation for the failure to append the required documents to the
original petition, this would constitute substantial compliance with the Rules of Court. In such
case, then, the petition should be reinstated. As this Court emphasized in Cusi-Hernandez v. Even assuming that an employer-employee relationship does exist between petitioners and
Diaz: private respondent, the former still cannot be held liable with Countrywide Bank for the illegal
dismissal of private respondent. Corporate officers are not personally liable for the money claims
of discharged corporate employees, unless they acted with evident malice and bad faith in
xxxx
terminating their employment.44

We must stress that "cases should be determined on the merits after full opportunity to all
First, we agree with petitioners that they are not corporate officers of the bank.
parties for ventilation of their causes and defenses, rather than on technicality or some
procedural imperfections. In that way, the ends of justice would be served better." Moreover, the
Court has held: It has been held that an "office" is created by the charter of the corporation and the officer is
elected by the directors or stockholders. On the other hand, an "employee" usually occupies no
office and generally is employed not by action of the directors or stockholders but by the
"Dismissal of appeals purely on technical grounds is frowned upon and the rules of procedure
managing officer of the corporation who also determines the compensation to be paid to such
ought not to be applied in a very rigid, technical sense, for they are adopted to help secure, not
employee.45
override, substantial justice, and thereby defeat their very aims."

Given this distinction, petitioners are neither officers nor employees of the bank. They are mere
Rules of procedure are mere tools designed to expedite the decision or resolution of cases and
depositors who sought to manage the bank in order to save it.
other matters pending in court. A strict and rigid application of rules that would result in
technicalities that tend to frustrate rather than promote substantial justice must be avoided.
(citations omitted) Next, settled is the rule in this jurisdiction that a corporation is vested by law with a legal
personality separate and distinct from those acting for and in its behalf and, in general, from the
people comprising it.46
In putting a premium on technical rules over the just resolution of the case, therefore, the CA
overlooked the right of petitioners to the full adjudication of their petition on its merits. Indeed,
while labor laws mandate the speedy administration of justice with least attention to
technicalities, this must be done without sacrificing the fundamental requisites of due process. 41
The general rule is that obligations incurred by the corporation, acting through its directors, WHEREFORE, premises considered, the petition is GRANTED. The February 28, 2002
officers, and employees, are its sole liabilities. However, solidary liability may be incurred, but Resolution in CA-G.R. SP No. 68680 of the Court of Appeals is REVERSED and SET ASIDE.
only under the following exceptional circumstances: The Decision of the Labor Arbiter in RAB-XI-01-50037-99, finding petitioners solidarily liable with
Countrywide Rural Bank of La Carlota is, likewise, REVERSED and SET ASIDE. No
pronouncement as to costs.
1. When directors and trustees or, in appropriate cases, the officers of a corporation: (a) vote for
or assent to patently unlawful acts of the corporation; (b) act in bad faith or with gross negligence
in directing the corporate affairs; (c) are guilty of conflict of interest to the prejudice of the SO ORDERED.
corporation, its stockholders or members, and other persons;

2. When a director or officer has consented to the issuance of watered stocks or who, having
knowledge thereof, did not forthwith file with the corporate secretary his written objection thereto;

3. When a director, trustee or officer has contractually agreed or stipulated to hold himself
personally and solidarily liable with the corporation; or

4. When a director, trustee or officer is made, by specific provision of law, personally liable for
his corporate action.47

Not one of these circumstances is present in this case.

Furthermore, the doctrine of piercing the veil of corporate fiction finds no application in the case.
Piercing the veil of corporate fiction may only be done when "the notion of legal entity is used to
defeat public convenience, justify wrong, protect fraud, or defend crime." 48

The general rule is that a corporation will be looked upon as a separate legal entity, unless and
until sufficient reason to the contrary appears. For the separate juridical personality of a
corporation to be disregarded, the wrongdoing must be clearly and convincingly established. It
cannot be presumed.49 Mere ownership by a single stockholder or by another corporation of all
or nearly all of the capital stock of a corporation is not in itself sufficient ground for disregarding
the separate corporate personality.50

In the case at bar, petitioners are not even stockholders of the bank but mere depositors. That
they assumed temporary control of the bank’s administration did not change the character of
their relationship with the bank. In fact, their bid to convert their interest in the bank to that of
stockholders failed as the BSP denied their plan to rehabilitate the bank.

Finally, we have noted petitioners’ Manifestation51 dated January 31, 2007 and this Court’s
decision in Atty. Andrea Uy and Felix Yusay v. Amalia Bueno.52

In previous cases, the Court has held, "When a court has laid down a principle of law as
applicable to a certain set of facts, it will adhere to that principle and apply it to all future cases in
which the facts are substantially the same. Stare decisis et non quieta movere. Stand by the
decision and disturb not what is settled. It simply means that a conclusion reached in one case
should be applied to those that follow if the facts are substantially the same, even though the
parties may be different. It comes from the basic principle of justice that like cases ought to be
decided alike. Thus, where the same question relating to the same event is brought by parties
similarly situated as in a previous case already litigated and decided by a competent court, the
rule of stare decisis is a bar to any attempt to relitigate the same issue."53

Petitioners’ liability, if there be any, must be determined in the proper action and at the proper
forum.
G.R. No. 170479 February 18, 2008 In trying to recover the amount he paid as down payment for the townhouse unit, respondent
Johnny Ong filed a complaint for Damages before the RTC of Cebu City, docketed as Civil Case
No. CEB-23687, against defendants Andre T. Almocera and FBMC alleging that defendants
ANDRE T. ALMOCERA, petitioner,
were guilty of fraudulent concealment and breach of contract when they sold to him a townhouse
vs.
unit without divulging that the same, at the time of the perfection of their contract, was already
JOHNNY ONG, respondent.
mortgaged with the Land Bank of the Philippines (LBP), with the latter causing the foreclosure of
the mortgage and the eventual sale of the townhouse unit to a third person.
DECISION
In their Answer, defendants denied liability claiming that the foreclosure of the mortgage on the
CHICO-NAZARIO, J.: townhouse unit was caused by the failure of complainant Johnny Ong to pay the balance of the
price of said townhouse unit.
Before Us is a Petition for Review on Certiorari under Rule 45 of the 1997 Rules of Civil
Procedure which seeks to set aside the Decision1 of the Court of Appeals dated 18 July 2005 in After the pre-trial conference was terminated, trial on the merits ensued. Respondent and his
CA-G.R. CV No. 75610 affirming in toto the Decision2 of Branch 11 of the Regional Trial Court brother, Thomas Y. Ong, took the witness stand. For defendants, petitioner testified.
(RTC) of Cebu City in Civil Case No. CEB-23687 and its Resolution3 dated 16 November 2005
denying petitioner’s motion for reconsideration. The RTC decision found petitioner Andre T.
In a Decision dated 20 May 2002, the RTC disposed of the case in this manner:
Almocera, Chairman and Chief Executive Officer of First Builder Multi-Purpose Cooperative
(FBMC), solidarily liable with FMBC for damages.
WHEREFORE, in view of all the foregoing premises, judgment is hereby rendered in
this case in favor of the plaintiff and against the defendants:
Stripped of non-essentials, the respective versions of the parties have been summarized by the
Court of Appeals as follows:
(a) Ordering the defendants to solidarily pay to the plaintiff the sum of P1,060,000.00,
together with a legal interest thereon at 6% per annum from April 21, 1999 until its full
Plaintiff Johnny Ong tried to acquire from the defendants a "townhome" described as
payment before finality of the judgment. Thereafter, if the amount adjudged remains
Unit No. 4 of Atrium Townhomes in Cebu City. As reflected in a Contract to Sell, the
unpaid, the interest rate shall be 12% per annum computed from the time when the
selling price of the unit was P3,400,000.00 pesos, for a lot area of eighty-eight (88)
judgment becomes final and executory until fully satisfied;
square meters with a three-storey building. Out of the purchase price, plaintiff was
able to pay the amount of P1,060,000.00. Prior to the full payment of this amount,
plaintiff claims that defendants Andre Almocera and First Builders fraudulently (b) Ordering the defendants to solidarily pay to the plaintiff the sum of P100,000.00 as
concealed the fact that before and at the time of the perfection of the aforesaid moral damages, the sum of P50,000.00 as attorney’s fee and the sum of P15,619.80
contract to sell, the property was already mortgaged to and encumbered with the Land as expenses of litigation; and
Bank of the Philippines (LBP). In addition, the construction of the house has long been
delayed and remains unfinished. On March 13, 1999, Lot 4-a covered by TCT No.
(c) Ordering the defendants to pay the cost of this suit.5
148818, covering the unit was advertised in a local tabloid for public auction for
foreclosure of mortgage. It is the assertion of the plaintiff that had it not for the
fraudulent concealment of the mortgage and encumbrance by defendants, he would The trial court ruled against defendants for not acting in good faith and for not complying with
have not entered into the contract to sell. their obligations under their contract with respondent. In the Contract to Sell 6 involving Unit 4 of
the Atrium Townhomes, defendants agreed to sell said townhouse to respondent
for P3,400,000.00. The down payment was P1,000,000.00, while the balance of P2,400,000.00
On the other hand, defendants assert that on March 20, 1995, First Builders Multi-
was to be paid in full upon completion, delivery and acceptance of the townhouse. Under the
purpose Coop. Inc., borrowed money in the amount of P500,000.00 from Tommy Ong,
contract which was signed on 10 January 1997, defendants agreed to complete and convey to
plaintiff’s brother. This amount was used to finance the documentation requirements of
respondent the unit within six months from the signing thereof.
the LBP for the funding of the Atrium Town Homes. This loan will be applied in
payment of one (1) town house unit which Tommy Ong may eventually purchase from
the project. When the project was under way, Tommy Ong wanted to buy another The trial court found that respondent was able to make a down payment or partial payment
townhouse for his brother, Johnny Ong, plaintiff herein, which then, the amount of P1,060,000.00 and that the defendants failed to complete the construction of, as well as
of P150,000.00 was given as additional partial payment. However, the particular unit deliver to respondent, the townhouse within six months from the signing of the contract.
was not yet identified. It was only on January 10, 1997 that Tommy Ong identified Unit Moreover, respondent was not informed by the defendants at the time of the perfection of their
No. 4 plaintiff’s chosen unit and again tendered P350,000.00 as his third partial contract that the subject townhouse was already mortgaged to LBP. The mortgage was
payment. When the contract to sell for Unit 4 was being drafted, Tommy Ong foreclosed by the LBP and the townhouse was eventually sold at public auction. It said that
requested that another contract to sell covering Unit 5 be made so as to give Johnny defendants were guilty of fraud in their dealing with respondent because the mortgage was not
Ong another option to choose whichever unit he might decide to have. When the disclosed to respondent when the contract was perfected. There was also non-compliance with
construction was already in full blast, defendants were informed by Tommy Ong that their obligations under the contract when they failed to complete and deliver the townhouse unit
their final choice was Unit 5. It was only upon knowing that the defendants will be at the agreed time. On the part of respondent, the trial court declared he was justified in
selling Unit 4 to some other persons for P4million that plaintiff changed his choice from suspending further payments to the defendants and was entitled to the return of the down
Unit 5 to Unit 4.4 payment.
Aggrieved, defendants appealed the decision to the Court of Appeals assigning the following as III. THE HONORABLE COURT OF APPEALS GRAVELY ERRED IN HOLDING THAT
errors: DEFENDANT ANDRE T. ALMOCERA IS SOLIDARILY LIABLE WITH THE
DEFENDANT COOPERATIVE FOR DAMAGES TO PLAINTIFF.10
1. THE LOWER COURT ERRED IN HOLDING THAT PLAINTIFF HAS A VALID
CAUSE OF ACTION FOR DAMAGES AGAINST DEFENDANT(S). It cannot be disputed that the contract entered into by the parties was a contract to sell. The
contract was denominated as such and it contained the provision that the unit shall be conveyed
by way of an Absolute Deed of Sale, together with the attendant documents of Ownership – the
2. THE LOWER COURT ERRED IN HOLDING THAT DEFENDANT ANDRE T.
Transfer Certificate of Title and Certificate of Occupancy – and that the balance of the contract
ALMOCERA IS SOLIDARILY LIABLE WITH THE COOPERATIVE FOR THE
price shall be paid upon the completion and delivery of the unit, as well as the acceptance
DAMAGES TO THE PLAINTIFF.7
thereof by respondent. All these clearly indicate that ownership of the townhouse has not passed
to respondent.
The Court of Appeals ruled that the defendants incurred delay when they failed to deliver the
townhouse unit to the respondent within six months from the signing of the contract to sell. It
In Serrano v. Caguiat, 11 we explained:
agreed with the finding of the trial court that the nonpayment of the balance of P2.4M by
respondent to defendants was proper in light of such delay and the fact that the property subject
of the case was foreclosed and auctioned. It added that the trial court did not err in giving A contract to sell is akin to a conditional sale where the efficacy or obligatory force of
credence to respondent’s assertion that had he known beforehand that the unit was used as the vendor’s obligation to transfer title is subordinated to the happening of a future and
collateral with the LBP, he would not have proceeded in buying the townhouse. Like the trial uncertain event, so that if the suspensive condition does not take place, the parties
court, the Court of Appeals gave no weight to defendants’ argument that had respondent paid would stand as if the conditional obligation had never existed. The suspensive
the balance of the purchase price of the townhouse, the mortgage could have been released. It condition is commonly full payment of the purchase price.
explained:
The differences between a contract to sell and a contract of sale are well-settled in
We cannot find fault with the choice of plaintiff not to further dole out money for a jurisprudence. As early as 1951, in Sing Yee v. Santos [47 O.G. 6372 (1951)], we held
property that in all events, would never be his. Moreover, defendants could, if they that:
were really desirous of satisfying their obligation, demanded that plaintiff pay the
outstanding balance based on their contract. This they had not done. We can fairly
"x x x [a] distinction must be made between a contract of sale in which title
surmise that defendants could not comply with their obligation themselves, because
passes to the buyer upon delivery of the thing sold and a contract to sell x x
as testified to by Mr. Almocera, they already signified to LBP that they cannot pay their
x where by agreement the ownership is reserved in the seller and is not to
outstanding loan obligations resulting to the foreclosure of the townhouse. 8
pass until the full payment of the purchase price is made. In the first case,
non-payment of the price is a negative resolutory condition; in the second
Moreover, as to the issue of petitioner’s solidary liability, it said that this issue was belatedly case, full payment is a positive suspensive condition. Being contraries, their
raised and cannot be treated for the first time on appeal. effect in law cannot be identical. In the first case, the vendor has lost and
cannot recover the ownership of the land sold until and unless the contract
of sale is itself resolved and set aside. In the second case, however, the title
On 18 July 2005, the Court of Appeals denied the appeal and affirmed in toto the decision of the
remains in the vendor if the vendee does not comply with the condition
trial court. The dispositive portion of the decision reads:
precedent of making payment at the time specified in the contract."

IN LIGHT OF ALL THE FOREGOING, this appeal is DENIED. The assailed decision
In other words, in a contract to sell, ownership is retained by the seller and
of the Regional Trial Court, Branch 11, Cebu City in Civil Case No. CEB-23687
is not to pass to the buyer until full payment of the price.
is AFFIRMED in toto.9

The Contract to Sell entered into by the parties contains the following pertinent provisions:
In a Resolution dated 16 November 2005, the Court of Appeals denied defendants’ motion for
reconsideration.
4. TERMS OF PAYMENT:
Petitioner is now before us pleading his case via a Petition for Review on Certiorari under Rule
45 of the 1997 Rules of Civil Procedure. The petition raises the following issues: 4a. ONE MILLION PESOS (P1,000,000.00) is hereby acknowledged as
Downpayment for the above-mentioned Contract Price.
I. THE HONORABLE COURT OF APPEALS GRAVELY ERRED IN HOLDING THAT
DEFENDANT HAS INCURRED DELAY. 4b. The Balance, in the amount of TWO MILLION FOUR HUNDRED PESOS
(P2,400,000.00) shall be paid thru financing Institution facilitated by the SELLER,
preferably Landbank of the Philippines (LBP).
II. THE HONORABLE COURT OF APPEALS GRAVELY ERRED IN SUSTAINING
RESPONDENT’S REFUSAL TO PAY THE BALANCE OF THE PURCHASE PRICE.
Upon completion, delivery and acceptance of the BUYER of the Townhouse Unit, the
BUYER shall have paid the Contract Price in full to the SELLER.
xxxx comply with his obligation. Where one of the parties to a contract did not perform the
undertaking to which he was bound by the terms of the agreement to perform, he is not entitled
to insist upon the performance of the other party.14
6. COMPLETION DATES OF THE TOWNHOUSE UNIT:

On the first assigned error, petitioner insists there was no delay when the townhouse unit was
The unit shall be completed and conveyed by way of an Absolute Deed of Sale
not completed within six months from the signing of the contract inasmuch as the mere lapse of
together with the attendant documents of Ownership in the name of the BUYER – the
the stipulated six (6) month period is not by itself enough to constitute delay on his part and that
Transfer Certificate of Title and Certificate of Occupancy within a period of six (6)
of FBMC, since the law requires that there must either be judicial or extrajudicial demand to fulfill
months from the signing of Contract to Sell.12
an obligation so that the obligor may be declared in default. He argues there was no evidence
introduced showing that a prior demand was made by respondent before the original action was
From the foregoing provisions, it is clear that petitioner and FBMC had the obligation to complete instituted in the trial court.
the townhouse unit within six months from the signing of the contract. Upon compliance
therewith, the obligation of respondent to pay the balance of P2,400,000.00 arises. Upon
We do not agree.
payment thereof, the townhouse shall be delivered and conveyed to respondent upon the
execution of the Absolute Deed of Sale and other relevant documents.
Demand is not necessary in the instant case. Demand by the respondent would be useless
because the impossibility of complying with their (petitioner and FBMC) obligation was due to
The evidence adduced shows that petitioner and FBMC failed to fulfill their obligation -- to
their fault. If only they paid their loans with the LBP, the mortgage on the subject townhouse
complete and deliver the townhouse within the six-month period. With petitioner and FBMC’s
would not have been foreclosed and thereafter sold to a third person.
non-fulfillment of their obligation, respondent refused to pay the balance of the contract price.
Respondent does not ask that ownership of the townhouse be transferred to him, but merely
asks that the amount or down payment he had made be returned to him. Anent the second assigned error, petitioner argues that if there was any delay, the same was
incurred by respondent because he refused to pay the balance of the contract price.
Article 1169 of the Civil Code reads:
We find his argument specious.
Art. 1169. Those obliged to deliver or to do something incur in delay from the time the
obligee judicially or extrajudicially demands from them the fulfillment of their obligation. As above-discussed, the obligation of respondent to pay the balance of the contract price was
conditioned on petitioner and FBMC’s performance of their obligation. Considering that the latter
did not comply with their obligation to complete and deliver the townhouse unit within the period
However, the demand by the creditor shall not be necessary in order that delay may
agreed upon, respondent could not have incurred delay. For failure of one party to assume and
exist:
perform the obligation imposed on him, the other party does not incur delay.15

(1) When the obligation or the law expressly so declares; or


Under the circumstances obtaining in this case, we find that respondent is justified in refusing to
pay the balance of the contract price. He was never in possession of the townhouse unit and he
(2) When from the nature and the circumstances of the obligation it appears that the can no longer be its owner since ownership thereof has been transferred to a third person who
designation of the time when the thing is to be delivered or the service is to be was not a party to the proceedings below. It would simply be the height of inequity if we are to
rendered was a controlling motive for the establishment of the contract; or require respondent to pay the balance of the contract price. To allow this would result in the
unjust enrichment of petitioner and FBMC. The fundamental doctrine of unjust enrichment is the
transfer of value without just cause or consideration. The elements of this doctrine which are
(3) When demand would be useless, as when the obligor has rendered it beyond his present in this case are: enrichment on the part of the defendant; impoverishment on the part of
power to perform. the plaintiff; and lack of cause. The main objective is to prevent one to enrich himself at the
expense of another. It is commonly accepted that this doctrine simply means a person shall not
In reciprocal obligations, neither party incurs in delay if the other does not comply or is be allowed to profit or enrich himself inequitably at another's expense. 16Hence, to allow
not ready to comply in a proper manner with what is incumbent upon him. From the petitioner and FBMC keep the down payment made by respondent amounting to P1,060,000.00
moment one of the parties fulfills his obligation, delay by the other begins. would result in their unjust enrichment at the expense of the respondent. Thus, said amount
should be returned.
The contract subject of this case contains reciprocal obligations which were to be fulfilled by the
parties, i.e., to complete and deliver the townhouse within six months from the execution of the What is worse is the fact that petitioner and FBMC intentionally failed to inform respondent that
contract to sell on the part of petitioner and FBMC, and to pay the balance of the contract price the subject townhouse which he was going to purchase was already mortgaged to LBP at the
upon completion and delivery of the townhouse on the part of the respondent. time of the perfection of their contract. This deliberate withholding by petitioner and FBMC of the
mortgage constitutes fraud and bad faith. The trial court had this say:
In the case at bar, the obligation of petitioner and FBMC which is to complete and deliver the
townhouse unit within the prescribed period, is determinative of the respondent’s obligation to In the light of the foregoing environmental circumstances and milieu, therefore, it
pay the balance of the contract price. With their failure to fulfill their obligation as stipulated in the appears that the defendants are guilty of fraud in dealing with the plaintiff. They
contract, they incurred delay and are liable for damages.13They cannot insist that respondent performed voluntary and willful acts which prevent the normal realization of the
prestation, knowing the effects which naturally and necessarily arise from such acts.
Their acts import a dishonest purpose or some moral obliquity and conscious doing of
a wrong. The said acts certainly gtive rise to liability for damages (8 Manresa 72;
Borrell-Macia 26-27; 3 Camus 34; O’Leary v. Macondray & Company, 454 Phil. 812;
Heredia v. Salinas, 10 Phil. 157). Article 1170 of the New Civil Code of the Philippines
provides expressly that "those who in the performance of their obligations are guilty of
fraud and those who in any manner contravene the tenor thereof are liable for
damages.17

On the last assigned error, petitioner contends that he should not be held solidarily liable with
defendant FBMC, because the latter is a separate and distinct entity which is the seller of the
subject townhouse. He claims that he, as Chairman and Chief Executive Officer of FBMC,
cannot be held liable because his representing FBMC in its dealings is a corporate act for which
only FBMC should be held liable.

This issue of piercing the veil of corporate fiction was never raised before the trial court. The
same was raised for the first time before the Court of Appeals which ruled that it was too late in
the day to raise the same. The Court of Appeals declared:

In the case below, the pleadings and the evidence of the defendants are one and the
same and never had it made to appear that Almocera is a person distinct and
separate from the other defendant. In fine, we cannot treat this error for the first time
on appeal. We cannot in good conscience, let the defendant Almocera raise the issue
of piercing the veil of corporate fiction just because of the adverse decision against
him. x x x.18

To allow petitioner to pursue such a defense would undermine basic considerations of due
process. Points of law, theories, issues and arguments not brought to the attention of the trial
court will not be and ought not to be considered by a reviewing court, as these cannot be raised
for the first time on appeal. It would be unfair to the adverse party who would have no
opportunity to present further evidence material to the new theory not ventilated before the trial
court.19

As to the award of damages granted by the trial court, and affirmed by the Court of Appeals, we
find the same to be proper and reasonable under the circumstances.

WHEREFORE, the petition is DENIED. The Decision of the Court of Appeals dated 18 July 2005
in CA-G.R. CV No. 75610 is AFFIRMED. Costs against the petitioner.

SO ORDERED.
G.R. No. 174938 October 1, 2014 35. Arbitration

GERARDO LANUZA, JR. AND ANTONIO O. OLBES, Petitioners, (1) Provided always that in case any dispute or difference shall arise between the Owner or the
vs. Project Manager on his behalf and the Contractor, either during the progress or after the
BF CORPORATION, SHANGRI-LA PROPERTIES, INC., ALFREDO C. RAMOS, RUFO B. completion or abandonment of the Works as to the construction of this Contract or as to any
COLAYCO, MAXIMO G. LICAUCO III, AND BENJAMIN C. RAMOS, Respondents. matter or thing of whatsoever nature arising there under or inconnection therewith (including any
matter or thing left by this Contract to the discretion of the Project Manager or the withholding by
the Project Manager of any certificate to which the Contractor may claim to be entitled or the
DECISION
measurement and valuation mentioned in clause 30(5)(a) of these Conditions or the rights and
liabilities of the parties under clauses 25, 26, 32 or 33 of these Conditions), the owner and the
LEONEN, J.: Contractor hereby agree to exert all efforts to settle their differences or dispute amicably. Failing
these efforts then such dispute or difference shall be referred to arbitration in accordance with
the rules and procedures of the Philippine Arbitration Law.
Corporate representatives may be compelled to submit to arbitration proceedings pursuant to a
contract entered into by the corporation they represent if there are allegations of bad faith or
malice in their acts representing the corporation. xxx xxx xxx

This is a Rule 45 petition, assailing the Court of Appeals' May 11, 2006 decision and October 5, (6) The award of such Arbitrators shall be final and binding on the parties. The decision of the
2006 resolution. The Court of Appeals affirmed the trial court's decision holding that petitioners, Arbitrators shall be a condition precedent to any right of legal action that either party may have
as director, should submit themselves as parties tothe arbitration proceedings between BF against the other. . . .12 (Underscoring in the original)
Corporation and Shangri-La Properties, Inc. (Shangri-La).
On August 19, 1993, BF Corporation opposed the motion to suspend proceedings.13
In 1993, BF Corporation filed a collection complaint with the Regional Trial Court against
Shangri-Laand the members of its board of directors: Alfredo C. Ramos, Rufo B.Colayco,
In the November 18, 1993 order, the Regional Trial Court denied the motion to suspend
Antonio O. Olbes, Gerardo Lanuza, Jr., Maximo G. Licauco III, and Benjamin C. Ramos. 1
proceedings.14

BF Corporation alleged in its complaint that on December 11, 1989 and May 30, 1991, it entered
On December 8, 1993, petitioners filed an answer to BF Corporation’s complaint, with
into agreements with Shangri-La wherein it undertook to construct for Shangri-La a mall and a
compulsory counter claim against BF Corporation and crossclaim against Shangri-La.15 They
multilevel parking structure along EDSA.2
alleged that they had resigned as members of Shangri-La’s board of directors as of July 15,
1991.16
Shangri-La had been consistent in paying BF Corporation in accordance with its progress billing
statements.3However, by October 1991, Shangri-La started defaulting in payment.4
After the Regional Trial Court denied on February 11, 1994 the motion for reconsideration of its
November 18, 1993 order, Shangri-La, Alfredo C. Ramos, Rufo B. Colayco,Maximo G. Licauco
BF Corporation alleged that Shangri-La induced BF Corporation to continue with the III, and Benjamin Ramos filed a petition for certiorari with the Court of Appeals.17
construction of the buildings using its own funds and credit despite Shangri-La’s
default.5 According to BF Corporation, ShangriLa misrepresented that it had funds to pay for its
On April 28, 1995, the Court of Appeals granted the petition for certiorari and ordered the
obligations with BF Corporation, and the delay in payment was simply a matter of delayed
submission of the dispute to arbitration.18
processing of BF Corporation’s progress billing statements.6

Aggrieved by the Court of Appeals’ decision, BF Corporation filed a petition for review on
BF Corporation eventually completed the construction of the buildings.7 Shangri-La allegedly
certiorari with this court.19On March 27, 1998, this court affirmed the Court of Appeals’ decision,
took possession of the buildings while still owing BF Corporation an outstanding balance. 8
directing that the dispute be submitted for arbitration.20

BF Corporation alleged that despite repeated demands, Shangri-La refused to pay the balance
Another issue arose after BF Corporation had initiated arbitration proceedings. BF Corporation
owed to it.9 It also alleged that the Shangri-La’s directors were in bad faith in directing Shangri-
and Shangri-La failed to agree as to the law that should govern the arbitration proceedings. 21 On
La’s affairs. Therefore, they should be held jointly and severally liable with Shangri-La for its
October 27, 1998, the trial court issued the order directing the parties to conduct the
obligations as well as for the damages that BF Corporation incurred as a result of Shangri-La’s
proceedings in accordance with Republic Act No. 876. 22
default.10

Shangri-La filed an omnibus motion and BF Corporation an urgent motion for clarification, both
On August 3, 1993, Shangri-La, Alfredo C. Ramos, Rufo B. Colayco, Maximo G. Licauco III, and
seeking to clarify the term, "parties," and whether Shangri-La’s directors should be included in
Benjamin C. Ramos filed a motion to suspend the proceedings in view of BF Corporation’s
the arbitration proceedings and served with separate demands for arbitration. 23
failure to submit its dispute to arbitration, in accordance with the arbitration clauseprovided in its
contract, quoted in the motion as follows:11
Petitioners filed their comment on Shangri-La’s and BF Corporation’s motions, praying that they personally nor did they undertake to shoulder Shangri-La’s obligations should it fail in its
be excluded from the arbitration proceedings for being non-parties to Shangri-La’s and BF obligations.38 BF Corporation also failed to establish fraud or bad faith on their part.39
Corporation’s agreement.24
Petitioners also argue that they are third parties to the contract between BF Corporation and
On July 28, 2003, the trial court issued the order directing service of demands for arbitration Shangri-La.40Provisions including arbitration stipulations should bind only the parties.41 Based on
upon all defendants in BF Corporation’s complaint.25 According to the trial court, Shangri-La’s our arbitration laws, parties who are strangers to an agreement cannot be compelled to
directors were interested parties who "must also be served with a demand for arbitration to give arbitrate.42
them the opportunity to ventilate their side of the controversy, safeguard their interest and fend
off their respective positions."26 Petitioners’ motion for reconsideration ofthis order was denied by
Petitioners point out thatour arbitration laws were enacted to promote the autonomy of parties in
the trial court on January 19, 2005.27
resolving their disputes.43 Compelling them to submit to arbitration is against this purpose and
may be tantamount to stipulating for the parties.44
Petitioners filed a petition for certiorari with the Court of Appeals, alleging grave abuse of
discretion in the issuance of orders compelling them to submit to arbitration proceedings despite
Separate comments on the petition werefiled by BF Corporation, and Maximo G. Licauco III,
being third parties to the contract between Shangri-La and BF Corporation.28
Alfredo C.Ramos and Benjamin C. Ramos.45

In its May 11, 2006 decision,29 the Court of Appeals dismissed petitioners’ petition for certiorari.
Maximo G. Licauco III Alfredo C. Ramos, and Benjamin C. Ramos agreed with petitioners that
The Court of Appeals ruled that ShangriLa’s directors were necessary parties in the arbitration
Shangri-La’sdirectors, being non-parties to the contract, should not be made personally liable for
proceedings.30 According to the Court of Appeals:
Shangri-La’s acts.46 Since the contract was executed only by BF Corporation and Shangri-La,
only they should be affected by the contract’s stipulation.47 BF Corporation also failed to
[They were] deemed not third-parties tothe contract as they [were] sued for their acts in specifically allege the unlawful acts of the directors that should make them solidarily liable with
representation of the party to the contract pursuant to Art. 31 of the Corporation Code, and that Shangri-La for its obligations.48
as directors of the defendant corporation, [they], in accordance with Art. 1217 of the Civil Code,
stand to be benefited or injured by the result of the arbitration proceedings, hence, being
Meanwhile, in its comment, BF Corporation argued that the courts’ ruling that the parties should
necessary parties, they must be joined in order to have complete adjudication of the controversy.
undergo arbitration "clearly contemplated the inclusion of the directors of the corporation[.]" 49 BF
Consequently, if [they were] excluded as parties in the arbitration proceedings and an arbitral
Corporation also argued that while petitioners were not parties to the agreement, they were still
award is rendered, holding [Shangri-La] and its board of directors jointly and solidarily liable to
impleaded under Section 31 of the Corporation Code.50Section 31 makes directors solidarily
private respondent BF Corporation, a problem will arise, i.e., whether petitioners will be bound
liable for fraud, gross negligence, and bad faith.51 Petitioners are not really third parties to the
bysuch arbitral award, and this will prevent complete determination of the issues and resolution
agreement because they are being sued as Shangri-La’s representatives, under Section 31 of
of the controversy.31
the Corporation Code.52

The Court of Appeals further ruled that "excluding petitioners in the arbitration proceedings . . .
BF Corporation further argued that because petitioners were impleaded for their solidary liability,
would be contrary to the policy against multiplicity of suits."32
they are necessary parties to the arbitration proceedings.53 The full resolution of all disputes in
the arbitration proceedings should also be done in the interest of justice. 54
The dispositive portion of the Court of Appeals’ decision reads:
In the manifestation dated September 6, 2007, petitioners informed the court that the Arbitral
WHEREFORE, the petition is DISMISSED. The assailed orders dated July 28, 2003 and Tribunal had already promulgated its decision on July 31, 2007.55 The Arbitral Tribunal denied
January 19, 2005 of public respondent RTC, Branch 157, Pasig City, in Civil Case No. 63400, BF Corporation’s claims against them.56Petitioners stated that "[they] were included by the
are AFFIRMED.33 Arbitral Tribunal in the proceedings conducted . . . notwithstanding [their] continuing objection
thereto. . . ."57 They also stated that "[their] unwilling participation in the arbitration case was
done ex abundante ad cautela, as manifested therein on several occasions."58 Petitioners
The Court of Appeals denied petitioners’ motion for reconsideration in the October 5, 2006
informed the court that they already manifested with the trial court that "any action taken on [the
resolution.34
Arbitral Tribunal’s decision] should be without prejudice to the resolution of [this] case." 59

On November 24, 2006, petitioners filed a petition for review of the May 11, 2006 Court of
Upon the court’s order, petitioners and Shangri-La filed their respective memoranda. Petitioners
Appeals decision and the October 5, 2006 Court of Appeals resolution. 35
and Maximo G. Licauco III, Alfredo C. Ramos, and Benjamin C. Ramos reiterated their
arguments that they should not be held liable for Shangri-La’s default and made parties to the
The issue in this case is whether petitioners should be made parties to the arbitration arbitration proceedings because only BF Corporation and Shangri-La were parties to the
proceedings, pursuant to the arbitration clause provided in the contract between BF Corporation contract.
and Shangri-La.
In its memorandum, Shangri-La argued that petitioners were impleaded for their solidary liability
Petitioners argue that they cannot be held personally liable for corporate acts or under Section 31 of the Corporation Code. Shangri-La added that their exclusion from the
obligations.36 The corporation is a separate being, and nothing justifies BF Corporation’s arbitration proceedings will result in multiplicity of suits, which "is not favored in this
allegation that they are solidarily liable with Shangri-La.37Neither did they bind themselves jurisdiction."60 It pointed out that the case had already been mooted by the termination of the
arbitration proceedings, which petitioners actively participated in. 61 Moreover, BF Corporation all matters in dispute between the parties shall be referred to arbitrators and to them alone, is
assailed only the correctness of the Arbitral Tribunal’s award and not the part absolving Shangri- contrary to public policy and cannot oust the courts of jurisdiction" (Manila Electric Co. vs. Pasay
La’s directors from liability.62 Transportation Co., 57 Phil., 600, 603), however, there are authorities which favor "the more
intelligent view that arbitration, as an inexpensive, speedy and amicable method of settling
disputes, and as a means of avoiding litigation, should receive every encouragement from the
BF Corporation filed a counter-manifestation with motion to dismiss63 in lieu of the required
courts which may be extended without contravening sound public policy or settled law" (3 Am.
memorandum.
Jur., p. 835). Congress has officially adopted the modern view when it reproduced in the new
Civil Code the provisions of the old Code on Arbitration. And only recently it approved Republic
In its counter-manifestation, BF Corporation pointed out that since "petitioners’ counterclaims Act No. 876 expressly authorizing arbitration of future disputes.72 (Emphasis supplied)
were already dismissed with finality, and the claims against them were likewise dismissed with
finality, they no longer have any interest orpersonality in the arbitration case. Thus, there is no
In view of our policy to adopt arbitration as a manner of settling disputes, arbitration clauses are
longer any need to resolve the present Petition, which mainly questions the inclusion of
liberally construed to favor arbitration. Thus, in LM Power Engineering Corporation v. Capitol
petitioners in the arbitration proceedings."64 The court’s decision in this case will no longer have
Industrial Construction Groups, Inc.,73 this court said:
any effect on the issue of petitioners’ inclusion in the arbitration proceedings. 65

Being an inexpensive, speedy and amicable method of settling disputes, arbitration — along with
The petition must fail.
mediation, conciliation and negotiation — is encouraged by the Supreme Court. Aside from
unclogging judicial dockets, arbitration also hastens the resolution of disputes, especially of the
The Arbitral Tribunal’s decision, absolving petitioners from liability, and its binding effect on BF commercial kind. It is thus regarded as the "wave of the future" in international civil and
Corporation, have rendered this case moot and academic. commercial disputes. Brushing aside a contractual agreement calling for arbitration between the
parties would be a step backward.
The mootness of the case, however, had not precluded us from resolving issues so that
principles may be established for the guidance of the bench, bar, and the public. In De la Consistent with the above-mentioned policy of encouraging alternative dispute resolution
Camara v. Hon. Enage,66 this court disregarded the fact that petitioner in that case already methods, courts should liberally construe arbitration clauses. Provided such clause is
escaped from prison and ruled on the issue of excessive bails: susceptible of an interpretation that covers the asserted dispute, an order to arbitrate should be
granted. Any doubt should be resolved in favor of arbitration.74(Emphasis supplied)
While under the circumstances a ruling on the merits of the petition for certiorari is notwarranted,
still, as set forth at the opening of this opinion, the fact that this case is moot and academic A more clear-cut statement of the state policy to encourage arbitration and to favor
should not preclude this Tribunal from setting forth in language clear and unmistakable, the interpretations that would render effective an arbitration clause was later expressed in Republic
obligation of fidelity on the part of lower court judges to the unequivocal command of the Act No. 9285:75
Constitution that excessive bail shall not be required.67
SEC. 2. Declaration of Policy.- It is hereby declared the policy of the State to actively promote
This principle was repeated in subsequent cases when this court deemed it proper to clarify party autonomy in the resolution of disputes or the freedom of the party to make their own
important matters for guidance.68 arrangements to resolve their disputes. Towards this end, the State shall encourage and actively
promote the use of Alternative Dispute Resolution (ADR) as an important means to achieve
speedy and impartial justice and declog court dockets. As such, the State shall provide means
Thus, we rule that petitioners may be compelled to submit to the arbitration proceedings in for the use of ADR as an efficient tool and an alternative procedure for the resolution of
accordance with Shangri-Laand BF Corporation’s agreement, in order to determine if the
appropriate cases. Likewise, the State shall enlist active private sector participation in the
distinction between Shangri-La’s personality and their personalities should be disregarded. settlement of disputes through ADR. This Act shall be without prejudice to the adoption by the
Supreme Court of any ADR system, such as mediation, conciliation, arbitration, or any
This jurisdiction adopts a policy in favor of arbitration. Arbitration allows the parties to avoid combination thereof as a means of achieving speedy and efficient means of resolving cases
litigation and settle disputes amicably and more expeditiously by themselves and through their pending before all courts in the Philippines which shall be governed by such rules as the
choice of arbitrators. Supreme Court may approve from time to time.

The policy in favor of arbitration has been affirmed in our Civil Code, 69 which was approved as ....
early as 1949. It was later institutionalized by the approval of Republic Act No. 876, 70 which
expressly authorized, made valid, enforceable, and irrevocable parties’ decision to submit their
SEC. 25. Interpretation of the Act.- In interpreting the Act, the court shall have due regard to the
controversies, including incidental issues, to arbitration. This court recognized this policy in policy of the law in favor of arbitration.Where action is commenced by or against multiple parties,
Eastboard Navigation, Ltd. v. Ysmael and Company, Inc.:71 one or more of whomare parties who are bound by the arbitration agreement although the civil
action may continue as to those who are not bound by such arbitration agreement. (Emphasis
As a corollary to the question regarding the existence of an arbitration agreement, defendant supplied)
raises the issue that, even if it be granted that it agreed to submit its dispute with plaintiff to
arbitration, said agreement is void and without effect for it amounts to removing said dispute
Thus, if there is an interpretation that would render effective an arbitration clause for purposes
from the jurisdiction of the courts in which the parties are domiciled or where the dispute ofavoiding litigation and expediting resolution of the dispute, that interpretation shall be adopted.
occurred. It is true that there are authorities which hold that "a clause in a contract providing that
Petitioners’ main argument arises from the separate personality given to juridical persons vis-à- 10. To establish pension, retirement, and other plans for the benefit of its directors,
vis their directors, officers, stockholders, and agents. Since they did not sign the arbitration trustees, officers and employees; and
agreement in any capacity, they cannot be forced to submit to the jurisdiction of the Arbitration
Tribunal in accordance with the arbitration agreement. Moreover, they had already resigned as
11. To exercise such other powers asmay be essential or necessary to carry out its
directors of Shangri-Laat the time of the alleged default.
purpose or purposes as stated in its articles of incorporation. (13a)

Indeed, as petitioners point out, their personalities as directors of Shangri-La are separate and
Because a corporation’s existence is only by fiction of law, it can only exercise its rights and
distinct from Shangri-La.
powers through itsdirectors, officers, or agents, who are all natural persons. A corporation
cannot sue or enter into contracts without them.
A corporation is an artificial entity created by fiction of law.76 This means that while it is not a
person, naturally, the law gives it a distinct personality and treats it as such. A corporation, in the
A consequence of a corporation’s separate personality is that consent by a corporation through
legal sense, is an individual with a personality that is distinct and separate from other persons
its representatives is not consent of the representative, personally. Its obligations, incurred
including its stockholders, officers, directors, representatives, 77 and other juridical entities. The
through official acts of its representatives, are its own. A stockholder, director, or representative
law vests in corporations rights,powers, and attributes as if they were natural persons with
does not become a party to a contract just because a corporation executed a contract through
physical existence and capabilities to act on their own.78 For instance, they have the power to
that stockholder, director or representative.
sue and enter into transactions or contracts. Section 36 of the Corporation Code enumerates
some of a corporation’s powers, thus:
Hence, a corporation’s representatives are generally not bound by the terms of the contract
executed by the corporation. They are not personally liable for obligations and liabilities incurred
Section 36. Corporate powers and capacity.– Every corporation incorporated under this Code
on or in behalf of the corporation.
has the power and capacity:

Petitioners are also correct that arbitration promotes the parties’ autonomy in resolving their
1. To sue and be sued in its corporate name;
disputes. This court recognized in Heirs of Augusto Salas, Jr. v. Laperal Realty
Corporation79 that an arbitration clause shall not apply to persons who were neither parties to the
2. Of succession by its corporate name for the period of time stated in the articles of contract nor assignees of previous parties, thus:
incorporation and the certificate ofincorporation;
A submission to arbitration is a contract. As such, the Agreement, containing the stipulation on
3. To adopt and use a corporate seal; arbitration, binds the parties thereto, as well as their assigns and heirs. But only
they.80 (Citations omitted)
4. To amend its articles of incorporation in accordance with the provisions of this
Code; Similarly, in Del Monte Corporation-USA v. Court of Appeals,81 this court ruled:

5. To adopt by-laws, not contrary to law, morals, or public policy, and to amend or The provision to submit to arbitration any dispute arising therefrom and the relationship of the
repeal the same in accordance with this Code; parties is part of that contract and is itself a contract. As a rule, contracts are respected as the
law between the contracting parties and produce effect as between them, their assigns and
heirs. Clearly, only parties to the Agreement . . . are bound by the Agreement and its arbitration
6. In case of stock corporations, to issue or sell stocks to subscribers and to sell
clause as they are the only signatories thereto.82 (Citation omitted)
treasury stocks in accordance with the provisions of this Code; and to admit members
to the corporation if it be a non-stock corporation;
This court incorporated these rulings in Agan, Jr. v. Philippine International Air Terminals Co.,
Inc.83 and Stanfilco Employees v. DOLE Philippines, Inc., et al.84
7. To purchase, receive, take or grant, hold, convey, sell, lease, pledge, mortgage and
otherwise deal with such real and personal property, including securities and bonds of
other corporations, as the transaction of the lawful business of the corporation may As a general rule, therefore, a corporation’s representative who did not personally bind himself
reasonably and necessarily require, subject to the limitations prescribed by law and or herself to an arbitration agreement cannot be forced to participate in arbitration proceedings
the Constitution; made pursuant to an agreement entered into by the corporation. He or she is generally not
considered a party to that agreement.
8. To enter into merger or consolidation with other corporations as provided in this
Code; However, there are instances when the distinction between personalities of directors,
officers,and representatives, and of the corporation, are disregarded. We call this piercing the
veil of corporate fiction.
9. To make reasonable donations, including those for the public welfare or for hospital,
charitable, cultural, scientific, civic, or similar purposes: Provided, That no corporation,
domestic or foreign, shall give donations in aid of any political party or candidate or for Piercing the corporate veil is warranted when "[the separate personality of a corporation] is used
purposes of partisan political activity; as a means to perpetrate fraud or an illegal act, or as a vehicle for the evasion of an existing
obligation, the circumvention of statutes, or to confuse legitimate issues." 85 It is also warranted in When there are allegations of bad faith or malice against corporate directors or representatives,
alter ego cases "where a corporation is merely a farce since it is a mere alter ego or business it becomes the duty of courts or tribunals to determine if these persons and the corporation
conduit of a person, or where the corporation is so organized and controlled and its affairs are so should be treated as one. Without a trial, courts and tribunals have no basis for determining
conducted as to make it merely an instrumentality, agency, conduit or adjunct of another whether the veil of corporate fiction should be pierced. Courts or tribunals do not have such prior
corporation."86 knowledge. Thus, the courts or tribunals must first determine whether circumstances exist
towarrant the courts or tribunals to disregard the distinction between the corporation and the
persons representing it. The determination of these circumstances must be made by one tribunal
When corporate veil is pierced, the corporation and persons who are normally treated as distinct
or court in a proceeding participated in by all parties involved, including current representatives
from the corporation are treated as one person, such that when the corporation is adjudged
of the corporation, and those persons whose personalities are impliedly the sameas the
liable, these persons, too, become liable as if they were the corporation.
corporation. This is because when the court or tribunal finds that circumstances exist warranting
the piercing of the corporate veil, the corporate representatives are treated as the corporation
Among the persons who may be treatedas the corporation itself under certain circumstances are itself and should be held liable for corporate acts. The corporation’s distinct personality is
its directors and officers. Section 31 of the Corporation Code provides the instances when disregarded, and the corporation is seen as a mere aggregation of persons undertaking a
directors, trustees, or officers may become liable for corporate acts: business under the collective name of the corporation.

Sec. 31. Liability of directors, trustees or officers. - Directors or trustees who willfully and Hence, when the directors, as in this case, are impleaded in a case against a corporation,
knowingly vote for or assent to patently unlawful acts of the corporation or who are guilty of alleging malice orbad faith on their part in directing the affairs of the corporation, complainants
gross negligence or bad faith in directing the affairs of the corporation or acquire any personal or are effectively alleging that the directors and the corporation are not acting as separate entities.
pecuniary interest in conflict with their duty as such directors or trustees shall be liable jointly and They are alleging that the acts or omissions by the corporation that violated their rights are also
severally for all damages resulting therefrom suffered by the corporation, its stockholders or the directors’ acts or omissions.90 They are alleging that contracts executed by the corporation
members and other persons. are contracts executed by the directors. Complainants effectively pray that the corporate veilbe
pierced because the cause of action between the corporation and the directors is the same.
When a director, trustee or officer attempts to acquire or acquires, in violation of his duty, any
interest adverse to the corporation in respect of any matter which has been reposed inhim in In that case, complainants have no choice but to institute only one proceeding against the
confidence, as to which equity imposes a disability upon him to deal in his own behalf, he shall parties. Under the Rules of Court, filing of multiple suits for a single cause of action is prohibited.
be liable as a trustee for the corporation and must account for the profits which otherwise would Institution of more than one suit for the same cause of action constitutes splitting the cause of
have accrued to the corporation. (n) action, which is a ground for the dismissal ofthe others. Thus, in Rule 2:

Based on the above provision, a director, trustee, or officer of a corporation may be made Section 3. One suit for a single cause of action. — A party may not institute more than one suit
solidarily liable with it for all damages suffered by the corporation, its stockholders or members, for a single cause of action. (3a)
and other persons in any of the following cases:
Section 4. Splitting a single cause of action;effect of. — If two or more suits are instituted on the
a) The director or trustee willfully and knowingly voted for or assented to a patently basis of the same cause of action, the filing of one or a judgment upon the merits in any one is
unlawful corporate act; available as a ground for the dismissal of the others. (4a)

b) The director or trustee was guilty of gross negligence or bad faith in directing It is because the personalities of petitioners and the corporation may later be found to be
corporate affairs; and indistinct that we rule that petitioners may be compelled to submit to arbitration.

c) The director or trustee acquired personal or pecuniary interest in conflict with his or However, in ruling that petitioners may be compelled to submit to the arbitration proceedings, we
her duties as director or trustee. are not overturning Heirs of Augusto Salas wherein this court affirmed the basic arbitration
principle that only parties to an arbitration agreement may be compelled to submit to arbitration.
In that case, this court recognizedthat persons other than the main party may be compelled to
Solidary liability with the corporation will also attach in the following instances:
submit to arbitration, e.g., assignees and heirs. Assignees and heirs may be considered parties
to an arbitration agreement entered into by their assignor because the assignor’s rights and
a) "When a director or officer has consented to the issuance of watered stocks or who, obligations are transferred to them upon assignment. In other words, the assignor’s rights and
having knowledge thereof, did not forthwith file with the corporate secretary his written obligations become their own rights and obligations. In the same way, the corporation’s
objection thereto";87 obligations are treated as the representative’s obligations when the corporate veil is pierced.
Moreover, in Heirs of Augusto Salas, this court affirmed its policy against multiplicity of suits and
unnecessary delay. This court said that "to split the proceeding into arbitration for some parties
b) "When a director, trustee or officer has contractually agreed or stipulated to hold and trial for other parties would "result in multiplicity of suits, duplicitous procedure and
himself personally and solidarily liable with the corporation"; 88 and unnecessary delay."91 This court also intimated that the interest of justice would be best
observed if it adjudicated rights in a single proceeding.92 While the facts of that case prompted
c) "When a director, trustee or officer is made, by specific provision of law, personally this court to direct the trial court to proceed to determine the issues of thatcase, it did not prohibit
liable for his corporate action."89 courts from allowing the case to proceed to arbitration, when circumstances warrant.
Hence, the issue of whether the corporation’s acts in violation of complainant’s rights, and the
incidental issue of whether piercing of the corporate veil is warranted, should be determined in a
single proceeding. Such finding would determine if the corporation is merely an aggregation of
persons whose liabilities must be treated as one with the corporation.

However, when the courts disregard the corporation’s distinct and separate personality from its
directors or officers, the courts do not say that the corporation, in all instances and for all
purposes, is the same as its directors, stockholders, officers, and agents. It does not result in an
absolute confusion of personalities of the corporation and the persons composing or
representing it. Courts merely discount the distinction and treat them as one, in relation to a
specific act, in order to extend the terms of the contract and the liabilities for all damages to
erring corporate officials who participated in the corporation’s illegal acts. This is done so that
the legal fiction cannot be used to perpetrate illegalities and injustices.

Thus, in cases alleging solidary liability with the corporation or praying for the piercing of the
corporate veil, parties who are normally treated as distinct individuals should be made to
participate in the arbitration proceedings in order to determine ifsuch distinction should indeed
be disregarded and, if so, to determine the extent of their liabilities.

In this case, the Arbitral Tribunal rendered a decision, finding that BF Corporation failed to prove
the existence of circumstances that render petitioners and the other directors solidarily liable. It
ruled that petitioners and Shangri-La’s other directors were not liable for the contractual
obligations of Shangri-La to BF Corporation. The Arbitral Tribunal’s decision was made with the
participation of petitioners, albeit with their continuing objection. In view of our discussion above,
we rule that petitioners are bound by such decision.

WHEREFORE, the petition is DENIED. The Court of Appeals' decision of May 11, 2006 and
resolution of October 5, 2006 are AFFIRMED.

SO ORDERED.
G.R. No. 108734 May 29, 1996 On November 27, 1985, the National Labor Relations Commission (NLRC) dismissed the motion
for reconsideration filed by petitioner on the ground that the said decision had already become
final and executory.2
CONCEPT BUILDERS, INC., petitioner,
vs.
THE NATIONAL LABOR RELATIONS COMMISSION, (First Division); and Norberto Marabe; On October 16, 1986, the NLRC Research and Information Department made the finding that
Rodolfo Raquel, Cristobal Riego, Manuel Gillego, Palcronio Giducos, Pedro Aboigar, private respondents' back wages amounted to P199,800.00.3
Norberto Comendador, Rogelio Salut, Emilio Garcia, Jr., Mariano Rio, Paulina Basea,
Alfredo Albera, Paquito Salut, Domingo Guarino, Romeo Galve, Dominador Sabina, Felipe
On October 29, 1986, the Labor Arbiter issued a writ of execution directing the sheriff to execute
Radiana, Gavino Sualibio, Moreno Escares, Ferdinand Torres, Felipe Basilan, and Ruben
the Decision, dated December 19, 1984. The writ was partially satisfied through garnishment of
Robalos, respondents.
sums from petitioner's debtor, the Metropolitan Waterworks and Sewerage Authority, in the
amount of P81,385.34. Said amount was turned over to the cashier of the NLRC.
HERMOSISIMA, JR., J.:
On February 1, 1989, an Alias Writ of Execution was issued by the Labor Arbiter directing the
The corporate mask may be lifted and the corporate veil may be pierced when a corporation is sheriff to collect from herein petitioner the sum of P117,414.76, representing the balance of the
just but the alter ego of a person or of another corporation. Where badges of fraud exist; where judgment award, and to reinstate private respondents to their former positions.
public convenience is defeated; where a wrong is sought to be justified thereby, the corporate
fiction or the notion of legal entity should come to naught. The law in these instances will regard
On July 13, 1989, the sheriff issued a report stating that he tried to serve the alias writ of
the corporation as a mere association of persons and, in case of two corporations, merge them
execution on petitioner through the security guard on duty but the service was refused on the
into one.
ground that petitioner no longer occupied the premises.

Thus, where a sister corporation is used as a shield to evade a corporation's subsidiary liability
On September 26, 1986, upon motion of private respondents, the Labor Arbiter issued a second
for damages, the corporation may not be heard to say that it has a personality separate and
alias writ of execution.
distinct from the other corporation. The piercing of the corporate veil comes into play.

The said writ had not been enforced by the special sheriff because, as stated in his progress
This special civil action ostensibly raises the question of whether the National Labor Relations
report, dated November 2, 1989:
Commission committed grave abuse of discretion when it issued a "break-open order" to the
sheriff to be enforced against personal property found in the premises of petitioner's sister
company. 1. All the employees inside petitioner's premises at 355 Maysan Road, Valenzuela, Metro
Manila, claimed that they were employees of Hydro Pipes Philippines, Inc. (HPPI) and not by
respondent;
Petitioner Concept Builders, Inc., a domestic corporation, with principal office at 355 Maysan
Road, Valenzuela, Metro Manila, is engaged in the construction business. Private respondents
were employed by said company as laborers, carpenters and riggers. 2. Levy was made upon personal properties he found in the premises;

On November, 1981, private respondents were served individual written notices of termination of 3. Security guards with high-powered guns prevented him from removing the properties he had
employment by petitioner, effective on November 30, 1981. It was stated in the individual notices levied upon.4
that their contracts of employment had expired and the project in which they were hired had
been completed.
The said special sheriff recommended that a "break-open order" be issued to enable him to
enter petitioner's premises so that he could proceed with the public auction sale of the aforesaid
Public respondent found it to be, the fact, however, that at the time of the termination of private personal properties on November 7, 1989.
respondent's employment, the project in which they were hired had not yet been finished and
completed. Petitioner had to engage the services of sub-contractors whose workers performed
the functions of private respondents. On November 6, 1989, a certain Dennis Cuyegkeng filed a third-party claim with the Labor
Arbiter alleging that the properties sought to be levied upon by the sheriff were owned by Hydro
(Phils.), Inc. (HPPI) of which he is the Vice-President.
Aggrieved, private respondents filed a complaint for illegal dismissal, unfair labor practice and
non-payment of their legal holiday pay, overtime pay and thirteenth-month pay against petitioner.
On November 23, 1989, private respondents filed a "Motion for Issuance of a Break-Open
Order," alleging that HPPI and petitioner corporation were owned by the same
On December 19, 1984, the Labor Arbiter rendered judgment 1 ordering petitioner to reinstate incorporator/stockholders. They also alleged that petitioner temporarily suspended its business
private respondents and to pay them back wages equivalent to one year or three hundred operations in order to evade its legal obligations to them and that private respondents were
working days. willing to post an indemnity bond to answer for any damages which petitioner and HPPI may
suffer because of the issuance of the break-open order.
In support of their claim against HPPI, private respondents presented duly certified copies of the petitioner. HPPI also alleged that the two corporations are engaged in two different kinds of
General Informations Sheet, dated May 15, 1987, submitted by petitioner to the Securities businesses, i.e., HPPI is a manufacturing firm while petitioner was then engaged in construction.
Exchange Commission (SEC) and the General Information Sheet, dated May 25, 1987,
submitted by HPPI to the Securities and Exchange Commission.
On March 2, 1990, the Labor Arbiter issued an Order which denied private respondents' motion
for break-open order.
The General Information Sheet submitted by the petitioner revealed the following:
Private respondents then appealed to the NLRC. On April 23, 1992, the NLRC set aside the
1. Breakdown of Subscribed Capital
order of the Labor Arbiter, issued a break-open order and directed private respondents to file a
Name of Stockholder Amount Subscribed
bond. Thereafter, it directed the sheriff to proceed with the auction sale of the properties already
HPPI P 6,999,500.00
levied upon. It dismissed the third-party claim for lack of merit.
Antonio W. Lim 2,900,000.00
Dennis S. Cuyegkeng 300.00
Elisa C. Lim 100,000.00 Petitioner moved for reconsideration but the motion was denied by the NLRC in a Resolution,
Teodulo R. Dino 100.00 dated December 3, 1992.
Virgilio O. Casino 100.00
2. Board of Directors
Hence, the resort to the present petition.
Antonio W. Lim Chairman
Dennis S. Cuyegkeng Member
Elisa C. Lim Member Petitioner alleges that the NLRC committed grave abuse of discretion when it ordered the
Teodulo R. Dino Member execution of its decision despite a third-party claim on the levied property. Petitioner further
Virgilio O. Casino Member contends, that the doctrine of piercing the corporate veil should not have been applied, in this
3. Corporate Officers case, in the absence of any showing that it created HPPI in order to evade its liability to private
Antonio W. Lim President respondents. It also contends that HPPI is engaged in the manufacture and sale of steel,
Dennis S. Cuyegkeng Assistant to the President concrete and iron pipes, a business which is distinct and separate from petitioner's construction
Elisa O. Lim Treasurer business. Hence, it is of no consequence that petitioner and HPPI shared the same premises,
Virgilio O. Casino Corporate Secretary the same President and the same set of officers and subscribers. 7
4. Principal Office
355 Maysan Road
Valenzuela, Metro Manila.5 We find petitioner's contention to be unmeritorious.

On the other hand, the General Information Sheet of HPPI revealed the following: It is a fundamental principle of corporation law that a corporation is an entity separate and
distinct from its stockholders and from other corporations to which it may be connected.8 But,
1. Breakdown of Subscribed Capital this separate and distinct personality of a corporation is merely a fiction created by law for
Name of Stockholder Amount Subscribed convenience and to promote justice.9 So, when the notion of separate juridical personality is
Antonio W. Lim P 400,000.00 used to defeat public convenience, justify wrong, protect fraud or defend crime, or is used as a
Elisa C. Lim 57,700.00 device to defeat the labor laws,10 this separate personality of the corporation may be
AWL Trading 455,000.00 disregarded or the veil of corporate fiction pierced.11 This is true likewise when the corporation is
Dennis S. Cuyegkeng 40,100.00 merely an adjunct, a business conduit or an alter ego of another corporation. 12
Teodulo R. Dino 100.00
Virgilio O. Casino 100.00
2. Board of Directors The conditions under which the juridical entity may be disregarded vary according to the peculiar
Antonio W. Lim Chairman facts and circumstances of each case. No hard and fast rule can be accurately laid down, but
Elisa C. Lim Member certainly, there are some probative factors of identity that will justify the application of the
Dennis S. Cuyegkeng Member doctrine of piercing the corporate veil, to wit:
Virgilio O. Casino Member
Teodulo R. Dino Member 1. Stock ownership by one or common ownership of both corporations.
3. Corporate Officers
Antonio W. Lim President
Dennis S. Cuyegkeng Assistant to the President 2. Identity of directors and officers.
Elisa C. Lim Treasurer
Virgilio O. Casino Corporate Secretary 3. The manner of keeping corporate books and records.
4. Principal Office
355 Maysan Road, Valenzuela, Metro Manila.6
4. Methods of conducting the business.13
On February 1, 1990, HPPI filed an Opposition to private respondents' motion for issuance of a
break-open order, contending that HPPI is a corporation which is separate and distinct from The SEC en banc explained the "instrumentality rule" which the courts have applied in
disregarding the separate juridical personality of corporations as follows:
Where one corporation is so organized and controlled and its affairs are and/or premises. Under this circumstances, (sic) it cannot be said that the
conducted so that it is, in fact, a mere instrumentality or adjunct of the other, property levied upon by the sheriff were not of respondents.16
the fiction of the corporate entity of the "instrumentality" may be
disregarded. The control necessary to invoke the rule is not majority or even
Clearly, petitioner ceased its business operations in order to evade the payment to private
complete stock control but such domination of instances, policies and
respondents of back wages and to bar their reinstatement to their former positions. HPPI is
practices that the controlled corporation has, so to speak, no separate mind,
obviously a business conduit of petitioner corporation and its emergence was skillfully
will or existence of its own, and is but a conduit for its principal. It must be
orchestrated to avoid the financial liability that already attached to petitioner corporation.
kept in mind that the control must be shown to have been exercised at the
time the acts complained of took place. Moreover, the control and breach of
duty must proximately cause the injury or unjust loss for which the complaint The facts in this case are analogous to Claparols v. Court of Industrial Relations, 17 where we
is made. had the occasion to rule:

The test in determining the applicability of the doctrine of piercing the veil of corporate fiction is Respondent court's findings that indeed the Claparols Steel and Nail Plant,
as follows: which ceased operation of June 30, 1957, was SUCCEEDED by the
Claparols Steel Corporation effective the next day, July 1, 1957, up to
December 7, 1962, when the latter finally ceased to operate, were not
1. Control, not mere majority or complete stock control, but complete
disputed by petitioner. It is very clear that the latter corporation was a
domination, not only of finances but of policy and business practice in
continuation and successor of the first entity . . . . Both predecessors and
respect to the transaction attacked so that the corporate entity as to this
successor were owned and controlled by petitioner Eduardo Claparols and
transaction had at the time no separate mind, will or existence of its own;
there was no break in the succession and continuity of the same business.
This "avoiding-the-liability" scheme is very patent, considering that 90% of
2. Such control must have been used by the defendant to commit fraud or the subscribed shares of stock of the Claparols Steel Corporation (the
wrong, to perpetuate the violation of a statutory or other positive legal duty second corporation) was owned by respondent . . . Claparols himself, and
or dishonest and unjust act in contravention of plaintiff's legal rights; and all the assets of the dissolved Claparols Steel and Nail plant were turned
over to the emerging Claparols Steel Corporation.
3. The aforesaid control and breach of duty must proximately cause the
injury or unjust loss complained of. It is very obvious that the second corporation seeks the protective shield of
a corporate fiction whose veil in the present case could, and should, be
pierced as it was deliberately and maliciously designed to evade its financial
The absence of any one of these elements prevents "piercing the corporate
obligation to its employees.
veil." In applying the "instrumentality" or "alter ego" doctrine, the courts are
concerned with reality and not form, with how the corporation operated and
the individual defendant's relationship to that operation.14 In view of the failure of the sheriff, in the case at bar, to effect a levy upon the property subject of
the execution, private respondents had no other recourse but to apply for a break-open order
after the third-party claim of HPPI was dismissed for lack of merit by the NLRC. This is in
Thus the question of whether a corporation is a mere alter ego, a mere sheet or paper
consonance with Section 3, Rule VII of the NLRC Manual of Execution of Judgment which
corporation, a sham or a subterfuge is purely one of fact. 15
provides that:

In this case, the NLRC noted that, while petitioner claimed that it ceased its business operations
Should the losing party, his agent or representative, refuse or prohibit the
on April 29, 1986, it filed an Information Sheet with the Securities and Exchange Commission on
Sheriff or his representative entry to the place where the property subject of
May 15, 1987, stating that its office address is at 355 Maysan Road, Valenzuela, Metro Manila.
execution is located or kept, the judgment creditor may apply to the
On the other hand, HPPI, the third-party claimant, submitted on the same day, a similar
Commission or Labor Arbiter concerned for a break-open order.
information sheet stating that its office address is at 355 Maysan Road, Valenzuela, Metro
Manila.
Furthermore, our perusal of the records shows that the twin requirements of due notice and
hearing were complied with. Petitioner and the third-party claimant were given the opportunity to
Furthermore, the NLRC stated that:
submit evidence in support of their claim.

Both information sheets were filed by the same Virgilio O. Casiño as the
Hence, the NLRC did not commit any grave abuse of discretion when it affirmed the break-open
corporate secretary of both corporations. It would also not be amiss to note
order issued by the Labor Arbiter.
that both corporations had the same president, the same board of directors,
the same corporate officers, and substantially the same subscribers.
Finally, we do not find any reason to disturb the rule that factual findings of quasi-judicial
agencies supported by substantial evidence are binding on this Court and are entitled to great
From the foregoing, it appears that, among other things, the respondent
respect, in the absence of showing of grave abuse of a discretion. 18
(herein petitioner) and the third-party claimant shared the same address
WHEREFORE, the petition is DISMISSED and the assailed resolutions of the NLRC, dated April
23, 1992 and December 3, 1992, are AFFIRMED.

SO ORDERED.
G.R. No. 182770 September 17, 2014 to introducing Manlapaz to Engineer Carmelo Neri (Neri), CLN’s general manager; that it was
actually Manlapaz and Neri who agreed on the terms and conditions of the agreement; that
when the complaint for damages was filed against her, she was abroad; and that she did not
WPM INTERNATIONAL TRADING, INC. and WARLITO P. MANLAPAZ, Petitioners,
know of the case until she returned to the Philippines and received a copy of the decision of the
vs.
RTC.
FE CORAZON LABAYEN, Respondent.

In her prayer, the respondent sought indemnification in the amount of ₱112,876.60 plus interest
DECISION
at 12%per annum from June 18, 1990 until fully paid; and 20% of the award as attorney’s fees.
She likewise prayed that an award of ₱100,000.00 as moral damages and ₱20,000.00 as
BRION, J.: attorney’s fees be paid to her.

We review in this petition for review on certiorari1 the decision2 dated September 28, 2007 and In his defense, Manlapaz claims that it was his fellow incorporator/director Edgar Alcansajewho
the resolution3 dated April 28, 2008 of the Court of Appeals (CA) in CA-G.R. CV No. 68289 that was in-charge with the daily operations of the Quickbite outlets; that when Alcansaje left WPM,
affirmed with modification the decision4 of the Regional Trial Court (RTC), Branch 77, Quezon the remaining directors were compelled to hire the respondent as manager; that the respondent
City. had entered intothe renovation agreement with CLN in her own personal capacity; that when he
found the amount quoted by CLN too high, he instructed the respondent to either renegotiate for
a lower price or to look for another contractor; that since the respondent had exceeded her
The Factual Background authority as agent of WPM, the renovation agreement should only bind her; and that since WPM
has a separate and distinct personality, Manlapaz cannot be made liable for the respondent’s
The respondent, Fe Corazon Labayen, is the owner of H.B.O. Systems Consultants, a claim.
management and consultant firm. The petitioner, WPM International Trading, Inc. (WPM), is a
domestic corporation engaged in the restaurant business, while Warlito P. Manlapaz (Manlapaz) Manlapaz prayed for the dismissal of the complaint for lack of cause of action, and by way of
is its president.
counterclaim, for the award of ₱350,000.00 as moral and exemplary damages and ₱50,000.00
attorney’s fees.
Sometime in 1990, WPM entered into a management agreement with the respondent, by virtue
of which the respondent was authorized to operate, manage and rehabilitate Quickbite, a
The RTC, through an order dated March 2, 1993 declared WPM in default for its failure to file a
restaurant owned and operated by WPM. As part of her tasks, the respondent looked for a responsive pleading.
contractor who would renovate the two existing Quickbite outlets in Divisoria, Manila and
Lepanto St., University Belt, Manila. Pursuant to the agreement, the respondent engaged the
services of CLN Engineering Services (CLN) to renovate Quickbite-Divisoria at the cost of The Decision of the RTC
₱432,876.02.
In its decision, the RTC held that the respondent is entitled to indemnity from Manlapaz. The
On June 13, 1990, Quickbite-Divisoria’s renovation was finally completed, and its possession RTC found that based on the records, there is a clear indication that WPM is a mere
was delivered to the respondent. However, out of the ₱432,876.02 renovation cost, only the instrumentality or business conduit of Manlapaz and as such, WPM and Manlapaz are
amount of ₱320,000.00 was paid to CLN, leaving a balance of ₱112,876.02. considered one and the same. The RTC also found that Manlapaz had complete control over
WPM considering that he is its chairman, president and treasurer at the same time. The RTC
thus concluded that Manlapaz is liable in his personal capacity to reimburse the respondent the
Complaint for Sum of Money (Civil Case No. Q-90-7013) amount she paid to CLN inconnection with the renovation agreement.

On October 19, 1990, CLN filed a complaint for sum of money and damages before the RTC The petitioners appealed the RTC decision with the CA. There, they argued that in view of the
against the respondent and Manlapaz, which was docketed as Civil Case No. Q-90-7013. CLN
respondent’s act of entering into a renovation agreement with CLN in excess of her authority as
later amended the complaint to exclude Manlapaz as defendant. The respondent was declared WPM’s agent, she is not entitled to indemnity for the amount she paid. Manlapaz also contended
in default for her failure to file a responsive pleading. that by virtue ofWPM’s separate and distinct personality, he cannot be madesolidarily liable with
WPM.
The RTC, in its January 28, 1991 decision, found the respondent liable to pay CLN actual
damages inthe amount of ₱112,876.02 with 12% interest per annum from June 18,1990 (the The Ruling of the Court of Appeals
date of first demand) and 20% of the amount recoverable as attorney’s fees.

On September 28, 2007, the CA affirmed, with modification on the award of attorney’s fees, the
Complaint for Damages (Civil Case No. Q-92-13446) decision of the RTC.The CA held that the petitioners are barred from raising as a defense the
respondent’s alleged lack of authority to enter into the renovation agreement in view of their tacit
Thereafter, the respondent instituted a complaint for damages against the petitioners, WPM and ratification of the contract.
Manlapaz. The respondent alleged that in Civil Case No. Q-90-7013, she was adjudged liable for
a contract that she entered into for and in behalf of the petitioners, to which she should be
entitled to reimbursement; that her participation in the management agreement was limited only
The CA likewise affirmed the RTC ruling that WPM and Manlapaz are one and the same based On the Application ofthe Principle of Piercing the Veil of Corporate Fiction
on the following: (1) Manlapaz is the principal stockholder of WPM; (2) Manlapaz had complete
control over WPM because he concurrently held the positions of president, chairman of the
The rule is settled that a corporation has a personality separate and distinct from the persons
board and treasurer, in violation of the Corporation Code; (3) two of the four other stockholders
acting for and in its behalf and, in general, from the people comprising it. 9 Following this
of WPM are employed by Manlapaz either directly or indirectly; (4) Manlapaz’s residence is the
principle, the obligations incurred by the corporate officers, orother persons acting as corporate
registered principal office of WPM; and (5) the acronym "WPM" was derived from Manlapaz’s
agents, are the direct accountabilities ofthe corporation they represent, and not theirs. Thus, a
initials. The CA applied the principle of piercing the veil of corporate fiction and agreed with the
director, officer or employee of a corporation is generally not held personally liable for obligations
RTC that Manlapaz cannot evade his liability by simply invoking WPM’s separate and distinct
incurred by the corporation;10 it is only in exceptional circumstances that solidary liability will
personality.
attach to them.

After the CA's denial of their motion for reconsideration, the petitioners filed the present petition
Incidentally, the doctrine of piercing the corporate veil applies only in three (3) basic instances,
for review on certiorari under Rule 45 of the Rules of Court.
namely: a) when the separate and distinct corporate personality defeats public convenience, as
when the corporate fiction is used as a vehicle for the evasion of an existing obligation; b) in
The Petition fraud cases, or when the corporate entity is used to justify a wrong, protect a fraud, or defend a
crime; or c) is used in alter ego cases, i.e., where a corporation is essentially a farce, since it is a
mere alter ego or business conduit of a person, or where the corporation is so organized and
The petitioners submit that the CA gravely erred in sustaining the RTC’s application of the
controlled and its affairs so conducted as to make it merely aninstrumentality, agency, conduit or
principle of piercing the veil of corporate fiction. They argue that the legal fiction of corporate
adjunct of another corporation.11
personality could only be discarded upon clear and convincing proof that the corporation is being
used as a shield to avoid liability or to commit a fraud. Since the respondent failed to establish
that any of the circumstances that would warrant the piercing is present, Manlapaz claims that Piercing the corporate veil based on the alter ego theory requires the concurrence of three
he cannot be made solidarily liable with WPM to answerfor damages allegedly incurred by the elements, namely:
respondent.
(1) Control, not mere majority or complete stock control, but complete domination, not only of
The petitioners further argue that, assuming they may be held liable to reimburse to the finances but of policy and business practice in respect to the transaction attacked so that the
respondentthe amount she paid in Civil Case No. Q-90-7013, such liability is only limited to the corporate entity as to this transaction had at the time no separate mind, will or existence of its
amount of ₱112,876.02, representing the balance of the obligation to CLN, and should not own;
include the twelve 12% percent interest, damages and attorney’s fees.
(2) Such control must have beenused by the defendant to commit fraud or wrong, to perpetuate
The Issues the violation of a statutory or other positive legal duty, or dishonest and unjust act in
contravention of plaintiff’s legal right; and
The core issues are: (1) whether WPM is a mere instrumentality, alter-ego, and business conduit
of Manlapaz; and (2) whether Manlapaz is jointly and severally liable with WPM to the (3) The aforesaid control and breach of duty must have proximately caused the injury or unjust
respondent for reimbursement, damages and interest. loss complained of.

Our Ruling The absence of any ofthese elements prevents piercing the corporate veil. 12

We find merit in the petition. In the present case, the attendantcircumstances do not establish that WPM is a mere alter ego
of Manlapaz.
We note, at the outset, that the question of whether a corporation is a mere instrumentality or
alter-ego of another is purely one of fact.5 This is also true with respect to the question of Aside from the fact that Manlapaz was the principal stockholder of WPM, records do not show
whether the totality of the evidence adduced by the respondentwarrants the application of the that WPM was organized and controlled, and its affairs conducted in a manner that made it
piercing the veil of corporate fiction doctrine.6 merely an instrumentality, agency, conduit or adjunct ofManlapaz. As held in Martinez v. Court of
Appeals,13 the mere ownership by a singlestockholder of even all or nearly all of the capital
stocks ofa corporation is not by itself a sufficient ground to disregard the separate corporate
Generally, factual findings of the lower courts are accorded the highest degree of respect, if not
personality. To disregard the separate juridical personality of a corporation, the wrongdoing must
finality. When adopted and confirmed by the CA, these findings are final and conclusive and may
be clearly and convincingly established.14
not be reviewed on appeal,7save in some recognized exceptions8 among others, when the
judgment is based on misapprehension of facts.
Likewise, the records of the case do not support the lower courts’ finding that Manlapaz had
control or domination over WPM or its finances. That Manlapaz concurrentlyheld the positions of
We have reviewed the records and found that the application of the principle of piercing the veil
president, chairman and treasurer, or that the Manlapaz’s residence is the registered principal
of corporate fiction is unwarranted in the present case.
office of WPM, are insufficient considerations to prove that he had exercised absolutecontrol
over WPM.
In this connection, we stress thatthe control necessary to invoke the instrumentality or alter ego WHEREFORE, in light of the foregoing, the decision dated September 28, 2007 of the Court of
rule is not majority or even complete stock control but such domination of finances, policies and Appeals in CA-G.R. CV No. 68289 is MODIFIED and.that petitioner Warlito P. Manlapaz is
practices that the controlled corporation has, so tospeak, no separate mind, will or existence of ABSOLVED from any liability under the renovation agreement.
its own, and is but a conduit for its principal. The control must be shown to have been exercised
at the time the acts complained of took place. Moreover, the control and breach of duty must
SO ORDERED.
proximately cause the injury or unjust loss for which the complaint is made.

Here, the respondent failed to prove that Manlapaz, acting as president, had absolute control
over WPM.1âwphi1 Even granting that he exercised a certain degree of control over the
finances, policies and practices of WPM, in view of his position as president, chairman and
treasurer of the corporation, such control does not necessarily warrant piercing the veil of
corporate fiction since there was not a single proof that WPM was formed to defraud CLN or the
respondent, or that Manlapaz was guilty of bad faith or fraud.

On the contrary, the evidence establishes that CLN and the respondent knew and acted on the
knowledgethat they were dealing with WPM for the renovation of the latter’s restaurant, and not
with Manlapaz. That WPM later reneged on its monetary obligation to CLN, resulting to the filing
of a civil case for sum of money against the respondent, does not automatically indicate fraud, in
the absence of any proof to support it.

This Court also observed that the CA failed to demonstrate how the separate and distinct
personalityof WPM was used by Manlapaz to defeat the respondent’s right for reimbursement.
Neither was there any showing that WPM attempted to avoid liability or had no property against
which to proceed.

Since no harm could be said to have been proximately caused by Manlapaz for which the latter
could be held solidarily liable with WPM, and considering that there was no proof that WPM had
insufficient funds, there was no sufficient justification for the RTC and the CA to have ruled that
Manlapaz should be held jointly and severally liable to the respondent for the amount she paid to
CLN. Hence, only WPM is liable to indemnify the respondent.

Finally, we emphasize that the piercing of the veil of corporate fiction is frowned upon and thus,
must be done with caution.15 It can only be done if it has been clearly established that the
separate and distinct personality of the corporation is used to justify a wrong, protect fraud, or
perpetrate a deception. The court must be certain that the corporate fiction was misused to such
an extent that injustice, fraud, or crime was committed against another, in disregard of its rights;
it cannot be presumed.

On the Award of Moral Damages

On the award of moral damages, we find the same in order in view of WPM's unjustified refusal
to pay a just debt. Under Article 2220 of the New Civil Code, 16 moral damages may be awarded
in cases of a breach of contract where the defendant acted fraudulently or in bad faith or was
guilty of gross negligence amounting to bad faith.

In the present case, when payment for the balance of the renovation cost was demanded, WPM,
instead of complying with its obligation, denied having authorized the respondent to contract in
its behalf and accordingly refused to pay. Such cold refusal to pay a just debt amounts to a
breach of contract in bad faith, as contemplated by Article 2220. Hence, the CA's order to pay
moral damages was in order.
G.R. No. L-5081 February 24, 1954 Ramon Sangalang 70 " 70,000.00
Maximo Cristobal 55 " 55,000.00
MARVEL BUILDING CORPORATION, ET AL., plaintiffs-appellees,
vs. Antonio Cristobal 45 " 45,000.00
SATURNINO DAVID, in his capacity as Collector, Bureau of Internal Revenue, defendant- P1,025,000.00
appellant.

LABRADOR, J.:
Maria B Castro was elected President and Maximo Cristobal, Secretary-Treasurer (Exhibit A).

This action was brought by plaintiffs as stockholders of the Marvel Building Corporation to enjoin
the defendant Collector of Internal Revenue from selling at public auction various properties The Wise Building was purchased on September 4, 1946, the purchase being made in the name
described in the complaint, including three parcels of land, with the buildings situated thereon, of Dolores Trinidad, wife of Amado A. Yatco (Exhibit V), and the Aguinaldo Building, on January
known as the Aguinaldo Building, the Wise Building, and the Dewey Boulevard-Padre Faura 17, 1947, in the name of Segundo Esguerra, Sr. (Exhibit M). Both building were purchased for
Mansion, all registered in the name of the said corporation. Said properties were seized and P1,800,000, but as the corporation had only P1,025,000, the balance of the purchase price was
distrained by defendant to collect war profits taxes assessed against plaintiff Maria B. Castro obtained as loans from the Insular Life Assurance Co., Ltd. and the Philippine Guaranty Co., Inc.
(Exhibit B). Plaintiffs allege that the said three properties (lands and buildings) belong to Marvel (Exhibit C).
Building Corporation and not to Maria B. Castro, while the defendant claims that Maria B. Castro
is the true and sole owner of all the subscribed stock of the Marvel Building Corporation, Of the incorporators of the Marvel Building Corporation, Maximo Cristobal and Antonio Cristobal
including those appearing to have been subscribed and paid for by the other members, and are half-brothers of Maria B. Castro, Maria Cristobal is a half-sister, and Segundo Esguerra, Sr.
consequently said Maria B. Castro is also the true and exclusive owner of the properties seized. a brother-in-law, husband of Maria Cristobal, Maria B. Castro's half-sister. Maximo B. Cristobal
The trial court held that the evidence, which is mostly circumstantial, fails to show to its did not file any income tax returns before the year 1946, except for three years 1939 and 1940,
satisfaction that Maria B. Castro is the true owner of all the stock certificates of the corporation, but in these years he was exempted from the tax. He has not filed any war profits tax return
because the evidence is susceptible of two interpretations and an interpretation may not be (Exhibit 54). Antonio Cristobal, Segundo Esguerra, Sr. and Jose T. Lopez did not file any income
made which would deprive one of the property without due process of law. tax returns for the years prior to 1946, and neither did they file any war profits tax returns (Exhibit
52). Maria Cristobal filed income tax returns for the year 1929 to 1942, but they were exempt
It appears that on September 15, 1950, the Secretary of Finance, upon consideration of the from the tax (Exhibit 53). Benita A. Lamagna did not file any income tax returns prior to 1945,
report of a special committee assigned to study the war profits tax case of Mrs. Maria B. Castro, except for 1942 which was exempt. She did not file any was profits tax (Exhibit 55). Ramon M.
recommended the collection of P3,593,950.78 as war profits taxes for the latter, and on Sangalang did not file income tax returns up to 1945 except for the years 1936, 1937, 1938,
September 22, 1953 the President instructed the Collector that steps be taken to collect the 1939 and 1940. He has not filed any war profits tax return (Exhibit 57). Amada A. Yatco did not
same (Exhibits 114, 114-A to 114-D). Pursuant thereto various properties, including the three file income tax returns prior to 1945, except for the years 1937, 1938, 1939, 1941 and 1942, but
above mentioned, were seized by the Collector of Internal Revenue on October 31, 1950. On these were exempt. He did not file any war profits tax return (Exhibit 58).
November 13, 1950, the original complaint in this case was filed. After trial, the Court of First
Instance of Manila rendered judgment ordering the release of the properties mentioned, and Antonio Cristobal's income in 1946 is P15,630, and in 1947, P4,550 (Exhibits 59-60); Maximo B.
enjoined the Collector of Internal Revenue from selling the same. The Collector of Internal Cristobal's income in 1946 is P19,759.10, in 1947, P9,773.47 (Exhibits 61-62); Segundo
Revenue has appealed to this Court against the judgment. Esquerra's income in 1946 is P5,500, in 1947, P7,754.32 (Exhibits 63-64); Jose T. Lopez's
income in 1946 is P20,785, in 1947, P14,302.77 (Exhibits 69-70); Benita A. Lamagna's income
The following facts are not disputed, or are satisfactorily proved by the evidence: in 1945 is P1,559, in 1946, P6,463.36, in 1947, P6,189.79 and her husband's income in 1947 is
P10,825.53 (Exhibits 65-68); Ramon M. Sangalang's income in 1945 is P5,500, in 1946,
P18,300.00 (Exhibits 71-72); Santiago Tan's income in 1945 is P456, in 1947 is P9,167.95, and
The Articles of Incorporation of the Marvel Building Corporation is dated February 12, 1947 and in 1947, P7,620.11 (Exhibits 73-75); and Amado Yatco's income in 1945 is P12,600, in 1946,
according to it the capital stock is P2,000,000, of which P1,025,000 was (at the time of P23,960, and in 1947, P11,160 (Exhibits 76-78).
incorporation) subscribed and paid for by the following incorporators:
In October, 1945 Maria B. Castro, Nicasio Yatco, Maxima Cristobal de Esquerra, Maria Cristobal
Maria B. Castro 250 shares P 250,000.00 Lopez and Maximo Cristobal organized the Maria B. Castro, Inc. with capital stock of P100,000,
of which Maria B. Castro subscribed for P99,600 and all others for P100 each. This was
Amado A. Yatco 100 " 100,000.00 increased in 1950 to P500,000 and Maria B. Castro subscribed P76,000 and the others P1,000
Santiago Tan 100 " 100,000.00 each (Exhibit 126).
Jose T. Lopez 90 " 90,000.00
It does not appear that the stockholders or the board of directors of the Marvel Building
Benita Lamagna 90 " 90,000.00
Corporation have ever held a business meeting, for no books thereof or minutes meeting were
C.S. Gonzales 80 " 80,000.00 ever mentioned by the officers thereof or presented by them at the trial. The by-laws of the
corporation, if any had ever been approved, has not been presented. Neither does it appear that
Maria Cristobal 70 " 70,000.00
any report of the affairs of the corporation has been made, either of its transactions or accounts.
Segundo Esguerra, Sr. 75 " 75,000.00
From the book of accounts of the corporation, advances to the Marvel Building Corporation of As to the supposed enmity of the Llamados towards the plaintiff Maria B. Castro, we note that,
P125,000 were made by Maria B. Castro in 1947, P102,916.05 in 1948 and P102,916.05 in supposing that there really was such enmity, it does not appear that it was of such magnitude or
1948, and P160,910.96 in 1949 (Exhibit 118). force as could have induced the Llamados and Maria B. Castro were close friends way back in
1947 and up to 1949; but that at the time of the trial the friendship had been marred by
misunderstandings. We believe that in 1948 and 1949 the Llamados were trusted friends of
The main issue involved in these proceedings is: Is Maria B. Castro the owner of all the shares
Maria B. Castro, and this explains why they had knowledge of her secret transactions. The
of stocks of Marvel Building Corporation and the other stockholders mere dummies of hers?
younger Llamado even made advances for the hand of Maria B. Castro's daughter, and this at
the time when as a bookkeeper he was entrusted with checking up the certificates of stock.
The most important evidence presented by the Collector of Internal Revenue to prove his claim When the older Llamado kept secret the existence of the endorsed certificates, the friendship
that Maria B. Castro is the sole and exclusive owner of the shares of stock of the Marvel Building between the two families was yet intact; hence, the existence of the endorsed certificates must
Corporation is supposed endorsement in blank of the shares of stock issued in the name of the have been kept to himself by the older Llamado. All the above circumstances reinforce our belief
other incorporators, and the possession thereof by Maria B. Castro. The existence of said that the Llamados had personal knowledge of the facts they testify to, and the existence of this
endorsed certificates was testified to by witnesses Felipe Aquino, internal revenue examiner, knowledge in turn renders improbable plaintiffs' claim that their testimonies were biased.
Antonio Mariano, examiner, and Crispin Llamado, Under Secretary of Finance, who declared as
follows: Towards the end of the year 1948 and about the beginning of the year 1949, while
Attempt was also made by the plaintiffs to show by expert evidence that the endorsement could
Aquino and Mariano were examining the books and papers were furnished by its secretary,
have been superimposed, i.e., that the signatures made on other papers and these were pasted
Maximo Cristobal, they came across an envelope containing eleven stock certificates, bound
and thereafter the documents photographed. Judicial experience is to the effect that the expert
together by an Acco fastener, which (certificates) corresponded in number and in amount on
witnesses can always be obtained for both sides of an issue, most likely because expert
their face to the subscriptions of the stockholders that all the certificates, except that in the name
witnesses are no longer impermeable to the influence of fees (II Wigmore, Sec. 563(2), p. 646).
of Maria B. Castro, were endorsed in the bank by the subscribers; that as the two revenue
And if parties are capable of paying fees, expert opinion should be received with caution. In the
agents could not agree what to do with the certificates, Aquino brought them to Under-Secretary
case at bar, the opinion on the supposed superimposition was merely a possibility, and we note
of Finance Llamado, who thereupon suggested that photostatic copies thereof be taken; that this
various circumstances which proved that the signatures were not superimposed and corroborate
was done, and the photostatic copies taken are (Exhibits 4, 5, 6, 7, 8, 9, 10, 11, 12 and 13; and
defendant's claim that they were genuine. In the first place,, the printed endorsement contains a
in that July, 1950, copy-cat copies of the above photostats were taken, and said copy-cat copies
very heavy line at the bottom for the signature of the endorsee. This line in almost all the
are Exhibits 40-49.
endorsements is as clear as the printed letters above it, and at the points where the letters of the
signature extend down and transversed it (the line), there is no indication that the line is covered
Julio Llamado, bookkeeper of the Marvel Building Corporation from 1947 to May, 1948, also by a superimposed paper. Again in these places both the signatures and the lines are clear and
testified that he was the one who had prepared the original certificates, putting therein the distinct where they cross one another. Had there been superimpositions the above features
number of shares in words in handprint; that the originals were given to him by Maria B. Castro could not have been possible. In the second place, Maria B. Castro admitted having signed 25
for comparison with the articles of incorporation; that they were not yet signed by the President stock certificates, but only eleven were issued (t. s. n., p. 662). No explanation is given by her
and by the Secretary-Treasurer when he had the certificates; and that after the checking he why she had to sign as many as 25 forms when there were only eleven subscribers and eleven
returned all of them to Mrs. Castro. He recognized the photostats, Exhibit 4 to 13 as photostats forms to be filed. This circumstances corroborate the young Llamado's declaration that two sets
of the said originals. He also declared that he also prepared a set of stock certificates, similar to of certificates had been prepared. The nineteen issue must be Exhibits H, H-1 to H-7 and J, or
the certificates which were copied in the photostats, the number of shares, and the date issue, Nos. 30 to 38, and the stock certificates endorsed whose photostatic copies are Exhibits 4 to 13.
and that the certificates he had prepared are Exhibits H, H-1 to H-7 and J (Exhibits 30-38). This It is to be remembered also, that it is a common practice among unscrupulous merchants to
set of certificates was made by him first and the set of which photostats were taken, a few days carry two sets of books, one set for themselves and another to be shown to tax collectors. This
later. practice could not have been unknown to Maria B. Castro, who apparently had been able to
evade the payment of her war profits taxes. These circumstances, coupled with the testimony of
Julio Llamado that two sets of certificates were given to him for checking, show to an impartial
The plaintiffs offered a half-hearted denial of the existence of the endorsed blank certificates,
mind the existence of the set of certificates endorsed in blank, thus confirming the testimonies of
Maximo Cristobal, secretary of the corporation, saying that no investigation was ever made by the defendant's witnesses, Aquino, Mariano and Crispin Llamado, and thus discrediting the
Aquino and Mariano in which said certificates were discovered by the latter. They, however, obviously partial testimony of the expert presented by plaintiffs. The genuineness of the
vigorously attack the credibility of the witnesses for the defendant, imputing to the Llamados,
signatures on the endorsements is not disputed. How could the defendant have secured these
enmity against Maria B. Castro, and to Aquino and Mariano, a very doubtful conduct in not genuine signatures? Plaintiffs offer no explanation for this, although they do not question them. It
divulging the existence of the certificates either Lobrin, Chief Income Tax Examiner, or to the follows that the genuine signatures must have been made on the stock certificates themselves.
Collector of Internal Revenue, both their immediate chiefs. Reliance is also placed on a
certificate, Exhibit W, wherein Aquino and others declare that the certificates (Exhibits 30 to 38,
or H, H-1 to H-7 and J) were regular and were not endorsed when the same were examined. In Next in importance among the evidence submitted by the defendant collector to prove his
connection with this certificate, Exhibit W, we note that it states that the certificates examined contention that Maria B. Castro is the sole owner of the shares of stock of the Marvel Building
were Exhibits 30 to 38, the existence or character of which are not disputed. But the statement Corporation, is the fact that the other stockholders did not have incomes in such amounts, during
contains nothing to the effect that the above certificates were the only one existence, according the time of the organization of the corporation in 1947, or immediately thereto, as to enable them
to their knowledge. Again the certificate was issued for an examination on September 1949, not to pay in full for their supposed subscriptions. This fact is proved by their income tax returns, or
by Aquino and Mariano at the end of 1948 or the beginning of 1949. The certificate, therefore, the absence thereof. Let us take Amado A. Yatco as an example. Before 1945 his returns were
neither denies the existence of the endorsed certificates, nor that Aquino and Mariano had made exempt from the tax, in 1945 he had P12,600 and in 1946, P23,000. He has four children. How
an examination of the papers of the corporation at the end of the year 1948. It ca not, therefore, could he have paid P100,000 in 1945 and 1946? Santiago Tan who also contributed P100,000
discredit the testimonies of the defendant's witnesses. had no appreciable income before 1946, and this year an income of only P9,167.95. Jose T.
Lopez also did not file any income tax returns before 1940 and 1946 he had an income of only
P20,785, whereas he is supposed to have subscribed P90,000 worth of stock early in 1947. the fact that twenty-five certificates were signed by the president of the corporation, for no
Benita Lamagna had no returns either up to 1945, except in 1942, which was exempt and in justifiable reason, the fact that two sets of certificates were issued, the undisputed fact that Maria
1945 she had an income of P1,550 and in 1946, P6,463.36. In the same situation are all the B. Castro had made enormous profits and, therefore, had a motive to hide them to evade the
others, and besides, brothers and sisters and brother-in-law of Maria B. Castro. On the other payment of taxes, the fact that the other subscribers had no incomes of sufficient magnitude to
hand, Maria B. Castro had been found to have made enormous gains or profits in her business justify their big subscriptions, the fact that the subscriptions were not receipted for and deposited
such that the taxes thereon were assessed at around P3,000,000. There was, therefore, a prima by the treasurer in the name of the corporation but were kept by Maria B. Castro herself, the fact
facie case out by the defendant collector that Maria B. Castro had furnished (& all the money that the stockholders or the directors never appeared to have ever met to discuss the business
that the Marvel Building Corporation had. of the corporation, the fact that Maria B. Castro advanced big sums of money to the corporation
without any previous arrangement or accounting, and the fact that the books of accounts were
kept as if they belonged to Maria B. Castro alone — these facts are of patent and potent
In order the meet the above evidence only three of the plaintiffs testified, namely, Maximo
significance. What are their necessary implications? Maria B. Castro would not have asked them
Cristobal, the corporation's secretary, who made the general assertion on the witness stand that
to endorse their stock certificates, or be keeping these in her possession, if they were really the
the other stockholders paid for their shares in full, Maria B. Castro, who stated that payments for
owners. They never would have consented that Maria B. Castro keep the funds without receipts
the subscription were made to her, and C.S. Gonzales, who admitted that Maria B. Castro, paid
or accounting, nor that she manages the business without their knowledge or concurrence, were
for his subscription. After a careful study of the above testimonies, however, we find them
they owners of the stocks in their own rights. Each and every one of the facts all set forth above,
subject to various objections. Maximo Cristobal declared that he issued provincial receipts for
in the same manner, is inconsistent with the claim that the stockholders, other than Maria B.
the subscriptions supposedly paid to him in 1946; but none of the supposed receipts were
Castro, own their shares in their own right. On the other hand, each and every one of them, and
presented. If the subscriptions were really received by him, big as the amounts were, he would
all of them, can point to no other conclusion than that Maria B. Castro was the sole and
have been able to tell specifically, by dates and in fix amounts, when and how the payments
exclusive owner of the shares and that they were only her dummies.
were made. The general assertion of alleged payments, without the concrete days and amounts
of payments, are, according to our experience, positive identifications of untruthfulness, for when
a witness testified to a fact that actually occurs, the act is concretely stated and no In our opinion, the facts and circumstances duly set forth above, all of which have been proved
generalization is made. to our satisfaction, prove conclusively and beyond reasonable doubt (section 89, Rule 123 of the
Rules of Court and section 42 of the Provisional law for the application of the Penal Code) that
Maria B. Castro is the sole and exclusive owner of all the shares of stock of the Marvel Building
With respect to Maria B. Castro's testimony, we find it to be as untruthful as that of Cristobal.
Corporation and that the other partners are her dummies.
She declared that payments of the subscriptions took place between July and December, 1946,
and that first payments were first deposited by her in the National City Bank of New York. A
study of her account in said bank (Exhibit 82), however, fails to show the alleged deposit of the Wherefore, the judgment appealed from should be, as it hereby is, reversed and the action filed
subscriptions during the year 1946 (See Exhibits 83-112). This fact completely belies her by plaintiffs-appellees, dismissed, with costs against plaintiffs-appellees. So ordered.
assertion. As to the testimony of C.S. Gonzales that Maria B. Castro advanced his subscription,
there is nothing in the evidence to corroborate it, and the circumstances show otherwise. If he
had really been a stockholder and Maria B. Castro advanced his subscription, the agreement
between him and Castro should have been put in writing, the amount advanced being quite
considerable (P80,000), and it appearing further that Gonzales is no close relative or confidant
of Castro.

Lastly, it is significant that the plaintiffs, the supposed subscribers, who should have come to
court to assert that they actually paid for their subscriptions, and are not mere dummies, did not
do so. They could not have afforded such a costly indifference, valued at from P70,000 to
P100,000 each, if they were not actual dummies. This failure on their part to take the witness
stand to deny or refute the charge that they were mere dummies is to us of utmost significance.
What could have been easier to disprove the charge that they were dummies, than for them to
come to court and show their receipts and testify on the payments they have paid on their
subscriptions? This they, however refused to do so. They had it in their power to rebut the
charges, but they chose to keep silent. The non-production of evidence that would naturally have
been produced by an honest and therefore fearless claimant permits the inference that its tenor
is unfavorable to the party's cause (II Wigmore, Sec. 285, p.162). A party's silence to adverse
testimony is equivalent to an admission of its truth (Ibid, Sec. 289, p. 175).

Our consideration of the evidence submitted on both sides leads us to a conclusion exactly
opposite that arrived at by the trial court. In general the evidence offered by the plaintiffs is
testimonial and direct evidence, easy of fabrication; that offered by defendant, documentary and
circumstantial, not only difficult of fabrication but in most cases found in the possession of
plaintiffs. There is very little room for choice as between the two. The circumstantial evidence is
not only convincing; it is conclusive. The existence of endorsed certificates, discovered by the
internal revenue agents between 1948 and 1949 in the possession of the Secretary-Treasurer,
G.R. No. L-13203 January 28, 1961 The assessment was disputed by the petitioner, and a reinvestigation of the case having been
made by the agents of the Bureau of Internal Revenue, the respondent Collector in his letter
dated November 15, 1952 countermanded his demand for sales tax deficiency on the ground
YUTIVO SONS HARDWARE COMPANY, petitioner,
that "after several investigations conducted into the matter no sufficient evidence could be
vs.
gathered to sustain the assessment of this Office based on the theory that Southern Motors is a
COURT OF TAX APPEALS and COLLECTOR OF INTERNAL REVENUE, respondents.
mere instrumentality or subsidiary of Yutivo." The withdrawal was subject, however, to the
general power of review by the now defunct Board of Tax Appeals. The Secretary of Finance to
GUTIERREZ DAVID, J.: whom the papers relative to the case were endorsed, apparently not agreeing with the
withdrawal of the assessment, returned them to the respondent Collector for reinvestigation.
This is a petition for review of a decision of the Court of Tax Appeals ordering petitioner to pay to
respondent Collector of Internal Revenue the sum of P1,266,176.73 as sales tax deficiency for After another investigation, the respondent Collector, in a letter to petitioner dated December 16,
the third quarter of 1947 to the fourth quarter of 1950; inclusive, plus 75% surcharge thereon, 1954, redetermined that the aforementioned tax assessment was lawfully due the government
equivalent to P349,632.54, or a sum total of P2,215,809.27, plus costs of the suit. and in addition assessed deficiency sales tax due from petitioner for the four quarters of 1950;
the respondents' last demand was in the total sum of P2,215,809.27 detailed as follows:
From the stipulation of facts and the evidence adduced by both parties, it appears that petitioner
Yutivo Sons Hardware Co. (hereafter referred to as Yutivo) is a domestic corporation, organized
Deficiency Sales 75%
under the laws of the Philippines, with principal office at 404 Dasmariñas St., Manila. Tax Surcharge Total Amount Due
Incorporated in 1916, it was engaged, prior to the last world war, in the importation and sale of
hardware supplies and equipment. After the liberation, it resumed its business and until June of Assessment (First) of November 7,
1946 bought a number of cars and trucks from General Motors Overseas Corporation (hereafter 1950 for deficiency sales Tax for the
referred to as GM for short), an American corporation licensed to do business in the Philippines. period from 3rd Qrtr 1947 to 4th Qrtr
As importer, GM paid sales tax prescribed by sections 184, 185 and 186 of the Tax Code on the 1949 inclusive P1,031,296.60 P773,473.45 P1,804,769.05
basis of its selling price to Yutivo. Said tax being collected only once on original sales, Yutivo Additional Assessment for period from
paid no further sales tax on its sales to the public. 1st to 4th Qrtr 1950, inclusive 234,880.13 176,160.09 411,040.22
Total amount demanded per letter of
On June 13, 1946, the Southern Motors, Inc. (hereafter referred to as SM) was organized to December 16, 1954 P1,266,176.73 P949,632.54 P2,215,809.27
engage in the business of selling cars, trucks and spare parts. Its original authorized capital
stock was P1,000,000 divided into 10,000 shares with a par value of P100 each.
This second assessment was contested by the petitioner Yutivo before the Court of Tax
At the time of its incorporation 2,500 shares worth P250,000 appear to have been subscribed Appeals, alleging that there is no valid ground to disregard the corporate personality of SM and
into equal proportions by Yu Khe Thai, Yu Khe Siong, Hu Kho Jin, Yu Eng Poh, and Washington to hold that it is an adjunct of petitioner Yutivo; (2) that assuming the separate personality of SM
Sycip. The first three named subscribers are brothers, being sons of Yu Tiong Yee, one of may be disregarded, the sales tax already paid by Yutivo should first be deducted from the
Yutivo's founders. The latter two are respectively sons of Yu Tiong Sin and Albino Sycip, who selling price of SM in computing the sales tax due on each vehicle; and (3) that the surcharge
are among the founders of Yutivo. has been erroneously imposed by respondent. Finding against Yutivo and sustaining the
respondent Collector's theory that there was no legitimate or bona fide purpose in the
organization of SM — the apparent objective of its organization being to evade the payment of
After the incorporation of SM and until the withdrawal of GM from the Philippines in the middle of taxes — and that it was owned (or the majority of the stocks thereof are owned) and controlled
1947, the cars and tracks purchased by Yutivo from GM were sold by Yutivo to SM which, in by Yutivo and is a mere subsidiary, branch, adjunct, conduit, instrumentality or alter ego of the
turn, sold them to the public in the Visayas and Mindanao. latter, the Court of Tax Appeals — with Judge Roman Umali not taking part — disregarded its
separate corporate existence and on April 27, 1957, rendered the decision now complained of.
When GM decided to withdraw from the Philippines in the middle of 1947, the U.S. manufacturer Of the two Judges who signed the decision, one voted for the modification of the computation of
of GM cars and trucks appointed Yutivo as importer for the Visayas and Mindanao, and Yutivo the sales tax as determined by the respondent Collector in his decision so as to give allowance
continued its previous arrangement of selling exclusively to SM. In the same way that GM used for the reduction of the tax already paid (resulting in the reduction of the assessment to
to pay sales taxes based on its sales to Yutivo, the latter, as importer, paid sales tax prescribed P820,509.91 exclusive of surcharges), while the other voted for affirmance. The dispositive part
on the basis of its selling price to SM, and since such sales tax, as already stated, is collected of the decision, however, affirmed the assessment made by the Collector. Reconsideration of
only once on original sales, SM paid no sales tax on its sales to the public. this decision having been denied, Yutivo brought the case to this Court thru the present petition
for review.

On November 7, 1950, after several months of investigation by revenue officers started in July,
1948, the Collector of Internal Revenue made an assessment upon Yutivo and demanded from It is an elementary and fundamental principle of corporation law that a corporation is an entity
the latter P1,804,769.85 as deficiency sales tax plus surcharge covering the period from the third separate and distinct from its stockholders and from other corporation petitions to which it may
quarter of 1947 to the fourth quarter of 1949; or from July 1, 1947 to December 31, 1949, be connected. However, "when the notion of legal entity is used to defeat public convenience,
claiming that the taxable sales were the retail sales by SM to the public and not the sales at justify wrong, protect fraud, or defend crime," the law will regard the corporation as an
wholesale made by, Yutivo to the latter inasmuch as SM and Yutivo were one and the same association of persons, or in the case of two corporations merge them into one. (Koppel [Phil.],
corporation, the former being the subsidiary of the latter. Inc. vs. Yatco, 77 Phil. 496, citing I Fletcher Cyclopedia of Corporation, Perm Ed., pp. 135 136;
United States vs. Milwaukee Refrigeration Transit Co., 142 Fed., 247, 255 per Sanborn, J.)
Another rule is that, when the corporation is the "mere alter ego or business conduit of a person, governing payment of advance sales tax by the importer based on the landed cost of the
it may be disregarded." (Koppel [Phil.], Inc. vs. Yatco, supra.) imported article, increased by mark-ups of 25%, 50%, and 100%, depending on whether the
imported article is taxed under sections 186, 185 and 184, respectively, of the Tax Code. Under
Republic Act No. 594, the amount at which the article is sold is immaterial to the amount of the
After going over the voluminous record of the present case, we are inclined to rule that the Court
sales tax. And yet after the passage of that Act, SM continued to exist up to the present and
of Tax Appeals was not justified in finding that SM was organized for no other purpose than to
operates as it did many years past in the promotion and pursuit of the business purposes for
defraud the Government of its lawful revenues. In the first place, this corporation was organized
which it was organized.
in June, 1946 when it could not have caused Yutivo any tax savings. From that date up to June
30, 1947, or a period of more than one year, GM was the importer of the cars and trucks sold to
Yutivo, which, in turn resold them to SM. During that period, it is not disputed that GM as In the third place, sections 184 to 186 of the said Code provides that the sales tax shall be
importer, was the one solely liable for sales taxes. Neither Yutivo or SM was subject to the sales collected "once only on every original sale, barter, exchange . . , to be paid by the manufacturer,
taxes on their sales of cars and trucks. The sales tax liability of Yutivo did not arise until July 1, producer or importer." The use of the word "original" and the express provision that the tax was
1947 when it became the importer and simply continued its practice of selling to SM. The collectible "once only" evidently has made the provisions susceptible of different interpretations.
decision, therefore, of the Tax Court that SM was organized purposely as a tax evasion device In this connection, it should be stated that a taxpayer has the legal right to decrease the amount
runs counter to the fact that there was no tax to evade. of what otherwise would be his taxes or altogether avoid them by means which the law permits.
(U.S. vs. Isham 17 Wall. 496, 506; Gregory vs. Helvering 293 U.S. 465, 469; Commr. vs. Tower,
327 U.S. 280; Lawton vs. Commr 194 F (2d) 380). Any legal means by the taxpayer to reduce
Making the observation from a newspaper clipping (Exh. "T") that "as early as 1945 it was
taxes are all right Benry vs. Commr. 25 T. Cl. 78). A man may, therefore, perform an act that he
known that GM was preparing to leave the Philippines and terminate its business of importing
honestly believes to be sufficient to exempt him from taxes. He does not incur fraud thereby
vehicles," the court below speculated that Yutivo anticipated the withdrawal of GM from business
even if the act is thereafter found to be insufficient. Thus in the case of Court Holding Co. vs.
in the Philippines in June, 1947. This observation, which was made only in the resolution on the
Commr. 2 T. Cl. 531, it was held that though an incorrect position in law had been taken by the
motion for reconsideration, however, finds no basis in the record. On the other hand, GM had
corporation there was no suppression of the facts, and a fraud penalty was not justified.
been an importer of cars in the Philippines even before the war and had but recently resumed its
operation in the Philippines in 1946 under an ambitious plan to expand its operation by
establishing an assembly plant here, so that it could not have been expected to make so drastic The evidence for the Collector, in our opinion, falls short of the standard of clear and convincing
a turnabout of not merely abandoning the assembly plant project but also totally ceasing to do proof of fraud. As a matter of fact, the respondent Collector himself showed a great deal of doubt
business as an importer. Moreover, the newspaper clipping, Exh. "T", was published on March or hesitancy as to the existence of fraud. He even doubted the validity of his first assessment
24, 1947, and clipping, merely reported a rumored plan that GM would abandon the assembly dated November 7, 1959. It must be remembered that the fraud which respondent Collector
plant project in the Philippines. There was no mention of the cessation of business by GM which imputed to Yutivo must be related to its filing of sales tax returns of less taxes than were legally
must not be confused with the abandonment of the assembly plant project. Even as respect the due. The allegation of fraud, however, cannot be sustained without the showing that Yutivo, in
assembly plant, the newspaper clipping was quite explicit in saying that the Acting Manager filing said returns, did so fully knowing that the taxes called for therein called for therein were
refused to confirm that rumor as late as March 24, 1947, almost a year after SM was organized. less than what were legally due. Considering that respondent Collector himself with the aid of his
legal staff, and after some two years of investigation and duty of investigation and study
concluded in 1952 that Yutivo's sales tax returns were correct — only to reverse himself after
At this juncture, it should be stated that the intention to minimize taxes, when used in the context
another two years — it would seem harsh and unfair for him to say in 1954 that Yutivo fully knew
of fraud, must be proved to exist by clear and convincing evidence amounting to more than mere
in October 1947 that its sales tax returns were inaccurate.
preponderance, and cannot be justified by a mere speculation. This is because fraud is never
lightly to be presumed. (Vitelli & Sons vs. U.S 250 U.S. 355; Duffin vs. Lucas, 55 F (2d) 786;
Budd vs. Commr., 43 F (2d) 509; Maryland Casualty Co. vs. Palmette Coal Co., 40 F (2d) 374; On this point, one other consideration would show that the intent to save taxes could not have
Schoonfield Bros., Inc. vs. Commr., 38 BTA 943; Charles Heiss vs. Commr 36 BTA 833; existed in the minds of the organizers of SM. The sales tax imposed, in theory and in practice, is
Kerbaugh vs. Commr 74 F (2d) 749; Maddas vs. Commr., 114 F. (2d) 548; Moore vs. Commr., passed on to the vendee, and is usually billed separately as such in the sales invoice. As
37 BTA 378; National City Bank of New York vs. Commr., 98 (2d) 93; Richard vs. Commr., 15 pointed out by petitioner Yutivo, had not SM handled the retail, the additional tax that would
BTA 316; Rea Gane vs. Commr., 19 BTA 518). (See also Balter, Fraud Under Federal Law, pp. have been payable by it, could have been easily passed off to the consumer, especially since
301-302, citing numerous authorities: Arroyo vs. Granada, et al., 18 Phil. 484.) Fraud is never the period covered by the assessment was a "seller's market" due to the post-war scarcity up to
imputed and the courts never sustain findings of fraud upon circumstances which, at the most, late 1948, and the imposition of controls in the late 1949.
create only suspicion. (Haygood Lumber & Mining Co. vs. Commr., 178 F (2d) 769; Dalone vs.
Commr., 100 F (2d) 507).
It is true that the arrastre charges constitute expenses of Yutivo and its non-inclusion in the
selling price by Yutivo cost the Government P4.00 per vehicle, but said non-inclusion was
In the second place, SM was organized and it operated, under circumstance that belied any explained to have been due to an inadvertent accounting omission, and could hardly be
intention to evade sales taxes. "Tax evasion" is a term that connotes fraud thru the use of considered as proof of willful channelling and fraudulent evasion of sales tax. Mere
pretenses and forbidden devices to lessen or defeat taxes. The transactions between Yutivo and understatement of tax in itself does not prove fraud. (James Nicholson, 32 BTA 377, affirmed 90
SM, however, have always been in the open, embodied in private and public documents, F. (2) 978, cited in Merten's Sec. 55.11 p. 21) The amount involved, moreover, is extremely
constantly subject to inspection by the tax authorities. As a matter of fact, after Yutivo became small inducement for Yutivo to go thru all the trouble of organizing SM. Besides, the non-
the importer of GM cars and trucks for Visayas and Mindanao, it merely continued the method of inclusion of these small arrastre charges in the sales tax returns of Yutivo is clearly shown in the
distribution that it had initiated long before GM withdrew from the Philippines. records of Yutivo, which is uncharacteristic of fraud (See Insular Lumber Co. vs. Collector, G.R.
No. L-719, April 28, 1956.)
On the other hand, if tax saving was the only justification for the organization of SM, such
justification certainly ceased with the passage of Republic Act No. 594 on February 16, 1951,
We are, however, inclined to agree with the court below that SM was actually owned and SM is under the management and control of Yutivo by virtue of a management contract entered
controlled by petitioner as to make it a mere subsidiary or branch of the latter created for the into between the two parties. In fact, the controlling majority of the Board of Directors of Yutivo is
purpose of selling the vehicles at retail and maintaining stores for spare parts as well as service also the controlling majority of the Board of Directors of SM. At the same time the principal
repair shops. It is not disputed that the petitioner, which is engaged principally in hardware officers of both corporations are identical. In addition both corporations have a common
supplies and equipment, is completely controlled by the Yutivo, Young or Yu family. The comptroller in the person of Simeon Sy, who is a brother-in-law of Yutivo's president, Yu Khe
founders of the corporation are closely related to each other either by blood or affinity, and most Thai. There is therefore no doubt that by virtue of such control, the business, financial and
of its stockholders are members of the Yu (Yutivo or Young) family. It is, likewise, admitted that management policies of both corporations could be directed towards common ends.
SM was organized by the leading stockholders of Yutivo headed by Yu Khe Thai. At the time of
its incorporation 2,500 shares worth P250,000.00 appear to have been subscribed in five equal
Another aspect relative to Yutivo's control over SM operations relates to its cash transactions. All
proportions by Yu Khe Thai, Yu Khe Siong, Yu Khe Jin, Yu Eng Poh and Washington Sycip. The
cash assets of SM were handled by Yutivo and all cash transactions of SM were actually
first three named subscribers are brothers, being the sons of Yu Tien Yee, one of Yutivo's
maintained thru Yutivo. Any and all receipts of cash by SM including its branches were
founders. Yu Eng Poh and Washington Sycip are respectively sons of Yu Tiong Sing and Alberto
transmitted or transferred immediately and directly to Yutivo in Manila upon receipt thereof.
Sycip who are co-founders of Yutivo. According to the Articles of Incorporation of the said
Likewise, all expenses, purchases or other obligations incurred by SM are referred to Yutivo
subscriptions, the amount of P62,500 was paid by the aforenamed subscribers, but actually the
which in turn prepares the corresponding disbursement vouchers and payments in relation there,
said sum was advanced by Yutivo. The additional subscriptions to the capital stock of SM and
the payment being made out of the cash deposits of SM with Yutivo, if any, or in the absence
subsequent transfers thereof were paid by Yutivo itself. The payments were made, however,
thereof which occurs generally, a corresponding charge is made against the account of SM in
without any transfer of funds from Yutivo to SM. Yutivo simply charged the accounts of the
Yutivo's books. The payments for and charges against SM are made by Yutivo as a matter of
subscribers for the amount allegedly advanced by Yutivo in payment of the shares. Whether a
course and without need of any further request, the latter would advance all such cash
charge was to be made against the accounts of the subscribers or said subscribers were to
requirements for the benefit of SM. Any and all payments and cash vouchers are made on
subscribe shares appears to constitute a unilateral act on the part of Yutivo, there being no
Yutivo stationery and made under authority of Yutivo's corporate officers, without any copy
showing that the former initiated the subscription.
thereof being furnished to SM. All detailed records such as cash disbursements, such as
expenses, purchases, etc. for the account of SM, are kept by Yutivo and SM merely keeps a
The transactions were made solely by and between SM and Yutivo. In effect, it was Yutivo who summary record thereof on the basis of information received from Yutivo.
undertook the subscription of shares, employing the persons named or "charged" with
corresponding account as nominal stockholders. Of course, Yu Khe Thai, Yu Khe Jin, Yu Khe
All the above plainly show that cash or funds of SM, including those of its branches which are
Siong and Yu Eng Poh were manifestly aware of these subscriptions, but considering that they
directly remitted to Yutivo, are placed in the custody and control of Yutivo, resources and subject
were the principal officers and constituted the majority of the Board of Directors of both Yutivo
to withdrawal only by Yutivo. SM's being under Yutivo's control, the former's operations and
and SM, their subscriptions could readily or easily be that of Yutivo's Moreover, these persons
existence became dependent upon the latter.
were related to death other as brothers or first cousins. There was every reason for them to
agree in order to protect their common interest in Yutivo and SM.
Consideration of various other circumstances, especially when taken together, indicates that
Yutivo treated SM merely as its department or adjunct. For one thing, the accounting system
The issued capital stock of SM was increased by additional subscriptions made by various
maintained by Yutivo shows that it maintained a high degree of control over SM accounts. All
person's but except Ng Sam Bak and David Sycip, "payments" thereof were effected by merely
transactions between Yutivo and SM are recorded and effected by mere debit or credit entries
debiting 'or charging the accounts of said stockholders and crediting the corresponding amounts
against the reciprocal account maintained in their respective books of accounts and indicate the
in favor of SM, without actually transferring cash from Yutivo. Again, in this instance, the
dependency of SM as branch upon Yutivo.
"payments" were Yutivo, by effected by the mere unilateral act of Yutivo a accounts of the virtue
of its control over the individual persons charged, would necessarily exercise preferential rights
and control directly or indirectly, over the shares, it being the party which really undertook to pay Apart from the accounting system, other facts corroborate or independently show that SM is a
or underwrite payment thereof. branch or department of Yutivo. Even the branches of SM in Bacolod, Iloilo, Cebu, and Davao
treat Yutivo — Manila as their "Head Office" or "Home Office" as shown by their letters of
remittances or other correspondences. These correspondences were actually received by Yutivo
The shareholders in SM are mere nominal stockholders holding the shares for and in behalf of
and the reference to Yutivo as the head or home office is obvious from the fact that all cash
Yutivo, so even conceding that the original subscribers were stockholders bona fide Yutivo was
collections of the SM's branches are remitted directly to Yutivo. Added to this fact, is that SM
at all times in control of the majority of the stock of SM and that the latter was a mere subsidiary
may freely use forms or stationery of Yutivo
of the former.

The fact that SM is a mere department or adjunct of Yutivo is made more patent by the fact that
True, petitioner and other recorded stockholders transferred their shareholdings, but the
arrastre conveying, and charges paid for the "operation of receiving, loading or unloading" of
transfers were made to their immediate relatives, either to their respective spouses and children
imported cars and trucks on piers and wharves, were charged against SM. Overtime charges for
or sometimes brothers or sisters. Yutivo's shares in SM were transferred to immediate relatives
the unloading of cars and trucks as requested by Yutivo and incurred as part of its acquisition
of persons who constituted its controlling stockholders, directors and officers. Despite these
cost thereof, were likewise charged against and treated as expenses of SM. If Yutivo were the
purported changes in stock ownership in both corporations, the Board of Directors and officers of
importer, these arrastre and overtime charges were Yutivo's expenses in importing goods and
both corporations remained unchanged and Messrs. Yu Khe Thai, Yu Khe Siong Hu Khe Jin and
not SM's. But since those charges were made against SM, it plainly appears that Yutivo had sole
Yu Eng Poll (all of the Yu or Young family) continued to constitute the majority in both boards. All
authority to allocate its expenses even as against SM in the sense that the latter is a mere
these, as observed by the Court of Tax Appeals, merely serve to corroborate the fact that there
adjunct, branch or department of the former.
was a common ownership and interest in the two corporations.
Proceeding to another aspect of the relation of the parties, the management fees due from SM any provision of existing law to the contrary not withstanding, repeal or modify the decision of the
to Yutivo were taken up as expenses of SM and credited to the account of Yutivo. If it were to be chief of said Bureaus or offices when advisable in public interest."
assumed that the two organizations are separate juridical entities, the corresponding receipts or
receivables should have been treated as income on the part of Yutivo. But such management
It should here also be stated that the assessment in question was consistently protested by
fees were recorded as "Reserve for Bonus" and were therefore a liability reserve and not an
petitioner, making several requests for reinvestigation thereof. Under the circumstances,
income account. This reserve for bonus were subsequently distributed directly to and credited in
petitioner may be considered to have waived the defense of prescription.
favor of the employees and directors of Yutivo, thereby clearly showing that the management
fees were paid directly to Yutivo officers and employees.
"Estoppel has been employed to prevent the application of the statute of limitations against the
government in certain instances in which the taxpayer has taken some affirmative action to
Briefly stated, Yutivo financed principally, if not wholly, the business of SM and actually extended
prevent the collection of the tax within the statutory period. It is generally held that a taxpayer is
all the credit to the latter not only in the form of starting capital but also in the form of credits
estopped to repudiate waivers of the statute of limitations upon which the government relied.
extended for the cars and vehicles allegedly sold by Yutivo to SM as well as advances or loans
The cases frequently involve dissolved corporations. If no waiver has been given, the cases
for the expenses of the latter when the capital had been exhausted. Thus, the increases in the
usually show come conduct directed to a postponement of collection, such, for example, as
capital stock were made in advances or "Guarantee" payments by Yutivo and credited in favor of
some variety of request to apply an overassessment. The taxpayer has 'benefited' and 'is not in
SM. The funds of SM were all merged in the cash fund of Yutivo. At all times Yutivo thru officers
a position to contest' his tax liability. A definite representation of implied authority may be
and directors common to it and SM, exercised full control over the cash funds, policies,
involved, and in many cases the taxpayer has received the 'benefit' of being saved from the
expenditures and obligations of the latter.
inconvenience, if not hardship of immediate collection. "

Southern Motors being but a mere instrumentality, or adjunct of Yutivo, the Court of Tax Appeals
Conceivably even in these cases a fully informed Commissioner may err to the sorrow of the
correctly disregarded the technical defense of separate corporate entity in order to arrive at the
revenues, but generally speaking, the cases present a strong combination of equities against the
true tax liability of Yutivo.
taxpayer, and few will seriously quarrel with their application of the doctrine of estoppel."
(Mertens Law of Federal Income Taxation, Vol. 10-A, pp. 159-160.)
Petitioner contends that the respondent Collector had lost his right or authority to issue the
disputed assessment by reason of prescription. The contention, in our opinion, cannot be
It is also claimed that section 9 of Executive Order No. 401-A, series of 1951 — es involving an
sustained. It will be noted that the first assessment was made on November 7, 1950 for
original assessment of more than P5,000 — refers only to compromises and refunds of taxes,
deficiency sales tax from 1947 to 1949. The corresponding returns filed by petitioner covering
but not to total withdrawal of the assessment. The contention is without merit. A careful
the said period was made at the earliest on October 1, as regards the third quarter of 1947, so
examination of the provisions of both sections 8 and 9 of Executive Order No. 401-A, series of
that it cannot be claimed that the assessment was not made within the five-year period
1951, reveals the procedure prescribed therein is intended as a check or control upon the
prescribed in section 331 of the Tax Code invoked by petitioner. The assessment, it is admitted,
powers of the Collector of Internal Revenue in respect to assessment and refunds of taxes. If it
was withdrawn by the Collector on insufficiency of evidence, but November 15, 1952 due to
be conceded that a decision of the Collector of Internal Revenue on partial remission of taxes is
insufficiency of evidence, but the withdrawal was made subject to the approval of the Secretary
subject to review by the Secretary of Finance and the Board of Tax Appeals, then with more
of Finance and the Board of Tax Appeals, pursuant to the provisions of section 9 of Executive
reason should the power of the Collector to withdraw totally an assessment be subject to such
Order No. 401-A, series of 1951. The decision of the previous assessment of November 7,
review.
Collector countermanding the as 1950 was forwarded to the Board of Tax Appeals through the
Secretary of Finance but that official, apparently disagreeing with the decision, sent it back for
re-investigation. Consequently, the assessment of November 7, 1950 cannot be considered to We find merit, however, in petitioner's contention that the Court of Tax Appeals erred in the
have been finally withdrawn. That the assessment was subsequently reiterated in the decision of imposition of the 5% fraud surcharge. As already shown in the early part of this decision, no
respondent Collector on December 16, 1954 did not alter the fact that it was made seasonably. element of fraud is present.
In this connection, it would appear that a warrant of distraint and levy had been issued on March
28, 1951 in relation with this case and by virtue thereof the properties of Yutivo were placed
under constructive distraint. Said warrant and constructive distraint have not been lifted up to the Pursuant to Section 183 of the National Internal Revenue Code the 50% surcharge should be
present, which shows that the assessment of November 7, 1950 has always been valid and added to the deficiency sales tax "in case a false or fraudulent return is willfully made." Although
the sales made by SM are in substance by Yutivo this does not necessarily establish fraud nor
subsisting.
the willful filing of a false or fraudulent return.

Anent the deficiency sale tax for 1950, considering that the assessment thereof was made on
The case of Court Holding Co. v. Commissioner of Internal Revenue (August 9, 1943, 2 TC 531,
December 16, 1954, the same was assessed well within the prescribed five-year period.
541-549) is in point. The petitioner Court Holding Co. was a corporation consisting of only two
stockholders, to wit: Minnie Miller and her husband Louis Miller. The only assets of third
Petitioner argues that the original assessment of November 7, 1950 did not extend the husband and wife corporation consisted of an apartment building which had been acquired for a
prescriptive period on assessment. The argument is untenable, for, as already seen, the very low price at a judicial sale. Louis Miller, the husband, who directed the company's business,
assessment was never finally withdrawn, since it was not approved by the Secretary of Finance verbally agreed to sell this property to Abe C. Fine and Margaret Fine, husband and wife, for the
or of the Board of Tax Appeals. The authority of the Secretary to act upon the assessment sum of $54,000.00, payable in various installments. He received $1,000.00 as down payment.
cannot be questioned, for he is expressly granted such authority under section 9 of Executive The sale of this property for the price mentioned would have netted the corporation a handsome
Order No. 401-And under section 79 (c) of the Revised Administrative Code, he has "direct profit on which a large corporate income tax would have to be paid. On the afternoon of
control, direction and supervision over all bureaus and offices under his jurisdiction and may, February 23, 1940, when the Millers and the Fines got together for the execution of the
document of sale, the Millers announced that their attorney had called their attention to the large
corporate tax which would have to be paid if the sale was made by the corporation itself. So Yutivo deliberately made a false return for the purpose of defrauding the government of its
instead of proceeding with the sale as planned, the Millers approved a resolution to declare a revenues which will justify the imposition of the surcharge penalty.
dividend to themselves "payable in the assets of the corporation, in complete liquidation and
surrender of all the outstanding corporate stock." The building, which as above stated was the
We likewise find meritorious the contention that the Tax Court erred in computing the alleged
only property of the corporation, was then transferred to Mr. and Mrs. Miller who in turn sold it to
deficiency sales tax on the selling price of SM without previously deducting therefrom the sales
Mr. and Mrs. Fine for exactly the same price and under the same terms as had been previously
tax due thereon. The sales tax provisions (sees. 184.186, Tax Code) impose a tax on original
agreed upon between the corporation and the Fines.
sales measured by "gross selling price" or "gross value in money". These terms, as interpreted
by the respondent Collector, do not include the amount of the sales tax, if invoiced separately.
The return filed by the Court Holding Co. with the respondent Commissioner of Internal Revenue Thus, General Circular No. 431 of the Bureau of Internal Revenue dated July 29, 1939, which
reported no taxable gain as having been received from the sale of its assets. The Millers, of implements sections 184.186 of the Tax Code provides: "
course, reported a long term capital gain on the exchange of their corporate stock with the
corporate property. The Commissioner of Internal Revenue contended that the liquidating
. . .'Gross selling price' or gross value in money' of the articles sold, bartered, exchanged,
dividend to stockholders had no purpose other than that of tax avoidance and that, therefore, the
transferred as the term is used in the aforecited sections (sections 184, 185 and 186) of the
sale by the Millers to the Fines of the corporation's property was in substance a sale by the
National Internal Revenue Code, is the total amount of money or its equivalent which the
corporation itself, for which the corporation is subject to the taxable profit thereon. In requiring
purchaser pays to the vendor to receive or get the goods. However, if a manufacturer, producer,
the corporation to pay the taxable profit on account of the sale, the Commissioner of Internal
or importer, in fixing the gross selling price of an article sold by him has included an amount
Revenue, imposed a surcharge of 25% for delinquency, plus an additional surcharge as fraud
intended to cover the sales tax in the gross selling price of the articles, the sales tax shall be
penalties.
based on the gross selling price less the amount intended to cover the tax, if the same is billed
to the purchaser as a separate item.
The U. S. Court of Tax Appeals held that the sale by the Millers was for no other purpose than to
avoid the tax and was, in substance, a sale by the Court Holding Co., and that, therefore, the
General Circular No. 440 of the same Bureau reads:
said corporation should be liable for the assessed taxable profit thereon. The Court of Tax
Appeals also sustained the Commissioner of Internal Revenue on the delinquency penalty of
25%. However, the Court of Tax Appeals disapproved the fraud penalties, holding that an Amount intended to cover the tax must be billed as a separate em so as not to pay a tax on the
attempt to avoid a tax does not necessarily establish fraud; that it is a settled principle that a tax. — On sales made after he third quarter of 1939, the amount intended to cover the sales tax
taxpayer may diminish his tax liability by means which the law permits; that if the petitioner, the must be billed to the purchaser as separate items in the, invoices in order that the reduction
Court Holding Co., was of the opinion that the method by which it attempted to effect the sale in thereof from the gross ailing price may be allowed in the computation of the merchants'
question was legally sufficient to avoid the imposition of a tax upon it, its adoption of that percentage tax on the sales. Unless billed to the purchaser as a separate item in the invoice, the
methods not subject to censure; and that in taking a position with respect to a question of law, amounts intended to cover the sales tax shall be considered as part of the gross selling price of
the substance of which was disclosed by the statement indorsed on it return, it may not be said the articles sold, and deductions thereof will not be allowed, (Cited in Dalupan, Nat. Int. Rev.
that that position was taken fraudulently. We quote in full the pertinent portion of the decision of Code, Annotated, Vol. II, pp. 52-53.)
the Court of Tax Appeals: .
Yutivo complied with the above circulars on its sales to SM, and as separately billed, the sales
". . . The respondent's answer alleges that the petitioner's failure to report as income the taxable taxes did not form part of the "gross selling price" as the measure of the tax. Since Yutivo had
profit on the real estate sale was fraudulent and with intent to evade the tax. The petitioner filed previously billed the sales tax separately in its sales invoices to SM General Circulars Nos. 431
a reply denying fraud and averring that the loss reported on its return was correct to the best of and 440 should be deemed to have been complied. Respondent Collector's method of
its knowledge and belief. We think the respondent has not sustained the burden of proving a computation, as opined by Judge Nable in the decision complained of —
fraudulent intent. We have concluded that the sale of the petitioner's property was in substance
a sale by the petitioner, and that the liquidating dividend to stockholders had no purpose other
. . . is unfair, because . . .(it is) practically imposing tax on a tax already paid. Besides, the
than that of tax avoidance. But the attempt to avoid tax does not necessarily establish fraud. It is
a settled principle that a taxpayer may diminish his liability by any means which the law adoption of the procedure would in certain cases elevate the bracket under which the tax is
permits. United States v. Isham, 17 Wall. 496; Gregory v. Helvering, supra; Chrisholm v. based. The late payment is already penalized, thru the imposition of surcharges, by adopting the
theory of the Collector, we will be creating an additional penalty not contemplated by law."
Commissioner, 79 Fed. (2d) 14. If the petitioner here was of the opinion that the method by
which it attempted to effect the sale in question was legally sufficient to avoid the imposition of
tax upon it, its adoption of that method is not subject to censure. Petitioner took a position with If the taxes based on the sales of SM are computed in accordance with Gen. Circulars Nos. 431
respect to a question of law, the substance of which was disclosed by the statement endorsed and 440 the total deficiency sales taxes, exclusive of the 25% and 50% surcharges for late
on its return. We can not say, under the record before us, that that position was taken payment and for fraud, would amount only to P820,549.91 as shown in the following
fraudulently. The determination of the fraud penalties is reversed." computation:

When GM was the importer and Yutivo, the wholesaler, of the cars and trucks, the sales tax was Gross Sales of Sales Taxes Due and
paid only once and on the original sales by the former and neither the latter nor SM paid taxes Rates of Sales Total Gross Selling Price
Vehicles Exclusive Computed under Gen.
on their subsequent sales. Yutivo might have, therefore, honestly believed that the payment by Tax Charged to the Public
of Sales Tax Cir Nos. 431 & 400
it, as importer, of the sales tax was enough as in the case of GM Consequently, in filing its return
on the basis of its sales to SM and not on those by the latter to the public, it cannot be said that 5% P11,912,219.57 P595,610.98 P12,507,83055
7% 909,559.50 63,669.16 973,228.66
10% 2,618,695.28 261,869.53 2,880,564.81
15% 3,602,397.65 540,359.65 4,142,757.30
20% 267,150.50 53,430.10 320,580.60
30% 837,146.97 251,114.09 1,088,291.06
50% 74,244.30 37,122.16 111,366.46
75% 8,000.00 6,000.00 14,000.00
TOTAL P20,220,413.77 P1,809,205.67 P22,038,619.44

Less Taxes Paid by Yutivo 988,655.76


Deficiency Tax still due P820,549.91

This is the exact amount which, according to Presiding Judge Nable of the Court of Tax Appeals,
Yutivo would pay, exclusive of the surcharges.

Petitioner finally contends that the Court of Tax Appeals erred or acted in excess of its
jurisdiction in promulgating judgment for the affirmance of the decision of respondent Collector
by less than the statutory requirement of at least two votes of its judges. Anent this contention,
section 2 of Republic Act No. 1125, creating the Court of Tax Appeals, provides that "Any two
judges of the Court of Tax Appeals shall constitute a quorum, and the concurrence of two judges
shall be necessary to promulgate decision thereof. . . . " It is on record that the present case was
heard by two judges of the lower court. And while Judge Nable expressed his opinion on the
issue of whether or not the amount of the sales tax should be excluded from the gross selling
price in computing the deficiency sales tax due from the petitioner, the opinion, apparently, is
merely an expression of his general or "private sentiment" on the particular issue, for he
concurred the dispositive part of the decision. At any rate, assuming that there is no valid
decision for lack of concurrence of two judges, the case was submitted for decision of the court
below on March 28, 1957 and under section 13 of Republic Act 1125, cases brought before said
court hall be decided within 30 days after submission thereof. "If no decision is rendered by the
Court within thirty days from the date a case is submitted for decision, the party adversely
affected by said ruling, order or decision, may file with said Court a notice of his intention to
appeal to the Supreme Court, and if no decision has as yet been rendered by the Court, the
aggrieved party may file directly with the Supreme Court an appeal from said ruling, order or
decision, notwithstanding the foregoing provisions of this section." The case having been
brought before us on appeal, the question raised by petitioner as become purely academic.

IN VIEW OF THE FOREGOING, the decision of the Court of Tax Appeals under review is
hereby modified in that petitioner shall be ordered to pay to respondent the sum of P820,549.91,
plus 25% surcharge thereon for late payment.

So ordered without costs.


G.R. No. L-5677 May 25, 1953 2. That the workers of the "La Campana Coffee Factory, Inc." are less than thirty-one;

LA CAMPANA FACTORY, INC., and TAN TONG doing business under the trial name "LA 3. That the petitioning union has no legal capacity to sue, because its registration as an
CAMPANA GAUGAU PACKING", petitioners, organized union has been revoked by the Department of Labor on September 5, 1951; and
vs.
KAISAHAN NG MGA MANGGAGAWA SA LA CAMPANA (KKM) and THE COURT OF
4. That there is an existing valid contract between the respondent "La Campana Gaugau
INDUSTRIAL RELATIONS, respondents.
Packing" and the intervenor PLOW, where-in the petitioner's members are contracting parties
bound by said contract.
REYES, J.:
Several hearings were held on the above motions, in the course of which ocular inspections
Tan Tong, one of the herein petitioners, has since 1932 been engaged in the business of buying were also made, and on the basis of the evidence received and the facts observed in the ocular
and selling gaugau under the trade name La Campana Gaugau Packing with an establishment inspections, the Court of Industrial Relations denied the said motions in its order of January 14,
in Binondo, Manila, which was later transferred to España Extension, Quezon City. But on July 1952, because if found as a fact that:
6, 1950, Tan Tong, with himself and members of his family corporation known as La Campana
Factory Co., Inc., with its principal office located in the same place as that of La Campana
A. While the coffee corporation is a family corporation with Mr. Tan Tong, his wife, and children
Gaugau Packing.
as the incorporations and stockhelders (Exhibit 1), the La Campana Gaugau Packing is merely a
business name (Exhibit 4).
About a year before the formation of the corporation, or on July 11, 1949, Tan Tong had entered
into a collective bargaining agreement with the Philippine Legion of Organized Workers, known
B. According to the contract of lease (Exhibit 23), Mr. Tan Tong., propriety and manager of the
as PLOW for short, to which the union of Tan Tong's employees headed by Manuel E. Sadde
Ka Campana Gaugau Factory, leased a space of 200 square meters in the bodega housing the
was then affiliated. Seceding, however, from the PLOW, Tan Tong's employees later formed
gaugau factory to his son Tan Keng Lim, manager of the La Campana Coffee Factory. But the
their own organization known as Kaisahan Ng Mga Manggagawa Sa La Campana, one of the
lease was executed only on September 1, 1951, while the dispute between the parties was
herein respondents, and applied for registration in the Department of Labor as an independent
pending before the Court.
entity. Pending consideration of this application, the Department gave the new organization legal
standing by issuing it a permit as an affiliate to the Kalipunan Ng Mga Manggagawa.
C. There is only one entity La Campana Starch and Coffee Factory, as shown by the signboard
(Exhibit 1), the advertisement in the delivery trucks (Exhibit I-1), the packages of gaugau(Exhibit
On July 19, 1951, the Kaisahan Ng Mga Manggagawa Sa La Campana, hereinafter to be
K), and delivery forms (Exhibits J, J-1, and J-2).
referred to as the respondent Kaisahan, which, as of that date, counted with 66 members —
workers all of them of both La Campana Gaugau Packing and La Campana Coffee Factory Co.,
Inc. — presented a demand for higher wages and more privileges, the demand being addressed D. All the laborers working in the gaugau or in the coffee factory receive their pay from the same
to La Campana Starch and Coffee Factory, by which name they sought to designate, so it person, the cashier, Miss Natividad Garcia, secretary of Mr. Tan Tong; and they are transferred
appears, the La Campana Gaugau Packing and the La Campana Coffee Factory Co., Inc. As from the gaugau to the coffee and vice-versa as the management so requires.
the demand was not granted and an attempt at settlement through the mediation of the
Conciliation Service of the Department of Labor had given no result, the said Department
certified the dispute to the Court of Industrial Relations on July 17, 1951, the case being there E. There has been only one payroll for the entire La Campana personnel and only one person
docketed as Case No. 584-V. preparing the same — Miss Natividad Garcia, secretary of Mr. Tan Tong. But after the case at
bar was certified to this Court on July 17, 1951, the company began making separate payrolls for
the coffee factory (Exhibits M-2 and M-3, and for the gaugau factory (Exhibits O-2, O-3 and O-4).
With the case already pending in the industrial court, the Secretary of Labor, on September 5, It is to be noted that before July 21, 1951, the coffee payrolls all began with number "41-Maria
1951, revoked the Kalipunan Ng Mga Kaisahang Manggagawa's permit as a labor union on the Villanueva" with 24 or more laborers (Exhibits M and M-1), whereas beginning July 21, 1951, the
strength of information received that it was dominated by subversive elements, and, in payrolls for the coffee factory began with No. 1-Loreta Bernabe with only 14 laborers (Exhibits
consequence, on the 20th of the same month, also suspended the permit of its affiliate, the M-2 and M-3).
respondent Kaisahan.
F. During the ocular inspection made in the factory on August 26, 1951 the Court has found the
We have it from the court's order of January 15, 1952, which forms one of the annexes to the following:
present petition, that following the revocation of the Kaisahan's permit, "La Campana Gaugau
and Coffee Factory" (obviously the combined name of La Campana Gaugau Packing and La
Campana Coffee Factory Co., Inc,) and the PLOW, which had been allowed to intervene as a In the ground floor and second floor of the gaugau factory there were hundreds of bags of raw
party having an interest in the dispute, filed separate motions for the dismissal of the case on the coffee behind the pile of gaugau sacks. There were also women employees working paper
wrappers for gaugau, and, in the same place there were about 3,000 cans to be used as
following grounds:
containers for coffee.

1. That the action is directed against two different entities with distinct personalities, with "La
The Court found out also that there were 16 trucks used both for the delivery of coffee and
Campana Starch Factory" and the "La Campana Coffee Factory, Inc.";
gaugau. To show that those trucks carried both coffee and gaugau, the union president invited
the Court to examine the contents of delivery truck No. T-582 parked in a garage between the
gaugau building and the coffee factory, and upon examination, there were found inside the said In the present case Tan Tong appears to be the owner of the gaugau factory. And the coffee
truck boxes of gaugau and cans of coffee, factory, though an incorporated business, is in reality owned exclusively by Tan Tong and his
family. As found by the Court of industrial Relations, the two factories have but one office, one
management and one payroll, except after July 17, the day the case was certified to the Court of
and held that:
Industrial Relations, when the person who was discharging the office of cashier for both
branches of the business began preparing separate payrolls for the two. And above all, it should
. . . there is only one management for the business of gaugau and coffee with whom the laborers not be overlooked that, as also found by the industrial court, the laborers of the gaugau factory
are dealing regarding their work. Hence, the filing of action against the Ka Campana Starch and and the coffee factory were interchangeable, that is, the laborers from the gaugau factory were
Coffee Factory is proper and justified. sometimes transferred to the coffee factory and vice-versa. In view of all these, the attempt to
make the two factories appears as two separate businesses, when in reality they are but one, is
but a device to defeat the ends of the law (the Act governing capital and labor relations) and
With regards to the alleged lack of personality, it is to be noted that before the certification of the should not be permitted to prevail.
case to this Court on July 17, 1951, the petitioner Kaisahan Ng Mga Manggagawa Sa La
Campana, had a separate permit from the Department of Labor. This permit was suspended on
September 30, 1951. (Exhibit M-Intervenor, page 55, of the record). It is not true that, on July 17, The second point raised by petitioners is likewise with-out merit. In the first place, there being
1951, when this case forwarded to this Court, the petitioner's permit, as an independent union, more than 30 laborers involved and the Secretary of Labor having certified the dispute to the
had not yet been issued, for the very Exhibit MM-Intervenor regarding the permit, conclusively Court of Industrial Relations, that court duly acquired jurisdiction over the case (International Oil
shows the preexistence of said permit. (Annex G.) Factory vs. NLU, Inc. 73 Phil., 401; section 4, C. A. 103). This jurisdiction was not when the
Department of Labor suspended the permit of the respondent Kaisahan as a labor organization.
For once jurisdiction is acquired by the Court of Industrial Relations it is retained until the case is
Their motion for reconsideration of the above order having been denied, Tan Tong and La
completely decided. (Manila Hotel Employees Association vs. Manila Hotel Co. et al., 73 Phil.,
Campana Coffee Factory, Inc. (same as La Campana Coffee Factory Co., Inc.), later joined by 374.)
the PLOW, filed the present petition for certiorari on the grounds that the Court of Industrial
Relations had no jurisdiction to take cognizance of the case, for the reason, according to them,
"(1) that the petitioner La Campana Coffee Factory, Inc. has only 14 employees, only 5 of whom In view of the foregoing, the petition is denied, with costs against the petitioner.
are members of the respondent union and therefore the absence of the jurisdictional number
(30) as provided by sections 1 and 4 of Commonwealth Act No. 103; and, (2) that the
suspension of respondent union's permit by the Secretary of Labor has the effect of taking away
the union's right to collective bargaining under section 2 of Commonwealth Act No. 213 and
consequently, its personality to sue for ad in behalf of its members."

As to the first ground, petitioners obviously do not question the fact that the number of
employees of the La Campana Gaugau Packing involved in the case is more than the
jurisdictional number (31) required bylaw, but they do contend that the industrial court has no
jurisdiction to try the case as against La Campana Coffee Factory, Inc. because the latter has
allegedly only 14 laborers and only of these are members of the respondent Kaisahan. This
contention loses force when it is noted that, as found by the industrial court — and this finding is
conclusive upon us — La Campana Gaugau Packing and La Campana Coffee Factory Co. Inc.,
are operating under one single management, that is, as one business though with two trade
names. True, the coffee factory is a corporation and, by legal fiction, an entity existing separate
and apart fro the persons composing it, that is, Tan Tong and his family. But it is settled that this
fiction of law, which has been introduced as a matter of convenience and to subserve the ends
of justice cannot be invoked to further an end subversive of that purpose.

Disregarding Corporate Entity. — The doctrine that a corporation is a legal entity existing
separate and apart from the person composing it is a legal theory introduced for purposes of
convenience and to subserve the ends of justice. The concept cannot, therefore, be extended to
a point beyond its reason and policy, and when invoked in support of an end subversive of this
policy, will be disregarded by the courts. Thus, in an appropriate case and in furtherance of the
ends of justice, a corporation and the individual or individuals owning all its stocks and assets
will be treated as identical, the corporate entity being disregarded where used as a cloak or
cover for fraud or illegality. (13 Am. Jur., 160-161.)

. . . A subsidiary or auxiliary corporation which is created by a parent corporation merely as an


agency for the latter may sometimes be regarded as identical with the parent corporation,
especially if the stockholders or officers of the two corporations are substantially the same or
their system of operation unified. (Ibid. 162; see Annotation 1 A. L. R. 612, s. 34 A. L. R. 599.)
G.R. No. L-20502 February 26, 1965 A factor that should not be overlooked is that Emilio and Rodolfo Cano are here indicted, not in
their private capacity, but as president and manager, respectively, of Emilio Cano Enterprises,
Inc. Having been sued officially their connection with the case must be deemed to be impressed
EMILIO CANO ENTERPRISES, INC., petitioner,
with the representation of the corporation. In fact, the court's order is for them to reinstate
vs.
Honorata Cruz to her former position in the corporation and incidentally pay her the wages she
COURT OF INDUSTRIAL RELATIONS, ET AL., respondents.
had been deprived of during her separation. Verily, the order against them is in effect against the
corporation. No benefit can be attained if this case were to be remanded to the court a
BAUTISTA ANGELO, J.: quo merely in response to a technical substitution of parties for such would only cause an
unwarranted delay that would work to Honorata's prejudice. This is contrary to the spirit of the
law which enjoins a speedy adjudication of labor cases disregarding as much as possible the
In a complaint for unfair labor practice filed before the Court of Industrial Relations on June 6,
technicalities of procedure. We, therefore, find unmeritorious the relief herein prayed for.
1956 by a prosecutor of the latter court, Emilio, Ariston and Rodolfo, all surnamed Cano, were
made respondents in their capacity as president and proprietor, field supervisor and manager,
respectively, of Emilio Cano Enterprises, Inc. WHEREFORE, petition is dismissed, with costs.

After trial, Presiding Judge Jose S. Bautista rendered decision finding Emilio Cano and Rodolfo
Cano guilty of the unfair labor practice charge, but absolved Ariston for insufficiency of evidence.
As a consequence, the two were ordered, jointly and severally, to reinstate Honorata Cruz, to
her former position with payment of backwages from the time of her dismissal up to her
reinstatement, together with all other rights and privileges thereunto appertaining.

Meanwhile, Emilio Cano died on November 14, 1958, and the attempt to have the case
dismissed against him having failed, the case was appealed to the court en banc, which in due
course affirmed the decision of Judge Bautista. An order of execution was issued on August 23,
1961 the dispositive part of which reads: (1) to reinstate Honorata Cruz to her former position as
ordered in the decision; and (2) to deposit with the court the amount of P7,222.58 within ten
days from receipt of the order, failing which the court will order either a levy on respondents'
properties or the filing of an action for contempt of court.

The order of execution having been directed against the properties of Emilio Cano Enterprises,
Inc. instead of those of the respondents named in the decision, said corporation filed an ex
parte motion to quash the writ on the ground that the judgment sought to be enforced was not
rendered against it which is a juridical entity separate and distinct from its officials. This motion
was denied. And having failed to have it reconsidered, the corporation interposed the present
petition for certiorari.

The issue posed before us is: Can the judgment rendered against Emilio and Rodolfo Cano in
their capacity as officials of the corporation Emilio Cano Enterprises, Inc. be made effective
against the property of the latter which was not a party to the case?

The answer must be in the affirmative. While it is an undisputed rule that a corporation has a
personality separate and distinct from its members or stockholders because of a fiction of the
law, here we should not lose sight of the fact that the Emilio Cano Enterprises, Inc. is a closed
family corporation where the incorporators and directors belong to one single family. Thus, the
following are its incorporators: Emilio Cano, his wife Juliana, his sons Rodolfo and Carlos, and
his daughter-in-law Ana D. Cano. Here is an instance where the corporation and its members
can be considered as one. And to hold such entity liable for the acts of its members is not to
ignore the legal fiction but merely to give meaning to the principle that such fiction cannot be
invoked if its purpose is to use it as a shield to further an end subversive of justice. 1 And so it
has been held that while a corporation is a legal entity existing separate and apart from the
persons composing it, that concept cannot be extended to a point beyond its reason and policy,
and when invoked in support of an end subversive of this policy it should be disregarded by the
courts (12 Am. Jur. 160-161).
G.R. No. L-28694 May 13, 1981 part, and that the illness for which compensation is sought is not an occupational disease,
hence, not compensable under the law. 8 The extension requested was denied. The Motion for
Reconsideration was likewise denied in an Order issued by the Chief of Section of the Regional
TELEPHONE ENGINEERING & SERVICE COMPANY, INC., petitioner,
Office dated December 28, 1967 9 predicated on two grounds: that the alleged mistake or
vs.
negligence was not excusable, and that the basis of the award was not the theory of direct
WORKMEN'S COMPENSATION COMMISSION, PROVINCIAL SHERIFF OF RIZAL and
causation alone but also on that of aggravation. On January 28, 1968, an Order of execution
LEONILA SANTOS GATUS, for herself and in behalf of her minor children, Teresita,
was issued by the same Office.
Antonina and Reynaldo, all surnamed GATUS, respondents.

On February 3, 1968, petitioner filed an "Urgent Motion to Compel Referee to Elevate the
MELENCIO-HERRERA, J.:
Records to the Workmen's Compensation Commission for Review." 10 Meanwhile, the Provincial
Sheriff of Rizal levied on and attached the properties of TESCO on February 17, 1968, and
These certiorari proceedings stem from the award rendered against petitioner Telephone scheduled the sale of the same at public auction on February 26, 1968. On February 28, 1968,
Engineering and Services, Co., Inc. (TESCO) on October 6, 1967 by the Acting Referee of the Commission issued an Order requiring petitioner to submit verified or true copies of the
Regional Office No. 4, Quezon City Sub-Regional Office, Workmen's Compensation Section, in Motion for Reconsideration and/or Petition to Set Aside Award and Order of December 28, 1967,
favor of respondent Leonila S. Gatus and her children, dependents of the deceased employee and to show proof that said Motion for Reconsideration was filed within the reglementary period,
Pacifico L. Gatus. The principal contention is that the award was rendered without jurisdiction as with the warning that failure to comply would result in the dismissal of the Motion. However,
there was no employer-employee relationship between petitioner and the deceased. before this Order could be released, TESCO filed with this Court, on February 22, 1968, The
present petition for "Certiorari with Preliminary Injunction" seeking to annul the award and to
enjoin the Sheriff from levying and selling its properties at public auction.
Petitioner is a domestic corporation engaged in the business of manufacturing telephone
equipment with offices at Sheridan Street, Mandaluyong, Rizal. Its Executive Vice-President and
General Manager is Jose Luis Santiago. It has a sister company, the Utilities Management On February 29, 1968, this Court required respondents to answer the Petition but denied
Corporation (UMACOR), with offices in the same location. UMACOR is also under the Injunction. 11 TESCO'S Urgent Motion dated April 2, 1968, for the issuance of a temporary
management of Jose Luis Santiago. restraining order to enjoin the Sheriff from proceeding with the auction sale of its properties was
denied in our Resolution dated May 8, 1968.
On September 8, 1964, UMACOR employed the late Pacifica L. Gatus as Purchasing Agent. On
May 16, 1965, Pacifico L. Gatus was detailed with petitioner company. He reported back to TESCO asserts:
UMACOR on August 1, 1965. On January 13, 1967, he contracted illness and although he
retained to work on May 10, 1967, he died nevertheless on July 14, 1967 of "liver cirrhosis with
I. That the respondent Workmen's Compensation Commission has no jurisdiction nor authority
malignant degeneration."
to render the award (Annex 'D', Petition) against your petitioner there being no employer-
employee relationship between it and the deceased Gatus;
On August 7, 1967, his widow, respondent Leonila S. Gatus, filed a "Notice and Claim for
Compensation" with Regional Office No. 4, Quezon City Sub-Regional Office, Workmen's
II. That petitioner can never be estopped from questioning the jurisdiction of respondent
Compensation Section, alleging therein that her deceased husband was an employee of
commission especially considering that jurisdiction is never conferred by the acts or omission of
TESCO, and that he died of liver cirrhosis. 1 On August 9, 1967, and Office wrote petitioner
the parties;
transmitting the Notice and for Compensation, and requiring it to submit an Employer's Report of
Accident or Sickness pursuant to Section 37 of the Workmen's Compensation Act (Act No.
3428). 2 An "Employer's Report of Accident or Sickness" was thus submitted with UMACOR III. That this Honorable Court has jurisdiction to nullify the award of respondent commission.
indicated as the employer of the deceased. The Report was signed by Jose Luis Santiago. In
answer to questions Nos. 8 and 17, the employer stated that it would not controvert the claim for
TESCO takes the position that the Commission has no jurisdiction to render a valid award in this
compensation, and admitted that the deceased employee contracted illness "in regular
suit as there was no employer-employee relationship between them, the deceased having been
occupation." 3 On the basis of this Report, the Acting Referee awarded death benefits in the
an employee of UMACOR and not of TESCO. In support of this contention, petitioner submitted
amount of P5,759.52 plus burial expenses of P200.00 in favor of the heirs of Gatus in a letter-
photostat copies of the payroll of UMACOR for the periods May 16-31, 1967 and June 1-15,
award dated October 6, 1967 4 against TESCO.
1967 12 showing the name of the deceased as one of the three employees listed under the
Purchasing Department of UMACOR. It also presented a photostat copy of a check of UMACOR
Replying on October 27, 1967, TESCO, through Jose Luis Santiago, informed the Acting payable to the deceased representing his salary for the period June 14 to July 13, 1967. 13
Referee that it would avail of the 15-days-notice given to it to state its non-conformity to the
award and contended that the cause of the illness contracted by Gatus was in no way
Both public and private respondents contend, on the other hand, that TESCO is estopped from
aggravated by the nature of his work. 5
claiming lack of employer – employee relationship.

On November 6, 1967, TESCO requested for an extension of ten days within which to file a
To start with, a few basic principles should be re-stated the existence of employer-employee
Motion for Reconsideration, 6 and on November 15, 1967, asked for an additional extension of
relationship is the jurisdictional foundation for recovery of compensation under the Workmen's
five days. 7 TESCO filed its "Motion for Reconsideration and/or Petition to Set Aside Award" on
Compensation Law. 14 The lack of employer-employee relationship, however, is a matter of
November 18, 1967, alleging as grounds therefor, that the admission made in the "Employer's
defense that the employer should properly raise in the proceedings below. The determination of
Report of Accident or Sickness" was due to honest mistake and/or excusable negligence on its
this relationship involves a finding of fact, which is conclusive and binding and not subject to
review by this Court. 15

Viewed in the light of these criteria, we note that it is only in this Petition before us that petitioner
denied, for the first time, the employer-employee relationship. In fact, in its letter dated October
27, 1967 to the Acting Referee, in its request for extension of time to file Motion for
Reconsideration, in its "Motion for Reconsideration and/or Petition to Set Aside Award," and in
its "Urgent Motion to Compel the Referee to Elevate Records to the Commission for Review,"
petitioner represented and defended itself as the employer of the deceased. Nowhere in said
documents did it allege that it was not the employer. Petitioner even admitted that TESCO and
UMACOR are sister companies operating under one single management and housed in the
same building. Although respect for the corporate personality as such, is the general rule, there
are exceptions. In appropriate cases, the veil of corporate fiction may be pierced as when the
same is made as a shield to confuse the legitimate issues. 16

While, indeed, jurisdiction cannot be conferred by acts or omission of the parties, TESCO'S
denial at this stage that it is the employer of the deceased is obviously an afterthought, a devise
to defeat the law and evade its obligations. 17 This denial also constitutes a change of theory on
appeal which is not allowed in this jurisdiction. 18 Moreover, issues not raised before the
Workmen's Compensation Commission cannot be raised for the first time on appeal. 19For that
matter, a factual question may not be raised for the first time on appeal to the Supreme Court. 20

This certiorari proceeding must also be held to have been prematurely brought. Before a petition
for certiorari can be instituted, all remedies available in the trial Court must be exhausted
first. 21 certiorari cannot be resorted to when the remedy of appeal is present. 22 What is sought
to be annulled is the award made by the Referee. However, TESCO did not pursue the remedies
available to it under Rules 23, 24 and 25 of the Rules of the Workmen's Compensation
Commission, namely, an appeal from the award of the Referee, within fifteen days from notice,
to the Commission; a petition for reconsideration of the latter's resolution, if adverse, to the
Commission en banc; and within ten days from receipt of an unfavorable decision by the latter,
an appeal to this Court. As petitioner had not utilized these remedies available to it, certiorari win
not he, it being prematurely filed. As this Court ruled in the case of Manila Jockey Club, Inc. vs.
Del Rosario, 2 SCRA 462 (1961).

An aggrieved party by the decision of a Commissioner should seek a reconsideration of the


decision by the Commission en banc. If the decision is adverse to him, he may appeal to the
Supreme Court. An appeal brought to the Supreme Court without first resorting to the remedy
referred to is premature and may be dismissed.

Although this rule admits of exceptions, as where public welfare and the advancement of public
policy so dictate, the broader interests of justice so require, or where the Orders complained of
were found to be completely null and void or that the appeal was not considered the appropriate
remedy, 23 the case at bar does not fan within any of these exceptions. WHEREFORE, this
Petition is hereby dismissed.

SO ORDERED.
G.R. No. L-68661 July 22, 1986 decision to effect a shutdown on September 8, 1982 and to circularize a memorandum on
November 2, 1982 announcing the cessation of operations.
NATIONAL FEDERATION OF LABOR UNION (NAFLU) AND TERESITA LORENZO, ET
AL., petitioners, The company alleged further that it had no more plant and building because they were allegedly
vs. repossessed by the Pioneer Texturizing Corporation for the failure of respondent to pay rentals
HON. MINISTER BLAS OPLE, as Minister of Labor and Employment; LAWMAN as evidenced by the letter of Mr. Eugenio Tan dated August 10, 1982 stating that respondent is
INDUSTRIAL/LIBRA GARMENTS/DOLPHIN ENTERPRISES, respondents. given fifteen (15) days to settle its accounts, otherwise an action for repossession and ejectment
would be instituted against it.
GUTIERREZ, JR., J.:
Nonetheless, the company offered to pay every employee affected by the shutdown a separation
pay of P328.95 each.
The only issue raised in this petition is whether or not, on the basis of the findings of the public
respondent that the respondent company was guilty of unfair labor practice, the petitioners
should be reinstated to their former positions without loss of seniority rights and with full On June 6, 1983, the National Federation of Labor Unions (NAFLU) submitted a position paper
backwages. alleging that it was certified by the Bureau of Labor Relations as the sole and exclusive
bargaining agent of all the rank and file employees of the said factory. Negotiations followed in
October 1981 until January 1982. The management refused to grant substantial economic
The background facts which led to the filing of the instant petition are summarized in the
demands to the workers, hence, the union declared a strike in July 1982. Thru the efforts of the
assailed decision as follows:
Bureau of Labor Relations, the strike was settled in July 1982. The management agreed as
follows: Wage increase, Pl.00 for the first year; Pl.00 for the second year and P1.00 for the third
On September 8, 1982, the National Federation of Labor Union (NAFLU) filed a request for year of the contract. Vacation and sick leaves were also granted and other fringe benefits. The
conciliation before the Bureau of Labor Relations requesting for the intervention in its dispute collective bargaining agreement was suppose to be effective September 1982.
with management involving certain money claims, refusal to conclude a collective agreement
after such has been negotiated and run-away shop undertaken by management in order to bust
But the actual partial shutdown began in August 1982. It appears moreover that at night,
the union.
machines were dismantled, hauled out and then installed at No. 43 Engineering Road, Araneta
University compound, Malabon, Metro Manila and the name of Lawman was changed to LIBRA
Several conferences were conducted by the Bureau to settle the dispute amicably. In the course GARMENTS. Under that name, new applicants for employment were called even as the
of the proceedings, however, management unilaterally declared a temporary shutdown on company continued to manufacture the same products but under the name of LIBRA
September 15, 1982. "On September 23, 1982, the management of Lawman Industrial promised GARMENTS. When this was discovered by the workers, LIBRA GARMENTS was changed to
the union 'that it will start the normalization of operations at Lawman effective January, 1983. DOLPHIN GARMENTS.

On October 11, 1982, after all efforts to mediate the charges of unfair labor practice and non- On March 17,1983, the Minister of Labor and Employment issued an order, stating:
payment of certain money claims have failed, the union filed its notice of strike.
In view of the foregoing, this office hereby assumes jurisdiction over the dispute at Lawman
On November 9, 1982, the firm offered payment of P200,000. as complete settlement of all Industrial Corporation pursuant to Art. 264 (g) of the Labor Code. All employees affected by the
claims inclusive of the separation pay from the company. The union rejected the offer which it extended shutdown which is highly irregular, are ordered to return to work and management is
felt was tantamount to a proposal to eliminate the union and final separation of its members from directed to accept all returning workers under the same terms and conditions prevailing previous
the company. to the illegal shutdown. Management is further directed to pay severance compensation
including all unpaid wages previous to the shutdown and after March 15, 1983 in the event that
the company cannot resume operations, All pending cases including Case No. 11-695-82
Efforts of conciliation proved futile. Until the last conference on January 6, 1983, the company
(NAFLU v. Lawman) are hereby ordered consolidated to this Office for resolution. Pending the
had failed to resume operations alleging poor business conditions. determination of the charges on illegal lockout run-away-shop and the pending money claims
against the company, Lawman Industrial is hereby enjoined from transferring ownership or
Meanwhile, the union filed a complaint for unfair labor practice against the management of otherwise effecting any encumbrance or any of its existing assets in favor of any third party
Lawman sometime December 1982 docketed as Case No. 11-695-82 (NAFLU v. Lawman) without a prior clearance from this Office and timely notice to the union. The company is likewise
pending before the Metro Manila Branch of the NLRC. prohibited from terminating the employment of any of its employees pending the outcome of this
dispute.
Notwithstanding the commitment of management to resume operations in January, 1983 and
even with the expiration on March 15, 1983 of the provisional shutdown, the period of shutdown This order automatically enjoins a strike or lockout.
was extended without notifying this Office of such extension. On March 17, 1983, this Office
issued the Order now in question. On July 31, 1984, the public respondent modified its earlier order and directed the private
respondent to pay all accrued wages and benefits including a one month's pay for its failure to
On May 20, 1983, respondent filed a motion for reconsideration alleging that it had suffered comply with the requirement of notice under Batas Pambansa Blg. 130, as amended and
losses as shown by its financial statements. In view thereof, it informed this Ministry of its separation pay for all dismissed employees equivalent to one month's pay or one-half month's
pay for every year of service whichever is higher computed up to January, 1983 when the Maritima v. United Seamen's Union, 104 Phil. 7; Talisay Silay Mining Co. v. Court of Industrial
company had declared its intention to actually close its operations. However, despite a finding Relations, 106 Phil. 1081; Velez v. PAV Watchmen's Union, 107 Phil. 689; Phil. Sugar Institute
that the private respondent company was guilty of unfair labor practice, the public respondent v. Court of Industrial Relations, et al., 109 Phil. 452; Big Five Products Workers Union v. Court of
did not order the reinstatement of the employees concerned "because the company has Industrial Relations, 8 SCRA 559; and MD Transit and Taxi Co. v. De Guzman, 7 SCRA 726).
declared that it had already ceased its operations completely." It is this order for non-
reinstatement which is now before us.
After finding that Lawman Industrial Corporation had transferred its business operations to Libra
Garments Enterprises, which later changed its name to Dolphin Garments Enterprises, the
The petition is impressed with merit. public respondent cannot deny reinstatement to the petitioners simply because Lawman
Industrial Corporation has ceased its operations.
We see no reason to disturb the findings of fact of the public respondent, supported as they are
by substantial evidence in the light of the well established principle that findings of administrative As Libra/Dolphin Garments is but an alter-ego of the old employer, Lawman Industrial, the
agencies which have acquired expertise because their jurisdiction is confined to specific matters former must bear the consequences of the latter's unfair acts by reinstating the petitioners to
are geiterary accorded not only respect but at times even finality, and that judicial review by this their former positions without loss of seniority rights (See Phil. Land-Air-Sea Labor Union
Court on labor cases does not go so far as to evaluate the sufficiency of the evidence upon (PLASLU) v. Sy Indong Co. Rice and Corn Mill, 11 SCRA 277).
which the Deputy Minister and the Regional Director based their determinations but are limited
to issues of jurisdiction or grave abuse of discretion (Special Events and Central Shipping Office
To justify its closure, the respondent company argues that it can no longer continue its
Workers Union v. San Miguel Corporation, 122 SCRA 557).
operations due to serious losses, and in support thereof, presented its financial statements for
1980-1981 and from January to June, 1986.
The findings of the Minister of Labor and Employment embodied in its July 31, 1984 decision are
categorical:
The alleged losses of the respondent company are more apparent than real. The argument of
the private respondent are refuted by the petitioners:
It is clear from the records of this case that the company bargained in bad faith with the union
when pending the negotiation of their collective agreement, the company declared a temporary
As of December 1981, LAWMAN's Cost of Goods Manufactured and Sold was P2,065,822.26
cessation of its operations which in reality was an illegal lockout. Evidently, the company also
while on June 30, 1982, it was P 3,768,609.22. The alleged reason was the entry of Direct Labor
maintained run-away shop when it started transferring its machine first to Libra and then to
under the 'Statement of Cost of Goods Manufactured and Sold' amounting to P 1,703,768.27 for
Dolphin Garments. Failure on the part of the company to comply with the requirements of notice
1982. This could only mean that there was a sudden increase in production of LAWMAN
and due process to the employees and the Labor Ministry one month before the intended
necessitating an additional and huge labor cost, Comparing this with the past year (1981), the
'closure' of the firm is clearly against the law.
entry for Direct Labor was only P 398,863.40. This tremendous increase in Direct Labor for the
six months ending June 1982 was not sufficiently explained by LAWMAN in the proceedings
There is also evidence on the record that even after the alleged 'shutdown' the company was still below.
operating in the name of Lawman Industrial although production was being carried out by
another firm called Libra Garments (later Dolphin Garments). When the company declared in its
Even on the entry Administrative Salaries has been increased to justify losses. For June 30,
position paper dated May 20, 1983 that all the machines of Lawman had been repossessed by
1982, LAWMAN spent a sizable P213,752.85 whereas for December 30, 1981, it only spent
the owner, Pioneer Texturizing Corporation, it admitted the fact that it has violated the 17 March
P47,889.20 without any justifiable reason at all.
Order of this Office enjoining any encumbrance or transfer of the properties of Lawman without
prior clearance from this Office. The evident bad faith, fraud and deceit committed by the
company to the prejudice of both the union and the employees who have existing wage claims, In addition, the Solicitor General submits the following observations: xxx xxx xxx
some of which are due for execution, leads us to affirm the union's position that the veil of
corporate fiction should be pierced in order to safeguard the right to self-organization and certain
... [T]he net sales of LAWMAN for the year 1981 was, P2,117,203.95 whereas for the shorter
vested rights which had accrued in favor of the union.
period of January to June 1982, its next sales was already P2,359,479.25, surpassing its entire
1981 sales. This clearly shows that the firm was experiencing a sales upswing at the time of its
It is very obvious from the above findings that the second corporation seeks the protective shield shutdown,
of a corporate fiction to achieve an illegal purpose. As enunciated in the case of Claparols v.
Court of Industrial Relations (65 SCRA 613) its veil in the present case should, therefore, be
pierced as it was deliberately and maliciously designed to evade its financial obligations to its Following the precedent set in Lepanto Consolidated Mining Co. v. Encarnacion et al (136 SCRA
256) and cases cited therein, the petitioner-workers should be reinstated but with backwages not
employees. It is an established principle that when the veil of corporate fiction is made as a
shield to perpetrate a fraud or to confuse legitimate issues (here, the relation of employer- exceeding three years.
employee), the same should be pierced (A.D. Santos, Inc. v. Vasquez, 22 SCRA 1156).
WHEREFORE, the petition for review is GRANTED. The appealed decision dated July 31, 1984
Thus, as Lawman Industrial Corporation was guilty of unfair labor practice, the public is hereby SET ASIDE. The private respondent is ordered to reinstate the petitioners to positions
respondent's order for reinstatement should follow as a matter of right. In National Mines and in LIBRA/DOLPHIN GARMENTS with backwages of not more than three (3) years each and
without loss of seniority rights and benefits being enjoyed by them prior to the alleged closure of
Allied Workers Union v. National Labor Relations Commission (118 SCRA 637), this Court held
that it is an established rule that an employer who commits an unfair labor practice may be Lawman's Industrial Corporation. SO ORDERED.
required to reinstate with fun backwages the workers affected by such act (See also Compana
G.R. No. L-69494 May 29, 1987 Quirino Avenue, Paranaque, Rizal. The compound, building, plant, equipment, machinery,
laboratory and bodega were the same as those occupied and used by RANSOM. The UNION
claims that ROSARIO thrives to this day.
A.C. RANSOM LABOR UNION-CCLU, petitioner,
vs.
NATIONAL LABOR RELATIONS COMMISSION, First Division A.C. RANSOM (PHIIS.) Writs of execution were issued successively against RANSOM on June 23, 1976, and February
CORPORATION RUBEN HERNANDEZ, MAXIMO C. HERNANDEZ, SR., PORFIRIO R. 17, 1977, to no avail.
VALENCIA, LAURA H. CORNEJO, FRANCISCO HERNANDEZ, CELESTINO C.
HERNANDEZ and MA. ROSARIO HERNANDEZ, respondents.
On December 18, 1978, the UNION again filed an ex-parte Motion for Writ of Execution and
Garnishment praying that the Writ issue against the Officers/Agents of RANSOM personally and
RESOLUTION or their estates, as the case may be, considering their success in hiding or shielding the assets
of said company. RANSOM countered that the CIR Decision, dated August 19, 1972, could no
longer be enforced by mere Motion because more than five (5) years had already lapsed.
MELENCIO-HERRERA, J.:

Acting on the Motion, Labor Arbiter Tito F. Genilo issued, on March 11, 1980, an Order, the
In a joint Decision in two earlier cases rendered by the then Court of Industrial Relations (CIR)
pertinent part of which reads:
on August 19, 1972, it declared in the dispositive portion thereof:

Under the circumstances and pursuant to the decision aforementioned, especially that portion
IN VIEW OF ALL THE FOREGOING, ... the A.C. Ransom Philippine Corporation is guilty of
holding the respondent corporation's officers and agents liable, the following officers of the
unfair labor practice of interference and discrimination herein above held and specified; ordering
respondent corporation — as appears in the record-are hereby deemed included parties
said corporation, its officers and agents to cease and desist from committing the same: finding
respondents in their official capacity:
the strike legal and justified; and to reinstate immediately ... , to their respective positions with
backwages from July 25, 1969 until actually reinstated, without loss of seniority rights and other
privileges appurtenant to their employment. (Emphasis supplied). 1 a) Ruben Hernandez (President, per his testimony on August 21, 1974);

This Court affirmed that Decision when it denied the Petition for Review filed by RANSOM on b) Maximo C. Hernandez, Jr. (Director);
February 26, 1973 in G.R. Nos. L-36226-68.
c) Porfirio N. Valencia (Director);
The backwages due the 22 employees having been computed at P 199,276.00 by the (CIR)
Examiner, successive Motions for Execution were filed by the UNION on January 27, 1973 and
d) Laura H. Cornejo (Director);
March 1, 1973, all of which RANSOM opposed stressing its "precarious financial position if
immediate execution of the backwages would be ordered." Upon the UNION's Motion of April 22,
1973 asking the CIR that RANSOM be ordered to deposit with the Court the backwages due e) Francisco Hernandez (Chairman of the Board);
them. RANSOM manifested that it did not have the necessary funds to deposit and asked that
the employees' earnings elsewhere during this suspension be deducted. After several hearings,
a recomputation was made and the award of P199,276.00 was reduced to P 164,984.00. 2 f) Celestino C. Hernandez (Director); and

The records show that, upon application filed by RANSOM on April 2, 1973, it was granted g) Ma. Rosario Hernandez (Director).
clearance by the Secretary of Labor on June 7, 1973 to cease operation and terminate
employment effective May 1, 1973, without prejudice to the right of subject employees to seek Consequently, let a writ of execution be issued for P 164,984.00 against respondent corporation
redress of grievances under existing laws and decrees. 3 The reasons given by RANSOM for the and its officers/agents enumerated above.
clearance application were financial difficulties on account of obligations incurred prior to 1966.
SO ORDERED. (Emphasis supplied) 4
On January 21, 1974, the UNION filed another Motion for Execution alleging that although
RANSOM had assumed a posture of suffering from business reverse, its officers and principal
stockholders had organized a new corporation, the Rosario Industrial Corporation (thereinafter It appears that among the persons named in the aforequoted Order, Ma. Rosario Hernandez
called ROSARIO), using the same equipment, personnel, business stocks and the same place died in 1971; Francisco Hernandez died in 1977: and Celestino C. Hernandez passed away in
of business. For its part, RANSOM declared that ROSARIO is a distinct and separate 1979. And Maximo Hernandez who was named in the CIR Decision, died in 1966. 5
corporation, which was organized long before these instant cases were decided adversely
against RANSOM. The NLRC, on appeal, modified the Decision by relieving the officers and agents of liability as
follows:
It appears that sometime in 1969, ROSARIO, a closed corporation, was, in fact, established. It
was engaged in the same line of business as RANSOM with the same Hernandez family as the As to the liability of the respondent's officers and agents, we agree with the contention of the
owners, the same officers, the same President, the same counsel and the same address at 555 respondent-appellant that there is nothing in the order dated March 11, 1980 that would justify
the holding of the individual officers and agents of respondent in their personal capacity. As a RANSOM's officers and agents of liability. It is also for that reason that in our Decision of June
general rule, officers of the corporation are not liable personally for the official acts unless they 10, 1986 we set aside said NLRC Decision and reinstated the Order of Labor Arbiter Genilo, with
have exceeded the scope of their authority. In the absence of evidence showing that the officers modification, in that we limited liability for backwages due the 22 UNION members to the
mentioned in the Order of the Labor Arbiter dated March 11, 1980 have exceeded their authority, President of RANSOM in 1974 jointly and severally with other Presidents of the same
the writ of execution can not be enforced against them, especially' so since they were not given corporation who had been elected as such after 1972 or up to the time the corporation life was
a chance to be heard. terminated, since the President should also be deemed included in the term "employer. "

WHEREFORE, the Order appealed from is hereby affirmed, except as modified above. The foregoing, however, limits the scope of liability and deviates from the CIR Decision, affirmed
by this Court in 1973, holding the officers and agents of RANSOM liable. In other words, the
officers and agents listed in the Genilo Order except for those who have since passed away,
SO ORDERED. 6
should, as affirmed by this Court, be held jointly and severally liable for the payment of
backwages to the 22 strikers.
Reconsideration sought by the UNION from the NLRC was denied, hence this special civil action
of Certiorari.
This finding does not ignore the legal fiction that a corporation has a personality separate and
distinct from its stockholders and members, for, as this Court had held "where the incorporators
On June 10, 1986, this Court promulgated its Decision, the dispositive portion of which decrees: and directors belong to a single family, the corporation and its members can be considered as
one in order to avoid its being used as an instrument to commit injustice," 10 or to further an end
subversive of justice. 11 In the case of Claparols vs. CIR 12 involving almost similar facts as in
WHEREFORE, the questioned Decision of the National Labor Relations Commission is SET this case, it was also held that the shield of corporate fiction should be pierced when it is
ASIDE, and the Order of the Labor Arbiter Tito F. Genilo of March 11, 1980 is reinstated with the
deliberately and maliciously designed to evade financial obligations to employees. To the same
modification that personal liability for the backwages due the 22 strikers shall be limited to effect was this Court's rulings in still other cases:
Ruben Hernandez, who was President of RANSOM in 1974, jointly and severally with other
Presidents of the same corporation who had been elected as such after 1972 or up to the time
the corporate life was terminated. When the notion of legal entity is used as a means to perpetrate fraud or an illegal act or as a
vehicle for the evasion of an existing obligation, the circumvention of statutes, and or confuse
legitimate issues the veil which protects the corporation will be lifted (Villa Rey Transit, Inc. vs.
Both parties have moved for reconsideration. Private respondents point out that they were never
Ferrer, 25 SCRA 846 [1968]; Republic vs. Razon, 20 SCRA 234 [1967]; A.D. Santos, Inc. vs.
impleaded as parties in the Trial Court, and that their personal liabilities were never at issue; that Vasquez, 22 SCRA 1156 [1968]; Telephone Eng'g. & Service Company, Inc. vs. WCC, 104
judgment holding Ruben Hernandez personally liable is tantamount to deprivation of property SCRA 354 [1981]).
without due process of law; and that he was not an officer of the corporation at the time the
unfair labor practices were committed.
The alleged bankruptcy of RANSOM furnishes no justification for non-payment of backwages to
the employees concerned taking into consideration Article 110 of the Labor Code, which
The UNION on the other hand, in its own Motion for Reconsideration, prays that the veil of provides:
corporate fiction be pierced and that the Decision be modified, in that all the individual private
respondents and not only the President, should be held jointly and severally liable with
RANSOM. On November 4, 1986, it further filed an Urgent Motion for Preliminary Mandatory ART. 110. Worker preference in case of bankruptcy. - In the event of bankruptcy or liquidation of
Injunction "directing private respondents to deposit the amount of P 199,276.00 or to put up a an employer's business, his workers shall enjoy first preference as regards wages due them for
supersedeas bond of the same sum." services rendered during the period prior to the bankruptcy or liquidation, any provision of law to
the contrary notwithstanding. Unpaid wages shag be paid in full before other creditors may
establish any claim to a share in the assets of the employer.
Incontrovertible is the fact that RANSOM was found guilty by the CIR, in its Decision of August
19, 1972, of unfair labor practice; that its officers and agents were ordered to cease and desist
from further committing acts constitutive of the same, and to reinstate immediately the 22 union The term "wages" refers to all remunerations, earnings and other benefits in terms of money
members to their respective positions with backwages from July 25, 1969 until actually accruing to the employees or workers for services rendered. They are to be paid in full before
reinstated. other creditors may establish any claim to a share in the assets of the employer.

The CIR Decision became final, conclusive, and executory after this Court denied the RANSOM Section 10. Payment of wages in case of bankruptcy.-Unpaid wages earned by the employees
petition for review in 1973. In other words, this Court upheld that portion of the judgment before the declaration of bankruptcy or judicial liquidation of the employer's business shall be
ordering the officers and agents of RANSOM to reinstate the laborers concerned, with given first preference and shall be paid in full before other creditors may establish any claim to a
backwages. The inclusion of the officers and agents was but proper since a corporation, as an share in the assets of the employer. 13
artificial being, can act only through them. It was also pursuant to the CIR Act (CA No. 103
), 7 the Industrial Peace Act (R.A. 875) 8 the Minimum Wage Law (R.A. 602). 9 Consequently,
The foregoing provisions are but in consonance with the principles of social justice and
when, in resolving the UNION's Motion for Writ of Execution and Garnishment in the Order of
protection to labor guaranteed by past and present Constitutions and are not really being given
March 11, 1980, Labor Arbiter Genilo named the seven (17) private respondents herein as the
any retroactive effect when applied herein.
RANSOM officers and agents, who should be held liable (supra), he merely implemented the
already final and executory CIR decision of August 19, 1972. The NLRC, on appeal to it by
RANSOM, could not have modified the CIR Decision, as affirmed by this Court, by relieving
The Decision of the CIR was rendered on August 19, 1972. Clearance to RANSOM to cease The plea of the UNION for the restoration of the original computation of P199,276.00 or to grant
operations and terminate employment granted by the Secretary of Labor was made effective on the 22 Union members three (3) years backwages is rejected. It is the amount of P164,984.00
May 1, 1973. The right of the employees concerned to backwages awarded them, therefore, had as backwages, which was the subject of the Writ of Execution issued by the Labor Arbiter
already vested at the time and even before clearance was granted. Note should also be taken of pursuant to the CIR Decision of 1972.
the fact that the clearance was without prejudice to the right of subject employees to seek
redress of grievances under existing laws and decrees.
With the conclusions arrived at, the UNION's Urgent Motion for a Writ of Preliminary Mandatory
Injunction directing private respondents to deposit the amount due as backwages in the
The worker preference applies even if the employer's properties are encumbered by means of a meantime, need no longer be acted on.
mortgage contract, as in this case. So that, when machinery and equipment of RANSOM were
sold to Revelations Manufacturing Corporation for P 2M in 1975, the right of the 22 laborers to
A final and executory Decision in favor of the UNION obtained in 1972 and affirmed by this Court
be paid from the proceeds should have been recognized, even though it is claimed that those
in 1973 has remained unsatisfied to this date despite no less than ten (10) Motions for Execution
proceeds were turned over to the Commercial Bank and Trust Company (Comtrust) in payment
over a period of fourteen (14) years, not to mention the fact that this is the second time that this
of RANSOM obligations, since the workers' preference is over and above the claim of other
case is before this Court. The detriment and prejudice caused the employees concerned is
creditors.
subversive of the ends of justice. This protracted litigation must end and labor should now enjoy
the just deserts of its legal victory.
The contention, therefore, of the heirs of the late Maximo C. Hernandez, Sr. that since they paid
from their own personal funds the balance of the amount owing by RANSOM to Comtrust they
ACCORDINGLY, private respondents' Motion for Reconsideration is hereby denied with
are the "preferential creditors" of RANSOM, is clearly without merit. Workers are to be paid in full
FINALITY; the Motion for Reconsideration filed by petitioner is granted in part; and the
before other creditors may establish any claim to a share in the assets of the employer.
dispositive portion of the Decision, dated June 10, 1986, is hereby amended to read as follows:

... even if the employer's properties are encumbered by means of a mortgage contract, still the
WHEREFORE, the questioned Decision of the National Labor Relations Commission is SET
workers' wages which enjoy first preference in case of bankruptcy or liquidation are duly
ASIDE, and the Order of Labor Arbiter Tito F. Genilo of March 11, 1980 is reinstated with the
protected by an automatic first lien over and above all other earlier encumbrances on the said
modification that Rosario Industrial Corporation and its officers and agents are hereby held
properties. Otherwise, workers' wages may be imperilled by foreclosure of mortgages, and as a
jointly and severally liable with the surviving private respondents for the payment of the
consequence, the aforecited provision of the New Labor Code would be rendered
backwages due the 22 union members.
meaningless. 14

Rosario Industrial Corporation is hereby ordered to reinstate the 22 union members or, if this is
Aggravating RANSOM's clear evasion of payment of its financial obligations is the organization
not possible, to award them separation pay equivalent at least to one (1) month pay or to one (1)
of a "run-away corporation," ROSARIO, in 1969 at the time the unfair labor practice case was
month salary for every year of service actually rendered by them with A.C. Ransom (Phils).
pending before the CIR by the same persons who were the officers and stockholders of
Corporation, whichever is higher.
RANSOM, engaged in the same line of business as RANSOM, producing the same line of
products, occupying the same compound, using the same machineries, buildings, laboratory,
bodega and sales and accounts departments used by RANSOM, and which is still in existence. This decision is immediately executory.
Both corporations were closed corporations owned and managed by members of the same
family. Its organization proved to be a convenient instrument to avoid payment of backwages
and the reinstatement of the 22 workers. This is another instance where the fiction of separate SO ORDERED.
and distinct corporate entities should be disregarded.

It is very obvious that the second corporation seeks the protective shield of a corporate fiction
whose veil in the present case could, and should, be pierced as it was deliberately and
maliciously designed to evade its financial obligation to its employees.

... When a notion of legal entity is used to. defeat public convenience, justify wrong, protect
fraud, or defend crime, the law will regard the corporation as an association or persons, or, in the
case of two corporations, will merge them into one. 15

The corporation will be treated merely as an aggregation of individuals or, where there are two
corporations, they will be merged as one, the one being merely regarded as part of the
instrumentality of the other. 16

The UNION's plea, therefore, for the reinstatement of the 22 strikers in ROSARIO should be
favorably heard. However, ROSARIO shall have the option to award them separation pay
equivalent to one-half month for every year of service actually rendered by the 22 strikers.
G.R. No. L-10510 March 17, 1961 Defendant-appellee RICARDO RODRIGUEZ is hereby ordered to pay to the plaintiffs-appellants
Dominga de los Reyes and Sabino Padilla the sum of P1,742.64 with legal interest thereon from
the time of the filing of the complaint and until it is fully paid. In addition thereto the defendants-
M. MC CONNEL, W. P. COCHRANE, RICARDO RODRIGUEZ, ET AL., petitioners,
appellees Cirilo Paredes, Ursula Tolentino and Ricardo Rodriguez shall pay the costs
vs.
proportionately in both instances.
THE COURT OF APPEALS and DOMINGA DE LOS REYES, assisted by her husband,
SABINO PADILLA,respondents.
IT IS SO ORDERED.
REYES, J.B.L., J.:
Cirilo Paredes and Ursula Tolentino then resorted to this court. We granted certiorari.
The issue before us in the correctness of the decision of the Court of Appeals that, under the
circumstances of record, there was justification for disregarding the corporate entity of the Park On the main issue whether the individual stockholders maybe held liable for obligations
Rite Co., Inc., and holding its controlling stockholders personally responsible for a judgment contracted by the corporation, this Court has already answered the question in the affirmative
against the corporation. wherever circumstances have shown that the corporate entity is being used as an alter ego or
business conduit for the sole benefit of the stockholders, or else to defeat public convenience,
justify wrong, protect fraud, or defend crime (Koppel [Phil.] Inc. vs. Yatco, 77 Phil. 496; Arnold
The Court of Appeals found that the Park Rite Co., Inc., a Philippine corporation, was originally
vs. Willits and Patterson, 44 Phil. 364).
organized on or about April 15, 1947, with a capital stock of 1,500 shares at P1.00 a share. The
corporation leased from Rafael Perez Rosales y Samanillo a vacant lot on Juan Luna street
(Manila) which it used for parking motor vehicles for a consideration. The Court of Appeals has made express findings to the following effect:

It turned out that in operating its parking business, the corporation occupied and used not only There is no question that a wrong has been committed by the so-called Park Rite Co., Inc., upon
the Samanillo lot it had leased but also an adjacent lot belonging to the respondents-appellees the plaintiffs when it occupied the lot of the latter without its prior knowledge and consent and
Padilla, without the owners' knowledge and consent. When the latter discovered the truth around without paying the reasonable rentals for the occupation of said lot. There is also no doubt in our
October of 1947, they demanded payment for the use and occupation of the lot. mind that the corporation was a mere alter ego or business conduit of the defendants Cirilo
Paredes and Ursula Tolentino, and before them — the defendants M. McConnel, W. P.
Cochrane, and Ricardo Rodriguez. The evidence clearly shows that thesepersons completely
The corporation (then controlled by petitioners Cirilo Parades and Ursula Tolentino, who had
dominated and controlled the corporation and that the functions of the corporation were solely
purchased and held 1,496 of its 1,500 shares) disclaimed liability, blaming the original
for their benefits.
incorporators, McConnel, Rodriguez and Cochrane. Whereupon, the lot owners filed against it a
complaint for forcible entry in the Municipal Court of Manila on 7 October 1947 (Civil Case No.
4031). When it was originally organized on or about April 15, 1947, the original incorporators were M.
McConnel, W. P. Cochrane, Ricardo Rodriguez, Benedicto M. Dario and Aurea Ordrecio with a
capital stock of P1,500.00 divided into 1,500 shares at P1.00 a share. McConnel and Cochrane
Judgment was rendered in due course on 13 November 1947, ordering the Park Rite Co., Inc. to
each owned 500 shares, Ricardo Rodriguez 408 shares, and Dario and Ordrecio 1 share each.
pay P7,410.00 plus legal interest as damages from April 15, 1947 until return of the lot.
It is obvious that the shares of the last two named persons were merely qualifying shares. Then
Restitution not having been made until 31 January 1948, the entire judgment amounted to
or about August 22, 1947 the defendants Cirilo Paredes and Ursula Tolentino purchased 1,496
P11,732.50. Upon execution, the corporation was found without any assets other than P550.00
shares of the said corporation and the remaining four shares were acquired by Bienvenido J.
deposited in Court. After their application to the judgment credit, there remained a balance of
Claudio, Quintin C. Paredes, Segundo Tarictican, and Paulino Marquez at one share each. It is
P11,182.50 outstanding and unsatisfied.
obvious that the last four shares bought by these four persons were merely qualifying shares
and that to all intents and purposes the spouses Cirilo Paredes and Ursula Tolentino composed
The judgment creditors then filed suit in the Court of First Instance of Manila against the the so-called Park Rite Co., Inc. That the corporation was a mere extension of their personality is
corporation and its past and present stockholders, to recover from them, jointly and severally, shown by the fact that the office of Cirilo Paredes and that of Park Rite Co., Inc. were located in
the unsatisfied balance of the judgment, plus legal interest and costs. The Court of First Instance the same building, in the same floor and in the same room — at 507 Wilson Building. This is
denied recovery; but on appeal, the Court of Appeals (CA-G.R. No. 8434-R) reversed, finding further shown by the fact that the funds of the corporation were kept by Cirilo Paredes in his own
that the corporation was a mere alter ego or business conduit of the principal stockholders that name (p. 14, November 8, 1950, T.S.N.) The corporation itself had no visible assets, as correctly
controlled it for their own benefit, and adjudged them responsible for the amounts demanded by found by the trial court, except perhaps the toll house, the wire fence around the lot and the
the lot owners, as follows: signs thereon. It was for this reason that the judgment against it could not be fully satisfied.
(Emphasis supplied).
WHEREFORE, premises considered, the decision appealed from is reversed. Defendants-
appellees Cirilo Paredes and Ursula Tolentino are hereby declared liable to the plaintiffs- The facts thus found can not be varied by us, and conclusively show that the corporation is a
appellants for the rentals due on the lot in question from August 22, 1947 to January 31, 1948 at mere instrumentality of the individual stockholder's, hence the latter must individually answer for
the rate of P1,235.00 a month, with legal interest thereon from the time of the filing of the the corporate obligations. While the mere ownership of all or nearly all of the capital stock of a
complaint. Deducting the P550.00 which was paid at the time when the corporation was already corporation is a mere business conduit of the stockholder, that conclusion is amply justified
acquired by the said defendants-appellees Cirilo Paredes and Ursula Tolentino, they are hereby where it is shown, as in the case before us, that the operations of the corporation were so
ordered to pay to plaintiffs-appellants Dominga de los Reyes and Sabino Padilla the sum of merged with those of the stockholders as to be practically indistinguishable from them. To hold
P6,036.66 with legal interest therein from the time of the filing of the complaint until fully paid. the latter liable for the corporation's obligations is not to ignore the corporation's separate entity,
but merely to apply the established principle that such entity can not be invoked or used for
purposes that could not have been intended by the law that created that separate personality.

The petitioners-appellants insist that the Court could have no jurisdiction over an action to
enforce a judgment within five (5) years from its rendition, since the Rules of Court provide for
enforcement by mere motion during those five years. The error of this stand is apparent,
because the second action, originally begun in the Court of First Instance, was not an action to
enforce the judgment of the Municipal Court, but an action to have non-parties to the judgment
held responsible for its payment.

Finding no error in the judgment appealed from, the same is hereby affirmed, with costs against
petitioners-appellants Cirilo Paredes and Ursula Tolentino.
G.R. No. L-41337 June 30, 1988 any force and effect. The Sheriff is ordered to return the said machinery to its owner, the
Philippine American Drug Co.
TAN BOON BEE & CO., INC., petitioner,
vs. Petitioner filed a Motion For Reconsideration (Ibid., pp. 7093) and an Addendum to Motion for
THE HONORABLE HILARION U. JARENCIO, PRESIDING JUDGE OF BRANCH XVIII of the Reconsideration (Ibid., pp. 94-08), but in an Order dated August 13, 1975, the same was denied
Court of First Instance of Manila, GRAPHIC PUBLISHING, INC., and PHILIPPINE for lack of merit (Ibid., p. 109). Hence, the instant petition.
AMERICAN CAN DRUG COMPANY, respondents.
In a Resolution dated September 12, 1975, the Second Division of this Court resolved to require
PARAS, J.: the respondents to comment, and to issue a temporary restraining order (Rollo, p. 111 ). After
submission of the parties' Memoranda, the case was submitted for decision in the Resolution of
November 28, 1975 (Ibid., p. 275).
This is a petition for certiorari, with prayer for preliminary injunction, to annul and set aside the
March 26, 1975 Order of the then Court of First Instance of Manila, Branch XXIII, setting aside
the sale of "Heidelberg" cylinder press executed by the sheriff in favor of the herein petitioner, as Petitioner, to support its stand, raised two (2) issues, to wit:
well as the levy on the said property, and ordering the sheriff to return the said machinery to its
owner, herein private respondent Philippine American Drug Company.
I

Petitioner herein, doing business under the name and style of Anchor Supply Co., sold on credit
THE RESPONDENT JUDGE GRAVELY EXCEEDED, IF NOT ACTED WITHOUT
to herein private respondent Graphic Publishing, Inc. (GRAPHIC for short) paper products
JURISDICTION WHEN HE ACTED UPON THE MOTION OF PADCO, NOT ONLY BECAUSE
amounting to P55,214.73. On December 20, 1972, GRAPHIC made partial payment by check to
SECTION 17, RULE 39 OF THE RULES OF COURT WAS NOT COMPLIED WITH, BUT ALSO
petitioner in the total amount of P24,848.74; and on December 21, 1972, a promissory note was
BECAUSE THE CLAIMS OF PADCO WHICH WAS NOT A PARTY TO THE CASE COULD
executed to cover the balance of P30,365.99. In the said promissory note, it was stipulated that
NOT BE VENTILATED IN THE CASE BEFORE HIM BUT IN INDEPENDENT PROCEEDING.
the amount will be paid on monthly installments and that failure to pay any installment would
make the amount immediately demandable with an interest of 12% per annum. On September 6,
1973, for failure of GRAPHIC to pay any installment, petitioner filed with the then Court of First II
Instance of Manila, Branch XXIII, presided over by herein respondent judge, Civil Case No.
91857 for a Sum of Money (Rollo, pp. 36-38). Respondent judge declared GRAPHIC in default
for failure to file its answer within the reglementary period and plaintiff (petitioner herein) was THE RESPONDENT JUDGE GRAVELY ABUSED HIS DISCRETION WHEN HE REFUSED TO
allowed to present its evidence ex parte. In a Decision dated January 18, 1974 (Ibid.,pp. 39-40), PIERCE THE PADCO'S (IDENTITY) AND DESPITE THE ABUNDANCE OF EVIDENCE
CLEARLY SHOWING THAT PADCO WAS CONVENIENTLY SHIELDING UNDER THE
the trial court ordered GRAPHIC to pay the petitioner the sum of P30,365.99 with 12% interest
from March 30, 1973 until fully paid, plus the costs of suit. On motion of petitioner, a writ of THEORY OF CORPORATE PETITION.
execution was issued by respondent judge; but the aforestated writ having expired without the
sheriff finding any property of GRAPHIC, an alias writ of execution was issued on July 2, 1974. Petitioner contends that respondent judge gravely exceeded, if not, acted without jurisdiction, in
nullifying the sheriffs sale not only because Section 17, Rule 39 of the Rules of Court was not
Pursuant to the said issued alias writ of execution, the executing sheriff levied upon one (1) unit complied with, but more importantly because PADCO could not have litigated its claim in the
printing machine Identified as "Original Heidelberg Cylinder Press" Type H 222, NR 78048, same case, but in an independent civil proceeding.
found in the premises of GRAPHIC. In a Notice of Sale of Execution of Personal Property dated
July 29, 1974, said printing machine was scheduled for auction sale on July 26, 1974 at 10:00 This contention is well-taken.
o'clock at 14th St., Cor. Atlanta St., Port Area, Manila (lbid., p. 45); but in a letter dated July 19,
1974, herein private respondent, Philippine American Drug Company (PADCO for short) had
informed the sheriff that the printing machine is its property and not that of GRAPHIC, and In the case of Bayer Philippines, Inc. vs. Agana (63 SCRA 355, 366-367 [1975]), this Court
accordingly, advised the sheriff to cease and desist from carrying out the scheduled auction sale categorically ruled as follows:
on July 26, 1974. Notwithstanding the said letter, the sheriff proceeded with the scheduled
auction sale, sold the property to the petitioner, it being the highest bidder, and issued a In other words, constitution, Section 17 of Rule 39 of the Revised Rules of Court, the rights of
Certificate of Sale in favor of petitioner (Rollo, p. 48). More than five (5) hours after the auction third-party claimants over certain properties levied upon by the sheriff to satisfy the judgment
sale and the issuance of the certificate of sale, PADCO filed an "Affidavit of Third Party Claim" should not be decided inthe action where the third-party claims have been presented, but in the
with the Office of the City Sheriff (Ibid., p. 47). Thereafter, on July 30,1974, PADCO filed with the separate action instituted by the claimants.
Court of First Instance of Manila, Branch XXIII, a Motion to Nullify Sale on Execution (With
Injunction) (Ibid., pp, 49-55), which was opposed by the petitioner (Ibid., pp. 5668). Respondent
judge, in an Order dated March 26, 1975 (Ibid., pp. 64-69), ruled in favor of PADCO. The ... Otherwise stated, the court issuing a writ of execution is supposed to enforce the authority
decretal portion of the said order, reads: only over properties of the judgment debtor, and should a third party appeal- to claim the
property levied upon by the sheriff, the procedure laid down by the Rules is that such claim
should be the subject of a separate and independent action.
WHEREFORE, the sale of the 'Heidelberg cylinder press executed by the Sheriff in favor of the
plaintiff as well as the levy on the said property is hereby set aside and declared to be without
xxx xxx xxx
... This rule is dictated by reasons of convenience, as "intervention is more likely to inject Considering the aforestated principles and the circumstances established in this case,
confusion into the issues between the parties in the case . . . with which the third-party claimant respondent judge should have pierced PADCO's veil of corporate Identity.
has nothing to do and thereby retard instead of facilitate the prompt dispatch of the controversy
which is the underlying objective of the rules of pleading and practice." Besides, intervention
Respondent PADCO argues that if respondent judge erred in not piercing the veil of its corporate
may not be permitted after trial has been concluded and a final judgment rendered in the case.
fiction, the error is merely an error of judgment and not an error of jurisdiction correctable by
appeal and not by certiorari.
However, the fact that petitioner questioned the jurisdiction of the court during the initial hearing
of the case but nevertheless actively participated in the trial, bars it from questioning now the
To this argument of respondent, suffice it to say that the same is a mere technicality. In the case
court's jurisdiction. A party who voluntarily participated in the trial, like the herein petitioner,
of Rubio vs. Mariano (52 SCRA 338, 343 [1973]), this Court ruled:
cannot later on raise the issue of the court's lack of jurisdiction (Philippine National Bank vs.
Intermediate Appellate Court, 143 SCRA [1986]).
While We recognize the fact that these movants — the MBTC, the Phillips spouses, the Phillips
corporation and the Hacienda Benito, Inc.— did raise in their respective answers the issue as to
As to the second issue (the non-piercing of PADCO's corporate Identity) the decision of
the propriety of the instant petition for certiorari on the ground that the remedy should have been
respondent judge is as follows:
appeal within the reglementary period, We considered such issue as a mere technicality which
would have accomplished nothing substantial except to deny to the petitioner the right to litigate
The plaintiff, however, contends that the controlling stockholders of the Philippine American the matters he raised ...
Drug Co. are also the same controlling stockholders of the Graphic Publishing, Inc. and,
therefore, the levy upon the said machinery which was found in the premises occupied by the
Litigations should, as much as possible, be decided on their merits and not on technicality (De
Graphic Publishing, Inc. should be upheld. This contention cannot be sustained because the two
las Alas vs. Court of Appeals, 83 SCRA 200, 216 [1978]). Every party-litigant must be afforded
corporations were duly incorporated under the Corporation Law and each of them has a juridical
the amplest opportunity for the proper and just determination of his cause, free from the
personality distinct and separate from the other and the properties of one cannot be levied upon
unacceptable plea of technicalities (Heirs of Ceferino Morales vs. Court of Appeals, 67 SCRA
to satisfy the obligation of the other. This legal preposition is elementary and fundamental.
304, 310 [1975]).

It is true that a corporation, upon coming into being, is invested by law with a personality
PREMISES CONSIDERED, the March 26,1975 Order of the then Court of First Instance of
separate and distinct from that of the persons composing it as well as from any other legal entity
Manila, is ANNULLED and SET ASIDE, and the Temporary Restraining Order issued is hereby
to which it may be related (Yutivo & Sons Hardware Company vs. Court of Tax Appeals, 1
made permanent.
SCRA 160 [1961]; and Emilio Cano Enterprises, Inc. vs. CIR, 13 SCRA 290 [1965]). As a matter
of fact, the doctrine that a corporation is a legal entity distinct and separate from the members
and stockholders who compose it is recognized and respected in all cases which are within SO ORDERED.
reason and the law (Villa Rey Transit, Inc. vs. Ferrer, 25 SCRA 845 [1968]). However, this
separate and distinct personality is merely a fiction created by law for convenience and to
promote justice (Laguna Transportation Company vs. SSS, 107 Phil. 833 [1960]). Accordingly,
this separate personality of the corporation may be disregarded, or the veil of corporate fiction
pierced, in cases where it is used as a cloak or cover for fraud or illegality, or to work an
injustice, or where necessary to achieve equity or when necessary for the protection of creditors
(Sulo ng Bayan, Inc. vs. Araneta, Inc., 72 SCRA 347 [1976]). Corporations are composed of
natural persons and the legal fiction of a separate corporate personality is not a shield for the
commission of injustice and inequity (Chenplex Philippines, Inc., et al. vs. Hon. Pamatian et al.,
57 SCRA 408 (19741). Likewise, this is true when the corporation is merely an adjunct, business
conduit or alter ego of another corporation. In such case, the fiction of separate and distinct
corporation entities should be disregarded (Commissioner of Internal Revenue vs. Norton &
Harrison, 11 SCRA 714 [1964]).

In the instant case, petitioner's evidence established that PADCO was never engaged in the
printing business; that the board of directors and the officers of GRAPHIC and PADCO were the
same; and that PADCO holds 50% share of stock of GRAPHIC. Petitioner likewise stressed that
PADCO's own evidence shows that the printing machine in question had been in the premises of
GRAPHIC since May, 1965, long before PADCO even acquired its alleged title on July 11, 1966
from Capitol Publishing. That the said machine was allegedly leased by PADCO to GRAPHIC on
January 24, 1966, even before PADCO purchased it from Capital Publishing on July 11, 1966,
only serves to show that PADCO's claim of ownership over the printing machine is not only farce
and sham but also unbelievable.
G.R. No. L-33172 October 18, 1979 VIEWED IN THE LIGHT OF ALL THE FOREGOING, judgment is hereby rendered in favor of
plaintiffs and against the defendants declaring that:
ERNESTO CEASE, CECILIA CEASE, MARION CEASE, TERESA CEASE-LACEBAL and the
F.L. CEASE PLANTATION CO., INC. as Trustee of properties of the defunct TIAONG 1) The assets or properties of the defunct Tiaong Milling and Plantation Company now
MILLING & PLANTATION CO., petitioners, appearing under the name of F.L. Cease Plantation Company as Trustee, is the estate also of
vs. the deceased Forrest L. Cease and ordered divided, share and share alike, among his six
HONORABLE COURT OF APPEALS, (Special Seventh Division), HON. MANOLO L. children the plaintiffs and the defendants in accordance with Rule 69, Rules of Court;
MADDELA, Presiding Judge, Court of First Instance of Quezon, BENJAMIN CEASE and
FLORENCE CEASE, respondents.
2) The Resolution to Sell dated October 12, 1959 and the Transfer and Conveyance with Trust
Agreement is hereby set aside as improper and illegal for the purposes and effect that it was
GUERRERO, J: intended and, therefore, null and void;

Appeal by certiorari from the decision of the Court of Appeals in CA-G.R. No. 45474, entitled 3) That F.L. Cease Plantation Company is removed as 'Trustee for interest against the estate
"Ernesto Cease, et al. vs. Hon. Manolo L. Maddela, Judge of the Court of First Instance of and essential to the protection of plaintiffs' rights and is hereby ordered to deliver and convey all
Quezon, et al." 1 which dismissed the petition for certiorari, mandamus, and prohibition instituted the properties and assets of the defunct Tiaong Milling now under its name, custody and control
by the petitioners against the respondent judge and the private respondents. to whomsoever be appointed as Receiver - disqualifying and of the parties herein - the latter to
act accordingly upon proper assumption of office; and
The antecedents of the case, as found by the appellate court, are as follows:
4) Special Proceedings No. 3893 for administration is terminated and dismissed; the instant
case to proceed but on issues of damages only and for such action inherently essential for
IT RESULTING: That the antecedents are not difficult to understand; sometime in June 1908,
partition.
one Forrest L. Cease common predecessor in interest of the parties together with five (5) other
American citizens organized the Tiaong Milling and Plantation Company and in the course of its
corporate existence the company acquired various properties but at the same time all the other SO ORDERED.
original incorporators were bought out by Forrest L. Cease together with his children namely
Ernest, Cecilia, Teresita, Benjamin, Florence and one Bonifacia Tirante also considered a
Lucena City, December 27, 1969., pp. 122-a-123, rollo.
member of the family; the charter of the company lapsed in June 1958; but whether there were
steps to liquidate it, the record is silent; on 13 August 1959, Forrest L. Cease died and by
extrajudicial partition of his shares, among the children, this was disposed of on 19 October upon receipt of that, defendants there filled a notice of appeal p. 129, rollo together with an
1959; it was here where the trouble among them came to arise because it would appear that appeal bond and a record on appeal but the plaintiffs moved to dismiss the appeal on the
Benjamin and Florence wanted an actual division while the other children wanted ground that the judgment was in fact interlocutory and not appealable p. 168 rollo and this
reincorporation; and proceeding on that, these other children Ernesto, Teresita and Cecilia and position of defendants was sustained by trial Judge, His Honor ruling that
aforementioned other stockholder Bonifacia Tirante proceeded to incorporate themselves into
the F.L. Cease Plantation Company and registered it with the Securities and Exchange
Commission on 9 December, 1959; apparently in view of that, Benjamin and Florence for their IN VIEW OF THE FOREGOING, the appeal interposed by plaintiffs is hereby dismissed as
part initiated a Special Proceeding No. 3893 of the Court of First Instance of Tayabas for the premature and the Record on Appeal is necessarily disapproved as improper at this stage of the
settlement of the estate of Forest L. Cease on 21 April, 1960 and one month afterwards on 19 proceedings.
May 1960 they filed Civil Case No. 6326 against Ernesto, Teresita and Cecilia Cease together
with Bonifacia Tirante asking that the Tiaong Milling and Plantation Corporation be declared SO ORDERED.
Identical to F.L. Cease and that its properties be divided among his children as his intestate
heirs; this Civil Case was resisted by aforestated defendants and notwithstanding efforts of the
plaintiffs to have the properties placed under receivership, they were not able to succeed Lucena City, April 27, 1970.
because defendants filed a bond to remain as they have remained in possession; after that and
already, during the pendency of Civil Case No. 6326 specifically on 21 May, 1961 apparently on and so it was said defendants brought the matter first to the Supreme Court, on mandamus on
the eve of the expiry of the three (3) year period provided by the law for the liquidation of 20 May, 1970 to compel the appeal and certiorari and prohibition to annul the order of 27 April,
corporations, the board of liquidators of Tiaong Milling executed an assignment and conveyance 1970 on the ground that the decision was "patently erroneous" p. 16, rollo; but the Supreme
of properties and trust agreement in favor of F.L. Cease Plantation Co. Inc. as trustee of the Court remanded the case to this Court of Appeals by resolution of 27 May 1970, p. 173, and this
Tiaong Milling and Plantation Co. so Chat upon motion of the plaintiffs trial Judge ordered that Court of Appeals on 1 July 1970 p. 175 dismissed the petition so far as the mandamus was
this alleged trustee be also included as party defendant; now this being the situation, it will be concerned taking the view that the decision sought to be appealed dated 27 December, 1969
remembered that there were thus two (2) proceedings pending in the Court of First Instance of was interlocutory and not appealable but on motion for reconsideration of petitioners and since
Quezon namely Civil Case No. 6326 and Special Proceeding No. 3893 but both of these were there was possible merit so far as its prayer for certiorari and prohibition was concerned, by
assigned to the Honorable Respondent Judge Manolo L. Maddela p. 43 and the case was finally resolution of the Court on 19 August, 1970, p. 232, the petition was permitted to go ahead in that
heard and submitted upon stipulation of facts pp, 34-110, rollo; and trial Judge by decision dated capacity; and it is the position of petitioners that the decision of 27 December, 1969 as well as
27 December 1969 held for the plaintiffs Benjamin and Florence, the decision containing the the order of 27 April, 1970 suffered of certain fatal defects, which respondents deny and on their
following dispositive part: part raise the preliminary point that this Court of Appeals has no authority to give relief to
petitioners because not
in aid of its appellate jurisdiction, the same time, We expressed Our displeasure in the appointment of the branch clerk of court or
any other court personnel for that matter as receiver. (p. 102, rollo).
and that the questions presented cannot be raised for the first time before this Court of Appeals;
2. Meanwhile, sensing that the appointed receiver was making some attempts to take
possession of the properties, petitioners filed in this present appeal an urgent petition to restrain
Respondent Court of Appeals in its decision promulgated December 9, 1970 dismissed the
proceedings in the lower court. We resolved the petition on January 29, 1975 by issuing a
petition with costs against petitioners, hence the present petition to this Court on the following
corresponding temporary restraining order enjoining the court a quo from implementing its
assignment of errors:
decision of December 27, 1969, more particularly, the taking over by a receiver of the properties
subject of the litigation, and private respondents Benjamin and Florence Cease from proceeding
THE COURT OF APPEALS ERRED - or taking any action on the matter until further orders from this Court (pp. 99-100, rollo). Private
respondents filed a motion for reconsideration of Our resolution of January 29, 1975. After
weighing the arguments of the parties and taking note of Our resolution in G.R. No. L-35629
I. IN SANCTIONING THE WRONGFUL EXERCISE OF JURISDICTION BEYOND THE LIMITS which upheld the appointment of a receiver, We issued another resolution dated April 11, 1975
OF AUTHORITY CONFERRED BY LAW UPON THE LOWER COURT, WHEN IT PROCEEDED
lifting effective immediately Our previous temporary restraining order which enforced the earlier
TO HEAR, ADJUDGE AND ADJUDICATE - resolution of January 29, 1975 (pp. 140-141, rollo).

(a) Special Proceedings No. 3893 for the settlement of the Estate of Forrest L. Cease,
3. On February 6, 1976, private respondents filed an urgent petition to restrain proceedings
simultaneously and concurrently with - below in view of the precipitate replacement of the court appointed receiver Mayor Francisco
Escueta (vice Mr. Eleno M. Joyas) and the appointment of Mr. Guillermo Lagrosa on the eve of
(b) Civil Case No. 6326, wherein the lower Court ordered Partition under Rule 69, Rules of Court respondent Judge Maddela's retirement (p. 166, rollo). The urgent petition was denied in Our
- resolution of February 18, 1976 (p. 176, rollo).

THE ISSUE OF LEGAL OWNERSHIP OF THE PROPERTIES COMMONLY INVOLVED IN 4. Several attempts at a compromise agreement failed to materialize. A Tentative Compromise
BOTH ACTIONS HAVING BEEN RAISED AT THE OUTSET BY THE TIAONG MILLING AND Agreement dated July 30, 1975 was presented to the Court on August 6, 1976 for the signature
PLANTATION COMPANY, AS THE REGISTERED OWNER OF SUCH PROPERTIES UNDER of the parties, but respondents "unceremoniously" repudiated the same by leaving the courtroom
ACT 496. without the permission of the court (Court of First Instance of Quezon, Branch 11) as a result of
which respondents and their counsel were cited for contempt (p. 195, 197, rollo) that
respondents' reason for the repudiation appears to be petitioners' failure to render an audited
II. IN AFFIRMING - UNSUPPORTED BY ANY EVIDENCE WHATSOEVER NOR CITATION OF account of their administration covering the period from May 31, 1961 up to January 29, 1974,
ANY LAW TO JUSTIFY - THE UNWARRANTED CONCLUSION THAT SUBJECT plus the inclusion of a provision on waiver and relinquishment by respondents of whatever rights
PROPERTIES, FOUND BY THE LOWER COURT AND THE COURT OF APPEALS AS that may have accrued to their favor by virtue of the lower court's decision and the affirmative
ACTUALLY REGISTERED IN THE NAME OF PETITIONER CORPORATION AND/OR ITS decision of the appellate court.
PREDECESSOR IN INTEREST, THE TIAONG MILLING AND PLANTATION COMPANY,
DURING ALL THE 50 YEARS OF ITS CORPORATE EXISTENCE "ARE ALSO PROPERTIES
OF THE ESTATE OF FOREST L. CEASE." We go now to the alleged errors committed by the respondent Court of Appeals.

III. IN AFFIRMING THE ARBITRARY CONCLUSION OF THE LOWER COURT THAT ITS As can be gleaned from petitioners' brief and the petition itself, two contentions underlie the first
DECISION OF DECEMBER 27,1969 IS AN "INTERLUCUTORY DECISION." IN DISMISSED assigned error. First, petitioners argue that there was an irregular and arbitrarte termination and
NG THE PETITION FOR WRIT OF MANDAMUS, AND IN AFFIRMING THE MANIFESTLY dismissal of the special proceedings for judicial administration simultaneously ordered in the
UNJUST JUDGMENT RENDERED WHICH CONTRADICTS THE FINDINGS OF ULTIMATE lower court . s decision in Civil Case No. 6326 adjudicating the partition of the estate, without
FACTS THEREIN CONTAINED. categorically, reasoning the opposition to the petition for administration Second, that the issue of
ownership had been raised in the lower court when Tiaong Milling asserted title over the
properties registered in its corporate name adverse to Forrest L. Cease or his estate, and that
During the period that ensued after the filing in this Court of the respective briefs and the the said issue was erroneously disposed of by the trial court in the partition proceedings when it
subsequent submission of the case for decision, some incidents had transpired, the summary of concluded that the assets or properties of the defunct company is also the estate of the
which may be stated as follows: deceased proprietor.

1. Separate from this present appeal, petitioners filed a petition for certiorari and prohibition in The propriety of the dismissal and termination of the special proceedings for judicial
this Court, docketed as G.R. No. L-35629 (Ernesto Cease, et al. vs. Hon. Manolo L. Maddela, et administration must be affirmed in spite of its rendition in another related case in view of the
al.) which challenged the order of respondent judge dated September 27, 1972 appointing his established jurisprudence which favors partition when judicial administration become,
Branch Clerk of Court, Mr. Eleno M. Joyas, as receiver of the properties subject of the appealed unnecessary. As observed by the Court of Appeals, the dismissal at first glance is wrong, for the
civil case, which order, petitioners saw as a virtual execution of the lower court's judgment (p. reason that what was actually heard was Civil Case No. 6326. The technical consistency,
92, rollo). In Our resolution of November 13, 1972, issued in G.R. No. L-35629, the petition was however, it is far less importance than the reason behind the doctrinal rule against placing an
denied since respondent judge merely appointed an auxilliary receiver for the preservation of the estate under administration. Judicial rulings consistently hold the view that where partition is
properties as well as for the protection of the interests of all parties in Civil Case No. 6326; but at possible, either judicial or extrajudicial, the estate should not be burdened with an administration
proceeding without good and compelling reason. When the estate has no creditors or pending Petitioners' argument has only theoretical persuasion, to say the least, rather apparent than real.
obligations to be paid, the beneficiaries in interest are not bound to submit the property to judicial It must be remembered that when Tiaong Milling adduced its defense and raised the issue of
administration which is always long and costly, or to apply for the appointment of an ownership, its corporate existence already terminated through the expiration of its charter. It is
administrator by the court, especially when judicial administration is unnecessary and clear in Section 77 of Act No. 1459 (Corporation Law) that upon the expiration of the charter
superfluous. Thus - period, the corporation ceases to exist and is dissolved ipso facto except for purposes
connected with the winding up and liquidation. The provision allows a three year, period from
expiration of the charter within which the entity gradually settles and closes its affairs, disposes
When a person dies without leaving pending obligations to be paid, his heirs, whether of age or
and convey its property and to divide its capital stock, but not for the purpose of continuing the
not, are bound to submit the property to a judicial administration, which is always long and
business for which it was established. At this terminal stage of its existence, Tiaong Milling may
costly, or to apply for the appointment of an administrator by the court. It has been uniformly held
no longer persist to maintain adverse title and ownership of the corporate assets as against the
that in such case the judicial administration and the appointment of an administrator are
prospective distributees when at this time it merely holds the property in trust, its assertion of
superfluous and unnecessary proceedings (Ilustre vs. Alaras Frondosa, 17 Phil., 321;
ownership is not only a legal contradiction, but more so, to allow it to maintain adverse interest
Malahacan vs. Ignacio, 19 Phil, 434; Bondad vs. Bondad, 34 Phil., 232; Baldemor vs.
would certainly thwart the very purpose of liquidation and the final distribute loll of the assets to
Malangyaon, 34 Phil., 367; Fule vs. Fule, 46 Phil., 317). Syllabus, Intestate estate of the
the proper, parties.
deceased Luz Garcia. Pablo G. Utulo vs. Leona Pasion Viuda de Garcia, 66 Phil. 302.

We agree with the Court of Appeals in its reasoning that substance is more important than form
Where the estate has no debts, recourse may be had to an administration proceeding only if the
when it sustained the dismissal of Special Proceedings No. 3893, thus -
heirs have good reasons for not resorting to an action for partition. Where partition is possible,
either in or out of court, the estate should not be burdened with an administration proceeding
without good and compelling reasons. (Intestate Estate of Mercado vs. Magtibay, 96 Phil. 383) a) As to the dismissal of Special Proceedings No. 3893, of course, at first glance, this was
wrong, for the reason that the case trial had been heard was Civil Case No. 6326; but what
should not be overlooked either is Chat respondent Judge was the same Judge that had before
In the records of this case, We find no indication of any indebtedness of the estate. No creditor
him in his own sala, said Special Proceedings No. 3893, p. 43 rollo, and the parties to the
has come up to charge the estate within the two-year period after the death of Forrest L. Cease,
present Civil Case No. 6326 had themselves asked respondent Judge to take judicial notice of
hence, the presumption under Section 1, Rule 74 that the estate is free from creditors must
the same and its contents page 34, rollo; it is not difficult to see that when respondent Judge in
apply. Neither has the status of the parties as legal heirs, much less that of respondents, been
par. 4 of the dispositive part of his decision complained of, ordered that,
raised as an issue. Besides, extant in the records is the stipulation of the parties to submit the
pleadings and contents of the administration proceedings for the cognizance of the trial judge in
adjudicating the civil case for partition (Respondents' Brief, p, 20, rollo). As respondents 4) Special Proceedings No. 3893 for administration is terminated and dismissed; the instant
observe, the parties in both cases are the same, so are the properties involved; that actual case to proceed but on issues of damages only and for such action inherently essential or
division is the primary objective in both actions; the theory and defense of the respective parties partition. p. 123, rollo,
are likewise common; and that both cases have been assigned to the same respondent judge.
We feel that the unifying effect of the foregoing circumstances invites the wholesome exception
in truth and in fact, His Honor was issuing that order also within Civil Case No. 632 but in
to the structures of procedural rule, thus allowing, instead, room for judicial flexibility.
connection with Special Proceedings No. 389:3: for substance is more important Chan form, the
Respondent judge's dismissal of the administration proceedings then, is a judicious move,
contending par ties in both proceedings being exactly the same, but not only this, let it not be
appreciable in today's need for effective and speedy administration of justice. There being ample
forgotten that when His Honor dismissed Special Proceedings No. 3893, that dismissal precisely
reason to support the dismissal of the special proceedings in this appealed case, We cannot see
was a dismissal that petitioners herein had themselves sought and solicited from respondent
in the records any compelling reason why it may not be dismissed just the same even if
Judge as petitioners themselves are in their present petition pp. 5-6, rollo; this Court must find
considered in a separate action. This is inevitably certain specially when the subject property
difficulty in reconciling petitioners' attack with the fact that it was they themselves that had
has already been found appropriate for partition, thus reducing the petition for administration to a
insisted on that dismissal; on the principle that not he who is favored but he who is hurt by a
mere unnecessary solicitation.
judicial order is he only who should be heard to complain and especially since extraordinary
legal remedies are remedies in extermies granted to parties ' who have been the victims not
The second point raised by petitioners in their first assigned error is equally untenable. In effect, merely of errors but of grave wrongs, and it cannot be seen how one who got what he had asked
petitioners argue that the action for partition should not have prospered in view of the repudiation could be heard to claim that he had been the victim of a wrong, petitioners should not now
of the co-ownership by Tiaong Milling and Plantation Company when, as early in the trial court, it complain of an order they had themselves asked in order to attack such an order afterwards; if at
already asserted ownership and corporate title over the properties adverse to the right of all, perhaps, third parties, creditors, the Bureau of Internal Revenue, might have been
ownership of Forrest L. Cease or his estate. We are not unmindful of the doctrine relied upon by prejudiced, and could have had the personality to attack that dismissal of Special Proceedings
petitioners in Rodriguez vs. Ravilan, 17 Phil. 63 wherein this Court held that in an action for No. 3893, but not petitioners herein, and it is not now for this Court of Appeals to protect said
partition, it is assumed that the parties by whom it is prosecuted are all co-owners or co- third persons who have not come to the Court below or sought to intervene herein;
proprietors of the property to be divided, and that the question of common ownership is not to be
argued, not the fact as to whether the intended parties are or are not the owners of the property
On the second assigned error, petitioners argue that no evidence has been found to support the
in question, but only as to how and in what manner and proportion the said property of common
conclusion that the registered properties of Tiaong Milling are also properties of the estate of
ownership shall be distributed among the interested parties by order of the Court. Consistent
Forrest L. Cease; that on the contrary, said properties are registered under Act No. 496 in the
with this dictum, it has been field that if any party to a suit for partition denies the pro-
name of Tiaong Milling as lawful owner and possessor for the last 50 years of its corporate
indivisocharacter of the estate whose partition is sought, and claims instead, exclusive title
existence.
thereto the action becomes one for recovery of property cognizable in the courts of ordinary
jurisdiction. 2
We do not agree. In reposing ownership to the estate of Forrest L. Cease, the trial court indeed In any of these cases, the notion of corporate entity will be pierced or disregarded, and the
found strong support, one that is based on a well-entrenched principle of law. In sustaining corporation will be treated merely as an association of persons or, where there are two
respondents' theory of "merger of Forrest L. Cease and The Tiaong Milling as one personality", corporations, they will be merged as one, the one being merely regarded as part or the
or that "the company is only the business conduit and alter ego of the deceased Forrest L. instrumentality of the otter (Koppel [Phil.] Inc. vs. Yatco, 77 Phil. 496, Yutivo Sons Hardware
Cease and the registered properties of Tiaong Milling are actually properties of Forrest L. Cease Company vs. Court of Tax Appeals, supra).
and should be divided equally, share and share alike among his six children, ... ", the trial court
did aptly apply the familiar exception to the general rule by disregarding the legal fiction of
So must the case at bar add to this jurisprudence. An indubitable deduction from the findings of
distinct and separate corporate personality and regarding the corporation and the individual
the trial court cannot but lead to the conclusion that the business of the corporation is largely, if
member one and the same. In shredding the fictitious corporate veil, the trial judge narrated the
not wholly, the personal venture of Forrest L. Cease. There is not even a shadow of a showing
undisputed factual premise, thus:
that his children were subscribers or purchasers of the stocks they own. Their participation as
nominal shareholders emanated solely from Forrest L. Cease's gratuitous dole out of his own
While the records showed that originally its incorporators were aliens, friends or third-parties in shares to the benefit of his children and ultimately his family.
relation of one to another, in the course of its existence, it developed into a close family
corporation. The Board of Directors and stockholders belong to one family the head of which
Were we sustain the theory of petitioners that the trial court acted in excess of jurisdiction or
Forrest L. Cease always retained the majority stocks and hence the control and management of
abuse of discretion amounting to lack of jurisdiction in deciding Civil Case No. 6326 as a case
its affairs. In fact, during the reconstruction of its records in 1947 before the Security and
for partition when the defendant therein, Tiaong Milling and Plantation Company, Inc. as
Exchange Commission only 9 nominal shares out of 300 appears in the name of his 3 eldest
registered owner asserted ownership of the assets and properties involved in the litigation, which
children then and another person close to them. It is likewise noteworthy to observe that as his
theory must necessarily be based on the assumption that said assets and properties of Tiaong
children increase or perhaps become of age, he continued distributing his shares among them
Milling and Plantation Company, Inc. now appearing under the name of F. L. Cease Plantation
adding Florence, Teresa and Marion until at the time of his death only 190 were left to his name.
Company as Trustee are distinct and separate from the estate of Forrest L. Cease to which
Definitely, only the members of his family benefited from the Corporation.
petitioners and respondents as legal heirs of said Forrest L. Cease are equally entitled share
and share alike, then that legal fiction of separate corporate personality shall have been used to
The accounts of the corporation and therefore its operation, as well as that of the family appears delay and ultimately deprive and defraud the respondents of their successional rights to the
to be indistinguishable and apparently joined together. As admitted by the defendants estate of their deceased father. For Tiaong Milling and Plantation Company shall have been able
(Manifestation of Compliance with Order of March 7, 1963 [Exhibit "21"] the corporation 'never' to extend its corporate existence beyond the period of its charter which lapsed in June, 1958
had any account with any banking institution or if any account was carried in a bank on its under the guise and cover of F. L, Cease Plantation Company, Inc. as Trustee which would be
behalf, it was in the name of Mr. Forrest L. Cease. In brief, the operation of the Corporation is against the law, and as Trustee shall have been able to use the assets and properties for the
merged with those of the majority stockholders, the latter using the former as his instrumentality benefit of the petitioners, to the great prejudice and defraudation. of private respondents. Hence,
and for the exclusive benefits of all his family. From the foregoing indication, therefore, there is it becomes necessary and imperative to pierce that corporate veil.
truth in plaintiff's allegation that the corporation is only a business conduit of his father and an
extension of his personality, they are one and the same thing. Thus, the assets of the
Under the third assigned error, petitioners claim that the decision of the lower court in the
corporation are also the estate of Forrest L. Cease, the father of the parties herein who are all
partition case is not interlocutory but rather final for it consists of final and determinative
legitimate children of full blood.
dispositions of the contentions of the parties. We find no merit in petitioners' stand.

A rich store of jurisprudence has established the rule known as the doctrine of disregarding or
Under the 1961 pronouncement and ruling of the Supreme Court in Vda. de Zaldarriaga vs.
piercing the veil of corporate fiction. Generally, a corporation is invested by law with a
Enriquez, 1 SCRA 1188 (and the sequel case of Vda. de Zaldarriaga vs. Zaldarriaga, 2 SCRA
personality separate and distinct from that of the persons composing it as well as from that of
356), the lower court's dismissal of petitioners' proposed appeal from its December 27, 1969
any other legal entity to which it may be related. By virtue of this attribute, a corporation may not,
judgment as affirmed by the Court of Appeals on the ground of prematurity in that the judgment
generally, be made to answer for acts or liabilities of its stockholders or those of the legal entities
was not final but interlocutory was in order. As was said in said case:
to which it may be connected, and vice versa. This separate and distinct personality is, however,
merely a fiction created by law for convenience and to promote the ends of justice (Laguna
Transportation Company vs. Social Security System, L-14606, April 28, 1960; La Campana It is true that in Africa vs. Africa, 42 Phil. 934 and other cases it was held - contrary to the rule
Coffee Factory, Inc. vs. Kaisahan ng mga Manggagawa sa La Campana, L-5677, May 25, laid down in Ron vs. Mojica, 8 Phil. 328; Rodriguez vs. Ravilan, 17 Phil. 63 - that in a partition
1953). For this reason, it may not be used or invoked for ends subversive of the policy and case where defendant relies on the defense of exclusive ownership, the action becomes one for
purpose behind its creation (Emiliano Cano Enterprises, Inc. vs. CIR, L-20502, Feb. 26, 1965) or title and the decision or order directing partition is final, but the ruling to this effect has been
which could not have been intended by law to which it owes its being McConnel vs. Court of expressly reversed in the Fuentebella case which, in our opinion, expresses the correct view,
Appeals, L- 10510, March 17, 1961, 1 SCRA 722). This is particularly true where the fiction is considering that a decision or order directing partition is not final because it leaves something
used to defeat public convenience, justify wrong, protect fraud, defend crime (Yutivo Sons more to be done in the trial court for the complete disposition of the case, namely, the
Hardware Company vs. Court of Tax Appeals, L-13203, Jan. 28, 1961, 1 SCRA 160), confuse appointment of commissioners, the proceedings to be had before them, the submission of their
legitimate legal or judicial issues (R. F. Sugay & Co. vs. Reyes, L-20451, Dec. 28, 1964), report which, according to law, must be set for hearing. In fact, it is only after said hearing that
perpetrate deception or otherwise circumvent the law (Gregorio Araneta, Inc. vs. reason de the court may render a final judgment finally disposing of the action (Rule 71, section 7, Rules of
Paterno, L-2886, Aug. 22, 1952, 49 O.G. 721). This is likewise true where the corporate entity is Court). (1 SCRA at page 1193).
being used as an alter ego, adjunct, or business conduit for the sole benefit of the stockholders
or of another corporate entity (McConnel vs. Court of Appeals, supra; Commissioner of Internal
It should be noted, however, that the said ruling in Zaldarriaga as based on Fuentebella vs.
Revenue vs. Norton Harrison Co., L-7618, Aug. 31, 1964).
Carrascoso, XIV Lawyers Journal 305 (May 27, 1942), has been expressly abandoned by the
Court in Miranda vs. Court of Appeals, 71 SCRA 295; 331-333 (June 18, 1976) wherein Mr. backwages less earnings elsewhere during his layoff) and that the only reason given in
Justice Teehankee, speaking for the Court, laid down the following doctrine: Fuentebelia for the contrary ruling, viz, "the general harm that would follow from throwing the
door open to multiplicity of appeals in a single case" of lesser import and consequence.
(Emphasis copied).
The Court, however, deems it proper for the guidance of the bench and bar to now declare as is
clearly indicated from the compelling reasons and considerations hereinabove stated:
The miranda ruling has since then been applied as the new rule by a unanimous Court in Valdez
vs. Bagasao, 82 SCRA 22 (March 8, 1978).
- that the Court considers the better rule to be that stated in H. E. Heacock Co. vs. American
Trading Co., to wit, that where the primary purpose of a case is to ascertain and determine who
between plaintiff and defendant is the true owner and entitled to the exclusive use of the If there were a valid genuine claim of Exclusive ownership of the inherited properties on the part
disputed property, "the judgment . . . rendered by the lower court [is] a judgment on the merits as of petitioners to respondents' action for partition, then under the Miranda ruling, petitioners would
to those questions, and [that] the order of the court for an accounting was based upon, and is be sustained, for as expressly held therein " the general rule of partition that an appeal will not
incidental to the judgment on the merits. That is to say, that the judgment . . . [is] a final lie until the partition or distribution proceedings are terminated will not apply where appellant
judgment ... that in this kind of a case an accounting is a mere incident to the judgment; that claims exclusive ownership of the whole property and denies the adverse party's right to any
an appeal lies from the rendition of the judgment as rendered ... "(as is widely held by a great partition."
number of judges and members of the bar, as shown by the cases so decided and filed and still
pending with the Court) for the fundamental reasons therein stated that "this is more in harmony
But this question has now been rendered moot and academic for the very issue of exclusive
with the administration of justice and the spirit and intent of the [Rules]. If on appeal the
ownership claimed by petitioners to deny and defeat respondents' right to partition - which is the
judgment of the lower court is affirmed, it would not in the least work an injustice to any of the
very core of their rejected appeal - has been squarely resolved herein against them, as if the
legal rights of [appellee]. On the other hand, if for any reason this court should reverse the
appeal had been given due course. The Court has herein expressly sustained the trial court's
judgment of the lower court, the accounting would be a waste of time and money, and might
findings, as affirmed by the Court of Appeals, that the assets or properties of the defunct
work a material injury to the [appellant]; and
company constitute the estate of the deceased proprietor (supra at page 7) and the defunct
company's assertion of ownership of the properties is a legal contradiction and would but thwart
- that accordingly, the contrary ruling in Fuentebella vs. Carrascoso which expressly reversed the liquidation and final distribution and partition of the properties among the parties hereof as
the Heacock case and a line of similar decisions and ruled that such a decision for recovery of children of their deceased father Forrest L. Cease. There is therefore no further hindrance to
property with accounting "is not final but merely interlocutory and therefore not appealable" and effect the partition of the properties among the parties in implementation of the appealed
subsequent cases adhering to the same must be now in turn abandoned and set aside. judgment.

Fuentebella adopted instead the opposite line of conflicting decisions mostly in partition One last consideration. Parties are brothers and sisters, legal heirs of their deceased father,
proceedings and exemplified by Ron vs. Mojica 8 Phil. 928 (under the old Code of Civil Forrest L. Cease. By all rights in law and jurisprudence, each is entitled to share and share alike
Procedure) that an order for partition of real property is not final and appealable until after in the estate, which the trial court correctly ordained and sustained by the appellate court.
the actual partition of the property as reported by the court appointed commissioners and Almost 20 years have lapsed since the filing of Special Proceedings No. 3893 for the
approved by the court in its judgment accepting the report. lt must be especially noted that such administration of the Estate of Forrest L. Cease and Civil Case No. 6326 for liquidation and
rule governing partitions is now so expressly provided and spelled out in Rule 69 of the Rules of partition of the assets of the defunct Tiaong Milling and Plantation Co., Inc. A succession of
Court, with special reference to Sections 1, 2, 3, 6, 7 and 11, to wit, that there must first be a receivers were appointed by the court to take, keep in possession, preserve and manage
preliminar, order for partition of the real estate (section 2) and where the parties-co-owners properties of the corporation which at one time showed an income of P386,152.90 and expenses
cannot agree, the court appointed commissioners make a plan of actual partition which must first of P308,405.01 for the period covering January 1, 1960 to August 31, 1967 as per Summary of
be passed upon and accepted by the trial court and embodied in a judgment to be rendered by it Operations of Commissioner for Finance appointed by the Court (Brief for Respondents, p. 38).
(sections 6 and 11). In partition cases, it must be further borne in mind that Rule 69, section 1 In the meantime, ejectment cases were filed by and against the heirs in connection with the
refers to "a person having the right to compel the partition of real estate," so that the general rule properties involved, aggravating the already strained relations of the parties. A prudent and
of partition that an appeal will not lie until the partition or distribution proceedings are terminated practical realization of these circumstances ought and must constrain the parties to give each
will not apply where appellant claims exclusive ownership of the whole property and denies the one his due in law and with fairness and dispatch that their basic rights be enjoyed. And by
adverse party's right to any partition, as was the ruling in Villanueva vs. Capistrano and Africa vs remanding this case to the court a quo for the actual partition of the properties, the substantial
.Africa, supra, Fuentebellas express rehearsal of these cases must likewise be deemed now rights of everyone of the heirs have not been impaired, for in fact, they have been preserved and
also abandoned in view of the Court's expressed preference for the rationale of the Heacock maintained.
case.
WHEREFORE, IN VIEW OF THE FOREGOING, the judgment appealed from is hereby
The Court's considered opinion is that imperative considerations of public policy and of sound AFFIRMED with costs against the petitioners.
practice in the courts and adherence to the constitutional mandate of simplified, just, speedy and
inexpensive determination of every action call for considering such judgments for recovery of
SO ORDERED.
property with accounting as final judgments which are duly appealable (and would therefore
become final and executory if not appealed within the reglementary period) with
the accounting as a mere incident of the judgment to be rendered during the course of the
appeal as provided in Rule 39, section 4 or to be implemented at the execution stage upon final
affirmance on appeal of the judgment (as in Court of Industrial Relations unfair labor practice
cases ordering the reinstatement of the worker with accounting, computation and payment of his
G.R. No. L-15121 August 31, 1962 both the civil and criminal cases were simultaneously tried by agreement of the parties in said
case. In the Counterclaim of the Answer, defendant alleges that in view of the filing of this
complaint which is a clearly unfounded civil action merely to harass the defendant, it was
GREGORIO PALACIO, in his own behalf and in behalf of his minor child,
compelled to engage the services of a lawyer for an agreed amount of P500.00.
MARIO PALACIO, plaintiffs-appellants,
vs.
FELY TRANSPORTATION COMPANY, defendant-appellee. During the trial, plaintiffs presented the transcript of the stenographic notes of the trial of the
case of "People of the Philippines vs. Alfredo Carillo, Criminal Case No. Q-1084," in the Court of
First Instance of Rizal, Quezon City (Branch IV), as Exhibit "A".
REGALA, J.:

It appears from Exhibit "A" that Gregorio Palacio, one of the herein plaintiffs, testified that Mario
This is an appeal by the plaintiffs from the decision of the Court of First Instance of Manila which
Palacio, the other plaintiff, is his son; that as a result of the reckless driving of accused Alfredo
dismissed their complaint.
Carillo, his child Mario was injured and hospitalized from December 24, 1952, to January 8,
1953; that during all the time that his child was in the hospital, he watched him during the night
Originally taken to the Court of Appeals, this appeal was certified to this Court on the ground that and his wife during the day; that during that period of time he could not work as he slept during
it raises purely questions of law. the day; that before his child was injured, he used to earn P10.00 a day on ordinary days and on
Sundays from P20 to P50 a Sunday; that to meet his expenses he had to sell his compressor
and electric drill for P150 only; and that they could have been sold for P300 at the lowest price.
The parties in this case adopt the following findings of fact of the lower court:

During the trial of the criminal case against the driver of the jeep in the Court of First Instance of
In their complaint filed with this Court on May 15, 1954, plaintiffs allege, among other things,
Quezon City (Criminal Case No. Q-1084) an attempt was unsuccessfully made by the
"that about December, 1952, the defendant company hired Alfredo Carillo as driver of AC-787 prosecution to prove moral damages allegedly suffered by herein plaintiff Gregorio Palacio.
(687) (a registration for 1952) owned and operated by the said defendant company; that on Likewise an attempt was made in vain by the private prosecutor in that case to prove the agreed
December 24, 1952, at about 11:30 a.m., while the driver Alfonso (Alfredo) Carillo was driving
attorney's fees between him and plaintiff Gregorio Palacio and the expenses allegedly incurred
AC-687 at Halcon Street, Quezon City, wilfully, unlawfully and feloniously and in a negligent, by the herein plaintiffs in connection with that case. During the trial of this case, plaintiff Gregorio
reckless and imprudent manner, run over a child Mario Palacio of the herein plaintiff Gregorio Palacio testified substantially to the same facts.
Palacio; that on account of the aforesaid injuries, Mario Palacio suffered a simple fracture of the
right tenor (sic), complete third, thereby hospitalizing him at the Philippine Orthopedic Hospital
from December 24, 1952, up to January 8, 1953, and continued to be treated for a period of five The Court of First Instance of Quezon City in its decision in Criminal Case No. 1084 (Exhibit "2")
months thereafter; that the plaintiff Gregorio Palacio herein is a welder by occupation and owner determined and thoroughly discussed the civil liability of the accused in that case. The
of a small welding shop and because of the injuries of his child he has abandoned his shop dispositive part thereof reads as follows:
where he derives income of P10.00 a day for the support of his big family; that during the period
that the plaintiff's (Gregorio Palacio's) child was in the hospital and who said child was under
IN VIEW OF THE FOREGOING, the Court finds the accused Alfredo Carillo y Damaso guilty
treatment for five months in order to meet the needs of his big family, he was forced to sell one
beyond reasonable doubt of the crime charged in the information and he is hereby sentenced to
air compressor (heavy duty) and one heavy duty electric drill, for a sacrifice sale of P150.00
suffer imprisonment for a period of Two Months & One Day of Arresto Mayor; to indemnify the
which could easily sell at P350.00; that as a consequence of the negligent and reckless act of
offended party, by way of consequential damages, in the sum of P500.00 which the Court
the driver Alfredo Carillo of the herein defendant company, the herein plaintiffs were forced to
deems reasonable; with subsidiary imprisonment in case of insolvency but not to exceed ¹/3 of
litigate this case in Court for an agreed amount of P300.00 for attorney's fee; that the herein
the principal penalty imposed; and to pay the costs.
plaintiffs have now incurred the amount of P500.00 actual expenses for transportation,
representation and similar expenses for gathering evidence and witnesses; and that because of
the nature of the injuries of plaintiff Mario Palacio and the fear that the child might become a On the basis of these facts, the lower court held action is barred by the judgment in the criminal
useless invalid, the herein plaintiff Gregorio Palacio has suffered moral damages which could be case and, that under Article 103 of the Revised Penal Code, the person subsidiarily liable to pay
conservatively estimated at P1,200.00. damages is Isabel Calingasan, the employer, and not the defendant corporation.

On May 23, 1956, defendant Fely Transportation Co., filed a Motion to Dismiss on the grounds Against that decision the plaintiffs appealed, contending that:
(1) that there is no cause of action against the defendant company, and (2) that the cause of
action is barred by prior judgment..
THE LOWER COURT ERRED IN NOT SUSTAINING THAT THE DEFENDANT-APPELLEE IS
SUBSIDIARILY LIABLE FOR DAMAGES AS A RESULT OF CRIMINAL CASE NO. Q-1084 OF
In its Order, dated June 8, 1956, this Court deferred the determination of the grounds alleged in THE COURT OF FIRST INSTANCE OF QUEZON CITY FOR THE REASON THAT THE
the Motion to Dismiss until the trial of this case. INCORPORATORS OF THE FELY TRANSPORTATION COMPANY, THE DEFENDANT-
APPELLEE HEREIN, ARE ISABELO CALINGASAN HIMSELF, HIS SON AND DAUGHTERS;
On June 20, 1956, defendant filed its answer. By way of affirmative defenses, it alleges (1) that
complaint states no cause of action against defendant, and (2) that the sale and transfer of the THE LOWER COURT ERRED IN NOT CONSIDERING THAT THE INTENTION OF ISABELO
jeep AC-687 by Isabelo Calingasan to the Fely Transportation was made on December 24, CALINGASAN IN INCORPORATING THE FELY TRANSPORTATION COMPANY, THE
1955, long after the driver Alfredo Carillo of said jeep had been convicted and had served his DEFENDANT-APPELLEE HEREIN, WAS TO EVADE HIS CIVIL LIABILITY AS A RESULT OF
sentence in Criminal Case No. Q-1084 of the Court of First Instance of Quezon City, in which THE CONVICTION OF HIS DRIVER OF VEHICLE AC-687 THEN OWNED BY HIM:
THE LOWER COURT ERRED IN HOLDING THAT THE CAUSE OF ACTION OF THE
PLAINTIFFS-APPELLANTS IS BARRED BY PRIOR JUDGMENT.

With respect to the first and second assignments of errors, plaintiffs contend that the defendant
corporate should be made subsidiarily liable for damages in the criminal case because the sale
to it of the jeep in question, after the conviction of Alfred Carillo in Criminal Case No. Q-1084 of
the Court of First Instance of Quezon City was merely an attempt on the part of Isabelo
Calingasan its president and general manager, to evade his subsidiary civil liability.

The Court agrees with this contention of the plaintiffs. Isabelo Calingasan and defendant Fely
Transportation may be regarded as one and the same person. It is evident that Isabelo
Calingasan's main purpose in forming the corporation was to evade his subsidiary civil
liability1 resulting from the conviction of his driver, Alfredo Carillo. This conclusion is borne out by
the fact that the incorporators of the Fely Transportation are Isabelo Calingasan, his wife, his
son, Dr. Calingasan, and his two daughters. We believe that this is one case where the
defendant corporation should not be heard to say that it has a personality separate and distinct
from its members when to allow it to do so would be to sanction the use of the fiction of
corporate entity as a shield to further an end subversive of justice. (La Campana Coffee Factory,
et al. v. Kaisahan ng mga Manggagawa, etc., et al., G.R. No. L-5677, May 25, 1953)
Furthermore, the failure of the defendant corporation to prove that it has other property than the
jeep (AC-687) strengthens the conviction that its formation was for the purpose above indicated.

And while it is true that Isabelo Calingasan is not a party in this case, yet, is held in the case of
Alonso v. Villamor, 16 Phil. 315, this Court can substitute him in place of the defendant
corporation as to the real party in interest. This is so in order to avoid multiplicity of suits and
thereby save the parties unnecessary expenses and delay. (Sec. 2, Rule 17, Rules of Court;
Cuyugan v. Dizon. 79 Phil. 80; Quison v. Salud, 12 Phil. 109.)

Accordingly, defendants Fely Transportation and Isabelo Calingasan should be held subsidiarily
liable for P500.00 which Alfredo Carillo was ordered to pay in the criminal case and which
amount he could not pay on account of insolvency.

We also sustain plaintiffs' third assignment of error and hold that the present action is not barred
by the judgment of the Court of First Instance of Quezon City in the criminal case. While there
seems to be some confusion on part of the plaintiffs as to the theory on which the is based —
whether ex-delito or quasi ex-delito (culpa aquiliana) — We are convinced, from the discussion
prayer in the brief on appeal, that they are insisting the subsidiary civil liability of the defendant.
As a matter of fact, the record shows that plaintiffs merely presented the transcript of the
stenographic notes (Exhibit "A") taken at the hearing of the criminal case, which Gregorio
Palacio corroborated, in support of their claim for damages. This rules out the defense of res
judicata, because such liability proceeds precisely from the judgment in the criminal action,
where the accused was found guilty and ordered to pay an indemnity in the sum P500.00.

WHEREFORE, the decision of the lower court is hereby reversed and defendants Fely
Transportation and Isabelo Calingasan are ordered to pay, jointly and severally, the plaintiffs the
amount of P500.00 and the costs.
G.R. No. 159121 February 3, 2005 "When the company took over the operation of Hacienda Pamplona in 1993, it did not absorb all
the workers of Hacienda Pamplona. Some, however, were hired by the company during harvest
season as coconut hookers or ‘sakador,’ coconut filers, coconut haulers, coconut scoopers or
PAMPLONA PLANTATION COMPANY, INC. and/or JOSE LUIS BONDOC, petitioners,
‘lugiteros,’ and charcoal makers.
vs.
RODEL TINGHIL, MARYGLENN SABIHON, ESTANISLAO BOBON, CARLITO TINGHIL,
BONIFACIO TINGHIL, NOLI TINGHIL, EDGAR TINGHIL, ERNESTO ESTOMANTE, SALLY "Sometime in 1995, Pamplona Plantation Leisure Corporation was established for the purpose
TOROY, BENIGNO TINGHIL JR., ROSE ANN NAPAO, DIOSDADO TINGHIL, ALBERTO of engaging in the business of operating tourist resorts, hotels, and inns, with complementary
TINGHIL, ANALIE TINGHIL, and ANTONIO ESTOMANTE,respondents. facilities, such as restaurants, bars, boutiques, service shops, entertainment, golf courses, tennis
courts, and other land and aquatic sports and leisure facilities.
DECISION
"On 15 December 1996, the Pamplona Plantation Labor Independent Union (PAPLIU)
conducted an organizational meeting wherein several [respondents] who are either union
PANGANIBAN, J.:
members or officers participated in said meeting.

To protect the rights of labor, two corporations with identical directors, management, office and
"Upon learning that some of the [respondents] attended the said meeting, [Petitioner] Jose Luis
payroll should be treated as one entity only. A suit by the employees against one corporation
Bondoc, manager of the company, did not allow [respondents] to work anymore in the plantation.
should be deemed as a suit against the other. Also, the rights and claims of workers should not
be prejudiced by the acts of the employer that tend to confuse them about its corporate identity.
The corporate fiction must yield to truth and justice. "Thereafter, on various dates, [respondents] filed their respective complaints with the NLRC,
Sub-Regional Arbitration Branch No. VII, Dumaguete City against [petitioners] for unfair labor
practice, illegal dismissal, underpayment, overtime pay, premium pay for rest day and holidays,
The Case
service incentive leave pay, damages, attorney’s fees and 13th month pay.

Before us is a Petition for Review1 under Rule 45 of the Rules of Court, seeking to annul the
"On 09 October 1997, [respondent] Carlito Tinghil amended his complaint to implead Pamplona
January 31, 2003 Decision2 and the June 17, 2003 Resolution3 of the Court of Appeals (CA) in
Plantation Leisure Corporation x x x.
CA-GR SP No. 62813. The assailed Decision disposed as follows:

"On 31 August 1998, Labor Arbiter Jose G. Gutierrez rendered a decision finding [respondents],
"WHEREFORE, in view of the foregoing, the petition is GRANTED. The assailed decision of
except Rufino Bacubac, Antonio Cañolas and Felix Torres who were complainants in another
public respondent NLRC dated 19 July 2000 [is] REVERSED and SET ASIDE and a new one
case, to be entitled to separation pay.
entered DIRECTING private respondents to reinstate petitioners, except Rufino Bacubac, Felix
Torres and Antonio Canolas, to their former positions without loss of seniority rights plus
payment of full backwages. However, if reinstatement is no longer feasible, a one-month salary xxxxxxxxx
for every year of service shall be paid the petitioners as ordered by the Labor Arbiter in his
decision dated 31 August 1998 plus payment of full backwages computed from date of illegal
"[Petitioners] appealed the Labor Arbiter’s decision to [the] NLRC. In the assailed decision dated
dismissal to the finality of this decision."4
19 July 2000, the NLRC’s Fourth Division reversed the Labor Arbiter, ruling that [respondents],
except Carlito Tinghil, failed to implead Pamplona Plantation Leisure Corporation, an
The Decision5 of the National Labor Relations Commission (NLRC),6 reversed by the CA, indispensable party and that ‘there exist no employer-employee relation between the parties.’
disposed as follows:
xxxxxxxxx
"WHEREFORE, premises considered, the decision appealed from is hereby REVERSED, and
another one entered DISMISSING the complaint."7
"[Respondents] filed a motion for reconsideration which was denied by [the] NLRC in a
Resolution dated 06 December 2000."8
The June 17, 2003 Resolution denied petitioners’ Motion for Reconsideration.
Respondents elevated the case to the CA via a Petition for Certiorari under Rule 65 of the Rules
The Facts of Court.

The CA summarized the antecedents as follows: Ruling of the Court of Appeals

"Sometime in 1993, [Petitioner] Pamplona Plantations Company, Inc. (company for brevity) was Guided by the fourfold test for determining the existence of an employer-employee relationship,
organized for the purpose of taking over the operations of the coconut and sugar plantation of the CA held that respondents were employees of petitioner-company. Finding there was a
Hacienda Pamplona located in Pamplona, Negros Oriental. It appears that Hacienda Pamplona "power to hire," the appellate court considered the admission of petitioners in their Comment that
was formerly owned by a certain Mr. Bower who had in his employ several agricultural workers. they had hired respondents as coconut filers, coconut scoopers, charcoal makers, or as
pieceworkers. The fact that respondents were paid by piecework did not mean that they were
not employees of the company. Further, the CA ruled that petitioners necessarily exercised considered, would justify a different conclusion; (9) the findings of fact are conclusions without
control over the work they performed, since the latter were working within the premises of the citation of the specific evidence on which they are based; and (10) when the findings of fact of
plantation. According to the CA, the mere existence -- not necessarily the actual exercise -- of the CA are premised on the absence of evidence but such findings are contradicted by the
the right to control the manner of doing work sufficed to meet the fourth element of an employer- evidence on record.17
employee relation.
The very same reason that constrained the appellate court to review the factual findings of the
The appellate court also held that respondents were regular employees, because the tasks they NLRC impels this Court to take its own look at the facts. Normally, the Supreme Court is not a
performed were necessary and indispensable to the operation of the company. Since there was trier of facts.18 However, since the findings of the CA and the NLRC on this point were
no compliance with the twin requirements of a valid and/or authorized cause and of procedural conflicting, we waded through the records to find out if there was basis for the former’s reversal
due process, their dismissal was illegal. of the NLRC’s Decision. We shall discuss our factual findings together with our review of the
main issue.
Hence, this Petition.9
Main Issue:
Issues
Piercing the Corporate Veil
In their Memorandum, petitioners submit the following issues for our consideration:
Petitioners contend that the CA should have dismissed the case for the failure of respondents
(except Carlito Tinghil) to implead the Pamplona Plantation Leisure Corporation, an
"1. Whether or not the finding of the Court of Appeals that herein respondents are employees of
indispensable party, for being the true and real employer. Allegedly, respondents admitted in
Petitioner Pamplona Plantation Company, Inc. is contrary to the admissions of the respondents
their Affidavits dated February 3, 1998,19 that they had been employed by the leisure corporation
themselves.
and/or engaged to perform activities that pertained to its business.

"2. Whether or not the Court of Appeals has decided in a way not in accord with law and
Further, as the NLRC allegedly noted in their individual Complaints, respondents specifically
jurisprudence, and with grave abuse of discretion, in not dismissing the respondents’ complaint
averred that they had worked in the "golf course" and performed related jobs in the "recreational
for failure to implead Pamplona Plantation Leisure Corp., which is an indispensable party to this
facilities" of the leisure corporation. Hence, petitioners claim that, as a sugar and coconut
case.
plantation company separate and distinct from the Pamplona Plantation Leisure Corporation, the
petitioner-company is not the real party in interest.
"3. Whether or not the Court of Appeals has decided in a way not in accord with law and
jurisprudence, and with grave abuse of discretion in ordering reinstatement or payment of
We are not persuaded.
separation pay and backwages to the respondents, considering the lack of employer-employee
relationship between petitioner and respondents."10
An examination of the facts reveals that, for both the coconut plantation and the golf course,
there is only one management which the laborers deal with regarding their work. 20 A portion of
The main issue raised is whether the case should be dismissed for the non-joinder of the
the plantation (also called Hacienda Pamplona) had actually been converted into a golf course
Pamplona Plantation Leisure Corporation. The other issues will be taken up in the discussion of
and other recreational facilities. The weekly payrolls issued by petitioner-company bore the
the main question.
name "Pamplona Plantation Co., Inc."21 It is also a fact that respondents all received their pay
from the same person, Petitioner Bondoc -- the managing director of the company. Since the
The Court’s Ruling workers were working for a firm known as Pamplona Plantation Co., Inc., the reason they sued
their employer through that name was natural and understandable.
The Petition lacks merit.
True, the Petitioner Pamplona Plantation Co., Inc., and the Pamplona Plantation Leisure
Corporation appear to be separate corporate entities. But it is settled that this fiction of law
Preliminary Issue: cannot be invoked to further an end subversive of justice.22

Factual Matters The principle requiring the piercing of the corporate veil mandates courts to see through the
protective shroud that distinguishes one corporation from a seemingly separate one.23 The
Section 1 of Rule 45 of the Rules of Court states that only questions of law are entertained in corporate mask may be removed and the corporate veil pierced when a corporation is the mere
appeals by certiorari to the Supreme Court. However, jurisprudence has recognized several alter ego of another.24 Where badges of fraud exist, where public convenience is defeated,
exceptions in which factual issues may be resolved by this Court:11 (1) the legal conclusions where a wrong is sought to be justified thereby, or where a separate corporate identity is used to
made by the lower tribunal are speculative;12 (2) its inferences are manifestly mistaken,13 evade financial obligations to employees or to third parties, 25 the notion of separate legal entity
absurd, or impossible; (3) the lower court committed grave abuse of discretion; (4) the judgment should be set aside26 and the factual truth upheld. When that happens, the corporate character
is based on a misapprehension of facts;14 (5) the findings of fact of the lower tribunals are is not necessarily abrogated.27 It continues for other legitimate objectives. However, it may be
conflicting;15 (6) the CA went beyond the issues; (7) the CA’s findings are contrary to the pierced in any of the instances cited in order to promote substantial justice.
admissions of the parties;16 (8) the CA manifestly overlooked facts not disputed which, if
In the present case, the corporations have basically the same incorporators and directors and petitioners. It would certainly be unjust to prejudice the claims of the workers because of the
are headed by the same official. Both use only one office and one payroll and are under one misleading actions of their employer.
management. In their individual Affidavits, respondents allege that they worked under the
supervision and control of Petitioner Bondoc -- the common managing director of both the
Non-Joinder of Parties
petitioner-company and the leisure corporation. Some of the laborers of the plantation also work
in the golf course.28 Thus, the attempt to make the two corporations appear as two separate
entities, insofar as the workers are concerned, should be viewed as a devious but obvious Granting for the sake of argument that the Pamplona Plantation Leisure Corporation is an
means to defeat the ends of the law. Such a ploy should not be permitted to cloud the truth and indispensable party that should be impleaded, NLRC’s outright dismissal of the Complaints was
perpetrate an injustice. still erroneous.

We note that this defense of separate corporate identity was not raised during the proceedings The non-joinder of indispensable parties is not a ground for the dismissal of an action. 34 At any
before the labor arbiter. The main argument therein raised by petitioners was their alleged lack stage of a judicial proceeding and/or at such times as are just, parties may be added on the
of employer-employee relationship with, and power of control over, the means and methods of motion of a party or on the initiative of the tribunal concerned. 35 If the plaintiff refuses to implead
work of respondents because of the seasonal nature of the latter’s work. 29 an indispensable party despite the order of the court, that court may dismiss the complaint for
the plaintiff’s failure to comply with the order. The remedy is to implead the non-party claimed to
be indispensable.36 In this case, the NLRC did not require respondents to implead the Pamplona
Neither was the issue of non-joinder of indispensable parties raised in petitioners’ appeal before
Plantation Leisure Corporation as respondent; instead, the Commission summarily dismissed
the NLRC.30Nevertheless, in its Decision31 dated July 19, 2000, the Commission concluded that
the Complaints.
the plantation company and the leisure corporation were two separate and distinct corporations,
and that the latter was an indispensable party that should have been impleaded. We quote
below pertinent portions of that Decision: In any event, there is no need to implead the leisure corporation because, insofar as
respondents are concerned, the leisure corporation and petitioner-company are one and the
same entity. Salvador v. Court of Appeals37 has held that this Court has "full powers, apart from
"Respondent posits that it is engaged in operating and maintaining sugar and coconut
that power and authority which is inherent, to amend the processes, pleadings, proceedings and
plantation. The positions of complainants could only be determined through their individual
decisions by substituting as party-plaintiff the real party-in-interest."
complaints. Yet all complainants alleged in their affidavits x x x that they were working at the
‘golf course.’ Worthy to note that only Carlito Tinghil amended his complaint to include Pamplona
Leisure Corporation, which respondents maintain is a separate corporation established in 1995. In Alonso v. Villamor,38 we had the occasion to state thus:
Thus, xxx Pamplona Plantation Co., Inc. and Pamplona Leisure Corporation are two separate
and distinct corporations. Except for Carlito Tinghil the complainants have the wrong party
"There is nothing sacred about processes or pleadings, their forms or contents. Their sole
respondent. Pamplona Leisure Corporation is an indispensable party without which there could
be no final determination of the case."32 purpose is to facilitate the application of justice to the rival claims of contending parties. They
were created, not to hinder and delay, but to facilitate and promote, the administration of justice.
They do not constitute the thing itself, which courts are always striving to secure to litigants.
Indeed, it was only after this NLRC Decision was issued that the petitioners harped on the They are designed as the means best adapted to obtain that thing. In other words, they are a
separate personality of the Pamplona Plantation Co., Inc., vis-à-vis the Pamplona Plantation means to an end. When they lose the character of the one and become the other, the
Leisure Corporation. administration of justice is at fault and courts are correspondingly remiss in the performance of
their obvious duty."
As cited above, the NLRC dismissed the Complaints because of the alleged admission of
respondents in their Affidavits that they had been working at the golf course. However, it failed to The controlling principle in the interpretation of procedural rules is liberality, so that they may
appreciate the rest of their averments. Just because they worked at the golf course did not promote their object and assist the parties in obtaining just, speedy and inexpensive
necessarily mean that they were not employed to do other tasks, especially since the golf course determination of every action and proceeding.39When the rules are applied to labor cases, this
was merely a portion of the coconut plantation. Even petitioners admitted that respondents had liberal interpretation must be upheld with even greater vigor. 40Without in any way depriving the
been hired as coconut filers, coconut scoopers or charcoal makers. 33 Consequently, NLRC’s employer of its legal rights, the thrust of statutes and rules governing labor cases has been to
conclusion derived from the Affidavits of respondents stating that they were employees of the benefit workers and avoid subjecting them to great delays and hardships. This intent holds
Pamplona Plantation Leisure Corporation alone was the result of an improper selective especially in this case, in which the plaintiffs are poor laborers.
appreciation of the entire evidence.
Employer-Employee Relationship
Furthermore, we note that, contrary to the NLRC’s findings, some respondents indicated that
their employer was the Pamplona Plantation Leisure Corporation, while others said that it was
the Pamplona Plantation Co., Inc. But in all these Affidavits, both the leisure corporation and Petitioners insist that respondents are not their employees, because the former exercised no
control over the latter’s work hours and method of performing tasks. Thus, petitioners contend
petitioner-company were identified or described as entities engaged in the development and
operation of sugar and coconut plantations, as well as recreational facilities such as a golf that under the "control test," the workers were independent contractors.
course. These allegations reveal that petitioner successfully confused the workers as to who
their true and real employer was. All things considered, their faulty belief that the plantation We disagree. As shown by the evidence on record, petitioners hired respondents, who
company and the leisure corporation were one and the same can be attributed solely to performed tasks assigned by their respective officers-in-charge, who in turn were all under the
direct supervision and control of Petitioner Bondoc. These allegations are contained in the
workers’ Affidavits, which were never disputed by petitioners. Also uncontroverted are the
payrolls bearing the name of the plantation company and signed by Petitioner Bondoc. Some of
these payrolls include the time records of the employees. These documents prove that
petitioner-company exercised control and supervision over them.

To operate against the employer, the power of control need not have been actually exercised.
Proof of the existence of such power is enough. 41 Certainly, petitioners wielded that power to
hire or dismiss, as well as to check on the progress and the quality of work of the laborers.

Jurisprudence provides other equally important considerations 42 that support the conclusion that
respondents were not independent contractors. First, they cannot be said to have carried on an
independent business or occupation.43They are not engaged in the business of filing, scooping
and hauling coconuts and/or operating and maintaining a plantation and a golf course. Second,
they do not have substantial capital or investment in the form of tools, equipment, machinery,
work premises, and other implements needed to perform the job, work or service under their
own account or responsibility.44 Third, they have been working exclusively for petitioners for
several years. Fourth, there is no dispute that petitioners are in the business of growing coconut
trees for commercial purposes. There is no question, either, that a portion of the plantation was
converted into a golf course and other recreational facilities. Clearly, respondents performed
usual, regular and necessary services for petitioners’ business.

WHEREFORE, the Petition is DENIED, and the assailed Decision AFFIRMED. Costs against
the petitioners.

SO ORDERED.
G.R. No. L-67626 April 18, 1989 Upon inquiry, private respondent found that no loan application was ever filed by Akron with
DBP. 12
JOSE REMO, JR., petitioner,
vs. In the meantime, Akron paid rentals of P500.00 a day pursuant to a subsequent agreement,
THE HON. INTERMEDIATE APPELLATE COURT and E.B. MARCHA TRANSPORT from April 27, 1978 (the end of the 90-day period to pay the balance) to May 31, 1978.
COMPANY, INC., represented by APIFANIO B. MARCHA, respondents. Thereafter, no more rental payments were made.

GANCAYCO, J.: On June 17, 1978, Coprada wrote private respondent begging for a grace period of until the end
of the month to pay the balance of the purchase price; that he will update the rentals within the
week; and in case he fails, then he will return the 13 units should private respondent elect to get
A corporation is an entity separate and distinct from its stockholders. While not in fact and in
back the same. 13 Private respondent, through counsel, wrote Akron on August 1, 1978
reality a person, the law treats a corporation as though it were a person by process of fiction or
demanding the return of the 13 trucks and the payment of P25,000.00 back rentals covering the
by regarding it as an artificial person distinct and separate from its individual stockholders. 1
period from June 1 to August 1, 1978. 14

However, the corporate fiction or the notion of legal entity may be disregarded when it "is used
Again, Coprada wrote private respondent on August 8, 1978 asking for another grace period of
to defeat public convenience, justify wrong, protect fraud, or defend crime" in which instances
up to August 31, 1978 to pay the balance, stating as well that he is expecting the approval of his
"the law will regard the corporation as an association of persons, or in case of two corporations,
loan application from a certain financing company, and that ten (10) trucks have been returned
will merge them into one." The corporate fiction may also be disregarded when it is the "mere
to Bagbag, Novaliches. 15 On December 9, 1978, Coprada informed private respondent anew
alter ego or business conduit of a person." 2 There are many occasions when this Court pierced
that he had returned ten (10) trucks to Bagbag and that a resolution was passed by the board of
the corporate veil because of its use to protect fraud and to justify wrong. 3 The herein petition
directors confirming the deed of assignment to private respondent of P475,000 from the
for review of a. resolution of the Intermediate Appellate Court dated February 8, 1984 seeking
proceeds of a loan obtained by Akron from the State Investment House, Inc. 16
the reversal thereof and the reinstatement of its earlier decision dated June 30, 1983 in AC-G.R.
No. 68496-R 4 calls for the application of the foregoing principles.
In due time, private respondent filed a compliant for the recovery of P525,000.00 or the return of
the 13 trucks with damages against Akron and its officers and directors, Feliciano Coprada,
In the latter part of December, 1977 the board of directors of Akron Customs Brokerage
Dario D. Punzalan, Jemina Coprada, Lucia Lacaste, Wilfredo Layug, Arcadio de la Cruz,
Corporation (hereinafter referred to as Akron), composed of petitioner Jose Remo, Jr., Ernesto
Francisco Clave, Vicente Martinez, Pacifico Dollario and petitioner with the then Court of First
Bañares, Feliciano Coprada, Jemina Coprada, and Dario Punzalan with Lucia Lacaste as
Instance of Rizal. Only petitioner answered the complaint denying any participation in the
Secretary, adopted a resolution authorizing the purchase of thirteen (13) trucks for use in its
transaction and alleging that Akron has a distinct corporate personality. He was, however,
business to be paid out of a loan the corporation may secure from any lending institution. 5
declared in default for his failure to attend the pre-trial.

Feliciano Coprada, as President and Chairman of Akron, purchased thirteen trucks from private
In the meanwhile, petitioner sold all his shares in Akron to Coprada. It also appears that Akron
respondent on January 25, 1978 for and in consideration of P525,000.00 as evidenced by a
amended its articles of incorporation thereby changing its name to Akron Transport International,
deed of absolute sale. 6 In a side agreement of the same date, the parties agreed on a
Inc. which assumed the liability of Akron to private respondent.
downpayment in the amount of P50,000.00 and that the balance of P475,000.00 shall be paid
within sixty (60) days from the date of the execution of the agreement. The parties also agreed
that until said balance is fully paid, the down payment of P50,000.00 shall accrue as rentals of After an ex parte reception of the evidence of the private respondent, a decision was rendered
the 13 trucks; and that if Akron fails to pay the balance within the period of 60 days, then the on October 28, 1980, the dispositive part of which reads as follows:
balance shall constitute as a chattel mortgage lien covering said cargo trucks and the parties
may allow an extension of 30 days and thereafter private respondent may ask for a revocation of
Finding the evidence sufficient to prove the case of the plaintiff, judgment is hereby rendered in
the contract and the reconveyance of all said trucks. 7
favor of the plaintiff and against the defendants, ordering them jointly and severally to pay;

The obligation is further secured by a promissory note executed by Coprada in favor of Akron. It
a — the purchase price of the trucks in the amount of P525,000.00 with ... legal rate (of interest)
is stated in the promissory note that the balance shall be paid from the proceeds of a loan
from the filing of the complaint until the full amount is paid;
obtained from the Development Bank of the Philippines (DBP) within sixty (60) days. 8 After the
lapse of 90 days, private respondent tried to collect from Coprada but the latter promised to pay
only upon the release of the DBP loan. Private respondent sent Coprada a letter of demand b — rentals of Bagbag property at P1,000.00 a month from August 1978 until the premises is
dated May 10, 1978. 9 In his reply to the said letter, Coprada reiterated that he was applying for cleared of the said trucks;
a loan from the DBP from the proceeds of which payment of the obligation shall be made. 10
c — attorneys fees of P10,000.00, and
Meanwhile, two of the trucks were sold under a pacto de retro sale to a certain Mr. Bais of the
Perpetual Loans and Savings Bank at Baclaran. The sale was authorized by a board resolution
made in a meeting held on March 15, 1978. 11 d — costs of suit.
The P50,000.00 given as down payment shall pertain as rentals of the trucks from June 1 to respondent was duly informed in a letter. 19Indeed, the new corporation confirmed and assumed
August 1, 1978 which is P25,000.00 (see demand letter of Atty. Aniano Exhibit "T") and the the obligation of the old corporation. There is no indication of an attempt on the part of Akron to
remaining P25,000.00 shall be from August 1, 1978 until the trucks are removed totally from the evade payment of its obligation to private respondent.
place." 17
There is the fact that petitioner sold his shares in Akron to Coprada during the pendency of the
A motion for new trial filed by petitioner was denied so he appealed to the then Intermediate case. Since petitioner has no personal obligation to private respondent, it is his inherent right as
Appellate Court (IAC) wherein in due course a decision was rendered on June 30, 1 983 setting a stockholder to dispose of his shares of stock anytime he so desires.
aside the said decision as far as petitioner is concemed. However, upon a motion for
reconsideration filed by private respondent dent, the IAC, in a resolution dated February 8,1984,
Mention is also made of the alleged "dumping" of 10 units in the premises of private respondent
set aside the decision dated June 30, 1983. The appellate court entered another decision
at Bagbag, Novaliches which to the mind of the Court does not prove fraud and instead appears
affirming the appealed decision of the trial court, with costs against petitioner.
to be an attempt on the part of Akron to attend to its obligations as regards the said trucks. Again
petitioner has no part in this.
Hence, this petition for review wherein petitioner raises the following issues:
If the private respondent is the victim of fraud in this transaction, it has not been clearly shown
I. The Intermediate Appellate Court (IAC) erred in disregarding the corporate fiction and in that petitioner had any part or participation in the perpetration of the same. Fraud must be
holding the petitioner personally liable for the obligation of the Corporation which decision is established by clear and convincing evidence. If at all, the principal character on whom fault
patently contrary to law and the applicable decision thereon. should be attributed is Feliciano Coprada, the President of Akron, whom private respondent
dealt with personally all through out. Fortunately, private respondent obtained a judgment
against him from the trial court and the said judgment has long been final and executory.
II. The Intermediate Appellate Court (IAC) committed grave error of law in its decision by
sanctioning the merger of the personality of the corporation with that of the petitioner when the
latter was held liable for the corporate debts. 18 WHEREFORE, the petition is GRANTED. The questioned resolution of the Intermediate
Appellate Court dated February 8,1984 is hereby set aside and its decision dated June 30,1983
setting aside the decision of the trial court dated October 28, 1980 insofar as petitioner is
We reverse.
concemed is hereby reinstated and affirmed, without costs.

The environmental facts of this case show that there is no cogent basis to pierce the corporate
SO ORDERED.
veil of Akron and hold petitioner personally liable for its obligation to private respondent. While it
is true that in December, 1977 petitioner was still a member of the board of directors of Akron
and that he participated in the adoption of a resolution authorizing the purchase of 13 trucks for
the use in the brokerage business of Akron to be paid out of a loan to be secured from a lending
institution, it does not appear that said resolution was intended to defraud anyone and more
particularly private respondent. It was Coprada, President and Chairman of Akron, who
negotiated with said respondent for the purchase of 13 cargo trucks on January 25, 1978. It was
Coprada who signed a promissory note to guarantee the payment of the unpaid balance of the
purchase price out of the proceeds of a loan he supposedly sought from the DBP. The word
"WE' in the said promissory note must refer to the corporation which Coprada represented in the
execution of the note and not its stockholders or directors. Petitioner did not sign the said
promissory note so he cannot be personally bound thereby.

Thus, if there was any fraud or misrepresentation that was foisted on private respondent in that
there was a forthcoming loan from the DBP when it fact there was none, it is Coprada who
should account for the same and not petitioner.

As to the sale through pacto de retro of the two units to a third person by the corporation by
virtue of a board resolution, petitioner asserts that he never signed said resolution. Be that as it
may, the sale is not inherently fraudulent as the 13 units were sold through a deed of absolute
sale to Akron so that the corporation is free to dispose of the same. Of course, it was stipulated
that in case of default in payment to private respondent of the balance of the consideration, a
chattel mortgage lien shag be constituted on the 13 units. Nevertheless, said mortgage is a prior
lien as against the pacto de retro sale of the 2 units.

As to the amendment of the articles of incorporation of Akron thereby changing its name to
Akron Transport International, Inc., petitioner alleges that the change of corporate name was in
order to include trucking and container yard operations in its customs brokerage of which private
G.R. No. 85416 July 24, 1990 7. Per another certification issued by the Licensing Division of this Office, it also appears that
another corporation, Philsa International Placement & Services Corp., composed of practically
the same set of incorporators/stockholders, was registered as a licensed private employment
FRANCISCO V. DEL ROSARIO, petitioner,
agency whose license was issued on November 5, 1981, represented by the same Mr.
vs.
Francisco V. del Rosario as its President/ General Manager.
NATIONAL LABOR RELATIONS COMMISSION and LEONARDO V. ATIENZA, respondents.

and an application of the ruling of the Court in A.C. Ransom Labor Union-CCLU v. NLRC, G.R.
CORTES, J.:
No. 69494, June 10, 1986, 142 SCRA 269.

In POEA Case No. 85-06-0394, the Philippine Overseas Employment Administration (POEA)
However, we find that the NLRC's reliance on the findings of the POEA and the ruling in A. C.
promulgated a decision on February 4, 1986 dismissing the complaint for money claims for lack
Ransom is totally misplaced.
of merit. The decision was appealed to the National Labor Relations Commission (NLRC), which
on April 30, 1987 reversed the POEA decision and ordered Philsa Construction and Trading Co.,
Inc. (the recruiter) and Arieb Enterprises (the foreign employer) to jointly and severally pay 1. Under the law a corporation is bestowed juridical personality, separate and distinct from its
private respondent the peso equivalent of $16,039.00, as salary differentials, and $2,420.03, as stockholders [Civil Code, Art. 44; Corporation Code, sec. 2]. But when the juridical personality of
vacation leave benefits. The case was elevated to the Supreme Court, but the petition was the corporation is used to defeat public convenience, justify wrong, protect fraud or defend
dismissed on August 31, 1987 and entry of judgment was made on September 24, 1987. crime, the corporation shall be considered as a mere association of persons [Koppel (Phil.), Inc.
v. Yatco, 77 Phil. 496 (1946), citing 1 Fletcher, Cyclopedia of Corporations, 135-136; see also
Palay, Inc. v. Clave, G.R. No. 56076, September 21, 1983, 124 SCRA 638], and its responsible
A writ of execution was issued by the POEA but it was returned unsatisfied as Philsa was no
officers and/or stockholders shall be held individually liable [Namarco v. Associated Finance Co.,
longer operating and was financially incapable of satisfying the judgment. Private respondent
Inc., G.R. No. L-20886, April 27, 1967, 19 SCRA 962]. For the same reasons, a corporation shall
moved for the issuance of an alias writ against the officers of Philsa. This motion was opposed
be liable for the obligations of a stockholder [Palacio v. Fely Transportation Company, G.R. No.
by the officers, led by petitioner, the president and general manager of the corporation.
L-15121, August 31, 1962, 5 SCRA 1011; Emilio Cano Enterprises, Inc. v. Court of Industrial
Relations, G.R. No. L-20502, February 26, 1965, 13 SCRA 290], or a corporation and its
On February 12, 1988, the POEA issued a resolution, the dispositive portion of which read: successor-in-interest shall be considered as one and the liability of the former shall attach to the
latter [Koppel v. Yatco, supra; Liddell & Co. v. Collector of Internal Revenue, G.R. No. L-9687,
June 30, 1961, 2 SCRA 632].
WHEREFORE, premises considered, let an alias writ of Execution be issued and the handling
sheriff is ordered to execute against the properties of Mr. Francisco V. del -Rosario and if
insufficient, against the cash and/or surety bond of Bonding Company concerned for the full But for the separate juridical personality of a corporation to be disregarded, the wrongdoing must
satisfaction of the judgment awarded. be clearly and convincingly established. It cannot be presumed.

Petitioner appealed to the NLRC. On September 23, 1988, the NLRC dismissed the appeal. On In this regard we find the NLRC's decision wanting. The conclusion that Philsa allowed its
October 21, 1988, petitioner's motion for reconsideration was denied. license to expire so as to evade payment of private respondent's claim is not supported by the
facts. Philsa's corporate personality therefore remains inviolable.
Thus, this petition was filed on October 28, 1988, alleging that the NLRC gravely abused its
discretion. On November 10, 1988 the Court issued a temporary restraining order enjoining the Consider the following undisputed facts:
enforcement of the NLRC's decision dated September 23, 1988 and resolution dated October
21, 1988. The petition was given due course on June 14, 1989.
(1) Private respondent filed his complaint with the POEA on June 4, 1985;

After considering the undisputed facts and the arguments raised in the pleadings, the Court finds
(2) The last renewal of Philsa's license expired on October 12, 1985;
grave abuse of discretion on the part of the NLRC.

(3) The POEA dismissed private respondent's complaint on February 4, 1986;


The action of the NLRC affirming the issuance of an alias writ of execution against petitioner, on
the theory that the corporate personality of Philsa should be disregarded, was founded primarily
on the following findings of the POEA — (4) Philsa was delisted for inactivity on August 15, 1986; *

xxx xxx xxx (5) The dismissal of the complaint was appealed to the NLRC and it was only on April 30, 1987
that the judgment awarding differentials and benefits to private respondent was rendered.
6. Per the certification issued by the Licensing Division of this Office, it appears that Philsa
Construction & Trading Co., Inc., with office address at 126 Pioneer St., Mandaluyong, Metro Thus, at the time Philsa allowed its license to lapse in 1985 and even at the time it was delisted
Manila, represented by Mr. Francisco V. del Rosario, President and General Manager, was in 1986, there was yet no judgment in favor of private respondent. An intent to evade payment of
formerly a registered construction contractor whose authority was originally issued on July 21, his claims cannot therefore be implied from the expiration of Philsa's license and its delisting.
1978 but was already delisted from the list of agencies/entities on August 15, 1986 for inactivity;
Neither will the organization of Philsa International Placement and Services Corp. and its At this juncture, the Court finds it appropriate to point out that a judgment against a recruiter
registration with the POEA as a private employment agency imply fraud since it was organized should initially be enforced against the cash and surety bonds filed with the POEA. As provided
and registered in 1981, several years before private respondent filed his complaint with the in the POEA Rules and Regulations —
POEA in 1985. The creation of the second corporation could not therefore have been in
anticipation of private respondent's money claims and the consequent adverse judgment against
... The bonds shall answer for all valid and legal claims arising from violations of the conditions
Philsa
for the grant and use of the license or authority and contracts of employment. The bonds shall
likewise guarantee compliance with the provisions of the Labor Code and its implementing rules
Likewise, substantial identity of the incorporators of the two corporations does not necessarily and regulations relating to recruitment and placement, the rules of the Administration and
imply fraud. relevant issuances of the Ministry and all liabilities which the Administration may impose. ...
[Rule II, see. 4.]
The circumstances of this case distinguish it from those in earlier decisions of the Court in labor
cases where the veil of corporate fiction was pierced. Quite evidently, these bonds do not answer for a single specific liability, but for all sorts of
liabilities of the recruiter to the worker and to the POEA. Moreover, the obligations guaranteed
by the bonds are continuing. Thus, the bonds are subject to replenishment when they are
In La Campana Coffee Factory, Inc. v. Kaisahan ng Manggagawa sa La Campana (KKM) 93
garnished, and failure to replenish shall cause the suspension or cancellation of the recruiter's
Phil. 160 (1953), La Campana Coffee Factory, Inc. and La Campana Gaugau Packing were
license [Rule II, sec. 19]. Furthermore, a cash bond shall be refunded to a recruiter who
substantially owned by the same person. They had one office, one management, and a single
surrenders his license only upon posting of a surety bond of similar amount valid for three (3)
payroll for both businesses. The laborers of the gaugau factory and the coffee factory were also
years [Rule II, sec. 20]. All these, to ensure recovery from the recruiter.
interchangeable, i.e., the workers in one factory worked also in the other factory.

It is therefore surprising why the POEA ordered execution "against the properties of Mr.
In Claparols v. Court of Industrial Relations, G.R. No. L-30822, July 31, 1975, 65 SCRA 613, the
Francisco V. del Rosario and if insufficient, against the cash and/or surety bond of Bonding
Claparols Steel and Nail Plant, which was ordered to pay its workers backwages, ceased
Company concerned for the till satisfaction of the judgment awarded" in complete disregard of
operations on June 30, 1957 and was succeeded on the next day, July 1, 1957 by the Claparols
the scheme outlined in the POEA Rules and Regulations. On this score alone, the NLRC should
Steel Corporation. Both corporations were substantially owned and controlled by the same
not have affirmed the POEA.
person and there was no break or cessation in operations. Moreover, all the assets of the steel
and nail plant were transferred to the new corporation.
WHEREFORE, the petition is GRANTED and the decision and resolution of the NLRC, dated
September 23, 1988 and October 21, 1988, respectively, in POEA Case No. 85-06-0394 are
2. As earlier stated, we also find that, contrary to the NLRC'S holding, the ruling in A. C.
SET ASIDE. The temporary restraining order issued by the Court on November 10, 1988 is
Ransom is inapplicable to this case. In A. C. Ransom, the Court said:
MADE PERMANENT.

... In the instant case, it would appear that RANSOM, in 1969, foreseeing the possibility or
SO ORDERED.
probability of payment of back wages to the 22 strikers, organized ROSARIO to replace
RANSOM, with the latter to be eventually phased out if the 22 strikers win their case. RANSOM
actually ceased operations on May 1, 1973, after the December 19, 1972 Decision of the Court
of Industrial Relations was promulgated against RANSOM. [At p. 274.]

The distinguishing marks of fraud were therefore clearly apparent in A. C. Ransom. A new
corporation was created, owned by the same family, engaging in the same business and
operating in the same compound.

Thus, considering that the non-payment of the workers was a continuing situation, the Court
adjudged its President, the "responsible officer" of the corporation, personally liable for the
backwages awarded, he being the chief operation officer or "manager" who could be held
criminally liable for violations of Republic Act No. 602 (the old Minimum Wage Law.)

In the case now before us, not only has there been a failure to establish fraud, but it has also not
been shown that petitioner is the corporate officer responsible for private respondent's
predicament. It must be emphasized that the claim for differentials and benefits was actually
directed against the foreign employer. Philsa became liable only because of its undertaking to be
jointly and severally bound with the foreign employer, an undertaking required by the rules of the
POEA [Rule II, sec. 1(d) (3)], together with the filing of cash and surety bonds [Rule 11, sec. 4],
in order to ensure that overseas workers shall find satisfaction for awards in their favor.
G.R. No. 96490 February 3, 1992 The existing impasse led the petitioner and private respondent to enter into a submission
agreement on September 6, 1990. The parties jointly requested the public respondent to act as
voluntary arbitrator in the resolution of the pending labor dispute pertaining to the proper
INDOPHIL TEXTILE MILL WORKERS UNION-PTGWO, petitioner,
interpretation of the CBA provision.
vs.
VOLUNTARY ARBITRATOR TEODORICO P. CALICA and INDOPHIL TEXTILE MILLS,
INC., respondents. After the parties submitted their respective position papers and replies, the public respondent
Voluntary Arbitrator rendered its award on December 8, 1990, the dispositive portion of which
provides as follows:
MEDIALDEA, J.:

PREMISES CONSIDERED, it would be a strained interpretation and application of the


This is a petition for certiorari seeking the nullification of the award issued by the respondent
questioned CBA provision if we would extend to the employees of Acrylic the coverage clause of
Voluntary Arbitrator Teodorico P. Calica dated December 8, 1990 finding that Section 1 (c),
Indophil Textile Mills CBA. Wherefore, an award is made to the effect that the proper
Article I of the Collective Bargaining Agreement between Indophil Textile Mills, Inc. and Indophil
interpretation and application of Sec. l, (c), Art. I, of the 1987 CBA do (sic) not extend to the
Textile Mill Workers Union-PTGWO does not extend to the employees of Indophil Acrylic
employees of Acrylic as an extension or expansion of Indophil Textile Mills, Inc. (Rollo, p.21)
Manufacturing Corporation as an extension or expansion of Indophil Textile Mills, Incorporated.

Hence, this petition raising four (4) issues, to wit:


The antecedent facts are as follows:

1. WHETHER OR NOT THE RESPONDENT ARBITRATOR ERRED IN INTERPRETING


Petitioner Indophil Textile Mill Workers Union-PTGWO is a legitimate labor organization duly
SECTION 1(c), ART I OF THE CBA BETWEEN PETITIONER UNION AND RESPONDENT
registered with the Department of Labor and Employment and the exclusive bargaining agent of
COMPANY.
all the rank-and-file employees of Indophil Textile Mills, Incorporated. Respondent Teodorico P.
Calica is impleaded in his official capacity as the Voluntary Arbitrator of the National Conciliation
and Mediation Board of the Department of Labor and Employment, while private respondent 2. WHETHER OR NOT INDOPHIL ACRYLIC IS A SEPARATE AND DISTINCT ENTITY FROM
Indophil Textile Mills, Inc. is a corporation engaged in the manufacture, sale and export of yarns RESPONDENT COMPANY FOR PURPOSES OF UNION REPRESENTATION.
of various counts and kinds and of materials of kindred character and has its plants at Barrio
Lambakin. Marilao, Bulacan.
3. WHETHER OR NOT THE RESPONDENT ARBITRATOR GRAVELY ABUSED HIS
DISCRETION AMOUNTING TO LACK OR IN EXCESS OF HIS JURISDICTION.
In April, 1987, petitioner Indophil Textile Mill Workers Union-PTGWO and private respondent
Indophil Textile Mills, Inc. executed a collective bargaining agreement effective from April 1,
4. WHETHER OR NOT THE RESPONDENT ARBITRATOR VIOLATED PETITIONER UNION'S
1987 to March 31, 1990.
CARDINAL PRIMARY RIGHT TO DUE PROCESS. (Rollo, pp. 6-7)

On November 3, 1967 Indophil Acrylic Manufacturing Corporation was formed and registered
The central issue submitted for arbitration is whether or not the operations in Indophil Acrylic
with the Securities and Exchange Commission. Subsequently, Acrylic applied for registration
Corporation are an extension or expansion of private respondent Company. Corollary to the
with the Board of Investments for incentives under the 1987 Omnibus Investments Code. The
aforementioned issue is the question of whether or not the rank-and-file employees working at
application was approved on a preferred non-pioneer status.
Indophil Acrylic should be recognized as part of, and/or within the scope of the bargaining unit.

In 1988, Acrylic became operational and hired workers according to its own criteria and
Petitioner maintains that public respondent Arbitrator gravely erred in interpreting Section l(c),
standards. Sometime in July, 1989, the workers of Acrylic unionized and a duly certified
Article I of the CBA in its literal meaning without taking cognizance of the facts adduced that the
collective bargaining agreement was executed.
creation of the aforesaid Indophil Acrylic is but a devise of respondent Company to evade the
application of the CBA between petitioner Union and respondent Company.
In 1990 or a year after the workers of Acrylic have been unionized and a CBA executed, the
petitioner union claimed that the plant facilities built and set up by Acrylic should be considered
Petitioner stresses that the articles of incorporation of the two corporations establish that the two
as an extension or expansion of the facilities of private respondent Company pursuant to Section
entities are engaged in the same kind of business, which is the manufacture and sale of yarns of
1(c), Article I of the CBA, to wit,.
various counts and kinds and of other materials of kindred character or nature.

c) This Agreement shall apply to the Company's plant facilities and installations and to any
Contrary to petitioner's assertion, the public respondent through the Solicitor General argues
extension and expansion thereat. (Rollo, p.4)
that the Indophil Acrylic Manufacturing Corporation is not an alter ego or an adjunct or business
conduit of private respondent because it has a separate legitimate business purpose. In addition,
In other words, it is the petitioner's contention that Acrylic is part of the Indophil bargaining unit. the Solicitor General alleges that the primary purpose of private respondent is to engage in the
business of manufacturing yarns of various counts and kinds and textiles. On the other hand, the
primary purpose of Indophil Acrylic is to manufacture, buy, sell at wholesale basis, barter, import,
The petitioner's contention was opposed by private respondent which submits that it is a juridical
export and otherwise deal in yarns of various counts and kinds. Hence, unlike private
entity separate and distinct from Acrylic.
respondent, Indophil Acrylic cannot manufacture textiles while private respondent cannot buy or It should be emphasized that in rendering the subject arbitral award, the voluntary arbitrator
import yarns. Teodorico Calica, a professor of the U.P. Asian Labor Education Center, now the Institute for
Industrial Relations, found that the existing law and jurisprudence on the matter, supported the
private respondent's contentions. Contrary to petitioner's assertion, public respondent cited facts
Furthermore, petitioner emphasizes that the two corporations have practically the same
and the law upon which he based the award. Hence, public respondent did not abuse his
incorporators, directors and officers. In fact, of the total stock subscription of Indophil Acrylic,
discretion.
P1,749,970.00 which represents seventy percent (70%) of the total subscription of
P2,500,000.00 was subscribed to by respondent Company.
Under the doctrine of piercing the veil of corporate entity, when valid grounds therefore exist, the
legal fiction that a corporation is an entity with a juridical personality separate and distinct from
On this point, private respondent cited the case of Diatagon Labor Federation v. Ople, G.R. No.
its members or stockholders may be disregarded. In such cases, the corporation will be
L-44493-94, December 3, 1980, 10l SCRA 534, which ruled that two corporations cannot be
considered as a mere association of persons. The members or stockholders of the corporation
treated as a single bargaining unit even if their businesses are related. It submits that the fact
will be considered as the corporation, that is liability will attach directly to the officers and
that there are as many bargaining units as there are companies in a conglomeration of
stockholders. The doctrine applies when the corporate fiction is used to defeat public
companies is a positive proof that a corporation is endowed with a legal personality distinctly its
convenience, justify wrong, protect fraud, or defend crime, or when it is made as a shield to
own, independent and separate from other corporations (see Rollo, pp. 160-161).
confuse the legitimate issues, or where a corporation is the mere alter ego or business conduit
of a person, or where the corporation is so organized and controlled and its affairs are so
Petitioner notes that the foregoing evidence sufficiently establish that Acrylic is but an extension conducted as to make it merely an instrumentality, agency, conduit or adjunct of another
or expansion of private respondent, to wit: corporation. (Umali et al. v. Court of Appeals, G.R. No. 89561, September 13, 1990, 189 SCRA
529, 542)
(a) the two corporations have their physical plants, offices and facilities situated in the same
compound, at Barrio Lambakin, Marilao, Bulacan; In the case at bar, petitioner seeks to pierce the veil of corporate entity of Acrylic, alleging that
the creation of the corporation is a devise to evade the application of the CBA between petitioner
Union and private respondent Company. While we do not discount the possibility of the
(b) many of private respondent's own machineries, such as dyeing machines, reeling, boiler, similarities of the businesses of private respondent and Acrylic, neither are we inclined to apply
Kamitsus among others, were transferred to and are now installed and being used in the Acrylic the doctrine invoked by petitioner in granting the relief sought. The fact that the businesses of
plant;
private respondent and Acrylic are related, that some of the employees of the private respondent
are the same persons manning and providing for auxilliary services to the units of Acrylic, and
(c) the services of a number of units, departments or sections of private respondent are provided that the physical plants, offices and facilities are situated in the same compound, it is our
to Acrylic; and considered opinion that these facts are not sufficient to justify the piercing of the corporate veil of
Acrylic.
(d) the employees of private respondent are the same persons manning and servicing the units
of Acrylic. (see Rollo, pp. 12-13) In the same case of Umali, et al. v. Court of Appeals (supra), We already emphasized that "the
legal corporate entity is disregarded only if it is sought to hold the officers and stockholders
directly liable for a corporate debt or obligation." In the instant case, petitioner does not seek to
Private respondent insists that the existence of a bonafide business relationship between Acrylic impose a claim against the members of the Acrylic.
and private respondent is not a proof of being a single corporate entity because the services
which are supposedly provided by it to Acrylic are auxiliary services or activities which are not
really essential in the actual production of Acrylic. It also pointed out that the essential services Furthermore, We already ruled in the case of Diatagon Labor Federation Local 110 of the
are discharged exclusively by Acrylic personnel under the control and supervision of Acrylic ULGWP v. Ople (supra) that it is grave abuse of discretion to treat two companies as a single
managers and supervisors. bargaining unit when these companies are indubitably distinct entities with separate juridical
personalities.
In sum, petitioner insists that the public respondent committed grave abuse of discretion
amounting to lack or in excess of jurisdiction in erroneously interpreting the CBA provision and in Hence, the Acrylic not being an extension or expansion of private respondent, the rank-and-file
failing to disregard the corporate entity of Acrylic. employees working at Acrylic should not be recognized as part of, and/or within the scope of the
petitioner, as the bargaining representative of private respondent.
We find the petition devoid of merit.
All premises considered, the Court is convinced that the public respondent Voluntary Arbitrator
did not commit grave abuse of discretion in its interpretation of Section l(c), Article I of the CBA
Time and again, We stress that the decisions of voluntary arbitrators are to be given the highest that the Acrylic is not an extension or expansion of private respondent.
respect and a certain measure of finality, but this is not a hard and fast rule, it does not preclude
judicial review thereof where want of jurisdiction, grave abuse of discretion, violation of due
process, denial of substantial justice, or erroneous interpretation of the law were brought to our ACCORDINGLY, the petition is DENIED and the award of the respondent Voluntary Arbitrator
attention. (see Ocampo, et al. v. National Labor Relations Commission, G.R. No. 81677, 25 July are hereby AFFIRMED.
1990, First Division Minute Resolution citing Oceanic Bic Division (FFW) v. Romero, G.R. No. L-
43890, July 16, 1984, 130 SCRA 392) SO ORDERED.
G.R. No. 142616 July 31, 2001 dismissed the petition. Petitioner thus seeks recourse to this Court and raises the following
errors:
PHILIPPINE NATIONAL BANK, petitioner,
vs. 1.
RITRATTO GROUP INC., RIATTO INTERNATIONAL, INC., and DADASAN GENERAL
MERCHANDISE,respondents.
THE COURT OF APPEALS PALPABLY ERRED IN NOT DISMISSING THE COMPLAINT A
QUO, CONSIDERING THAT BY THE ALLEGATIONS OF THE COMPLAINT, NO CAUSE OF
KAPUNAN, J.: ACTION EXISTS AGAINST PETITIONER, WHICH IS NOT A REAL PARTY IN INTEREST
BEING A MERE ATTORNEY-IN-FACT AUTHORIZED TO ENFORCE AN ANCILLARY
CONTRACT.
In a petition for review on certiorari under Rule 45 of the Revised Rules of Court, petitioner
seeks to annul and set aside the Court of Appeals' decision in C.A. CV G.R. S.P. No. 55374
dated March 27, 2000, affirming the Order issuing a writ of preliminary injunction of the Regional 2.
Trial Court of Makati, Branch 147 dated June 30, 1999, and its Order dated October 4, 1999,
which denied petitioner's motion to dismiss.
THE COURT OF APPEALS PALPABLY ERRED IN ALLOWING THE TRIAL COURT TO ISSUE
IN EXCESS OR LACK OF JURISDICTION A WRIT OF PRELIMINARY INJUNCTION OVER
The antecedents of this case are as follows: AND BEYOND WHAT WAS PRAYED FOR IN THE COMPLAINT A QUO CONTRARY
TO CHIEF OF STAFF, AFP VS. GUADIZ JR., 101 SCRA 827.2
Petitioner Philippine National Bank is a domestic corporation organized and existing under
Philippine law. Meanwhile, respondents Ritratto Group, Inc., Riatto International, Inc. and Petitioner prays, inter alia, that the Court of Appeals' Decision dated March 27, 2000 and the trial
Dadasan General Merchandise are domestic corporations, likewise, organized and existing court's Orders dated June 30, 1999 and October 4, 1999 be set aside and the dismissal of the
under Philippine law. complaint in the instant case.3

On May 29, 1996, PNB International Finance Ltd. (PNB-IFL) a subsidiary company of PNB, In their Comment, respondents argue that even assuming arguendo that petitioner and PNB-IFL
organized and doing business in Hong Kong, extended a letter of credit in favor of the are two separate entities, petitioner is still the party-in-interest in the application for preliminary
respondents in the amount of US$300,000.00 secured by real estate mortgages constituted over injunction because it is tasked to commit acts of foreclosing respondents'
four (4) parcels of land in Makati City. This credit facility was later increased successively to properties.4 Respondents maintain that the entire credit facility is void as it contains stipulations
US$1,140,000.00 in September 1996; to US$1,290,000.00 in November 1996; to in violation of the principle of mutuality of contracts. 5 In addition, respondents justified the act of
US$1,425,000.00 in February 1997; and decreased to US$1,421,316.18 in April 1998. the court a quo in applying the doctrine of "Piercing the Veil of Corporate Identity" by stating that
Respondents made repayments of the loan incurred by remitting those amounts to their loan petitioner is merely an alter ego or a business conduit of PNB-IFL.6
account with PNB-IFL in Hong Kong.
The petition is impressed with merit.
However, as of April 30, 1998, their outstanding obligations stood at US$1,497,274.70. Pursuant
to the terms of the real estate mortgages, PNB-IFL, through its attorney-in-fact PNB, notified the
Respondents, in their complaint, anchor their prayer for injunction on alleged invalid provisions
respondents of the foreclosure of all the real estate mortgages and that the properties subject
of the contract:
thereof were to be sold at a public auction on May 27, 1999 at the Makati City Hall.

GROUNDS
On May 25, 1999, respondents filed a complaint for injunction with prayer for the issuance of a
writ of preliminary injunction and/or temporary restraining order before the Regional Trial Court
of Makati. The Executive Judge of the Regional Trial Court of Makati issued a 72-hour temporary I
restraining order. On May 28, 1999, the case was raffled to Branch 147 of the Regional Trial
Court of Makati. The trial judge then set a hearing on June 8, 1999. At the hearing of the
application for preliminary injunction, petitioner was given a period of seven days to file its THE DETERMINATION OF THE INTEREST RATES BEING LEFT TO THE SOLE
DISCRETION OF THE DEFENDANT PNB CONTRAVENES THE PRINCIPAL OF MUTUALITY
written opposition to the application. On June 15, 1999, petitioner filed an opposition to the
application for a writ of preliminary injunction to which the respondents filed a reply. On June 25, OF CONTRACTS.
1999, petitioner filed a motion to dismiss on the grounds of failure to state a cause of action and
the absence of any privity between the petitioner and respondents. On June 30, 1999, the trial II
court judge issued an Order for the issuance of a writ of preliminary injunction, which writ was
correspondingly issued on July 14, 1999. On October 4, 1999, the motion to dismiss was denied
by the trial court judge for lack of merit. THERE BEING A STIPULATION IN THE LOAN AGREEMENT THAT THE RATE OF INTEREST
AGREED UPON MAY BE UNILATERALLY MODIFIED BY DEFENDANT, THERE WAS NO
STIPULATION THAT THE RATE OF INTEREST SHALL BE REDUCED IN THE EVENT THAT
Petitioner, thereafter, in a petition for certiorari and prohibition assailed the issuance of the writ of THE APPLICABLE MAXIMUM RATE OF INTEREST IS REDUCED BY LAW OR BY THE
preliminary injunction before the Court of Appeals. In the impugned decision, 1 the appellate court MONETARY BOARD.7
Based on the aforementioned grounds, respondents sought to enjoin and restrain PNB from the Company on the ground that Southern had acquired the entire capital stock of Lenoir Car Works,
foreclosure and eventual sale of the property in order to protect their rights to said property by hence, the latter corporation was but a mere instrumentality of the former. The Tennessee
reason of void credit facilities as bases for the real estate mortgage over the said property. 8 Supreme Court stated that as a general rule the stock ownership alone by one corporation of the
stock of another does not thereby render the dominant corporation liable for the torts of the
subsidiary unless the separate corporate existence of the subsidiary is a mere sham, or unless
The contract questioned is one entered into between respondent and PNB-IFL, not PNB. In their
the control of the subsidiary is such that it is but an instrumentality or adjunct of the dominant
complaint, respondents admit that petitioner is a mere attorney-in-fact for the PNB-IFL with full
corporation. Said Court then outlined the circumstances which may be useful in the
power and authority to, inter alia, foreclose on the properties mortgaged to secure their loan
determination of whether the subsidiary is but a mere instrumentality of the parent-corporation:
obligations with PNB-IFL. In other words, herein petitioner is an agent with limited authority and
specific duties under a special power of attorney incorporated in the real estate mortgage. It is
not privy to the loan contracts entered into by respondents and PNB-IFL. The Circumstance rendering the subsidiary an instrumentality. It is manifestly impossible to
catalogue the infinite variations of fact that can arise but there are certain common
circumstances which are important and which, if present in the proper combination, are
The issue of the validity of the loan contracts is a matter between PNB-IFL, the petitioner's
controlling.
principal and the party to the loan contracts, and the respondents. Yet, despite the recognition
that petitioner is a mere agent, the respondents in their complaint prayed that the petitioner PNB
be ordered to re-compute the rescheduling of the interest to be paid by them in accordance with These are as follows:
the terms and conditions in the documents evidencing the credit facilities, and crediting the
amount previously paid to PNB by herein respondents.9
(a) The parent corporation owns all or most of the capital stock of the subsidiary.

Clearly, petitioner not being a part to the contract has no power to re-compute the interest rates
(b) The parent and subsidiary corporations have common directors or officers.
set forth in the contract. Respondents, therefore, do not have any cause of action against
petitioner.
(c) The parent corporation finances the subsidiary.
The trial court, however, in its Order dated October 4, 1994, ruled that since PNB-IFL, is a wholly
owned subsidiary of defendant Philippine National Bank, the suit against the defendant PNB is a (d) The parent corporation subscribes to all the capital stock of the subsidiary or otherwise
suit against PNB-IFL.10 In justifying its ruling, the trial court, citing the case of Koppel Phil. Inc. causes its incorporation.
vs. Yatco,11 reasoned that the corporate entity may be disregarded where a corporation is the
mere alter ego, or business conduit of a person or where the corporation is so organized and
controlled and its affairs are so conducted, as to make it merely an instrumentality, agency, (e) The subsidiary has grossly inadequate capital.
conduit or adjunct of another corporation.12
(f) The parent corporation pays the salaries and other expenses or losses of the subsidiary.
We disagree.
(g) The subsidiary has substantially no business except with the parent corporation or no assets
The general rule is that as a legal entity, a corporation has a personality distinct and separate except those conveyed to or by the parent corporation.
from its individual stockholders or members, and is not affected by the personal rights,
obligations and transactions of the latter.13 The mere fact that a corporation owns all of the (h) In the papers of the parent corporation or in the statements of its officers, the subsidiary is
stocks of another corporation, taken alone is not sufficient to justify their being treated as one described as a department or division of the parent corporation, or its business or financial
entity. If used to perform legitimate functions, a subsidiary's separate existence may be responsibility is referred to as the parent corporation's own.
respected, and the liability of the parent corporation as well as the subsidiary will be confined to
those arising in their respective business. The courts may in the exercise of judicial discretion
step in to prevent the abuses of separate entity privilege and pierce the veil of corporate entity. (i) The parent corporation uses the property of the subsidiary as its own.

We find, however, that the ruling in Koppel finds no application in the case at bar. In said case, (j) The directors or executives of the subsidiary do not act independently in the interest of the
this Court disregarded the separate existence of the parent and the subsidiary on the ground subsidiary but take their orders from the parent corporation.
that the latter was formed merely for the purpose of evading the payment of higher taxes. In the
case at bar, respondents fail to show any cogent reason why the separate entities of the PNB (k) The formal legal requirements of the subsidiary are not observed.
and PNB-IFL should be disregarded.
The Tennessee Supreme Court thus ruled:
While there exists no definite test of general application in determining when a subsidiary may
be treated as a mere instrumentality of the parent corporation, some factors have been identified
that will justify the application of the treatment of the doctrine of the piercing of the corporate veil. In the case at bar only two of the eleven listed indicia occur, namely, the ownership of most of
The case of Garrett vs. Southern Railway Co.14 is enlightening. The case involved a suit against the capital stock of Lenoir by Southern, and possibly subscription to the capital stock of Lenoir. .
the Southern Railway Company. Plaintiff was employed by Lenoir Car Works and alleged that he . The complaint must be dismissed.
sustained injuries while working for Lenoir. He, however, filed a suit against Southern Railway
Similarly, in this jurisdiction, we have held that the doctrine of piercing the corporate veil is an SECTION 3. Grounds for issuance of preliminary injunction. — A preliminary injunction may be
equitable doctrine developed to address situations where the separate corporate personality of a granted when it is established:
corporation is abused or used for wrongful purposes. The doctrine applies when the corporate
fiction is used to defeat public convenience, justify wrong, protect fraud or defend crime, or when
(a) That the applicant is entitled to the relief demanded, and the whole or part of such relief
it is made as a shield to confuse the legitimate issues, or where a corporation is the mere alter
consists in restraining the commission or continuance of the act or acts complained of, or in
ego or business conduit of a person, or where the corporation is so organized and controlled and
requiring the performance of an act or acts, either for a limited period or perpetually,
its affairs are so conducted as to make it merely an instrumentality, agency, conduit or adjunct of
another corporation.15
(b) That the commission, continuance or non-performance of the acts or acts complained of
during the litigation would probably work injustice to the applicant; or
In Concept Builders, Inc. v. NLRC,16 we have laid the test in determining the applicability of the
doctrine of piercing the veil of corporate fiction, to wit:
(c) That a party, court, agency or a person is doing, threatening, or is attempting to do, or is
procuring or suffering to be done, some act or acts probably in violation of the rights of the
1. Control, not mere majority or complete control, but complete domination, not only of finances
applicant respecting the subject of the action or proceeding, and tending to render the judgment
but of policy and business practice in respect to the transaction attacked so that the corporate
ineffectual.
entity as to this transaction had at the time no separate mind, will or existence of its own.

Thus, an injunctive remedy may only be resorted to when there is a pressing necessity to avoid
2. Such control must have been used by the defendant to commit fraud or wrong, to perpetuate
injurious consequences which cannot be remedied under any standard
the violation of a statutory or other positive legal duty, or dishonest and, unjust act in
compensation.21 Respondents do not deny their indebtedness. Their properties are by their own
contravention of plaintiffs legal rights; and,
choice encumbered by real estate mortgages. Upon the non-payment of the loans, which were
secured by the mortgages sought to be foreclosed, the mortgaged properties are properly
3. The aforesaid control and breach of duty must proximately cause the injury or unjust loss subject to a foreclosure sale. Moreover, respondents questioned the alleged void stipulations in
complained of. the contract only when petitioner initiated the foreclosure proceedings. Clearly, respondents
have failed to prove that they have a right protected and that the acts against which the writ is to
be directed are violative of said right.22 The Court is not unmindful of the findings of both the trial
The absence of any one of these elements prevents "piercing the corporate veil." In applying the
court and the appellate court that there may be serious grounds to nullify the provisions of the
"instrumentality" or "alter ego" doctrine, the courts are concerned with reality and not form, with
loan agreement. However, as earlier discussed, respondents committed the mistake of filing the
how the corporation operated and the individual defendant's relationship to the operation. 17
case against the wrong party, thus, they must suffer the consequences of their error.

Aside from the fact that PNB-IFL is a wholly owned subsidiary of petitioner PNB, there is no
All told, respondents do not have a cause of action against the petitioner as the latter is not privy
showing of the indicative factors that the former corporation is a mere instrumentality of the latter
to the contract the provisions of which respondents seek to declare void. Accordingly, the case
are present. Neither is there a demonstration that any of the evils sought to be prevented by the
before the Regional Trial Court must be dismissed and the preliminary injunction issued in
doctrine of piercing the corporate veil exists. Inescapably, therefore, the doctrine of piercing the
connection therewith, must be lifted.
corporate veil based on the alter ego or instrumentality doctrine finds no application in the case
at bar.
IN VIEW OF THE FOREGOING, the petition is hereby GRANTED. The assailed decision of the
Court of Appeals is hereby REVERSED. The Orders dated June 30, 1999 and October 4, 1999
In any case, the parent-subsidiary relationship between PNB and PNB-IFL is not the significant
of the Regional Trial Court of Makati, Branch 147 in Civil Case No. 99-1037 are hereby
legal relationship involved in this case since the petitioner was not sued because it is the parent
ANNULLED and SET ASIDE and the complaint in said case DISMISSED.
company of PNB-IFL. Rather, the petitioner was sued because it acted as an attorney-in-fact of
PNB-IFL in initiating the foreclosure proceedings. A suit against an agent cannot without
compelling reasons be considered a suit against the principal. Under the Rules of Court, every SO ORDERED.
action must be prosecuted or defended in the name of the real party-in-interest, unless
otherwise authorized by law or these Rules.18 In mandatory terms, the Rules require that
"parties-in-interest without whom no final determination can be had, an action shall be joined
either as plaintiffs or defendants."19 In the case at bar, the injunction suit is directed only against
the agent, not the principal.

Anent the issuance of the preliminary injunction, the same must be lifted as it is a mere
provisional remedy but adjunct to the main suit.20 A writ of preliminary injunction is an ancillary or
preventive remedy that may only be resorted to by a litigant to protect or preserve his rights or
interests and for no other purpose during the pendency of the principal action. The dismissal of
the principal action thus results in the denial of the prayer for the issuance of the writ. Further,
there is no showing that respondents are entitled to the issuance of the writ. Section 3, Rule 58,
of the 1997 Rules of Civil Procedure provides:
G.R. No. 199687 March 24, 2014 When the Writ of Execution was returned unsatisfied, private respondents
moved for the issuance of an alias writ of execution to hold Export and
Industry Bank, Inc. liable for the judgment obligation as E- Securities is "a
PACIFIC REHOUSE CORPORATION, Petitioners,
wholly-owned controlled and dominated subsidiary of Export and Industry
vs.
Bank, Inc., and is[,] thus[,] a mere alter ego and business conduit of the
COURT OF APPEALS and EXPORT AND INDUSTRY BANK, INC., Respondents.
latter. E-Securities opposed the motion[,] arguing that it has a corporate
personality that is separate and distinct from petitioner. On July 27, 2011,
x-----------------------x private respondents filed their (1) Reply attaching for the first time a sworn
statement executed by Atty. Ramon F. Aviado, Jr., the former corporate
secretary of petitioner and E-Securities, to support their alter ego theory;
G.R. No. 201537
and (2) Ex-Parte Manifestation alleging service of copies of the Writ of
Execution and Motion for Alias Writ of Execution on petitioner.
PACIFIC REHOUSE CORPORATION, PACIFIC CONCORDE CORPORATION, MIZPAH
HOLDINGS, INC., FORUM HOLDINGS CORPORATION and EAST ASIA OIL COMPANY,
On July 29, 2011, the RTC concluded that E-Securities is a mere business
INC., Petitioners,
conduit or alter ego of petitioner, the dominant parent corporation, which
vs. justifies piercing of the veil of corporate fiction. The trial court brushed aside
EXPORT AND INDUSTRY BANK, INC., Respondent.
E-Securities’ claim of denial of due process on petitioner as "xxx case
records show that notices regarding these proceedings had been tendered
DECISION to the latter, which refused to even receive them. Clearly, [petitioner] had
been sufficiently put on notice and afforded the chance to give its side[,]
yet[,] it chose not to." Thus, the RTC disposed as follows:
REYES, J.:

WHEREFORE, xxx,
On the scales of justice precariously lie the right of a prevailing party to his victor's cup, no more,
no less; and the right of a separate entity from being dragged by the ball and chain of the
vanquished party. Let an Alias Writ of Execution be issued relative to the
above-entitled case and pursuant to the RESOLUTION
dated October 18, 2005 and to this Order directing
The facts of this case as garnered from the Decision1 dated April 26, 2012 of the Court of defendant EIB Securities, Inc., and/or Export and
Appeals (CA) in CA-G.R. SP No. 120979 are as follows: Industry Bank, Inc., to fully comply therewith.

We trace the roots of this case to a complaint instituted with the Makati City The Branch Sheriff of this Court is directed to cause the
Regional Trial Court (RTC), Branch 66, against EIB Securities Inc. (E- immediate implementation of the given alias writ in
Securities) for unauthorized sale of 32,180,000 DMCI shares of private accordance with the Order of Execution to be issued
respondents Pacific Rehouse Corporation, Pacific Concorde Corporation, anew by the Branch Clerk of Court.
Mizpah Holdings, Inc., Forum Holdings Corporation, and East Asia Oil
Company, Inc. In its October 18, 2005 Resolution, the RTC rendered
judgment on the pleadings. The fallo reads: SO ORDERED. x x x

WHEREFORE, premises considered, judgment is With this development, petitioner filed an Omnibus
hereby rendered directing the defendant [E- Motion (Ex Abundanti Cautela) questioning
Securities] to return the plaintiffs’ [private the alias writ because it was not impleaded as a party
respondents herein] 32,180,000 DMCI shares, as of to the case. The RTC denied the motion in its Order
judicial demand. dated August 26, 2011 and directed the garnishment of
P1,465,799,000.00, the total amount of the 32,180,000
DMCI shares at P45.55 per share, against petitioner
On the other hand, plaintiffs are directed to reimburse and/or E-Securities.2 x x x. (Citations omitted)
the defendant the amount of [P]10,942,200.00,
representing the buy back price of the 60,790,000 KPP
shares of stocks at [P]0.18 per share. The Regional Trial Court (RTC) ratiocinated that being one and the same entity in the eyes of
the law, the service of summons upon EIB Securities, Inc. (E-Securities) has bestowed
jurisdiction over both the parent and wholly-owned subsidiary.3 The RTC cited the cases of Sps.
SO ORDERED. x x x Violago v. BA Finance Corp. et al.4 and Arcilla v. Court of Appeals5where the doctrine of piercing
the veil of corporate fiction was applied notwithstanding that the affected corporation was not
The Resolution was ultimately affirmed by the Supreme Court and attained brought to the court as a party. Thus, the RTC held in its Order6 dated August 26, 2011:
finality.
WHEREFORE, premises considered, the Motion for Reconsideration with Let the HEARING be set on September 27, 2011 at 2:00 in the afternoon at
Motion to Inhibit filed by defendant EIB Securities, Inc. is denied for lack of the Paras Hall, Main Building, Court of Appeals, to determine the necessity
merit. The Omnibus Motion Ex Abundanti C[au]tela is likewise denied for of issuing a writ of preliminary injunction. The Division Clerk of Court is
lack of merit. DIRECTED to notify the parties and their counsel with dispatch.

Pursuant to Rule 39, Section 10 (a) of the Rules of Court, the Branch Clerk xxxx
of Court or the Branch Sheriff of this Court is hereby directed to acquire
32,180,000 DMCI shares of stock from the Philippine Stock Exchange at the
SO ORDERED.11
cost of EIB Securities, Inc. and Export and Industry Bank[,] Inc. and to
deliver the same to the plaintiffs pursuant to this Court’s Resolution dated
October 18, 2005. Pacific Rehouse Corporation (Pacific Rehouse), Pacific Concorde Corporation, Mizpah Holdings,
Inc., Forum Holdings Corporation and East Asia Oil Company, Inc. (petitioners) filed their
Comment12 to Export Bank’s petition and proffered that the cases mentioned by Export Bank are
To implement this Order, let GARNISHMENT issue against ALL THOSE
inapplicable owing to their clearly different factual antecedents. The petitioners alleged that
HOLDING MONEYS, PROPERTIES OF ANY AND ALL KINDS, REAL OR
unlike the other cases, there are circumstances peculiar only to E-Securities and Export Bank
PERSONAL BELONGING TO OR OWNED BY DEFENDANT EIB
such as: 499,995 out of 500,000 outstanding shares of stocks of E-Securities are owned by
SECURITIES, INC. AND/OR EXPORT AND INDUSTRY BANK[,] INC., [sic]
Export Bank;13 Export Bank had actual knowledge of the subject matter of litigation as the
in such amount as may be sufficient to acquire 32,180,000 DMCI shares of
lawyers who represented E-Securities are also lawyers of Export Bank.14 As an alter ego, there
stock to the Philippine Stock Exchange, based on the closing price of
is no need for a finding of fraud or illegality before the doctrine of piercing the veil of corporate
Php45.55 per share of DMCI shares as of August 1, 2011, the date of the
fiction can be applied.15
issuance of the Alias Writ of Execution, or the total amount of
PhP1,465,799,000.00.
After oral arguments before the CA, the parties were directed to file their respective
memoranda.16
SO ORDERED.7

On October 25, 2011, the CA issued a Resolution,17 granting Export Bank’s application for the
CA-G.R. SP No. 120979
issuance of a writ of preliminary injunction, viz:

Export and Industry Bank, Inc. (Export Bank) filed before the CA a petition for certiorari with
WHEREFORE, finding [Export Bank’s] application for the ancillary injunctive
prayer for the issuance of a temporary restraining order (TRO)8 seeking the nullification of the
relief to be meritorious, and it further appearing that there is urgency and
RTC Order dated August 26, 2011 for having been made with grave abuse of discretion
necessity in restraining the same, a Writ of Preliminary Injunction is hereby
amounting to lack or excess of jurisdiction. In its petition, Export Bank made reference to several
GRANTED and ISSUED against the Sheriff of the Regional Trial Court of
rulings9 of the Court upholding the separate and distinct personality of a corporation.
Makati City, Branch 66, or his deputies, agents, representatives or any
person acting in their behalf from executing the July 29, 2011 and August
In a Resolution10 dated September 2, 2011, the CA issued a 60-day TRO enjoining the 26, 2011 Orders. Public respondents are ordered to CEASE and DESIST
execution of the Orders of the RTC dated July 29, 2011 and August 26, 2011, which granted the from enforcing and implementing the subject orders until further notice from
issuance of an alias writ of execution and ordered the garnishment of the properties of E- this Court.18
Securities and/or Export Bank. The CA also set a hearing to determine the necessity of issuing a
writ of injunction, viz:
The petitioners filed a Manifestation19 and Supplemental Manifestation20 challenging the above-
quoted CA resolution for lack of concurrence of Associate Justice Socorro B. Inting (Justice
Considering the amount ordered to be garnished from petitioner Export and Inting), who was then on official leave.
Industry Bank, Inc. and the fiduciary duty of the banking institution to the
public, there is grave and irreparable injury that may be caused to [Export
On December 22, 2011, the CA, through a Special Division of Five, issued another
Bank] if the assailed Orders are immediately implemented. We thus resolve
Resolution,21 which reiterated the Resolution dated October 25, 2011 granting the issuance of a
to GRANT the Temporary Restraining Order effective for a period of sixty
writ of preliminary injunction.
(60) days from notice, restraining/enjoining the Sheriff of the Regional Trial
Court of Makati City or his deputies, agents, representatives or any person
acting in their behalf from executing the July 29, 2011 and August 26, 2011 On January 2, 2012, one of the petitioners herein, Pacific Rehouse filed before the Court a
Orders. [Export Bank] is DIRECTED to POST a bond in the sum of fifty petition for certiorari22under Rule 65, docketed as G.R. No. 199687, demonstrating its objection
million pesos (P50,000,000.00) within ten (10) days from notice, to answer to the Resolutions dated October 25, 2011 and December 22, 2011 of the CA.
for any damage which private respondents may suffer by reason of this
Temporary Restraining Order; otherwise, the same shall automatically
become ineffective. On April 26, 2012, the CA rendered the assailed Decision23 on the merits of the case, granting
Export Bank’s petition. The CA disposed of the case in this wise:
We GRANT the petition. The Orders dated July 29, 2011 and August 26, G.R. No. 199687
2011 of the Makati City Regional Trial Court, Branch 66, insofar as [Export
Bank] is concerned, are NULLIFIED. The Writ of Preliminary Injunction
The Resolution dated October 25, 2011 was initially challenged by the petitioners in its
(WPI) is rendered PERMANENT.
Manifestation29 and Supplemental Manifestation30 due to the lack of concurrence of Justice
Inting, which according to the petitioners rendered the aforesaid resolution null and void.
SO ORDERED.24
To the petitioners’ mind, Section 5, Rule VI of the Internal Rules of the CA (IRCA)31 requires the
The CA explained that the alter ego theory cannot be sustained because ownership of a submission of the resolution granting an application for TRO or preliminary injunction to the
subsidiary by the parent company is not enough justification to pierce the veil of corporate absent Justice/s when they report back to work for ratification, modification or recall, such that
fiction. There must be proof, apart from mere ownership, that Export Bank exploited or misused when the absent Justice/s do not agree with the issuance of the TRO or preliminary injunction,
the corporate fiction of E-Securities. The existence of interlocking incorporators, directors and the resolution is recalled and without force and effect. 32 Since the resolution which granted the
officers between the two corporations is not a conclusive indication that they are one and the application for preliminary injunction appears short of the required number of consensus, owing
same.25 The records also do not show that Export Bank has complete control over the business to the absence of Justice Inting’s signature, the petitioners contest the validity of said resolution.
policies, affairs and/or transactions of E-Securities. It was solely E-Securities that contracted the
obligation in furtherance of its legitimate corporate purpose; thus, any fall out must be confined
The petitioners also impugn the CA Resolution dated December 22, 2011 rendered by the
within its limited liability.26
Special Division of Five. The petitioners maintain that pursuant to Batas Pambansa Bilang
12933 and the IRCA,34 such division is created only when the three members of a division cannot
The petitioners, without filing a motion for reconsideration, filed a Petition for Review27 under reach a unanimous vote in deciding a case on the merits. 35Furthermore, for petitioner Pacific
Rule 45 docketed as G.R. No. 201537,28 impugning the Decision dated April 26, 2012 of the CA. Rehouse, this Resolution is likewise infirm because the purpose of the formation of the Special
Division of Five is to decide the case on the merits and not to grant Export Bank’s application for
a writ of preliminary injunction.36
Considering that G.R. Nos. 199687 and 201537 originated from the same set of facts, involved
the same parties and raised intertwined issues, the cases were then consolidated per Resolution
dated September 26, 2012, for a thorough discussion of the merits of the case. We hold that the opposition to the CA resolutions is already nugatory because the CA has
already rendered its Decision on April 16, 2012, which disposed of the substantial merits of the
case. Consequently, the petitioners’ concern that the Special Division of Five should have been
Issues
created to resolve cases on the merits has already been addressed by the rendition of the CA
Decision dated April 16, 2012.
In précis, the issues for resolution of this Court are the following:
"It is well-settled that courts will not determine questions that have become moot and academic
In G.R. No. 199687, because there is no longer any justiciable controversy to speak of. The judgment will not serve
any useful purpose or have any practical legal effect because, in the nature of things, it cannot
be enforced."37 In such cases, there is no actual substantial relief to which the petitioners would
WHETHER THE CA COMMITTED GRAVE ABUSE OF DISCRETION IN be entitled to and which would be negated by the dismissal of the petition. 38Thus, it would be
GRANTING EXPORT BANK’S APPLICATION FOR THE ISSUANCE OF A futile and pointless to address the issue in G.R. No. 199687 as this has become moot and
WRIT OF PRELIMINARY INJUNCTION. academic.

In G.R. No. 201537, G.R. No. 201537

I. The petitioners bewail that the certified true copy of the CA Decision dated April 26, 2012 along
with its Certification at the bottom portion were not signed by the Chairperson 39 of the Special
WHETHER THE CA COMMITTED A REVERSIBLE ERROR IN RULING Division of Five; thus, it is not binding upon the parties.40 The petitioners quoted this Court’s
THAT EXPORT BANK MAY NOT BE HELD LIABLE FOR A FINAL AND pronouncement in Limkaichong v. Commission on Elections,41that a decision must not only be
EXECUTORY JUDGMENT AGAINST E-SECURITIES IN AN ALIAS WRIT signed by the Justices who took part in the deliberation, but must also be promulgated to be
OF EXECUTION BY PIERCING ITS VEIL OF CORPORATE FICTION; and considered a Decision.42

II. A cursory glance on a copy of the signature page43 of the decision attached to the records would
show that, indeed, the same was not signed by CA Associate Justice Magdangal M. De Leon.
However, it must be noted that the CA, on May 7, 2012, issued a Resolution44 explaining that
WHETHER THE CA COMMITTED A REVERSIBLE ERROR IN RULING due to inadvertence, copies of the decision not bearing the signature of the Chairperson were
THAT THE ALTER EGO DOCTRINE IS NOT APPLICABLE. sent to the parties on the same day of promulgation. The CA directed the Division Clerk of Court
to furnish the parties with copies of the signature page with the Chairperson’s signature.
Ruling of the Court Consequently, as the mistake was immediately clarified and remedied by the CA, the lack of the
Chairperson’s signature on the copies sent to the parties has already become a non-issue.
It must be emphasized that the instant cases sprang from Pacific Rehouse Corporation v. EIB commencing from its filing of an Omnibus Motion53 by way of special appearance during the
Securities, Inc.45which was decided by this Court last October 13, 2010. Significantly, Export execution stage until the filing of its Comment54 before the Court wherein it was pleaded that
Bank was not impleaded in said case but was unexpectedly included during the execution stage, "RTC [of] Makati[, Branch] 66 never acquired jurisdiction over Export [B]ank. Export [B]ank was
in addition to E-Securities, against whom the writ of execution may be enforced in the not pleaded as a party in this case. It was never served with summons by nor did it voluntarily
Order46 dated July 29, 2011 of the RTC. In including Export Bank, the RTC considered E- appear before RTC [of] Makati[, Branch] 66 so as to be subjected to the latter’s jurisdiction." 55
Securities as a mere business conduit of Export Bank. 47 Thus, one of the arguments interposed
by the latter in its Opposition48 that it was never impleaded as a defendant was simply set aside.
In dispensing with the requirement of service of summons or voluntary appearance of Export
Bank, the RTC applied the cases of Violago and Arcilla. The RTC concluded that in these cases,
This action by the RTC begs the question: may the RTC enforce the alias writ of execution the Court decided that the doctrine of piercing the veil of corporate personality can be applied
against Export Bank? even when one of the affected parties has not been brought to the Court as a party. 56

The question posed before us is not novel. A closer perusal on the rulings of this Court in Violago and Arcilla, however, reveals that the
RTC misinterpreted the doctrines on these cases. We agree with the CA that these cases are
not congruent to the case at bar. In Violago, Spouses Pedro and Florencia Violago (Spouses
The Court already ruled in Kukan International Corporation v. Reyes49 that compliance with the
Violago) filed a third party complaint against their cousin Avelino Violago (Avelino), who is also
recognized modes of acquisition of jurisdiction cannot be dispensed with even in piercing the veil
the president of Violago Motor Sales Corporation (VMSC), for selling them a vehicle which was
of corporate fiction, to wit:
already sold to someone else. VMSC was not impleaded as a third party defendant. Avelino
contended that he was not a party to the transaction personally, but VMSC. The Court ruled that
The principle of piercing the veil of corporate fiction, and the resulting treatment of two related "[t]he fact that VMSC was not included as defendant in [Spouses Violago’s] third party complaint
corporations as one and the same juridical person with respect to a given transaction, is does not preclude recovery by Spouses Violago from Avelino; neither would such non-inclusion
basically applied only to determine established liability; it is not available to confer on the court a constitute a bar to the application of the piercing-of-the-corporate-veil doctrine."57 It should be
jurisdiction it has not acquired, in the first place, over a party not impleaded in a case. Elsewise pointed out that although VMSC was not made a third party defendant, the person who was
put, a corporation not impleaded in a suit cannot be subject to the court’s process of found liable in Violago, Avelino, was properly made a third party defendant in the first instance.
piercing the veil of its corporate fiction. In that situation, the court has not acquired The present case could not be any more poles apart from Violago, because Export Bank, the
jurisdiction over the corporation and, hence, any proceedings taken against that corporation and parent company which was sought to be accountable for the judgment against E-Securities, is
its property would infringe on its right to due process. Aguedo Agbayani, a recognized authority not a party to the main case.
on Commercial Law, stated as much:
In Arcilla, meanwhile, Calvin Arcilla (Arcilla) obtained a loan in the name of Csar Marine
"23. Piercing the veil of corporate entity applies to determination of liability not of jurisdiction. x x Resources, Inc. (CMRI) from Emilio Rodulfo. A complaint was then filed against Arcilla for non-
x payment of the loan. CMRI was not impleaded as a defendant. The trial court eventually ordered
Arcilla to pay the judgment creditor for such loan. Arcilla argued that he is not personally liable
for the adjudged award because the same constitutes a corporate liability which cannot even
This is so because the doctrine of piercing the veil of corporate fiction comes to play only
bind the corporation as the latter is not a party to the collection suit. The Court made the
during the trial of the case after the court has already acquired jurisdiction over the
succeeding observations:
corporation. Hence, before this doctrine can be applied, based on the evidence presented, it is
imperative that the court must first have jurisdiction over the corporation. x x x" 50 (Citations
omitted) [B]y no stretch of even the most fertile imagination may one be able to conclude that the
challenged Amended Decision directed Csar Marine Resources, Inc. to pay the amounts
adjudged. By its clear and unequivocal language, it is the petitioner who was declared liable
From the preceding, it is therefore correct to say that the court must first and foremost acquire therefor and consequently made to pay. x x x, even if We are to assume arguendo that the
jurisdiction over the parties; and only then would the parties be allowed to present evidence for
obligation was incurred in the name of the corporation, the petitioner would still be personally
and/or against piercing the veil of corporate fiction. If the court has no jurisdiction over the liable therefor because for all legal intents and purposes, he and the corporation are one and the
corporation, it follows that the court has no business in piercing its veil of corporate fiction same. Csar Marine Resources, Inc. is nothing more than his business conduit and alter ego. The
because such action offends the corporation’s right to due process.
fiction of a separate juridical personality conferred upon such corporation by law should be
disregarded. x x x.58 (Citation omitted)
"Jurisdiction over the defendant is acquired either upon a valid service of summons or the
defendant’s voluntary appearance in court. When the defendant does not voluntarily submit to
It is important to bear in mind that although CMRI was not a party to the suit, it was Arcilla, the
the court’s jurisdiction or when there is no valid service of summons, ‘any judgment of the court defendant himself who was found ultimately liable for the judgment award. CMRI and its
which has no jurisdiction over the person of the defendant is null and void.’" 51 "The defendant properties were left untouched from the main case, not only because of the application of the
must be properly apprised of a pending action against him and assured of the opportunity to
alter ego doctrine, but also because it was never made a party to that case.
present his defenses to the suit. Proper service of summons is used to protect one’s right to due
process."52
The disparity between the instant case and those of Violago and Arcilla is that in said cases,
although the corporations were not impleaded as defendant, the persons made liable in the end
As Export Bank was neither served with summons, nor has it voluntarily appeared before the were already parties thereto since the inception of the main case. Consequently, it cannot be
court, the judgment sought to be enforced against E-Securities cannot be made against its said that the Court had, in the absence of fraud and/or bad faith, applied the doctrine of piercing
parent company, Export Bank. Export Bank has consistently disputed the RTC jurisdiction,
the veil of corporate fiction to make a non-party liable. In short, liabilities attached only to those how the corporation operated and the individual defendant’s relationship to that
who are parties. None of the non-party corporations (VMSC and CMRI) were made liable for the operation.64 Hence, all three elements should concur for the alter ego doctrine to be applicable.
judgment award against Avelino and Arcilla.
In its decision, the RTC maintained that the subsequently enumerated factors betray the true
The Alter Ego Doctrine is not applicable nature of E-Securities as a mere alter ego of Export Bank:

"The question of whether one corporation is merely an alter ego of another is purely one of fact. 1. Defendant EIB Securities, a subsidiary corporation 100% totally owned by Export and Industry
So is the question of whether a corporation is a paper company, a sham or subterfuge or Bank, Inc., was only re-activated by the latter in 2002-2003 and the continuance of its operations
whether petitioner adduced the requisite quantum of evidence warranting the piercing of the veil was geared for no other reason tha[n] to serve as the securities brokerage arm of said
of respondent’s corporate entity."59 parent corporation bank;

As a rule, the parties may raise only questions of law under Rule 45, because the Supreme 2. It was the parent corporation bank that provided and infused the fresh working cash capital
Court is not a trier of facts. Generally, we are not duty-bound to analyze again and weigh the needed by defendant EIB Securities which prior thereto was non-operating and severely cash-
evidence introduced in and considered by the tribunals below. 60 However, justice for all is of strapped. [This was so attested by the then Corporate Secretary of both corporations, Atty.
primordial importance that the Court will not think twice of reviewing the facts, more so because Ramon Aviado, Jr., in his submitted Sworn Statement which is deemed allowable "evidence on
the RTC and the CA arrived in contradicting conclusions. motion", under Sec. 7, Rule 133, Rules on Evidence; Bravo vs. Borja, 134 SCRA 438];

"It is a fundamental principle of corporation law that a corporation is an entity separate and 3. For effective control purposes, defendant EIB Securities and its operating office and staff are
distinct from its stockholders and from other corporations to which it may be connected. But, this all housed in Exportbank Plaza located at Chino Roces cor. Sen. Gil Puyat Avenue, Makati City
separate and distinct personality of a corporation is merely a fiction created by law for which is the same building w[h]ere the bank parent corporation has its headquarters;
convenience and to promote justice. So, when the notion of separate juridical personality is used
to defeat public convenience, justify wrong, protect fraud or defend crime, or is used as a device
4. As shown in the General Information Sheets annually filed with the S.E.C. from 2002 to 2011,
to defeat the labor laws, this separate personality of the corporation may be disregarded or the
both defendant EIB Securities and the bank parent corporation share common key Directors and
veil of corporate fiction pierced. This is true likewise when the corporation is merely an adjunct, a
corporate officers. Three of the 5-man Board of Directors of defendant EIB Securities are
business conduit or an alter ego of another corporation."61
Directors of the bank parent corporation, namely: Jaime C. Gonzales, Pauline C. Tan and
Dionisio E. Carpio, Jr. In addition, Mr. Gonzales is Chairman of the Board of both corporations,
"Where one corporation is so organized and controlled and its affairs are conducted so that it is, whereas Pauline C. Tan is concurrently President/General Manager of EIB Securities, and
in fact, a mere instrumentality or adjunct of the other, the fiction of the corporate entity of the Dionisio Carpio Jr., is not only director of the bank, but also Director Treasurer of defendant EIB
"instrumentality" may be disregarded. The control necessary to invoke the rule is not majority or Securities;
even complete stock control but such domination of finances, policies and practices that the
controlled corporation has, so to speak, no separate mind, will or existence of its own, and is but
5. As admitted by the bank parent corporation in its consolidated audited financial statements[,]
a conduit for its principal. It must be kept in mind that the control must be shown to have been
EIB Securities is a CONTROLLED SUBSIDIARY, and for which reason its financial condition
exercised at the time the acts complained of took place. Moreover, the control and breach of
and results of operations are included and integrated as part of the group’s consolidated
duty must proximately cause the injury or unjust loss for which the complaint is made."62
financial statements, examined and audited by the same auditing firm;

The Court has laid down a three-pronged control test to establish when the alter ego doctrine
6. The lawyers handling the suits and legal matters of defendant EIB Securities are the same
should be operative:
lawyers in the Legal Department of the bank parent corporation.1âwphi1 The Court notes that in
[the] above-entitled suit, the lawyers who at the start represented said defendant EIB Securities
(1) Control, not mere majority or complete stock control, but complete domination, not only of and filed all the pleadings and filings in its behalf are also the lawyers in the Legal Services
finances but of policy and business practice in respect to the transaction attacked so that the Division of the bank parent corporation. They are Attys. Emmanuel A. Silva, Leonardo C. Bool,
corporate entity as to this transaction had at the time no separate mind, will or existence of its Riva Khristine E. Maala and Ma. Esmeralda R. Cunanan, all of whom worked at the Legal
own; Services Division of Export Industry Bank located at 36/F, Exportbank Plaza, Don Chino Roces
Avenue, cor. Sen. Gil Puyat Avenue, Makati City.
(2) Such control must have been used by the defendant to commit fraud or wrong, to perpetuate
the violation of a statutory or other positive legal duty, or dishonest and unjust act in 7. Finally[,] and this is very significant, the control and sway that the bank parent corporation
contravention of plaintiff’s legal right; and held over defendant EIB Securities was prevailing in June 2004 when the very act complained of
in plaintiff’s Complaint took place, namely the unauthorized disposal of the 32,180,000 DMCI
shares of stock. Being then under the direction and control of the bank parent corporation, the
(3) The aforesaid control and breach of duty must [have] proximately caused the injury or unjust
unauthorized disposal of those shares by defendant EIB Securities is attributable to, and the
loss complained of.63
responsibility of the former.65

The absence of any one of these elements prevents ‘piercing the corporate veil’ in applying the
All the foregoing circumstances, with the exception of the admitted stock ownership, were
‘instrumentality’ or ‘alter ego’ doctrine, the courts are concerned with reality and not form, with
however not properly pleaded and proved in accordance with the Rules of Court. 66 These were
merely raised by the petitioners for the first time in their Motion for Issuance of an Alias Writ of In closing, we understand that the petitioners are disgruntled at the turnout of this case-that they
Execution67 and Reply,68 which the Court cannot consider. "Whether the separate personality of cannot enforce the award due them on its entirety; however, the Court cannot supplant a remedy
the corporation should be pierced hinges on obtaining facts appropriately pleaded or proved."69 which is not sanctioned by our laws and prescribed rules.

Albeit the RTC bore emphasis on the alleged control exercised by Export Bank upon its WHEREFORE, the petition in G.R. No. 199687 is hereby DISMISSED for having been rendered
subsidiary E-Securities, "[c]ontrol, by itself, does not mean that the controlled corporation is a moot and academic. The petition in G.R. No. 201537, meanwhile, is hereby DENIED for lack of
mere instrumentality or a business conduit of the mother company. Even control over the merit. Consequently, the Decision dated April 26, 2012 of the Court of Appeals in CA-G.R. SP
financial and operational concerns of a subsidiary company does not by itself call for No. 120979 is AFFIRMED.
disregarding its corporate fiction. There must be a perpetuation of fraud behind the control or at
least a fraudulent or illegal purpose behind the control in order to justify piercing the veil of
SO ORDERED.
corporate fiction. Such fraudulent intent is lacking in this case."70

Moreover, there was nothing on record demonstrative of Export Bank’s wrongful intent in setting
up a subsidiary, E-Securities. If used to perform legitimate functions, a subsidiary’s separate
existence shall be respected, and the liability of the parent corporation as well as the subsidiary
will be confined to those arising in their respective business. 71 To justify treating the sole
stockholder or holding company as responsible, it is not enough that the subsidiary is so
organized and controlled as to make it "merely an instrumentality, conduit or adjunct" of its
stockholders. It must further appear that to recognize their separate entities would aid in the
consummation of a wrong.72

As established in the main case73 and reiterated by the CA, the subject 32,180,000 DMCI shares
which E-Securities is obliged to return to the petitioners were originally bought at an average
price of P0.38 per share and were sold for an average price of P0.24 per share. The proceeds
were then used to buy back 61,100,000 KPP shares earlier sold by E-Securities. Quite
unexpectedly however, the total amount of these DMCI shares ballooned to
P1,465,799,000.00.74 It must be taken into account that this unexpected turnabout did not inure
to the benefit of E-Securities, much less Export Bank.

Furthermore, ownership by Export Bank of a great majority or all of stocks of E-Securities and
the existence of interlocking directorates may serve as badges of control, but ownership of
another corporation, per se, without proof of actuality of the other conditions are insufficient to
establish an alter ego relationship or connection between the two corporations, which will justify
the setting aside of the cover of corporate fiction. The Court has declared that "mere ownership
by a single stockholder or by another corporation of all or nearly all of the capital stock of a
corporation is not of itself sufficient ground for disregarding the separate corporate personality."
The Court has likewise ruled that the "existence of interlocking directors, corporate officers and
shareholders is not enough justification to pierce the veil of corporate fiction in the absence of
fraud or other public policy considerations."75

While the courts have been granted the colossal authority to wield the sword which pierces
through the veil of corporate fiction, concomitant to the exercise of this power, is the
responsibility to uphold the doctrine of separate entity, when rightly so; as it has for so long
encouraged businessmen to enter into economic endeavors fraught with risks and where only a
few dared to venture.

Hence, any application of the doctrine of piercing the corporate veil should be done with caution.
A court should be mindful of the milieu where it is to be applied. It must be certain that the
corporate fiction was misused to such an extent that injustice, fraud, or crime was committed
against another, in disregard of its rights. The wrongdoing must be clearly and convincingly
established; it cannot be presumed. Otherwise, an injustice that was never unintended may
result from an erroneous application.76
G.R. Nos. 111810-11 June 16, 1995 reiterating the grounds they relied upon in their opposition to private respondents' motion to
implead. A reply was filed by private respondents, and a rejoinder was then filed by petitioners.
In turn, private respondents filed a sub-rejoinder.
JAMES YU and WILSON YOUNG, petitioners,
vs.
THE NATIONAL LABOR RELATIONS COMMISSION, LABOR ARBITER DANIEL C. CUETO, Subsequently, an amended complaint was filed by private respondents against TDI and
TANDUAY DISTILLERY INC., FERNANDO DURAN, EDUARDO PALIWAN, ROQUE ESTOCE petitioners Yu and Young "doing business under the name and style of Tanduay Distillers".
AND RODRIGO SANTOS, respondents.
In her decision dated May 24, 1989, Labor Arbiter Daisy Cauton-Barcelona held:
MELO, J.:
In treating the motion to implead a motion to admit amended complaint with leave, the same [is]
Before us is a petition for certiorari assailing the decision of public respondent National Labor hereby given due course and all pleadings filed by respondents James Yu and Wilson Young
Relations Commission (NLRC) promulgated on August 25, 1993 in the cases of Fernando are hereby treated as their responsive pleadings in the light of speedy disposition of justice and
Duran, et al. vs. Tanduay Distillery, Inc., docketed as NLRC NCR Case No. 00-04-01737-88, the basic rule that administrative fora, such as this office, are not governed by technical rules of
and Rodrigo Santos vs. Tanduay Distillery, Inc., docketed as NLRC NCR Case No. 00-06- proceedings.
02546-88.
(p. 52, Rollo).
The relevant antecedent facts as gathered from the record are as follows:
In the same decision, it was disposed:
Private respondents-employees Fernando Duran, Eduardo Paliwan, Roque Estoce, and Rodrigo
Santos were employees of respondent corporation Tanduay Distillery, Inc, (TDI).
WHEREFORE, judgment is hereby rendered and declaring that the retrenchment is illegal
thereby ordering respondent Tanduay Distillery, Inc., to reinstate the complainants to their
On March 29, 1988, 22 employees of TDI, including private respondents employees, received a former position with backwages up to the time of change of ownership, if one has taken place.
memorandum from TDI terminating their services. for reasons of retrenchment, effective 30 days
from receipt thereof or not later than the close of business hours on April 28, 1988.
That in the event of change in management it (Tanduay Distillery, Inc.,) is hereby ordered to pay
the complainants their respective separation benefits computed at the rate of one (1) month for
On April 26, 1988, all 22 employees of TDI filed an application for the issuance of a temporary every year of service. This is without prejudice to the letter of Mr. James Yu as officer-in-charge
restraining order against their retrenchment. The labor arbiter issued the restraining order the of Tanduay Distillers dated June 17, 1988 to the President of the Tanduay Distillery, Inc., Labor
following day. However, due to the 20-day lifetime of the temporary restraining order, and Union.
because of the on-going negotiations for the sale of TDI the retrenchment pushed through.
Parenthetically, it should be mentioned that although all 22 employees were retrenched, the
(pp. 57-58, Rollo.)
instant petition involves only the 4 individual respondents herein, namely, Fernando Duran,
Eduardo Paliwan, Roque Estoce, and Rodrigo Santos.
Only TDI appealed said decision to the National Labor Relations Commission, but on June 18,
1991, said commission promulgated an affirmance decision (p. 102, Rollo). TDI filed a motion for
On June 14, 1988, the First Pacific Metro Corporation moved that it be dropped as a party to the
reconsideration, but the same was denied on August 15, 1991.
case on the ground that its projected purchase of the assets of TDI was not consummated. The
participation of First Pacific was later in effect held to be irrelevant (decision dated May 24, 1989;
Annex G, pp. 50-58, Rollo). On June 1, 1988, or after respondents-employees had ceased as Thereupon, private respondents-employees on September 16, 1991 filed a motion for execution
such employees, a new buyer of TDI's assets, Twin Ace Holdings, Inc. took over the business. (Annex Q, pp. 103-106, Rollo) praying that NLRC through the labor arbiter, "[i]ssue the
Twin Ace assumed the business name Tanduay Distillers. necessary writ for the execution of the entire decision dated May 24, 1989, including the actual
reinstatement of the complainants to their former position without loss of seniority and benefits
against Tanduay Distillery, Inc., and/or Tanduay Distillers, James Yu and Wilson Young."
On August 8, 1988, the employees filed a motion to implead herein petitioners James Yu and
Wilson Young, doing business under the name and style of Tanduay Distillers, as party
respondents in said cases. Petitioners filed an opposition thereto, asserting that they are On September 24, 1993, petitioners filed an opposition (Annex R, pp. 108-110, Rollo) to the
representatives of Tanduay Distillers an entity distinct and separate from TDI, the previous motion for execution on the ground that "the Motion for Execution is without any basis in so far
owner, and that there is no employer-employee relationship between Tanduay Distillers and as it prays for the issuance of a writ of execution against respondent Tanduay Distillers, which is
private respondents. Respondents-employees filed a reply to the opposition stating that an entity separate and distinct from respondent Tanduay Distillery, Inc., and respondents James
petitioner James Yu as officer-in-charge of Tanduay Distillers had informed the president of TDI Yu and Wilson Young." Respondents-employees filed their reply thereto (Annex S, pp. 111-
labor union of Tanduay Distillers' decision to hire everybody with a clean slate on a probation 115, Rollo), and in turn petitioners filed their rejoinder (Annex T, pp. 116-118, Rollo), to which
basis. private respondents filed their sur-rejoinder (Annex U, pp. 119-122, Rollo). On December 18,
1991, respondent TDI filed its comment on the motion for execution (Annex V, pp. 124-
129, Rollo), while petitioners on January 10, 1992, filed a joint comment (Annex W, pp. 130-
On November 16, 1988, private respondents filed a motion for leave to file an amended
132, Rollo) to private respondents' sur-rejoinder as well to the comment filed by respondent TDI.
complaint impleading petitioners as respondents. Petitioners filed an opposition thereto
Subsequently, TDI filed a manifestation dated April 24, 1992 (Annex X, pp. 133-135, Rollo), Fernando Duran, Rodrigo Santos, Roque Estoce and Eduardo Daliwan to their respective
stating — positions.

2. At the hearing held on March 23, 1992, individual complainants, assisted by their counsel, (p 145, Rollo.)
Atty. Noel Cruz, agreed to be paid the total amount of P86,049.83, in full satisfaction of the
Company's liability as stated in the dispositive portion of Labor Arbiter Barcelona's decision
Consequently, a writ of execution was issued by Labor Arbiter Cueto on December 16, 1992.
promulgated on May 24, 1989 and affirmed by the Second Division of the NLRC on June 18,
1991, which reads as follows:
To stop the implementation of the writ of execution, petitioners filed a petition
for certiorari (Annex AA, pp. 146-158, Rollo before respondent NLRC, praying that —
WHEREFORE, judgment is hereby rendered declaring that the retrenchment is illegal thereby
ordering respondent Tanduay Distillery, Inc. to reinstate the complainants to their former position
with backwages up to the time of the change of ownership, if one has taken place. 1. Immediately upon filing of the instant case, a temporary restraining order he issued, to wit.

That in the event of change in management it (Tanduay Distillery, Inc.( is hereby ordered to pay a) Enjoining and restraining the respondents from implementing the Order dated November 17,
the complainants their respective separation benefits computed at the rate of one (1) month of 1992;
every year of service. This is without prejudice to the letter of Mr. James Yu as officer-in-charge
of Tanduay Distillers dated June 17, 1988 to the President of the Tanduay Distillery, Inc., Labor
Union. b.) Commanding the public respondent to desist from acting on the Order,

c.) Commanding the respondents to desist from committing any other act judicial to the
No Costs.
petitioners/appellants.

SO ORDERED.
2. After the appropriate proceedings, a writ of preliminary injunction be issued so enjoining the
respondents;
1. In accordance with the aforequoted decision, complainants shall be paid the
amounts appearing opposite their respective names: 3. After hearing on the merits, the Order dated November 17, 1992 be set aside and an
injunction be issued permanently enjoining the respondents from committing the aforesaid acts
and to comply strictly with terms of the Decision and the NLRC;
Rodrigo F. Santos P20,282.03
Roque Estoce 20,092.50
4. Ordering the respondents, jointly and severally, to pay petitioner's fees in the amount of
Eduardo Daliwan 19,973.40 P100,000.00 and to pay the cost of suit.
Fernando A. Duran 25,702.00
————— On August 25, 1993, respondent NLRC promulgated its decision, the dispositive portion of which
reads:
Total P86,049.83
In view of the foregoing premises, the petition/appeal with prayer for preliminary injunction is
=========
hereby dismissed for lack of merit.

The petitioners respondents are hereby directed to re re-employ/re-hire respondents-


4. The foregoing amounts shall be satisfied out of the cash bond deposited by the Company with complainants immediately upon receipt of this decision.
the Cashier of the NLRC. The excess amounting to P7,076.44 must revert to the Company.
(p. 36, Rollo.)
(pp. 134-135, Rollo.)
Thus, the present petition where petitioners pray that —
On November 17, 1992, respondent NLRC, through Labor Arbiter Daniel C. Cueto, issued an
order (Annex Z, pp. 139-145, Rollo), resolving the motion for execution as follows:
1. Immediately upon filing of the instant case, a temporary restraining order be issued, to wit:

Based on the foregoing considerations, this Branch finds the Motion for Writ of Execution filed by
the complainants meritorious and in order. Accordingly, let a Writ of Execution be issued against a) Restraining and prohibiting the respondents form implementing the ORDER dated November
17, 1992 and the NLRC Certiorari Decision.
Tanduay Distiller, Inc., Wilson Young and James Yu to immediately reinstate complainants
b) Commanding the respondents to desist from committing any other act prejudicial to the However, Labor Arbiter Cueto went beyond what was disposed by the decision and issued an
petitioners. order dated November 17, 1992 (Annex Z, Petition, pp. 139-145, Rollo) which required

2. After the appropriate proceedings, a writ of preliminary injunction be issued so enjoining the . . . Tanduay Distillers, Inc., Wilson Young and James Yu to immediately reinstate complainants
respondents; Fernando Duran, Rodrigo Santos, Roque Estoce and Eduardo Daliwan to heir respective
positions.
3. After appropriate proceedings, the ORDER dated November 17, 1992 and the
NLRC CertiorariDecision be set aside and a injunction be issued permanently enjoining the (p. 145, Rollo.)
respondents from committing the aforesaid acts and to comply strictly with the terms of the
Arbiter Decision and the NLRC Decision;
Subsequently, a writ of execution was issued on December 16, 1992 pursuant to the order of
November 17, 1992.
4. Ordering the respondents, jointly and severally, to pay petitioners' attorney's fees in the
amount of P100,000.00 and to pay the costs of suit.
The order of execution dated November 17, 1992 in effect amended the decision dated May 24,
1989 for the former orders petitioners and Tanduay Distillers to reinstate private respondents
(pp. 26-27, Rollo.) employees whereas the decision dated May 24, 1989, as we have discussed above, does not so
decree, This cannot be done. It is beyond the power and competence of Labor Arbiter Cueto to
amend a final decision, The writ of execution must not go beyond the scope of the judgment.
The issue posed by the present petition is whether respondent NLRC committed grave abuse of
discretion in holding petitioners Yu and Young liable under the decision dated May 24, 1989
which decreed that: As Chief Justice Moran opined: "The writ of execution must conform to the judgment which is to
be executed as it may not vary the terms of the judgment it seeks to enforce. Nor may it go
beyond the terms of the judgment, sought to be executed. Where the execution is not in
WHEREFORE, judgment is hereby rendered declaring that the retrenchment is illegal thereby
harmony with the judgment which gives it life and exceeds it, it has pro tanto no validity. To
ordering respondent Tanduay Distillery Inc., to reinstate the complainants to their former position
maintain otherwise would be to ignore the constitutional provision against depriving a person of
with backwages up to the time of the change ownership, if one has taken place.
his property without due process of law" (Moran, Comments on the Rules of Court, Vol. I 1952
Ed., p. 809; cited in Villoria vs. Piccio, supra).
That in the event of change in management it (Tanduay Distillery, Inc.) is hereby ordered to pay
the complainants their respective separation benefits corrupted at the rate of one (1) month for
(Gamboa's Incorporated vs. Court of
every year of service. This is without prejudice to the letter of Mr. James Yu as officer-in-charge
Appeals, 72 SCRA 131, 137-138 [1976])
of Tanduay Distillers dated June 17, 1988 to the President of the Tanduay Distillery, Inc., Labor
Union.
The order of execution and the writ of execution ordering petitioners and Tanduay Distillers to
reinstate private respondents employees are, therefore, null and void.
(pp. 57-58, Rollo.)

2. Neither may be said that petitioners and Tanduay Distillers are one and the same as TDI, as
We hold that petitioners, for a number of reasons which we shall discuss below, may not be held
seems to be the impression of respondents when they impleaded petitioners as party
answerable and liable under the final judgment of Labor Arbiter Cauton-Barcelona.
respondents in their compliant for unfair labor practice, illegal lay off, and separation benefits.

1. Admittedly, the decision dated May 24, 1989 is now final and executory, as only respondent
Such a stance is not supported by the facts. The name of the company for whom the petitioners
TDI appealed said decision and its appeal was later dismissed by respondent NLRC. It is
are working is Twin Ace Holdings Corporation, As stated by the Solicitor General, Twin Ace is
fundamental that a final and executory decision cannot be amended or corrected (First
part of the Allied Bank Group although it conducts the rum business under the name of Tanduay
Integrated Bonding and Insurance Company, Inc, vs. Hernando, 199 SCRA 796 [1991]) except
Distillers. The use of a similar sounding or almost identical name is an obvious device to
for clerical errors or mistakes (Maramba vs. Lozano, 20 SCRA 474 [1967]); Reyes vs. Court of
capitalize on the goodwill which Tanduay Rum has built over the years. Twin Ace or Tanduay
Appeals, 189 SCRA 46 [1990]). A definitive judgment is no longer subject to change, revision,
Distillers, on one hand, and Tanduay Distillery Inc. (TDI), on the other, are distinct and separate
amendment, or reversal (Miranda vs. Court of Appeals, 71 SCRA 295 [1976], and the court
corporations. There is nothing to suggest that the owners of TDI, have any common relationship
loses jurisdiction over it, except to order its execution (PY Eng Chong vs. Herrera, 70 SCRA 130
as to identify it with Allied Bank Group which runs Tanduay Distillers. The dissertation of the
(1976]).
Court in Diatagon Labor Federation Local 110 of the ULGWP vs. Ople, et al. (101 SCRA 534
[1980]) is worthy of restatement, thusly:
An examination of the aforequoted dispositive portion of the decision shows that the same does
not in any manner obligate Tanduay Distillers, or even petitioners Yu and Young for that matter,
We hold that the director of labor Relations acted with grave abuse of discretion in treating the
to reinstate respondents. Only TDI was held liable to reinstate respondents up to the time of
two companies as a single bargaining unit. The ruling is arbitrary and untenable because the two
change of ownership, and for separation benefits.
companies are indubitably distinct entities with separate juridical personalities.
The fact that their businesses are related and that the 236 employees of Georgia Pacific In La Campana Coffee Factory. Inc. vs. Kaisahan ng Mangagawa sa La Campana (KKM), (93
International Corporation were originally employees of Lianga Bay Logging Co., Inc, is not a Phil, 160 [1953]), La Campana Coffee Factory, Inc. and La Campana Gaugau Packing were
justification for disregarding their separate personalities. Hence, the 236 employees, who are substantially owned by the same person. They had one office, one management, and a single
now attached to Georgia Pacific International should not be allowed to vote in the certification payroll for both businesses. The laborers of the gaugau factory and the coffee factory were also
election at the Lianga Bay Logging Co., Inc. They should vote at a separate certification election interchangeable, the workers in one factory worked also in the other factory.
to determine the collective bargaining representative of the employees of Georgia Pacific
International Corporation.
In Claparols vs. Court of Industrial Relations (65 SCRA 613 (1975]), the Claparols Steel and Nail
Plant, which was ordered to pay its workers backwages, ceased operations on June 30, 1956
(at pp. 540-541.) and was succeeded on the very next day, July 1, 1957, by the Claparols Steel Corporation. Both
corporations were substantially owned and controlled by the same person and there was no
break or cessation in operations. Moreover, all the assets of the steel and nail plant were
It is basic that a corporation is invested by law with a personality separate and distinct from
transferred to the new corporation.
those of the persons composing it as well as from that of any other legal entity to which it may be
related (Palay, Inc. et al. vs. Clave, et al., 124 SCRA 641 [1983]).
In fine, the fiction of separate and distinct corporate entities cannot, in the instant case, be
disregarded and brushed aside, there being not the least indication that the second corporation
The genuine nature of the sale to Twin Ace is evidenced by the fact that Twin Ace was only a
is a dummy or serves as a client of the first corporate entity.
subsequent interested buyer. At the time when termination notices were sent to its employees,
TDI was negotiating with the First Pacific Metro Corporation for the sale of its assets. Only after
First Pacific gave up its efforts to acquire the assets did Twin Ace or Tanduay Distillers come In the case at bench, since TDI and Twin Ace or Tanduay Distillers are two separate and distinct
into the picture. Respondents-employees have not presented any proof as to communality of entities, the order for Tanduay Distillers (and petitioners) to reinstate respondents-employees is
ownership and management to support their contention that the two companies are one firm or obviously without legal and factual basis.
closely related. The doctrine of piercing the veil of corporate entity applies when the corporate
fiction is used to defeat public convenience, justify wrong, protect fraud, or defend crime or
3. Nor could the order and writ to reinstate be anchored on the vague and seemingly uncalled for
where a corporation is the mere alter ego or business conduit of a person (Indophil Textile Mill
alternative disposition in the Barcelona decision that —
Workers Union vs. Calica, 205 SCRA 697, 703 (1992]). To disregard the separate juridical
personality of a corporation, the wrong-doing must be clearly and convincingly established. It
cannot be presumed (Del Rosario vs. NLRC, 187 SCRA 777, 7809 [1990]). . . . This is without prejudice to the letter of Mr. James Yu as officer-in-charge of Tanduay
Distillers dated June 16, 1988 to the President of the Tanduay Distillery, Inc. labor Union.
The complaint for unfair labor practice, illegal lay off, and separation benefits was filed against
TDI. Only later when the manufacture and sale of Tanduay products was taken over by Twin The June 11, 1988 letter referred to was addressed to Benjamin C. Agaloos, president of the
Ace or Tanduay Distillers were James Yu and Wilson Young impleaded. Tanduay Distillery Labor Union by James Yu in his capacity as officer-in-charge of Tanduay
Distillers.
The corporation itself — Twin Ace or Tanduay Distillers — was never made a party to the case.
It pertinently reads:
Another factor to consider is that TDI as a corporation or its shares of stock were not purchased
by Twin Ace. The buyer limited itself to purchasing most of the assets, equipment, and Please be informed that our company stands firm on its decision to hire everybody with a clean
machinery of TDI. Thus, Twin Ace or Tanduay Distillers did not take over the corporate slate effective June 1, 1988 on a probationary basis while those currently casual or contractual
personality of DTI although they manufacture the same product at the same plant with the same employees shall retain the same employment status. In the same manner that the new company
equipment and machinery. Obviously, the trade name "Tanduay" went with the sale because the stood firm on its decision to grant a 10% across-the-board increase to all employees, which in
new firm does business as Tanduay Distillers and its main product of rum is sold as Tanduay fact has been received by employees concerned.
Rum. There is no showing, however, that TDI itself was absorbed by Twin Ace or that it ceased
to exist as a separate corporation, In point of fact TDI is now herein a party respondent
represented by its own counsel. (p. 88, Rollo.)

Significantly, TDI in the petition at hand has taken the side of its former employees and argues We do not find in the decision of Labor Arbiter Cauton Barcelona or in the letter of James Yu
against Tanduay Distillers. In its memorandum filed on January 9, 1995, TDI argues that it was what the respondents are trying to read into it. Labor Arbiter Cauton-Barcelona found the
retrenchment effected by TDI illegal and ordered TDI to reinstate the complainants and that if
not alone its liability which arbiter recognized "but also of James Yu and Wilson Young
representatives of Twin Ace and/or the Allied Bank Group doing business under the name there is a change of management, then separation benefits would be paid. There is, however, no
"TANDUAY DISTILLERS," to whom the business and assets of TDI were sold." If TDI and order in the decision directing Twin Ace or Tanduay Distillers to hire or reinstate herein four
individual respondents.
Tanduay, Distillers are one and the same group or one is a continuation of the other, the two
would not be fighting each other in this case. TDI would not argue strongly "that the petition
for certiorari filed by James Yu and Wilson Young be dismissed for lack of merit." It is obvious The letter of James Yu does not mention any reinstatement. It assures the president of the labor
that the second corporation, Twin Ace or Tanduay Distillers, is an entity separate and distinct, union that Tanduay Distillers stood firm on its decision to hire employees with a clean slate on a
from the first corporation, TDI. The circumstances of this case are different from the earlier probationary basis. The fact that the employees of the former employer (TDI) would be hired on
decisions of the Court in labor cases where the veil of corporate fiction was pierced. a probationary basis shows that there was no employer-employee relationship between
individual respondents and Twin Ace. Any one who joins the buyer corporation comes in as an
outsider who is newly hired and who starts on a probationary basis until he proves he deserves
to be on a permanent status. His application can be rejected in the exercise of the hiring
authority's discretion.

There is thus no legal basis for Labor Arbiter Cueto or the NLRC to compel Twin Ace or
Tanduay Distillers, or petitioners to "reinstate" the four individual respondents. The letter of
James Yu to the union president was a unilateral and gratuitous offer with no consideration. It
refers to people who still have to be hired. New hires had to be investigated or evaluated if they
have "clean slates." Twin Ace or Tanduay Distillers and petitioners are being compelled by
public respondents to reinstate workers who were never their employees. There is no showing
that the sale of assets by TDI to Tanduay Distillers included a condition that employees of the
former would be absorbed by the latter.

Employees of TDI had been terminated in their employment as of April 28, 1988. Petitioners
state that Twin Ace bought the assets of TDI after the employment of the individual respondents
had been terminated. True, Labor Arbiter Cauton-Barcelona declared the retrenchment program
of TDI as illegal. This decision, however, did not convert the individual respondents into
employees of the firm which purchased the assets of the former employer. It merely held TDI
liable for the consequences of the illegal retrenchment.

Labor Arbiter Cueto and the NLRC, therefore, committed grave abuse of discretion when they
read into the decision of Labor Arbiter Cauton- Barcelona something which did not appear
therein. And even assuming that Labor Arbiter Cauton-Barcelona formally included an order for
the petitioners to hire the individual respondents, there would be no factual or legal basis for
such an order. An employer-employee relation is created by contract, and can not be forced
upon either party simply upon order of a labor arbiter. The hiring of employees is one of the
recognized prerogatives of management.

4. Another factor which militates against the claim for reinstatement of the individual respondents
is their having received separation pay as part of a compromise agreement in the course of their
litigation with TDI. Rodrigo F. Santos received P20,282.03; Roque Estoce, P20,092.50; Eduardo
Daliwan, P19,973.40; and Fernando A. Duran, P25,702.00. These amounts correspond to their
entitlement to separation benefits. Having received separation pay from a former employer, how
can they compel, as a matter of right, another company to hire them on a supposed
"reinstatement" basis? The orders executing the earlier decision of Labor Arbiter Cauton-
Barcelona and directing petitioners to immediately reinstate the four individual respondents to
their former positions are, thus, characterized by grave abuse of discretion. There are no "former
positions" to which individual respondents may be reinstated because they never hired by Twin
Ace/Tanduay Distillers and had never worked for it.

WHEREFORE, the petition is hereby GRANTED, The questioned Order of the Labor Arbiter
Daniel C. Cueto dated November 17, 1992 and the decision of the National Labor Relations
Commission upholding said order are set aside as null and void. No special pronouncement is
made as to costs.

SO ORDERED.
G.R. No. 191525 December 13, 2017 I/AME then filed a petition with the CA to contest the judgment of the RTC, which was eventually
denied by the appellate court.
INTERNATIONAL ACADEMY OF MANAGEMENT AND ECONOMICS (I/AME), Petitioner
vs. THE CA RULING
LITTON AND COMPANY, INC., Respondent
The CA upheld the Judgment and Order of the RTC and held that no grave abuse of discretion
DECISION was committed when the trial court pierced the corporate veil of I/AME. 14

SERENO, CJ.: It took note of how Santos had utilized I/ AME to insulate the Makati real property covered by
TCT No. 187565 from the execution of the judgment rendered against him, for the following
reasons:
Before us is a Petition for Review on Certiorari under Rule 45 of the Rules of Court assailing the
Court of Appeals (CA) Decision1 and Resolution2 in CA-G.R. SP No. 107727.
First, the Deed of Absolute Sale dated 31 August 1979 indicated that Santos, being the
.President, was representing I/AME as the vendee. 15 However, records show that it was only in
The CA affirmed the Judgment3 and Order4 of the Regional Trial Court (RTC) of Manila in
1985 that I/AME was organized as a juridical entity. 16 Obviously, Santos could not have been
Special Civil Action No. 06-115547 reinstating the Order5 of the Metropolitan Trial Court (Me TC)
President of a non-existent corporation at that time.17
of Manila in favor of Litton and Company, Inc. (Litton).

Second, the CA noted that the subject real property was transferred to I/AME during the
THE FACTS
pendency of the appeal for the revival of the judgment in the ejectment case in the CA.18

The facts, as culled from the records, are as follows:


Finally, the CA observed that the Register of Deeds of Makati City issued TCT No. 187565 only
on 17 November 1993, fourteen (14) years after the execution of the Deed of Absolute Sale and
Atty. Emmanuel T. Santos (Santos), a lessee to two (2) buildings owned by Litton, owed the more than eight (8) years after I/AME was incorporated.19
latter rental arrears as well as his share of the payment of realty taxes. 6
Thus, the CA concluded that Santos merely used I/ AME as a shield to protect his property from
Consequently, Litton filed a complaint for unlawful detainer against Santos before the MeTC of the coverage of the writ of execution; therefore, piercing the veil of corporate fiction is proper. 20
Manila. The MeTC ruled in Litton’s favor and ordered Santos to vacate A.I.D. Building and Litton
Apartments and to pay various sums of money representing unpaid arrears, realty taxes,
THE ISSUES
penalty, andattorney’s fees.7

The issues boil down to the alleged denial of due process when the court pierced the corporate
It appears however that the judgment was not executed. Litton subsequently filed an action for
veil of I/ AME and its property was made to answer for the liability of Santos.
revival of judgment, which was granted by the RTC. 8 Santos then appealed the RTC decision to
the CA, which nevertheless affirmed the RTC.9 The said CA decision became final and
executory on 22 March 1994.10 OUR RULING

On l 1 November 1996, the sheriff of the MeTC of Manila levied on a piece of real property We deny the petition.
covered by Transfer Certificate of Title (TCT) No. 187565 and registered in the name of
International Academy of Management and Economics Incorporated (I/AME), in order to execute
There was no violation of due
the judgment against Santos.11 The annotations on TCT No. 187565 indicated that such
process against I/AME
was "only up to the extent of the share of Emmanuel T. Santos."12

Petitioner avers that its right to due process was violated when it was dragged into the case and
I/AME filed with Me TC a "Motion to Lift or Remove Annotations Inscribed in TCT No. 187565 of
its real property made an object of a writ of execution in a judgment against Santos. It argues
the Register of Deeds of Makati City."13 I/AME claimed that it has a separate and distinct
that since it was not impleaded in the main case, the court a quo never acquired jurisdiction over
personality from Santos; hence, its properties should not be made to answer for the latter's
it. Indeed, compliance with the recognized modes of acquisition ofjurisdiction cannot be
liabilities. The motion was denied in an Order dated 29 October 2004.
dispensed with even in piercing the veil of corporation.21

Upon motion for reconsideration of I/AME, the Me TC reversed its earlier ruling and ordered the
In a petition for review on certiorari under Rule 45, only questions of law shall be entertained.
cancellation of the annotations of levy as well as the writ of execution. Litton then elevated the
This Court considers the determination of the existence of any of the circumstances that would
case to the RTC, which in turn reversed the Order granting I/AME’s motion for reconsideration
warrant the piercing of the veil of corporate fiction as a question of fact which ordinarily cannot
and reinstated the original Order dated 29 October 2004.
be the subject of a petition for review on certiorariunder Rule 45. We will only take cognizance of
factual issues if the findings of the lower court are not supported by the evidence on record or
are based on a misapprehension of facts.22 Once the CA affirms the factual findings of the trial contract he entered into as lessee with the Littons as lessor. He was not able to comply with this
court, such findings are deemed final and conclusive and thus, may not be reviewed on appeal, particular obligation, and in fact, refused to comply therewith.
unless the judgment of the CA depends on a misapprehension of facts, which if properly
considered, would justify a different conclusion.23 Such exception however, is not applicable in
This Court agrees with the CA that Santos used I/AME as a means to defeat judicial processes
this case.
and to evade his obligation to Litton.30 Thus, even while I/AME was not imp leaded in the main
case and yet was so named in a writ of execution to satisfy a court judgment against Santos, it is
The 29 October 2004 MeTC judgment, the RTC judgment, and the CA decision are one in vulnerable to the piercing of its corporate veil. We will further expound on this matter.
accord on the matters presented before this Court.
Piercing the Corporate Veil may
In general, corporations, whether stock or non-stock, are treated as separate and distinct legal Apply to Non-stock Corporations
entities from the natural persons composing them. The privilege of being considered a distinct
and separate entity is confined to legitimate uses, and is subject to equitable limitations to
Petitioner I/AME argues that the doctrine of piercing the corporate veil applies only to stock
prevent its being exercised for fraudulent, unfair or illegal purposes.24 However, once equitable
corporations, and not to non-stock, nonprofit corporations such as I/AME since there are no
limitations are breached using the coverture of the corporate veil, courts may step in to pierce
stockholders to hold liable in such a situation but instead only members. Hence, they do not
the same.
have investments or shares of stock or assets to answer for possible liabilities.

As we held in Lanuza, Jr. v. BF Corporation:25


Thus, no one in a non-stock corporation can be held liable in case the corporate veil is
disregarded or pierced.31
Piercing the corporate veil is warranted when "[the separate personality of a
corporation] is used as a means to perpetrate fraud or an illegal act, or as a
The CA disagreed. It ruled that since the law does not make a distinction between a stock and
vehicle for the evasion of an existing obligation, the circumvention of
non-stock corporation, neither should there be a distinction in case the doctrine of piercing the
statutes, or to confuse legitimate issues." It is also warranted in alter ego
veil of corporate fiction has to be applied. While I/AME is an educational institution, the CA
cases "where a corporation is merely a farce since it is a mere alter ego or
further ruled, it still is a registered corporation conducting its affairs as such. 32
business conduit of a person, or where the corporation is so organized and
controlled and its affairs are so conducted as to make it merely an
instrumentality, agency, conduit or adjunct of another corporation." This Court agrees with the CA.

When [the] corporate veil is pierced, the corporation and persons who are In determining the propriety of applicability of piercing the veil of corporate fiction, this Court, in a
normally treated as distinct from the corporation are treated as one person, number of cases, did not put in issue whether a corporation is a stock or non-stock corporation.
such that when the corporation is adjudged liable, these persons, too, In Sula ng Bayan, Inc. v. Gregorio Araneta, Inc. ,33 we considered but ultimately refused to
become liable as if they were the corporation. pierce the corporate veil of a non-stock non-profit corporation which sought to institute an action
for reconveyance of real property on behalf of its members. This Court held that the non-stock
corporation had no personality to institute a class suit on behalf of its members, considering that
The piercing of the corporate veil is premised on the fact that the corporation concerned must
the non-stock corporation was not an assignee or transferee of the real property in question, and
have been properly served with summons or properly subjected to the jurisdiction of the court a
did not have an identity that was one and the same as its members.
quo. Corollary thereto, it cannot be subjected to a writ of execution meant for another in violation
of its right to due process.26
In another case, this Court did not put in issue whether the corporation is a non-stock, non-profit,
non-governmental corporation in considering the application of the doctrine of piercing of
There exists, however, an exception to this rule: if it is shown "by clear and convincing proof that
corporate veil. In Republic of the Philippines v. Institute for Social Concern, 34 while we did not
the separate and distinct personality of the corporation was purposefully employed to evade a
allow the piercing of the corporate veil, this Court affirmed the finding of the CA that the
legitimate and binding commitment and perpetuate a fraud or like wrongdoings. " 27
Chairman of the Institute for Social Concern cannot be held jointly and severally liable with the
aforesaid non-governmental organization (NGO) at the time the Memorandum of Agreement
The resistance of the Court to offend the right to due process of a corporation that is a nonparty was entered into with the Philippine Government. We found no fraud in that case committed by
in a main case, may disintegrate not only when its director, officer, shareholder, trustee or the Chairman that would have justified the piercing of the corporate veil of the NG0.35
member is a party to the main case, but when it finds facts which show that piercing of the
corporate veil is merited.28
In the United States, from which we have adopted our law on corporations, non-profit
corporations are not immune from the doctrine of piercing the corporate veil.1âwphi1 Their
Thus, as the Court has already ruled, a party whose corporation is vulnerable to piercing of its courts view piercing of the corporation asan equitable remedy, which justifies said courts to
corporate veil cannot argue violation of due process.29 scrutm1ze any organization however organized and in whatever manner it operates. Moreover,
control of ownership does not hinge on stock ownership.
In this case, the Court confirms the lower courts' findings that Santos had an existing obligation
based on a court judgment that he owed monthly rentals and unpaid realty taxes under a lease As held in Barineau v. Barineau:36
[t]he mere fact that the corporation involved is a nonprofit corporation does was his business conduit and alter ego. Thus, the acquired properties were actually properties of
not by itself preclude a court from applying the equitable remedy of piercing the decedent and as such, should be divided among the decedent's legitimate children in the
the corporate veil. The equitable character of the remedy permits a court to partition of his estate.43
look to the substance of the organization, and its decision is not controlled
by the statutory framework under which the corporation was formed and
In another instance, this Court allowed the piercing of the corporate veil against another natural
operated. While it may appear to be impossible for a person to exercise
person, in Arcilla v. Court of Appeals. 44 The case stemmed from a complaint for sum of money
ownership control over a non-stock, not-for-profit corporation, a person can
against Arcilla for his failure to pay his loan from the private respondent. Arcilla, in his defense,
be held personally liable under the alter ego theory if the evidence shows
alleged that the loan was in the name of his family corporation, CSAR Marine Resources, Inc.
that the person controlling the corporation did in fact exercise control, even
He further argued that the CA erred in holding CSAR Marine Resources liable to the private
though there was no stock ownership.
respondent since the latter was not impleaded as a party in the case. This Court allowed the
piercing of the corporate veil and held that Arcilla used "his capacity as President, x x x [as] a
In another U.S. case, Public Interest Bounty Hunters v. Board of Governors of Federal Reserve sanctuary for a defense x x x to avoid complying with the liability adjudged against him x x x.
System,37 the U.S. Court allowed the piercing of the corporate veil of the Foundation headed by "45 We held that his liability remained attached even if he was impleaded as a party, and not the
the plaintiff, in order to avoid inequitable results. Plaintiff was found to be the sole trustee, the corporation, to thecollection case and even if he ceased to be corporate president. 46 Indeed,
sole member of the board, and the sole financial contributor to the Foundation. In the end, the even if Arcilla had ceased to be corporate president, he remained personally liable for the
Court found that the plaintiff used the Foundation to avoid paying attorneys’ fees. judgment debt to pay his personal loan, for we treated him and the corporation as one and the
same. CSAR Marine was deemed his alter ego.
The concept of equitable ownership, for stock or non-stock corporations, in piercing of the
corporate veil scenarios, may also be considered. An equitable owner is an individual who is a We find similarities with Arcilla and the instant case. Like Arcilla, Santos: (1) was adjudged liable
non-shareholder defendant, who exercises sufficient control or considerable authority over the to pay on a judgment against him; (2) he became President of a corporation; (3) he formed a
corporation to the point of completely disregarding the corporate form and acting as though its corporation to conceal assets which were supposed to pay for the judgment against his favor; (4)
assets are his or her alone to manage and distribute.38 the corporation which has Santos as its President, is being asked by the court to pay on the
judgment; and (5) he may not use as a defense that he is no longer President of I/AME
(although a visit to the website of the school shows he is the current President). 47
Given the foregoing, this Court sees no reason why a non-stock corporation such as I/AME, may
not be scrutinized for purposes of piercing the corporate veil or fiction.
This Court agrees with the CA that I/AME is the alter ego of Santos and Santos - the natural
person - is the alter ego of I/AME. Santos falsely represented himself as President of I/AME in
Piercing the Corporate Veil may
the Deed of Absolute Sale when he bought the Makati real property, at a time when I/ AME had
Apply to Natural Persons
not yet existed. Uncontroverted facts in this case also reveal the findings of Me TC showing
Santos and I/ AME as being one and the same person:
The petitioner also insists that the piercing of the corporate veil cannot be applied to a natural
person - in this case, Santos - simply because as a human being, he has no corporate veil
(1) Santos is the conceptualizer and implementor of I/AME;
shrouding or covering his person.39

(2) Santos’ contribution is ₱1,200,000.00 (One Million Two Hundred Thousand Pesos) out of the
a) When the Corporation is the Alter Ego of a Natural Person
₱1,500,000.00 (One Million Five Hundred Thousand Pesos), making him the majority contributor
of I/AME; and,
As cited in Sula ng Bayan, Inc. v. Araneta, Inc. ,40 "[t]he doctrine of alter ego is based upon
the misuse of a corporation by an individual for wrongful or inequitable purposes, and in such
(3) The building being occupied by I/AME is named after Santos using his known nickname (to
case the court merely disregards the corporate entity and holds the individual responsible for
date it is called, the "Noli Santos Inte1national Tower").48
acts knowingly and intentionally done in the name of the corporation." This, Santos has done in
this case. Santos formed I/AME, using the non-stock corporation, to evade paying his judgment
creditor, Litton. This Court deems I/AME and Santos as alter egos of each other based on the former’s own
admission in its pleadings before the trial court. In its Answer (to Amended Petition) with the
RTC entitled Litton and Company, Inc. v. Hon. Hernandez-Calledo, Civil Case No. 06-115547,
The piercing of the corporate veil may apply to corporations as well as natural persons involved
I/AME admitted the allegations found in paragraphs 2, 4 and 5 of the amended petition of Litton,
with corporations. This Court has held that the "corporate mask may be lifted and the corporate
particularly paragraph number 4 which states:
veil may be pierced when a corporation is just but the alter ego of a person or of another
corporation."41
4. Respondent, International Academy of Management and Economics
Inc. (hereinafter referred to as Respondent I/ AME), is a corporation
We have considered a deceased natural person as one and the same with his corporaticc to
organized and existing under Philippine laws with address at 1061
protect the succession rights of his legal heirs to his estate. In Cease v. Court of Appeals, 42 the
Metropolitan Avenue, San Antonie Village, Makati City, where it may be
predecessor-in-interest organized a close corporation which acquired properties during its
served with summons and other judicial processes. It is the corporate
existence. When he died intestate, trouble ensued amongst his children on whether or not to
entity used by Respondent Santos as his alter ego for the purpose of
consider his company one and the same with his person. The Court agreed with the trial court
when it pierced the corporate veil of the decedent's corporation. It found that said corporation
shielding his assets from the reach of his creditors, one of which is of Fely Transportation to run after its corporate assets and pay the subsidiary civil liability of the
herein Petitioner.49 (Emphases ours) company's president and general manager.

Hence, I/AME is the alter ego of the natural person, Santos, which the latter used to evade the This notwithstanding, the equitable remedy of reverse corporate piercing or reverse piercing was
execution on the Makati property, thus frustrating the satisfaction of the judgment won by Litton. not meant to encourage a creditor’s failure to undertake such remedies that could have
otherwise been available, to the detriment of other creditors.56
b) Reverse Piercing of the Corporate Veil
Reverse corporate piercing is an equitable remedy which if utilized cavalierly, may lead to
disastrous consequences for both stock and non-stock corporations. We are aware that ordinary
This Court in Arcilla pierced the corporate veil of CSAR Marine Resources to satisfy a money
judgment collection procedures or other legal remedies are preferred over that which would risk
judgment against its erstwhile President, Arcilla.
damage to third parties (for instance, innocent stockholders or voluntary creditors) with
unprotected interests in the assets of the beleaguered corporation.57
We borrow from American parlance what is called reverse piercing or reverse corporate piercing
or piercing the corporate veil "in reverse."
Thus, this Court would recommend the application of the current 1997 Rules on Civil Procedure
on Enforcement of Judgments. Under the current Rules of Court on Civil Procedure, when it
As held in the U.S. Case, C.F. Trust, Inc., v. First Flight Limited Partnership, 50 "in a traditional comes to satisfaction by levy, a judgment obligor is given the option to immediately choose
veil-piercing action, a court disregards the existence of the corporate entity so a claimant can which property or part thereof may be levied upon to satisfy the judgment. If the judgmentobligor
reach the assets of a corporate insider. In a reverse piercing action, however, the plaintiff seeks does not exercise the option, personal properties, if any, shall be first levied and then on real
to reach the assets of a corporation to satisfy claims against a corporate insider." properties if the personal properties are deemed insufficient to answer for the judgment. 58

"Reverse-piercing flows in the opposite direction (of traditional corporate veil-piercing) and In the instant case, it may be possible for this Court to recommend that Litton run after the other
makes the corporation liable for the debt of the shareholders." 51 properties of Santos that could satisfy the money judgment - first personal, then other real
properties other than that of the school. However, if we allow this, we frustrate the decades-old
yet valid MeTC judgment which levied on the real property now titled under the name of the
It has two (2) types: outsider reverse piercing and insider reverse piercing. Outsider reverse school. Moreover, this Court will unwittingly condone the action of Santos in hiding all these
piercing occurs when a party with a claim against an individual or corporation attempts to be
years behind the corporate form to evade paying his obligation under the judgment in the court a
repaid with assets of a corporation owned or substantially controlled by the defendant. 52 In quo. This we cannot countenance without being a party to the injustice.
contrast, in insider reverse piercing, the controlling members will attempt to ignore the corporate
fiction in order to take advantage of a benefit available to the corporation, such as an interest in
a lawsuit or protection of personal assets.53 Thus, the reverse piercing of the corporate veil of I/AME to enforce the levy on execution of the
Makati real property where the school now stands is applied.
Outsider reverse veil-piercing is applicable in the instant case. Litton, as judgment creditor,
seeks the Court's intervention to pierce the corporate veil of I/AME in order to make its Makati WHEREFORE, in view of the foregoing, the instant petition is DENIED. The CA Decision in CA-
real property answer for a judgment against Santos, who formerly owned and still substantially G.R. SP No. 107727 dated 30 October 2009 and its Resolution on 12 March 2010 are
controls I/AME. hereby AFFIRMED. The MeTC Order dated 29 October 2004 is hereby REINSTATED.

In the U.S. case Acree v. McMahan, 54 the American court held that "[ o ]utsider reverse veil- Accordingly, the MeTC of Manila, Branch 2, is hereby DIRECTED to execute with dispatch the
piercing extends the traditional veil-piercing doctrine to permit a third-party creditor to pierce the MeTC Order dated 29 October 2004 against Santos.
veil to satisfy the debts of an individual out of the corporation's assets."
SO ORDERED.
The Court has pierced the corporate veil in a reverse manner in the instances when the scheme
was to avoid corporate assets to be included in the estate of a decedent as in the Cease case
and when the corporation was used to escape a judgment to pay a debt as in the Arcilla case.

In a 1962 Philippine case, this Court also employed what we now call reverse-piercing of the
corporate veil. In Palacio v. Fely Transportation Co., 55 we found that the president and general
manager of the private respondent company formed the corporation to evade his subsidiary civil
liability resulting from the conviction of his driver who ran over the child of the petitioner, causing
injuries and medical expenses. The Court agreed with the plaintiffs that the president and
general manager, and Fely Transportation, may be regarded as one and the same person.
Thus, even if the president and general manager was not a party to the case, we reversed the
lower court and declared both him and the private respondent company, jointly and severally
liable to the plaintiffs. Thus, this Court allowed the outsider-plaintiffs to pierce the corporate veil
G.R. No. L-19891 July 31, 1964 On March 17, 1962, the lower court rendered judgment embodying the contents of the said
compromise agreement, the dispositive portion of which reads —
J.R.S. BUSINESS CORPORATION, J.R. DA SILVA and A.J. BELTRAN, petitioners,
vs. WHEREFORE, the Court hereby approves the above-quoted compromise agreement and
IMPERIAL INSURANCE, INC., MACARIO M. OFILADA, Sheriff of Manila and renders judgment in accordance therewith, enjoining the parties to comply faithfully and strictly
HON. AGUSTIN MONTESA, Judge of the Court of First Instance of Manila, respondents. with the terms and conditions thereof, without special pronouncement as to costs.

Felipe N. Aurea for petitioners. Wherefore, the parties respectfully pray that the foregoing stipulation of facts be admitted and
Tañada, Teehankee and Carreon for respondent Imperial Insurance, Inc. approved by this Honorable Court, without prejudice to the parties adducing other evidence to
prove their case not covered by this stipulation of facts. 1äwphï1.ñët
PAREDES, J.:
On May 15, 1962, one day after the date fixed in the compromise agreement, within which the
judgment debt would be paid, but was not, respondent Imperial Insurance Inc., filed a "Motion for
Petitioner J. R. Da Silva, is the President of the J.R.S. Business Corporation, an establishment
the Insurance of a Writ of Execution". On May 23, 1962, a Writ of Execution was issued by
duly franchised by the Congress of the Philippines, to conduct a messenger and delivery
respondent Sheriff of Manila and on May 26, 1962, Notices of Sale were sent out for the auction
express service. On July 12, 1961, the respondent Imperial Insurance, Inc., presented with the
of the personal properties of the petitioner J.R.S. Business Corporation. On June 2, 1962, a
CFI of Manila a complaint (Civ. Case No. 47520), for sum of money against the petitioner
Notice of Sale of the "whole capital stocks of the defendants JRS Business Corporation, the
corporation. After the defendants therein have submitted their Answer, the parties entered into a
business name, right of operation, the whole assets, furnitures and equipments, the total
Compromise Agreement, assisted by their respective counsels, the pertinent portions of which
liabilities, and Net Worth, books of accounts, etc., etc." of the petitioner corporation was, handed
recite:
down. On June 9, the petitioner, thru counsel, presented an "Urgent Petition for Postponement
of Auction Sale and for Release of Levy on the Business Name and Right to Operate of
1) WHEREAS, the DEFENDANTS admit and confess their joint and solidary indebtedness to the Defendant JRS Business Corporation", stating that petitioners were busy negotiating for a loan
PLAINTIFF in the full sum of PESOS SIXTY ONE THOUSAND ONE HUNDRED SEVENTY- with which to pay the judgment debt; that the judgment was for money only and, therefore,
TWO & 32/100 (P61,172.32), Philippine Currency, itemized as follows: plaintiff (respondent Insurance Company) was not authorized to take over and appropriate for its
own use, the business name of the defendants; that the right to operate under the franchise, was
not transferable and could not be considered a personal or immovable, property, subject to levy
a) Principal P50,000.00 and sale. On June 10, 1962, a Supplemental Motion for Release of Execution, was filed by
counsel of petitioner JRS Business Corporation, claiming that the capital stocks thereof, could
not be levied upon and sold under execution. Under date of June 20, 1962, petitioner's counsel
b) Interest at 12% per annum 5,706.14 presented a pleading captioned "Very Urgent Motion for Postponement of Public Auction Sale
and for Ruling on Motion for Release of Levy on the Business Name, Right to Operate and
Capital Stocks of JRS Business Corporation". The auction sale was set for June 21, 1962. In
said motion, petitioners alleged that the loan they had applied for, was to be secured within the
c) Liquidated damages at 7% per annum 3,330.58 next ten (10) days, and they would be able to discharge the judgment debt. Respondents
opposed the said motion and on June 21, 1962, the lower court denied the motion for
postponement of the auction sale.
d) Costs of suit 135.60
In the sale which was conducted in the premises of the JRS Business Corporation at 1341 Perez
St., Paco, Manila, all the properties of said corporation contained in the Notices of Sale dated
e) Attorney's fees 2,000.00
May 26, 1962, and June 2, 1962 (the latter notice being for the whole capital stocks of the
defendant, JRS Business Corporation, the business name, right of operation, the whole assets,
furnitures and equipments, the total liabilities and Net Worth, books of accounts, etc., etc.), were
bought by respondent Imperial Insurance, Inc., for P10,000.00, which was the highest bid
2) WHEREAS, the DEFENDANTS bind themselves, jointly and severally, and hereby promise to offered. Immediately after the sale, respondent Insurance Company took possession of the
pay their aforementioned obligation to the PLAINTIFF at its business address at 301-305 proper ties and started running the affairs and operating the business of the JRS Business
Banquero St., (Ground Floor), Regina Building, Escolta, Manila, within sixty (60) days from Corporation. Hence, the present appeal.
March 16, 1962 or on or before May 14, 1962;
It would seem that the matters which need determination are (1) whether the respondent Judge
3) WHEREAS, in the event the DEFENDANTS FAIL to pay in full the total amount of PESOS acted without or in excess of his jurisdiction or with grave abuse of discretion in promulgating the
SIXTY ONE THOUSAND ONE HUNDRED SEVENTY TWO & 32/100 (P61,172.32), Philippine Order of June 21, 1962, denying the motion for postponement of the scheduled sale at public
Currency, for any reason whatsoever, on May 14, 1962, the PLAINTIFF shall be entitled, as a auction, of the properties of petitioner; and (2) whether the business name or trade name,
matter of right, to move for the execution of the decision to be rendered in the above-entitled franchise (right to operate) and capital stocks of the petitioner are properties or property rights
case by this Honorable Court based on this COMPROMISE AGREEMENT. which could be the subject of levy, execution and sale.
The respondent Court's act of postponing the scheduled sale was within the discretion of The right to operate a messenger and express delivery service, by virtue of a legislative
respondent Judge, the exercise of which, one way or the other, did not constitute grave abuse of enactment, is admittedly a secondary franchise (R.A. No. 3260, entitled "An Act granting the
discretion and/or excess of jurisdiction. There was a decision rendered and the corresponding JRS Business Corporation a franchise to conduct a messenger and express service)" and, as
writ of execution was issued. Respondent Judge had jurisdiction over the matter and erroneous such, under our corporation law, is subject to levy and sale on execution together and including
conclusions of law or fact, if any, committed in the exercise of such jurisdiction are merely errors all the property necessary for the enjoyment thereof. The law, however, indicates the procedure
of judgment, not correctible by certiorari (Villa Rey Transit v. Bello, et al., L-18957, April 23, under which the same (secondary franchise and the properties necessary for its enjoyment) may
1963, and cases cited therein.) be sold under execution. Said franchise can be sold under execution, when such sale is
especially decreed and ordered in the judgment and it becomes effective only when the sale is
confirmed by the Court after due notice (Sec. 56, Corp. Law). The compromise agreement and
The corporation law, on forced sale of franchises, provides —
the judgment based thereon, do not contain any special decree or order making the franchise
answerable for the judgment debt. The same thing may be stated with respect to petitioner's
Any franchise granted to a corporation to collect tolls or to occupy, enjoy, or use public property trade name or business name and its capital stock. Incidentally, the trade name or business
or any portion of the public domain or any right of way over public property or the public domain, name corresponds to the initials of the President of the petitioner corporation and there can be
and any rights and privileges acquired under such franchise may be levied upon and sold under no serious dispute regarding the fact that a trade name or business name and capital stock are
execution, together with the property necessary for the enjoyment, the exercise of the powers, necessarily included in the enjoyment of the franchise. Like that of a franchise, the law
and the receipt of the proceeds of such franchise or right of way, in the same manner and with mandates, that property necessary for the enjoyment of said franchise, can only be sold to
like effect as any other property to satisfy any judgment against the corporation: Provided, That satisfy a judgment debt if the decision especially so provides. As We have stated heretofore, no
the sale of the franchise or right of way and the property necessary for the enjoyment, the such directive appears in the decision. Moreover, a trade name or business name cannot be
exercise of the powers, and the receipt of the proceeds of said franchise or right of way is sold separately from the franchise, and the capital stock of the petitioner corporation or any other
especially decreed and ordered in the judgment: And provided, further, That the sale shall not corporation, for the matter, represents the interest and is the property of stockholders in the
become effective until confirmed by the court after due notice. (Sec. 56, Corporation Law.) corporation, who can only be deprived thereof in the manner provided by law (Therbee v. Baker,
35 N.E. Eq. [8 Stew.] 501, 505; In re Wells' Estate, 144 N.W. 174, 177, Wis. 294, cited in 6
Words and Phrases, 109).
In the case of Gulf Refining Co. v. Cleveland Trust Co., 108 So., 158, it was held —

It, therefore, results that the inclusion of the franchise, the trade name and/or business name
The first question then for decision is the meaning of the word "franchise" in the statute.
and the capital stock of the petitioner corporation, in the sale of the properties of the JRS
Business Corporation, has no justification. The sale of the properties of petitioner corporation is
"A franchise is a special privilege conferred by governmental authority, and which does not set aside, in so far as it authorizes the levy and sale of its franchise, trade name and capital
belong to citizens of the country generally as a matter of common right. ... Its meaning depends stocks. Without pronouncement as to costs.
more or less upon the connection in which the word is employed and the property and
corporation to which it is applied. It may have different significations.

"For practical purposes, franchises, so far as relating to corporations, are divisible into (1)
corporate or general franchises; and (2) special or secondary franchises. The former is the
franchise to exist as a corporation, while the latter are certain rights and privileges conferred
upon existing corporations, such as the right to use the streets of a municipality to lay pipes or
tracks, erect poles or string wires." 2 Fletcher's Cyclopedia Corp. See. 1148; 14 C.J. p. 160;
Adams v. Yazon & M. V. R. Co., 24 So. 200, 317, 28 So. 956, 77 Miss. 253, 60 L.R.A. 33 et seq.

The primary franchise of a corporation that is, the right to exist as such, is vested "in the
individuals who compose the corporation and not in the corporation itself" (14 C.J. pp. 160, 161;
Adams v. Railroad, supra; 2 Fletcher's Cyclopedia Corp. Secs. 1153, 1158; 3 Thompson on
Corporations 2d Ed.] Secs. 2863, 2864), and cannot be conveyed in the absence of a legislative
authority so to do (14A CJ. 543, 577; 1 Fletcher's Cyc. Corp. Sec. 1224; Memphis & L.R.R. Co.
v. Berry 5 S. Ct. 299, 112 U.S. 609, 28 L.E.d. 837; Vicksburg Waterworks Co. v. Vicksburg, 26
S. Ct. 660, 202 U.S. 453, 50 L.E.d. 1102, 6 Ann. Cas. 253; Arthur v. Commercial & Railroad
Bank, 9 Smedes & M. 394, 48 Am. Dec. 719), but the specify or secondary franchises of a
corporation are vested in the corporation and may ordinarily be conveyed or mortgaged under a
general power granted to a corporation to dispose of its property (Adams v. Railroad, supra; 14A
C.J. 542, 557; 3 Thompson on Corp. [2nd Ed.] Sec. 2909), except such special or secondary
franchises as are charged with a public use (2 Fletcher's Cyc. Corp. see. 1225; 14A C.J. 544; 3
Thompson on Corp. [2d Ed.] sec. 2908; Arthur v. Commercial & R.R. Bank, supra; McAllister v.
Plant, 54 Miss. 106).
G.R. No. 148076 January 12, 2011 On July 28, 1998, Carandang assumed office as general manager and chief operating officer of
RPN.8
ANTONIO M. CARANDANG, Petitioner,
vs. On April 19, 1999, Carandang and other RPN officials were charged with grave misconduct
HONORABLE ANIANO A. DESIERTO, OFFICE OF THE OMBUDSMAN, Respondent. before the Ombudsman. The charge alleged that Carandang, in his capacity as the general
manager of RPN, had entered into a contract with AF Broadcasting Incorporated despite his
being an incorporator, director, and stockholder of that corporation; that he had thus held
x - - - - - - - - - - - - - - - - - - - - - - -x
financial and material interest in a contract that had required the approval of his office; and that
the transaction was prohibited under Section 7 (a) and Section 9 of Republic Act No. 6713
G.R. No. 153161 (Code of Conduct and Ethical Standards for Public Officials and Employees), thereby rendering
him administratively liable for grave misconduct.
ANTONIO M. CARANDANG, Petitioner,
vs. Carandang sought the dismissal of the administrative charge on the ground that the
SANDIGANBAYAN (FIFTH DIVISION), Respondent. Ombudsman had no jurisdiction over him because RPN was not a government-owned or -
controlled corporation.9
DECISION
On May 7, 1999, the Ombudsman suspended Carandang from his positions in RPN.
BERSAMIN, J.:
On September 8, 1999, Carandang manifested that he was no longer interested and had no
further claim to his positions in RPN. He was subsequently replaced by Edgar San Luis.10
Petitioner Antonio M. Carandang (Carandang) challenges the jurisdiction over him of the
Ombudsman and of the Sandiganbayan on the ground that he was being held to account for
acts committed while he was serving as general manager and chief operating officer of Radio In its decision dated January 26, 2000,11 the Ombudsman found Carandang guilty of grave
Philippines Network, Inc. (RPN), which was not a government-owned or -controlled corporation; misconduct and ordered his dismissal from the service.
hence, he was not a public official or employee.
Carandang moved for reconsideration on two grounds: (a) that the Ombudsman had no
In G.R. No. 148076, Carandang seeks the reversal of the decision1 and resolution2 promulgated jurisdiction over him because RPN was not a government-owned or -controlled corporation; and
by the Court of Appeals (CA) affirming the decision3 of the Ombudsman dismissing him from the (b) that he had no financial and material interest in the contract that required the approval of his
service for grave misconduct. office.12

In G.R. No. 153161, Carandang assails on certiorari the resolutions dated October 17, The Ombudsman denied Carandang’s motion for reconsideration on March 15, 2000.13
20014 and March 14, 20025of the Sandiganbayan (Fifth Division) that sustained the
Sandiganbayan’s jurisdiction over the criminal complaint charging him with violation of Republic
On appeal (CA G.R. SP No. 58204),14 the CA affirmed the decision of the Ombudsman on
Act No. 3019 (Anti-Graft and Corrupt Practices Act).
February 12, 2001, stating:

Antecedents
The threshold question to be resolved in the present case is whether or not the Office of the
Ombudsman has jurisdiction over the herein petitioner.
Roberto S. Benedicto (Benedicto) was a stockholder of RPN, a private corporation duly
registered with the Securities and Exchange Commission (SEC). 6 In March 1986, the
It is therefore of paramount importance to consider the definitions of the following basic terms, to
Government ordered the sequestration of RPN’s properties, assets, and business. On
wit: A public office"is the right, authority and duty, created and conferred by law, by which for a
November 3, 1990, the Presidential Commission on Good Government (PCGG) entered into a
given period, either fixed by law or enduring at the pleasure of the creating power, an individual
compromise agreement with Benedicto, whereby he ceded to the Government, through the
is invested with some portion of the sovereign functions of the state to be exercised by him for
PCGG, all his shares of stock in RPN. Consequently, upon motion of the PCGG, the
the benefit of the public." (San Andres, Catanduanes vs. Court of Appeals, 284 SCRA 276:
Sandiganbayan (Second Division) directed the president and corporate secretary of RPN to
Chapter I, Section 1, Mechem, A Treatise on Law of Public Offices and Officers). The individual
transfer to the PCGG Benedicto’s shares representing 72.4% of the total issued and outstanding
so invested is called the public officer which "includes elective and appointive officials and
capital stock of RPN.
employees, permanent or temporary, whether in the classified or unclassified or exemption
service receiving compensation, even nominal, from the government as defined in xxx [Sec. 2
However, Benedicto moved for a reconsideration, contending that his RPN shares ceded to the (a) of Republic Act No. 3019 as amended]." (Sec. 2 (b) of Republic Act No. 3019 as
Government, through the PCGG, represented only 32.4% of RPN’s outstanding capital stock, amended. Unless the powers conferred are of this nature, the individual is not a public officer.
not 72.4%. Benedicto’s motion for reconsideration has remained unresolved to this date. 7
With these time-honored definitions and the substantial findings of the Ombudsman, We are
Administrative Complaint for Grave Misconduct constrained to conclude that, indeed, the herein petitioner (Antonio M. Carandang) is a public
officer. Precisely, since he (Antonio M. Carandang) was appointed by then President Joseph
Ejercito Estrada as general manager and chief operating officer of RPN-9 (page 127 of the and controlled corporation. Another thing, RPN 9, though predominantly tackles proprietary
Rollo). As a presidential appointee, the petitioner derives his authority from the Philippine functions—those intended for private advantage and benefit, still, it is irrefutable that RPN-9 also
Government. It is luce clarius that the function of the herein petitioner (as a presidential performs governmental roles in the interest of health, safety and for the advancement of public
appointee), relates to public duty, i.e., to represent the interest of the Philippine Government in good and welfare, affecting the public in general.
RPN-9 and not purely personal matter, thus, the matter transcends the petitioner’s personal
pique or pride.
xxx

xxx
Coming now to the last assignment of error- While it may be considered in substance that the
"latest GIS clearly shows that petitioner was no longer a stockholder of record of AF
Having declared earlier that the herein petitioner is a public officer, it follows therefore that, that Broadcasting Corporation at the time of his assumption of Office in RPN 9 x x x" (Petitioner’s
jurisdiction over him is lodged in the Office of the Ombudsman. Reply [to Comment]; page 317 of the Rollo), still severing ties from AF Broadcasting Corporation
does not convince this Court fully well to reverse the finding of the Ombudsman that Antonio
Carandang "appears to be liable for Grave Misconduct" (page 10 of the Assailed Decision; page
It is worth remembering that as protector of the people, the Ombudsman has the power, function
36 of the Rollo). Note that, as a former stockholder of AF Broadcasting Corporation, it is
and duty to act promptly on complaints filed in any form or manner against officers or employees
improbable that the herein petitioner was completely oblivious of the developments therein and
of the Government, or of any, subdivision, agency or instrumentality thereof, including
unaware of the contracts it (AF Broadcasting Corporation) entered into. By reason of his past
government-owned or controlled corporations, and enforce their administrative, civil and criminal
(Antonio Carandang) association with the officers of the AF Broadcasting Corporation, it is
liability in every case where the evidence warrants in order to promote efficient service by the
unbelievable that herein petitioner could simply have ignored the contract entered into between
Government to the people. (Section 13 of Republic Act No. 6770).
RPN-9 and AF Broadcasting Corporation and not at all felt to reap the benefits thereof.
Technically, it is true that herein petitioner did not directly act on behalf of AF Broadcasting
xxx Corporation, however, We doubt that he (herein petitioner) had no financial and/or material
interest in that particular transaction requiring the approval of his office—a fact that could not
have eluded Our attention.
Accordingly, the Office of the Ombudsman is, therefore, clothed with the proper armor when it
assumed jurisdiction over the case filed against the herein petitioner. x x x
xxx
xxx
WHEREFORE, premises considered and pursuant to applicable laws and jurisprudence on the
matter, the present Petition for Review is hereby DENIED for lack of merit. The assailed decision
It appears that RPN-9 is a private corporation established to install, operate and manage radio
(dated January 26, 2000) of the Office of the Ombudsman in OMB-ADM-0-99-0349 is hereby
broadcasting and/or television stations in the Philippines (pages 59-79 of the Rollo). On March 2, AFFIRMED in toto. No pronouncement as to costs.
1986, when RPN-9 was sequestered by the Government on ground that the same was
considered as an illegally obtained property (page 3 of the Petition for Review; page 2 of the
Respondent’s Comment; pages 10 and 302 of the Rollo), RPN-9 has shed-off its private status. SO ORDERED.15
In other words, there can be no gainsaying that as of the date of its sequestration by the
Government, RPN-9, while retaining its own corporate existence, became a government-owned
After the denial of his motion for reconsideration,16 Carandang commenced G.R. No. 148076.
or controlled corporation within the Constitutional precept.

Violation of Section 3 (g), Republic Act No. 3019


Be it noted that a government-owned or controlled corporation "refers to any agency organized
as a stock or non-stock corporation, vested with functions relating to public needs whether
government or proprietary in nature, and owned by the Government directly or through its On January 17, 2000, the Ombudsman formally charged Carandang in the Sandiganbayan with
instrumentalities either wholly, or, where applicable as in the case of stock corporations, to the a violation of Section 3 (g) of RA 3019 by alleging in the following information, 17 viz:
extent of at least fifty-one (51) percent of its capital stock; Provided, That government-owned or
controlled corporations may be further categorized by the department of Budget, the Civil
That sometime on September 8, 1998 or thereabouts, in Quezon City, Philippines and within the
Service, and the Commission on Audit for purposes of the exercise and discharge of their
jurisdiction of this Honorable Court, accused ANTONIO M. CARANDANG, a high ranking officer
respective powers, functions and responsibilities with respect to such corporations." (Section 2
(HRO) being then the General Manager of Radio Philippines Network, Inc. (RPN-9), then a
[13], Executive Order No. 292).
government owned and controlled corporation, did then and there willfully, unlawfully and
criminally give unwarranted benefits to On Target Media Concept, Inc. (OTMCI) through
Contrary to the claim of the petitioner, this Court is of the view and so holds that RPN-9 perfectly manifest partiality and gross inexcusable negligence and caused the government undue injury,
falls under the foregoing definition. For one, "the government’s interest to RPN-9 amounts to by pre-terminating the existing block time contract between RPN 9 and OTMCI for the telecast of
72.4% of RPN’s capital stock with an uncontested portion of 32.4% and a contested or litigated "Isumbong Mo Kay Tulfo" which assured the government an income of Sixty Four Thousand and
portion of 40%." (page 3 of the Petition for Review; pages 8-9 of the Respondent’s Comment). Nine Pesos (P 64,009.00) per telecast and substituting the same with a more onerous co-
On this score, it ought to be pointed out that while the forty percent (40%) of the seventy two production agreement without any prior study as to the profitability thereof, by which agreement
point four percent (72.4%) is still contested and litigated, until the matter becomes formally RPN-9 assumed the additional obligation of taking part in the promotions, sales and proper
settled, the government, for all interests and purposes still has the right over said portion, for the marketing of the program, with the end result in that in a period of five (5) months RPN-9 was
law is on its side. Hence, We can safely say that for the moment, RPN-9 is a government owned able to realize an income of only Seventy One Thousand One Hundred Eighty Five Pesos (P
71,185.00), and further, by waiving RPN-9’s collectible from OTMCI for August 1-30, 1998 in the Section 2 (13) of Executive Order No. 292 (Administrative Code of 1987)26 renders a similar
amount of Three Hundred Twenty Thousand and Forty Five Pesos (P 320,045.00). definition of government-owned or -controlled corporations:

Carandang moved to quash the information,18 arguing that Sandiganbayan had no jurisdiction Section 2. General Terms Defined. – Unless the specific words of the text or the context as a
because he was not a public official due to RPN not being a government-owned or -controlled whole or a particular statute, shall require a different meaning:
corporation.
xxx
The Sandiganbayan denied Carandang’s motion to quash on October 17, 2001. 19
(13) government-owned or controlled corporations refer to any agency organized as a stock or
After the denial by the Sandiganbayan of his motion for reconsideration, 20 Carandang initiated non-stock corporation vested with functions relating to public needs whether governmental or
G.R. No. 153161.21 proprietary in nature, and owned by the government directly or indirectly through its
instrumentalities either wholly, or where applicable as in the case of stock corporations to the
extent of at least 51% of its capital stock.
On May 27, 2002, Carandang moved to defer his arraignment and pre-trial, citing the pendency
of G.R. No. 153161.22
It is clear, therefore, that a corporation is considered a government-owned or -controlled
corporation only when the Government directly or indirectly owns or controls at least a majority
On July 29, 2002, the Court directed the parties in G.R. No. 153161 to maintain the status quo
or 51% share of the capital stock. Applying this statutory criterion, the Court ruled in Leyson, Jr.
until further orders.23
v. Office of the Ombudsman:27

On November 20, 2006, G.R. No. 148076 was consolidated with G.R. No. 153161. 24
But these jurisprudential rules invoked by petitioner in support of his claim that the CIIF
companies are government owned and/or controlled corporations are incomplete without
Issue resorting to the definition of "government owned or controlled corporation" contained in par. (13),
Sec.2, Introductory Provisions of the Administrative Code of 1987, i.e., any agency organized as
a stock or non-stock corporation vested with functions relating to public needs whether
Carandang insists that he was not a public official considering that RPN was not a government- governmental or proprietary in nature, and owned by the government directly or indirectly
owned or -controlled corporation; and that, consequently, the Ombudsman and the
through its instrumentalities either wholly, or where applicable as in the case of stock
Sandiganbayan had no jurisdiction over him. He prays that the administrative and criminal corporations to the extent of at least fifty-one (51) percent of its capital stock. The definition
complaints filed against him should be dismissed. Accordingly, decisive is whether or not RPN mentions three (3) requisites, namely, first, any agency organized as a stock or non-stock
was a government-owned or -controlled corporation.
corporation; second, vested with functions relating to public needs whether governmental or
proprietary in nature; and, third, owned by the Government directly or through its
Ruling instrumentalities either wholly, or, where applicable as in the case of stock corporations, to the
extent of at least fifty-one (51) of its capital stock.
We find the petitions to be meritorious.
In the present case, all three (3) corporations comprising the CIIF companies were organized as
stock corporations. The UCPB-CIIF owns 44.10% of the shares of LEGASPI OIL, xxx.
It is not disputed that the Ombudsman has jurisdiction over administrative cases involving grave Obviously, the below 51% shares of stock in LEGASPI OIL removes this firm from the
misconduct committed by the officials and employees of government-owned or -controlled definition of a government owned or controlled corporation. x x x The Court thus concludes
corporations; and that the Sandiganbayan has jurisdiction to try and decide criminal actions that the CIIF are, as found by public respondent, private corporations not within the scope of its
involving violations of R.A. 3019 committed by public officials and employees, including jurisdiction.28
presidents, directors and managers of government-owned or -controlled corporations. The
respective jurisdictions of the respondents are expressly defined and delineated by the law. 25
Consequently, RPN was neither a government-owned nor a controlled corporation because of
the Government’s total share in RPN’s capital stock being only 32.4%.
Similarly, the law defines what are government-owned or -controlled corporations. For one,
Section 2 of Presidential Decree No. 2029 (Defining Government Owned or Controlled
Corporations and Identifying Their Role in National Development) states: Parenthetically, although it is true that the Sandiganbayan (Second Division) ordered the transfer
to the PCGG of Benedicto’s shares that represented 72.4% of the total issued and outstanding
capital stock of RPN, such quantification of Benedicto’s shareholding cannot be controlling in
Section 2. A government-owned or controlled corporation is a stock or a non-stock corporation, view of Benedicto’s timely filing of a motion for reconsideration whereby he clarified and insisted
whether performing governmental or proprietary functions, which is directly chartered by a that the shares ceded to the PCGG had accounted for only 32.4%, not 72.4%, of RPN’s
special law or if organized under the general corporation law is owned or controlled by the outstanding capital stock. With the extent of Benedicto’s holdings in RPN remaining unresolved
government directly, or indirectly through a parent corporation or subsidiary corporation, to the with finality, concluding that the Government held the majority of RPN’s capital stock as to make
extent of at least a majority of its outstanding capital stock or of its outstanding voting capital RPN a government-owned or -controlled corporation would be bereft of any factual and legal
stock. basis.
Even the PCGG and the Office of the President (OP) have recognized RPN’s status as being "Sec.2. Definition. – A government owned- or controlled corporation is a stock or non-stock
neither a government-owned nor -controlled corporation. corporation, whether performing governmental or proprietary functions which is directly
chartered by special law or organized under the general corporation law is owned or controlled
by the government directly, or indirectly through a parent corporation or subsidiary corporation,
In its Opinion/Clarification dated August 18, 1999, the PCGG communicated to San Luis as the
to the extent of at least a majority of its outstanding capital stock or of its outstanding voting
president and general manager of RPN regarding a case involving RPN and Carandang: 29
capital stock;

MR. EDGAR S. SAN LUIS


Provided, that a corporation organized under the general corporation law under private
President & General Manager
ownership at least a majority of the shares of stock of which were conveyed to a government
Radio Philippines Network, Inc.
corporation in satisfaction of debts incurred with a government financial institution, whether by
Broadcast City, Capitol Hills
foreclosure or otherwise, or a subsidiary corporation of a government corporation organized
Diliman, Quezon City
exclusively to own and manage, or lease, or operate specific physical assets acquired by a
government financial institution in satisfaction of debts incurred therewith, and which in any case
Sir: by enunciated policy of the government is required to be disposed of to private ownership within
a specified period of time, shall not be considered a government-owned or controlled
corporation before such disposition and even if the ownership or control thereof is subsequently
This refers to your letter dated August 4, 1999, seeking "PCGG’s position
transferred to another government-owned or controlled corporation."
on the following:

A government-owned or controlled corporation is either "parent" corporation, i.e., one "created


"1. Whether RPN-9 is a GOCC x x x or a private corporation outside the by special law" (Sec. 3 (a), PD 2029) or a "subsidiary" corporation, i.e, one created pursuant to
scope of OGCC and COA’s control given 32% Government ownership x x x. law where at least a majority of the outstanding voting capital stock of which is owned by parent
government corporation and/or other government-owned subsidiaries. (Sec. 3 (b), PD 2029).
xxx
RPN-9 may not likewise be considered as an "acquired asset corporation" which is one
It appears that under the RP-Benedicto Compromise Agreement dated November 3, 1990 – organized under the general corporation law (1) under private ownership at least a majority of
validity of which has been sustained by the Supreme Court in G.R. No. 96087, March 31, 1992, the shares of stock of which were conveyed to a government corporation in satisfaction of debts
(Guingona, Jr. vs. PCGG, 207 SCRA 659) – Benedicto ceded all his rights, interest and/or incurred with a government financial institution, whether by foreclosure or otherwise, or (2) as a
participation, if he has any, in RPN-9, among others, to the government which rights, interest subsidiary corporation of a government corporation organized exclusively to own and manage,
and/or participation per PCGG’s understanding, include 9,494,327.50 shares of stock, i.e, about or lease, or operate specific physical assets acquired by a government financial institution in
72.4% of the total issued and outstanding capital stock of RPN-9. satisfaction of debts incurred therewith, and which in any case by enunciated policy of the
government is required to be disposed of to private ownership within a specified period of time"
(Sec 3 c, PD 2029), for the following reasons:
Accordingly, the Sandiganbayan (Second Division), on motion of the government through
PCGG, ordered the president and corporate secretary of the RPN-9 to "effect the immediate
cancellation and transfer of the 9,494,327.50 shares corresponding to Benedicto’s proprietary 1. as noted above, the uncontested (not litigated) RPN-9 shares of the government is only
interest in RPN-9 to the Republic of the Philippines c/o PCGG" (Sandiganbayan’s Resolution of 32.4% (not a majority) of its capital stock;
February 3, 1998 in Civil Case No. 0034, RP vs. Roberto Benedicto, et. al.) Benedicto, however,
filed a motion for reconsideration of said Resolution, contending that the number of RPN-9
2. said 32.4% shares of stock, together with the contested/litigated 40%, were not conveyed to a
shares ceded by him embraces only his personal holdings and those of his immediate family and government corporation or the government "in satisfaction of debts incurred with government
nominees totaling 4,161,207.5 shares but excluding the RPN-9 shares in the name of Far East financial institution, whether by foreclosure or otherwise;
Managers and Investors, Inc. ("FEMIE"), which is about 40%, as they are corporate
properties/assets of FEMIE and not his personal holdings. Said motion for reconsideration is still
pending resolution by the Sandiganbayan. 3. RPN-9 was not organized as a subsidiary corporation of a government corporation organized
exclusively to own and manage, or lease, or operate specific physical assets acquired by a
government financial institution in satisfaction of debts incurred therewith.
xxx

It should be parenthetically noted that the 32.4% or 72.4% shares of stocks were turned over to
We agree with your x x x view that RPN-9 is not a government owned or controlled
the government by virtue of a compromise agreement between the government and Benedicto in
corporation within the contemplation of the Administrative Code of 1987, for admittedly,
Civil Case No. 0034 which is "a civil action against Defendants Roberto S. Benedicto, Ferdinand
RPN-9 was organized for private needs and profits, and not for public needs and was not
E. Marcos, Imelda R. Marcos" and others, to recover from them ill-gotten wealth" (Amended
specifically vested with functions relating to public needs.
Complaint, Aug. 12, 1987, Civil Case No. 0034, p. 2.) As the case between the government and
Benedicto, his family and nominees was compromised, no judicial pronouncement was made as
Neither could RPN-9 be considered a "government-owned or controlled corporation" to the character or nature of the assets and properties turned over by Benedicto to the
under Presidential Decree (PD) No. 2029 dated February 4, 1986, which defines said terms government – whether they are ill-gotten wealth or not.30
as follows:
The PCGG’s Opinion/Clarification was affirmed by the OP itself on February 10, 2000: 31
WHEREFORE, we grant the petitions in G.R. No. 148076 and G.R. No. 153161.

February 10, 2000 We reverse and set aside the decision promulgated on February 12, 2001 by the Court of
Appeals in C.A.-G.R. SP No. 58204, and dismiss the administrative charge for grave misconduct
against the petitioner.
Mr. Edgar S. San Luis
President and General Manager
Radio Philippines Network Inc. We annul and set aside the resolutions dated October 17, 2001 and March 14, 2002, as well as
Broadcasting City, Capitol Hills, Diliman Quezon City the order dated March 15, 2002, all issued by the Sandiganbayan (Fifth Division) in Criminal
Case No. 25802, and dismiss Criminal Case No. 25802 as against the petitioner.
Dear President San Luis,
SO ORDERED.
xxx

Relative thereto, please be informed that we affirm the PCGG’s opinion


that RPNI is not a government-owned and/or controlled corporation
(GOCC). Section 2 (13), Introductory Provisions of the Administrative Code
of 1987 defines a GOCC as an agency organized as a stock or non-stock
corporation vested with functions relating to public needs whether
governmental or proprietary in nature, and owned by the government
directly or indirectly through its instrumentalities either wholly, or where
applicable as in the case of stock corporations to the extent of at least 51%
of its capital stock. As government ownership over RPNI is only 32.4%
of its capital stock, pending the final judicial determination of the true
and legal ownership of RPNI, the corporation is deemed private.32

Even earlier, a similar construction impelled the Ombudsman to dismiss a criminal complaint for
violation of R.A. 3019 filed against certain

RPN officials, as the Ombudsman’s resolution dated December 15, 1997 indicates, 33 a pertinent
portion of which is quoted thus:

This is not to mention the fact that the other respondents, the RPN officials, are outside the
jurisdiction of this Office (Office of the Ombudsman); they are employed by a private corporation
registered with the Securities and Exchange Commission, the RPN, which is not a
government owned or controlled corporation x x x34

Considering that the construction of a statute given by administrative agencies deserves


respect,35 the uniform administrative constructions of the relevant aforequoted laws defining
what are government-owned or -controlled corporations as applied to RPN is highly persuasive.

Lastly, the conclusion that Carandang was a public official by virtue of his having been appointed
as general manager and chief operating officer of RPN by President Estrada deserves no
consideration. President Estrada’s intervention was merely to recommend Carandang’s
designation as general manager and chief operating officer of RPN to the PCGG, which then
cast the vote in his favor vis-à-vis said positions.36 Under the circumstances, it was RPN’s Board
of Directors that appointed Carandang to his positions pursuant to RPN’s By-Laws.371avvphi1

In fine, Carandang was correct in insisting that being a private individual he was not subject to
the administrative authority of the Ombudsman and to the criminal jurisdiction of the
Sandiganbayan.38
why such an argument is far from persuasive. "The premise that the funds could be spoken as
public character may be accepted in the sense that the People Homesite and Housing
Corporation was a government-owned entity. It does not follow though that they were exempt.
G.R. No. L-33112 June 15, 1978 from garnishment. National Shipyard and Steel Corporation v. Court of Industrial Relations is
squarely in point. As was explicitly stated in the opinion of the then Justice, later Chief Justice,
PHILIPPINE NATIONAL BANK, petitioner, Concepcion: "The allegation to the effect that the funds of the NASSCO are public funds of the
vs. government, and that, as such, the same may not be garnished, attached or levied upon, is
HON. JUDGE JAVIER PABALAN, Judge of the Court of First Instance, Branch III, La untenable for, as a government owned and controlled corporation, the NASSCO has a
Union, AGOO TOBACCO PLANTERS ASSOCIATION, INC., PHILIPPINE VIRGINIA personality of its own. distinct and separate from that of the Government. It has — pursuant to
TOBACCO ADMINISTRATION, and PANFILO P. JIMENEZ, Deputy Sheriff, La Section 2 of Executive Order No. 356, dated October 23, 1950 ... , pursuant to which The
Union, respondents. NASSCO has been established — all the powers of a corporation under the Corporation Law ...
." Accordingly, it may be sue and be sued and may be subjected to court processes just like any
other corporation (Section 13, Act No. 1459, as amended.)" ... To repeat, the ruling was the
FERNANDO, Acting C.J.: appropriate remedy for the prevailing party which could proceed against the funds of a corporate
entity even if owned or controlled by the government." 12
The reliance of petitioner Philippine National Bank in this certiorari and prohibition proceeding
against respondent Judge Javier Pabalan who issued a writ of execution, 1 followed thereafter 2. The National Shipyard and Steel Corporation decision was not the first of its kind. The ruling
by a notice of garnishment of the funds of respondent Philippine Virginia Tobacco therein could be inferred from the judgment announced in Manila Hotel Employees Association
Administration, 2 deposited with it, is on the fundamental constitutional law doctrine of non- v. Manila Hotel Company, decided as far back as 1941. 13 In the language of its ponente Justice
suability of a state, it being alleged that such funds are public in character. This is not the first Ozaeta "On the other hand, it is well-settled that when the government enters into commercial
time petitioner raised that issue. It did so before in Philippine National Bank v. Court of industrial business, it abandons its sovereign capacity and is to be treated like any other corporation.
Relations, 3 decided only last January. It did not meet with success, this Court ruling in (Bank of the United States v. Planters' Bank, 9 Wheat. 904, 6 L.ed. 244). By engaging in a
accordance with the two previous cases of National Shipyard and Steel Corporation 4 and particular business thru the instrumentality of a corporation, the government divests itself pro
Manila Hotel Employees Association v. Manila Hotel Company,5 that funds of public corporations hac vice of its sovereign character, so as to render the corporation subject to the rules of law
which can sue and be sued were not exempt from garnishment. As respondent Philippine governing private corporations." 14 It is worth mentioning that Justice Ozaeta could find support
Virginia Tobacco Administration is likewise a public corporation possessed of the same for such a pronouncement from the leading American Supreme Court case of united States v.
attributes,6 a similar outcome is indicated. This petition must be dismissed. Planters' Bank, 15 with the opinion coming from the illustrious Chief Justice Marshall. It was
handed down more than one hundred fifty years ago, 1824 to be exact. It is apparent, therefore,
It is undisputed that the judgment against respondent Philippine Virginia Tobacco Administration that petitioner Bank could it legally set forth as a bar or impediment to a notice of garnishment
had reached the stage of finality. A writ of execution was, therefore, in order. It was accordingly the doctrine of non-suability.
issued on December 17, 1970. 7There was a notice of garnishment for the full amount
mentioned in such writ of execution in the sum of P12,724,66. 8 In view of the objection, WHEREFORE, this petition for certiorari and prohibition is dismissed. No costs.
however, by petitioner Philippine National Bank on the above ground, coupled with an inquiry as
to whether or not respondent Philippine Virginia Tobacco Administration had funds deposited
with petitioner's La Union branch, it was not until January 25, 1971 that the order sought to be
set aside in this certiorari proceeding was issued by respondent Judge. 9 Its dispositive portion
reads as follows: Conformably with the foregoing, it is now ordered, in accordance with law, that
sufficient funds of the Philippine Virginia Tobacco Administration now deposited with the
Philippine National Bank, La Union Branch, shall be garnished and delivered to the plaintiff
immediately to satisfy the Writ of Execution for one-half of the amount awarded in the decision of
November 16, 1970." 10 Hence this certiorari and prohibition proceeding.

As noted at the outset, petitioner Philippine National Bank would invoke the doctrine of non-
suability. It is to be admitted that under the present Constitution, what was formerly implicit as a
fundamental doctrine in constitutional law has been set forth in express terms: "The State may
not be sued without its consent." 11 If the funds appertained to one of the regular departments or
offices in the government, then, certainly, such a provision would be a bar to garnishment. Such
is not the case here. Garnishment would lie. Only last January, as noted in the opening
paragraph of this decision, this Court, in a case brought by the same petitioner precisely
invoking such a doctrine, left no doubt that the funds of public corporations could properly be
made the object of a notice of garnishment. Accordingly, this petition must fail.

1. The alleged grave abuse of discretion, the basis of this certiorari proceeding, was sought to
be justified on the failure of respondent Judge to set aside the notice of garnishment of funds
belonging to respondent Philippine Virginia Tobacco Administration. This excerpt from the
aforecited decision of Philippine National Bank v. Court of Industrial Relations makes manifest
G.R. No. L-22619 December 2, 1924 of developing coal deposits in the Philippine Island and of mining, extracting, transporting and
selling the coal contained in said deposits." (Sec. 2, Act No. 2705.) By the same law (Act No.
2705) the Government of the Philippine Islands is made the majority stockholder, evidently in
NATIONAL COAL COMPANY, plaintiff-appellee,
order to insure proper government supervision and control, and thus to place the Government in
vs.
a position to render all possible encouragement, assistance and help in the prosecution and
THE COLLECTOR OF INTERNAL REVENUE, defendant-appellant.
furtherance of the company's business.

JOHNSON, J.:
On May 14, 1917, two months after the passage of Act No. 2705, creating the National Coal
Company, the Philippine Legislature passed Act No. 2719 "to provide for the leasing and
This action was brought in the Court of First Instance of the City of Manila on the 17th day of development of coal lands in the Philippine Islands." On October 18, 1917, upon petition of the
July, 1923, for the purpose of recovering the sum of P12,044.68, alleged to have been paid National Coal Company, the Governor-General, by Proclamation No. 39, withdrew "from
under protest by the plaintiff company to the defendant, as specific tax on 24,089.3 tons of coal. settlement, entry, sale or other disposition, all coal-bearing public lands within the Province of
Said company is a corporation created by Act No. 2705 of the Philippine Legislature for the Zamboanga, Department of Mindanao and Sulu, and the Island of Polillo, Province of Tayabas."
purpose of developing the coal industry in the Philippine Islands and is actually engaged in coal Almost immediately after the issuance of said proclamation the National Coal Company took
mining on reserved lands belonging to the Government. It claimed exemption from taxes under possession of the coal lands within the said reservation, with an area of about 400 hectares,
the provision of sections 14 and 15 of Act No. 2719, and prayed for a judgment ordering the without any further formality, contract or lease. Of the 30,000 shares of stock issued by the
defendant to refund to the plaintiff said sum of P12,044.68, with legal interest from the date of company, the Government of the Philippine Islands is the owner of 29,809 shares, that is, of 99
the presentation of the complaint, and costs against the defendant. 1/3 per centum of the whole capital stock.

The defendant answered denying generally and specifically all the material allegations of the If we understand the theory of the plaintiff-appellee, it is, that it claims to be the owner of the land
complaint, except the legal existence and personality of the plaintiff. As a special defense, the from which it has mined the coal in question and is therefore subject to the provisions of section
defendant alleged (a) that the sum of P12,044.68 was paid by the plaintiff without protests, and 15 of Act No. 2719 and not to the provisions of the section 1496 of the Administrative Code. That
(b) that said sum was due and owing from the plaintiff to the Government of the Philippine contention of the plaintiff leads us to an examination of the evidence upon the question of the
Islands under the provisions of section 1496 of the Administrative Code and prayed that the ownership of the land from which the coal in question was mined. Was the plaintiff the owner of
complaint be dismissed, with costs against the plaintiff. the land from which the coal in question was mined? If the evidence shows the affirmative, then
the judgment should be affirmed. If the evidence shows that the land does not belong to the
plaintiff, then the judgment should be reversed, unless the plaintiff's rights fall under section 3 of
Upon the issue thus presented, the case was brought on for trial. After a consideration of the said Act.
evidence adduced by both parties, the Honorable Pedro Conception, judge, held that the words
"lands owned by any person, etc.," in section 15 of Act No. 2719 should be understood to mean
"lands held in lease or usufruct," in harmony with the other provision of said Act; that the coal The only witness presented by the plaintiff upon the question of the ownership of the land in
lands possessed by the plaintiff, belonging to the Government, fell within the provisions of question was Mr. Dalmacio Costas, who stated that he was a member of the board of directors
section 15 of Act No. 2719; and that a tax of P0.04 per ton of 1,016 kilos on each ton of coal of the plaintiff corporation; that the plaintiff corporation took possession of the land in question by
extracted therefrom, as provided in said section, was the only tax which should be collected from virtue of the proclamation of the Governor-General, known as Proclamation No. 39 of the year
the plaintiff; and sentenced the defendant to refund to the plaintiff the sum of P11,081.11 which 1917; that no document had been issued in favor of the plaintiff corporation; that said
is the difference between the amount collected under section 1496 of the Administrative Code corporation had received no permission from the Secretary of Agriculture and Natural
and the amount which should have been collected under the provisions of said section 15 of Act Resources; that it took possession of said lands covering an area of about 400 hectares, from
No. 2719. From that sentence the defendant appealed, and now makes the following which the coal in question was mined, solely, by virtue of said proclamation (Exhibit B, No. 39).
assignments of error:
Said proclamation (Exhibit B) was issued by Francis Burton Harrison, then Governor-General,
I. The court below erred in holding that section 15 of Act No. 2719 does not refer to coal lands on the 18th day of October, 1917, and provided: "Pursuant to the provision of section 71 of Act
owned by persons and corporations. No. 926, I hereby withdraw from settlement, entry, sale, or other disposition, all coal-bearing
public lands within the Province of Zamboanga, Department of Mindanao and Sulu, and the
Island of Polillo, Province of Tayabas." It will be noted that said proclamation only provided that
II. The court below erred in holding that the plaintiff was not subject to the tax prescribed in all coal-bearing public lands within said province and island should be withdrawn from
section 1496 of the Administrative Code. settlement, entry, sale, or other disposition. There is nothing in said proclamation which
authorizes the plaintiff or any other person to enter upon said reversations and to mine coal, and
The question confronting us in this appeal is whether the plaintiff is subject to the taxes under no provision of law has been called to our attention, by virtue of which the plaintiff was entitled to
section 15 of Act No. 2719, or to the specific taxes under section 1496 of the Administrative enter upon any of the lands so reserved by said proclamation without first obtaining permission
Code. therefor.

The plaintiff corporation was created on the 10th day of March, 1917, by Act No. 2705, for the The plaintiff is a private corporation. The mere fact that the Government happens to the majority
purpose of developing the coal industry in the Philippine Island, in harmony with the general plan stockholder does not make it a public corporation. Act No. 2705, as amended by Act No. 2822,
of the Government to encourage the development of the natural resources of the country, and to makes it subject to all of the provisions of the Corporation Law, in so far as they are not
provided facilities therefor. By said Act, the company was granted the general powers of a inconsistent with said Act (No. 2705). No provisions of Act No. 2705 are found to be inconsistent
corporation "and such other powers as may be necessary to enable it to prosecute the business with the provisions of the Corporation Law. As a private corporation, it has no greater rights,
powers or privileges than any other corporation which might be organized for the same purpose section 1496 of the Administrative Code. That being the issue, an examination of the provisions
under the Corporation Law, and certainly it was not the intention of the Legislature to give it a of Act No. 2719 becomes necessary.
preference or right or privilege over other legitimate private corporations in the mining of coal.
While it is true that said proclamation No. 39 withdrew "from settlement, entry, sale, or other
An examination of said Act (No. 2719) discloses the following facts important for consideration
disposition of coal-bearing public lands within the Province of Zamboanga . . . and the Island of
here:
Polillo," it made no provision for the occupation and operation by the plaintiff, to the exclusion of
other persons or corporations who might, under proper permission, enter upon the operate coal
mines. First. All "coal-bearing lands of the public domain in the Philippine Islands shall not be disposed
of in any manner except as provided in this Act." Second. Provisions for leasing by the Secretary
of Agriculture and Natural Resources of "unreserved, unappropriated coal-bearing public lands,"
On the 14th day of May, 1917, and before the issuance of said proclamation, the Legislature of
and the obligation to the Government which shall be imposed by said Secretary upon the
the Philippine Island in "an Act for the leasing and development of coal lands in the Philippine
lessee.lawphi1.net
Islands" (Act No. 2719), made liberal provision. Section 1 of said Act provides: "Coal-bearing
lands of the public domain in the Philippine Island shall not be disposed of in any manner except
as provided in this Act," thereby giving a clear indication that no "coal-bearing lands of the public Third. The internal revenue duty and tax which must be paid upon coal-bearing lands owned by
domain" had been disposed of by virtue of said proclamation. any person, firm, association or corporation.

Neither is there any provision in Act No. 2705 creating the National Coal Company, nor in the To repeat, it will be noted, first, that Act No. 2719 provides an internal revenue duty and tax upon
amendments thereof found in Act No. 2822, which authorizes the National Coal Company to unreserved, unappropriated coal-bearing public lands which may be leased by the Secretary of
enter upon any of the reserved coal lands without first having obtained permission from the Agriculture and Natural Resources; and, second, that said Act (No. 2719) provides an internal
Secretary of Agriculture and Natural Resources.lawphi1.net revenue duty and tax imposed upon any person, firm, association or corporation, who may be
the owner of "coal-bearing lands." A reading of said Act clearly shows that the tax imposed
thereby is imposed upon two classes of persons only — lessees and owners.
The following propositions are fully sustained by the facts and the law:

The lower court had some trouble in determining what was the correct interpretation of section
(1) The National Coal Company is an ordinary private corporation organized under Act No. 2705,
15 of said Act, by reason of what he believed to be some difference in the interpretation of the
and has no greater powers nor privileges than the ordinary private corporation, except those
language used in Spanish and English. While there is some ground for confusion in the use of
mentioned, perhaps, in section 10 of Act No. 2719, and they do not change the situation here.
the language in Spanish and English, we are persuaded, considering all the provisions of said
Act, that said section 15 has reference only to persons, firms, associations or corporations which
(2) It mined on public lands between the month of July, 1920, and the months of March, 1922, had already, prior to the existence of said Act, become the owners of coal lands. Section 15
24,089.3 tons of coal. cannot certainty refer to "holders or lessees of coal lands' for the reason that practically all of the
other provisions of said Act has reference to lessees or holders. If section 15 means that the
persons, firms, associations, or corporation mentioned therein are holders or lessees of coal
(3) Upon demand of the Collector of Internal Revenue it paid a tax of P0.50 a ton, as taxes
lands only, it is difficult to understand why the internal revenue duty and tax in said section was
under the provisions of article 1946 of the Administrative Code on the 15th day of December,
made different from the obligations mentioned in section 3 of said Act, imposed upon lessees or
1922.
holders.

(4) It is admitted that it is neither the owner nor the lessee of the lands upon which said coal was
From all of the foregoing, it seems to be made plain that the plaintiff is neither a lessee nor an
mined.
owner of coal-bearing lands, and is, therefore, not subject to any other provisions of Act No.
2719. But, is the plaintiff subject to the provisions of section 1496 of the Administrative Code?
(5) The proclamation of Francis Burton Harrison, Governor-General, of the 18th day of October,
1917, by authority of section 1 of Act No. 926, withdrawing from settlement, entry, sale, or other
Section 1496 of the Administrative Code provides that "on all coal and coke there shall be
dispositon all coal-bearing public lands within the Province of Zamboanga and the Island of
collected, per metric ton, fifty centavos." Said section (1496) is a part of article, 6 which provides
Polillo, was not a reservation for the benefit of the National Coal Company, but for any person or
for specific taxes. Said article provides for a specific internal revenue tax upon all things
corporation of the Philippine Islands or of the United States.
manufactured or produced in the Philippine Islands for domestic sale or consumption, and upon
things imported from the United States or foreign countries. It having been demonstrated that the
(6) That the National Coal Company entered upon said land and mined said coal, so far as the plaintiff has produced coal in the Philippine Islands and is not a lessee or owner of the land from
record shows, without any lease or other authority from either the Secretary of Agriculture and which the coal was produced, we are clearly of the opinion, and so hold, that it is subject to pay
Natural Resources or any person having the power to grant a leave or authority. the internal revenue tax under the provisions of section 1496 of the Administrative Code, and is
not subject to the payment of the internal revenue tax under section 15 of Act No. 2719, nor to
any other provisions of said Act.
From all of the foregoing facts we find that the issue is well defined between the plaintiff and the
defendant. The plaintiff contends that it was liable only to pay the internal revenue and other
fees and taxes provided for under section 15 of Act No. 2719; while the defendant contends, Therefore, the judgment appealed from is hereby revoked, and the defendant is hereby relieved
under the facts of record, the plaintiff is obliged to pay the internal revenue duty provided for in from all responsibility under the complaint. And, without any finding as to costs, it is so ordered.
G.R. No. 79182 September 11, 1991 2. On June 15, 1985, Danilo Mercado went on vacation leave without prior leave,
against company policy, rules and regulations. (Petitioner's Memorandum, Rollo, p.
195).
PNOC-ENERGY DEVELOPMENT CORPORATION, petitioner,
vs.
NATIONAL LABOR RELATIONS COMMISSION (Third Division) and DANILO On September 23, 1985, private respondent Mercado filed a complaint for illegal dismissal,
MERCADO, respondents. retirement benefits, separation pay, unpaid wages, etc. against petitioner PNOC-EDC before the
NLRC Regional Arbitration Branch No. VII docketed as Case No. RAB-VII-0556-85.
PARAS, J.:
After private respondent Mercado filed his position paper on December 16, 1985 (Annex "B" of
the Petition, Rollo, pp. 28-40), petitioner PNOC-EDC filed its Position Paper/Motion to Dismiss
This is a petition for certiorari to set aside the Resolution * dated July 3, 1987 of respondent
on January 15, 1986, praying for the dismissal of the case on the ground that the Labor Arbiter
National Labor Relations Commission (NLRC for brevity) which affirmed the decision dated April
and/or the NLRC had no jurisdiction over the case (Annex "C" of the Petition, Rollo, pp. 41-45),
30, 1986 of Labor Arbiter Vito J. Minoria of the NLRC, Regional Arbitration Branch No. VII at
which was assailed by private respondent Mercado in his Opposition to the Position
Cebu City in Case No. RAB-VII-0556-85 entitled "Danilo Mercado, Complainant, vs. Philippine
Paper/Motion to Dismiss dated March 12, 1986 (Annex "D" of the Petition, Rollo, pp. 46-50).
National Oil Company-Energy Development Corporation, Respondent", ordering the
reinstatement of complainant Danilo Mercado and the award of various monetary claims.
The Labor Arbiter ruled in favor of private respondent Mercado. The dispositive onion of said
decision reads as follows:
The factual background of this case is as follows:

WHEREFORE, in view of the foregoing, respondents are hereby ordered:


Private respondent Danilo Mercado was first employed by herein petitioner Philippine National
Oil Company-Energy Development Corporation (PNOC-EDC for brevity) on August 13, 1979. He
held various positions ranging from clerk, general clerk to shipping clerk during his employment 1) To reinstate complainant to his former position with full back wages from the date of
at its Cebu office until his transfer to its establishment at Palimpinon, Dumaguete, Oriental his dismissal up to the time of his actual reinstatement without loss of seniority rights
Negros on September 5, 1984. On June 30, 1985, private respondent Mercado was dismissed. and other privileges;
His last salary was P1,585.00 a month basic pay plus P800.00 living allowance (Labor Arbiter's
Decision, Annex "E" of Petition, Rollo, p. 52).
2) To pay complainant the amount of P10,000.00 representing his personal share of
his savings account with the respondents;
The grounds for the dismissal of Mercado are allegedly serious acts of dishonesty committed as
follows:
3) To pay complainants the amount of P30,000.00 moral damages; P20,000.00
exemplary damages and P5,000.00 attorney's fees;
1. On ApriI 12, 1985, Danilo Mercado was ordered to purchase 1,400 pieces of nipa
shingles from Mrs. Leonardo Nodado of Banilad, Dumaguete City, for the total
4) To pay complainant the amount of P792.50 as his proportionate 13th month pay for
purchase price of Pl,680.00. Against company policy, regulations and specific orders,
1985.
Danilo Mercado withdrew the nipa shingles from the supplier but paid the amount of
P1,000.00 only. Danilo Mercado appropriated the balance of P680.00 for his personal
use; Respondents are hereby further ordered to deposit the aforementioned amounts with
this Office within ten days from receipt of a copy of this decision for further disposition.
2. In the same transaction stated above, the supplier agreed to give the company a
discount of P70.00 which Danilo Mercado did not report to the company; SO ORDERED.
(Labor Arbiter's Decision, Rollo, p. 56)
3. On March 28, 1985, Danilo Mercado was instructed to contract the services of Fred
R. Melon of Dumaguete City, for the fabrication of rubber stamps, for the total amount The appeal to the NLRC was dismissed for lack of merit on July 3, 1987 and the assailed
of P28.66. Danilo Mercado paid the amount of P20.00 to Fred R. Melon and decision was affirmed.
appropriated for his personal use the balance of P8.66.
Hence, this petition.
In addition, private respondent, Danilo Mercado violated company rules and
regulations in the following instances:
The issues raised by petitioner in this instant petition are:

1. On June 5, 1985, Danilo Mercado was absent from work without leave, without
1. Whether or not matters of employment affecting the PNOC-EDC, a government-
proper turn-over of his work, causing disruption and delay of company work activities;
owned and controlled corporation, are within the jurisdiction of the Labor Arbiter and
the NLRC.
2. Assuming the affirmative, whether or not the Labor Arbiter and the NLRC are As regards the second issue, the record shows that PNOC-EDC's accusations of dishonesty and
justified in ordering the reinstatement of private respondent, payment of his savings, violations of company rules are not supported by evidence. Nonetheless, while acknowledging
and proportionate 13th month pay and payment of damages as well as attorney's fee. the rule that administrative bodies are not governed by the strict rules of evidence, petitioner
PNOC-EDC alleges that the labor arbiter's propensity to decide the case through the position
papers submitted by the parties is violative of due process thereby rendering the decision null
Petitioner PNOC-EDC alleges that it is a corporation wholly owned and controlled by the
and void (Ibid., p. 196).
government; that the Energy Development Corporation is a subsidiary of the Philippine National
Oil Company which is a government entity created under Presidential Decree No. 334, as
amended; that being a government-owned and controlled corporation, it is governed by the Civil On the other hand, private respondent contends that as can be seen from petitioner's Motion for
Service Law as provided for in Section 1, Article XII-B of the 1973 Constitution, Section 56 of Reconsideration and/or Appeal dated July 28, 1986 (Annex "F" of the Petition, Rollo, pp. 57- 64),
Presidential Decree No. 807 (Civil Service Decree) and Article 277 of Presidential Decree No. the latter never questioned the findings of facts of the Labor Arbiter but simply limited its
442, as amended (Labor Code). objection to the lack of legal basis in view of its stand that the NLRC had no jurisdiction over the
case (Private Respondent's Memorandum, Rollo, p. 104).
The 1973 Constitution provides:
Petitioner PNOC-EDC filed its Position Paper/Motion to Dismiss dated January 15, 1986 (Annex
"C" of the Petition Rollo, pp. 41-45) before the Regional Arbitration Branch No. VII of Cebu City
The Civil Service embraces every branch, agency, subdivision and instrumentality of
and its Motion for Reconsideration and/or Appeal dated July 28, 1986 (Annex "F" of the Petition,
the government including government-owned or controlled corporations.
Rollo, pp. 57-64) before the NLRC of Cebu City. Indisputably, the requirements of due process
are satisfied when the parties are given an opportunity to submit position papers. What the
Petitioner PNOC-EDC argued that since Labor Arbiter Minoria rendered the decision at the time fundamental law abhors is not the absence of previous notice but rather the absolute lack of
when the 1973 Constitution was in force, said decision is null and void because under the 1973 opportunity to ventilate a party's side. There is no denial of due process where the party
Constitution, government-owned and controlled corporations were governed by the Civil Service submitted its position paper and flied its motion for reconsideration (Odin Security Agency vs. De
Law. Even assuming that PNOC-EDC has no original or special charter and Section 2(i), Article la Serna, 182 SCRA 472 [February 21, 1990]). Petitioner's subsequent Motion for
IX-B of the 1987 Constitution provides that: Reconsideration and/or Appeal has the effect of curing whatever irregularity might have been
committed in the proceedings below (T.H. Valderama and Sons, Inc. vs. Drilon, 181 SCRA 308
[January 22, 1990]).
The Civil Service embraces all branches, subdivision, instrumentalities and agencies
of the Government, including government-owned or controlled corporations with
original charters. Furthermore, it has been consistently held that findings of administrative agencies which have
acquired expertise because their jurisdiction is confined to specific matters are accorded not only
respect but even finality (Asian Construction and Development Corporation vs. NLRC, 187
such circumstances cannot give validity to the decision of the Labor Arbiter (Ibid., pp. 192-193). SCRA 784 [July 27, 1990]; Lopez Sugar Corporation vs. Federation of Free Workers, 189 SCRA
179 [August 30, 1990]). Judicial review by this Court does not go so far as to evaluate the
This issue has already been laid to rest in the case of PNOC-EDC vs. Leogardo, 175 SCRA 26 sufficiency of the evidence but is limited to issues of jurisdiction or grave abuse of discretion
(July 5, 1989), involving the same petitioner and the same issue, where this Court ruled that the (Filipinas Manufacturers Bank vs. NLRC, 182 SCRA 848 [February 28, 1990]). A careful study of
doctrine that employees of government-owned and/or con controlled corporations, whether the records shows no substantive reason to depart from these established principles.
created by special law or formed as subsidiaries under the General Corporation law are
governed by the Civil Service Law and not by the Labor Code, has been supplanted by the While it is true that loss of trust or breach of confidence is a valid ground for dismissing an
present Constitution. "Thus, under the present state of the law, the test in determining whether a
employee, such loss or breach of trust must have some basis (Gubac v. NLRC, 187 SCRA 412
government-owned or controlled corporation is subject to the Civil Service Law are the manner [July 13, 1990]). As found by the Labor Arbiter, the accusations of petitioner PNOC-EDC against
of its creation, such that government corporations created by special charter are subject to its private respondent Mercado have no basis. Mrs. Leonardo Nodado, from whom the nipa
provisions while those incorporated under the General Corporation Law are not within its
shingles were purchased, sufficiently explained in her affidavit (Rollo, p. 36) that the total
coverage." purchase price of P1,680.00 was paid by respondent Mercado as agreed upon. The alleged
discount given by Mrs. Nodado is not supported by evidence as well as the alleged appropriation
Specifically, the PNOC-EDC having been incorporated under the General Corporation Law was of P8.66 from the cost of fabrication of rubber stamps. The Labor Arbiter, likewise, found no
held to be a government owned or controlled corporation whose employees are subject to the evidence to support the alleged violation of company rules. On the contrary, he found
provisions of the Labor Code (Ibid.). respondent Mercado's explanation in his affidavit (Rollo, pp. 38-40) as to the alleged violations
to be satisfactory. Moreover, these findings were never contradicted by petitioner petitioner
PNOC-EDC.
The fact that the case arose at the time when the 1973 Constitution was still in effect, does not
deprive the NLRC of jurisdiction on the premise that it is the 1987 Constitution that governs
because it is the Constitution in place at the time of the decision (NASECO v. NLRC, G.R. No. PREMISES CONSIDERED, the petition is DENIED and the resolution of respondent NLRC
69870, 168 SCRA 122 [1988]). dated July 3, 1987 is AFFIRMED with the modification that the moral damages are reduced to
Ten Thousand (P10,000.00) Pesos, and the exemplary damages reduced to Five Thousand
(P5,000.00) Pesos.
In the case at bar, the decision of the NLRC was promulgated on July 3, 1987. Accordingly, this
case falls squarely under the rulings of the aforementioned cases.
SO ORDERED.
G.R. No. 150416 July 21, 2006 The donation was allegedly accepted by one Liberato Rayos, an elder of the Seventh Day
Adventist Church, on behalf of the donee.
SEVENTH DAY ADVENTIST CONFERENCE CHURCH OF SOUTHERN PHILIPPINES, INC.,
and/or represented by MANASSEH C. ARRANGUEZ, BRIGIDO P. GULAY, FRANCISCO M. Twenty-one years later, however, on February 28, 1980, the same parcel of land was sold by the
LUCENARA, DIONICES O. TIPGOS, LORESTO C. MURILLON, ISRAEL C. NINAL, GEORGE spouses Cosio to the Seventh Day Adventist Church of Northeastern Mindanao Mission (SDA-
G. SOMOSOT, JESSIE T. ORBISO, LORETO PAEL and JOEL BACUBAS, petitioners, NEMM).5 TCT No. 4468 was thereafter issued in the name of SDA-NEMM.6
vs.
NORTHEASTERN MINDANAO MISSION OF SEVENTH DAY ADVENTIST, INC., and/or
Claiming to be the alleged donee’s successors-in-interest, petitioners asserted ownership over
represented by JOSUE A. LAYON, WENDELL M. SERRANO, FLORANTE P. TY and
the property. This was opposed by respondents who argued that at the time of the donation,
JETHRO CALAHAT and/or SEVENTH DAY ADVENTIST CHURCH [OF] NORTHEASTERN
SPUM-SDA Bayugan could not legally be a donee
MINDANAO MISSION,* Respondents.

because, not having been incorporated yet, it had no juridical personality. Neither were
DECISION
petitioners members of the local church then, hence, the donation could not have been made
particularly to them.
CORONA, J.:
On September 28, 1987, petitioners filed a case, docketed as Civil Case No. 63 (a suit for
This petition for review on certiorari assails the Court of Appeals (CA) decision1 and cancellation of title, quieting of ownership and possession, declaratory relief and reconveyance
resolution2 in CA-G.R. CV No. 41966 affirming, with modification, the decision of the Regional with prayer for preliminary injunction and damages), in the RTC of Bayugan, Agusan del Sur.
Trial Court (RTC) of Bayugan, Agusan del Sur, Branch 7 in Civil Case No. 63. After trial, the trial court rendered a decision7 on November 20, 1992 upholding the sale in favor
of respondents.
This case involves a 1,069 sq. m. lot covered by Transfer Certificate of Title (TCT) No. 4468 in
Bayugan, Agusan del Sur originally owned by Felix Cosio and his wife, Felisa Cuysona. On appeal, the CA affirmed the RTC decision but deleted the award of moral damages and
attorney’s fees.8Petitioners’ motion for reconsideration was likewise denied. Thus, this petition.
On April 21, 1959, the spouses Cosio donated the land to the South Philippine Union Mission of
Seventh Day Adventist Church of Bayugan Esperanza, Agusan (SPUM-SDA Bayugan).3 Part of The issue in this petition is simple: should SDA-NEMM’s ownership of the lot covered by TCT
the deed of donation read: No. 4468 be upheld?9We answer in the affirmative.

KNOW ALL MEN BY THESE PRESENTS: The controversy between petitioners and respondents involves two supposed transfers of the lot
previously owned by the spouses Cosio: (1) a donation to petitioners’ alleged predecessors-in-
interest in 1959 and (2) a sale to respondents in 1980.
That we Felix Cosio[,] 49 years of age[,] and Felisa Cuysona[,] 40 years of age, [h]usband and
wife, both are citizen[s] of the Philippines, and resident[s] with post office address in the Barrio of
Bayugan, Municipality of Esperanza, Province of Agusan, Philippines, do hereby grant, convey Donation is undeniably one of the modes of acquiring ownership of real property. Likewise,
and forever quit claim by way of Donation or gift unto the South Philippine [Union] Mission of ownership of a property may be transferred by tradition as a consequence of a sale.
Seventh Day Adventist Church of Bayugan, Esperanza, Agusan, all the rights, title, interest,
claim and demand both at law and as well in possession as in expectancy of in and to all the
Petitioners contend that the appellate court should not have ruled on the validity of the donation
place of land and portion situated in the Barrio of Bayugan, Municipality of Esperanza, Province
since it was not among the issues raised on appeal. This is not correct because an appeal
of Agusan, Philippines, more particularly and bounded as follows, to wit:
generally opens the entire case for review.

1. a parcel of land for Church Site purposes only.


We agree with the appellate court that the alleged donation to petitioners was void.

2. situated [in Barrio Bayugan, Esperanza].


Donation is an act of liberality whereby a person disposes gratuitously of a thing or right in favor
of another personwho accepts it. The donation could not have been made in favor of an entity
3. Area: 30 meters wide and 30 meters length or 900 square meters. yet inexistent at the time it was made. Nor could it have been accepted as there was yet no one
to accept it.
4. Lot No. 822-Pls-225. Homestead Application No. V-36704, Title No. P-285.
The deed of donation was not in favor of any informal group of SDA members but a supposed
SPUM-SDA Bayugan (the local church) which, at the time, had neither juridical personality nor
5. Bounded Areas
capacity to accept such gift.

North by National High Way; East by Bricio Gerona; South by Serapio Abijaron and West by
Declaring themselves a de facto corporation, petitioners allege that they should benefit from the
Feliz Cosio xxx. 4
donation.
But there are stringent requirements before one can qualify as a de facto corporation: he would not have disposed of it for a mere P2,000.00 in two installments but for P50,000.00
or P60,000.00. According to him, the P2,000.00 was not a consideration of the sale but only a
form of help extended.
(a) the existence of a valid law under which it may be incorporated;

A thorough analysis and perusal, nonetheless, of the Deed of Absolute Sale disclosed
(b) an attempt in good faith to incorporate; and
that it has the essential requisites of contracts pursuant to xxx Article 1318 of the Civil
Code, except that the consideration of P2,000.00 is somewhat insufficient for a [1,069-square
(c) assumption of corporate powers.10 meter] land. Would then this inadequacy of the consideration render the contract invalid?

While there existed the old Corporation Law (Act 1459),11 a law under which SPUM-SDA Article 1355 of the Civil Code provides:
Bayugan could have been organized, there is no proof that there was an attempt to incorporate
at that time.
Except in cases specified by law, lesion or inadequacy of cause shall not invalidate a contract,
unless there has been fraud, mistake or undue influence.
The filing of articles of incorporation and the issuance of the certificate of incorporation are
essential for the existence of a de facto corporation.12 We have held that an organization not
No evidence [of fraud, mistake or undue influence] was adduced by [petitioners].
registered with the Securities and Exchange Commission (SEC) cannot be considered a
corporation in any concept, not even as a corporation de facto.13 Petitioners themselves
admitted that at the time of the donation, they were not registered with the SEC, nor did they xxx
even attempt to organize14 to comply with legal requirements.
Well-entrenched is the rule that a Certificate of Title is generally a conclusive evidence of
Corporate existence begins only from the moment a certificate of incorporation is issued. No [ownership] of the land. There is that strong and solid presumption that titles were legally
such certificate was ever issued to petitioners or their supposed predecessor-in-interest at the issued and that they are valid. It is irrevocable and indefeasible and the duty of the Court is to
time of the donation. Petitioners obviously could not have claimed succession to an entity that see to it that the title is maintained and respected unless challenged in a direct proceeding. xxx
never came to exist. Neither could the principle of separate juridical personality apply since there The title shall be received as evidence in all the Courts and shall be conclusive as to all matters
was never any corporation15 to speak of. And, as already stated, some of the representatives of contained therein.
petitioner Seventh Day Adventist Conference Church of Southern Philippines, Inc. were not even
members of the local church then, thus, they could not even claim that the donation was
[This action was instituted almost seven years after the certificate of title in respondents’ name
particularly for them.16
was issued in 1980.]20

"The de facto doctrine thus effects a compromise between two conflicting public interest[s]—the
According to Art. 1477 of the Civil Code, the ownership of the thing sold shall be transferred to
one opposed to an unauthorized assumption of corporate privileges; the other in favor of doing
the vendee upon the actual or constructive delivery thereof. On this, the noted author Arturo
justice to the parties and of establishing a general assurance of security in business dealing with
Tolentino had this to say:
corporations."17

The execution of [a] public instrument xxx transfers the ownership from the vendor to the vendee
Generally, the doctrine exists to protect the public dealing with supposed corporate entities, not
who may thereafter exercise the rights of an owner over the same21
to favor the defective or non-existent corporation.18

Here, transfer of ownership from the spouses Cosio to SDA-NEMM was made upon constructive
In view of the foregoing, petitioners’ arguments anchored on their supposed de facto status hold
delivery of the property on February 28, 1980 when the sale was made through a public
no water. We are convinced that there was no donation to petitioners or their supposed
instrument.22 TCT No. 4468 was thereafter issued and it remains in the name of SDA-NEMM.
predecessor-in-interest.

WHEREFORE, the petition is hereby DENIED.


On the other hand, there is sufficient basis to affirm the title of SDA-NEMM. The factual findings
of the trial court in this regard were not convincingly disputed. This Court is not a trier of facts.
Only questions of law are the proper subject of a petition for review on certiorari. 19 Costs against petitioners.

Sustaining the validity of respondents’ title as well as their right of ownership over the property, SO ORDERED.
the trial court stated:

[W]hen Felix Cosio was shown the Absolute Deed of Sale during the hearing xxx he
acknowledged that the same was his xxx but that it was not his intention to sell the controverted
property because he had previously donated the same lot to the South Philippine Union Mission
of SDA Church of Bayugan-Esperanza. Cosio avouched that had it been his intendment to sell,
G.R. No. 141735 June 8, 2005 On 18 June 1993, petitioner received a memorandum from Islamic Bank [AIIBP] Chairman
Roberto F. De Ocampo charging him with Dishonesty in the Performance of Official Duties
and/or Conduct Prejudicial to the Best Interest of the Service and preventively suspending him.
SAPPARI K. SAWADJAAN, petitioner,
vs.
THE HONORABLE COURT OF APPEALS, THE CIVIL SERVICE COMMISSION and AL- In his memorandum dated 8 September 1993, petitioner informed the Investigating Committee
AMANAH INVESTMENT BANK OF THE PHILIPPINES, respondents. that he could not submit himself to the jurisdiction of the Committee because of its alleged
partiality. For his failure to appear before the hearing set on 17 September 1993, after the
hearing of 13 September 1993 was postponed due to the Manifestation of even date filed by
DECISION
petitioner, the Investigating Committee declared petitioner in default and the prosecution was
allowed to present its evidence ex parte.
CHICO-NAZARIO, J.:
On 08 December 1993, the Investigating Committee rendered a decision, the pertinent portions
This is a petition for certiorari under Rule 65 of the Rules of Court of the Decision1 of the Court of of which reads as follows:
Appeals of 30 March 1999 affirming Resolutions No. 94-4483 and No. 95-2754 of the Civil
Service Commission (CSC) dated 11 August 1994 and 11 April 1995, respectively, which in turn
In view of respondent SAWADJAAN’S abject failure to perform his duties and assigned tasks as
affirmed Resolution No. 2309 of the Board of Directors of the Al-Amanah Islamic Investment
appraiser/inspector, which resulted to the prejudice and substantial damage to the Bank,
Bank of the Philippines (AIIBP) dated 13 December 1993, finding petitioner guilty of Dishonesty
respondent should be held liable therefore. At this juncture, however, the Investigating
in the Performance of Official Duties and/or Conduct Prejudicial to the Best Interest of the
Committee is of the considered opinion that he could not be held liable for the administrative
Service and dismissing him from the service, and its Resolution2 of 15 December 1999
offense of dishonesty considering the fact that no evidence was adduced to show that he
dismissing petitioner’s Motion for Reconsideration.
profited or benefited from being remiss in the performance of his duties. The record is bereft of
any evidence which would show that he received any amount in consideration for his non-
The records show that petitioner Sappari K. Sawadjaan was among the first employees of the performance of his official duties.
Philippine Amanah Bank (PAB) when it was created by virtue of Presidential Decree No. 264 on
02 August 1973. He rose through the ranks, working his way up from his initial designation as
This notwithstanding, respondent cannot escape liability. As adverted to earlier, his failure to
security guard, to settling clerk, bookkeeper, credit investigator, project analyst, appraiser/
perform his official duties resulted to the prejudice and substantial damage to the Islamic Bank
inspector, and eventually, loans analyst.3
for which he should be held liable for the administrative offense of CONDUCT PREJUDICIAL TO
THE BEST INTEREST OF THE SERVICE.
In February 1988, while still designated as appraiser/investigator, Sawadjaan was assigned to
inspect the properties offered as collaterals by Compressed Air Machineries and Equipment
Premises considered, the Investigating Committee recommends that respondent SAPPARI
Corporation (CAMEC) for a credit line of Five Million Pesos (P5,000,000.00). The properties
SAWADJAAN be meted the penalty of SIX (6) MONTHS and ONE (1) DAY SUSPENSION from
consisted of two parcels of land covered by Transfer Certificates of Title (TCTs) No. N-130671
office in accordance with the Civil Service Commission’s Memorandum Circular No. 30, Series
and No. C-52576. On the basis of his Inspection and Appraisal Report, 4 the PAB granted the
of 1989.
loan application. When the loan matured on 17 May 1989, CAMEC requested an extension of
180 days, but was granted only 120 days to repay the loan.5
On 13 December 1993, the Board of Directors of the Islamic Bank [AIIBP] adopted Resolution
6 No. 2309 finding petitioner guilty of Dishonesty in the Performance of Official Duties and/or
In the meantime, Sawadjaan was promoted to Loans Analyst I on 01 July 1989.
Conduct Prejudicial to the Best Interest of the Service and imposing the penalty of Dismissal
from the Service.
In January 1990, Congress passed Republic Act 6848 creating the AIIBP and repealing P.D. No.
264 (which created the PAB). All assets, liabilities and capital accounts of the PAB were
On reconsideration, the Board of Directors of the Islamic Bank [AIIBP] adopted the Resolution
transferred to the AIIBP,7 and the existing personnel of the PAB were to continue to discharge
No. 2332 on 20 February 1994 reducing the penalty imposed on petitioner from dismissal to
their functions unless discharged.8 In the ensuing reorganization, Sawadjaan was among the
suspension for a period of six (6) months and one (1) day.
personnel retained by the AIIBP.

On 29 March 1994, petitioner filed a notice of appeal to the Merit System Protection Board
When CAMEC failed to pay despite the given extension, the bank, now referred to as the AIIBP,
(MSPB).
discovered that TCT No. N-130671 was spurious, the property described therein non-existent,
and that the property covered by TCT No. C-52576 had a prior existing mortgage in favor of one
Divina Pablico. On 11 August 1994, the CSC adopted Resolution No. 94-4483 dismissing the appeal for lack of
merit and affirming Resolution No. 2309 dated 13 December 1993 of the Board of Directors of
Islamic Bank.
On 08 June 1993, the Board of Directors of the AIIBP created an Investigating Committee to
look into the CAMEC transaction, which had cost the bank Six Million Pesos (P6,000,000.00) in
losses.9 The subsequent events, as found and decided upon by the Court of Appeals, 10 are as On 11 April 1995, the CSC adopted Resolution No. 95-2574 denying petitioner’s Motion for
follows: Reconsideration.
On 16 June 1995, the instant petition was filed with the Honorable Supreme Court on the The second assignment of error must likewise fail. The issue is raised for the first time via this
following assignment of errors: petition for certiorari.Petitioner submitted himself to the jurisdiction of the CSC. Although he
could have raised the alleged lack of jurisdiction in his Motion for Reconsideration of Resolution
No. 94-4483 of the CSC, he did not do so. By filing the Motion for Reconsideration, he is
I. Public respondent Al-Amanah Islamic Investment Bank of the Philippines has
estopped from denying the CSC’s jurisdiction over him, as it is settled rule that a party who asks
committed a grave abuse of discretion amounting to excess or lack of jurisdiction
for an affirmative relief cannot later on impugn the action of the tribunal as without jurisdiction
when it initiated and conducted administrative investigation without a validly
after an adverse result was meted to him. Although jurisdiction over the subject matter of a case
promulgated rules of procedure in the adjudication of administrative cases at the
may be objected to at any stage of the proceedings even on appeal, this particular rule,
Islamic Bank.
however, means that jurisdictional issues in a case can be raised only during the proceedings in
said case and during the appeal of said case (Aragon v. Court of Appeals, 270 SCRA 603). The
II. Public respondent Civil Service Commission has committed a grave abuse of case at bar is a petition [for] certiorari and not an appeal.
discretion amounting to lack of jurisdiction when it prematurely and falsely assumed
jurisdiction of the case not appealed to it, but to the Merit System Protection Board.
But even on the merits the argument must falter. Item No. 1 of CSC Resolution No. 93-2387
dated 29 June 1993, provides:
III. Both the Islamic Bank and the Civil Service Commission erred in finding petitioner
Sawadjaan of having deliberately reporting false information and therefore guilty of
Decisions in administrative cases involving officials and employees of the civil service
Dishonesty and Conduct Prejudicial to the Best Interest of the Service and penalized
appealable to the Commission pursuant to Section 47 of Book V of the Code (i.e., Administrative
with dismissal from the service.
Code of 1987) including personnel actions such as contested appointments shall now be
appealed directly to the Commission and not to the MSPB.
On 04 July 1995, the Honorable Supreme Court En Banc referred this petition to this Honorable
Court pursuant to Revised Administrative Circular No. 1-95, which took effect on 01 June 1995.
In Rubenecia v. Civil Service Commission, 244 SCRA 640, 651, it was categorically held:

We do not find merit [in] the petition.


. . . The functions of the MSPB relating to the determination of administrative disciplinary cases
were, in other words, re-allocated to the Commission itself.
Anent the first assignment of error, a reading of the records would reveal that petitioner raises
for the first time the alleged failure of the Islamic Bank [AIIBP] to promulgate rules of procedure
Be that as it may, "(i)t is hornbook doctrine that in order `(t)o ascertain whether a court (in this
governing the adjudication and disposition of administrative cases involving its personnel. It is a
case, administrative agency) has jurisdiction or not, the provisions of the law should be inquired
rule that issues not properly brought and ventilated below may not be raised for the first time on
into.’ Furthermore, `the jurisdiction of the court must appear clearly from the statute law or it will
appeal, save in exceptional circumstances (Casolita, Sr. v. Court of Appeals, 275 SCRA 257)
not be held to exist.’"(Azarcon v. Sandiganbayan, 268 SCRA 747, 757) From the provision of
none of which, however, obtain in this case. Granting arguendo that the issue is of such
law abovecited, the Civil Service Commission clearly has jurisdiction over the Administrative
exceptional character that the Court may take cognizance of the same, still, it must fail. Section
Case against petitioner.
26 of Republic Act No. 6848 (1990) provides:

Anent the third assignment of error, we likewise do not find merit in petitioner’s proposition that
Section 26. Powers of the Board. The Board of Directors shall have the broadest powers to
he should not be liable, as in the first place, he was not qualified to perform the functions of
manage the Islamic Bank, x x x The Board shall adopt policy guidelines necessary to carry out
appraiser/investigator because he lacked the necessary training and expertise, and therefore,
effectively the provisions of this Charter as well as internal rules and regulations necessary for
should not have been found dishonest by the Board of Directors of Islamic Bank [AIIBP] and the
the conduct of its Islamic banking business and all matters related to personnel organization,
CSC. Petitioner himself admits that the position of appraiser/inspector is "one of the most
office functions and salary administration. (Italics ours)
serious [and] sensitive job in the banking operations." He should have been aware that
accepting such a designation, he is obliged to perform the task at hand by the exercise of more
On the other hand, Item No. 2 of Executive Order No. 26 (1992) entitled "Prescribing Procedure than ordinary prudence. As appraiser/investigator, he is expected, among others, to check the
and Sanctions to Ensure Speedy Disposition of Administrative Cases" directs, "all administrative authenticity of the documents presented by the borrower by comparing them with the originals
agencies" to "adopt and include in their respective Rules of Procedure" provisions designed to on file with the proper government office. He should have made it sure that the technical
abbreviate administrative proceedings. descriptions in the location plan on file with the Bureau of Lands of Marikina, jibe with that
indicated in the TCT of the collateral offered by CAMEC, and that the mortgage in favor of the
Islamic Bank was duly annotated at the back of the copy of the TCT kept by the Register of
The above two (2) provisions relied upon by petitioner does not require the Islamic Bank [AIIBP]
Deeds of Marikina. This, petitioner failed to do, for which he must be held liable. That he did not
to promulgate rules of procedure before administrative discipline may be imposed upon its profit from his false report is of no moment. Neither the fact that it was not deliberate or willful,
employees. The internal rules of procedures ordained to be adopted by the Board refers to that detracts from the nature of the act as dishonest. What is apparent is he stated something to be a
necessary for the conduct of its Islamic banking business and all matters related to "personnel
fact, when he really was not sure that it was so.
organization, office functions and salary administration." On the contrary, Section 26 of RA 6848
gives the Board of Directors of the Islamic Bank the "broadest powers to manage the Islamic
Bank." This grant of broad powers would be an idle ceremony if it would be powerless to Wherefore, above premises considered, the instant Petition is DISMISSED, and the assailed
discipline its employees. Resolutions of the Civil Service Commission are hereby AFFIRMED.
On 24 March 1999, Sawadjaan’s counsel notified the court a quo of his change of address,11 but Parte Urgent Motion for Extension of Time to File Reply Memorandum (To: CSC and AIIBP’s
apparently neglected to notify his client of this fact. Thus, on 23 July 1999, Sawadjaan, by Memorandum);33 15) Reply Memorandum (To: CSC’s Memorandum) With Ex-Parte Urgent
himself, filed a Motion for New Trial12 in the Court of Appeals based on the following grounds: Motion for Additional Extension of time to File Reply Memorandum (To: AIIBP’s
fraud, accident, mistake or excusable negligence and newly discovered evidence. He claimed Memorandum);34 and 16) Reply Memorandum (To: OGCC’s Memorandum for Respondent
that he had recently discovered that at the time his employment was terminated, the AIIBP had AIIBP).35
not yet adopted its corporate by-laws. He attached a Certification13 by the Securities and
Exchange Commission (SEC) that it was only on 27 May 1992 that the AIIBP submitted its draft
Petitioner’s efforts are unavailing, and we deny his petition for its procedural and substantive
by-laws to the SEC, and that its registration was being held in abeyance pending certain
flaws.
corrections being made thereon. Sawadjaan argued that since the AIIBP failed to file its by-laws
within 60 days from the passage of Rep. Act No. 6848, as required by Sec. 51 of the said law,
the bank and its stockholders had "already forfeited its franchise or charter, including its license The general rule is that the remedy to obtain reversal or modification of the judgment on the
to exist and operate as a corporation,"14 and thus no longer have "the legal standing and merits is appeal. This is true even if the error, or one of the errors, ascribed to the court
personality to initiate an administrative case." rendering the judgment is its lack of jurisdiction over the subject matter, or the exercise of power
in excess thereof, or grave abuse of discretion in the findings of fact or of law set out in the
decision.36
Sawadjaan’s counsel subsequently adopted his motion, but requested that it be treated as a
motion for reconsideration.15 This motion was denied by the court a quo in its Resolution of 15
December 1999.16 The records show that petitioner’s counsel received the Resolution of the Court of Appeals
denying his motion for reconsideration on 27 December 1999. The fifteen day reglamentary
period to appeal under Rule 45 of the Rules of Court therefore lapsed on 11 January 2000. On
Still disheartened, Sawadjaan filed the present petition for certiorari under Rule 65 of the Rules
23 February 2000, over a month after receipt of the resolution denying his motion for
of Court challenging the above Decision and Resolution of the Court of Appeals on the ground
reconsideration, the petitioner filed his petition for certiorari under Rule 65.
that the court a quo erred: i) in ignoring the facts and evidences that the alleged Islamic Bank
has no valid by-laws; ii) in ignoring the facts and evidences that the Islamic Bank lost its juridical
personality as a corporation on 16 April 1990; iii) in ignoring the facts and evidences that the It is settled that a special civil action for certiorari will not lie as a substitute for the lost remedy of
alleged Islamic Bank and its alleged Board of Directors have no jurisdiction to act in the manner appeal,37 and though there are instances38 where the extraordinary remedy of certiorari may be
they did in the absence of a valid by-laws; iv) in not correcting the acts of the Civil Service resorted to despite the availability of an appeal,39 we find no special reasons for making out an
Commission who erroneously rendered the assailed Resolutions No. 94-4483 and No. 95-2754 exception in this case.
as a result of fraud, falsification and/or misrepresentations committed by Farouk A. Carpizo and
his group, including Roberto F. de Ocampo; v) in affirming an unconscionably harsh and/or
excessive penalty; and vi) in failing to consider newly discovered evidence and reverse its Even if we were to overlook this fact in the broader interests of justice and treat this as a special
decision accordingly. civil action for certiorari under Rule 65,40 the petition would nevertheless be dismissed for failure
of the petitioner to show grave abuse of discretion. Petitioner’s recurrent argument, tenuous at
its very best, is premised on the fact that since respondent AIIBP failed to file its by-laws within
Subsequently, petitioner Sawadjaan filed an "Ex-parte Urgent Motion for Additional Extension of the designated 60 days from the effectivity of Rep. Act No. 6848, all proceedings initiated by
Time to File a Reply (to the Comments of Respondent Al-Amanah Investment Bank of the AIIBP and all actions resulting therefrom are a patent nullity. Or, in his words, the AIIBP and its
Philippines),17 Reply (to Respondent’s Consolidated Comment,)18 and Reply (to the Alleged officers and Board of Directors,
Comments of Respondent Al-Amanah Islamic Bank of the Philippines)."19 On 13 October 2000,
he informed this Court that he had terminated his lawyer’s services, and, by himself, prepared
. . . [H]ave no legal authority nor jurisdiction to manage much less operate the Islamic Bank, file
and filed the following: 1) Motion for New Trial;20 2) Motion to Declare Respondents in Default
and/or Having Waived their Rights to Interpose Objection to Petitioner’s Motion for New administrative charges and investigate petitioner in the manner they did and allegedly passed
Trial;21 3) Ex-Parte Urgent Motions to Punish Attorneys Amado D. Valdez, Elpidio J. Vega, Alda Board Resolution No. 2309 on December 13, 1993 which is null and void for lack of an
(sic) authorized and valid by-laws. The CIVIL SERVICE COMMISSION was therefore affirming,
G. Reyes, Dominador R. Isidoro, Jr., and Odilon A. Diaz for Being in Contempt of Court & to
Inhibit them from Appearing in this Case Until they Can Present Valid Evidence of Legal erroneously, a null and void "Resolution No. 2309 dated December 13, 1993 of the Board of
Authority;22 4) Opposition/Reply (to Respondent AIIBP’s Alleged Comment);23 5) Ex- Directors of Al-Amanah Islamic Investment Bank of the Philippines" in CSC Resolution No. 94-
4483 dated August 11, 1994. A motion for reconsideration thereof was denied by the CSC in its
Parte Urgent Motion to Punish Atty. Reynaldo A. Pineda for Contempt of Court and the Issuance
of a Commitment Order/Warrant for His Arrest;24 6) Reply/Opposition (To the Formal Notice of Resolution No. 95-2754 dated April 11, 1995. Both acts/resolutions of the CSC are erroneous,
Withdrawal of Undersigned Counsel as Legal Counsel for the Respondent Islamic Bank with resulting from fraud, falsifications and misrepresentations of the alleged Chairman and CEO
Roberto F. de Ocampo and the alleged Director Farouk A. Carpizo and his group at the alleged
Opposition to Petitioner’s Motion to Punish Undersigned Counsel for Contempt of Court for the
Issuance of a Warrant of Arrest);25 7) Memorandum for Petitioner;26 8) Opposition to SolGen’s Islamic Bank.41
Motion for Clarification with Motion for Default and/or Waiver of Respondents to File their
Memorandum;27 9) Motion for Contempt of Court and Inhibition/Disqualification with Opposition Nowhere in petitioner’s voluminous pleadings is there a showing that the court a quo committed
to OGCC’s Motion for Extension of Time to File Memorandum; 28 10) Motion for Enforcement (In grave abuse of discretion amounting to lack or excess of jurisdiction reversible by a petition
Defense of the Rule of Law);29 11) Motion and Opposition (Motion to Punish OGCC’s Attorneys for certiorari. Petitioner already raised the question of AIIBP’s corporate existence and lack of
Amado D. Valdez, Efren B. Gonzales, Alda G. Reyes, Odilon A. Diaz and Dominador R. Isidoro, jurisdiction in his Motion for New Trial/Motion for Reconsideration of 27 May 1997 and was
Jr., for Contempt of Court and the Issuance of a Warrant for their Arrest; and Opposition to their denied by the Court of Appeals. Despite the volume of pleadings he has submitted thus far, he
Alleged "Manifestation and Motion" Dated February 5, 2002);30 12) Motion for Reconsideration of has added nothing substantial to his arguments.
Item (a) of Resolution dated 5 February 2002 with Supplemental Motion for Contempt of
Court;3113) Motion for Reconsideration of Portion of Resolution Dated 12 March 2002; 32 14) Ex-
The AIIBP was created by Rep. Act No. 6848. It has a main office where it conducts business, On appeal to the CSC, the Commission found that Sawadjaan’s failure to perform his official
has shareholders, corporate officers, a board of directors, assets, and personnel. It is, in fact, duties greatly prejudiced the AIIBP, for which he should be held accountable. It held that:
here represented by the Office of the Government Corporate Counsel, "the principal law office of
government-owned corporations, one of which is respondent bank."42 At the very least, by its
. . . (I)t is crystal clear that respondent SAPPARI SAWADJAAN was remiss in the performance
failure to submit its by-laws on time, the AIIBP may be considered a de
of his duties as appraiser/inspector. Had respondent performed his duties as
facto corporation43 whose right to exercise corporate powers may not be inquired into collaterally
appraiser/inspector, he could have easily noticed that the property located at Balintawak,
in any private suit to which such corporations may be a party.44
Caloocan City covered by TCT No. C-52576 and which is one of the properties offered as
collateral by CAMEC is encumbered to Divina Pablico. Had respondent reflected such fact in his
Moreover, a corporation which has failed to file its by-laws within the prescribed period does appraisal/inspection report on said property the ISLAMIC BANK would not have approved
not ipso facto lose its powers as such. The SEC Rules on Suspension/Revocation of the CAMEC’s loan of P500,000.00 in 1987 and CAMEC’s P5 Million loan in 1988, respondent
Certificate of Registration of Corporations,45details the procedures and remedies that may be knowing fully well the Bank’s policy of not accepting encumbered properties as collateral.
availed of before an order of revocation can be issued. There is no showing that such a
procedure has been initiated in this case.
Respondent SAWADJAAN’s reprehensible act is further aggravated when he failed to check and
verify from the Registry of Deeds of Marikina the authenticity of the property located at Mayamot,
In any case, petitioner’s argument is irrelevant because this case is not a corporate controversy, Antipolo, Rizal covered by TCT No. N-130671 and which is one of the properties offered as
but a labor dispute; and it is an employer’s basic right to freely select or discharge its employees, collateral by CAMEC for its P5 Million loan in 1988. If he only visited and verified with the
if only as a measure of self-protection against acts inimical to its interest.46 Regardless of Register of Deeds of Marikina the authenticity of TCT No. N-130671 he could have easily
whether AIIBP is a corporation, a partnership, a sole proprietorship, or a sari-sari store, it is an discovered that TCT No. N-130671 is fake and the property described therein non-existent.
undisputed fact that AIIBP is the petitioner’s employer. AIIBP chose to retain his services during
its reorganization, controlled the means and methods by which his work was to be performed,
...
paid his wages, and, eventually, terminated his services.47

This notwithstanding, respondent cannot escape liability. As adverted to earlier, his failure to
And though he has had ample opportunity to do so, the petitioner has not alleged that he is
perform his official duties resulted to the prejudice and substantial damage to the ISLAMIC
anything other than an employee of AIIBP. He has neither claimed, nor shown, that he is a
BANK for which he should be held liable for the administrative offense of CONDUCT
stockholder or an officer of the corporation. Having accepted employment from AIIBP, and
PREJUDICIAL TO THE BEST INTEREST OF THE SERVICE.49
rendered his services to the said bank, received his salary, and accepted the promotion given
him, it is now too late in the day for petitioner to question its existence and its power to terminate
his services. One who assumes an obligation to an ostensible corporation as such, cannot resist From the foregoing, we find that the CSC and the court a quo committed no grave abuse of
performance thereof on the ground that there was in fact no corporation. 481avvphi1 discretion when they sustained Sawadjaan’s dismissal from service. Grave abuse of discretion
implies such capricious and whimsical exercise of judgment as equivalent to lack of jurisdiction,
or, in other words, where the power is exercised in an arbitrary or despotic manner by reason of
Even if we were to consider the facts behind petitioner Sawadjaan’s dismissal from service, we
passion or personal hostility, and it must be so patent and gross as to amount to an evasion of
would be hard pressed to find error in the decision of the AIIBP.
positive duty or to a virtual refusal to perform the duty enjoined or to act at all in contemplation of
law.50 The records show that the respondents did none of these; they acted in accordance with
As appraiser/investigator, the petitioner was expected to conduct an ocular inspection of the the law.
properties offered by CAMEC as collaterals and check the copies of the certificates of title
against those on file with the Registry of Deeds. Not only did he fail to conduct these routine
WHEREFORE, the petition is DISMISSED. The Decision of the Court of Appeals of 30 March
checks, but he also deliberately misrepresented in his appraisal report that after reviewing the
1999 affirming Resolutions No. 94-4483 and No. 95-2754 of the Civil Service Commission, and
documents and conducting a site inspection, he found the CAMEC loan application to be in
its Resolution of 15 December 1999 are hereby affirmed. Costs against the petitioner.
order. Despite the number of pleadings he has filed, he has failed to offer an alternative
explanation for his actions.
SO ORDERED.
When he was informed of the charges against him and directed to appear and present his side
on the matter, the petitioner sent instead a memorandum questioning the fairness and
impartiality of the members of the investigating committee and refusing to recognize their
jurisdiction over him. Nevertheless, the investigating committee rescheduled the hearing to give
the petitioner another chance, but he still refused to appear before it.

Thereafter, witnesses were presented, and a decision was rendered finding him guilty of
dishonesty and dismissing him from service. He sought a reconsideration of this decision and
the same committee whose impartiality he questioned reduced their recommended penalty to
suspension for six months and one day. The board of directors, however, opted to dismiss him
from service.
G.R. No. 136448 November 3, 1999 ii. Accrued interest for P27,904.02
on Invoice No. 14413 for
P146,868.00 dated February 13,
LIM TONG LIM, petitioner,
1990;
vs.
PHILIPPINE FISHING GEAR INDUSTRIES, INC., respondent.
iii. Accrued interest of P12,920.00
on Invoice No. 14426 for
PANGANIBAN, J.:
P68,000.00 dated February 19,
1990;
A partnership may be deemed to exist among parties who agree to borrow money to pursue a
business and to divide the profits or losses that may arise therefrom, even if it is shown that they
c. P50,000.00 as and for attorney's fees, plus
have not contributed any capital of their own to a "common fund." Their contribution may be in
P8,500.00 representing P500.00 per appearance in
the form of credit or industry, not necessarily cash or fixed assets. Being partner, they are all
court;
liable for debts incurred by or on behalf of the partnership. The liability for a contract entered into
on behalf of an unincorporated association or ostensible corporation may lie in a person who
may not have directly transacted on its behalf, but reaped benefits from that contract. d. P65,000.00 representing P5,000.00 monthly rental
for storage charges on the nets counted from
September 20, 1990 (date of attachment) to September
The Case
12, 1991 (date of auction sale);

In the Petition for Review on Certiorari before us, Lim Tong Lim assails the November 26, 1998
e. Cost of suit.
Decision of the Court of Appeals in CA-GR CV
41477, 1 which disposed as follows:
With respect to the joint liability of defendants for the principal
obligation or for the unpaid price of nets and floats in the amount
WHEREFORE, [there being] no reversible error in the appealed decision,
of P532,045.00 and P68,000.00, respectively, or for the total
the same is hereby affirmed. 2
amount P600,045.00, this Court noted that these items were
attached to guarantee any judgment that may be rendered in
The decretal portion of the Quezon City Regional Trial Court (RTC) ruling, which was affirmed by favor of the plaintiff but, upon agreement of the parties, and, to
the CA, reads as follows: avoid further deterioration of the nets during the pendency of this
case, it was ordered sold at public auction for not less than
P900,000.00 for which the plaintiff was the sole and winning
WHEREFORE, the Court rules: bidder. The proceeds of the sale paid for by plaintiff was
deposited in court. In effect, the amount of P900,000.00 replaced
1. That plaintiff is entitled to the writ of preliminary attachment issued by this the attached property as a guaranty for any judgment that plaintiff
Court on September 20, 1990; may be able to secure in this case with the ownership and
possession of the nets and floats awarded and delivered by the
sheriff to plaintiff as the highest bidder in the public auction sale. It
2. That defendants are jointly liable to plaintiff for the following amounts, has also been noted that ownership of the nets [was] retained by
subject to the modifications as hereinafter made by reason of the special the plaintiff until full payment [was] made as stipulated in the
and unique facts and circumstances and the proceedings that transpired invoices; hence, in effect, the plaintiff attached its own properties.
during the trial of this case; It [was] for this reason also that this Court earlier ordered the
attachment bond filed by plaintiff to guaranty damages to
a. P532,045.00 representing [the] unpaid purchase defendants to be cancelled and for the P900,000.00 cash bidded
price of the fishing nets covered by the Agreement plus and paid for by plaintiff to serve as its bond in favor of defendants.
P68,000.00 representing the unpaid price of the floats
not covered by said Agreement; From the foregoing, it would appear therefore that whatever
judgment the plaintiff may be entitled to in this case will have to
b. 12% interest per annum counted from date of be satisfied from the amount of P900,000.00 as this amount
plaintiff's invoices and computed on their respective replaced the attached nets and floats. Considering, however, that
amounts as follows: the total judgment obligation as computed above would amount to
only P840,216.92, it would be inequitable, unfair and unjust to
award the excess to the defendants who are not entitled to
i. Accrued interest of P73,221.00 damages and who did not put up a single centavo to raise the
on Invoice No. 14407 for amount of P900,000.00 aside from the fact that they are not the
P385,377.80 dated February 9, owners of the nets and floats. For this reason, the defendants are
1990;
hereby relieved from any and all liabilities arising from the b) If the four (4) vessel[s] and the fishing net will be sold
monetary judgment obligation enumerated above and for plaintiff at a higher price than P5,750,000.00 whatever will be
to retain possession and ownership of the nets and floats and for the excess will be divided into 3: 1/3 Lim Tong Lim; 1/3
the reimbursement of the P900,000.00 deposited by it with the Antonio Chua; 1/3 Peter Yao;
Clerk of Court.
c) If the proceeds of the sale the vessels will be less
SO ORDERED. 3 than P5,750,000.00 whatever the deficiency shall be
shouldered and paid to JL Holding Corporation by 1/3
Lim Tong Lim; 1/3 Antonio Chua; 1/3 Peter Yao. 11
The Facts

The trial court noted that the Compromise Agreement was silent as to the nature of their
On behalf of "Ocean Quest Fishing Corporation," Antonio Chua and Peter Yao entered into a
obligations, but that joint liability could be presumed from the equal distribution of the profit and
Contract dated February 7, 1990, for the purchase of fishing nets of various sizes from the
loss. 21
Philippine Fishing Gear Industries, Inc. (herein respondent). They claimed that they were
engaged in a business venture with Petitioner Lim Tong Lim, who however was not a signatory
to the agreement. The total price of the nets amounted to P532,045. Four hundred pieces of Lim appealed to the Court of Appeals (CA) which, as already stated, affirmed the RTC.
floats worth P68,000 were also sold to the Corporation. 4
Ruling of the Court of Appeals
The buyers, however, failed to pay for the fishing nets and the floats; hence, private respondents
filed a collection suit against Chua, Yao and Petitioner Lim Tong Lim with a prayer for a writ of
In affirming the trial court, the CA held that petitioner was a partner of Chua and Yao in a fishing
preliminary attachment. The suit was brought against the three in their capacities as general
business and may thus be held liable as a such for the fishing nets and floats purchased by and
partners, on the allegation that "Ocean Quest Fishing Corporation" was a nonexistent
for the use of the partnership. The appellate court ruled:
corporation as shown by a Certification from the Securities and Exchange Commission. 5 On
September 20, 1990, the lower court issued a Writ of Preliminary Attachment, which the sheriff
enforced by attaching the fishing nets on board F/B Lourdes which was then docked at the The evidence establishes that all the defendants including herein appellant
Fisheries Port, Navotas, Metro Manila. Lim Tong Lim undertook a partnership for a specific undertaking, that is for
commercial fishing . . . . Oviously, the ultimate undertaking of the
defendants was to divide the profits among themselves which is what a
Instead of answering the Complaint, Chua filed a Manifestation admitting his liability and
partnership essentially is . . . . By a contract of partnership, two or more
requesting a reasonable time within which to pay. He also turned over to respondent some of the
persons bind themselves to contribute money, property or industry to a
nets which were in his possession. Peter Yao filed an Answer, after which he was deemed to
common fund with the intention of dividing the profits among themselves
have waived his right to cross-examine witnesses and to present evidence on his behalf,
(Article 1767, New Civil Code). 13
because of his failure to appear in subsequent hearings. Lim Tong Lim, on the other hand, filed
an Answer with Counterclaim and Crossclaim and moved for the lifting of the Writ of
Attachment. 6 The trial court maintained the Writ, and upon motion of private respondent, Hence, petitioner brought this recourse before this Court. 14
ordered the sale of the fishing nets at a public auction. Philippine Fishing Gear Industries won
the bidding and deposited with the said court the sales proceeds of P900,000. 7
The Issues

On November 18, 1992, the trial court rendered its Decision, ruling that Philippine Fishing Gear
Industries was entitled to the Writ of Attachment and that Chua, Yao and Lim, as general In his Petition and Memorandum, Lim asks this Court to reverse the assailed Decision on the
partners, were jointly liable to pay respondent. 8 following grounds:

The trial court ruled that a partnership among Lim, Chua and Yao existed based (1) on the I THE COURT OF APPEALS ERRED IN HOLDING, BASED ON A
testimonies of the witnesses presented and (2) on a Compromise Agreement executed by the COMPROMISE AGREEMENT THAT CHUA, YAO AND PETITIONER LIM
ENTERED INTO IN A SEPARATE CASE, THAT A PARTNERSHIP
three 9 in Civil Case No. 1492-MN which Chua and Yao had brought against Lim in the RTC of
Malabon, Branch 72, for (a) a declaration of nullity of commercial documents; (b) a reformation AGREEMENT EXISTED AMONG THEM.
of contracts; (c) a declaration of ownership of fishing boats; (d) an injunction and (e)
damages. 10 The Compromise Agreement provided: II SINCE IT WAS ONLY CHUA WHO REPRESENTED THAT HE WAS
ACTING FOR OCEAN QUEST FISHING CORPORATION WHEN HE
a) That the parties plaintiffs & Lim Tong Lim agree to BOUGHT THE NETS FROM PHILIPPINE FISHING, THE COURT OF
APPEALS WAS UNJUSTIFIED IN IMPUTING LIABILITY TO PETITIONER
have the four (4) vessels sold in the amount of
P5,750,000.00 including the fishing net. This LIM AS WELL.
P5,750,000.00 shall be applied as full payment for
P3,250,000.00 in favor of JL Holdings Corporation III THE TRIAL COURT IMPROPERLY ORDERED THE SEIZURE AND
and/or Lim Tong Lim; ATTACHMENT OF PETITIONER LIM'S GOODS.
In determining whether petitioner may be held liable for the fishing nets and floats from (5) That Lim, Chua and Yao agreed that the refurbishing, re-equipping,
respondent, the Court must resolve this key issue: whether by their acts, Lim, Chua and Yao repairing, dry docking and other expenses for the boats would be
could be deemed to have entered into a partnership. shouldered by Chua and Yao;

This Court's Ruling (6) That because of the "unavailability of funds," Jesus Lim again extended
a loan to the partnership in the amount of P1 million secured by a check,
because of which, Yao and Chua entrusted the ownership papers of two
The Petition is devoid of merit.
other boats, Chua's FB Lady Anne Mel and Yao's FB Tracy to Lim Tong
Lim.
First and Second Issues:
(7) That in pursuance of the business agreement, Peter Yao and Antonio
Existence of a Partnership Chua bought nets from Respondent Philippine Fishing Gear, in behalf of
and Petitioner's Liability "Ocean Quest Fishing Corporation," their purported business name.

In arguing that he should not be held liable for the equipment purchased from respondent,
(8) That subsequently, Civil Case No. 1492-MN was filed in the Malabon
petitioner controverts the CA finding that a partnership existed between him, Peter Yao and
RTC, Branch 72 by Antonio Chua and Peter Yao against Lim Tong Lim for
Antonio Chua. He asserts that the CA based its finding on the Compromise Agreement alone.
(a) declaration of nullity of commercial documents; (b) reformation of
Furthermore, he disclaims any direct participation in the purchase of the nets, alleging that the
contracts; (c) declaration of ownership of fishing boats; (4) injunction; and
negotiations were conducted by Chua and Yao only, and that he has not even met the
(e) damages.
representatives of the respondent company. Petitioner further argues that he was a lessor, not a
partner, of Chua and Yao, for the "Contract of Lease " dated February 1, 1990, showed that he
had merely leased to the two the main asset of the purported partnership — the fishing boat F/B (9) That the case was amicably settled through a Compromise Agreement
Lourdes. The lease was for six months, with a monthly rental of P37,500 plus 25 percent of the executed between the parties-litigants the terms of which are already
gross catch of the boat. enumerated above.

We are not persuaded by the arguments of petitioner. The facts as found by the two lower courts From the factual findings of both lower courts, it is clear that Chua, Yao and Lim had decided to
clearly showed that there existed a partnership among Chua, Yao and him, pursuant to Article engage in a fishing business, which they started by buying boats worth P3.35 million, financed
1767 of the Civil Code which provides: by a loan secured from Jesus Lim who was petitioner's brother. In their Compromise Agreement,
they subsequently revealed their intention to pay the loan with the proceeds of the sale of the
boats, and to divide equally among them the excess or loss. These boats, the purchase and the
Art. 1767 — By the contract of partnership, two or more persons bind
repair of which were financed with borrowed money, fell under the term "common fund" under
themselves to contribute money, property, or industry to a common fund,
Article 1767. The contribution to such fund need not be cash or fixed assets; it could be an
with the intention of dividing the profits among themselves.
intangible like credit or industry. That the parties agreed that any loss or profit from the sale and
operation of the boats would be divided equally among them also shows that they had indeed
Specifically, both lower courts ruled that a partnership among the three existed based on the formed a partnership.
following factual findings: 15
Moreover, it is clear that the partnership extended not only to the purchase of the boat, but also
(1) That Petitioner Lim Tong Lim requested Peter Yao who was engaged in to that of the nets and the floats. The fishing nets and the floats, both essential to fishing, were
commercial fishing to join him, while Antonio Chua was already Yao's obviously acquired in furtherance of their business. It would have been inconceivable for Lim to
partner; involve himself so much in buying the boat but not in the acquisition of the aforesaid equipment,
without which the business could not have proceeded.
(2) That after convening for a few times, Lim, Chua, and Yao verbally
agreed to acquire two fishing boats, the FB Lourdes and the FB Nelson for Given the preceding facts, it is clear that there was, among petitioner, Chua and Yao, a
the sum of P3.35 million; partnership engaged in the fishing business. They purchased the boats, which constituted the
main assets of the partnership, and they agreed that the proceeds from the sales and operations
thereof would be divided among them.
(3) That they borrowed P3.25 million from Jesus Lim, brother of Petitioner
Lim Tong Lim, to finance the venture.
We stress that under Rule 45, a petition for review like the present case should involve only
questions of law. Thus, the foregoing factual findings of the RTC and the CA are binding on this
(4) That they bought the boats from CMF Fishing Corporation, which
Court, absent any cogent proof that the present action is embraced by one of the exceptions to
executed a Deed of Sale over these two (2) boats in favor of Petitioner Lim the rule. 16 In assailing the factual findings of the two lower courts, petitioner effectively goes
Tong Lim only to serve as security for the loan extended by Jesus Lim; beyond the bounds of a petition for review under Rule 45.
Compromise Agreement general partners for all debts, liabilities and damages incurred or arising as
Not the Sole Basis of Partnership a result thereof: Provided however, That when any such ostensible
corporation is sued on any transaction entered by it as a corporation or on
Petitioner argues that the appellate court's sole basis for assuming the existence of a any tort committed by it as such, it shall not be allowed to use as a defense
partnership was the Compromise Agreement. He also claims that the settlement was entered its lack of corporate personality.
into only to end the dispute among them, but not to adjudicate their preexisting rights and
obligations. His arguments are baseless. The Agreement was but an embodiment of the
One who assumes an obligation to an ostensible corporation as such,
relationship extant among the parties prior to its execution.
cannot resist performance thereof on the ground that there was in fact no
corporation.
A proper adjudication of claimants' rights mandates that courts must review and thoroughly
appraise all relevant facts. Both lower courts have done so and have found, correctly, a
Thus, even if the ostensible corporate entity is proven to be legally nonexistent, a party may be
preexisting partnership among the parties. In implying that the lower courts have decided on the
estopped from denying its corporate existence. "The reason behind this doctrine is obvious —
basis of one piece of document alone, petitioner fails to appreciate that the CA and the RTC
an unincorporated association has no personality and would be incompetent to act and
delved into the history of the document and explored all the possible consequential combinations
appropriate for itself the power and attributes of a corporation as provided by law; it cannot
in harmony with law, logic and fairness. Verily, the two lower courts' factual findings mentioned
create agents or confer authority on another to act in its behalf; thus, those who act or purport to
above nullified petitioner's argument that the existence of a partnership was based only on the
act as its representatives or agents do so without authority and at their own risk. And as it is an
Compromise Agreement.
elementary principle of law that a person who acts as an agent without authority or without a
principal is himself regarded as the principal, possessed of all the right and subject to all the
Petitioner Was a Partner, liabilities of a principal, a person acting or purporting to act on behalf of a corporation which has
Not a Lessor no valid existence assumes such privileges and obligations and becomes personally liable for
contracts entered into or for other acts performed as such agent. 17
We are not convinced by petitioner's argument that he was merely the lessor of the boats to
Chua and Yao, not a partner in the fishing venture. His argument allegedly finds support in the
The doctrine of corporation by estoppel may apply to the alleged corporation and to a third party.
Contract of Lease and the registration papers showing that he was the owner of the boats,
In the first instance, an unincorporated association, which represented itself to be a corporation,
including F/B Lourdes where the nets were found.
will be estopped from denying its corporate capacity in a suit against it by a third person who
relied in good faith on such representation. It cannot allege lack of personality to be sued to
His allegation defies logic. In effect, he would like this Court to believe that he consented to the evade its responsibility for a contract it entered into and by virtue of which it received advantages
sale of his own boats to pay a debt of Chua and Yao, with the excess of the proceeds to be and benefits.
divided among the three of them. No lessor would do what petitioner did. Indeed, his consent to
the sale proved that there was a preexisting partnership among all three.
On the other hand, a third party who, knowing an association to be unincorporated, nonetheless
treated it as a corporation and received benefits from it, may be barred from denying its
Verily, as found by the lower courts, petitioner entered into a business agreement with Chua and corporate existence in a suit brought against the alleged corporation. In such case, all those who
Yao, in which debts were undertaken in order to finance the acquisition and the upgrading of the benefited from the transaction made by the ostensible corporation, despite knowledge of its legal
vessels which would be used in their fishing business. The sale of the boats, as well as the defects, may be held liable for contracts they impliedly assented to or took advantage of.
division among the three of the balance remaining after the payment of their loans, proves
beyond cavil that F/B Lourdes, though registered in his name, was not his own property but an
There is no dispute that the respondent, Philippine Fishing Gear Industries, is entitled to be paid
asset of the partnership. It is not uncommon to register the properties acquired from a loan in the
for the nets it sold. The only question here is whether petitioner should be held jointly 18 liable
name of the person the lender trusts, who in this case is the petitioner himself. After all, he is the
with Chua and Yao. Petitioner contests such liability, insisting that only those who dealt in the
brother of the creditor, Jesus Lim.
name of the ostensible corporation should be held liable. Since his name does not appear on
any of the contracts and since he never directly transacted with the respondent corporation,
We stress that it is unreasonable — indeed, it is absurd — for petitioner to sell his property to ergo, he cannot be held liable.
pay a debt he did not incur, if the relationship among the three of them was merely that of lessor-
lessee, instead of partners.
Unquestionably, petitioner benefited from the use of the nets found inside F/B Lourdes, the boat
which has earlier been proven to be an asset of the partnership. He in fact questions the
Corporation by Estoppel attachment of the nets, because the Writ has effectively stopped his use of the fishing vessel.

Petitioner argues that under the doctrine of corporation by estoppel, liability can be imputed only It is difficult to disagree with the RTC and the CA that Lim, Chua and Yao decided to form a
to Chua and Yao, and not to him. Again, we disagree. corporation. Although it was never legally formed for unknown reasons, this fact alone does not
preclude the liabilities of the three as contracting parties in representation of it. Clearly, under the
law on estoppel, those acting on behalf of a corporation and those benefited by it, knowing it to
Sec. 21 of the Corporation Code of the Philippines provides:
be without valid existence, are held liable as general partners.

Sec. 21. Corporation by estoppel. — All persons who assume to act as a


corporation knowing it to be without authority to do so shall be liable as
Technically, it is true that petitioner did not directly act on behalf of the corporation. However,
having reaped the benefits of the contract entered into by persons with whom he previously had
an existing relationship, he is deemed to be part of said association and is covered by the scope
of the doctrine of corporation by estoppel. We reiterate the ruling of the Court in Alonso
v. Villamor: 19

A litigation is not a game of technicalities in which one, more deeply


schooled and skilled in the subtle art of movement and position, entraps and
destroys the other. It is, rather, a contest in which each contending party
fully and fairly lays before the court the facts in issue and then, brushing
aside as wholly trivial and indecisive all imperfections of form and
technicalities of procedure, asks that justice be done upon the merits.
Lawsuits, unlike duels, are not to be won by a rapier's thrust. Technicality,
when it deserts its proper office as an aid to justice and becomes its great
hindrance and chief enemy, deserves scant consideration from courts.
There should be no vested rights in technicalities.

Third Issue:

Validity of Attachment

Finally, petitioner claims that the Writ of Attachment was improperly issued against the nets. We
agree with the Court of Appeals that this issue is now moot and academic. As previously
discussed, F/B Lourdes was an asset of the partnership and that it was placed in the name of
petitioner, only to assure payment of the debt he and his partners owed. The nets and the floats
were specifically manufactured and tailor-made according to their own design, and were bought
and used in the fishing venture they agreed upon. Hence, the issuance of the Writ to assure the
payment of the price stipulated in the invoices is proper. Besides, by specific agreement,
ownership of the nets remained with Respondent Philippine Fishing Gear, until full payment
thereof.

WHEREFORE, the Petition is DENIED and the assailed Decision AFFIRMED. Costs against
petitioner.

SO ORDERED.
G.R. No. L-12719 May 31, 1962 1. Whether the respondent Club is liable for the payment of the sum of 12,068.84, as fixed and
percentage taxes and surcharges prescribed in sections 182, 183 and 191 of the Tax Code,
under which the assessment was made, in connection with the operation of its bar and
THE COLLECTOR OF INTERNAL REVENUE, petitioner,
restaurant, during the periods mentioned above; and
vs.
THE CLUB FILIPINO, INC. DE CEBU, respondent.
2. Whether it is liable for the payment of the sum of P500.00 as compromise penalty.
PAREDES, J.:
Section 182, of the Tax Code states, "Unless otherwise provided, every person engaging in a
business on which the percentage tax is imposed shall pay in full a fixed annual tax of ten pesos
This is a petition to review the decision of the Court of Tax Appeals, reversing the decision of the
for each calendar year or fraction thereof in which such person shall engage in said business."
Collector of Internal Revenue, assessing against and demanding from the "Club Filipino, Inc. de
Section 183 provides in general that "the percentage taxes on business shall be payable at the
Cebu", the sum of P12,068.84 as fixed and percentage taxes, surcharge and compromise
end of each calendar quarter in the amount lawfully due on the business transacted during each
penalty, allegedly due from it as a keeper of bar and restaurant.
quarter; etc." And section 191, same Tax Code, provides "Percentage tax . . . Keepers of
restaurants, refreshment parlors and other eating places shall pay a tax three per centum, and
As found by the Court of Tax Appeals, the "Club Filipino, Inc. de Cebu," (Club, for short), is a keepers of bar and cafes where wines or liquors are served five per centum of their gross
civic corporation organized under the laws of the Philippines with an original authorized capital receipts . . .". It has been held that the liability for fixed and percentage taxes, as provided by
stock of P22,000.00, which was subsequently increased to P200,000.00, among others, to it these sections, does not ipso facto attach by mere reason of the operation of a bar and
"proporcionar, operar, y mantener un campo de golf, tenis, gimnesio (gymnasiums), juego de restaurant. For the liability to attach, the operator thereof must be engaged in the business as a
bolos (bowling alleys), mesas de billar y pool, y toda clase de juegos no prohibidos por leyes barkeeper and restaurateur. The plain and ordinary meaning of business is restricted to activities
generales y ordenanzas generales; y desarollar y cultivar deportes de toda clase y or affairs where profit is the purpose or livelihood is the motive, and the term business when
denominacion cualquiera para el recreo y entrenamiento saludable de sus miembros y used without qualification, should be construed in its plain and ordinary meaning, restricted to
accionistas" (sec. 2, Escritura de Incorporacion del Club Filipino, Inc. Exh. A). Neither in the activities for profit or livelihood (The Coll. of Int. Rev. v. Manila Lodge No. 761 of the BPOE
articles or by-laws is there a provision relative to dividends and their distribution, although it is [Manila Elks Club] & Court of Tax Appeals, G.R. No. L-11176, June 29, 1959, giving full
covenanted that upon its dissolution, the Club's remaining assets, after paying debts, shall be definitions of the word "business"; Coll. of Int. Rev. v. Sweeney, et al. [International Club of Iloilo,
donated to a charitable Philippine Institution in Cebu (Art. 27, Estatutos del Club, Exh. A-a.). Inc.], G.R. No. L-12178, Aug. 21, 1959, the facts of which are similar to the ones at bar; Manila
Polo Club v. B. L. Meer, etc., No. L-10854, Jan. 27, 1960).
The Club owns and operates a club house, a bowling alley, a golf course (on a lot leased from
the government), and a bar-restaurant where it sells wines and liquors, soft drinks, meals and Having found as a fact that the Club was organized to develop and cultivate sports of all class
short orders to its members and their guests. The bar-restaurant was a necessary incident to the and denomination, for the healthful recreation and entertainment of its stockholders and
operation of the club and its golf-course. The club is operated mainly with funds derived from members; that upon its dissolution, its remaining assets, after paying debts, shall be donated to
membership fees and dues. Whatever profits it had, were used to defray its overhead expenses a charitable Philippine Institution in Cebu; that it is operated mainly with funds derived from
and to improve its golf-course. In 1951. as a result of a capital surplus, arising from the re- membership fees and dues; that the Club's bar and restaurant catered only to its members and
valuation of its real properties, the value or price of which increased, the Club declared stock their guests; that there was in fact no cash dividend distribution to its stockholders and that
dividends; but no actual cash dividends were distributed to the stockholders. In 1952, a BIR whatever was derived on retail from its bar and restaurant was used to defray its overall
agent discovered that the Club has never paid percentage tax on the gross receipts of its bar overhead expenses and to improve its golf-course (cost-plus-expenses-basis), it stands to
and restaurant, although it secured B-4, B-9(a) and B-7 licenses. In a letter dated December 22, reason that the Club is not engaged in the business of an operator of bar and restaurant (same
1852, the Collector of Internal Revenue assessed against and demanded from the Club, the authorities, cited above).
following sums: —
It is conceded that the Club derived profit from the operation of its bar and restaurant, but such
fact does not necessarily convert it into a profit-making enterprise. The bar and restaurant are
As percentage tax on its gross receipts necessary adjuncts of the Club to foster its purposes and the profits derived therefrom are
during the tax years 1946 to 1951 P9,599.07 necessarily incidental to the primary object of developing and cultivating sports for the healthful
recreation and entertainment of the stockholders and members. That a Club makes some profit,
Surcharge therein 2,399.77 does not make it a profit-making Club. As has been remarked a club should always strive,
whenever possible, to have surplus (Jesus Sacred Heart College v. Collector of Int. Rev., G.R.
As fixed tax for the years 1946 to 1952 70.00 No. L-6807, May 24, 1954; Collector of Int. Rev. v. Sinco Educational Corp., G.R. No. L-9276,
Oct. 23, 1956).1äwphï1.ñët
Compromise penalty 500.00

It is claimed that unlike the two cases just cited (supra), which are non-stock, the appellee Club
The Club wrote the Collector, requesting for the cancellation of the assessment. The request is a stock corporation. This is unmeritorious. The facts that the capital stock of the respondent
having been denied, the Club filed the instant petition for review. Club is divided into shares, does not detract from the finding of the trial court that it is not
engaged in the business of operator of bar and restaurant. What is determinative of whether or
not the Club is engaged in such business is its object or purpose, as stated in its articles and by-
The dominant issues involved in this case are twofold: laws. It is a familiar rule that the actual purpose is not controlled by the corporate form or by the
commercial aspect of the business prosecuted, but may be shown by extrinsic evidence,
including the by-laws and the method of operation. From the extrinsic evidence adduced, the
Tax Court concluded that the Club is not engaged in the business as a barkeeper and
restaurateur.

Moreover, for a stock corporation to exist, two requisites must be complied with, to wit: (1) a
capital stock divided into shares and (2) an authority to distribute to the holders of such shares,
dividends or allotments of the surplus profits on the basis of the shares held (sec. 3, Act No.
1459). In the case at bar, nowhere in its articles of incorporation or by-laws could be found an
authority for the distribution of its dividends or surplus profits. Strictly speaking, it cannot,
therefore, be considered a stock corporation, within the contemplation of the corporation law.

A tax is a burden, and, as such, it should not be deemed imposed upon fraternal, civic, non-
profit, nonstock organizations, unless the intent to the contrary is manifest and patent" (Collector
v. BPOE Elks Club, et al., supra), which is not the case in the present appeal.

Having arrived at the conclusion that respondent Club is not engaged in the business as an
operator of a bar and restaurant, and therefore, not liable for fixed and percentage taxes, it
follows that it is not liable for any penalty, much less of a compromise penalty.

WHEREFORE, the decision appealed from is affirmed without costs.


G.R. No. L-43350 December 23, 1937 In dismissing the complaint against the defendant, the court below, reached the conclusion that
Exhibit B is invalid because of vice in consent and repugnancy to law. While we do not agree
with this conclusion, we have however voted to affirm the judgment appealed from the reasons
CAGAYAN FISHING DEVELOPMENT CO., INC., plaintiff-appellant,
which we shall presently state.
vs.
TEODORO SANDIKO, defendant-appellee.
The transfer made by Tabora to the Cagayan fishing Development Co., Inc., plaintiff herein, was
affected on May 31, 1930 (Exhibit A) and the actual incorporation of said company was affected
LAUREL, J.:
later on October 22, 1930 (Exhibit 2). In other words, the transfer was made almost five months
before the incorporation of the company. Unquestionably, a duly organized corporation has the
This is an appeal from a judgment of the Court of First Instance of Manila absolving the power to purchase and hold such real property as the purposes for which such corporation was
defendant from the plaintiff's complaint. formed may permit and for this purpose may enter into such contracts as may be necessary
(sec. 13, pars. 5 and 9, and sec. 14, Act No. 1459). But before a corporation may be said to be
lawfully organized, many things have to be done. Among other things, the law requires the filing
Manuel Tabora is the registered owner of four parcels of land situated in the barrio of Linao,
of articles of incorporation (secs. 6 et seq., Act. No. 1459). Although there is a presumption that
town of Aparri, Province of Cagayan, as evidenced by transfer certificate of title No. 217 of the all the requirements of law have been complied with (sec. 334, par. 31 Code of Civil Procedure),
land records of Cagayan, a copy of which is in evidence as Exhibit 1. To guarantee the payment in the case before us it can not be denied that the plaintiff was not yet incorporated when it
of a loan in the sum of P8,000, Manuel Tabora, on August 14, 1929, executed in favor of the
entered into a contract of sale, Exhibit A. The contract itself referred to the plaintiff as "una
Philippine National Bank a first mortgage on the four parcels of land above-mentioned. A second sociedad en vias de incorporacion." It was not even a de facto corporation at the time. Not being
mortgage in favor of the same bank was in April of 1930 executed by Tabora over the same in legal existence then, it did not possess juridical capacity to enter into the contract.
lands to guarantee the payment of another loan amounting to P7,000. A third mortgage on the
same lands was executed on April 16, 1930 in favor of Severina Buzon to whom Tabora was
indebted in the sum of P2,9000. These mortgages were registered and annotations thereof Corporations are creatures of the law, and can only come into existence in the manner
appear at the back of transfer certificate of title No. 217. prescribed by law. As has already been stated, general law authorizing the formation
of corporations are general offers to any persons who may bring themselves within
their provisions; and if conditions precedent are prescribed in the statute, or certain
On May 31, 1930, Tabora executed a public document entitled "Escritura de Transpaso de acts are required to be done, they are terms of the offer, and must be complied with
Propiedad Inmueble" (Exhibit A) by virtue of which the four parcels of land owned by him was
substantially before legal corporate existence can be acquired. (14 C. J., sec. 111, p.
sold to the plaintiff company, said to under process of incorporation, in consideration of one peso 118.)
(P1) subject to the mortgages in favor of the Philippine National Bank and Severina Buzon and,
to the condition that the certificate of title to said lands shall not be transferred to the name of the
plaintiff company until the latter has fully and completely paid Tabora's indebtedness to the That a corporation should have a full and complete organization and existence as an
Philippine National Bank. entity before it can enter into any kind of a contract or transact any business, would
seem to be self evident. . . . A corporation, until organized, has no being, franchises or
faculties. Nor do those engaged in bringing it into being have any power to bind it by
The plaintiff company filed its article incorporation with the Bureau of Commerce and Industry on contract, unless so authorized by the charter there is not a corporation nor does it
October 22, 1930 (Exhibit 2). A year later, on October 28, 1931, the board of directors of said possess franchise or faculties for it or others to exercise, until it acquires a complete
company adopted a resolution (Exhibit G) authorizing its president, Jose Ventura, to sell the four existence. (Gent vs. Manufacturers and Merchant's Mutual Insurance Company, 107
parcels of lands in question to Teodoro Sandiko for P42,000. Exhibits B, C and D were Ill., 652, 658.)
thereafter made and executed. Exhibit B is a deed of sale executed before a notary public by the
terms of which the plaintiff sold ceded and transferred to the defendant all its right, titles, and
interest in and to the four parcels of land described in transfer certificate in turn obligated himself Boiled down to its naked reality, the contract here (Exhibit A) was entered into not between
to shoulder the three mortgages hereinbefore referred to. Exhibit C is a promisory note for Manuel Tabora and a non-existent corporation but between the Manuel Tabora as owner of the
P25,300. drawn by the defendant in favor of the plaintiff, payable after one year from the date four parcels of lands on the one hand and the same Manuel Tabora, his wife and others, as
thereof. Exhibit D is a deed of mortgage executed before a notary public in accordance with mere promoters of a corporations on the other hand. For reasons that are self-evident, these
which the four parcels of land were given a security for the payment of the promissory note, promoters could not have acted as agent for a projected corporation since that which no legal
Exhibit C. All these three instrument were dated February 15, 1932. existence could have no agent. A corporation, until organized, has no life and therefore no
faculties. It is, as it were, a child in ventre sa mere. This is not saying that under no
circumstances may the acts of promoters of a corporation be ratified by the corporation if and
The defendant having failed to pay the sum stated in the promissory note, plaintiff, on January
when subsequently organized. There are, of course, exceptions (Fletcher Cyc. of Corps.,
25, 1934, brought this action in the Court of First Instance of Manila praying that judgment be permanent edition, 1931, vol. I, secs. 207 et seq.), but under the peculiar facts and
rendered against the defendant for the sum of P25,300, with interest at legal rate from the date circumstances of the present case we decline to extend the doctrine of ratification which would
of the filing of the complaint, and the costs of the suits. After trial, the court below, on December
result in the commission of injustice or fraud to the candid and unwary.(Massachusetts rule,
18, 1934, rendered judgment absolving the defendant, with costs against the plaintiff. Plaintiff Abbott vs. Hapgood, 150 Mass., 248; 22 N. E. 907, 908; 5 L. R. A., 586; 15 Am. St. Rep., 193;
presented a motion for new trial on January 14, 1935, which motion was denied by the trial court citing English cases; Koppel vs. Massachusetts Brick Co., 192 Mass., 223; 78 N. E., 128;
on January 19 of the same year. After due exception and notice, plaintiff has appealed to this
Holyoke Envelope Co., vs. U. S. Envelope Co., 182 Mass., 171; 65 N. E., 54.) It should be
court and makes an assignment of various errors. observed that Manuel Tabora was the registered owner of the four parcels of land, which he
succeeded in mortgaging to the Philippine National Bank so that he might have the necessary
funds with which to convert and develop them into fishery. He appeared to have met with
financial reverses. He formed a corporation composed of himself, his wife, and a few others.
From the articles of incorporation, Exhibit 2, it appears that out of the P48,700, amount of capital
stock subscribed, P45,000 was subscribed by Manuel Tabora himself and P500 by his wife,
Rufina Q. de Tabora; and out of the P43,300, amount paid on subscription, P42,100 is made to
appear as paid by Tabora and P200 by his wife. Both Tabora and His wife were directors and
the latter was treasurer as well. In fact, to this day, the lands remain inscribed in Tabora's name.
The defendant always regarded Tabora as the owner of the lands. He dealt with Tabora directly.
Jose Ventura, president of the plaintiff corporation, intervened only to sign the contract, Exhibit
B, in behalf of the plaintiff. Even the Philippine National Bank, mortgagee of the four parcels of
land, always treated Tabora as the owner of the same. (See Exhibits E and F.) Two civil suits
(Nos. 1931 and 38641) were brought against Tabora in the Court of First Instance of Manila and
in both cases a writ of attachment against the four parcels of land was issued. The Philippine
National Bank threatened to foreclose its mortgages. Tabora approached the defendant Sandiko
and succeeded in the making him sign Exhibits B, C, and D and in making him, among other
things, assume the payment of Tabora's indebtedness to the Philippine National Bank. The
promisory note, Exhibit C, was made payable to the plaintiff company so that it may not attached
by Tabora's creditors, two of whom had obtained writs of attachment against the four parcels of
land.

If the plaintiff corporation could not and did not acquire the four parcels of land here involved, it
follows that it did not possess any resultant right to dispose of them by sale to the defendant,
Teodoro Sandiko.

Some of the members of this court are also of the opinion that the transfer from Manuel Tabora
to the Cagayan Fishing Development Company, Inc., which transfer is evidenced by Exhibit A,
was subject to a condition precedent (condicion suspensiva), namely, the payment of the
mortgage debt of said Tabora to the Philippine National Bank, and that this condition not having
been complied with by the Cagayan Fishing Development Company, Inc., the transfer was
ineffective. (Art. 1114, Civil Code; Wise & Co. vs. Kelly and Lim, 37 Phil., 696; Manresa, vol. 8,
p. 141.) However, having arrived at the conclusion that the transfer by Manuel Tabora to the
Cagayan Fishing Development Company, Inc. was null because at the time it was affected the
corporation was non-existent, we deem it unnecessary to discuss this point.lawphil.net

The decision of the lower court is accordingly affirmed, with costs against the appellant. So
Ordered.
G.R. No. 137592 December 12, 2001 any corporation, association, and/or partnership presently registered with the
Commission.
ANG MGA KAANIB SA IGLESIA NG DIOS KAY KRISTO HESUS, H.S.K. SA BANSANG
PILIPINAS, INC.,petitioner, Let a copy of this Decision be furnished the Records Division and the Corporate and
vs. Legal Department [CLD] of this Commission for their records, reference and/or for
IGLESIA NG DIOS KAY CRISTO JESUS, HALIGI AT SUHAY NG whatever requisite action, if any, to be undertaken at their end.
KATOTOHANAN, respondent.
SO ORDERED.7
YNARES-SANTIAGO, J.:
Petitioner appealed to the SEC En Banc, where its appeal was docketed as SEC-AC No. 539. In
This is a petition for review assailing the Decision dated October 7, 19971 and the Resolution a decision dated March 4, 1996, the SEC En Banc affirmed the above decision, upon a finding
dated February 16, 19992 of the Court of Appeals in CA-G.R. SP No. 40933, which affirmed the that petitioner's corporate name was identical or confusingly or deceptively similar to that of
Decision of the Securities and Exchange and Commission (SEC) in SEC-AC No. 539.3 respondent's corporate name.8

Respondent Iglesia ng Dios Kay Cristo Jesus, Haligi at Suhay ng Katotohanan (Church of God Petitioner filed a petition for review with the Court of Appeals. On October 7, 1997, the Court of
in Christ Jesus, the Pillar and Ground of Truth),4 is a non-stock religious society or corporation Appeals rendered the assailed decision affirming the decision of the SEC En Banc. Petitioner's
registered in 1936. Sometime in 1976, one Eliseo Soriano and several other members of motion for reconsideration was denied by the Court of Appeals on February 16, 1992.
respondent corporation disassociated themselves from the latter and succeeded in registering
on March 30, 1977 a new non-stock religious society or corporation, named Iglesia ng Dios Kay
Hence, the instant petition for review, raising the following assignment of errors:
Kristo Hesus, Haligi at Saligan ng Katotohanan.

I
On July 16, 1979, respondent corporation filed with the SEC a petition to compel the Iglesia ng
Dios Kay Kristo Hesus, Haligi at Saligan ng Katotohanan to change its corporate name, which
petition was docketed as SEC Case No. 1774. On May 4, 1988, the SEC rendered judgment in THE HONORABLE COURT OF APPEALS ERRED IN CONCLUDING THAT PETITIONER HAS
favor of respondent, ordering the Iglesia ng Dios Kay Kristo Hesus, Haligi at Saligan ng NOT BEEN DEPRIVED OF ITS RIGHT TO PROCEDURAL DUE PROCESS, THE
Katotohanan to change its corporate name to another name that is not similar or identical to any HONORABLE COURT OF APPEALS DISREGARDED THE JURISPRUDENCE APPLICABLE
name already used by a corporation, partnership or association registered with the TO THE CASE AT BAR AND INSTEAD RELIED ON TOTALLY INAPPLICABLE
Commission.5No appeal was taken from said decision. JURISPRUDENCE.

It appears that during the pendency of SEC Case No. 1774, Soriano, et al., caused the II
registration on April 25, 1980 of petitioner corporation, Ang Mga Kaanib sa Iglesia ng Dios Kay
Kristo Hesus, H.S.K, sa Bansang Pilipinas. The acronym "H.S.K." stands for Haligi at Saligan ng
Katotohanan.6 THE HONORABLE COURT OF APPEALS ERRED IN ITS INTERPRETATION OF THE CIVIL
CODE PROVISIONS ON EXTINCTIVE PRESCRIPTION, THEREBY RESULTING IN ITS
FAILURE TO FIND THAT THE RESPONDENT'S RIGHT OF ACTION TO INSTITUTE THE SEC
On March 2, 1994, respondent corporation filed before the SEC a petition, docketed as SEC CASE HAS SINCE PRESCRIBED PRIOR TO ITS INSTITUTION.
Case No. 03-94-4704, praying that petitioner be compelled to change its corporate name and be
barred from using the same or similar name on the ground that the same causes confusion
among their members as well as the public. III

THE HONORABLE COURT OF APPEALS FAILED TO CONSIDER AND PROPERLY APPLY


Petitioner filed a motion to dismiss on the ground of lack of cause of action. The motion to
dismiss was denied. Thereafter, for failure to file an answer, petitioner was declared in default THE EXCEPTIONS ESTABLISHED BY JURISPRUDENCE IN THE APPLICATION OF
and respondent was allowed to present its evidence ex parte. SECTION 18 OF THE CORPORATION CODE TO THE INSTANT CASE.

On November 20, 1995, the SEC rendered a decision ordering petitioner to change its corporate IV
name. The dispositive portion thereof reads:
THE HONORABLE COURT OF APPEALS FAILED TO PROPERLY APPRECIATE THE SCOPE
PREMISES CONSIDERED, judgment is hereby rendered in favor of the petitioner OF THE CONSTITUTIONAL GUARANTEE ON RELIGIOUS FREEDOM, THEREBY FAILING
(respondent herein). TO APPLY THE SAME TO PROTECT PETITIONER'S RIGHTS.9

Respondent Mga Kaanib sa Iglesia ng Dios Kay Kristo Jesus (sic), H.S.K. sa Bansang Invoking the case of Legarda v. Court of Appeals,10 petitioner insists that the decision of the
Pilipinas (petitioner herein) is hereby MANDATED to change its corporate name to Court of Appeals and the SEC should be set aside because the negligence of its former counsel
another not deceptively similar or identical to the same already used by the Petitioner,
of record, Atty. Joaquin Garaygay, in failing to file an answer after its motion to dismiss was misleading or likely to injure in the exercise of its corporate functions, regardless of intent, may
denied by the SEC, deprived them of their day in court. be prevented by the corporation having a prior right, by a suit for injunction against the new
corporation to prevent the use of the name.18
The contention is without merit. As a general rule, the negligence of counsel binds the client.
This is based on the rule that any act performed by a lawyer within the scope of his general or Petitioner claims that it complied with the aforecited SEC guideline by adding not only two but
implied authority is regarded as an act of his client.11 An exception to the foregoing is where the eight words to their registered name, to wit: "Ang Mga Kaanib" and "Sa Bansang Pilipinas,
reckless or gross negligence of the counsel deprives the client of due process of law. 12 Said Inc.," which, petitioner argues, effectively distinguished it from respondent corporation.
exception, however, does not obtain in the present case.
The additional words "Ang Mga Kaanib" and "Sa Bansang Pilipinas, Inc." in petitioner's name
In Legarda v. Court of Appeals, the effort of the counsel in defending his client's cause consisted are, as correctly observed by the SEC, merely descriptive of and also referring to the members,
in filing a motion for extension of time to file answer before the trial court. When his client was or kaanib, of respondent who are likewise residing in the Philippines. These words can hardly
declared in default, the counsel did nothing and allowed the judgment by default to become final serve as an effective differentiating medium necessary to avoid confusion or difficulty in
and executory. Upon the insistence of his client, the counsel filed a petition to annul the distinguishing petitioner from respondent. This is especially so, since both petitioner and
judgment with the Court of Appeals, which denied the petition, and again the counsel allowed respondent corporations are using the same acronym — H.S.K.;19 not to mention the fact that
the denial to become final and executory. This Court found the counsel grossly negligent and both are espousing religious beliefs and operating in the same place. Parenthetically, it is well to
consequently declared as null and void the decision adverse to his client. mention that the acronym H.S.K. used by petitioner stands for "Haligi at Saligan ng
Katotohanan."20
The factual antecedents of the case at bar are different. Atty. Garaygay filed before the SEC a
motion to dismiss on the ground of lack of cause of action. When his client was declared in Then, too, the records reveal that in holding out their corporate name to the public, petitioner
default for failure to file an answer, Atty. Garaygay moved for reconsideration and lifting of the highlights the dominant words "IGLESIA NG DIOS KAY KRISTO HESUS, HALIGI AT SALIGAN
order of default.13 After judgment by default was rendered against petitioner corporation, Atty. NG KATOTOHANAN," which is strikingly similar to respondent's corporate name, thus making it
Garaygay filed a motion for extension of time to appeal/motion for reconsideration, and even more evident that the additional words "Ang Mga Kaanib" and "Sa Bansang Pilipinas, Inc.",
thereafter a motion to set aside the decision.14 are merely descriptive of and pertaining to the members of respondent corporation. 21

Evidently, Atty. Garaygay was only guilty of simple negligence. Although he failed to file an Significantly, the only difference between the corporate names of petitioner and respondent are
answer that led to the rendition of a judgment by default against petitioner, his efforts were the words SALIGAN and SUHAY. These words are synonymous — both mean ground,
palpably real, albeit bereft of zeal.15 foundation or support. Hence, this case is on all fours with Universal Mills Corporation v.
Universal Textile Mills, Inc.,22 where the Court ruled that the corporate names Universal Mills
Corporation and Universal Textile Mills, Inc., are undisputably so similar that even under the test
Likewise, the issue of prescription, which petitioner raised for the first time on appeal to the
of "reasonable care and observation" confusion may arise.
Court of Appeals, is untenable. Its failure to raise prescription before the SEC can only be
construed as a waiver of that defense.16 At any rate, the SEC has the authority to de-register at
all times and under all circumstances corporate names which in its estimation are likely to spawn Furthermore, the wholesale appropriation by petitioner of respondent's corporate name cannot
confusion. It is the duty of the SEC to prevent confusion in the use of corporate names not only find justification under the generic word rule. We agree with the Court of Appeals' conclusion
for the protection of the corporations involved but more so for the protection of the public. 17 that a contrary ruling would encourage other corporations to adopt verbatim and register an
existing and protected corporate name, to the detriment of the public.
Section 18 of the Corporation Code provides:
The fact that there are other non-stock religious societies or corporations using the names
Church of the Living God, Inc., Church of God Jesus Christ the Son of God the Head, Church of
Corporate Name. — No corporate name may be allowed by the Securities and
God in Christ & By the Holy Spirit, and other similar names, is of no consequence. It does not
Exchange Commission if the proposed name is identical or deceptively or confusingly
authorize the use by petitioner of the essential and distinguishing feature of respondent's
similar to that of any existing corporation or to any other name already protected by
registered and protected corporate name.23
law or is patently deceptive, confusing or is contrary to existing laws. When a change
in the corporate name is approved, the Commission shall issue an amended certificate
of incorporation under the amended name. We need not belabor the fourth issue raised by petitioner. Certainly, ordering petitioner to
change its corporate name is not a violation of its constitutionally guaranteed right to religious
freedom. In so doing, the SEC merely compelled petitioner to abide by one of the SEC
Corollary thereto, the pertinent portion of the SEC Guidelines on Corporate Names states:
guidelines in the approval of partnership and corporate names, namely its undertaking to
manifest its willingness to change its corporate name in the event another person, firm, or entity
(d) If the proposed name contains a word similar to a word already used as part of the has acquired a prior right to the use of the said firm name or one deceptively or confusingly
firm name or style of a registered company, the proposed name must contain two similar to it.
other words different from the name of the company already registered;
WHEREFORE, in view of all the foregoing, the instant petition for review is DENIED. The
Parties organizing a corporation must choose a name at their peril; and the use of a name appealed decision of the Court of Appeals is AFFIRMED in toto. SO ORDERED.
similar to one adopted by another corporation, whether a business or a nonprofit organization, if
G.R. No. L-56763 December 15, 1982 would be served by dismissing the same and ordering that it be filed in Manila (Sy vs. Pineda,
CA-G.R. No. SP-10775). That decision was appealed to this Court.
JOHN SY and UNIVERSAL PARTS SUPPLY CORPORATION, petitioners,
vs. There is no question that the venue was improperly laid in this case. The place of business of
TYSON ENTERPRISES, INC., JUDGE GREGORIO G. PINEDA of the Court of First Instance plaintiff Tyson Enterprises, Inc., which for purposes of venue is considered as its residence (18
of Rizal, Pasig Branch XXI and COURT OF APPEALS, respondents. C.J.S 583; Clavecilla Radio system vs. Antillon, L-22238, February 18, 1967, 19 SCRA 379),
because a corporation has a personality separate and distinct from that of its officers and
stockholders.
AQUINO, J:

Consequently, the collection suit should have been filed in Manila, the residence of plaintiff
This is a case about the venue of a collection suit. On August 29, 1979, Tyson Enterprises, Inc.
corporation and the place designated in its sales invoice, or it could have been filed also in
filed against John Sy and Universal Parts Supply Corporation in the Court of First Instance of
Bacolod City, the residence of defendant Sy.
Rizal, Pasig Branch XXI, a complaint for the collection of P288,534.58 plus interest, attorney's
fees and litigation expenses (Civil Case No. 34302).
We hold that the trial court and the Court of Appeals erred in ruling that the defendants, now the
petitioners, waived their objection to the improper venue. As the trial court proceeded in defiance
It is alleged in the complaint that John Sy, doing business under the trade name, Universal Parts
of the Rules of Court in not dismissing the case, prohibition lies to restrain it from acting in the
Supply, is a resident of Fuentebella Subdivision, Bacolod City and that his co-defendant,
case (Enriquez vs. Macadaeg, 84 Phil. 674).
Universal Parts Supply Corporation, allegedly controlled by Sy, is doing business in Bacolod
City.
Section 4, Rule 4 of the Rules of Court provides that, "when improper venue is not objected to in
a motion to dismiss it is deemed waived" and it can no longer be pleaded as an affirmative
Curiously enough, there is no allegation in the complaint as to the office or place of business of
defense in the answer (Sec. 5, Rule 16).
plaintiff Tyson Enterprises, Inc., a firm actually doing business at 1024 Magdalena, now G.
Masangkay Street, Binondo, Manila (p. 59, Rollo).
In this case, the petitioners, before filing their answer, filed a motion to dismiss based on
improper venue. That motion was seasonably filed (Republic vs. Court of First Instance of
What is alleged is the postal address or residence of Dominador Ti, the president and general
Manila, L-30839, November 28, 1975, 68 SCRA 231, 239). The fact that they filed a motion for a
manager of plaintiff firm, which is at 26 Xavier Street, Greenhills Subdivision, San Juan, Rizal.
bill of particulars before they filed their motion to dismiss did not constitute a waiver of their
The evident purpose of alleging that address and not mentioning the place of business of
objection to the venue.
plaintiff firm was to justify the filing of the suit in Pasig, Rizal instead of in Manila.

It should be noted that the provision of Section 377 of the Code of Civil Procedure that "the
Defendant Sy and Universal Parts Supply Corporation first filed a motion for extension of time to
failure of a defendant to object to the venue of the action at the time of entering his appearance
file their answer and later a motion for a bill of particulars. The latter motion was denied. Then,
in the action shall be deemed a waiver on his part of all objection to the place or tribunal in which
they filed a motion to dismiss on the ground of improper venue.
the action is brought" is not found in the Rules of Court.

They invoked the provision of section 2(b), Rule 4 of the Rules of Court that personal actions
And the provision of section 4, Rule 5 of the 1940 Rules of Court that "when improper venue is
"may be commenced and tried where the defendant or any of the defendants resides or may be
not objected to prior to the trial, it is deemed waived" is not reproduced in the present Rules of
found, or where the plaintiffs or any of the plaintiffs resides, at the election of the plaintiff."
Court.

To strengthen that ground, they also cited the stipulation in the sales invoice that "the parties
To repeat, what section 4 of Rule 4 of the present Rules of court provides is that the objection to
expressly submit to the jurisdiction of the Courts of the City of Manila for any legal action arising
improper venue should be raised in a motion to dismiss seasonably filed and, if not so raised,
out of" the transaction which stipulation is quoted in paragraph 4 of plaintiff's complaint.
then the said objection is waived. Section 4 does not provide that the objection based on
improper venue should be interposed by means of a special appearance or before any pleading
The plaintiff opposed the motion to dismiss on the ground that the defendants had waived the is filed.
objection based on improper venue because they had previously filed a motion for a bill of
particulars which was not granted. The trial court denied the motion to dismiss on the ground
The rules on venue, like the other procedural rules, are designed to insure a just and orderly
that by filing a motion for a bill of particulars the defendants waived their objection to the venue.
administration of justice or the impartial and evenhanded determination of every action and
That denial order was assailed in a petition for certiorari and prohibition in the Court of Appeals
proceeding. Obviously, this objective will not be attained if the plaintiff is given unrestricted
which issued on July 29, 1980 a restraining order, enjoining respondent judge from acting on the
freedom to choose the court where he may file his complaint or petition.
case. He disregarded the restraining order (p. 133, Rollo).

The choice of venue should not be left to the plaintiff's whim or caprice. He may be impelled by
The Appellate Court in its decision of October 6, 1980 dismissed the petition. It ruled that the
some ulterior motivation in choosing to file a case in a particular court even if not allowed by the
parties did not intend Manila as the exclusive venue of the actions arising under their
rules on venue.
transactions and that since the action was filed in Pasig, which is near Manila, no useful purpose
As perspicaciously observed by Justice Moreland, the purpose of procedure is not to restrict the
court's jurisdiction over the subject matter but to give it effective facility "in righteous action", "to
facilitate and promote the administration of justice" or to insure "just judgments" by means of a
fair hearing. If that objective is not achieved, then "the administration of justice becomes
incomplete and unsatisfactory and lays itself open to grave criticism." (Manila Railroad Co. vs.
Attorney General, 20 Phil. 523, 530.)

The case of Marquez Lim Cay vs. Del Rosario, 55 Phil. 962, does not sustain the trial court's
order of denial because in that case the defendants, before filing a motion to dismiss on the
ground of improper venue, interposed a demurrer on the ground that the complaint does not
state a cause of action. Then, they filed a motion for the dissolution of an attachment, posted a
bond for its dissolution and later filed a motion for the assessment of the damages caused by the
attachment. All those acts constituted a submission to the trial court's jurisdiction and a waiver of
the objection based on improper venue under section 377 of the Code of Civil Procedure.

The instant case is similar to Evangelista vs. Santos, 86 Phil. 387, where the plaintiffs sued the
defendant in the Court of First Instance of Rizal on the assumption that he was a resident of
Pasay City because he had a house there. Upon receipt of the summons, the defendant filed a
motion to dismiss based on improper venue. He alleged under oath that he was a resident of
Iloilo City.

This Court sustained the dismissal of the complaint on the ground of improper venue, because
the defendant was really a resident of Iloilo City. His Pasay City residence was used by his
children who were studying in Manila. Same holding in Casilan vs. Tomassi, 90 Phil. 765; Corre
vs. Corre, 100 Phil. 321; Calo vs. Bislig Industries, Inc., L-19703, January 30, 1967, 19 SCRA
173; Adamos vs. J. M. Tuason, Co., Inc.,. L-21957, October 14, 1968, 25 SCRA 529.

Where one Cesar Ramirez, a resident of Quezon City, sued in the Court of First Instance of
Manila Manuel F. Portillo, a resident of Caloocan City, for the recovery of a sum of money, the
trial court erred in not granting Portillo's motion to dismiss the complaint on the ground of
improper venue This Court issued the writ of prohibition to restrain the trial court from
proceeding in the case (Portillo vs. Judge Reyes and Ramirez, 113 Phil. 288).

WHEREFORE, the decision of the Court of Appeals and the order of respondent judge denying
the motion to dismiss are reversed and set aside. The writ of prohibition is granted. Civil Case
No. 34302 should be considered dismissed without prejudice to refiling - it in the Court of First
Instance of Manila or Bacolod City at the election of plaintiff which should be allowed to withdraw
the documentary evidence submitted in that case. All the proceedings in said case, including the
decision, are also set aside. Costs against Tyson Enterprises, Inc.

SOORDERED.
G.R. No. L-23606 July 29, 1968 1963, cannot be availed of by the said corporation, for the reason that its term of existence had
already expired when the said law took effect in short, said law has no retroactive effect."
ALHAMBRA CIGAR & CIGARETTE MANUFACTURING COMPANY, INC., petitioner,
vs. On December 3, 1963, Alhambra's counsel sought reconsideration of SEC's ruling aforesaid,
SECURITIES & EXCHANGE COMMISSION, respondent. refiled the amended articles of incorporation.

SANCHEZ, J.: On September 8, 1964, SEC, after a conference hearing, issued an order denying the
reconsideration sought.
To the question — May a corporation extend its life by amendment of its articles of incorporation
effected during the three-year statutory period for liquidation when its original term of existence Alhambra now invokes the jurisdiction of this Court to overturn the conclusion below. 1
had already expired? — the answer of the Securities and Exchange Commissioner was in the
negative. Offshoot is this appeal.
1. Alhambra relies on Republic Act 3531, which amended Section 18 of the Corporation Law.
Well it is to take note of the old and the new statutes as they are framed. Section 18, prior to and
That problem emerged out of the following controlling facts: after its modification by Republic Act 3531, covers the subject of amendment of the articles of
incorporation of private corporations. A provision thereof which remains unaltered is that a
corporation may amend its articles of incorporation "by a majority vote of its board of directors or
Petitioner Alhambra Cigar and Cigarette Manufacturing Company, Inc. (hereinafter referred to
trustees and ... by the vote or written assent of the stockholders representing at least two-thirds
simply as Alhambra) was duly incorporated under Philippine laws on January 15, 1912. By its
of the subscribed capital stock ... "
corporate articles it was to exist for fifty (50) years from incorporation. Its term of existence
expired on January 15, 1962. On that date, it ceased transacting business, entered into a state
of liquidation. But prior to amendment by Republic Act 3531, an explicit prohibition existed in Section 18, thus:

Thereafter, a new corporation. — Alhambra Industries, Inc. — was formed to carry on the ... Provided, however, That the life of said corporation shall not be extended by said
business of Alhambra. amendment beyond the time fixed in the original articles: ...

On May 1, 1962, Alhambra's stockholders, by resolution named Angel S. Gamboa trustee to This was displaced by Republic Act 3531 which enfranchises all private corporations to extend
take charge of its liquidation. their corporate existence. Thus incorporated into the structure of Section 18 are the following:

On June 20, 1963 — within Alhambra's three-year statutory period for liquidation - Republic Act ... Provided, however, That should the amendment consist in extending the corporate
3531 was enacted into law. It amended Section 18 of the Corporation Law; it empowered life, the extension shall not exceed fifty years in any one instance: Provided, further,
domestic private corporations to extend their corporate life beyond the period fixed by the That the original articles, and amended articles together shall contain all provisions
articles of incorporation for a term not to exceed fifty years in any one instance. Previous to required by law to be set out in the articles of incorporation: ...
Republic Act 3531, the maximum non-extendible term of such corporations was fifty years.
As we look in retrospect at the facts, we find these: From July 15 to October 28, 1963, when
On July 15, 1963, at a special meeting, Alhambra's board of directors resolved to amend Alhambra made its attempt to extend its corporate existence, its original term of fifty years had
paragraph "Fourth" of its articles of incorporation to extend its corporate life for an additional fifty already expired (January 15, 1962); it was in the midst of the three-year grace period statutorily
years, or a total of 100 years from its incorporation. fixed in Section 77 of the Corporation Law, thus: .

On August 26, 1963, Alhambra's stockholders, representing more than two-thirds of its SEC. 77. Every corporation whose charter expires by its own limitation or is annulled
subscribed capital stock, voted to approve the foregoing resolution. The "Fourth" paragraph of by forfeiture or otherwise, or whose corporate existence for other purposes is
Alhambra's articles of incorporation was thus altered to read: terminated in any other manner, shall nevertheless be continued as a body corporate
for three years after the time when it would have been so dissolved, for the purpose of
prosecuting and defending suits by or against it and of enabling it gradually to settle
FOURTH. That the term for which said corporation is to exist is fifty (50) years from
and close its affairs, to dispose of and convey its property and to divide its capital
and after the date of incorporation, and for an additional period of fifty (50) years
stock, but not for the purpose of continuing the business for which it was established. 2
thereafter.

Plain from the language of the provision is its meaning: continuance of a "dissolved" corporation
On October 28, 1963, Alhambra's articles of incorporation as so amended certified correct by its
as a body corporate for three years has for its purpose the final closure of its affairs, and no
president and secretary and a majority of its board of directors, were filed with respondent
other; the corporation is specifically enjoined from "continuing the business for which it was
Securities and Exchange Commission (SEC).
established". The liquidation of the corporation's affairs set forth in Section 77 became
necessary precisely because its life had ended. For this reason alone, the corporate existence
On November 18, 1963, SEC, however, returned said amended articles of incorporation to and juridical personality of that corporation to do business may no longer be extended.
Alhambra's counsel with the ruling that Republic Act 3531 "which took effect only on June 20,
Worth bearing in mind, at this juncture, is the basic development of corporation law. nothing to extend. Here it was proposed nearly three years after the corporate life of
the association had expired to revivify the dead body, and to make that relate back
some two years and eight months. In other words, the association for two years and
The common law rule, at the beginning, was rigid and inflexible in that upon its dissolution, a
eight months had only existed for the purpose of winding up its business, and, after
corporation became legally dead for all purposes. Statutory authorizations had to be provided for
this length of time, it was proposed to revivify it and make it a live corporation for the
its continuance after dissolution "for limited and specified purposes incident to complete
two years and eight months daring which it had not been such.
liquidation of its affairs".3 Thus, the moment a corporation's right to exist as an "artificial person"
ceases, its corporate powers are terminated "just as the powers of a natural person to take part
in mundane affairs cease to exist upon his death".4 There is nothing left but to conduct, as it The law gives a certain length of time for the filing of records in this court, and
were, the settlement of the estate of a deceased juridical person. provides that the time may be extended by the court, but under this provision it has
uniformly been held that when the time was expired, there is nothing to extend, and
that the appeal must be dismissed... So, when the articles of a corporation have
2. Republic Act 3531, amending Section 18 of the Corporation Law, is silent, it is true, as to
expired, it is too late to adopt an amendment extending the life of a corporation; for,
when such act of extension may be made. But even with a superficial knowledge of corporate
the corporation having expired, this is in effect to create a new corporation ..."7
principles, it does not take much effort to reach a correct conclusion. For, implicit in Section 77
heretofore quoted is that the privilege given to prolongcorporate life under the amendment must
be exercised before the expiry of the term fixed in the articles of incorporation. True it is, that the Alabama Supreme Court has stated in one case.8 that a corporation
empowered by statute to renew its corporate existence may do so even after the expiration of its
corporate life, provided renewal is taken advantage of within the extended statutory period for
Silence of the law on the matter is not hard to understand. Specificity is not really necessary.
purposes of liquidation. That ruling, however, is inherently weak as persuasive authority for the
The authority to prolong corporate life was inserted by Republic Act 3531 into a section of the
situation at bar for at least two reasons: First. That case was a suit for mandamus to compel a
law that deals with the power of a corporation to amend its articles of incorporation. (For, the
former corporate officer to turn over books and records that came into his possession and
manner of prolongation is through an amendment of the articles.) And it should be clearly
control by virtue of his office. It was there held that such officer was obliged to surrender his
evident that under Section 77 no corporation in a state of liquidation can act in any way, much
books and records even if the corporation had already expired. The holding on the continued
less amend its articles, "for the purpose of continuing the business for which it was established".
existence of the corporation was a mere dictum. Second. Alabama's law is different.
Corporations in that state were authorized not only to extend but also to renew their corporate
All these dilute Alhambra's position that it could revivify its corporate life simply because when it existence.That very case defined the word "renew" as follows; "To make new again; to restore to
attempted to do so, Alhambra was still in the process of liquidation. It is surely impermissible for freshness; to make new spiritually; to regenerate; to begin again; to recommence; to resume; to
us to stretch the law — that merely empowers a corporation to act in liquidation — to inject restore to existence, to revive; to re-establish; to recreate; to replace; to grant or obtain an
therein the power to extend its corporate existence. extension of Webster's New International Dict.; 34 Cyc. 1330; Carter v. Brooklyn Life Ins.
Co., 110 N.Y. 15, 21, 22, 17 N.E. 396; 54 C.J. 379. Sec".9
3. Not that we are alone in this view. Fletcher has written: "Since the privilege of extension is
purely statutory, all of the statutory conditions precedent must be complied with in order that the On this point, we again draw from Fletcher: "There is a broad distinction between the extension
extension may be effectuated. And, generally these conditions must be complied with, and the of a charter and the grant of a new one. To renew a charter is to revive a charter which has
steps necessary to effect the extension must be taken, during the life of the corporation, and expired, or, in other words, "to give a new existence to one which has been forfeited, or which
before the expiration of the term of existence as original fixed by its charter or the general law, has lost its vitality by lapse of time". To "extend" a charter is "to increase the time for the
since, as a rule, the corporation is ipso facto dissolved as soon as that time expires. So where existence of one which would otherwise reach its limit at an earlier period". 10 Nowhere in our
the extension is by amendment of the articles of incorporation, the amendment must be adopted statute — Section 18, Corporation Law, as amended by Republic Act 3531 — do we find the
before that time. And, similarly, the filing and recording of a certificate of extension after that time word "renew" in reference to the authority given to corporations to protract their lives. Our law
cannot relate back to the date of the passage of a resolution by the stockholders in favor of the limits itself to extension of corporate existence. And, as so understood, extension may be
extension so as to save the life of the corporation. The contrary is true, however, and the made only before the term provided in the corporate charter expires.
doctrine of relation will apply, where the delay is due to the neglect of the officer with whom the
certificate is required to be filed, or to a wrongful refusal on his part to receive it. And statutes in
Alhambra draws attention to another case11 which declares that until the end of the extended
some states specifically provide that a renewal may be had within a specified time before or after
period for liquidation, a dissolved corporation "does not become an extinguished entity". But this
the time fixed for the termination of the corporate existence". 5
statement was obviously lifted out of context. That case dissected the question whether or not
suits can be commenced by or against a corporation within its liquidation period. Which was
The logic of this position is well expressed in a foursquare case decided by the Court of Appeals answered in the affirmative. For, the corporation still exists for the settlement of its affairs.
of Kentucky.6There, pronouncement was made as follows:
People, ex rel. vs. Green,12 also invoked by Alhambra, is as unavailing. There, although the
... But section 561 (section 2147) provides that, when any corporation expires by the corporation amended its articles to extend its existence at a time when it had no legal authority
terms of its articles of incorporation, it may be thereafter continued to act for the yet, it adopted the amended articles later on when it had the power to extend its life and during
purpose of closing up its business, but for no other purpose. The corporate life of the its original term when it could amend its articles.
Home Building Association expired on May 3, 1905. After that date, by the mandate of
the statute, it could continue to act for the purpose of closing up its business, but for
The foregoing notwithstanding, Alhambra falls back on the contention that its case is arguably
no other purpose. The proposed amendment was not made until January 16, 1908, or
within the purview of the law. It says that before cessation of its corporate life, it could not have
nearly three years after the corporation expired by the terms of the articles of
extended the same, for the simple reason that Republic Act 3531 had not then become law. It
incorporation. When the corporate life of the corporation was ended, there was
must be remembered that Republic Act 3531 took effect on June 20, 1963, while the original 5. Alhambra pleads for munificence in interpretation, one which brushes technicalities aside.
term of Alhambra's existence expired before that date — on January 15, 1962. The mischief that Bases for this posture are that Republic Act 3531 is a remedial statute, and that extension of
flows from this theory is at once apparent. It would certainly open the gates for all defunct corporate life is beneficial to the economy.
corporations — whose charters have expired even long before Republic Act 3531 came into
being — to resuscitate their corporate existence.
Alhambra's stance does not induce assent. Expansive construction is possible only when there
is something to expand. At the time of the passage of Republic Act 3531, Alhambra's corporate
4. Alhambra brings into argument Republic Act 1932, which amends Section 196 of the life had already expired. It had overstepped the limits of its limited existence. No life there is to
Insurance Act, now reading as follows: 1äwphï1.ñët prolong.

SEC. 196. Any provision of law to the contrary notwithstanding, every domestic life Besides, a new corporation — Alhambra Industries, Inc., with but slight change in
insurance corporation, formed for a limited period under the provisions of its articles of stockholdings15 — has already been established. Its purpose is to carry on, and it actually does
incorporation, may extend its corporate existence for a period not exceeding fifty years carry on,16 the business of the dissolved entity. The beneficial-effects argument is off the mark.
in any one instance by amendment to its articles of incorporation on or before the
expiration of the term so fixed in said articles ...
The way the whole case shapes up then, the only possible drawbacks of Alhambra might be
that, instead of the new corporation (Alhambra Industries, Inc.) being written off, the old one
To be observed is that the foregoing statute — unlike Republic Act 3531 — expressly authorizes (Alhambra Cigar & Cigarette Manufacturing Company, Inc.) has to be wound up; and that the old
domestic insurance corporations to extend their corporate existence "on or before the expiration corporate name cannot be retained fully in its exact form. 17What is important though is that the
of the term" fixed in their articles of incorporation. Republic Act 1932 was approved on June 22, word Alhambra, the name that counts [it has goodwill], remains.
1957, long before the passage of Republic Act 3531 in 1963. Congress, Alhambra points out,
must have been aware of Republic Act 1932 when it passed Republic Act 3531. Since the
FOR THE REASONS GIVEN, the ruling of the Securities and Exchange Commission of
phrase "on or before", etc., was omitted in Republic Act 3531, which contains no similar
November 18, 1963, and its order of September 8, 1964, both here under review, are hereby
limitation, it follows, according to Alhambra, that it is not necessary to extend corporate
affirmed.
existence on or before the expiration of its original term.

Costs against petitioner Alhambra Cigar & Cigarette Manufacturing Company, Inc. So ordered.
That Republic Act 3531 stands mute as to when extention of corporate existence may be made,
assumes no relevance. We have already said, in the face of a familiar precept, that a defunct
corporation is bereft of any legal faculty not otherwise expressly sanctioned by law.

Illuminating here is the explanatory note of H.B. 1774, later Republic Act 3531 — now in dispute.
Its first paragraph states that "Republic Act No. 1932 allows the automatic extension of the
corporate existence of domestic life insurance corporations upon amendment of their articles of
incorporation on or before the expiration of the terms fixed by said articles". The succeeding
lines are decisive: "This is a good law, a sane and sound one. There appears to be no valid
reason why it should not be made to apply to other private corporations.13

The situation here presented is not one where the law under consideration is ambiguous, where
courts have to put in harness extrinsic aids such as a look at another statute to disentangle
doubts. It is an elementary rule in legal hermeneutics that where the terms of the law are clear,
no statutory construction may be permitted. Upon the basic conceptual scheme under which
corporations operate, and with Section 77 of the Corporation Law particularly in mind, we find no
vagueness in Section 18, as amended by Republic Act 3531. As we view it, by directing
attention to Republic Act 1932, Alhambra would seek to create obscurity in the law; and, with
that, ask of us a ruling that such obscurity be explained. This, we dare say, cannot be done.

The pari materia rule of statutory construction, in fact, commands that statutes must be
harmonized with each other.14 So harmonizing, the conclusion is clear that Section 18 of the
Corporation Law, as amended by Republic Act 3531 in reference to extensions of corporate
existence, is to be read in the same light as Republic Act 1932. Which means that domestic
corporations in general, as with domestic insurance companies, can extend corporate existence
only on or before the expiration of the term fixed in their charters.
G.R. No. 176579 June 28, 2011 Thereafter, First Pacific announced that it would exercise its right of first refusal as a PTIC
stockholder and buy the 111,415 PTIC shares by matching the bid price of Parallax. However,
First Pacific failed to do so by the 1 February 2007 deadline set by IPC and instead, yielded its
WILSON P. GAMBOA, Petitioner,
right to PTIC itself which was then given by IPC until 2 March 2007 to buy the PTIC shares. On
vs.
14 February 2007, First Pacific, through its subsidiary, MPAH, entered into a Conditional Sale
FINANCE SECRETARY MARGARITO B. TEVES, FINANCE UNDERSECRETARY JOHN P.
and Purchase Agreement of the 111,415 PTIC shares, or 46.125 percent of the outstanding
SEVILLA, AND COMMISSIONER RICARDO ABCEDE OF THE PRESIDENTIAL
capital stock of PTIC, with the Philippine Government for the price of ₱25,217,556,000 or
COMMISSION ON GOOD GOVERNMENT (PCGG) IN THEIR CAPACITIES AS CHAIR AND
US$510,580,189. The sale was completed on 28 February 2007.
MEMBERS, RESPECTIVELY, OF THE PRIVATIZATION COUNCIL, CHAIRMAN ANTHONI
SALIM OF FIRST PACIFIC CO., LTD. IN HIS CAPACITY AS DIRECTOR OF METRO PACIFIC
ASSET HOLDINGS INC., CHAIRMAN MANUEL V. PANGILINAN OF PHILIPPINE LONG Since PTIC is a stockholder of PLDT, the sale by the Philippine Government of 46.125 percent
DISTANCE TELEPHONE COMPANY (PLDT) IN HIS CAPACITY AS MANAGING DIRECTOR of PTIC shares is actually an indirect sale of 12 million shares or about 6.3 percent of the
OF FIRST PACIFIC CO., LTD., PRESIDENT NAPOLEON L. NAZARENO OF PHILIPPINE outstanding common shares of PLDT. With the sale, First Pacific’s common shareholdings
LONG DISTANCE TELEPHONE COMPANY, CHAIR FE BARIN OF THE SECURITIES in PLDT increased from 30.7 percent to 37 percent, thereby increasing the common
EXCHANGE COMMISSION, and PRESIDENT FRANCIS LIM OF THE PHILIPPINE STOCK shareholdings of foreigners in PLDT to about 81.47 percent. This violates Section 11, Article
EXCHANGE, Respondents. XII of the 1987 Philippine Constitution which limits foreign ownership of the capital of a public
PABLITO V. SANIDAD and ARNO V. SANIDAD, Petitioners-in-Intervention. utility to not more than 40 percent.3

DECISION On the other hand, public respondents Finance Secretary Margarito B. Teves, Undersecretary
John P. Sevilla, and PCGG Commissioner Ricardo Abcede allege the following relevant facts:
CARPIO, J.:
On 9 November 1967, PTIC was incorporated and had since engaged in the business of
investment holdings. PTIC held 26,034,263 PLDT common shares, or 13.847 percent of the total
The Case
PLDT outstanding common shares. PHI, on the other hand, was incorporated in 1977, and
became the owner of 111,415 PTIC shares or 46.125 percent of the outstanding capital stock of
This is an original petition for prohibition, injunction, declaratory relief and declaration of nullity of PTIC by virtue of three Deeds of Assignment executed by Ramon Cojuangco and Luis Tirso
the sale of shares of stock of Philippine Telecommunications Investment Corporation (PTIC) by Rivilla. In 1986, the 111,415 PTIC shares held by PHI were sequestered by the PCGG, and
the government of the Republic of the Philippines to Metro Pacific Assets Holdings, Inc. (MPAH), subsequently declared by this Court as part of the ill-gotten wealth of former President
an affiliate of First Pacific Company Limited (First Pacific). Ferdinand Marcos. The sequestered PTIC shares were reconveyed to the Republic of the
Philippines in accordance with this Court’s decision4 which became final and executory on 8
August 2006.
The Antecedents

The Philippine Government decided to sell the 111,415 PTIC shares, which represent 6.4
The facts, according to petitioner Wilson P. Gamboa, a stockholder of Philippine Long Distance percent of the outstanding common shares of stock of PLDT, and designated the Inter-Agency
Telephone Company (PLDT), are as follows:1 Privatization Council (IPC), composed of the Department of Finance and the PCGG, as the
disposing entity. An invitation to bid was published in seven different newspapers from 13 to 24
On 28 November 1928, the Philippine Legislature enacted Act No. 3436 which granted PLDT a November 2006. On 20 November 2006, a pre-bid conference was held, and the original
franchise and the right to engage in telecommunications business. In 1969, General Telephone deadline for bidding scheduled on 4 December 2006 was reset to 8 December 2006. The
and Electronics Corporation (GTE), an American company and a major PLDT stockholder, sold extension was published in nine different newspapers.
26 percent of the outstanding common shares of PLDT to PTIC. In 1977, Prime Holdings, Inc.
(PHI) was incorporated by several persons, including Roland Gapud and Jose Campos, Jr.
During the 8 December 2006 bidding, Parallax Capital Management LP emerged as the highest
Subsequently, PHI became the owner of 111,415 shares of stock of PTIC by virtue of three bidder with a bid of ₱25,217,556,000. The government notified First Pacific, the majority owner
Deeds of Assignment executed by PTIC stockholders Ramon Cojuangco and Luis Tirso Rivilla. of PTIC shares, of the bidding results and gave First Pacific until 1 February 2007 to exercise its
In 1986, the 111,415 shares of stock of PTIC held by PHI were sequestered by the Presidential
right of first refusal in accordance with PTIC’s Articles of Incorporation. First Pacific announced
Commission on Good Government (PCGG). The 111,415 PTIC shares, which represent about its intention to match Parallax’s bid.
46.125 percent of the outstanding capital stock of PTIC, were later declared by this Court to be
owned by the Republic of the Philippines.2
On 31 January 2007, the House of Representatives (HR) Committee on Good Government
conducted a public hearing on the particulars of the then impending sale of the 111,415 PTIC
In 1999, First Pacific, a Bermuda-registered, Hong Kong-based investment firm, acquired the shares. Respondents Teves and Sevilla were among those who attended the public hearing.
remaining 54 percent of the outstanding capital stock of PTIC. On 20 November 2006, the Inter-
The HR Committee Report No. 2270 concluded that: (a) the auction of the government’s
Agency Privatization Council (IPC) of the Philippine Government announced that it would sell the 111,415 PTIC shares bore due diligence, transparency and conformity with existing legal
111,415 PTIC shares, or 46.125 percent of the outstanding capital stock of PTIC, through a procedures; and (b) First Pacific’s intended acquisition of the government’s 111,415 PTIC
public bidding to be conducted on 4 December 2006. Subsequently, the public bidding was reset shares resulting in First Pacific’s 100% ownership of PTIC will not violate the 40 percent
to 8 December 2006, and only two bidders, Parallax Venture Fund XXVII (Parallax) and Pan- constitutional limit on foreign ownership of a public utility since PTIC holds only 13.847
Asia Presidio Capital, submitted their bids. Parallax won with a bid of ₱25.6 billion or US$510
million.
percent of the total outstanding common shares of PLDT.5 On 28 February 2007, First The Issue
Pacific completed the acquisition of the 111,415 shares of stock of PTIC.
This Court is not a trier of facts. Factual questions such as those raised by petitioner, 9 which
Respondent Manuel V. Pangilinan admits the following facts: (a) the IPC conducted a public indisputably demand a thorough examination of the evidence of the parties, are generally
bidding for the sale of 111,415 PTIC shares or 46 percent of the outstanding capital stock of beyond this Court’s jurisdiction. Adhering to this well-settled principle, the Court shall confine the
PTIC (the remaining 54 percent of PTIC shares was already owned by First Pacific and its resolution of the instant controversy solely on the threshold and purely legal issue of whether
affiliates); (b) Parallax offered the highest bid amounting to ₱25,217,556,000; (c) pursuant to the the term "capital" in Section 11, Article XII of the Constitution refers to the total common shares
right of first refusal in favor of PTIC and its shareholders granted in PTIC’s Articles of only or to the total outstanding capital stock (combined total of common and non-voting preferred
Incorporation, MPAH, a First Pacific affiliate, exercised its right of first refusal by matching the shares) of PLDT, a public utility.
highest bid offered for PTIC shares on 13 February 2007; and (d) on 28 February 2007, the sale
was consummated when MPAH paid IPC ₱25,217,556,000 and the government delivered the
The Ruling of the Court
certificates for the 111,415 PTIC shares. Respondent Pangilinan denies the other allegations of
facts of petitioner.
The petition is partly meritorious.
On 28 February 2007, petitioner filed the instant petition for prohibition, injunction, declaratory
relief, and declaration of nullity of sale of the 111,415 PTIC shares. Petitioner claims, among Petition for declaratory relief treated as petition for mandamus
others, that the sale of the 111,415 PTIC shares would result in an increase in First Pacific’s
common shareholdings in PLDT from 30.7 percent to 37 percent, and this, combined with
Japanese NTT DoCoMo’s common shareholdings in PLDT, would result to a total foreign At the outset, petitioner is faced with a procedural barrier. Among the remedies petitioner seeks,
only the petition for prohibition is within the original jurisdiction of this court, which however is not
common shareholdings in PLDT of 51.56 percent which is over the 40 percent constitutional
limit.6 Petitioner asserts: exclusive but is concurrent with the Regional Trial Court and the Court of Appeals. The actions
for declaratory relief,10 injunction, and annulment of sale are not embraced within the original
jurisdiction of the Supreme Court. On this ground alone, the petition could have been dismissed
If and when the sale is completed, First Pacific’s equity in PLDT will go up from 30.7 percent to outright.
37.0 percent of its common – or voting- stockholdings, x x x. Hence, the consummation of the
sale will put the two largest foreign investors in PLDT – First Pacific and Japan’s NTT DoCoMo,
While direct resort to this Court may be justified in a petition for prohibition,11 the Court shall
which is the world’s largest wireless telecommunications firm, owning 51.56 percent of PLDT
common equity. x x x With the completion of the sale, data culled from the official website of the nevertheless refrain from discussing the grounds in support of the petition for prohibition since
New York Stock Exchange (www.nyse.com) showed that those foreign entities, which own at on 28 February 2007, the questioned sale was consummated when MPAH paid IPC
₱25,217,556,000 and the government delivered the certificates for the 111,415 PTIC shares.
least five percent of common equity, will collectively own 81.47 percent of PLDT’s common
equity. x x x
However, since the threshold and purely legal issue on the definition of the term "capital" in
x x x as the annual disclosure reports, also referred to as Form 20-K reports x x x which PLDT Section 11, Article XII of the Constitution has far-reaching implications to the national economy,
submitted to the New York Stock Exchange for the period 2003-2005, revealed that First Pacific the Court treats the petition for declaratory relief as one for mandamus.12
and several other foreign entities breached the constitutional limit of 40 percent ownership as
early as 2003. x x x"7 In Salvacion v. Central Bank of the Philippines,13 the Court treated the petition for declaratory
relief as one for mandamus considering the grave injustice that would result in the interpretation
of a banking law. In that case, which involved the crime of rape committed by a foreign tourist
Petitioner raises the following issues: (1) whether the consummation of the then impending sale
of 111,415 PTIC shares to First Pacific violates the constitutional limit on foreign ownership of a against a Filipino minor and the execution of the final judgment in the civil case for damages on
public utility; (2) whether public respondents committed grave abuse of discretion in allowing the the tourist’s dollar deposit with a local bank, the Court declared Section 113 of Central Bank
Circular No. 960, exempting foreign currency deposits from attachment, garnishment or any
sale of the 111,415 PTIC shares to First Pacific; and (3) whether the sale of common shares to
foreigners in excess of 40 percent of the entire subscribed common capital stock violates the other order or process of any court, inapplicable due to the peculiar circumstances of the case.
constitutional limit on foreign ownership of a public utility.8 The Court held that "injustice would result especially to a citizen aggrieved by a foreign guest
like accused x x x" that would "negate Article 10 of the Civil Code which provides that ‘in case of
doubt in the interpretation or application of laws, it is presumed that the lawmaking body
On 13 August 2007, Pablito V. Sanidad and Arno V. Sanidad filed a Motion for Leave to intended right and justice to prevail.’" The Court therefore required respondents Central Bank of
Intervene and Admit Attached Petition-in-Intervention. In the Resolution of 28 August 2007, the the Philippines, the local bank, and the accused to comply with the writ of execution issued in
Court granted the motion and noted the Petition-in-Intervention. the civil case for damages and to release the dollar deposit of the accused to satisfy the
judgment.
Petitioners-in-intervention "join petitioner Wilson Gamboa x x x in seeking, among others, to
enjoin and/or nullify the sale by respondents of the 111,415 PTIC shares to First Pacific or In Alliance of Government Workers v. Minister of Labor,14 the Court similarly brushed aside the
assignee." Petitioners-in-intervention claim that, as PLDT subscribers, they have a "stake in the procedural infirmity of the petition for declaratory relief and treated the same as one for
outcome of the controversy x x x where the Philippine Government is completing the sale of mandamus. In Alliance, the issue was whether the government unlawfully excluded petitioners,
government owned assets in [PLDT], unquestionably a public utility, in violation of the nationality who were government employees, from the enjoyment of rights to which they were entitled
restrictions of the Philippine Constitution." under the law. Specifically, the question was: "Are the branches, agencies, subdivisions, and
instrumentalities of the Government, including government owned or controlled corporations
included among the four ‘employers’ under Presidential Decree No. 851 which are required to the reservation of certain investments to Filipino citizens,21 in Section 4(2), Article XIV on the
pay their employees x x x a thirteenth (13th) month pay x x x ?" The Constitutional principle ownership of educational institutions,22 and in Section 11(2), Article XVI on the ownership of
involved therein affected all government employees, clearly justifying a relaxation of the advertising companies.23
technical rules of procedure, and certainly requiring the interpretation of the assailed presidential
decree.
Petitioner has locus standi

In short, it is well-settled that this Court may treat a petition for declaratory relief as one for
There is no dispute that petitioner is a stockholder of PLDT. As such, he has the right to question
mandamus if the issue involved has far-reaching implications. As this Court held in Salvacion:
the subject sale, which he claims to violate the nationality requirement prescribed in Section 11,
Article XII of the Constitution. If the sale indeed violates the Constitution, then there is a
The Court has no original and exclusive jurisdiction over a petition for declaratory possibility that PLDT’s franchise could be revoked, a dire consequence directly affecting
relief. However, exceptions to this rule have been recognized. Thus, where the petition petitioner’s interest as a stockholder.
has far-reaching implications and raises questions that should be resolved, it may be
treated as one for mandamus.15 (Emphasis supplied)
More importantly, there is no question that the instant petition raises matters of transcendental
importance to the public. The fundamental and threshold legal issue in this case, involving the
In the present case, petitioner seeks primarily the interpretation of the term "capital" in Section national economy and the economic welfare of the Filipino people, far outweighs any perceived
11, Article XII of the Constitution. He prays that this Court declare that the term "capital" refers to impediment in the legal personality of the petitioner to bring this action.
common shares only, and that such shares constitute "the sole basis in determining foreign
equity in a public utility." Petitioner further asks this Court to declare any ruling inconsistent with
In Chavez v. PCGG,24 the Court upheld the right of a citizen to bring a suit on matters of
such interpretation unconstitutional.
transcendental importance to the public, thus:

The interpretation of the term "capital" in Section 11, Article XII of the Constitution has far-
In Tañada v. Tuvera, the Court asserted that when the issue concerns a public right and the
reaching implications to the national economy. In fact, a resolution of this issue will determine
object of mandamus is to obtain the enforcement of a public duty, the people are
whether Filipinos are masters, or second class citizens, in their own country. What is at stake
regarded as the real parties in interest; and because it is sufficient that petitioner is a
here is whether Filipinos or foreigners will have effective control of the national economy.
citizen and as such is interested in the execution of the laws, he need not show that he
Indeed, if ever there is a legal issue that has far-reaching implications to the entire nation, and to
has any legal or special interest in the result of the action. In the aforesaid case, the
future generations of Filipinos, it is the threshhold legal issue presented in this case.
petitioners sought to enforce their right to be informed on matters of public concern, a right then
recognized in Section 6, Article IV of the 1973 Constitution, in connection with the rule that laws
The Court first encountered the issue on the definition of the term "capital" in Section 11, Article in order to be valid and enforceable must be published in the Official Gazette or otherwise
XII of the Constitution in the case of Fernandez v. Cojuangco, docketed as G.R. No. effectively promulgated. In ruling for the petitioners’ legal standing, the Court declared that the
157360.16 That case involved the same public utility (PLDT) and substantially the same private right they sought to be enforced ‘is a public right recognized by no less than the fundamental law
respondents. Despite the importance and novelty of the constitutional issue raised therein and of the land.’
despite the fact that the petition involved a purely legal question, the Court declined to resolve
the case on the merits, and instead denied the same for disregarding the hierarchy of
Legaspi v. Civil Service Commission, while reiterating Tañada, further declared that ‘when a
courts.17There, petitioner Fernandez assailed on a pure question of law the Regional Trial
mandamus proceeding involves the assertion of a public right, the requirement of
Court’s Decision of 21 February 2003 via a petition for review under Rule 45. The Court’s
personal interest is satisfied by the mere fact that petitioner is a citizen and, therefore,
Resolution, denying the petition, became final on 21 December 2004.
part of the general ‘public’ which possesses the right.’

The instant petition therefore presents the Court with another opportunity to finally settle
Further, in Albano v. Reyes, we said that while expenditure of public funds may not have been
this purely legal issuewhich is of transcendental importance to the national economy and a
involved under the questioned contract for the development, management and operation of the
fundamental requirement to a faithful adherence to our Constitution. The Court must forthwith
Manila International Container Terminal, ‘public interest [was] definitely involved
seize such opportunity, not only for the benefit of the litigants, but more significantly for the
considering the important role [of the subject contract] . . . in the economic development
benefit of the entire Filipino people, to ensure, in the words of the Constitution, "a self-reliant and
of the country and the magnitude of the financial consideration involved.’ We concluded
independent national economy effectively controlled by Filipinos."18 Besides, in the light of
that, as a consequence, the disclosure provision in the Constitution would constitute sufficient
vague and confusing positions taken by government agencies on this purely legal issue, present
authority for upholding the petitioner’s standing. (Emphasis supplied)
and future foreign investors in this country deserve, as a matter of basic fairness, a categorical
ruling from this Court on the extent of their participation in the capital of public utilities and other
nationalized businesses. Clearly, since the instant petition, brought by a citizen, involves matters of transcendental public
importance, the petitioner has the requisite locus standi.
Despite its far-reaching implications to the national economy, this purely legal issue has
remained unresolved for over 75 years since the 1935 Constitution. There is no reason for this Definition of the Term "Capital" in
Court to evade this ever recurring fundamental issue and delay again defining the term "capital," Section 11, Article XII of the 1987 Constitution
which appears not only in Section 11, Article XII of the Constitution, but also in Section 2, Article
XII on co-production and joint venture agreements for the development of our natural
resources,19 in Section 7, Article XII on ownership of private lands,20 in Section 10, Article XII on
Section 11, Article XII (National Economy and Patrimony) of the 1987 Constitution mandates the Any citizen or juridical entity desiring to operate a public utility must therefore meet the minimum
Filipinization of public utilities, to wit: nationality requirement prescribed in Section 11, Article XII of the Constitution. Hence, for a
corporation to be granted authority to operate a public utility, at least 60 percent of its "capital"
must be owned by Filipino citizens.
Section 11. No franchise, certificate, or any other form of authorization for the operation of
a public utility shall be granted except to citizens of the Philippines or to corporations or
associations organized under the laws of the Philippines, at least sixty per centum of The crux of the controversy is the definition of the term "capital." Does the term "capital" in
whose capital is owned by such citizens; nor shall such franchise, certificate, or authorization Section 11, Article XII of the Constitution refer to common shares or to the total outstanding
be exclusive in character or for a longer period than fifty years. Neither shall any such franchise capital stock (combined total of common and non-voting preferred shares)?
or right be granted except under the condition that it shall be subject to amendment, alteration,
or repeal by the Congress when the common good so requires. The State shall encourage
Petitioner submits that the 40 percent foreign equity limitation in domestic public utilities refers
equity participation in public utilities by the general public. The participation of foreign investors
only to common shares because such shares are entitled to vote and it is through voting that
in the governing body of any public utility enterprise shall be limited to their proportionate share
control over a corporation is exercised. Petitioner posits that the term "capital" in Section 11,
in its capital, and all the executive and managing officers of such corporation or association must
Article XII of the Constitution refers to "the ownership of common capital stock subscribed and
be citizens of the Philippines. (Emphasis supplied)
outstanding, which class of shares alone, under the corporate set-up of PLDT, can vote and
elect members of the board of directors." It is undisputed that PLDT’s non-voting preferred
The above provision substantially reiterates Section 5, Article XIV of the 1973 Constitution, thus: shares are held mostly by Filipino citizens.30 This arose from Presidential Decree No.
217,31 issued on 16 June 1973 by then President Ferdinand Marcos, requiring every applicant of
a PLDT telephone line to subscribe to non-voting preferred shares to pay for the investment cost
Section 5. No franchise, certificate, or any other form of authorization for the operation of
of installing the telephone line.32
a public utility shall be granted except to citizens of the Philippines or to corporations or
associations organized under the laws of the Philippines at least sixty per centum of the
capital of which is owned by such citizens, nor shall such franchise, certificate, or Petitioners-in-intervention basically reiterate petitioner’s arguments and adopt petitioner’s
authorization be exclusive in character or for a longer period than fifty years. Neither shall any definition of the term "capital."33 Petitioners-in-intervention allege that "the approximate foreign
such franchise or right be granted except under the condition that it shall be subject to ownership of common capital stock of PLDT x x x already amounts to at least 63.54% of the total
amendment, alteration, or repeal by the National Assembly when the public interest so requires. outstanding common stock," which means that foreigners exercise significant control over PLDT,
The State shall encourage equity participation in public utilities by the general public. The patently violating the 40 percent foreign equity limitation in public utilities prescribed by the
participation of foreign investors in the governing body of any public utility enterprise shall be Constitution.
limited to their proportionate share in the capital thereof. (Emphasis supplied)
Respondents, on the other hand, do not offer any definition of the term "capital" in Section 11,
The foregoing provision in the 1973 Constitution reproduced Section 8, Article XIV of the 1935 Article XII of the Constitution. More importantly, private respondents Nazareno and Pangilinan of
Constitution, viz: PLDT do not dispute that more than 40 percent of the common shares of PLDT are held by
foreigners.
Section 8. No franchise, certificate, or any other form of authorization for the operation of
a public utility shall be granted except to citizens of the Philippines or to corporations or In particular, respondent Nazareno’s Memorandum, consisting of 73 pages, harps mainly on the
other entities organized under the laws of the Philippines sixty per centum of the capital procedural infirmities of the petition and the supposed violation of the due process rights of the
of which is owned by citizens of the Philippines,nor shall such franchise, certificate, or "affected foreign common shareholders." Respondent Nazareno does not deny petitioner’s
authorization be exclusive in character or for a longer period than fifty years. No franchise or allegation of foreigners’ dominating the common shareholdings of PLDT. Nazareno stressed
right shall be granted to any individual, firm, or corporation, except under the condition that it mainly that the petition "seeks to divest foreign common shareholders purportedly
shall be subject to amendment, alteration, or repeal by the Congress when the public interest so exceeding 40% of the total common shareholdings in PLDT of their ownership over their
requires. (Emphasis supplied) shares." Thus, "the foreign natural and juridical PLDT shareholders must be impleaded in this
suit so that they can be heard."34 Essentially, Nazareno invokes denial of due process on behalf
of the foreign common shareholders.
Father Joaquin G. Bernas, S.J., a leading member of the 1986 Constitutional Commission,
reminds us that the Filipinization provision in the 1987 Constitution is one of the products of the
spirit of nationalism which gripped the 1935 Constitutional Convention. 25 The 1987 Constitution While Nazareno does not introduce any definition of the term "capital," he states that "among
"provides for the Filipinization of public utilities by requiring that any form of authorization for the the factual assertions that need to be established to counter petitioner’s allegations is the
operation of public utilities should be granted only to ‘citizens of the Philippines or to uniform interpretation by government agencies (such as the SEC), institutions and
corporations or associations organized under the laws of the Philippines at least sixty per corporations (such as the Philippine National Oil Company-Energy Development
centum of whose capital is owned by such citizens.’ The provision is [an express] recognition Corporation or PNOC-EDC) of including both preferred shares and common shares in
of the sensitive and vital position of public utilities both in the national economy and for "controlling interest" in view of testing compliance with the 40% constitutional limitation
national security."26 The evident purpose of the citizenship requirement is to prevent aliens on foreign ownership in public utilities."35
from assuming control of public utilities, which may be inimical to the national interest. 27 This
specific provision explicitly reserves to Filipino citizens control of public utilities, pursuant to an
Similarly, respondent Manuel V. Pangilinan does not define the term "capital" in Section 11,
overriding economic goal of the 1987 Constitution: to "conserve and develop our
Article XII of the Constitution. Neither does he refute petitioner’s claim of foreigners holding more
patrimony"28 and ensure "a self-reliant and independent national
than 40 percent of PLDT’s common shares. Instead, respondent Pangilinan focuses on the
economy effectively controlled by Filipinos."29
procedural flaws of the petition and the alleged violation of the due process rights of foreigners.
Respondent Pangilinan emphasizes in his Memorandum (1) the absence of this Court’s Thus, the 40% foreign ownership limitation should be interpreted to apply to both the beneficial
jurisdiction over the petition; (2) petitioner’s lack of standing; (3) mootness of the petition; (4) ownership and the controlling interest.
non-availability of declaratory relief; and (5) the denial of due process rights. Moreover,
respondent Pangilinan alleges that the issue should be whether "owners of shares in PLDT as
xxxx
well as owners of shares in companies holding shares in PLDT may be required to relinquish
their shares in PLDT and in those companies without any law requiring them to surrender their
shares and also without notice and trial." Clearly, therefore, the forty percent (40%) foreign equity limitation in public utilities prescribed by
the Constitution refers to ownership of shares of stock entitled to vote, i.e., common shares.
Furthermore, ownership of record of shares will not suffice but it must be shown that the legal
Respondent Pangilinan further asserts that "Section 11, [Article XII of the Constitution]
and beneficial ownership rests in the hands of Filipino citizens. Consequently, in the case of
imposes no nationality requirement on the shareholders of the utility company as a
petitioner PLDT, since it is already admitted that the voting interests of foreigners which would
condition for keeping their shares in the utility company." According to him, "Section 11
gain entry to petitioner PLDT by the acquisition of SMART shares through the Questioned
does not authorize taking one person’s property (the shareholder’s stock in the utility company)
Transactions is equivalent to 82.99%, and the nominee arrangements between the foreign
on the basis of another party’s alleged failure to satisfy a requirement that is a condition only for
principals and the Filipino owners is likewise admitted, there is, therefore, a violation of Section
that other party’s retention of another piece of property (the utility company being at least 60%
11, Article XII of the Constitution.
Filipino-owned to keep its franchise)."36

Parenthetically, the Opinions dated February 15, 1988 and April 14, 1987 cited by the Trial Court
The OSG, representing public respondents Secretary Margarito Teves, Undersecretary John P.
to support the proposition that the meaning of the word "capital" as used in Section 11, Article
Sevilla, Commissioner Ricardo Abcede, and Chairman Fe Barin, is likewise silent on the
XII of the Constitution allegedly refers to the sum total of the shares subscribed and paid-in by
definition of the term "capital." In its Memorandum 37 dated 24 September 2007, the OSG also
the shareholder and it allegedly is immaterial how the stock is classified, whether as common or
limits its discussion on the supposed procedural defects of the petition, i.e. lack of standing, lack
preferred, cannot stand in the face of a clear legislative policy as stated in the FIA which took
of jurisdiction, non-inclusion of interested parties, and lack of basis for injunction. The OSG does
effect in 1991 or way after said opinions were rendered, and as clarified by the above-quoted
not present any definition or interpretation of the term "capital" in Section 11, Article XII of the
Amendments. In this regard, suffice it to state that as between the law and an opinion rendered
Constitution. The OSG contends that "the petition actually partakes of a collateral attack on
by an administrative agency, the law indubitably prevails. Moreover, said Opinions are merely
PLDT’s franchise as a public utility," which in effect requires a "full-blown trial where all the
advisory and cannot prevail over the clear intent of the framers of the Constitution.
parties in interest are given their day in court."38

In the same vein, the SEC’s construction of Section 11, Article XII of the Constitution is at best
Respondent Francisco Ed Lim, impleaded as President and Chief Executive Officer of the
merely advisory for it is the courts that finally determine what a law means.39
Philippine Stock Exchange (PSE), does not also define the term "capital" and seeks the
dismissal of the petition on the following grounds: (1) failure to state a cause of action against
Lim; (2) the PSE allegedly implemented its rules and required all listed companies, including On the other hand, respondents therein, Antonio O. Cojuangco, Manuel V. Pangilinan, Carlos A.
PLDT, to make proper and timely disclosures; and (3) the reliefs prayed for in the petition would Arellano, Helen Y. Dee, Magdangal B. Elma, Mariles Cacho-Romulo, Fr. Bienvenido F. Nebres,
adversely impact the stock market. Ray C. Espinosa, Napoleon L. Nazareno, Albert F. Del Rosario, and Orlando B. Vea, argued
that the term "capital" in Section 11, Article XII of the Constitution includes preferred shares
since the Constitution does not distinguish among classes of stock, thus:
In the earlier case of Fernandez v. Cojuangco, petitioner Fernandez who claimed to be a
stockholder of record of PLDT, contended that the term "capital" in the 1987 Constitution refers
to shares entitled to vote or the common shares. Fernandez explained thus: 16. The Constitution applies its foreign ownership limitation on the corporation’s "capital," without
distinction as to classes of shares. x x x
The forty percent (40%) foreign equity limitation in public utilities prescribed by the Constitution
refers to ownership of shares of stock entitled to vote, i.e., common shares, considering that it is In this connection, the Corporation Code – which was already in force at the time the present
through voting that control is being exercised. x x x (1987) Constitution was drafted – defined outstanding capital stock as follows:

Obviously, the intent of the framers of the Constitution in imposing limitations and restrictions on Section 137. Outstanding capital stock defined. – The term "outstanding capital stock", as used
fully nationalized and partially nationalized activities is for Filipino nationals to be always in in this Code, means the total shares of stock issued under binding subscription agreements to
control of the corporation undertaking said activities. Otherwise, if the Trial Court’s ruling subscribers or stockholders, whether or not fully or partially paid, except treasury shares.
upholding respondents’ arguments were to be given credence, it would be possible for the
ownership structure of a public utility corporation to be divided into one percent (1%) common
stocks and ninety-nine percent (99%) preferred stocks. Following the Trial Court’s ruling Section 137 of the Corporation Code also does not distinguish between common and preferred
adopting respondents’ arguments, the common shares can be owned entirely by foreigners thus shares, nor exclude either class of shares, in determining the outstanding capital stock (the
"capital") of a corporation. Consequently, petitioner’s suggestion to reckon PLDT’s foreign equity
creating an absurd situation wherein foreigners, who are supposed to be minority shareholders,
control the public utility corporation. only on the basis of PLDT’s outstanding common shares is without legal basis. The language of
the Constitution should be understood in the sense it has in common use.

xxxx
xxxx
17. But even assuming that resort to the proceedings of the Constitutional Commission is Where the articles of incorporation provide for non-voting shares in the cases allowed by this
necessary, there is nothing in the Record of the Constitutional Commission (Vol. III) – which Code, the holders of such shares shall nevertheless be entitled to vote on the following matters:
petitioner misleadingly cited in the Petition x x x – which supports petitioner’s view that only
common shares should form the basis for computing a public utility’s foreign equity.
1. Amendment of the articles of incorporation;

xxxx
2. Adoption and amendment of by-laws;

18. In addition, the SEC – the government agency primarily responsible for implementing the
3. Sale, lease, exchange, mortgage, pledge or other disposition of all or substantially
Corporation Code, and which also has the responsibility of ensuring compliance with the
all of the corporate property;
Constitution’s foreign equity restrictions as regards nationalized activities x x x – has
categorically ruled that both common and preferred shares are properly considered in
determining outstanding capital stock and the nationality composition thereof. 40 4. Incurring, creating or increasing bonded indebtedness;

We agree with petitioner and petitioners-in-intervention. The term "capital" in Section 11, Article 5. Increase or decrease of capital stock;
XII of the Constitution refers only to shares of stock entitled to vote in the election of directors,
and thus in the present case only to common shares,41 and not to the total outstanding capital
6. Merger or consolidation of the corporation with another corporation or other
stock comprising both common and non-voting preferred shares.
corporations;

The Corporation Code of the Philippines42 classifies shares as common or preferred, thus:
7. Investment of corporate funds in another corporation or business in accordance
with this Code; and
Sec. 6. Classification of shares. - The shares of stock of stock corporations may be divided into
classes or series of shares, or both, any of which classes or series of shares may have such
rights, privileges or restrictions as may be stated in the articles of incorporation: Provided, That 8. Dissolution of the corporation.
no share may be deprived of voting rights except those classified and issued as
"preferred" or "redeemable" shares, unless otherwise provided in this Code: Provided, Except as provided in the immediately preceding paragraph, the vote necessary to approve a
further, That there shall always be a class or series of shares which have complete voting rights. particular corporate act as provided in this Code shall be deemed to refer only to stocks with
Any or all of the shares or series of shares may have a par value or have no par value as may voting rights.
be provided for in the articles of incorporation: Provided, however, That banks, trust companies,
insurance companies, public utilities, and building and loan associations shall not be permitted to
issue no-par value shares of stock. Indisputably, one of the rights of a stockholder is the right to participate in the control or
management of the corporation.43 This is exercised through his vote in the election of directors
because it is the board of directors that controls or manages the corporation. 44 In the absence of
Preferred shares of stock issued by any corporation may be given preference in the distribution provisions in the articles of incorporation denying voting rights to preferred shares, preferred
of the assets of the corporation in case of liquidation and in the distribution of dividends, or such shares have the same voting rights as common shares. However, preferred shareholders are
other preferences as may be stated in the articles of incorporation which are not violative of the often excluded from any control, that is, deprived of the right to vote in the election of directors
provisions of this Code: Provided, That preferred shares of stock may be issued only with a and on other matters, on the theory that the preferred shareholders are merely investors in the
stated par value. The Board of Directors, where authorized in the articles of incorporation, may corporation for income in the same manner as bondholders.45 In fact, under the Corporation
fix the terms and conditions of preferred shares of stock or any series thereof: Provided, That Code only preferred or redeemable shares can be deprived of the right to vote. 46 Common
such terms and conditions shall be effective upon the filing of a certificate thereof with the shares cannot be deprived of the right to vote in any corporate meeting, and any provision in the
Securities and Exchange Commission. articles of incorporation restricting the right of common shareholders to vote is invalid.47

Shares of capital stock issued without par value shall be deemed fully paid and non-assessable Considering that common shares have voting rights which translate to control, as opposed to
and the holder of such shares shall not be liable to the corporation or to its creditors in respect preferred shares which usually have no voting rights, the term "capital" in Section 11, Article XII
thereto: Provided; That shares without par value may not be issued for a consideration less than of the Constitution refers only to common shares. However, if the preferred shares also have the
the value of five (₱5.00) pesos per share: Provided, further, That the entire consideration right to vote in the election of directors, then the term "capital" shall include such preferred
received by the corporation for its no-par value shares shall be treated as capital and shall not shares because the right to participate in the control or management of the corporation is
be available for distribution as dividends. exercised through the right to vote in the election of directors. In short, the term "capital" in
Section 11, Article XII of the Constitution refers only to shares of stock that can vote in
the election of directors.
A corporation may, furthermore, classify its shares for the purpose of insuring compliance with
constitutional or legal requirements.
This interpretation is consistent with the intent of the framers of the Constitution to place in the
Except as otherwise provided in the articles of incorporation and stated in the certificate of stock, hands of Filipino citizens the control and management of public utilities. As revealed in the
deliberations of the Constitutional Commission, "capital" refers to the voting stock or controlling
each share shall be equal in all respects to every other share.
interest of a corporation, to wit:
MR. NOLLEDO. In Sections 3, 9 and 15, the Committee stated local or Filipino equity and MR. AZCUNA. But the control can be with the foreigners even if they are the minority. Let
foreign equity; namely, 60-40 in Section 3, 60-40 in Section 9 and 2/3-1/3 in Section 15. us say 40 percent of the capital is owned by them, but it is the voting capital, whereas, the
Filipinos own the nonvoting shares. So we can have a situation where the corporation is
controlled by foreigners despite being the minority because they have the voting capital.
MR. VILLEGAS. That is right.
That is the anomaly that would result here.

MR. NOLLEDO. In teaching law, we are always faced with this question: "Where do we base the
MR. BENGZON. No, the reason we eliminated the word "stock" as stated in the 1973 and
equity requirement, is it on the authorized capital stock, on the subscribed capital stock, or on
1935 Constitutions is that according to Commissioner Rodrigo, there are associations
the paid-up capital stock of a corporation"? Will the Committee please enlighten me on this?
that do not have stocks. That is why we say "CAPITAL."

MR. VILLEGAS. We have just had a long discussion with the members of the team from the UP
MR. AZCUNA. We should not eliminate the phrase "controlling interest."
Law Center who provided us a draft. The phrase that is contained here which we adopted
from the UP draft is "60 percent of voting stock."
MR. BENGZON. In the case of stock corporations, it is assumed.49 (Emphasis supplied)
MR. NOLLEDO. That must be based on the subscribed capital stock, because unless declared
delinquent, unpaid capital stock shall be entitled to vote. Thus, 60 percent of the "capital" assumes, or should result in, "controlling interest" in the
corporation. Reinforcing this interpretation of the term "capital," as referring to controlling interest
or shares entitled to vote, is the definition of a "Philippine national" in the Foreign Investments
MR. VILLEGAS. That is right.
Act of 1991,50 to wit:

MR. NOLLEDO. Thank you.


SEC. 3. Definitions. - As used in this Act:

With respect to an investment by one corporation in another corporation, say, a corporation with
a. The term "Philippine national" shall mean a citizen of the Philippines; or a domestic
60-40 percent equity invests in another corporation which is permitted by the Corporation Code,
partnership or association wholly owned by citizens of the Philippines; or a corporation
does the Committee adopt the grandfather rule?
organized under the laws of the Philippines of which at least sixty percent (60%) of the
capital stock outstanding and entitled to vote is owned and held by citizens of the
MR. VILLEGAS. Yes, that is the understanding of the Committee. Philippines; or a corporation organized abroad and registered as doing business in the
Philippines under the Corporation Code of which one hundred percent (100%) of the capital
stock outstanding and entitled to vote is wholly owned by Filipinos or a trustee of funds for
MR. NOLLEDO. Therefore, we need additional Filipino capital?
pension or other employee retirement or separation benefits, where the trustee is a Philippine
national and at least sixty percent (60%) of the fund will accrue to the benefit of Philippine
MR. VILLEGAS. Yes.48 nationals: Provided, That where a corporation and its non-Filipino stockholders own stocks in a
Securities and Exchange Commission (SEC) registered enterprise, at least sixty percent (60%)
of the capital stock outstanding and entitled to vote of each of both corporations must be owned
xxxx and held by citizens of the Philippines and at least sixty percent (60%) of the members of the
Board of Directors of each of both corporations must be citizens of the Philippines, in order that
MR. AZCUNA. May I be clarified as to that portion that was accepted by the Committee. the corporation, shall be considered a "Philippine national." (Emphasis supplied)

MR. VILLEGAS. The portion accepted by the Committee is the deletion of the phrase "voting In explaining the definition of a "Philippine national," the Implementing Rules and Regulations of
stock or controlling interest." the Foreign Investments Act of 1991 provide:

MR. AZCUNA. Hence, without the Davide amendment, the committee report would read: b. "Philippine national" shall mean a citizen of the Philippines or a domestic partnership or
"corporations or associations at least sixty percent of whose CAPITAL is owned by such association wholly owned by the citizens of the Philippines; or a corporation organized under
citizens." the laws of the Philippines of which at least sixty percent [60%] of the capital stock
outstanding and entitled to vote is owned and held by citizens of the Philippines; or a
trustee of funds for pension or other employee retirement or separation benefits, where the
MR. VILLEGAS. Yes. trustee is a Philippine national and at least sixty percent [60%] of the fund will accrue to the
benefit of the Philippine nationals; Provided, that where a corporation its non-Filipino
MR. AZCUNA. So if the Davide amendment is lost, we are stuck with 60 percent of the capital to stockholders own stocks in a Securities and Exchange Commission [SEC] registered enterprise,
be owned by citizens. at least sixty percent [60%] of the capital stock outstanding and entitled to vote of both
corporations must be owned and held by citizens of the Philippines and at least sixty percent
[60%] of the members of the Board of Directors of each of both corporation must be citizens of
MR. VILLEGAS. That is right. the Philippines, in order that the corporation shall be considered a Philippine national. The
control test shall be applied for this purpose.
Compliance with the required Filipino ownership of a corporation shall be determined on of public utilities in the hands of Filipinos. It also renders illusory the State policy of an
the basis of outstanding capital stock whether fully paid or not, but only such stocks independent national economy effectively controlled by Filipinos.
which are generally entitled to vote are considered.
The example given is not theoretical but can be found in the real world, and in fact exists in the
For stocks to be deemed owned and held by Philippine citizens or Philippine nationals, present case.
mere legal title is not enough to meet the required Filipino equity. Full beneficial
ownership of the stocks, coupled with appropriate voting rights is essential. Thus,
Holders of PLDT preferred shares are explicitly denied of the right to vote in the election of
stocks, the voting rights of which have been assigned or transferred to aliens cannot be
directors. PLDT’s Articles of Incorporation expressly state that "the holders of Serial Preferred
considered held by Philippine citizens or Philippine nationals.
Stock shall not be entitled to vote at any meeting of the stockholders for the election of
directors or for any other purpose or otherwise participate in any action taken by the
Individuals or juridical entities not meeting the aforementioned qualifications are corporation or its stockholders, or to receive notice of any meeting of stockholders." 51
considered as non-Philippine nationals. (Emphasis supplied)
On the other hand, holders of common shares are granted the exclusive right to vote in the
Mere legal title is insufficient to meet the 60 percent Filipino-owned "capital" required in the election of directors. PLDT’s Articles of Incorporation52 state that "each holder of Common
Constitution. Full beneficial ownership of 60 percent of the outstanding capital stock, coupled Capital Stock shall have one vote in respect of each share of such stock held by him on all
with 60 percent of the voting rights, is required. The legal and beneficial ownership of 60 percent matters voted upon by the stockholders, and the holders of Common Capital Stock shall
of the outstanding capital stock must rest in the hands of Filipino nationals in accordance with have the exclusive right to vote for the election of directors and for all other purposes."53
the constitutional mandate. Otherwise, the corporation is "considered as non-Philippine
national[s]."
In short, only holders of common shares can vote in the election of directors, meaning only
common shareholders exercise control over PLDT. Conversely, holders of preferred shares, who
Under Section 10, Article XII of the Constitution, Congress may "reserve to citizens of the have no voting rights in the election of directors, do not have any control over PLDT. In fact,
Philippines or to corporations or associations at least sixty per centum of whose capital is owned under PLDT’s Articles of Incorporation, holders of common shares have voting rights for all
by such citizens, or such higher percentage as Congress may prescribe, certain areas of purposes, while holders of preferred shares have no voting right for any purpose whatsoever.
investments." Thus, in numerous laws Congress has reserved certain areas of investments to
Filipino citizens or to corporations at least sixty percent of the "capital" of which is owned by
It must be stressed, and respondents do not dispute, that foreigners hold a majority of the
Filipino citizens. Some of these laws are: (1) Regulation of Award of Government Contracts or
common shares of PLDT. In fact, based on PLDT’s 2010 General Information Sheet
R.A. No. 5183; (2) Philippine Inventors Incentives Act or R.A. No. 3850; (3) Magna Carta for
(GIS),54 which is a document required to be submitted annually to the Securities and Exchange
Micro, Small and Medium Enterprises or R.A. No. 6977; (4) Philippine Overseas Shipping
Commission,55 foreigners hold 120,046,690 common shares of PLDT whereas Filipinos hold
Development Act or R.A. No. 7471; (5) Domestic Shipping Development Act of 2004 or R.A. No.
only 66,750,622 common shares.56 In other words, foreigners hold 64.27% of the total number of
9295; (6) Philippine Technology Transfer Act of 2009 or R.A. No. 10055; and (7) Ship Mortgage
PLDT’s common shares, while Filipinos hold only 35.73%. Since holding a majority of the
Decree or P.D. No. 1521. Hence, the term "capital" in Section 11, Article XII of the Constitution
common shares equates to control, it is clear that foreigners exercise control over PLDT. Such
is also used in the same context in numerous laws reserving certain areas of investments to
amount of control unmistakably exceeds the allowable 40 percent limit on foreign ownership of
Filipino citizens.
public utilities expressly mandated in Section 11, Article XII of the Constitution.

To construe broadly the term "capital" as the total outstanding capital stock, including both
Moreover, the Dividend Declarations of PLDT for 2009,57 as submitted to the SEC, shows that
common and non-votingpreferred shares, grossly contravenes the intent and letter of the
per share the SIP58preferred shares earn a pittance in dividends compared to the common
Constitution that the "State shall develop a self-reliant and independent national
shares. PLDT declared dividends for the common shares at ₱70.00 per share, while the
economy effectively controlled by Filipinos." A broad definition unjustifiably disregards who
declared dividends for the preferred shares amounted to a measly ₱1.00 per share. 59 So the
owns the all-important voting stock, which necessarily equates to control of the public utility.
preferred shares not only cannot vote in the election of directors, they also have very little and
obviously negligible dividend earning capacity compared to common shares.
We shall illustrate the glaring anomaly in giving a broad definition to the term "capital." Let us
assume that a corporation has 100 common shares owned by foreigners and 1,000,000 non-
As shown in PLDT’s 2010 GIS,60 as submitted to the SEC, the par value of PLDT common
voting preferred shares owned by Filipinos, with both classes of share having a par value of one
shares is ₱5.00 per share, whereas the par value of preferred shares is ₱10.00 per share. In
peso (₱1.00) per share. Under the broad definition of the term "capital," such corporation would
other words, preferred shares have twice the par value of common shares but cannot elect
be considered compliant with the 40 percent constitutional limit on foreign equity of public utilities
directors and have only 1/70 of the dividends of common shares. Moreover, 99.44% of the
since the overwhelming majority, or more than 99.999 percent, of the total outstanding capital
preferred shares are owned by Filipinos while foreigners own only a minuscule 0.56% of the
stock is Filipino owned. This is obviously absurd.
preferred shares.61 Worse, preferred shares constitute 77.85% of the authorized capital stock of
PLDT while common shares constitute only 22.15%.62 This undeniably shows that beneficial
In the example given, only the foreigners holding the common shares have voting rights in the interest in PLDT is not with the non-voting preferred shares but with the common shares,
election of directors, even if they hold only 100 shares. The foreigners, with a minuscule equity blatantly violating the constitutional requirement of 60 percent Filipino control and Filipino
of less than 0.001 percent, exercise control over the public utility. On the other hand, the beneficial ownership in a public utility.
Filipinos, holding more than 99.999 percent of the equity, cannot vote in the election of directors
and hence, have no control over the public utility. This starkly circumvents the intent of the
framers of the Constitution, as well as the clear language of the Constitution, to place the control
The legal and beneficial ownership of 60 percent of the outstanding capital stock must rest in the executing, the legislature would have the power to ignore and practically nullify the mandate of
hands of Filipinos in accordance with the constitutional mandate. Full beneficial ownership of 60 the fundamental law. This can be cataclysmic. That is why the prevailing view is, as it has
percent of the outstanding capital stock, coupled with 60 percent of the voting rights, is always been, that —
constitutionally required for the State’s grant of authority to operate a public utility. The
undisputed fact that the PLDT preferred shares, 99.44% owned by Filipinos, are non-voting and
. . . in case of doubt, the Constitution should be considered self-executing rather than non-self-
earn only 1/70 of the dividends that PLDT common shares earn, grossly violates the
executing. . . . Unless the contrary is clearly intended, the provisions of the Constitution
constitutional requirement of 60 percent Filipino control and Filipino beneficial ownership of a
should be considered self-executing, as a contrary rule would give the legislature
public utility.
discretion to determine when, or whether, they shall be effective. These provisions would
be subordinated to the will of the lawmaking body, which could make them entirely meaningless
In short, Filipinos hold less than 60 percent of the voting stock, and earn less than 60 by simply refusing to pass the needed implementing statute. (Emphasis supplied)
percent of the dividends, of PLDT. This directly contravenes the express command in Section
11, Article XII of the Constitution that "[n]o franchise, certificate, or any other form of
In Manila Prince Hotel, even the Dissenting Opinion of then Associate Justice Reynato S. Puno,
authorization for the operation of a public utility shall be granted except to x x x corporations x x
later Chief Justice, agreed that constitutional provisions are presumed to be self-executing.
x organized under the laws of the Philippines, at least sixty per centum of whose capital is
Justice Puno stated:
owned by such citizens x x x."

Courts as a rule consider the provisions of the Constitution as self-executing, rather than as
To repeat, (1) foreigners own 64.27% of the common shares of PLDT, which class of shares
requiring future legislation for their enforcement. The reason is not difficult to discern. For if they
exercises the sole right to vote in the election of directors, and thus exercise control over PLDT;
are not treated as self-executing, the mandate of the fundamental law ratified by the
(2) Filipinos own only 35.73% of PLDT’s common shares, constituting a minority of the voting
sovereign people can be easily ignored and nullified by Congress. Suffused with wisdom
stock, and thus do not exercise control over PLDT; (3) preferred shares, 99.44% owned by
of the ages is the unyielding rule that legislative actions may give breath to constitutional
Filipinos, have no voting rights; (4) preferred shares earn only 1/70 of the dividends that
rights but congressional inaction should not suffocate them.
common shares earn;63 (5) preferred shares have twice the par value of common shares; and
(6) preferred shares constitute 77.85% of the authorized capital stock of PLDT and common
shares only 22.15%. This kind of ownership and control of a public utility is a mockery of the Thus, we have treated as self-executing the provisions in the Bill of Rights on arrests, searches
Constitution. and seizures, the rights of a person under custodial investigation, the rights of an accused, and
the privilege against self-incrimination. It is recognized that legislation is unnecessary to enable
courts to effectuate constitutional provisions guaranteeing the fundamental rights of life, liberty
Incidentally, the fact that PLDT common shares with a par value of ₱5.00 have a current stock
and the protection of property. The same treatment is accorded to constitutional provisions
market value of ₱2,328.00 per share,64 while PLDT preferred shares with a par value of ₱10.00
forbidding the taking or damaging of property for public use without just compensation.
per share have a current stock market value ranging from only ₱10.92 to ₱11.06 per share, 65 is
(Emphasis supplied)
a glaring confirmation by the market that control and beneficial ownership of PLDT rest with the
common shares, not with the preferred shares.
Thus, in numerous cases,67 this Court, even in the absence of implementing legislation, applied
directly the provisions of the 1935, 1973 and 1987 Constitutions limiting land ownership to
Indisputably, construing the term "capital" in Section 11, Article XII of the Constitution to include
Filipinos. In Soriano v. Ong Hoo,68this Court ruled:
both voting and non-voting shares will result in the abject surrender of our telecommunications
industry to foreigners, amounting to a clear abdication of the State’s constitutional duty to limit
control of public utilities to Filipino citizens. Such an interpretation certainly runs counter to the x x x As the Constitution is silent as to the effects or consequences of a sale by a citizen of his
constitutional provision reserving certain areas of investment to Filipino citizens, such as the land to an alien, and as both the citizen and the alien have violated the law, none of them should
exploitation of natural resources as well as the ownership of land, educational institutions and have a recourse against the other, and it should only be the State that should be allowed to
advertising businesses. The Court should never open to foreign control what the Constitution intervene and determine what is to be done with the property subject of the violation. We have
has expressly reserved to Filipinos for that would be a betrayal of the Constitution and of the said that what the State should do or could do in such matters is a matter of public policy,
national interest. The Court must perform its solemn duty to defend and uphold the intent and entirely beyond the scope of judicial authority. (Dinglasan, et al. vs. Lee Bun Ting, et al., 6 G. R.
letter of the Constitution to ensure, in the words of the Constitution, "a self-reliant and No. L-5996, June 27, 1956.) While the legislature has not definitely decided what policy
independent national economy effectively controlled by Filipinos." should be followed in cases of violations against the constitutional prohibition, courts of
justice cannot go beyond by declaring the disposition to be null and void as violative of
the Constitution. x x x (Emphasis supplied)
Section 11, Article XII of the Constitution, like other provisions of the Constitution expressly
reserving to Filipinos specific areas of investment, such as the development of natural resources
and ownership of land, educational institutions and advertising business, is self-executing. To treat Section 11, Article XII of the Constitution as not self-executing would mean that since
There is no need for legislation to implement these self-executing provisions of the Constitution. the 1935 Constitution, or over the last 75 years, not one of the constitutional provisions
The rationale why these constitutional provisions are self-executing was explained in Manila expressly reserving specific areas of investments to corporations, at least 60 percent of the
Prince Hotel v. GSIS,66 thus: "capital" of which is owned by Filipinos, was enforceable. In short, the framers of the 1935, 1973
and 1987 Constitutions miserably failed to effectively reserve to Filipinos specific areas of
investment, like the operation by corporations of public utilities, the exploitation by corporations
x x x Hence, unless it is expressly provided that a legislative act is necessary to enforce a
of mineral resources, the ownership by corporations of real estate, and the ownership of
constitutional mandate, the presumption now is that all provisions of the constitution are self-
educational institutions. All the legislatures that convened since 1935 also miserably failed to
executing. If the constitutional provisions are treated as requiring legislation instead of self-
enact legislations to implement these vital constitutional provisions that determine who will
effectively control the national economy, Filipinos or foreigners. This Court cannot allow such an
absurd interpretation of the Constitution.

This Court has held that the SEC "has both regulatory and adjudicative functions." 69 Under its
regulatory functions, the SEC can be compelled by mandamus to perform its statutory duty when
it unlawfully neglects to perform the same. Under its adjudicative or quasi-judicial functions, the
SEC can be also be compelled by mandamus to hear and decide a possible violation of any law
it administers or enforces when it is mandated by law to investigate such violation.1awphi1

Under Section 17(4)70 of the Corporation Code, the SEC has the regulatory function to reject or
disapprove the Articles of Incorporation of any corporation where "the required percentage of
ownership of the capital stock to be owned by citizens of the Philippines has not been
complied with as required by existing laws or the Constitution." Thus, the SEC is the
government agency tasked with the statutory duty to enforce the nationality requirement
prescribed in Section 11, Article XII of the Constitution on the ownership of public utilities. This
Court, in a petition for declaratory relief that is treated as a petition for mandamus as in the
present case, can direct the SEC to perform its statutory duty under the law, a duty that the SEC
has apparently unlawfully neglected to do based on the 2010 GIS that respondent PLDT
submitted to the SEC.

Under Section 5(m) of the Securities Regulation Code,71 the SEC is vested with the "power and
function" to "suspend or revoke, after proper notice and hearing, the franchise or
certificate of registration of corporations, partnerships or associations, upon any of the
grounds provided by law." The SEC is mandated under Section 5(d) of the same Code with
the "power and function" to "investigate x x x the activities of persons to ensure
compliance" with the laws and regulations that SEC administers or enforces. The GIS that all
corporations are required to submit to SEC annually should put the SEC on guard against
violations of the nationality requirement prescribed in the Constitution and existing laws. This
Court can compel the SEC, in a petition for declaratory relief that is treated as a petition for
mandamus as in the present case, to hear and decide a possible violation of Section 11, Article
XII of the Constitution in view of the ownership structure of PLDT’s voting shares, as admitted by
respondents and as stated in PLDT’s 2010 GIS that PLDT submitted to SEC.

WHEREFORE, we PARTLY GRANT the petition and rule that the term "capital" in Section 11,
Article XII of the 1987 Constitution refers only to shares of stock entitled to vote in the election of
directors, and thus in the present case only to common shares, and not to the total outstanding
capital stock (common and non-voting preferred shares). Respondent Chairperson of the
Securities and Exchange Commission is DIRECTED to apply this definition of the term "capital"
in determining the extent of allowable foreign ownership in respondent Philippine Long Distance
Telephone Company, and if there is a violation of Section 11, Article XII of the Constitution, to
impose the appropriate sanctions under the law.

SO ORDERED.
G.R. No. 117604 March 26, 1997 On 4 December 1986, VGCCI caused to be published in the newspaper Daily Express a notice
of auction sale of a number of its stock certificates, to be held on 10 December 1986 at 10:00
a.m. Included therein was Calapatia's own share of stock (Stock Certificate No. 1219).
CHINA BANKING CORPORATION, petitioner,
vs.
COURT OF APPEALS, and VALLEY GOLF and COUNTRY CLUB, INC., respondents. Through a letter dated 15 December 1986, VGCCI informed Calapatia of the termination of his
membership due to the sale of his share of stock in the 10 December 1986 auction. 11
KAPUNAN, J.:
On 5 May 1989, petitioner advised VGCCI that it is the new owner of Calapatia's Stock
Certificate No. 1219 by virtue of being the highest bidder in the 17 September 1985 auction and
Through a petition for review on certiorari under Rule 45 of the Revised Rules of Court,
requested that a new certificate of stock be issued in its name. 12
petitioner China Banking Corporation seeks the reversal of the decision of the Court of Appeals
dated 15 August 1994 nullifying the Securities and Exchange Commission's order and resolution
dated 4 June 1993 and 7 December 1993, respectively, for lack of jurisdiction. Similarly On 2 March 1990, VGCCI replied that "for reason of delinquency" Calapatia's stock was sold at
impugned is the Court of Appeals' resolution dated 4 September 1994 which denied petitioner's the public auction held on 10 December 1986 for P25,000.00. 13
motion for reconsideration.
On 9 March 1990, petitioner protested the sale by VGCCI of the subject share of stock and
The case unfolds thus: thereafter filed a case with the Regional Trial Court of Makati for the nullification of the 10
December 1986 auction and for the issuance of a new stock certificate in its name. 14
On 21 August 1974, Galicano Calapatia, Jr. (Calapatia, for brevity) a stockholder of private
respondent Valley Golf & Country Club, Inc. (VGCCI, for brevity), pledged his Stock Certificate On 18 June 1990, the Regional Trial Court of Makati dismissed the complaint for lack of
No. 1219 to petitioner China Banking Corporation (CBC, for brevity).1 jurisdiction over the subject matter on the theory that it involves an intra-corporate dispute and
on 27 August 1990 denied petitioner's motion for reconsideration.
On 16 September 1974, petitioner wrote VGCCI requesting that the aforementioned pledge
agreement be recorded in its books.2 On 20 September 1990, petitioner filed a complaint with the Securities and Exchange
Commission (SEC) for the nullification of the sale of Calapatia's stock by VGCCI; the
cancellation of any new stock certificate issued pursuant thereto; for the issuance of a new
In a letter dated 27 September 1974, VGCCI replied that the deed of pledge executed by
certificate in petitioner's name; and for damages, attorney's fees and costs of litigation.
Calapatia in petitioner's favor was duly noted in its corporate books.3

On 3 January 1992, SEC Hearing Officer Manuel P. Perea rendered a decision in favor of
On 3 August 1983, Calapatia obtained a loan of P20,000.00 from petitioner, payment of which
VGCCI, stating in the main that "(c)onsidering that the said share is delinquent, (VGCCI) had
was secured by the aforestated pledge agreement still existing between Calapatia and
valid reason not to transfer the share in the name of the petitioner in the books of (VGCCI) until
petitioner.4
liquidation of
delinquency." 15 Consequently, the case was dismissed. 16
Due to Calapatia's failure to pay his obligation, petitioner, on 12 April 1985, filed a petition for
extrajudicial foreclosure before Notary Public Antonio T. de Vera of Manila, requesting the latter
On 14 April 1992, Hearing Officer Perea denied petitioner's motion for reconsideration. 17
to conduct a public auction sale of the pledged stock.5

Petitioner appealed to the SEC en banc and on 4 June 1993, the Commission issued an order
On 14 May 1985, petitioner informed VGCCI of the above-mentioned foreclosure proceedings
reversing the decision of its hearing officer. It declared thus:
and requested that the pledged stock be transferred to its (petitioner's) name and the same be
recorded in the corporate books. However, on 15 July 1985, VGCCI wrote petitioner expressing
its inability to accede to petitioner's request in view of Calapatia's unsettled accounts with the The Commission en banc believes that appellant-petitioner has a prior right
club.6 over the pledged share and because of pledgor's failure to pay the principal
debt upon maturity, appellant-petitioner can proceed with the foreclosure of
the pledged share.
Despite the foregoing, Notary Public de Vera held a public auction on 17 September 1985 and
petitioner emerged as the highest bidder at P20,000.00 for the pledged stock. Consequently,
petitioner was issued the corresponding certificate of sale. 7 WHEREFORE, premises considered, the Orders of January 3, 1992 and
April 14, 1992 are hereby SET ASIDE. The auction sale conducted by
appellee-respondent Club on December 10, 1986 is declared NULL and
On 21 November 1985, VGCCI sent Calapatia a notice demanding full payment of his overdue
VOID. Finally, appellee-respondent Club is ordered to issue another
account in the amount of P18,783.24. 8 Said notice was followed by a demand letter dated 12
membership certificate in the name of appellant-petitioner bank.
December 1985 for the same amount9 and another notice dated 22 November 1986 for
P23,483.24. 10
SO ORDERED. 18
VGCCI sought reconsideration of the abovecited order. However, the SEC denied the same in WHETHER OR NOT RESPONDENT COURT OF APPEALS (Former Eighth
its resolution dated 7 December 1993. 19 Division) GRAVELY ERRED WHEN:

The sudden turn of events sent VGCCI to seek redress from the Court of Appeals. On 15 August 1. IT NULLIFIED AND SET ASIDE THE DECISION DATED JUNE 04, 1993
1994, the Court of Appeals rendered its decision nullifying and setting aside the orders of the AND ORDER DATED DECEMBER 07, 1993 OF THE SECURITIES AND
SEC and its hearing officer on ground of lack of jurisdiction over the subject matter and, EXCHANGE COMMISSION EN BANC, AND WHEN IT DISMISSED THE
consequently, dismissed petitioner's original complaint. The Court of Appeals declared that the COMPLAINT OF PETITIONER AGAINST RESPONDENT VALLEY GOLF
controversy between CBC and VGCCI is not intra-corporate. It ruled as follows: ALL FOR LACK OF JURISDICTION OVER THE SUBJECT MATTER OF
THE CASE;
In order that the respondent Commission can take cognizance of a case,
the controversy must pertain to any of the following relationships: (a) 2. IT FAILED TO AFFIRM THE DECISION OF THE SECURITIES AND
between the corporation, partnership or association and the public; (b) EXCHANGE COMMISSION EN BANC DATED JUNE 04, 1993 DESPITE
between the corporation, partnership or association and its stockholders, PREPONDERANT EVIDENCE SHOWING THAT PETITIONER IS THE
partners, members, or officers; (c) between the corporation, partnership or LAWFUL OWNER OF MEMBERSHIP CERTIFICATE NO. 1219 FOR ONE
association and the state in so far as its franchise, permit or license to SHARE OF RESPONDENT VALLEY GOLF.
operate is concerned, and (d) among the stockholders, partners or
associates themselves (Union Glass and Container Corporation vs. SEC,
The petition is granted.
November 28, 1983, 126 SCRA 31). The establishment of any of the
relationship mentioned will not necessarily always confer jurisdiction over
the dispute on the Securities and Exchange Commission to the exclusion of The basic issue we must first hurdle is which body has jurisdiction over the controversy, the
the regular courts. The statement made in Philex Mining Corp. vs. Reyes, regular courts or the SEC.
118 SCRA 602, that the rule admits of no exceptions or distinctions is not
that absolute. The better policy in determining which body has jurisdiction
over a case would be to consider not only the status or relationship of the P. D. No. 902-A conferred upon the SEC the following pertinent powers:
parties but also the nature of the question that is the subject of their
controversy (Viray vs. Court of Appeals, November 9, 1990, 191 SCRA 308, Sec. 3. The Commission shall have absolute jurisdiction, supervision and
322-323). control over all corporations, partnerships or associations, who are the
grantees of primary franchises and/or a license or permit issued by the
government to operate in the Philippines, and in the exercise of its authority,
Indeed, the controversy between petitioner and respondent bank which
involves ownership of the stock that used to belong to Calapatia, Jr. is not it shall have the power to enlist the aid and support of and to deputize any
within the competence of respondent Commission to decide. It is not any of and all enforcement agencies of the government, civil or military as well as
those mentioned in the aforecited case. any private institution, corporation, firm, association or person.

WHEREFORE, the decision dated June 4, 1993, and order dated December xxx xxx xxx
7, 1993 of respondent Securities and Exchange Commission (Annexes Y
and BB, petition) and of its hearing officer dated January 3, 1992 and April Sec. 5. In addition to the regulatory and adjudicative functions of the
14, 1992 (Annexes S and W, petition) are all nullified and set aside for lack Securities and Exchange Commission over corporations, partnerships and
of jurisdiction over the subject matter of the case. Accordingly, the complaint other forms of associations registered with it as expressly granted under
of respondent China Banking Corporation (Annex Q, petition) is existing laws and decrees, it shall have original and exclusive jurisdiction to
DISMISSED. No pronouncement as to costs in this instance. hear and decide cases involving:

SO ORDERED. 20 a) Devices or schemes employed by or any acts of the


board of directors, business associates, its officers or
Petitioner moved for reconsideration but the same was denied by the Court of Appeals in its partners, amounting to fraud and misrepresentation
resolution dated 5 October 1994. 21 which may be detrimental to the interest of the public
and/or of the stockholders, partners, members of
associations or organizations registered with the
Hence, this petition wherein the following issues were raised: Commission.

II b) Controversies arising out of intra-corporate or


partnership relations, between and among
stockholders, members, or associates; between any or
ISSUES
all of them and the corporation, partnership or
association of which they are stockholders, members or
associates, respectively; and between such VGCCI's aforequoted by-laws, a subject which irrefutably calls for the special competence of the
corporation, partnership or association and the State SEC.
insofar as it concerns their individual franchise or right
to exist as such entity;
We reiterate herein the sound policy enunciated by the Court in Abejo v. De la Cruz 27:

c) Controversies in the election or appointment of


6. In the fifties, the Court taking cognizance of the move to vest jurisdiction
directors, trustees, officers, or managers of such
in administrative commissions and boards the power to resolve specialized
corporations, partnerships or associations.
disputes in the field of labor (as in corporations, public transportation and
public utilities) ruled that Congress in requiring the Industrial Court's
d) Petitions of corporations, partnerships or intervention in the resolution of labor-management controversies likely to
associations to be declared in the state of suspension cause strikes or lockouts meant such jurisdiction to be exclusive, although it
of payments in cases where the corporation, did not so expressly state in the law. The Court held that under the "sense-
partnership or association possesses property to cover making and expeditious doctrine of primary jurisdiction . . . the courts cannot
all of its debts but foresees the impossibility of meeting or will not determine a controversy involving a question which is within the
them when they respectively fall due or in cases where jurisdiction of an administrative tribunal, where the question demands the
the corporation, partnership or association has no exercise of sound administrative discretion requiring the special knowledge,
sufficient assets to cover its liabilities, but is under the experience, and services of the administrative tribunal to determine
Management Committee created pursuant to this technical and intricate matters of fact, and a uniformity of ruling is essential
Decree. to comply with the purposes of the regulatory statute administered.

The aforecited law was expounded upon in Viray v. CA 22 and in the recent cases of Mainland In this era of clogged court dockets, the need for specialized administrative
Construction Co., Inc. v. Movilla 23 and Bernardo v. CA, 24 thus: boards or commissions with the special knowledge, experience and
capability to hear and determine promptly disputes on technical matters or
essentially factual matters, subject to judicial review in case of grave abuse
. . . .The better policy in determining which body has jurisdiction over a case
of discretion, has become well nigh indispensable. Thus, in 1984, the Court
would be to consider not only the status or relationship of the parties but
noted that "between the power lodged in an administrative body and a court,
also the nature of the question that is the subject of their controversy.
the unmistakable trend has been to refer it to the former. 'Increasingly, this
Court has been committed to the view that unless the law speaks clearly
Applying the foregoing principles in the case at bar, to ascertain which tribunal has jurisdiction and unequivocably, the choice should fall on [an administrative agency.]'"
we have to determine therefore whether or not petitioner is a stockholder of VGCCI and whether The Court in the earlier case of Ebon v. De Guzman, noted that the
or not the nature of the controversy between petitioner and private respondent corporation is lawmaking authority, in restoring to the labor arbiters and the NLRC their
intra-corporate. jurisdiction to award all kinds of damages in labor cases, as against the
previous P.D. amendment splitting their jurisdiction with the regular courts,
"evidently, . . . had second thoughts about depriving the Labor Arbiters and
As to the first query, there is no question that the purchase of the subject share or membership the NLRC of the jurisdiction to award damages in labor cases because that
certificate at public auction by petitioner (and the issuance to it of the corresponding Certificate setup would mean duplicity of suits, splitting the cause of action and
of Sale) transferred ownership of the same to the latter and thus entitled petitioner to have the
possible conflicting findings and conclusions by two tribunals on one and the
said share registered in its name as a member of VGCCI. It is readily observed that VGCCI did same claim."
not assail the transfer directly and has in fact, in its letter of 27 September 1974, expressly
recognized the pledge agreement executed by the original owner, Calapatia, in favor of
petitioner and has even noted said agreement in its corporate books. 25 In addition, Calapatia, In this case, the need for the SEC's technical expertise cannot be over-emphasized involving as
the original owner of the subject share, has not contested the said transfer. it does the meticulous analysis and correct interpretation of a corporation's by-laws as well as
the applicable provisions of the Corporation Code in order to determine the validity of VGCCI's
claims. The SEC, therefore, took proper cognizance of the instant case.
By virtue of the afore-mentioned sale, petitioner became a bona fide stockholder of VGCCI and,
therefore, the conflict that arose between petitioner and VGCCI aptly exemplies an intra-
corporate controversy between a corporation and its stockholder under Sec. 5(b) of P.D. 902-A. VGCCI further contends that petitioner is estopped from denying its earlier position, in the first
complaint it filed with the RTC of Makati (Civil Case No. 90-1112) that there is no intra-corporate
relations between itself and VGCCI.
An important consideration, moreover, is the nature of the controversy between petitioner and
private respondent corporation. VGCCI claims a prior right over the subject share anchored
mainly on Sec. 3, Art VIII of its by-laws which provides that "after a member shall have been VGCCI's contention lacks merit.
posted as delinquent, the Board may order his/her/its share sold to satisfy the claims of the Club.
. ." 26 It is pursuant to this provision that VGCCI also sold the subject share at public auction, of
In Zamora v. Court of Appeals, 28 this Court, through Mr. Justice Isagani A. Cruz, declared that:
which it was the highest bidder. VGCCI caps its argument by asserting that its corporate by-laws
should prevail. The bone of contention, thus, is the proper interpretation and application of
It follows that as a rule the filing of a complaint with one court which has no assist the parties in obtaining just, speedy and inexpensive determination of
jurisdiction over it does not prevent the plaintiff from filing the same every action or proceeding. The Court, therefore, feels that the central
complaint later with the competent court. The plaintiff is not estopped from issues of the case, albeit unresolved by the courts below, should now be
doing so simply because it made a mistake before in the choice of the settled specially as they involved pure questions of law. Furthermore, the
proper forum. . . . pleadings of the respective parties on file have amply ventilated their
various positions and arguments on the matter necessitating prompt
adjudication.
We remind VGCCI that in the same proceedings before the RTC of Makati, it categorically stated
(in its motion to dismiss) that the case between itself and petitioner is intra-corporate and
insisted that it is the SEC and not the regular courts which has jurisdiction. This is precisely the In the case at bar, since we already have the records of the case (from the proceedings before
reason why the said court dismissed petitioner's complaint and led to petitioner's recourse to the the SEC) sufficient to enable us to render a sound judgment and since only questions of law
SEC. were raised (the proper jurisdiction for Supreme Court review), we can, therefore, unerringly take
cognizance of and rule on the merits of the case.
Having resolved the issue on jurisdiction, instead of remanding the whole case to the Court of
Appeals, this Court likewise deems it procedurally sound to proceed and rule on its merits in the The procedural niceties settled, we proceed to the merits.
same proceedings.
VGCCI assails the validity of the pledge agreement executed by Calapatia in petitioner's favor. It
It must be underscored that petitioner did not confine the instant petition for review contends that the same was null and void for lack of consideration because the pledge
on certiorari on the issue of jurisdiction. In its assignment of errors, petitioner specifically raised agreement was entered into on 21 August
questions on the merits of the case. In turn, in its responsive pleadings, private respondent duly 1974 33 but the loan or promissory note which it secured was obtained by Calapatia much later
answered and countered all the issues raised by petitioner. or only on 3 August 1983. 34

Applicable to this case is the principle succinctly enunciated in the case of Heirs of Crisanta VGCCI's contention is unmeritorious.
Y. Gabriel-Almoradie v. Court of Appeals, 29 citing Escudero v. Dulay 30 and The Roman Catholic
Archbishop of Manila v. Court of Appeals. 31
A careful perusal of the pledge agreement will readily reveal that the contracting parties explicitly
stipulated therein that the said pledge will also stand as security for any future advancements (or
In the interest of the public and for the expeditious administration of justice renewals thereof) that Calapatia (the pledgor) may procure from petitioner:
the issue on infringement shall be resolved by the court considering that this
case has dragged on for years and has gone from one forum to another.
xxx xxx xxx

It is a rule of procedure for the Supreme Court to strive to settle the entire
This pledge is given as security for the prompt payment when due of all
controversy in a single proceeding leaving no root or branch to bear the
loans, overdrafts, promissory notes, drafts, bills or exchange, discounts, and
seeds of future litigation. No useful purpose will be served if a case or the
all other obligations of every kind which have heretofore been contracted, or
determination of an issue in a case is remanded to the trial court only to
which may hereafter be contracted, by the PLEDGOR(S) and/or
have its decision raised again to the Court of Appeals and from there to the
DEBTOR(S) or any one of them, in favor of the PLEDGEE, including
Supreme Court.
discounts of Chinese drafts, bills of exchange, promissory notes, etc.,
without any further endorsement by the PLEDGOR(S) and/or Debtor(s) up
We have laid down the rule that the remand of the case or of an issue to the to the sum of TWENTY THOUSAND (P20,000.00) PESOS, together with
lower court for further reception of evidence is not necessary where the the accrued interest thereon, as hereinafter provided, plus the costs, losses,
Court is in position to resolve the dispute based on the records before it and damages and expenses (including attorney's fees) which PLEDGEE may
particularly where the ends of justice would not be subserved by the remand incur in connection with the collection thereof. 35 (Emphasis ours.)
thereof. Moreover, the Supreme Court is clothed with ample authority to
review matters, even those not raised on appeal if it finds that their
The validity of the pledge agreement between petitioner and Calapatia cannot thus be held
consideration is necessary in arriving at a just disposition of the case.
suspect by VGCCI. As candidly explained by petitioner, the promissory note of 3 August 1983 in
the amount of P20,000.00 was but a renewal of the first promissory note covered by the same
In the recent case of China Banking Corp., et al. v. Court of Appeals, et al., 32 this Court, through pledge agreement.
Mr. Justice Ricardo J. Francisco, ruled in this wise:
VGCCI likewise insists that due to Calapatia's failure to settle his delinquent accounts, it had the
At the outset, the Court's attention is drawn to the fact that since the filing of right to sell the share in question in accordance with the express provision found in its by-laws.
this suit before the trial court, none of the substantial issues have been
resolved. To avoid and gloss over the issues raised by the parties, as what
Private respondent's insistence comes to naught. It is significant to note that VGCCI began
the trial court and respondent Court of Appeals did, would unduly prolong
sending notices of delinquency to Calapatia after it was informed by petitioner (through its letter
this litigation involving a rather simple case of foreclosure of mortgage.
dated 14 May 1985) of the foreclosure proceedings initiated against Calapatia's pledged share,
Undoubtedly, this will run counter to the avowed purpose of the rules, i.e., to
although Calapatia has been delinquent in paying his monthly dues to the club since 1975. A by-law of a corporation which provides that transfers of stock shall not be
Stranger still, petitioner, whom VGCCI had officially recognized as the pledgee of Calapatia's valid unless approved by the board of directors, while it may be enforced as
share, was neither informed nor furnished copies of these letters of overdue accounts until a reasonable regulation for the protection of the corporation against
VGCCI itself sold the pledged share at another public auction. By doing so, VGCCI completely worthless stockholders, cannot be made available to defeat the rights of
disregarded petitioner's rights as pledgee. It even failed to give petitioner notice of said auction third persons. (Farmers' and Merchants' Bank of Lineville vs. Wasson, 48
sale. Such actuations of VGCCI thus belie its claim of good faith. Iowa, 336.) (Emphasis ours.)

In defending its actions, VGCCI likewise maintains that petitioner is bound by its by-laws. It In order to be bound, the third party must have acquired knowledge of the pertinent by-laws at
argues in this wise: the time the transaction or agreement between said third party and the shareholder was entered
into, in this case, at the time the pledge agreement was executed. VGCCI could have easily
informed petitioner of its by-laws when it sent notice formally recognizing petitioner as pledgee of
The general rule really is that third persons are not bound by the by-laws of
one of its shares registered in Calapatia's name. Petitioner's belated notice of said by-laws at the
a corporation since they are not privy thereto (Fleischer v. Botica Nolasco,
time of foreclosure will not suffice. The ruling of the SEC en banc is particularly instructive:
47 Phil. 584). The exception to this is when third persons have actual or
constructive knowledge of the same. In the case at bar, petitioner had actual
knowledge of the by-laws of private respondent when petitioner foreclosed By-laws signifies the rules and regulations or private laws enacted by the
the pledge made by Calapatia and when petitioner purchased the share corporation to regulate, govern and control its own actions, affairs and
foreclosed on September 17, 1985. This is proven by the fact that prior concerns and its stockholders or members and directors and officers with
thereto, i.e., on May 14, 1985 petitioner even quoted a portion of private relation thereto and among themselves in their relation to it. In other words,
respondent's by-laws which is material to the issue herein in a letter it wrote by-laws are the relatively permanent and continuing rules of action adopted
to private respondent. Because of this actual knowledge of such by-laws by the corporation for its own government and that of the individuals
then the same bound the petitioner as of the time when petitioner purchased composing it and having the direction, management and control of its
the share. Since the by-laws was already binding upon petitioner when the affairs, in whole or in part, in the management and control of its affairs and
latter purchased the share of Calapatia on September 17, 1985 then the activities. (9 Fletcher 4166, 1982 Ed.)
petitioner purchased the said share subject to the right of the private
respondent to sell the said share for reasons of delinquency and the right of
The purpose of a by-law is to regulate the conduct and define the duties of
private respondent to have a first lien on said shares as these rights are
the members towards the corporation and among themselves. They are
provided for in the by-laws very very clearly. 36
self-imposed and, although adopted pursuant to statutory authority, have no
status as public law. (Ibid.)
VGCCI misunderstood the import of our ruling in Fleischer v. Botica Nolasco Co.: 37
Therefore, it is the generally accepted rule that third persons are not bound
And moreover, the by-law now in question cannot have any effect on the by by-laws, except when they have knowledge of the provisions either
appellee. He had no knowledge of such by-law when the shares were actually or constructively. In the case of Fleisher v. Botica Nolasco, 47 Phil.
assigned to him. He obtained them in good faith and for a valuable 584, the Supreme Court held that the by-law restricting the transfer of
consideration. He was not a privy to the contract created by said by-law shares cannot have any effect on the transferee of the shares in question as
between the shareholder Manuel Gonzales and the Botica Nolasco, Inc. he "had no knowledge of such by-law when the shares were assigned to
Said by-law cannot operate to defeat his rights as a purchaser. him. He obtained them in good faith and for a valuable consideration. He
was not a privy to the contract created by the by-law between the
shareholder . . .and the Botica Nolasco, Inc. Said by-law cannot operate to
An unauthorized by-law forbidding a shareholder to sell his shares without
defeat his right as a purchaser. (Emphasis supplied.)
first offering them to the corporation for a period of thirty days is not binding
upon an assignee of the stock as a personal contract, although his assignor
knew of the by-law and took part in its adoption. (10 Cyc., 579; Ireland vs. By analogy of the above-cited case, the Commission en banc is of the
Globe Milling Co., 21 R.I., 9.) opinion that said case is applicable to the present controversy. Appellant-
petitioner bank as a third party can not be bound by appellee-respondent's
by-laws. It must be recalled that when appellee-respondent communicated
When no restriction is placed by public law on the transfer of corporate
to appellant-petitioner bank that the pledge agreement was duly noted in the
stock, a purchaser is not affected by any contractual restriction of which he
club's books there was no mention of the shareholder-pledgor's unpaid
had no notice. (Brinkerhoff-Farris Trust & Savings Co. vs. Home Lumber
accounts. The transcript of stenographic notes of the June 25, 1991 Hearing
Co., 118 Mo., 447.)
reveals that the pledgor became delinquent only in 1975. Thus, appellant-
petitioner was in good faith when the pledge agreement was contracted.
The assignment of shares of stock in a corporation by one who has
assented to an unauthorized by-law has only the effect of a contract by, and
The Commission en banc also believes that for the exception to the general
enforceable against, the assignor; the assignee is not bound by such by-law
accepted rule that third persons are not bound by by-laws to be applicable
by virtue of the assignment alone. (Ireland vs. Globe Milling Co., 21 R.I., 9.)
and binding upon the pledgee, knowledge of the provisions of the VGCI By-
laws must be acquired at the time the pledge agreement was contracted.
Knowledge of said provisions, either actual or constructive, at the time of pledge; and it is the holder who must renew the pledge, if it is to be kept
foreclosure will not affect pledgee's right over the pledged share. Art. 2087 alive.
of the Civil Code provides that it is also of the essence of these contracts
that when the principal obligation becomes due, the things in which the
It is quite obvious from the aforequoted case that a membership share is quite
pledge or mortgage consists maybe alienated for the payment to the
different in character from a pawn ticket and to reiterate, petitioner was never informed
creditor.
of Calapatia's unpaid accounts and the restrictive provisions in VGCCI's by-laws.

In a letter dated March 10, 1976 addressed to Valley Golf Club, Inc., the
Finally, Sec. 63 of the Corporation Code which provides that "no shares of stock against which
Commission issued an opinion to the effect that:
the corporation holds any unpaid claim shall be transferable in the books of the corporation"
cannot be utilized by VGCCI. The term "unpaid claim" refers to "any unpaid claim arising from
According to the weight of authority, the pledgee's right unpaid subscription, and not to any indebtedness which a subscriber or stockholder may owe
is entitled to full protection without surrender of the the corporation arising from any other transaction." 40 In the case at bar, the subscription for the
certificate, their cancellation, and the issuance to him of share in question has been fully paid as evidenced by the issuance of Membership Certificate
new ones, and when done, the pledgee will be fully No. 1219. 41 What Calapatia owed the corporation were merely the monthly dues. Hence, the
protected against a subsequent purchaser who would aforequoted provision does not apply.
be charged with constructive notice that the certificate
is covered by the pledge. (12-A Fletcher 502)
WHEREFORE, premises considered, the assailed decision of the Court of Appeals is
REVERSED and the order of the SEC en banc dated 4 June 1993 is hereby AFFIRMED.
The pledgee is entitled to retain possession of the stock
until the pledgor pays or tenders to him the amount due
SO ORDERED.
on the debt secured. In other words, the pledgee has
the right to resort to its collateral for the payment of the
debts. (Ibid, 502)

To cancel the pledged certificate outright and the


issuance of new certificate to a third person who
purchased the same certificate covered by the pledge,
will certainly defeat the right of the pledgee to resort to
its collateral for the payment of the debt. The pledgor or
his representative or registered stockholders has no
right to require a return of the pledged stock until the
debt for which it was given as security is paid and
satisfied, regardless of the length of time which have
elapsed since debt was created. (12-A Fletcher 409)

A bona fide pledgee takes free from any latent or secret equities or liens in
favor either of the corporation or of third persons, if he has no notice thereof,
but not otherwise. He also takes it free of liens or claims that may
subsequently arise in favor of the corporation if it has notice of the pledge,
although no demand for a transfer of the stock to the pledgee on the
corporate books has been made. (12-A Fletcher 5634, 1982 ed., citing
Snyder v. Eagle Fruit Co., 75 F2d739) 38

Similarly, VGCCI's contention that petitioner is duty-bound to know its by-laws because of Art.
2099 of the Civil Code which stipulates that the creditor must take care of the thing pledged with
the diligence of a good father of a family, fails to convince. The case of Cruz & Serrano
v. Chua A. H. Lee, 39 is clearly not applicable:

In applying this provision to the situation before us it must be borne in mind


that the ordinary pawn ticket is a document by virtue of which the property in
the thing pledged passes from hand to hand by mere delivery of the ticket;
and the contract of the pledge is, therefore, absolvable to bearer. It results
that one who takes a pawn ticket in pledge acquires domination over the
G.R. No. 108905 October 23, 1997 VI. ANNUAL MEETING

GRACE CHRISTIAN HIGH SCHOOL, petitioner, The Annual Meeting of the members of the Association shall be held on
vs. the second Thursday of January of each year. Each Charter or
THE COURT OF APPEALS, GRACE VILLAGE ASSOCIATION, INC., ALEJANDRO G. Associate Member of the Association is entitled to vote. He shall be entitled
BELTRAN, and ERNESTO L. GO, respondents. to as many votes as he has acquired thru his monthly membership fees
only computed on a ratio of TEN (P10.00) PESOS for one vote.
MENDOZA, J.:
The Charter and Associate Members shall elect the Directors of the
Association. The candidates receiving the first fourteen (14) highest number
The question for decision in this case is the right of petitioner's representative to sit in the board
of votes shall be declared and proclaimed elected until their successors are
of directors of respondent Grace Village Association, Inc. as a permanent member thereof. For
elected and qualified. GRACE CHRISTIAN HIGH SCHOOL representative
fifteen years — from 1975 until 1989 — petitioner's representative had been recognized as a
is a permanent Director of the ASSOCIATION.
"permanent director" of the association. But on February 13, 1990, petitioner received notice
from the association's committee on election that the latter was "reexamining" (actually,
reconsidering) the right of petitioner's representative to continue as an unelected member of the This draft was never presented to the general membership for approval. Nevertheless, from
board. As the board denied petitioner's request to be allowed representation without election, 1975, after it was presumably submitted to the board, up to 1990, petitioner was given a
petitioner brought an action for mandamus in the Home Insurance and Guaranty Corporation. Its permanent seat in the board of directors of the association. On February 13, 1990, the
action was dismissed by the hearing officer whose decision was subsequently affirmed by the association's committee on election in a letter informed James Tan, principal of the school, that
appeals board. Petitioner appealed to the Court of Appeals, which in turn upheld the decision of "it was the sentiment that all directors should be elected by members of the association"
the HIGC's appeals board. Hence this petition for review based on the following contentions: because "to make a person or entity a permanent Director would deprive the right of voters to
vote for fifteen (15) members of the Board," and "it is undemocratic for a person or entity to hold
office in perpetuity." 4 For this reason, Tan was told that "the proposal to make the Grace
1. The Petitioner herein has already acquired a vested right to a permanent
Christian High School representative as a permanent director of the association, although
seat in the Board of Directors of Grace Village Association;
previously tolerated in the past elections should be reexamined." Following this advice, notices
were sent to the members of the association that the provision on election of directors of the
2. The amended By-laws of the Association drafted and promulgated by a 1968 by-laws of the association would be observed.
Committee on December 20, 1975 is valid and binding; and
Petitioner requested the chairman of the election committee to change the notice of election by
3. The Practice of tolerating the automatic inclusion of petitioner as a following the procedure in previous elections, claiming that the notice issued for the 1990
permanent member of the Board of Directors of the Association without the elections ran "counter to the practice in previous years" and was "in violation of the by-laws (of
benefit of election is allowed under the law.1 1975)" and "unlawfully deprive[d] Grace Christian High School of its vested right [to] a
permanent seat in the board." 5
Briefly stated, the facts are as follows:
As the association denied its request, the school brought suit for mandamus in the Home
Insurance and Guaranty Corporation to compel the board of directors of the association to
Petitioner Grace Christian High School is an educational institution offering preparatory, recognize its right to a permanent seat in the board. Petitioner based its claim on the following
kindergarten and secondary courses at the Grace Village in Quezon City. Private respondent
portion of the proposed amendment which, it contended, had become part of the by-laws of the
Grace Village Association, Inc., on the other hand, is an organization of lot and/or building association as Article VI, paragraph 2, thereof:
owners, lessees and residents at Grace Village, while private respondents Alejandro G. Beltran
and Ernesto L. Go were its president and chairman of the committee on election, respectively, in
1990, when this suit was brought. The Charter and Associate Members shall elect the Directors of the
Association. The candidates receiving the first fourteen (14) highest number
of votes shall be declared and proclaimed elected until their successors are
As adopted in 1968, the by-laws of the association provided in Article IV, as follows:
elected and qualified. GRACE CHRISTIAN HIGH SCHOOL representative
is a permanent Director of the ASSOCIATION.
The annual meeting of the members of the Association shall be held on the
first Sunday of January in each calendar year at the principal office of the
It appears that the opinion of the Securities and Exchange Commission on the validity of this
Association at 2:00 P.M. where they shall elect by plurality vote and by provision was sought by the association and that in reply to the query, the SEC rendered an
secret balloting, the Board of Directors, composed of eleven (11) members opinion to the effect that the practice of allowing unelected members in the board was contrary
to serve for one (1) year until their successors are duly elected and have
to the existing by-laws of the association and to §92 of the Corporation Code (B.P. Blg. 68).
qualified.2

Private respondent association cited the SEC opinion in its answer. Additionally, the association
It appears, that on December 20, 1975, a committee of the board of directors prepared a draft of
contended that the basis of the petition for mandamus was merely "a proposed by-laws which
an amendment to the by-laws, reading as follows:3 has not yet been approved by competent authority nor registered with the SEC or HIGC." It
argued that "the by-laws which was registered with the SEC on January 16, 1969 should be the Petitioner appealed to the Court of Appeals but petitioner again lost as the appellate court on
prevailing by-laws of the association and not the proposed amended by-laws."6 February 9, 1993, affirmed the decision of the HIGC. The Court of Appeals held that there was
no valid amendment of the association's by-laws because of failure to comply with the
requirement of its existing by-laws, prescribing the affirmative vote of the majority of the
In reply, petitioner maintained that the "amended by-laws is valid and binding" and that the
members of the association at a regular or special meeting called for the adoption of amendment
association was estopped from questioning the by-laws. 7
to the by-laws. Article XIX of the by-laws provides: 10

A preliminary conference was held on March 29, 1990 but nothing substantial was agreed upon.
The members of the Association by an affirmative vote of the majority at any
The parties merely agreed that the board of directors of the association should meet on April 17,
regular or special meeting called for the purpose, may alter, amend, change
1990 and April 24, 1990 for the purpose of discussing the amendment of the by-laws and a
or adopt any new by-laws.
possible amicable settlement of the case. A meeting was held on April 17, 1990, but the parties
failed to reach an agreement. Instead, the board adopted a resolution declaring the 1975
provision null and void for lack of approval by members of the association and the 1968 by-laws This provision of the by-laws actually implements §22 of the Corporation Law (Act No. 1459)
to be effective. which provides:

On June 20, 1990, the hearing officer of the HIGC rendered a decision dismissing petitioner's §22. The owners of a majority of the subscribed capital stock, or a majority
action. The hearing officer held that the amended by-laws, upon which petitioner based its claim, of the members if there be no capital stock, may, at a regular or special
"[was] merely a proposed by-laws which, although implemented in the past, had not yet been meeting duly called for the purpose, amend or repeal any by-law or adopt
ratified by the members of the association nor approved by competent authority"; that, on the new by-laws. The owners of two-thirds of the subscribed capital stock, or
contrary, in the meeting held on April 17, 1990, the directors of the association declared "the two-thirds of the members if there be no capital stock, may delegate to the
proposed by-law dated December 20, 1975 prepared by the committee on by-laws . . . null and board of directors the power to amend or repeal any by-law or to adopt new
void" and the by-laws of December 17, 1968 as the "prevailing by-laws under which the by-laws: Provided, however, That any power delegated to the board of
association is to operate until such time that the proposed amendments to the by-laws are directors to amend or repeal any by-law or adopt new by-laws shall be
approved and ratified by a majority of the members of the association and duly filed and considered as revoked whenever a majority of the stockholders or of the
approved by the pertinent government agency." The hearing officer rejected petitioner's members of the corporation shall so vote at a regular or special
contention that it had acquired a vested right to a permanent seat in the board of directors. He meeting. And provided, further, That the Director of the Bureau of
held that past practice in election of directors could not give rise to a vested right and that Commerce and Industry shall not hereafter file an amendment to the by-
departure from such practice was justified because it deprived members of association of their laws of any bank, banking institution or building and loan association, unless
right to elect or to be voted in office, not to say that "allowing the automatic inclusion of a accompanied by certificate of the Bank Commissioner to the effect that such
member representative of petitioner as permanent director [was] contrary to law and the amendments are in accordance with law.
registered by-laws of respondent association." 8
The proposed amendment to the by-laws was never approved by the majority of the members of
The appeals board of the HIGC affirmed the decision of the hearing officer in its resolution dated the association as required by these provisions of the law and by-laws. But petitioner contends
September 13, 1990. It cited the opinion of the SEC based on §92 of the Corporation Code that the members of the committee which prepared the proposed amendment were duly
which reads: authorized to do so and that because the members of the association thereafter implemented
the provision for fifteen years, the proposed amendment for all intents and purposes should be
considered to have been ratified by them. Petitioner contends: 11
§92. Election and term of trustees. — Unless otherwise provided in the
articles of incorporation or the by-laws, the board of trustees of non-stock
corporations, which may be more than fifteen (15) in number as may be Considering, therefore, that the "agents" or committee were duly authorized
fixed in their articles of incorporation or by-laws, shall, as soon as to draft the amended by-laws and the acts done by the "agents" were in
organized, so classify themselves that the term of office of one-third (1/3) of accordance with such authority, the acts of the "agents" from the very
the number shall expire every year; and subsequent elections of trustees beginning were lawful and binding on the homeowners (the principals) per
comprising one-third (1/3) of the board of trustees shall be held annually se without need of any ratification or adoption. The more has the amended
and trustees so elected shall have a term of three (3) years. Trustees by-laws become binding on the homeowners when the homeowners
thereafter elected to fill vacancies occurring before the expiration of a followed and implemented the provisions of the amended by-laws. This is
particular term shall hold office only for the unexpired period. not merely tantamount to tacit ratification of the acts done by duly
authorized "agents" but express approval and confirmation of what the
"agents" did pursuant to the authority granted to them.
The HIGC appeals board denied claims that the school "[was] being deprived of its
right to be a member of the Board of Directors of respondent association," because
the fact was that "it may nominate as many representatives to the Association's Board Corollarily, petitioner claims that it has acquired a vested right to a permanent seat in the board.
as it may deem appropriate." It said that "what is merely being upheld is the act of the Says petitioner:
incumbent directors of the Board of correcting a long standing practice which is not
anchored upon any legal basis." 9
The right of the petitioner to an automatic membership in the board of the
Association was granted by the members of the Association themselves and
this grant has been implemented by members of the board themselves all
through the years. Outside the present membership of the board, not a among the holders of stock or, where there is no stock, from the members
single member of the Association has registered any desire to remove the of the corporation: Provided, however, That in corporations, other than
right of herein petitioner to an automatic membership in the board. If there is banks, in which the United States has or may have a vested interest,
anybody who has the right to take away such right of the petitioner, it would pursuant to the powers granted or delegated by the Trading with the Enemy
be the individual members of the Association through a referendum and not Act, as amended, and similar Acts of Congress of the United States relating
the present board some of the members of which are motivated by personal to the same subject, or by Executive Order No. 9095 of the President of the
interest. United States, as heretofore or hereafter amended, or both, the directors
need not be elected from among the holders of the stock, or, where there is
no stock from the members of the corporation. (emphasis added)
Petitioner disputes the ruling that the provision in question, giving petitioner's
representative a permanent seat in the board of the association, is contrary to law.
Petitioner claims that that is not so because there is really no provision of law §29. At the meeting for the adoption of the original by-laws, or at such
prohibiting unelected members of boards of directors of corporations. Referring to §92 subsequent meeting as may be then determined, directors shall be elected
of the present Corporation Code, petitioner says: to hold their offices for one year and until their successors are elected and
qualified. Thereafter the directors of the corporation shall be elected
annually by the stockholders if it be a stock corporation or by the members if
It is clear that the above provision of the Corporation Code only provides for
it be a nonstock corporation, and if no provision is made in the by-laws for
the manner of election of the members of the board of trustees of non-stock
the time of election the same shall be held on the first Tuesday after the first
corporations which may be more than fifteen in number and which manner
Monday in January. Unless otherwise provided in the by-laws, two weeks'
of election is even subject to what is provided in the articles of incorporation
notice of the election of directors must be given by publication in some
or by-laws of the association thus showing that the above provisions [are]
newspaper of general circulation devoted to the publication of general news
not even mandatory.
at the place where the principal office of the corporation is established or
located, and by written notice deposited in the post-office, postage pre-paid,
Even a careful perusal of the above provision of the Corporation Code addressed to each stockholder, or, if there be no stockholders, then to each
would not show that it prohibits a non-stock corporation or association from member, at his last known place of residence. If there be no newspaper
granting one of its members a permanent seat in its board of directors or published at the place where the principal office of the corporation is
trustees. If there is no such legal prohibition then it is allowable provided it is established or located, a notice of the election of directors shall be posted
so provided in the Articles of Incorporation or in the by-laws as in the instant for a period of three weeks immediately preceding the election in at least
case. three public places, in the place where the principal office of the corporation
is established or located. (Emphasis added)
xxx xxx xxx
12
The present Corporation Code (B.P. Blg. 68), which took effect on May 1, 1980, similarly
provides:
If fact, the truth is that this is allowed and is being practiced by some
corporations duly organized and existing under the laws of the Philippines.
§23. The Board of Directors or Trustees. — Unless otherwise provided in
this Code, the corporate powers of all corporations formed under this Code
One example is the Plus XII Catholic Center, Inc. Under the by-laws of this shall be exercised, all business conducted and all property of such
corporation, that whoever is the Archbishop of Manila is considered a
corporations controlled and held by the board of directors or trustees to be
member of the board of trustees without benefit of election. And not only electedfrom among the holders of stocks, or where there is no stock, from
that. He also automatically sits as the Chairman of the Board of Trustees, among the members of the corporation, who shall hold office for one (1)
again without need of any election.
year and until their successors are elected and qualified. (Emphasis added)

Another concrete example is the Cardinal Santos Memorial Hospital, Inc. It These provisions of the former and present corporation law leave no room for doubt as to their
is also provided in the by-laws of this corporation that whoever is the
meaning: the board of directors of corporations must be elected from among the stockholders or
Archbishop of Manila is considered a member of the board of trustees year members. There may be corporations in which there are unelected members in the board but it
after year without benefit of any election and he also sits automatically as is clear that in the examples cited by petitioner the unelected members sit as ex
the Chairman of the Board of Trustees.
officio members, i.e., by virtue of and for as long as they hold a particular office. But in the case
of petitioner, there is no reason at all for its representative to be given a seat in the board. Nor
It is actually §§28 and 29 of the Corporation Law — not §92 of the present law or §29 of the does petitioner claim a right to such seat by virtue of an office held. In fact it was not given such
former one — which require members of the boards of directors of corporations to be elected. seat in the beginning. It was only in 1975 that a proposed amendment to the by-laws sought to
These provisions read: give it one.

§28. Unless otherwise provided in this Act, the corporate powers of all Since the provision in question is contrary to law, the fact that for fifteen years it has not been
corporations formed under this Act shall be exercised, all business questioned or challenged but, on the contrary, appears to have been implemented by the
conducted and all property of such corporations controlled and held by a members of the association cannot forestall a later challenge to its validity. Neither can it attain
board of not less than five nor more than eleven directors to be elected from validity through acquiescence because, if it is contrary to law, it is beyond the power of the
members of the association to waive its invalidity. For that matter the members of the
association may have formally adopted the provision in question, but their action would be of no
avail because no provision of the by-laws can be adopted if it is contrary to law. 13

It is probable that, in allowing petitioner's representative to sit on the board, the members of the
association were not aware that this was contrary to law. It should be noted that they did not
actually implement the provision in question except perhaps insofar as it increased the number
of directors from 11 to 15, but certainly not the allowance of petitioner's representative as an
unelected member of the board of directors. It is more accurate to say that the members merely
tolerated petitioner's representative and tolerance cannot be considered ratification.

Nor can petitioner claim a vested right to sit in the board on the basis of "practice." Practice, no
matter how long continued, cannot give rise to any vested right if it is contrary to law. Even less
tenable is petitioner's claim that its right is "coterminus with the existence of the association." 14

Finally, petitioner questions the authority of the SEC to render an opinion on the validity of the
provision in question. It contends that jurisdiction over this case is exclusively vested in the
HIGC.

But this case was not decided by the SEC but by the HIGC. The HIGC merely cited as authority
for its ruling the opinion of the SEC chairman. The HIGC could have cited any other authority for
the view that under the law members of the board of directors of a corporation must be elected
and it would be none the worse for doing so.

WHEREFORE, the decision of the Court of Appeals is AFFIRMED.

SO ORDERED.
G.R. No. 117188 August 7, 1997 On January 26, 1993, after due notice and hearing, private respondents obtained a favorable
ruling from HIGC Hearing Officer Danilo C. Javier who disposed of HIGC Case No. RRM-5-89
as follows:
LOYOLA GRAND VILLAS HOMEOWNERS (SOUTH) ASSOCIATION, INC., petitioner,
vs.
HON. COURT OF APPEALS, HOME INSURANCE AND GUARANTY CORPORATION, WHEREFORE, judgment is hereby rendered recognizing the Loyola Grand Villas
EMDEN ENCARNACION and HORATIO AYCARDO, respondents. Homeowners Association, Inc., under Certificate of Registration No. 04-197 as the
duly registered and existing homeowners association for Loyola Grand Villas
homeowners, and declaring the Certificates of Registration of Loyola Grand Villas
ROMERO, J.:
Homeowners (North) Association, Inc. and Loyola Grand Villas Homeowners (South)
Association, Inc. as hereby revoked or cancelled; that the receivership be terminated
May the failure of a corporation to file its by-laws within one month from the date of its and the Receiver is hereby ordered to render an accounting and turn-over to Loyola
incorporation, as mandated by Section 46 of the Corporation Code, result in its automatic Grand Villas Homeowners Association, Inc., all assets and records of the Association
dissolution? now under his custody and possession.

This is the issue raised in this petition for review on certiorari of the Decision1 of the Court of The South Association appealed to the Appeals Board of the HIGC. In its Resolution of
Appeals affirming the decision of the Home Insurance and Guaranty Corporation (HIGC). This September 8, 1993, the Board 4 dismissed the appeal for lack of merit.
quasi-judicial body recognized Loyola Grand Villas Homeowners Association (LGVHA) as the
sole homeowners' association in Loyola Grand Villas, a duly registered subdivision in Quezon
Rebuffed, the South Association in turn appealed to the Court of Appeals, raising two
City and Marikina City that was owned and developed by Solid Homes, Inc. It revoked the
issues. First, whether or not LGVHAI's failure to file its by-laws within the period prescribed by
certificates of registration issued to Loyola Grand Villas homeowners (North) Association
Section 46 of the Corporation Code resulted in the automatic dissolution of LGVHAI. Second,
Incorporated (the North Association for brevity) and Loyola Grand Villas Homeowners (South)
whether or not two homeowners' associations may be authorized by the HIGC in one "sprawling
Association Incorporated (the South Association).
subdivision." However, in the Decision of August 23, 1994 being assailed here, the Court of
Appeals affirmed the Resolution of the HIGC Appeals Board.
LGVHAI was organized on February 8, 1983 as the association of homeowners and residents of
the Loyola Grand Villas. It was registered with the Home Financing Corporation, the predecessor
In resolving the first issue, the Court of Appeals held that under the Corporation Code, a private
of herein respondent HIGC, as the sole homeowners' organization in the said subdivision under
corporation commences to have corporate existence and juridical personality from the date the
Certificate of Registration No. 04-197. It was organized by the developer of the subdivision and
Securities and Exchange Commission (SEC) issues a certificate of incorporation under its official
its first president was Victorio V. Soliven, himself the owner of the developer. For unknown
seal. The requirement for the filing of by-laws under Section 46 of the Corporation Code within
reasons, however, LGVHAI did not file its corporate by-laws.
one month from official notice of the issuance of the certificate of incorporation presupposes that
it is already incorporated, although it may file its by-laws with its articles of incorporation.
Sometime in 1988, the officers of the LGVHAI tried to register its by-laws. They failed to do Elucidating on the effect of a delayed filing of by-laws, the Court of Appeals said:
so. 2 To the officers' consternation, they discovered that there were two other organizations
within the subdivision — the North Association and the South Association. According to private
We also find nothing in the provisions cited by the petitioner, i.e., Section 46 and 22,
respondents, a non-resident and Soliven himself, respectively headed these associations. They
Corporation Code, or in any other provision of the Code and other laws which provide
also discovered that these associations had five (5) registered homeowners each who were also
or at least imply that failure to file the by-laws results in an automatic dissolution of the
the incorporators, directors and officers thereof. None of the members of the LGVHAI was listed
corporation. While Section 46, in prescribing that by-laws must be adopted within the
as member of the North Association while three (3) members of LGVHAI were listed as
period prescribed therein, may be interpreted as a mandatory provision, particularly
members of the South Association.3 The North Association was registered with the HIGC on
because of the use of the word "must," its meaning cannot be stretched to support the
February 13, 1989 under Certificate of Registration No. 04-1160 covering Phases West II, East
argument that automatic dissolution results from non-compliance.
III, West III and East IV. It submitted its by-laws on December 20, 1988.

We realize that Section 46 or other provisions of the Corporation Code are silent on
In July, 1989, when Soliven inquired about the status of LGVHAI, Atty. Joaquin A. Bautista, the
the result of the failure to adopt and file the by-laws within the required period. Thus,
head of the legal department of the HIGC, informed him that LGVHAI had been automatically
Section 46 and other related provisions of the Corporation Code are to be construed
dissolved for two reasons. First, it did not submit its by-laws within the period required by the
with Section 6 (1) of P.D. 902-A. This section empowers the SEC to suspend or
Corporation Code and, second, there was non-user of corporate charter because HIGC had not
revoke certificates of registration on the grounds listed therein. Among the grounds
received any report on the association's activities. Apparently, this information resulted in the
stated is the failure to file by-laws (see also II Campos: The Corporation Code, 1990
registration of the South Association with the HIGC on July 27, 1989 covering Phases West I,
ed., pp. 124-125). Such suspension or revocation, the same section provides, should
East I and East II. It filed its by-laws on July 26, 1989.
be made upon proper notice and hearing. Although P.D. 902-A refers to the SEC, the
same principles and procedures apply to the public respondent HIGC as it exercises
These developments prompted the officers of the LGVHAI to lodge a complaint with the HIGC. its power to revoke or suspend the certificates of registration or homeowners
They questioned the revocation of LGVHAI's certificate of registration without due notice and association. (Section 2 [a], E.O. 535, series 1979, transferred the powers and
hearing and concomitantly prayed for the cancellation of the certificates of registration of the authorities of the SEC over homeowners associations to the HIGC.)
North and South Associations by reason of the earlier issuance of a certificate of registration in
favor of LGVHAI.
We also do not agree with the petitioner's interpretation that Section 46, Corporation the corporation do any lawful act as such without by-laws. Surely, no law is indeed to
Code prevails over Section 6, P.D. 902-A and that the latter is invalid because it create chaos. 7
contravenes the former. There is no basis for such interpretation considering that
these two provisions are not inconsistent with each other. They are, in fact,
Petitioner asserts that P.D. No. 902-A cannot exceed the scope and power of the Corporation
complementary to each other so that one cannot be considered as invalidating the
Code which itself does not provide sanctions for non-filing of by-laws. For the petitioner, it is "not
other.
proper to assess the true meaning of Sec. 46 . . . on an unauthorized provision on such matter
contained in the said decree."
The Court of Appeals added that, as there was no showing that the registration of LGVHAI had
been validly revoked, it continued to be the duly registered homeowners' association in the
In their comment on the petition, private respondents counter that the requirement of adoption of
Loyola Grand Villas. More importantly, the South Association did not dispute the fact that
by-laws is not mandatory. They point to P.D. No. 902-A as having resolved the issue of whether
LGVHAI had been organized and that, thereafter, it transacted business within the period
said requirement is mandatory or merely directory. Citing Chung Ka Bio v. Intermediate
prescribed by law.
Appellate Court, 8 private respondents contend that Section 6(I) of that decree provides that non-
filing of by-laws is only a ground for suspension or revocation of the certificate of registration of
On the second issue, the Court of Appeals reiterated its previous ruling 5 that the HIGC has the corporations and, therefore, it may not result in automatic dissolution of the corporation.
authority to order the holding of a referendum to determine which of two contending associations Moreover, the adoption and filing of by-laws is a condition subsequent which does not affect the
should represent the entire community, village or subdivision. corporate personality of a corporation like the LGVHAI. This is so because Section 9 of the
Corporation Code provides that the corporate existence and juridical personality of a corporation
begins from the date the SEC issues a certificate of incorporation under its official seal.
Undaunted, the South Association filed the instant petition for review on certiorari. It elevates as
Consequently, even if the by-laws have not yet been filed, a corporation may be considered a de
sole issue for resolution the first issue it had raised before the Court of Appeals, i.e., whether or
facto corporation. To emphasize the fact the LGVHAI was registered as the sole homeowners'
not the LGVHAI's failure to file its by-laws within the period prescribed by Section 46 of the
association in the Loyola Grand Villas, private respondents point out that membership in the
Corporation Code had the effect of automatically dissolving the said corporation.
LGVHAI was an "unconditional restriction in the deeds of sale signed by lot buyers."

Petitioner contends that, since Section 46 uses the word "must" with respect to the filing of by-
In its reply to private respondents' comment on the petition, petitioner reiterates its argument that
laws, noncompliance therewith would result in "self-extinction" either due to non-occurrence of a
the word " must" in Section 46 of the Corporation Code is mandatory. It adds that, before the
suspensive condition or the occurrence of a resolutory condition "under the hypothesis that (by)
ruling in Chung Ka Bio v. Intermediate Appellate Court could be applied to this case, this Court
the issuance of the certificate of registration alone the corporate personality is deemed already
must first resolve the issue of whether or not the provisions of P.D. No. 902-A prescribing the
formed." It asserts that the Corporation Code provides for a "gradation of violations of
rules and regulations to implement the Corporation Code can "rise above and change" the
requirements." Hence, Section 22 mandates that the corporation must be formally organized and
substantive provisions of the Code.
should commence transaction within two years from date of incorporation. Otherwise, the
corporation would be deemed dissolved. On the other hand, if the corporation commences
operations but becomes continuously inoperative for five years, then it may be suspended or its The pertinent provision of the Corporation Code that is the focal point of controversy in this case
corporate franchise revoked. states:

Petitioner concedes that Section 46 and the other provisions of the Corporation Code do not Sec. 46. Adoption of by-laws. — Every corporation formed under this Code, must
provide for sanctions for non-filing of the by-laws. However, it insists that no sanction need be within one (1) month after receipt of official notice of the issuance of its certificate of
provided "because the mandatory nature of the provision is so clear that there can be no doubt incorporation by the Securities and Exchange Commission, adopt a code of by-laws
about its being an essential attribute of corporate birth." To petitioner, its submission is for its government not inconsistent with this Code. For the adoption of by-laws by the
buttressed by the facts that the period for compliance is "spelled out distinctly;" that the corporation, the affirmative vote of the stockholders representing at least a majority of
certification of the SEC/HIGC must show that the by-laws are not inconsistent with the Code, the outstanding capital stock, or of at least a majority of the members, in the case of
and that a copy of the by-laws "has to be attached to the articles of incorporation." Moreover, no non-stock corporations, shall be necessary. The by-laws shall be signed by the
sanction is provided for because "in the first place, no corporate identity has been completed." stockholders or members voting for them and shall be kept in the principal office of the
Petitioner asserts that "non-provision for remedy or sanction is itself the tacit proclamation that corporation, subject to the stockholders or members voting for them and shall be kept
non-compliance is fatal and no corporate existence had yet evolved," and therefore, there was in the principal office of the corporation, subject to inspection of the stockholders or
"no need to proclaim its demise." 6 In a bid to convince the Court of its arguments, petitioner members during office hours; and a copy thereof, shall be filed with the Securities and
stresses that: Exchange Commission which shall be attached to the original articles of incorporation.

. . . the word MUST is used in Sec. 46 in its universal literal meaning and corollary Notwithstanding the provisions of the preceding paragraph, by-laws may be adopted
human implication — its compulsion is integrated in its very essence — MUST is and filed prior to incorporation; in such case, such by-laws shall be approved and
always enforceable by the inevitable consequence — that is, "OR ELSE". The use of signed by all the incorporators and submitted to the Securities and Exchange
the word MUST in Sec. 46 is no exception — it means file the by-laws within one Commission, together with the articles of incorporation.
month after notice of issuance of certificate of registration OR ELSE. The OR ELSE,
though not specified, is inextricably a part of MUST . Do this or if you do not you are
In all cases, by-laws shall be effective only upon the issuance by the Securities and
"Kaput". The importance of the by-laws to corporate existence compels such meaning
Exchange Commission of a certification that the by-laws are not inconsistent with this
for as decreed the by-laws is "the government" of the corporation. Indeed, how can
Code.
The Securities and Exchange Commission shall not accept for filing the by-laws or any This exchange of views demonstrates clearly that automatic corporate dissolution for failure to
amendment thereto of any bank, banking institution, building and loan association, file the by-laws on time was never the intention of the legislature. Moreover, even without
trust company, insurance company, public utility, educational institution or other resorting to the records of deliberations of the Batasang Pambansa, the law itself provides the
special corporations governed by special laws, unless accompanied by a certificate of answer to the issue propounded by petitioner.
the appropriate government agency to the effect that such by-laws or amendments are
in accordance with law.
Taken as a whole and under the principle that the best interpreter of a statute is the statute itself
(optima statuli interpretatix est ipsum statutum), 14 Section 46 aforequoted reveals the legislative
As correctly postulated by the petitioner, interpretation of this provision of law begins with the intent to attach a directory, and not mandatory, meaning for the word "must" in the first sentence
determination of the meaning and import of the word "must" in this section Ordinarily, the word thereof. Note should be taken of the second paragraph of the law which allows the filing of the
"must" connotes an imperative act or operates to impose a duty which may be enforced. 9 It is by-laws even prior to incorporation. This provision in the same section of the Code rules out
synonymous with "ought" which connotes compulsion or mandatoriness. 10 However, the word mandatory compliance with the requirement of filing the by-laws "within one (1) month after
"must" in a statute, like "shall," is not always imperative. It may be consistent with an exercise of receipt of official notice of the issuance of its certificate of incorporation by the Securities and
discretion. In this jurisdiction, the tendency has been to interpret "shall" as the context or a Exchange Commission." It necessarily follows that failure to file the by-laws within that period
reasonable construction of the statute in which it is used demands or requires. 11 This is equally does not imply the "demise" of the corporation. By-laws may be necessary for the "government"
true as regards the word "must." Thus, if the languages of a statute considered as a whole and of the corporation but these are subordinate to the articles of incorporation as well as to the
with due regard to its nature and object reveals that the legislature intended to use the words Corporation Code and related statutes.15 There are in fact cases where by-laws are unnecessary
"shall" and "must" to be directory, they should be given that meaning.12 to corporate existence or to the valid exercise of corporate powers, thus:

In this respect, the following portions of the deliberations of the Batasang Pambansa No. 68 are In the absence of charter or statutory provisions to the contrary, by-laws are not
illuminating: necessary either to the existence of a corporation or to the valid exercise of the
powers conferred upon it, certainly in all cases where the charter sufficiently provides
for the government of the body; and even where the governing statute in express
MR. FUENTEBELLA. Thank you, Mr. Speaker.
terms confers upon the corporation the power to adopt by-laws, the failure to exercise
the power will be ascribed to mere nonaction which will not render void any acts of the
On page 34, referring to the adoption of by-laws, are we made to understand here, Mr. corporation which would otherwise be valid. 16 (Emphasis supplied.)
Speaker, that by-laws must immediately be filed within one month after the issuance?
In other words, would this be mandatory or directory in character?
As Fletcher aptly puts it:

MR. MENDOZA. This is mandatory.


It has been said that the by-laws of a corporation are the rule of its life, and that until
by-laws have been adopted the corporation may not be able to act for the purposes of
MR. FUENTEBELLA. It being mandatory, Mr. Speaker, what would be the effect of the its creation, and that the first and most important duty of the members is to adopt
failure of the corporation to file these by-laws within one month? them. This would seem to follow as a matter of principle from the office and functions
of by-laws. Viewed in this light, the adoption of by-laws is a matter of practical, if not
one of legal, necessity. Moreover, the peculiar circumstances attending the formation
MR. MENDOZA. There is a provision in the latter part of the Code which identifies and of a corporation may impose the obligation to adopt certain by-laws, as in the case of
describes the consequences of violations of any provision of this Code. One such a close corporation organized for specific purposes. And the statute or general laws
consequences is the dissolution of the corporation for its inability, or perhaps, incurring
from which the corporation derives its corporate existence may expressly require it to
certain penalties. make and adopt by-laws and specify to some extent what they shall contain and the
manner of their adoption. The mere fact, however, of the existence of power in the
MR. FUENTEBELLA. But it will not automatically amount to a dissolution of the corporation to adopt by-laws does not ordinarily and of necessity make the exercise of
corporation by merely failing to file the by-laws within one month. Supposing the such power essential to its corporate life, or to the validity of any of its acts. 17
corporation was late, say, five days, what would be the mandatory penalty?
Although the Corporation Code requires the filing of by-laws, it does not expressly provide for
MR. MENDOZA. I do not think it will necessarily result in the automatic or ipso the consequences of the non-filing of the same within the period provided for in Section 46.
facto dissolution of the corporation. Perhaps, as in the case, as you suggested, in the However, such omission has been rectified by Presidential Decree No. 902-A, the pertinent
case of El Hogar Filipino where a quo warrantoaction is brought, one takes into provisions on the jurisdiction of the SEC of which state:
account the gravity of the violation committed. If the by-laws were late — the filing of
the by-laws were late by, perhaps, a day or two, I would suppose that might be a Sec. 6. In order to effectively exercise such jurisdiction, the Commission shall possess
tolerable delay, but if they are delayed over a period of months — as is happening
the following powers:
now — because of the absence of a clear requirement that by-laws must be
completed within a specified period of time, the corporation must suffer certain
consequences. 13 xxx xxx xxx
(1) To suspend, or revoke, after proper notice and hearing, the franchise or certificate Section 19 of the Corporation Law, part of which is now Section 22 of the Corporation
of registration of corporations, partnerships or associations, upon any of the grounds Code, provided that the powers of the corporation would cease if it did not formally
provided by law, including the following: organize and commence the transaction of its business or the continuation of its works
within two years from date of its incorporation. Section 20, which has been reproduced
with some modifications in Section 46 of the Corporation Code, expressly declared
xxx xxx xxx
that "every corporation formed under this Act, must within one month after the filing of
the articles of incorporation with the Securities and Exchange Commission, adopt a
5. Failure to file by-laws within the required period; code of by-laws." Whether this provision should be given mandatory or only directory
effect remained a controversial question until it became academic with the adoption of
PD 902-A. Under this decree, it is now clear that the failure to file by-laws within the
xxx xxx xxx
required period is only a ground for suspension or revocation of the certificate of
registration of corporations.
In the exercise of the foregoing authority and jurisdiction of the Commission or by a
Commissioner or by such other bodies, boards, committees and/or any officer as may
Non-filing of the by-laws will not result in automatic dissolution of the corporation.
be created or designated by the Commission for the purpose. The decision, ruling or Under Section 6(I) of PD 902-A, the SEC is empowered to "suspend or revoke, after
order of any such Commissioner, bodies, boards, committees and/or officer may be proper notice and hearing, the franchise or certificate of registration of a corporation"
appealed to the Commission sitting en banc within thirty (30) days after receipt by the
on the ground inter alia of "failure to file by-laws within the required period." It is clear
appellant of notice of such decision, ruling or order. The Commission shall promulgate from this provision that there must first of all be a hearing to determine the existence
rules of procedures to govern the proceedings, hearings and appeals of cases falling of the ground, and secondly, assuming such finding, the penalty is not necessarily
with its jurisdiction.
revocation but may be only suspension of the charter. In fact, under the rules and
regulations of the SEC, failure to file the by-laws on time may be penalized merely
The aggrieved party may appeal the order, decision or ruling of the Commission with the imposition of an administrative fine without affecting the corporate existence
sitting en banc to the Supreme Court by petition for review in accordance with the of the erring firm.
pertinent provisions of the Rules of Court.
It should be stressed in this connection that substantial compliance with conditions
Even under the foregoing express grant of power and authority, there can be no automatic subsequent will suffice to perfect corporate personality. Organization and
corporate dissolutionsimply because the incorporators failed to abide by the required filing of by- commencement of transaction of corporate business are but conditions subsequent
laws embodied in Section 46 of the Corporation Code. There is no outright "demise" of corporate and not prerequisites for acquisition of corporate personality. The adoption and filing
existence. Proper notice and hearing are cardinal components of due process in any democratic of by-laws is also a condition subsequent. Under Section 19 of the Corporation Code,
institution, agency or society. In other words, the incorporators must be given the chance to a Corporation commences its corporate existence and juridical personality and is
explain their neglect or omission and remedy the same. deemed incorporated from the date the Securities and Exchange Commission issues
certificate of incorporation under its official seal. This may be done even before the
filing of the by-laws, which under Section 46 of the Corporation Code, must be
That the failure to file by-laws is not provided for by the Corporation Code but in another law is of adopted "within one month after receipt of official notice of the issuance of its
no moment. P.D. No. 902-A, which took effect immediately after its promulgation on March 11, certificate of incorporation." 21
1976, is very much apposite to the Code. Accordingly, the provisions abovequoted supply the
law governing the situation in the case at bar, inasmuch as the Corporation Code and P.D. No.
902-A are statutes in pari materia. Interpretare et concordare legibus est optimus interpretandi. That the corporation involved herein is under the supervision of the HIGC does not alter the
Every statute must be so construed and harmonized with other statutes as to form a uniform result of this case. The HIGC has taken over the specialized functions of the former Home
system of jurisprudence. 18 Financing Corporation by virtue of Executive Order No. 90 dated December 17, 1989. 22 With
respect to homeowners associations, the HIGC shall "exercise all the powers, authorities and
responsibilities that are vested on the Securities and Exchange Commission . . . , the provision
As the "rules and regulations or private laws enacted by the corporation to regulate, govern and of Act 1459, as amended by P.D. 902-A, to the contrary notwithstanding." 23
control its own actions, affairs and concerns and its stockholders or members and directors and
officers with relation thereto and among themselves in their relation to it," 19 by-laws are
indispensable to corporations in this jurisdiction. These may not be essential to corporate birth WHEREFORE, the instant petition for review on certiorari is hereby DENIED and the questioned
but certainly, these are required by law for an orderly governance and management of Decision of the Court of Appeals AFFIRMED. This Decision is immediately executory. Costs
corporations. Nonetheless, failure to file them within the period required by law by no means tolls against petitioner.
the automatic dissolution of a corporation.
SO ORDERED.
In this regard, private respondents are correct in relying on the pronouncements of this Court
in Chung Ka Bio v.Intermediate Appellate Court, 20 as follows:

. . . . Moreover, failure to file the by-laws does not automatically operate to dissolve a
corporation but is now considered only a ground for such dissolution.
G.R. No. 116631 October 28, 1998 But petitioner, however, failed to execute a document recognizing private respondent's beneficial
ownership over said share.
MARSH THOMSON, petitioner,
vs. Following AmCham's policy and practice, there was a yearly renewal of employment contract
COURT OF APPEALS and THE AMERICAN CHAMPER OF COMMERCE OF THE between the petitioner and private respondent. Separate letters of employment advice dated
PHILIPPINES, INC, respondents. October 1, 19864, as well March 4, 19885 and January 7, 19896, mentioned the MPC share. But
petitioner never acknowledged that private respondent is the beneficial owner of the share as
requested in follow-up requests, particularly one dated March 4, 1988 as follows:
QUISUMBING, J.:

Dear Marsh:
This is a petition for review on certiorari seeking the reversal of the Decision 1 of the Court of
Appeals on May 19, 1994, disposing as follows:
xxx xxx xxx
WHEREFORE, THE DECISION APPEALED FROM IS HEREBY SET
ASIDE. ANOTHER JUDGMENT IS ENTERED ORDERING DEFENDANT- All other provisions of your compensation/benefit package will remain the
APPELLEE MARSH THOMSON TO TRANSFER THE SAID MPC [Manila same and are summarized as follows:
Polo Club] SHARE TO THE NOMINEE OF THE APPELLANT.
xxx xxx xxx
The facts of the case are:
9) The Manila Polo Club membership provided by the
Petitioner Marsh Thomson (Thomson) was the Executive Vice-President and, later on, the Chamber for you and your family will continue on the
Management Consultant of private respondent, the American Chamber of Commerce of the same basis, to wit: all dues and other charges relating
Philippines, Inc. (AmCham) for over ten years, 1979-1989. to such membership shall be for your personal account
and, if you have not already done so, you will execute
such documents as are necessary to acknowledge that
While petitioner was still working with private respondent, his superior, A. Lewis Burridge, retired
the Chamber is the beneficial owner of your
as AmCham's President. Before Burridge decided to return to his home country, he wanted to
membership in the Club. 7
transfer his proprietary share in the Manila Polo Club (MPC) to petitioner. However, through the
intercession of Burridge, private respondent paid for the share but had it listed in petitioner's
name. This was made clear in an employment advice dated January 13, 1986, wherein When petitioner's contract of employment was up for renewal in 1989, he notified private
petitioner was informed by private respondent as follows: respondent that he would no longer be available as Executive Vice President after September
30, 1989. Still, the private respondent asked the petitioner to stay on for another six (6) months.
Petitioner indicated his acceptance of the consultancy arrangement with a counter-proposal in
xxx xxx xxx
his letter dated October 8, 1989, among others as follows:

11. If you so desire, the Chamber is willing to acquire


11.) Retention of the Polo Club share, subject to my
for your use a membership in the Manila Polo Club. The
reimbursing the purchase price to the Chamber, or one
timing of such acquisition shall be subject to the
hundred ten thousand pesos (P110,000.00).8
discretion of the Board based on the Chamber's
financial position. All dues and other charges relating to
such membership shall be for your personal account. If Private respondent rejected petitioner's counter-proposal.
the membership is acquired in your name, you would
execute such documents as necessary to acknowledge
Pending the negotiation for the consultancy arrangement, private respondent executed on
beneficial ownership thereof by the Chamber. 2
September 29, 1989 a Release and Quitclaim,9 stating that "AMCHAM, its directors, officers and
assigns, employees and/or representatives do hereby release, waive, abandon and discharge J.
xxx xxx xxx MARSH THOMSON from any and all existing claims that the AMCHAM, its directors, officers
and assigns, employees and/or representatives may have against J. MARSH
THOMSON." 10 The quitclaim, expressed in general terms, did not mention specifically the MPC
On April 25, 1986, Burridge transferred said proprietary share to petitioner, as confirmed in a
share.
letter3 of notification to the Manila Polo Club.

On April 5, 1990, private respondent, through counsel sent a letter to the petitioner demanding
Upon his admission as a new member of the MPC, petitioner paid the transfer fee of P40,000.00
the return and delivery of the MPC share which "it (AmCham) owns and placed in your
from his own funds; but private respondent subsequently reimbursed this amount. On November
(Thomson's) name." 11
19, 1986, MPC issued Proprietary Membership Certificate Number 3398 in favor of petitioner.
Failing to get a favorable response, private respondent filed on May 15, 1990, a complaint herein petitioner to transfer the MPC share to the nominee of private respondent, reasoning
against petitioner praying, inter alia, that the Makati Regional Trial Court render judgment thus:
ordering Thomson "to return the Manila Polo Club share to the plaintiff and transfer said share to
the nominee of plaintiff." 12
xxx xxx xxx
13
On February 28, 1992, the trial court promulgated its decision, thus:
The significant fact in the instant case is that the appellant [herein private
respondent] purchased the MPC share for the use of the appellee [herein
The foregoing considered judgment is rendered as follows: petitioner] and the latter expressly conformed thereto as shown in Exhibits
A-1, B, B-1, C, C-1, D, D-1. By such express conformity of the appellee, the
former was bound to recognize the appellant as the owner of the said share
1) The ownership of the contested Manila Polo Club
for a contract has the force of law between the parties. (Alim vs. CA, 200
share is adjudicated in favor of defendant Marsh
SCRA 450; Sasuhura Company, Inc., Ltd. vs. IAC, 205 SCRA 632) Aside
Thomson; and;
from the foregoing, the appellee conceded the true ownership of the said
share to the appellant when (1) he offered to buy the MPC share from the
2) Defendant shall pay plaintiff the sum of P300,000.00 appellant (Exhs. E and E-1) upon the termination of his employment; (2) he
obliged himself to return the MPC share after his six month consultancy
contract had elapsed, unless its return was earlier requested in writting
Because both parties thru their respective faults have somehow contributed (Exh. I); and (3) on cross-examination, he admitted that the proprietary
to the birth of this case, each shall bear the incidental expenses incurred. 14 share listed as one of the assets of the appellant corporation in its 1988
Corporate Income Tax Return, which he signed as the latter's Executive
In said decision, the trial court awarded the MPC share to defendant (petitioner now) on the Vice President (prior to its filing), refers to the Manila Polo Club Share (tsn.,
ground that the Articles of Incorporation and By-laws of Manila Polo Club prohibit artificial pp. 19-20, August 30, 1991). . . . 17
persons, such as corporations, to be club members, ratiocinating in this manner:
On 16 June 1994, petitioner filed a motion for reconsideration 18 of said decision. By
An assessment of the evidence adduced by both parties at the trial will resolution 19promulgated on August 4, 1994, the Court of Appeals denied the motion for
show clearly that it was the intention of the parties that a membership to reconsideration.
Manila Polo Club was to be secured by plaintiff [herein private respondent]
for defendant's [herein petitioner] use. The latter was to execute the In this petition for review, petitioner alleges the following errors of public respondent as grounds
necessary documents to acknowledge ownership of the Polo membership in
for our review:
favor of plaintiff. (Exh. C par 9) However, when the parties parted ways in
disagreement and with some degree of bitterness, the defendant had
second thoughts and decided to keep the membership for himself. This is I. The respondent Court of Appeals erred in setting
evident from the exhibits (E & G) where defendant asked that he retained aside the Decision dated 28 February 1992 of the
the Polo Club membership upon reimbursement of its purchase price; and Regional Trial Court, NCJR, Branch 65, Makati, Metro
where he showed his "profound disappointment, both at the previous Manila, in its Civil Case No. 90-1286, and in not
Board's unfair action, and at what I consider to be harsh terms, after my confirming petitioner's ownership over the MPC
long years of dedication to the Chamber's interest." membership share.

xxx xxx xxx II. The respondent Court of Appeals erred in ruling that
"the Quitclaim executed by AmCham in favor of
petitioner of September 29, 1989 was superseded by
Notwithstanding all these evidence in favor of plaintiff, however, defendant the contractual agreement entered into by the parties
may not be declared the owner of the contested membership be compelled on October 13, 1989 wherein again the appellee
to execute documents transferring the Polo Membership to plaintiff or the
acknowledged that the appellant owned the MPC
latter's nominee for the reason that this is prohibited by Polo Club's Articles share, there being absolutely no evidence to support
& By-Laws. . . . such a conclusion and/or such inference is manifestly
mistaken.
It is for the foregoing reasons that the Court rules that the ownership of the
questioned Polo Club membership be retained by defendant. 15 . . . . III. The respondent Court of Appeals erred in rendering
judgment ordering petitioner to transfer the contested
Not satisfied with the trial court's decision, private respondent appealed to the Court of Appeals. MPC share to a nominee of respondent AmCham
notwithstanding that: (a) AmCham has no standing in
the Manila Polo Club (MPG), and being an artificial
On May 19, 1994, the Court of Appeals (Former Special Sixth Division) promulgated its person, it is precluded under MPC's Articles of
decision 16 in said CA-G.R. CV No. 38417, reversing the, trial court's judgment and ordered Incorporation and governing rules and regulations from
owning a proprietary share or from becoming a member employment advice, dated March 4, 1988, private respondent once again, asked the petitioner to
thereof: and (b) even under AmCham's Articles of execute proof to recognize the trust agreement in writing:
Incorporation, the purposes for which it is dedicated,
becoming a stockholder or shareholder in other
The Manila Polo membership provided by the Chamber for you and your
corporation is not one of the express implied powers
family will continue on the same basis, to wit: all dues and other charges
fixed in AmCham's said corporate franchise. 20
relating to such membership shall be for your personal account and, if you
have not already done so, you will execute such documents as are
As posited above, these assigned errors show the disputed matters herein are mainly factual. As necessary to acknowledge that the Chamber is the beneficial owner of your
such they are best left to the trial and appellate courts' disposition. And this Court could have membership in the Club. 24
dismissed the petition outright, were it not for the opposite results reached by the courts below.
Moreover, for the enhanced appreciation of the jural relationship between the parties involving
Petitioner voluntarily affixed his signature to conform with the employment advice, including his
trust, this Court has given due course to the petition, which we now decide.
obligation stated therein — for him to execute the necessary document to recognize his
employer as the beneficial owner of the MPC share. Now, we cannot hear him claiming
After carefully considering the pleadings on record, we find there are two main issues to be otherwise, in derogation of said undertaking, without legal and equitable justification.
resolved: (1) Did respondent court err in holding that private respondent is the beneficial owner
of the disputed share? (2) Did the respondent court err in ordering petitioner to transfer said
For private respondent's intention to hold on to its beneficial ownership is not only presumed; it
share to private respondent's nominees?
was expressed in writing at the very outset. Although the share was placed in the name of
petitioner, his title is limited to the usufruct, that is, to enjoy the facilities and privileges of such
Petitioner claims ownership of the MPC share, asserting that he merely incurred a debt to membership in the club appertaining to the share. Such arrangement reflects a trust relationship
respondent when the latter advanced the funds for the purchase of the share. On the other governed by law and equity.
hand, private respondent asserts beneficial ownership whereby petitioner only holds the share in
his name, but the beneficial title belongs to private respondent. To resolve the first issue, we
While private respondent paid the purchase price for the share, petitioner was given legal title
must clearly distinguish a debt from a trust.
thereto. Thus, a resulting trust is presumed as a matter of law. The burden then shifted to the
transferee to show otherwise, that it was just a loan. Such resulting trust could have been
The beneficiary of a trust has beneficial interest in the trust property, while a creditor has merely rebutted by proof of a contrary intention by a showing that, in fact, no trust was intended.
a personal claim against the debtor. In trust, there is a fiduciary relation between a trustee and a Petitioner could have negated the trust agreement by contrary, consistent and convincing
beneficiary, but there is no such relation between a debtor and creditor. While a debt implies evidence on rebuttal. However, on the witness stand, petitioner failed to do so persuasively.
merely an obligation to pay a certain sum of money, a trust refers to a duty to deal with a specific
property for the benefit of another. If a creditor-debtor relationship exists, but not a fiduciary
On cross-examination, the petitioner testified as follows:
relationship between the parties, there is no express trust. However, it is understood that when
the purported trustee of funds is entitled to use them as his or her own (and commingle them
with his or her own money), a debtor-creditor relationship exists, not a trust. 21 ATTY. AQUINO (continuing)

In the present case, as the Executive Vice-President of AmCham, petitioner occupied a fiduciary Q. Okay, let me go to the cash advance that you
position in the business of AmCham. AmCham released the funds to acquire a share in the Club mentioned Mr. Witness, is there any document proving
for the use of petitioner but obliged him to "execute such document as necessary to that you claimed cash advance signed by an officer of
acknowledge beneficial ownership thereof by the Chamber". 22 A trust relationship is, therefore, the Chamber?
manifestly indicated.
A. I believe the best evidence is the check.
Moreover, petitioner failed to present evidence to support his allegation of being merely a debtor
when the private respondent paid the purchase price of the MPC share. Applicable here is the
rule that a trust arises in favor of one who pays the purchase money of property in the name of Q. Is there any document?
another, because of the presumption that he who pays for a thing intends a beneficial interest
therein for himself. 23 COURT

Although petitioner initiated the acquisition of the share, evidence on record shows that private Other than the Check?
respondent acquired said share with its funds. Petitioner did not pay for said share, although he
later wanted to, but according to his own terms, particularly the price thereof.
MR. THOMSON

Private respondent's evident purpose in acquiring the share was to provide additional incentive
and perks to its chosen executive, the petitioner himself. Such intention was repeated in the Nothing more.
yearly employment advice prepared by AmCham for petitioner's concurrence. In the cited
ATTY. AQUINO
Is there any application filed in the Chamber to avail of document does not lead to the purported conclusion that be intended to renounce private
this cash advance? respondent's beneficial title over its share in the Manila Polo Club. We, therefore, find no
reversible error in the respondent Court's holding that private respondent, AmCham, is the
beneficial owner of the share in dispute.
A. Verbal only.

Turning now to the second issue, the petitioner contends that the Articles of Incorporation and
Q. Nothing written, and can you tell to this Honorable
By-laws of Manila Polo Club prohibit corporate membership. However, private respondent does
Court what are the stipulations or conditions, or terms
not insist nor intend to transfer the club membership in its name but rather to its designated
of this transaction of securing this cash advance or
nominee. For as properly ruled by the Court of Appeals:
loan?

The matter prayed for does not involve the transfer of said share to the
xxx xxx xxx
appellant, an artificial person. The transfer sought is to the appellant's
nominee. Even if the MPC By-Laws and Articles prohibit corporate
COURT membership, there would be no violation of said prohibition for the
appellant's nominee to whom the said share is sought to be transferred
would certainly be a natural person. . . .
How are you going to repay the cash advance?

As to whether or not the transfer of said share the appellant's nominee


MR. THOMSON would be disapproved by the MPC, is a matter that should be raised at the
proper time, which is only if such transfer is disapproved by the MPC. 32
The cash advance, we never stipulate when I have to
repay it, but I presume that I would, when able to repay The Manila Polo Club does not necessarily prohibit the transfer of proprietary shares by its
the money. 25
members. The Club only restricts membership to deserving applicants in accordance with its
rules, when the amended Articles of Incorporation states that: "No transfer shall be valid except
In deciding whether the property was wrongfully appropriated or retained and what the intent of between the parties, and shall be registered in the Membership Book unless made in
the parties was at the time of the conveyance, the court must rely upon its impression of the accordance with these Articles and the By-Laws". 33Thus, as between parties herein, there is no
credibility of the witnesses. 26 Intent is a question of fact, the determination of which is not question that a transfer is feasible. Moreover, authority granted to a corporation to regulate the
reviewable unless the conclusion drawn by the trier is one which could not reasonably be transfer of its stock does not empower it to restrict the right of a stockholder to transfer his
drawn. 27 Petitioner's denial is not adequate to rebut the trust. Time and again, we have ruled shares, but merely authorizes the adoption of regulations as to the formalities and procedure to
that denials, if unsubstantiated by clear and convincing evidence, are deemed negative and self- be followed in effecting transfer. 34
serving evidence, unworthy of credence. 28
In this case, the petitioner was the nominee of the private respondent to hold the share and
The trust between the parties having been established, petitioner advanced an alternative enjoy the privileges of the club. But upon the expiration of petitioner's employment as officer and
defense that the private respondent waived the beneficial ownership of MPC share by issuing consultant of AmCham, the incentives that go with the position, including use of the MPC share,
the Release and Quitclaim in his favor. also ceased to exist. It now behooves petitioner to surrender said share to private respondent's
next nominee, another natural person. Obviously this arrangement of trust and confidence
cannot be defeated by the petitioner's citation of the MPC rules to shield his untenable position,
This argument is less than persuasive. The quitclaim executed by private respondent does not without doing violence to basic tenets of justice and fair dealing.
clearly show the intent to include therein the ownership over the MPC share. Private respondent
even asserts that at the time the Release and Quitclaim was executed on September 29, 1989,
the ownership of the MPC share was not controversial nor contested. Settled is the rule that a However, we still have to ascertain whether the rights of herein parties to the trust still subsist. It
waiver to be valid and effective must, in the first place, be couched in clear and unequivocal has been held that so long as there has been no denial or repudiation of the trust, the
terms which leave no doubt as to the intention of a party to give up a right or benefit which possession of the trustee of an express and continuing trust is presumed to be that of the
legally pertains to him. 29 A waiver may not be attributed to a person when the terms thereof do beneficiary, and the statute of limitations does not run between them. 35 With regard to a
not explicitly and clearly evidence an intent to abandon a right vested in such person. 30 If we constructive or a resulting trust, the statute of limitations does not begin to run until the trustee
apply the standard rule that waiver must be cast in clear and unequivocal terms, then clearly the clearly repudiates or disavows the trust and such disavowal is brought home to the other party,
general terms of the cited release and quitclaim indicates merely a clearance from general "cestui que trust". 36 The statute of limitations runs generally from the time when the act was
accountability, not specifically a waiver of AmCham's beneficial ownership of the disputed done by which the party became chargeable as a trustee by operation of law or when the
shares. beneficiary knew that he had a cause of action, 37 in the absence of fraud or concealment.

Additionally, the intention to waive a right or advantage must be shown clearly and convincingly, Noteworthy in the instant case, there was no declared or explicit repudiation of the trust existing
and when the only proof of intention rests in what a party does, his act should be so manifestly between the parties. Such repudiation could only be inferred as evident when the petitioner
consistent with, and indicative of, an intent to voluntarily relinquish the particular right or showed his intent to appropriate the MPC share for himself. Specifically, this happened when he
advantage that no other reasonable explanation of his conduct is possible. 31 Considering the requested to retain the MPC share upon his reimbursing the purchase price of P110,000, a
terms of the quitclaim executed by the President of private respondent, the tenor of the request denied promptly by private respondent. Eventually, petitioner refused to surrender the
share despite the written demand of private respondent. This act could then be construed as
repudiation of the trust. The statute of limitation could start to set in at this point in time. But
private respondent took immediate positive action. Thus, on May 15, 1990, private respondent
filed an action to recover the MPC share. Between the time of implicit repudiation of the trust on
October 9, 1989, as evidenced by petitioner's letter of said date, and private respondent's
institution of the action to recover the MPC share on May 15, 1990, only about seven months
bad lapsed. Our laws on the matter provide that actions to recover movables shall prescribe
eight years from the time the possession thereof is lot, 38 unless the possessor has acquired the
ownership by prescription for a less period of four years if in good faith. 39 Since the private
respondent filed the necessary action on time and the defense of good faith is not available to
the petitioner, there is no basis for any purported claim of prescription, after repudiation of the
trust, which will entitle petitioner to ownership of the disputed share. As correctly held by the
respondent court, petitioner has the obligation to transfer now said share to the nominee of
private respondent.

WHEREFORE, the Petition for Review on Certiorari is DENIED. The Decision of the Court of
Appeals of May 19, 1994, is AFFIRMED.

COSTS against petitioner.

SO ORDERED.
G.R. No. 121791 December 23, 1998 association Jaime Y. Ladao in a letter of 3 July 1987. His reply to the
aforesaid letter, claiming his employment regular, and viz a viz, referring to
submit his medical certificate, notwithstanding, to our mind, merely
ENRIQUE SALAFRANCA, petitioner,
underscored the need to define his position as, in fact, the Association's
vs.
Rules and Regulations were amended if but to put to rest the tenural (sic)
PHILAMLIFE (PAMPLONA) VILLAGE HOMEOWNERS ASSOCIATION, INC., BONIFACIO
limit of the office of the Administrative Officer in accordance with its earlier
DAZO and THE SECOND DIVISION, NATIONAL LABOR RELATIONS COMMISSION
intention, that it is co-terminus with that of the members of the Board of
(NLRC), respondents.
Directors.

ROMERO, J.:
WHEREFORE, the decision appealed from is hereby set aside.
Respondents are hereby ordered to pay herein appellee one half (1/2)
Petitioner Enrique Salafranca started working with the private respondent Philamlife Village month pay for every year of service representing his retirement pay.
Homeowners Association on May 1, 1981 as administrative officer for a period of six months.
From this date until December 31, 1983, petitioner was reappointed to his position three more
In view of the sudden turn of events, petitioner has elevated the case to this Court assigning the
times.1 As administrative officer, petitioner was generally responsible for the management of the
following errors:8
village's day to day activities.2 After petitioner's term of employment expired on December 31,
1983, he still continued to work in the same capacity, albeit, without the benefit of a renewed
contract. 1. The NLRC gravely abused its discretion when it ruled that the
employment of the Petitioner is not purely based on considerations of
Employer-Employee relationship.
Sometime in 1987, private respondent decided to amend its by-laws. Included therein was a
provision regarding officers, specifically, the position of administrative officer under which said
officer shall hold office at the pleasure of the Board of Directors. In view of this development, 2. Petitioner was illegally dismissed by private respondents.
private respondent, on July 3, 1987, informed the petitioner that his term of office shall be
coterminus with the Board of Directors which appointed him to his position. Furthermore, until he
As to the first assigned error by the petitioner, we need not dwell on this at length. We agree with
submits a medical certificate showing his state of health, his employment shall be on a month-to-
the Solicitor General's observation that an employer-employee relationship exists between the
month basis.3 Oddly, notwithstanding the failure of herein petitioner to submit his medical
petitioner and the private respondent. 9
certificate, he continued working until his termination in December 1992. 4Claiming that his
services had been unlawfully and unceremoniously dispensed with, petitioner filed a complaint
for illegal dismissal with money claims and for damages. 5 xxx xxx xxx

After the submission by the parties of their respective position papers and other pleadings, the The first element is present in this case. Petitioner was hired as
Labor Arbiter rendered a decision6 ordering private respondent to pay the petitioner the amount Administrative Officer by respondents. In fact, he was extended successive
of P257,833.33 representing his backwages, separation pay and 13th month pay. In justifying appointments by respondents.
the award, the Labor Arbiter elucidated:
The second element is also present since it is not denied that respondent
Respondents' contention that complainant's term of employment was co- PVHA paid petitioner a fixed salary for his services.
terminus with the term of Office of the Board of Directors, is wanting in
merit. Records show that complainant had been hired in 1981 while the
Amendment of the respondents' By-Laws making the position of an As to the third element, it can be seen from the Records that respondents
Administrative Officer co-terminus with the term of the Board of Directors had the power of dismissal over petitioner. In their letter dated December 7,
1992, respondents informed petitioner that they had decided to discontinue
was made in 1987. Evidently, the said Amendment would not be applicable
to the case of complainant who had become a regular employee long time his services. In their Position Paper submitted to the Labor Arbiter,
before the Amendment took place. Moreover, the Amendment should be respondents stated that petitioner "was dismissed for cause." (p. 17,
Record).
applied prospectively and not retroactively.

On appeal by the private respondent, the NLRC reversed the decision of the Labor Arbiter and With respect to the fourth and most important element, respondents
controlled the work of petitioner not only with respect to the ends to be
rendered a new one7 reducing petitioner's monetary award to only one-half (1/2) month pay for
every year of service representing his retirement pay. In other words, the NLRC viewed the achieved but also the means used in reaching such ends.
dismissal of the petitioner as a valid act by the private respondent.
Relative to the second assigned error of the petitioner, both the Solicitor General and the private
The fact that he continued to perform the function of the office of respondent take the stance that petitioner was not illegally dismissed. 10 On this aspect, we
administrative officer without extension or re-appointment thereafter, to our disagree with their contentions.
mind, did not in any way make his employment permanent as in fact, he
was even reminded of the nature of his position by then president of the
On the outset, there is no dispute that petitioner had already attained the status of a regular The essence of due process is to afford the party an opportunity to be heard and defend himself,
employee, as evidenced by his eleven years of service with the private respondent. Accordingly, to cleanse his name and reputation from any taint. It includes the twin requirements of notice
petitioner enjoys the right to security of tenure 11 and his services may be terminated only for and hearing. 22 This concept evolved from the basic tenet that one's employment or profession is
causes provided by law. 12 a property right protected by the constitutional guaranty of due process of law. 23 Hence, an
individual's separation from work must be founded on clearly-established facts, not on mere
conjectures and suspicions. 24
Viewed in this light, while private respondent has the right to terminate the services of petitioner,
this is subject to both substantive and procedural grounds. 13 The substantive causes for
dismissal are those provided in Articles 282 and 283 of the, Labor Code, 14 while the procedural In light of the foregoing, private respondent's arguments are clearly baseless and without merit.
grounds refer to the observance of the requirement of due process. 15 In all these instances, it is In truth, instead of protecting petitioner's reputation, private respondent succeeded in doing
the private respondent, being the employer, who must prove the validity of the dismissal. 16 exactly the opposite — it condemned the petitioner without even hearing his side. It is stating the
obvious that dismissal, being the ultimate penalty that can be meted out to an employee, should
be based on a clear or convincing ground. 25As such, a decision to terminate an employee
Having reviewed the records of this case carefully, we conclude that private respondent utterly
without fully apprising him of the facts, on the pretext that the twin requirements of notice and
failed to substantiate petitioner's dismissal, rendering the latter's termination illegal. At the risk of
hearing are unnecessary or useless, is an invalid and obnoxious exercise of management
being redundant, it must be stressed that these requirements are mandatory and non-
prerogative.
compliance therewith renders any judgment reached by the management void and inexistent. 17

Furthermore, private respondent, in an effort to validate the dismissal of the petitioner, posits the
While private respondent imputes "gross negligence," and "serious misconduct" as the causes of
theory that the latter's position is coterminus with that of the Village's Board of Directors, as
petitioner's dismissal, 18 not a shred of evidence was offered in support thereof, other than bare
provided for in its amended by-laws. 26
and uncorroborated allegations. The facts and circumstances regarding such alleged infractions
were never explained, While it is true that private respondent, through its president Bonifacio
Dazo, executed an affidavit narrating the alleged violations of the petitioner, 19 these were never Admittedly, the right to amend the by-laws lies solely in the discretion of the employer, this being
corroborated by concrete or competent evidence. It is settled that no undue importance should in the exercise of management prerogative or business judgment. However this right, extensive
be given to a sworn statement or affidavit as a piece of evidence because, being taken ex-parte, as it may be, cannot impair the obligation of existing contracts or rights.
an affidavit is almost always incomplete and inaccurate. 20 Furthermore, it must be noted that
when petitioner was terminated in 1992, these alleged infractions were never raised nor
Prescinding from these premises, private respondent's insistence that it can legally dismiss
communicated to him. In fact, these were only revealed after the complaint was filed by the
petitioner on the ground that his tenure has expired is untenable. To reiterate, petitioner, being a
petitioner in 1993. Why there was a delay was never adequately explained by private
regular employee, is entitled to security of tenure, hence, his services may only be terminated for
respondent.
causes provided by law. 27 A contrary interpretation would not find justification in the laws or the
Constitution. If we were to rule otherwise, it would enable an employer to remove any employee
Likewise, we note that Dazo himself was not presented as a witness to give the petitioner an from his employment by the simple expediency of amending its by-laws and providing that
opportunity to cross-examine him and propound clarificatory questions regarding matters his/her position shall cease to exist upon the occurrence of a specified event.
averred in his affidavit. All told, the foregoing lapses and the belated submission of the affidavit,
cast doubt as to the credibility of the allegations. In sum, the dismissal of the petitioner had no
If private respondent wanted to make the petitioner's position co-terminus with that of the Board
factual basis whatsoever. The rule is that unsubstantiated accusations without more, are not
of Directors, then the amendment must be effective after petitioner's stay with the private
tantamount to guilt. 21
respondent, not during his term. Obviously, the measure taken by the private respondent in
amending its by-laws is nothing but a devious, but crude, attempt to circumvent petitioner's right
As regards the issue of procedural due process, private respondent justifies its non-compliance to security of tenure as a regular employee guaranteed under the Labor Code. 28
therewith in this wise:
Interestingly, the Solicitor General is of the view that what actually transpired was that petitioner
The Association Officers, being his peers and friends had a problem was retired from his employment, considering the fact that in 1992 he was already 70 years old
however in terminating his services. He had been found to have committed and not terminated. 29
infractions as previously enumerated. PVHA could have proceeded with a
full-blown investigation to hear these charges, but the ordeal might break
While there seems to be a semblance of plausibility in this contention for the matter of extension
the old man's heart as this will surely affect his standing in the community.
of service of such employee or official is addressed to the sound discretion of the employer, still
So they decided to make their move as discreetly (but legally) as possible to
we have no doubt that this was just a mere after-thought — a dismissal disguised as retirement.
save the petitioner's reputation. Terminating him in accordance with the
provision of the by-laws of the Association without pointing out his
numerous faults and malfeasance in office and with one-half month pay for In the proceedings before the Labor Arbiter, it is noteworthy that private respondent never raised
every year of service in accordance with the Retirement Law was the best the issue of compulsory retirement, 30 as a cause for terminating petitioner's service. In its
and only alternative. appeal before the NLRC, this ground was never discussed. In fact, private respondent, in
justifying the termination of the petitioner, still anchored its claim on the applicability of the
amended by-laws. This omission is fatal to private respondent's cause, for the rule is well-settled
We are not impressed. The reasoning advanced by the private respondent is as puerile as it is
that matters, theories or arguments not brought out in the proceedings below will ordinarily not
preposterous.
be considered by a reviewing court, as they cannot be raised for the first time on appeal. 31
Undaunted, private respondent now asserts that the instant petition was filed out of With respect to the issue that petitioner, being a managerial employee, is not entitled to
time, 32 considering that the assailed NLRC decision was received on June 28, 1995 while this thirteenth month pay, Memorandum Order No. 28, as implemented by the Revised Guidelines
petition was filed on September 20, 1995. At this juncture, we take this opportunity to state that on the Implementation of the 13th Month Pay Law dated November 16, 1987, provides:
under the 1997 Rules of Civil Procedure, a petition for certiorari must now be instituted within
sixty days of receipt of the assailed judgment, order or resolution. 33 However, since this case
Sec. 1 of Presidential Decree No. 851 is hereby modified to the extent that
arose in 1995 and the aforementioned rule only took effect on July 1, 1997 then the old rule is
all employers are hereby required to pay all their rank and file employees a
applicable. Since prior to the effectivity of the new rule, a special civil action ofcertiorari should
13th month pay not later than December 24 of every year.
be instituted within a period of three months, 34 the instant petition which was filed on September
20, 1995 or two months and twenty-two days thereafter, was still within the reglementary period.
Clearly, therefore, the foregoing exempts managerial employees from this benefit. Of course,
this does not preclude an employer from granting other bonuses, in lieu of the 13th month pay,
With respect to the issue of the monetary award to be given to the petitioner, private respondent
to managerial employees in its discretion.
argues that he deserves only retirement pay and nothing more. This position would have been
tenable had petitioner not been illegally dismissed. However, since we have already ruled
petitioner's dismissal as without just cause and lacking due process, the award of backwages Finally, we cannot simply ignore private respondent's malicious scheme to remove petitioner
and reinstatement is proper. 35 from his position which is contrary to good customs and effected in an oppressive manner, thus
warranting an award of moral and exemplary damages to the petitioner. 41 Moreover, since
petitioner was forced to litigate and incur expenses to protect his right and interests, he is
In this particular case, reinstatement is no longer feasible since petitioner was already 70 years
entitled to attorney's fees.42
old at the time he was removed from his employment. As a substitute thereof, separation pay is
generally awarded, 36the amount of which must be equivalent to one-month salary for every year
of service.37 WHEREFORE, in view of the foregoing, the instant petition is GRANTED. The NLRC decision
dated June 15, 1995 is hereby REVERSED and SET ASIDE. Private respondent Philamlife
Village Homeowners Association is ORDERED: (1) to pay petitioner Enrique Salafranca
With respect to the amount of backwages which, incidentally is different from separation pay, 38 it
separation pay equivalent to one month salary for every year of service; (2) to pay his full
now settled that an illegally dismissed employee is entitled to its full payment as long as the
backwages in accordance with our ruling in Bustamante v. NLRC; 43(3) to pay his retirement pay
cause of action accrued after March 21, 1989. 39 Considering that petitioner was terminated from
in accordance with Article 287 of the Labor Code, as amended by Republic Act No. 7641, (4) to
the service on December 9, 1992, which is after March 21, 1989, he is entitled to full backwages
pay moral and exemplary damages in the amount of twenty thousand (P20,000.00) pesos and
from the time of the illegal dismissal without any, qualification or deduction. 40
ten thousand (P10,000.00) pesos, respectively; 44 and (5) to pay ten (10%) percent of the total
amount due to petitioner, as attorney's fees. Consequently, the respondent NLRC is ORDERED
As regards the issue of retirement pay, private respondent asserts that the correct amount to COMPUTE the total monetary benefits awarded in accordance with this decision and to
should be one-half (1/2) month salary for every year of service. This time we agree with private submit its compliance thereon within thirty (30) days from notice of this decision.
respondent's contention. The pertinent law is Article 287 of the Labor Code, as amended by
Republic Act No. 7641, which reads:
SO ORDERED.

Art. 287. Retirement. — Any employee may be retired upon reaching the
retirement age established in the collective bargaining agreement or other
applicable employment contract.

In case of retirement, the employee shall he entitled to receive such


retirement benefits as he may have earned under existing laws and any
collective bargaining agreement and other agreements: Provided, however,
That an employee's retirement benefits under any collective bargaining and
other agreements shall not be less than those provided herein.

In the absence of a retirement plan or agreement providing for retirement


benefits of employees in the establishment, an employee upon reaching the
age of sixty (60) years or more, but not beyond sixty-five (65) years which is
hereby declared the compulsory retirement age, who has served at least
five (5) years in the said establishment, may retire and shall be entitled to
retirement pay equivalent to at least one-half (1/2) month salary for every
year of service, a fraction of al least six (6) months being considered as one
whole year.

xxx xxx xxx


G.R. No. 121466 August 15, 1997 exercised no appropriate and proper supervision of the said classes which activities allegedly
violated certain rules and regulations of the Department of Education, Culture and Sports
(DECS). Furthermore, the claims, according to petitioner, were all exaggerated and that, at any
PMI COLLEGES, petitioner,
rate, private respondent abandoned his work at the time he should have commenced the same.
vs.
THE NATIONAL LABOR RELATIONS COMMISSION and ALEJANDRO GA LVA
N, respondents. In reply, private respondent belied petitioner's allegations contending, among others, that he
conducted lectures within the premises of petitioner's rented space located at 5th Floor,
Manufacturers Bldg., Sta. Cruz, Manila; that his students duly enrolled with the Registrar's Office
ROMERO, J.:
of petitioner; that shipyard and plant visits were conducted at Fort San Felipe, Cavite Naval
Base; that petitioner was fully aware of said shipyard and plant visits because it even wrote a
Subject of the instant petition for certiorari under Rule 65 of the Rules of Court is the letter for that purpose; and that basic seaman courses 41 and 42 were sanctioned by the DECS
resolution 1 of public respondent National Labor Relations Commission 2 rendered on August 4, as shown by the records of the Registrar's Office.
1995, affirming in toto the December 7, 1994 decision 3 of Labor Arbiter Pablo C. Espiritu
declaring petitioner PMI Colleges liable to pay private respondent Alejandro Galvan P405,000.00
Later in the proceedings below, petitioner manifested that Mr. Tomas G. Cloma, Jr., a member
in unpaid wages and P40,532.00 as attorney's fees.
of the petitioner's Board of Trustees wrote a letter 5 to the Chairman of the Board on May 23,
1994, clarifying the case of private respondent and stating therein, inter alia, that under
A chronicle of the pertinent events on record leading to the filing of the instant petition is as petitioner's by-laws only the Chairman is authorized to sign any contract and that private
follows: respondent, in any event, failed to submit documents on the alleged shipyard and plant visits in
Cavite Naval Base.
On July 7, 1991, petitioner, an educational institution offering courses on basic seaman's training
and other marine-related courses, hired private respondent as contractual instructor with an Attempts at amicable settlement having failed, the parties were required to submit their
agreement that the latter shall be paid at an hourly rate of P30.00 to P50.00, depending on the respective position papers. Thereafter, on June 16, 1994, the Labor Arbiter issued an order
description of load subjects and on the schedule for teaching the same. Pursuant to this declaring the case submitted for decision on the basis of the position papers which the parties
engagement, private respondent then organized classes in marine engineering. filed. Petitioner, however, vigorously opposed this order insisting that there should be a formal
trial on the merits in view of the important factual issues raised. In another order dated July 22,
1994, the Labor Arbiter impliedly denied petitioner's opposition, reiterating that the case was
Initially, private respondent and other instructors were compensated for services rendered during already submitted for decision. Hence, a decision was subsequently rendered by the Labor
the first three periods of the abovementioned contract. However, for reasons unknown to private Arbiter on December 7, 1994 finding for the private respondent. On appeal, the NLRC affirmed
respondent, he stopped receiving payment for the succeeding rendition of services. This claim of
the same in toto in its decision of August 4, 1995.
non-payment was embodied in a letter dated March 3, 1992, written by petitioner's Acting
Director, Casimiro A. Aguinaldo, addressed to its President, Atty. Santiago Pastor, calling
attention to and appealing for the early approval and release of the salaries of its instructors Aggrieved, petitioner now pleads for the Court to resolve the following issues in its favor, to wit:
including that of private respondent. It appeared further in said letter that the salary of private
respondent corresponding to the shipyard and plant visits and the ongoing on-the-job training of
I. Whether the money claims of private respondent representing salaries/wages as
Class 41 on board MV "Sweet Glory" of Sweet Lines, Inc. was not yet included. This request of
contractual instructor for class instruction, on-the-job training and shipboard and plant
the Acting Director apparently went unheeded. Repeated demands having likewise failed,
visits have valid legal and factual bases;
private respondent was soon constrained to file a complaint 4 before the National Capital Region
Arbitration Branch on September 14, 1993 seeking payment for salaries earned from the
following: (1) basic seaman course Classes 41 and 42 for the period covering October 1991 to II. Whether claims for salaries/wages for services relative to on-the-job training and
September 1992; (2) shipyard and plant visits and on-the-job training of Classes 41 and 42 for shipboard and plant visits by instructors, assuming the same were really conducted,
the period covering October 1991 to September 1992 on board M/V "Sweet Glory" vessel; and have valid bases;
(3) as Acting Director of Seaman Training Course for 3-1/2 months.
III. Whether the petitioner was denied its right to procedural due process; and
In support of the abovementioned claims, private respondent submitted documentary evidence
which were annexed to his complaint, such as the detailed load and schedule of classes with
IV. Whether the NLRC findings in its questioned resolution have sound legal and
number of class hours and rate per hour (Annex "A"); PMI Colleges Basic Seaman Training
factual support.
Course (Annex "B"); the aforementioned letter-request for payment of salaries by the Acting
Director of PMI Colleges (Annex "C"); unpaid load of private respondent (Annex "D"); and
vouchers prepared by the accounting department of petitioner but whose amounts indicated We see no compelling reason to grant petitioner's plea; the same must, therefore, be dismissed.
therein were actually never paid to private respondent (Exhibit "E").
At once, a mere perusal of the issues raised by petitioner already invites dismissal for
Private respondent's claims, as expected, were resisted by petitioner. It alleged that classes in demonstrated ignorance and disregard of settled rules on certiorari. Except perhaps for the third
the courses offered which complainant claimed to have remained unpaid were not held or issue, the rest glaringly call for a re-examination, evaluation and appreciation of the weight and
conducted in the school premises of PMI Colleges. Only private respondent, it was argued, knew sufficiency of factual evidence presented before the Labor Arbiter. This, of course, the Court
whether classes were indeed conducted. In the same vein, petitioner maintained that it cannot do in the exercise of its certiorari jurisdiction without transgressing the well-defined limits
thereof. The corrective power of the Court in this regard is confined only to jurisdictional issues In any event, granting that we may have to delve into the facts and evidence of the parties, we
and a determination of whether there is such grave abuse of discretion amounting to lack or still find no puissant justification for us to adjudge both the Labor Arbiter's and NLRC's
excess of jurisdiction on the part of a tribunal or agency. So unyielding and consistent are the appreciation of such evidence as indicative of any grave abuse of discretion.
decisional rules thereon that it is indeed surprising why petitioner's counsel failed to accord them
the observance they deserve.
First. Petitioner places so much emphasis on its argument that private respondent did not
produce a copy of the contract pursuant to which he rendered services. This argument is, of
6
Thus, in San Miguel Foods, Inc. Cebu B-Men Feed Plant v. Hon. Bienvenido Laguesma, we course, puerile. The absence of such copy does not in any manner negate the existence of a
were emphatic in declaring that: contract of employment since "(C)ontracts shall be obligatory, in whatever form they have
been entered into, provided all the essential requisites for their validity are present." 9 The only
exception to this rule is "when the law requires that a contract be in some form in order that it
This Court is definitely not the proper venue to consider this matter for it is not a trier
may be valid or enforceable, or that a contract be proved in a certain way." However, there is no
of facts. . . . Certiorari is a remedy narrow in its scope and inflexible in character. It is
requirement under the law that the contract of employment of the kind entered into by petitioner
not a general utility tool in the legal workshop. Factual issues are not a proper subject
with private respondent should be in any particular form. While it may have been desirable for
for certiorari, as the power of the Supreme Court to review labor cases is limited to the
private respondent to have produced a copy of his contract if one really exists, but the absence
issue of jurisdiction and grave abuse of discretion. . . . (Emphasis supplied).
thereof, in any case, does not militate against his claims inasmuch as:

Of the same tenor was our disquisition in Ilocos Sur Electric Cooperative, Inc. v. NLRC 7 where
No particular form of evidence is required to prove the existence of an employer-
we made plain that:
employee relationship. Any competent and relevant evidence to prove the relationship
may be admitted. For, if only documentary evidence would be required to show that
In certiorari proceedings under Rule 65 of the Rules of Court, judicial review by this relationship, no scheming employer would ever be brought before the bar of justice, as
Court does not go so far as to evaluate the sufficiency of evidence upon which the no employer would wish to come out with any trace of the illegality he has authored
Labor Arbiter and the NLRC based their determinations, the inquiry being limited considering that it should take much weightier proof to invalidate a written instrument. .
essentially to whether or not said public respondents had acted without or in excess of . . 10
its jurisdiction or with grave abuse of discretion. (Emphasis supplied).
At any rate, the vouchers prepared by petitioner's own accounting department and the letter-
To be sure, this does not mean that the Court would disregard altogether the evidence request of its Acting Director asking for payment of private respondent's services suffice to
presented. We merely declare that the extent of review of evidence we ordinarily provide in other support a reasonable conclusion that private respondent was employed with petitioner. How else
cases is different when it is a special civil action of certiorari. The latter commands us to merely could one explain the fact that private respondent was supposed to be paid the amounts
determine whether there is basis established on record to support the findings of a tribunal and mentioned in those documents if he were not employed? Petitioner's evidence is wanting in this
such findings meet the required quantum of proof, which in this instance, is substantial evidence. respect while private respondent affirmatively stated that the same arose out of his employment
Our deference to the expertise acquired by quasi-judicial agencies and the limited scope granted with petitioner. As between the two, the latter is weightier inasmuch as we accord affirmative
to us in the exercise of certiorari jurisdiction restrain us from going so far as to probe into the testimony greater value than a negative one. For the foregoing reasons, we find it difficult to
correctness of a tribunal's evaluation of evidence, unless there is palpable mistake and complete agree with petitioner's assertion that the absence of a copy of the alleged contract should nullify
disregard thereof in which case certiorari would be proper. In plain terms, private respondent's claims.
in certiorari proceedings, we are concerned with mere "errors of jurisdiction" and not "errors of
judgment." Thus:
Neither can we concede that such contract would be invalid just because the signatory thereon
was not the Chairman of the Board which allegedly violated petitioner's by-laws. Since by-laws
The rule is settled that the original and exclusive jurisdiction of this Court to review a operate merely as internal rules among the stockholders, they cannot affect or prejudice third
decision of respondent NLRC (or Executive Labor Arbiter as in this case) in a petition persons who deal with the corporation, unless they have knowledge of the same." 11 No proof
for certiorari under Rule 65 does not normally include an inquiry into the correctness of appears on record that private respondent ever knew anything about the provisions of said by-
its evaluation of the evidence. Errors of judgment, as distinguished from errors of laws. In fact, petitioner itself merely asserts the same without even bothering to attach a copy or
jurisdiction, are not within the province of a special civil action for certiorari, which is excerpt thereof to show that there is such a provision. How can it now expect the Labor Arbiter
merely confined to issues of jurisdiction or grave abuse of discretion. It is thus and the NLRC to believe it? That this allegation has never been denied by private respondent
incumbent upon petitioner to satisfactorily establish that respondent Commission or does not necessarily signify admission of its existence because technicalities of law and
executive labor arbiter acted capriciously and whimsically in total disregard of procedure and the rules obtaining in the courts of law do not strictly apply to proceedings of this
evidence material to or even decisive of the controversy, in order that the nature.
extraordinary writ of certiorari will lie. By grave abuse of discretion is meant such
capricious and whimsical exercise of judgment as is equivalent to lack of jurisdiction,
Second. Petitioner bewails the fact that both the Labor Arbiter and the NLRC accorded due
and it must be shown that the discretion was exercised arbitrarily or despotically.
weight to the documents prepared by private respondent since they are said to be self-serving.
For certiorari to lie there must be capricious, arbitrary and whimsical exercise of
"Self-serving evidence" is not to be literally taken as evidence that serves one's selfish
power, the very antithesis of the judicial prerogative in accordance with centuries of
interest. 12 The fact alone that most of the documents submitted in evidence by private
both civil law and common law traditions. 8
respondent were prepared by him does not make them self-serving since they have been
offered in the proceedings before the Labor Arbiter and that ample opportunity was given to
The Court entertains no doubt that the foregoing doctrines apply with equal force in the case at petitioner to rebut their veracity and authenticity. Petitioner, however, opted to merely deny them
bar. which denial, ironically, is actually what is considered self-serving evidence 13 and, therefore,
deserves scant consideration. In any event, any denial made by petitioner cannot stand against The petitioners did ask in their position paper for a hearing to thresh out some factual
the affirmative and fairly detailed manner by which private respondent supported his claims, matters pertinent to their case. However, they had no right or reason to assume that
such as the places where he conducted his classes, on-the-job training and shipyard and plant their request would be granted. The petitioners should have attached to their position
visits; the rate he applied and the duration of said rendition of services; the fact that he was paper all the documents that would prove their claim in case it was decided that no
indeed engaged as a contractual instructor by petitioner; and that part of his services was not yet hearing should be conducted or was necessary. In fact, the rules require that position
remunerated. These evidence, to reiterate, have never been effectively refuted by petitioner. papers shall be accompanied by all supporting documents, including affidavits of
witnesses in lieu of their direct testimony. 14
Third. As regards the amounts demanded by private respondent, we can only rely upon the
evidence presented which, in this case, consists of the computation of private respondent, as It must be noted that adequate opportunity was given to petitioner in the presentation of its
well as the findings of both the Labor Arbiter and the NLRC. Petitioner, it must be stressed, evidence, such as when the Labor Arbiter granted petitioner's Manifestation and Motion 15 dated
presented no satisfactory proof to the contrary. Absent such proof, we are constrained to rely July 22, 1994 allowing it to submit four more documents. This opportunity notwithstanding,
upon private respondent's otherwise straightforward explanation of his claims. petitioner still failed to fully proffer all its evidence which might help the Labor Arbiter in resolving
the issues. What it desired instead, as stated in its petition, 16 was to "require presentation of
witnesses buttressed by relevant documents in support thereof." But this is precisely the
Fourth. The absence of a formal hearing or trial before the Labor Arbiter is no cause for
opportunity given to petitioner when the Labor Arbiter granted its Motion and Manifestation. It
petitioner to impute grave abuse of discretion. Whether to conduct one or not depends on the
should have presented the documents it was proposing to submit. The affidavits of its witnesses
sole discretion of the Labor Arbiter, taking into account the position papers and supporting
would have sufficed in lieu of their direct testimony 17 to clarify what it perceives to be complex
documents submitted by the parties on every issue presented. If the Labor Arbiter, in his
factual issues. We rule that the Labor Arbiter and the NLRC were not remiss in their duty to
judgment, is confident that he can rely on the documents before him, he cannot be faulted for
afford petitioner due process. The essence of due process is merely that a party be afforded a
not conducting a formal trial anymore, unless it would appear that, in view of the particular
reasonable opportunity to be heard and to submit any evidence he may have in support of his
circumstances of a case, the documents, without more, are really insufficient.
defense. 18

As applied to the instant case, we can understand why the Labor Arbiter has opted not to
WHEREFORE, in view of the foregoing, the instant petition is hereby DISMISSED for lack of
proceed to trial, considering that private respondent, through annexes to his position paper, has
merit while the resolution of the National Labor Relations Commission dated August 4, 1995 is
adequately established that, first of all, he was an employee of petitioner; second, the nature and
hereby AFFIRMED.
character of his services, and finally, the amounts due him in consideration of his services.
Petitioner, it should be reiterated, failed to controvert them. Actually, it offered only four
documents later in the course of the proceedings. It has only itself to blame if it did not attach its SO ORDERED.
supporting evidence with its position paper. It cannot now insist that there be a trial to give it an
opportunity to ventilate what it should have done earlier. Section 3, Rule V of the New Rules of
Procedure of the NLRC is very clear on the matter:

Sec. 3. . . .

These verified position papers . . . shall be accompanied by all supporting documents


including the affidavits of their respective witnesses which shall take the place of the
latter's direct testimony. The parties shall thereafter not be allowed to allege facts, or
present evidence to prove facts, not referred to and any cause or causes of action not
included in the complaint or position papers, affidavits and other documents. . . .
(Emphasis supplied).

Thus, given the mandate of said rule, petitioner should have foreseen that the Labor Arbiter, in
view of the non-litigious nature of the proceedings before it, might not proceed at all to trial.
Petitioner cannot now be heard to complain of lack of due process. The following is apropos:

The petitioners should not have assumed that after they submitted their position
papers, the Labor Arbiter would call for a formal trial or hearing. The holding of a trial
is discretionary on the Labor Arbiter, it is not a matter of right of the parties, especially
in this case, where the private respondents had already presented their documentary
evidence.

xxx xxx xxx


G.R. No. L-19761 January 29, 1923 In the case before us the resolution releasing the shareholders from their obligation to pay 50
per centum of their respective subscriptions was an attempted withdrawal of so much capital
from the fund upon which the company's creditors were entitled ultimately to rely and, having
PHILIPPINE TRUST COMPANY, as assignee in insolvency of "La Cooperativa Naval
been effected without compliance with the statutory requirements, was wholly ineffectual.
Filipina," plaintiff-appellee,
vs.
MARCIANO RIVERA, defendant-appellant. The judgment will be affirmed with cost, and it is so ordered.

STREET, J.:

This action was instituted on November 21, 1921, in the Court of First Instance of Manila, by the
Philippine Trust Company, as assignee in insolvency of La Cooperativa Naval Filipina, against
Marciano Rivera, for the purpose of recovering a balance of P22,500, alleged to be due upon
defendant's subscription to the capital stock of said insolvent corporation. The trial judge having
given judgment in favor of the plaintiff for the amount sued for, the defendant appealed.

It appears in evidence that in 1918 the Cooperativa Naval Filipina was duly incorporated under
the laws of the Philippine Islands, with a capital of P100,000, divided into one thousand shares
of a par value of P100 each. Among the incorporators of this company was numbered the
defendant Mariano Rivera, who subscribed for 450 shares representing a value of P45,000, the
remainder of the stock being taken by other persons. The articles of incorporation were duly
registered in the Bureau of Commerce and Industry on October 30 of the same year.

In the course of time the company became insolvent and went into the hands of the Philippine
Trust Company, as assignee in bankruptcy; and by it this action was instituted to recover one-
half of the stock subscription of the defendant, which admittedly has never been paid.

The reason given for the failure of the defendant to pay the entire subscription is, that not long
after the Cooperativa Naval Filipina had been incorporated, a meeting of its stockholders
occurred, at which a resolution was adopted to the effect that the capital should be reduced by
50 per centum and the subscribers released from the obligation to pay any unpaid balance of
their subscription in excess of 50 per centum of the same. As a result of this resolution it seems
to have been supposed that the subscription of the various shareholders had been cancelled to
the extent stated; and fully paid certificate were issued to each shareholders for one-half of his
subscription. It does not appear that the formalities prescribed in section 17 of the Corporation
Law (Act No. 1459), as amended, relative to the reduction of capital stock in corporations were
observed, and in particular it does not appear that any certificate was at any time filed in the
Bureau of Commerce and Industry, showing such reduction.

His Honor, the trial judge, therefore held that the resolution relied upon the defendant was
without effect and that the defendant was still liable for the unpaid balance of his subscription. In
this we think his Honor was clearly right.

It is established doctrine that subscription to the capital of a corporation constitute a find to which
creditors have a right to look for satisfaction of their claims and that the assignee in insolvency
can maintain an action upon any unpaid stock subscription in order to realize assets for the
payment of its debts. (Velasco vs. Poizat, 37 Phil., 802.) A corporation has no power to release
an original subscriber to its capital stock from the obligation of paying for his shares, without a
valuable consideration for such release; and as against creditors a reduction of the capital stock
can take place only in the manner an under the conditions prescribed by the statute or the
charter or the articles of incorporation. Moreover, strict compliance with the statutory regulations
is necessary (14 C. J., 498, 620).
G.R. No. L-56655 July 25, 1983 there is no sufficient legal basis to set aside the
certificate issued by this Commission authorizing the
increase in capital stock of respondent corporation from
DATU TAGORANAO BENITO, petitioner,
P200,000.00 to Pl,000,000.00. Considering, however,
vs.
that petitioner has not waived his pre-emptive right to
SECURITIES AND EXCHANGE COMMISSION and JAMIATUL PHILIPPINE-AL ISLAMIA,
subscribe to the increased capitalization, respondent
INC., respondents..
corporation is hereby directed to allow petitioner to
subscribe thereto, at par value, proportionate to his
RELOVA, J.: present shareholdings, adding thereto the 2,540 shares
transferred to him by Mr. Domocao Alonto and Mrs.
Moki-in Alonto; (c) To direct as it hereby directs, the
On February 6, 1959, the Articles of Incorporation of respondent Jamiatul Philippine-Al Islamia, respondent corporation to immediately cancel
Inc. (originally Kamilol Islam Institute, Inc.) were filed with the Securities and Exchange Certificates of Stock Nos. 216, 223, 302, all in the name
Commission (SEC) and were approved on December 14, 1962. The corporation had an
of Domocao Alonto, and Certificate of Stock No. 217, in
authorized capital stock of P200,000.00 divided into 20,000 shares at a par value of P10.00 the name of Moki-in Alonto, upon their presentation by
each. Of the authorized capital stock, 8,058 shares worth P80,580.00 were subscribed and fully the petitioner and to issue new certificates
paid for. Herein petitioner Datu Tagoranao Benito subscribed to 460 shares worth P4,600.00.
corresponding thereto in the name of petitioner herein;
(d) To direct, as it hereby directs, respondent
On October 28, 1975, the respondent corporation filed a certificate of increase of its capital stock corporation to religiously comply with the requirement of
from P200,000.00 to P1,000,000.00. It was shown in said certificate that P191,560.00 worth of filing annual financial statements under pain of a more
shares were represented in the stockholders' meeting held on November 25, 1975 at which time drastic action; (e) To declare, as it hereby declares, as
the increase was approved. Thus, P110,980.00 worth of shares were subsequently issued by irregular, the election of the nine (9) members of the
the corporation from the unissued portion of the authorized capital stock of P200,000.00. Of the Board of Trustees of respondent corporation on
increased capital stock of P1,000,000.00, P160,000.00 worth of shares were subscribed by Mrs. October 30, 1976, for which reason, respondent
Fatima A. Ramos, Mrs. Tarhata A. Lucman and Mrs. Moki-in Alonto. corporation is hereby ordered to call a stockholders'
meeting to elect a new set of five (5) members of the
Board of Trustees, unless in the meantime the said
On November 18, 1976, petitioner Datu Tagoranao filed with respondent Securities and number is accordingly increased and the requirement of
Exchange Commission a petition alleging that the additional issue (worth P110,980.00) of law to make such increase effective have been
previously subscribed shares of the corporation was made in violation of his pre-emptive right to complied with. It is understood that the said
said additional issue and that the increase in the authorized capital stock of the corporation from stockholders' meeting be called within thirty (30) days
P200,000.00 to P1,000,000.00 was illegal considering that the stockholders of record were not from the time petitioner shall have subscribed to the
notified of the meeting wherein the proposed increase was in the agenda. Petitioner prayed that increased capitalization.'
the additional issue of shares of previously authorized capital stock as well as the shares issued
from the increase in capital stock of respondent corporation be cancelled; that the secretary of
respondent corporation be ordered to register the 2,540 shares acquired by him (petitioner) from be, as the same is hereby AFFIRMED, the same being in accordance with
Domocao Alonto and Moki-in Alonto; and that the corporation be ordered to render an law and the facts of the case. (pp. 28-29, Reno)
accounting of funds to the stockholders.
Hence, this petition for review by way of appeal from the aforementioned decision of the
In their answer, respondents denied the material allegations of the petition and, by way of Securities and Exchange Commission, petitioner contending that (1) the issuance of the 11,098
special defense, claimed that petitioner has no cause of action and that the stock certificates shares without the consent of the stockholders or of the Board of Directors, and in the absence
covering the shares alleged to have been sold to petitioner were only given to him as collateral of consideration, is null and void; (2) the increase in the authorized capital stock from
for the loan of Domocao Alonto and Moki-in Alonto. P200,000.00 to P1,000,000.00 without the consent or express waiver of the stockholders, is null
and void; (3) he is entitled to attorneys' fees, damages and expenses of litigation in filing this suit
against the directors of respondent corporation.
On July 11, 1980, Hearing Officer Ledor E. Macalalag of the Securities and Exchange
Commission, after due proceedings, rendered a decision which was affirmed by the Commission
En Banc during its executive session held on March 9, 1981, as follows: We are not persuaded. As aptly stated by the Securities and Exchange Commission in its
decision:
RESOLVED, That the decision of the hearing Officer in SEC Case No.
1392, dated July 11, 1980, the dispositive portion of which reads as follows: xxx xxx xxx

WHEREFORE, in view of the foregoing considerations, ... the questioned issuance of the unsubscribed portion of the capital stock
this Commission hereby rules: (a) That the issuance by worth P110,980.00 is ' not invalid even if assuming that it was made without
the corporation of its unissued shares was validly made notice to the stockholders as claimed by petitioner. The power to issue
and was not subject to the pre-emptive rights of shares of stocks in a corporation is lodged in the board of directors and no
stockholders, including the petitioner, herein; (b) That stockholders' meeting is necessary to consider it because additional
issuance of shares of stocks does not need approval of the stockholders. ... Findings of fact by an administrative board or official, following a hearing,
The by-laws of the corporation itself states that 'the Board of Trustees shall, are binding upon the courts and win not be disturbed except where the
in accordance with law, provide for the issue and transfer of shares of stock board or official has gone beyond his statutory authority, exercised
of the Institute and shall prescribe the form of the certificate of stock of the unconstitutional powers or clearly acted arbitrarily and without regard to his
Institute. (Art. V, Sec. 1). duty or with grave abuse of discretion. ...

Petitioner bewails the fact that in view of the lack of notice to him of such ACCORDINGLY, this petition is hereby dismissed for lack of merit.
subsequent issuance, he was not able to exercise his right of pre-emption
over the unissued shares. However, the general rule is that pre-emptive
SO ORDERED.
right is recognized only with respect to new issue of shares, and not with
respect to additional issues of originally authorized shares. This is on the
theory that when a corporation at its inception offers its first shares, it is
presumed to have offered all of those which it is authorized to issue. An
original subscriber is deemed to have taken his shares knowing that they
form a definite proportionate part of the whole number of authorized shares.
When the shares left unsubscribed are later re-offered, he cannot therefore
claim a dilution of interest. (Campos and Lopez-Campos Selected Notes
and Cases on Corporation Law, p. 855, citing Yasik V. Wachtel 25 Del. Ch.
247,17A. 2d 308 (1941). (pp. 33-34, Rollo)

With respect to the claim that the increase in the authorized capital stock was without the
consent, expressed or implied, of the stockholders, it was the finding of the Securities and
Exchange Commission that a stockholders' meeting was held on November 25,1975, presided
over by Mr. Ahmad Domocao Alonto, Chairman of the Board of Trustees and, among the many
items taken up then were the change of name of the corporation from Kamilol Islam Institute Inc.
to Jamiatul Philippine-Al Islamia, Inc., the increase of its capital stock from P200,000.00 to
P1,000,000.00, and the increase of the number of its Board of Trustees from five to nine.
"Despite the insistence of petitioner, this Commission is inclined to believe that there was a
stockholders' meeting on November 25, 1975 which approved the increase. The petitioner had
not sufficiently overcome the evidence of respondents that such meeting was in fact held. What
petitioner successfully proved, however, was the fact that he was not notified of said meeting
and that he never attended the same as he was out of the country at the time. The documentary
evidence of petitioner conclusively proved that he was attending the Mecca pilgrimage when the
meeting was held on November 25, 1975. (Exhs. 'Q', 'Q-14', 'R', 'S' and 'S-l'). While petitioner
doubts the authenticity of the alleged minutes of the proceedings (Exh. '4'), the Commission
notes with significance that said minutes contain numerous details of various items taken up
therein that would negate any claim that it was not authentic. Another thing that petitioner was
able to disprove was the allegation in the certificate of increase (Exh. 'E-l') that all stockholders
who did not subscribe to the increase of capital stock have waived their pre-emptive right to do
so. As far as the petitioner is concerned, he had not waived his pre-emptive right to subscribe as
he could not have done so for the reason that he was not present at the meeting and had not
executed a waiver, thereof. Not having waived such right and for reasons of equity, he may still
be allowed to subscribe to the increased capital stock proportionate to his present
shareholdings." (pp. 36-37, Rollo)

Well-settled is the rule that the findings of facts of administrative bodies will not be interfered with
by the courts in the absence of grave abuse of discretion on the part of said agencies, or unless
the aforementioned findings are not supported by substantial evidence. (Gokongwei, Jr. vs.
SEC, 97 SCRA 78). In a long string of cases, the Supreme Court has consistently adhered to the
rule that decisions of administrative officers are not to be disturbed by the courts except when
the former have acted without or in excess of their jurisdiction or with grave abuse of discretion
(Sichangco vs. Board of Commissioners of Immigration, 94 SCRA 61). Thus, in the case
of Deluao vs. Casteel ( L-21906, Dec. 24, 1968, 26 SCRA 475, 496, citing Pajo vs. Ago, et al., L-
15414, June 30, 1960) and Genitano vs. Secretary of Agriculture and Natural Resources, et
al. (L-2ll67, March 31, 1966), the Supreme Court held that:
G.R. No. 109491 February 28, 2001 issued by Hi-Cement through Lourdes M. de Leon, as treasurer, confirming the issuance of the
four checks in favor of E.T. Henry in payment for petroleum products.6
ATRIUM MANAGEMENT CORPORATION, petitioner,
vs. Respondent Hi-Cement presented as witness Ms. Erlinda Yap who testified that she was once a
COURT OF APPEALS, E.T. HENRY AND CO., LOURDES VICTORIA M. DE LEON, RAFAEL secretary to the treasurer of Hi-Cement, Lourdes M. de Leon, and as such she was familiar with
DE LEON, JR., AND HI-CEMENT CORPORATION, respondents. the four RCBC checks as the postdated checks issued by Hi-Cement to E.T. Henry upon
instructions of Ms. de Leon. She testified that E.T. Henry offered to give Hi-Cement a loan which
the subject checks would secure as collateral.7
----------------------------------------

On July 20, 1989, the Regional Trial Court, Manila, Branch 09 rendered a decision, the
G.R. No. 121794 February 28, 2001
dispositive portion of which reads:

LOURDES M. DE LEON, petitioner,


"WHEREFORE, in view of the foregoing considerations, and plaintiff having proved its
vs.
cause of action by preponderance of evidence, judgment is hereby rendered ordering
COURT OF APPEALS, ATRIUM MANAGEMENT CORPORATION, AND HI-CEMENT
all the defendants except defendant Antonio de las Alas to pay plaintiff jointly and
CORPORATION,respondents.
severally the amount of TWO MILLION (P2,000,000.00) PESOS with the legal rate of
interest from the filling of the complaint until fully paid, plus the sum of TWENTY
PARDO, J.: THOUSAND (P20,000.00) PESOS as and for attorney's fees and the cost of suit."

What is before the Court are separate appeals from the decision of the Court of Appeals, 1 ruling All other claims are, for lack of merit dismissed.
that Hi-Cement Corporation is not liable for four checks amounting to P2 million issued to E.T.
Henry and Co. and discounted to Atrium Management Corporation.
SO ORDERED."8

On January 3, 1983, Atrium Management Corporation filed with the Regional Trial Court, Manila
In due time, both Lourdes M. de Leon and Hi-Cement appealed to the Court of Appeals.9
an action for collection of the proceeds of four postdated checks in the total amount of P2
million. Hi-Cement Corporation through its corporate signatories, petitioner Lourdes M. de
Leon,2 treasurer, and the late Antonio de las Alas, Chairman, issued checks in favor of E.T. Lourdes M. de Leon submitted that the trial court erred in ruling that she was solidarilly liable
Henry and Co. Inc., as payee. E.T. Henry and Co., Inc., in turn, endorsed the four checks to with Hi-Cement for the amount of the check. Also, that the trial court erred in ruling that Atrium
petitioner Atrium Management Corporation for valuable consideration. Upon presentment for was an ordinary holder, not a holder in due course of the rediscounted checks.10
payment, the drawee bank dishonored all four checks for the common reason "payment
stopped". Atrium, thus, instituted this action after its demand for payment of the value of the
Hi-Cement on its part submitted that the trial court erred in ruling that even if Hi-Cement did not
checks was denied.3
authorize the issuance of the checks, it could still be held liable for the checks. And assuming
that the checks were issued with its authorization, the same was without any consideration,
After due proceedings, on July 20, 1989, the trial court rendered a decision ordering Lourdes M. which is a defense against a holder in due course and that the liability shall be borne alone by
de Leon, her husband Rafael de Leon, E.T. Henry and Co., Inc. and Hi-Cement Corporation to E.T. Henry.11
pay petitioner Atrium, jointly and severally, the amount of P2 million corresponding to the value
of the four checks, plus interest and attorney's fees.4
On March 17, 1993, the Court of Appeals promulgated its decision modifying the ruling of the
trial court, the dispositive portion of which reads:
On appeal to the Court of Appeals, on March 17, 1993, the Court of Appeals promulgated its
decision modifying the decision of the trial court, absolving Hi-Cement Corporation from liability
"Judgement is hereby rendered:
and dismissing the complaint as against it. The appellate court ruled that: (1) Lourdes M. de
Leon was not authorized to issue the subject checks in favor of E.T. Henry, Inc.; (2) The
issuance of the subject checks by Lourdes M. de Leon and the late Antonio de las Alas (1) dismissing the plaintiff's complaint as against defendants Hi-Cement Corporation
constituted ultra vires acts; and (3) The subject checks were not issued for valuable and Antonio De las Alas;
consideration.5
(2) ordering the defendants E.T. Henry and Co., Inc. and Lourdes M. de Leon, jointly
At the trial, Atrium presented as its witness Carlos C. Syquia who testified that in February 1981, and severally to pay the plaintiff the sum of TWO MILLION PESOS (P2,000,000.00)
Enrique Tan of E.T. Henry approached Atrium for financial assistance, offering to discount four with interest at the legal rate from the filling of the complaint until fully paid, plus
RCBC checks in the total amount of P2 million, issued by Hi-Cement in favor of E.T. Henry. P20,000.00 for attorney's fees.
Atrium agreed to discount the checks, provided it be allowed to confirm with Hi-Cement the fact
that the checks represented payment for petroleum products which E.T. Henry delivered to Hi-
Cement. Carlos C. Syquia identified two letters, dated February 6, 1981 and February 9, 1981 (3) Ordering the plaintiff and defendants E.T. Henry and Co., Inc. and Lourdes M. de
Leon, jointly and severally to pay defendant Hi-Cement Corporation, the sum of
P20,000.00 as and for attorney's fees.
With cost in this instance against the appellee Atrium Management Corporation and It is, however, our view that there is basis to rule that the act of issuing the checks was well
appellant Lourdes Victoria M. de Leon. within the ambit of a valid corporate act, for it was for securing a loan to finance the activities of
the corporation, hence, not an ultra viresact.
So ordered."12
"An ultra vires act is one committed outside the object for which a corporation is created as
defined by the law of its organization and therefore beyond the power conferred upon it by
Hence, the recourse to this Court.13
law"16 The term "ultra vires" is "distinguished from an illegal act for the former is merely voidable
which may be enforced by performance, ratification, or estoppel, while the latter is void and
The issues raised are the following: cannot be validated."17

In G. R. No. 109491 (Atrium, petitioner): The next question to determine is whether Lourdes M. de Leon and Antonio de las Alas were
personally liable for the checks issued as corporate officers and authorized signatories of the
check.
1. Whether the issuance of the questioned checks was an ultra vires act;

"Personal liability of a corporate director, trustee or officer along (although not necessarily) with
2. Whether Atrium was not a holder in due course and for value; and the corporation may so validly attach, as a rule, only when:

3. Whether the Court of Appeals erred in dismissing the case against Hi-Cement and "1. He assents (a) to a patently unlawful act of the corporation, or (b) for bad faith or
ordering it to pay P20,000.00 as attorney's fees.14 gross negligence in directing its affairs, or (c) for conflict of interest, resulting in
damages to the corporation, its stockholders or other persons;
In G. R. No. 121794 (de Leon, petitioner):
"2. He consents to the issuance of watered down stocks or who, having knowledge
1. Whether the Court of Appeals erred in holding petitioner personally liable for the Hi- thereof, does not forthwith file with the corporate secretary his written objection
Cement checks issued to E.T. Henry; thereto;

2. Whether the Court of Appeals erred in ruling that Atrium is a holder in due course; "3. He agrees to hold himself personally and solidarily liable with the corporation; or

3. Whether the Court of Appeals erred in ruling that petitioner Lourdes M. de Leon as "4. He is made, by a specific provision of law, to personally answer for his corporate
signatory of the checks was personally liable for the value of the checks, which were action."18
declared to be issued without consideration;
In the case at bar, Lourdes M. de Leon and Antonio de las Alas as treasurer and Chairman of
4. Whether the Court of Appeals erred in ordering petitioner to pay Hi-Cement Hi-Cement were authorized to issue the checks. However, Ms. de Leon was negligent when she
attorney's fees and costs.15 signed the confirmation letter requested by Mr. Yap of Atrium and Mr. Henry of E.T. Henry for
the rediscounting of the crossed checks issued in favor of E.T. Henry. She was aware that the
checks were strictly endorsed for deposit only to the payee's account and not to be further
We affirm the decision of the Court of Appeals. negotiated. What is more, the confirmation letter contained a clause that was not true, that is,
"that the checks issued to E.T. Henry were in payment of Hydro oil bought by Hi-Cement from
We first resolve the issue of whether the issuance of the checks was an ultra vires act. The E.T. Henry". Her negligence resulted in damage to the corporation. Hence, Ms. de Leon may be
record reveals that Hi-Cement Corporation issued the four (4) checks to extend financial held personally liable therefor.1âwphi1.nêt
assistance to E.T. Henry, not as payment of the balance of the P30 million pesos cost of hydro
oil delivered by E.T. Henry to Hi-Cement. Why else would petitioner de Leon ask for counterpart The next issue is whether or not petitioner Atrium was a holder of the checks in due course. The
checks from E.T. Henry if the checks were in payment for hydro oil delivered by E.T. Henry to Negotiable Instruments Law, Section 52 defines a holder in due course, thus:
Hi-Cement?

"A holder in due course is a holder who has taken the instrument under the following
Hi-Cement, however, maintains that the checks were not issued for consideration and that conditions:
Lourdes and E.T. Henry engaged in a "kiting operation" to raise funds for E.T. Henry, who
admittedly was in need of financial assistance. The Court finds that there was no sufficient
evidence to show that such is the case. Lourdes M. de Leon is the treasurer of the corporation (a) That it is complete and regular upon its face;
and is authorized to sign checks for the corporation. At the time of the issuance of the checks,
there were sufficient funds in the bank to cover payment of the amount of P2 million pesos.
(b) That he became the holder of it before it was overdue, and without
notice that it had been previously dishonored, if such was the fact;
(c) That he took it in good faith and for value;

(d) That at the time it was negotiated to him he had no notice of any infirmity
in the instrument or defect in the title of the person negotiating it."

In the instant case, the checks were crossed checks and specifically indorsed for deposit to
payee's account only. From the beginning, Atrium was aware of the fact that the checks were all
for deposit only to payee's account, meaning E.T. Henry. Clearly, then, Atrium could not be
considered a holder in due course.

However, it does not follow as a legal proposition that simply because petitioner Atrium was not
a holder in due course for having taken the instruments in question with notice that the same
was for deposit only to the account of payee E.T. Henry that it was altogether precluded from
recovering on the instrument. The Negotiable Instruments Law does not provide that a holder
not in due course can not recover on the instrument.19

The disadvantage of Atrium in not being a holder in due course is that the negotiable instrument
is subject to defenses as if it were non-negotiable.20 One such defense is absence or failure of
consideration.21

We need not rule on the other issues raised, as they merely follow as a consequence of the
foregoing resolutions.

WHEREFORE, the petitions are hereby DENIED. The decision and resolution of the Court of
Appeals in CA-G. R. CV No. 26686, are hereby AFFIRMED in toto.

No costs.

SO ORDERED.
G.R. No. 111448 January 16, 2002 Realty asked for payment of P1,500,000.00 as compensatory damages; P400,000.00 as
attorney's fees; and P500,000.00 as exemplary damages.
AF REALTY & DEVELOPMENT, INC. and ZENAIDA R. RANULLO, petitioners,
vs. In its answer, Dieselman alleged that there was no meeting of the minds between the parties in
DIESELMAN FREIGHT SERVICES, CO., MANUEL C. CRUZ, JR. and MIDAS the sale of the property and that it did not authorize any person to enter into such transaction on
DEVELOPMENT CORPORATION, respondents. its behalf.

SANDOVAL-GUTIERREZ, J.: Meanwhile, on July 30, 1988, Dieselman and Midas Development Corporation (Midas) executed
a Deed of Absolute Sale13 of the same property. The agreed price was P2,800.00 per square
meter. Midas delivered to Dieselman P500,000.00 as down payment and deposited the balance
Petition for review on certiorari assailing the Decision dated December 10, 1992 and the
of P5,300,000.00 in escrow account with the PCIBank.
Resolution (Amending Decision) dated August 5, 1993 of the Court of Appeals in CA-G.R. CV
No. 30133.
Constrained to protect its interest in the property, Midas filed on April 3, 1989 a Motion for Leave
to Intervene in Civil Case No. 56278. Midas alleged that it has purchased the property and took
Dieselman Freight Service Co. (Dieselman for brevity) is a domestic corporation and a
possession thereof, hence Dieselman cannot be compelled to sell and convey it to AF Realty.
registered owner of a parcel of commercial lot consisting of 2,094 square meters, located at 104
The trial court granted Midas' motion.
E. Rodriguez Avenue, Barrio Ugong, Pasig City, Metro Manila. The property is covered by
Transfer Certificate of Title No. 39849 issued by the Registry of Deeds of the Province of Rizal. 1
After trial, the lower court rendered the challenged Decision holding that the acts of Cruz, Jr.
bound Dieselman in the sale of the lot to AF Realty.14 Consequently, the perfected contract of
On May 10, 1988, Manuel C. Cruz, Jr., a member of the board of directors of Dieselman, issued
sale between Dieselman and AF Realty bars Midas' intervention. The trial court also held that
a letter denominated as "Authority To Sell Real Estate"2 to Cristeta N. Polintan, a real estate
Midas acted in bad faith when it initially paid Dieselman P500,000.00 even without seeing the
broker of the CNP Real Estate Brokerage. Cruz, Jr. authorized Polintan "to look for a
latter's title to the property. Moreover, the notarial report of the sale was not submitted to the
buyer/buyers and negotiate the sale" of the lot at P3,000.00 per square meter, or a total of
Clerk of Court of the Quezon City RTC and the balance of P5,300,000.00 purportedly deposited
P6,282,000.00. Cruz, Jr. has no written authority from Dieselman to sell the lot.
in escrow by Midas with a bank was not established.1âwphi1.nêt

In turn, Cristeta Polintan, through a letter3 dated May 19, 1988, authorized Felicisima ("Mimi")
The dispositive portion of the trial court's Decision reads:
Noble4 to sell the same lot.

"WHEREFORE, foregoing considered, judgment is hereby rendered ordering


Felicisima Noble then offered for sale the property to AF Realty & Development, Inc. (AF Realty)
defendant to execute and deliver to plaintiffs the final deed of sale of the property
at P2,500.00 per square meter.5 Zenaida Ranullo, board member and vice-president of AF
covered by the Transfer Certificate of Title No. 39849 of the Registry of Deed of Rizal,
Realty, accepted the offer and issued a check in the amount of P300,000.00 payable to the
Metro Manila District II, including the improvements thereon, and ordering defendants
order of Dieselman. Polintan received the check and signed an "Acknowledgement
to pay plaintiffs attorney's fees in the amount of P50,000.00 and to pay the costs.
Receipt"6 indicating that the amount of P300,000.00 represents the partial payment of the
property but refundable within two weeks should AF Realty disapprove Ranullo's action on the
matter. "The counterclaim of defendants is necessarily dismissed.

On June 29, 1988, AF Realty confirmed its intention to buy the lot. Hence, Ranullo asked "The counterclaim and/or the complaint in intervention are likewise dismissed
Polintan for the board resolution of Dieselman authorizing the sale of the property. However,
Polintan could only give Ranullo the original copy of TCT No. 39849, the tax declaration and tax
"SO ORDERED."15
receipt for the lot, and a photocopy of the Articles of Incorporation of Dieselman. 7

Dissatisfied, all the parties appealed to the Court of Appeals.


On August 2, 1988, Manuel F. Cruz, Sr., president of Dieselman, acknowledged receipt of the
said P300,000.00 as "earnest money" but required AF Realty to finalize the sale
at P4,000.00 per square meter.8 AF Realty replied that it has paid an initial down payment of AF Realty alleged that the trial court erred in not holding Dieselman liable for moral,
P300,000.00 and is willing to pay the balance.9 compensatory and exemplary damages, and in dismissing its counterclaim against Midas.

However, on August 13, 1988, Mr. Cruz, Sr. terminated the offer and demanded from AF Realty Upon the other hand, Dieselman and Midas claimed that the trial court erred in finding that a
the return of the title of the lot earlier delivered by Polintan.10 contract of sale between Dieselman and AF Realty was perfected. Midas further averred that
there was no bad faith on its part when it purchased the lot from Dieselman.
Claiming that there was a perfected contract of sale between them, AF Realty filed with the
Regional Trial Court, Branch 160, Pasig City a complaint for specific performance (Civil Case In its Decision dated December 10, 1992, the Court of Appeals reversed the judgment of the trial
No. 56278) against Dieselman and Cruz, Jr.. The complaint prays that Dieselman be ordered to court holding that since Cruz, Jr. was not authorized in writing by Dieselman to sell the subject
execute and deliver a final deed of sale in favor of AF Realty.11 In its amended complaint,12 AF property to AF Realty, the sale was not perfected; and that the Deed of Absolute Sale between
Dieselman and Midas is valid, there being no bad faith on the part of the latter. The Court of document duly certified by notary public under his hand and seal. x x x. Such a deed
Appeals then declared Dieselman and Cruz, Jr. jointly and severally liable to AF Realty for of sale being public document acknowledged before a notary public is admissible as to
P100,000.00 as moral damages; P100,000.00 as exemplary damages; and P100,000.00 as the date and fact of its execution without further proof of its due execution and delivery
attorney's fees.16 (Bael vs. Intermediate Appellate Court, 169 SCRA617; Joson vs. Baltazar, 194 SCRA
114) and to prove the defects and lack of consent in the execution thereof, the
evidence must be strong and not merely preponderant x x x."18
On August 5, 1993, the Court of Appeals, upon motions for reconsideration filed by the parties,
promulgated an Amending Decision, the dispositive portion of which reads:
We agree with the Court of Appeals.
"WHEREFORE, The Decision promulgated on October 10, 1992, is hereby
AMENDED in the sense that only defendant Mr. Manuel Cruz, Jr. should be made Section 23 of the Corporation Code expressly provides that the corporate powers of all
liable to pay the plaintiffs the damages and attorney's fees awarded therein, plus the corporations shall be exercised by the board of directors. Just as a natural person may authorize
amount of P300,000.00 unless, in the case of the said P300,000.00, the same is still another to do certain acts in his behalf, so may the board of directors of a corporation validly
deposited with the Court which should be restituted to plaintiffs. delegate some of its functions to individual officers or agents appointed by it. 19 Thus, contracts or
acts of a corporation must be made either by the board of directors or by a corporate agent duly
authorized by the board.20 Absent such valid delegation/authorization, the rule is that the
"SO ORDERED."17
declarations of an individual director relating to the affairs of the corporation, but not in the
course of, or connected with, the performance of authorized duties of such director, are held not
AF Realty now comes to this Court via the instant petition alleging that the Court of Appeals binding on the corporation.21
committed errors of law.
In the instant case, it is undisputed that respondent Cruz, Jr. has no written authority from the
The focal issue for consideration by this Court is who between petitioner AF Realty and board of directors of respondent Dieselman to sell or to negotiate the sale of the lot, much less
respondent Midas has a right over the subject lot. to appoint other persons for the same purpose. Respondent Cruz, Jr.'s lack of such authority
precludes him from conferring any authority to Polintan involving the subject realty. Necessarily,
neither could Polintan authorize Felicisima Noble. Clearly, the collective acts of respondent
The Court of Appeals, in reversing the judgment of the trial court, made the following Cruz, Jr., Polintan and Noble cannot bind Dieselman in the purported contract of sale.
ratiocination:

Petitioner AF Realty maintains that the sale of land by an unauthorized agent may be ratified
"From the foregoing scenario, the fact that the board of directors of Dieselman never where, as here, there is acceptance of the benefits involved. In this case the receipt by
authorized, verbally and in writing, Cruz, Jr. to sell the property in question or to look
respondent Cruz, Jr. from AF Realty of the P300,000.00 as partial payment of the lot effectively
for buyers and negotiate the sale of the subject property is undeniable. binds respondent Dieselman.22

"While Cristeta Polintan was actually authorized by Cruz, Jr. to look for buyers and We are not persuaded.
negotiate the sale of the subject property, it should be noted that Cruz, Jr. could not
confer on Polintan any authority which he himself did not have. Nemo dat quod non
habet. In the same manner, Felicisima Noble could not have possessed authority Involved in this case is a sale of land through an agent. Thus, the law on agency under the Civil
broader in scope, being a mere extension of Polintan's purported authority, for it is a Code takes precedence. This is well stressed in Yao Ka Sin Trading vs. Court of Appeals:23
legal truism in our jurisdiction that a spring cannot rise higher than its source.
Succinctly stated, the alleged sale of the subject property was effected through
"Since a corporation, such as the private respondent, can act only through its officers
persons who were absolutely without any authority whatsoever from Dieselman.
and agents, all acts within the powers of said corporation may be performed by
agents of its selection; and, except so far as limitations or restrictions may be
"The argument that Dieselman ratified the contract by accepting the P300,000.00 as imposed by special charter, by-law, or statutory provisions, the same general
partial payment of the purchase price of the subject property is equally untenable. The principles of law which govern the relation of agency for a natural person
sale of land through an agent without any written authority is void. govern the officer or agent of a corporation, of whatever status or rank, in
respect to his power to act for the corporation; and agents when once
appointed, or members acting in their stead, are subject to thesame rules,
xxx xxx xxx liabilities, and incapacities as are agents of individuals and private persons."
(Emphasis supplied)
"On the contrary, anent the sale of the subject property by Dieselman to intervenor
Midas, the records bear out that Midas purchased the same from Dieselman on 30 Pertinently, Article 1874 of the same Code provides:
July 1988. The notice of lis pendens was subsequently annotated on the title of the
property by plaintiffs on 15 August 1988. However, this subsequent annotation of the
notice of lis pendens certainly operated prospectively and did not retroact to make the "ART. 1874. When a sale of piece of land or any interest therein is through an
previous sale of the property to Midas a conveyance in bad faith. A subsequently agent, the authority of the latter shall be in writing; otherwise, the sale shall be
registered notice of lis pendens surely is not proof of bad faith. It must therefore be void." (Emphasis supplied)
borne in mind that the 30 July 1988 deed of sale between Midas and Dieselman is a
Considering that respondent Cruz, Jr., Cristeta Polintan and Felicisima Ranullo were not
authorized by respondent Dieselman to sell its lot, the supposed contract is void. Being a void
contract, it is not susceptible of ratification by clear mandate of Article 1409 of the Civil Code,
thus:

"ART. 1409. The following contracts are inexistent and void from the very
beginning:

xxx

(7) Those expressly prohibited or declared void by law.

"These contracts cannot be ratified. Neither can the right to set up the defense of
illegality be waived." (Emphasis supplied)

Upon the other hand, the validity of the sale of the subject lot to respondent Midas is
unquestionable. As aptly noted by the Court of Appeals,24 the sale was authorized by a board
resolution of respondent Dieselman dated May 27, 1988.1âwphi1.nêt

The Court of Appeals awarded attorney's fees and moral and exemplary damages in favor of
petitioner AF Realty and against respondent Cruz, Jr.. The award was made by reason of a
breach of contract imputable to respondent Cruz, Jr. for having acted in bad faith. We are no
persuaded. It bears stressing that petitioner Zenaida Ranullo, board member and vice-president
of petitioner AF Realty who accepted the offer to sell the property, admitted in her
testimony25 that a board resolution from respondent Dieselman authorizing the sale is necessary
to bind the latter in the transaction; and that respondent Cruz, Jr. has no such written authority.
In fact, despite demand, such written authority was not presented to her.26 This notwithstanding,
petitioner Ranullo tendered a partial payment for the unauthorized transaction. Clearly,
respondent Cruz, Jr. should not be held liable for damages and attorney's fees.

WHEREFORE, the assailed Decision and Resolution of the Court of Appeals are
hereby AFFIRMED withMODIFICATION in the sense that the award of damages and attorney's
fees is deleted. Respondent Dieselman is ordered to return to petitioner AF Realty its partial
payment of P300,000.00. Costs against petitioners.

SO ORDERED.
G.R. No. 161886 March 16, 2007 In his aforesaid letter, Cruz requested the board to take necessary action/actions to recover from
those elected to the aforementioned positions the salaries they have received.
FILIPINAS PORT SERVICES, INC., represented by stockholders, ELIODORO C. CRUZ and
MINDANAO TERMINAL AND BROKERAGE SERVICES, INC., Petitioners, On 15 September 1992, the board met and took up Cruz’s letter. The records do not show what
vs. specific action/actions the board had taken on the letter. Evidently, whatever action/actions the
VICTORIANO S. GO, ARSENIO LOPEZ CHUA, EDGAR C. TRINIDAD, HERMENEGILDO M. board took did not sit well with Cruz.
TRINIDAD, JESUS SYBICO, MARY JEAN D. CO, HENRY CHUA, JOSELITO S. JAYME,
ERNESTO S. JAYME, and ELIEZER B. DE JESUS, Respondents.
On 14 June 1993, Cruz, purportedly in representation of Filport and its stockholders, among
which is herein co-petitioner Mindanao Terminal and Brokerage Services, Inc. (Minterbro), filed
DECISION with the SEC a petition3 which he describes as a derivative suit against the herein respondents
who were then the incumbent members of Filport’s Board of Directors, for alleged acts of
mismanagement detrimental to the interest of the corporation and its shareholders at large,
GARCIA, J.:
namely:

Assailed and sought to be set aside in this petition for review on certiorari is the Decision 1 dated
1. creation of an executive committee in 1991 composed of seven (7) members of the
19 January 2004 of the Court of Appeals (CA) in CA-G.R. CV No. 73827, reversing an earlier
board with compensation of ₱500.00 for each member per meeting, an office which, to
decision of the Regional Trial Court (RTC) of Davao City and accordingly dismissing the
Cruz, is not provided for in the by-laws of the corporation and whose function merely
derivative suit instituted by petitioner Eliodoro C. Cruz for and in behalf of the stockholders of co-
duplicates those of the President and General Manager;
petitioner Filipinas Port Services, Inc. (Filport, hereafter).

2. increase in the emoluments of the Chairman, Vice-President, Treasurer and


The case is actually an intra-corporate dispute involving Filport, a domestic corporation engaged
Assistant General Manager which increases are greatly disproportionate to the volume
in stevedoring services with principal office in Davao City. It was initially instituted with the
and character of the work of the directors holding said positions;
Securities and Exchange Commission (SEC) where the case hibernated and remained
unresolved for several years until it was overtaken by the enactment into law, on 19 July 2000,
of Republic Act (R.A.) No. 8799, otherwise known as the Securities Regulation Code. From the 3. re-creation of the positions of Assistant Vice-Presidents (AVPs) for Corporate
SEC and consistent with R.A. No. 8799, the case was transferred to the RTC of Manila, Branch Planning, Operations, Finance and Administration, and the election thereto of board
14, sitting as a corporate court. Subsequently, upon respondents’ motion, the case eventually members Edgar C. Trinidad, Eliezer de Jesus, Mary Jean D. Co and Henry Chua,
landed at the RTC of Davao City where it was docketed as Civil Case No. 28,552-2001. RTC- respectively; and
Davao City, Branch 10, ruled in favor of the petitioners prompting respondents to go to the CA in
CA-G.R. CV No. 73827. This time, the respondents prevailed, hence, this petition for review by
4. creation of the additional positions of Special Assistants to the President and the
the petitioners.
Board Chairman, with Fortunato V. de Castro and Arsenio Lopez Chua elected to the
same, the directors elected/appointed thereto not doing any work to deserve the
The relevant facts: monthly remuneration of ₱13,050.00 each.

On 4 September 1992, petitioner Eliodoro C. Cruz, Filport’s president from 1968 until he lost his In the same petition, docketed as SEC Case No. 06-93-4491, Cruz alleged that despite
bid for reelection as Filport’s president during the general stockholders’ meeting in 1991, wrote a demands made upon the respondent members of the board of directors to desist from creating
letter2 to the corporation’s Board of Directors questioning the board’s creation of the following the positions in question and to account for the amounts incurred in creating the same, the
positions with a monthly remuneration of ₱13,050.00 each, and the election thereto of certain demands were unheeded. Cruz thus prayed that the respondent members of the board of
members of the board, to wit: directors be made to pay Filport, jointly and severally, the sums of money variedly representing
the damages incurred as a result of the creation of the offices/positions complained of and the
aggregate amount of the questioned increased salaries.
Asst. Vice-President for Corporate Planning - Edgar C. Trinidad (Director)

In their common Answer with Counterclaim,4 the respondents denied the allegations of
Asst. Vice-President for Operations - Eliezer B. de Jesus (Director)
mismanagement and materially averred as follows:

Asst. Vice-President for Finance - Mary Jean D. Co (Director)


1. the creation of the executive committee and the grant of per diems for the
attendance of each member are allowed under the by-laws of the corporation;
Asst. Vice-President for Administration - Henry Chua (Director)
2. the increases in the salaries/emoluments of the Chairman, Vice-President,
Special Asst. to the Chairman - Arsenio Lopez Chua (Director) Treasurer and Assistant General Manager were well within the financial capacity of
the corporation and well-deserved by the officers elected thereto; and
Special Asst. to the President - Fortunato V. de Castro
3. the positions of AVPs for Corporate Planning, Operations, Finance and In its decision6 of 19 January 2004, the CA, taking exceptions to the findings of the trial court
Administration were already in existence during the tenure of Cruz as president of the that the creation of the positions of Assistant Vice President for Corporate Planning, Special
corporation, and were merely recreated by the Board, adding that all those appointed Assistant to the President and Special Assistant to the Board Chairman was merely for
to said positions of Assistant Vice Presidents, as well as the additional position of accommodation purposes, granted the respondents’ appeal, reversed and set aside the
Special Assistants to the Chairman and the President, rendered services to deserve appealed decision of the trial court and accordingly dismissed the so-called derivative suit filed
their compensation. by Cruz, et al., thus:

In the same Answer, respondents further averred that Cruz and his co-petitioner Minterbro, while IN VIEW OF ALL THE FOREGOING, the instant appeal is GRANTED, the challenged decision
admittedly stockholders of Filport, have no authority nor standing to bring the so-called is REVERSED and SET ASIDE, and a new one entered DISMISSING Civil Case No. 28,552-
"derivative suit" for and in behalf of the corporation; that respondent Mary Jean D. Co has 2001 with no pronouncement as to costs.
already ceased to be a corporate director and so with Fortunato V. de Castro, one of those
holding an assailed position; and that no demand to cease and desist from further committing
SO ORDERED.
the acts complained of was made upon the board. By way of affirmative defenses, respondents
asserted that (1) the petition is not duly verified by petitioner Filport which is the real party-in-
interest; (2) Filport, as represented by Cruz and Minterbro, failed to exhaust remedies for Intrigued, and quite understandably, by the fact that, in its decision, the CA, before proceeding to
redress within the corporation before bringing the suit; and (3) the petition does not show that address the merits of the appeal, prefaced its disposition with the statement reading "[T]he
the stockholders bringing the suit are joined as nominal parties. In support of their counterclaim, appeal is bereft of merit,"7 thereby contradicting the very fallo of its own decision and the
respondents averred that Cruz filed the alleged derivative suit in bad faith and purely for discussions made in the body thereof, respondents filed with the appellate court a Motion For
harassment purposes on account of his non-reelection to the board in the 1991 general Nunc Pro Tunc Order,8 thereunder praying that the phrase "[T]he appeal is bereft of merit," be
stockholders’ meeting. corrected to read "[T]he appeal is impressed with merit." In its resolution9 of 23 April 2004, the
CA granted the respondents’ motion and accordingly effected the desired correction.
As earlier narrated, the derivative suit (SEC Case No. 06-93-4491) hibernated with the SEC for a
long period of time. With the enactment of R.A. No. 8799, the case was first turned over to the Hence, petitioners’ present recourse.
RTC of Manila, Branch 14, sitting as a corporate court. Thereafter, on respondents’ motion, it
was eventually transferred to the RTC of Davao City whereat it was docketed as Civil Case No.
Petitioners assigned four (4) errors allegedly committed by the CA. For clarity, we shall formulate
28,552-2001 and raffled to Branch 10 thereof.
the issues as follows:

On 10 December 2001, RTC-Davao City rendered its decision5 in the case. Even as it found that
1. Whether the CA erred in holding that Filport’s Board of Directors acted within its
(1) Filport’s Board of Directors has the power to create positions not provided for in the by-laws
of the corporation since the board is the governing body; and (2) the increases in the salaries of powers in creating the executive committee and the positions of AVPs for Corporate
the board chairman, vice-president, treasurer and assistant general manager are reasonable, Planning, Operations, Finance and Administration, and those of the Special Assistants
the trial court nonetheless rendered judgment against the respondents by ordering the directors to the President and the Board Chairman, each with corresponding remuneration, and
holding the positions of Assistant Vice President for Corporate Planning, Special Assistant to the in increasing the salaries of the positions of Board Chairman, Vice-President,
President and Special Assistant to the Board Chairman to refund to the corporation the salaries Treasurer and Assistant General Manager; and
they have received as such officers "considering that Filipinas Port Services is not a big
corporation requiring multiple executive positions" and that said positions "were just created for 2. Whether the CA erred in finding that no evidence exists to prove that (a) the
accommodation." We quote the fallo of the trial court’s decision. positions of AVP for Corporate Planning, Special Assistant to the President and
Special Assistant to the Board Chairman were created merely for accommodation,
WHEREFORE, judgment is rendered ordering: and (b) the salaries/emoluments corresponding to said positions were actually paid to
and received by the directors appointed thereto.

Edgar C. Trinidad under the third and fourth causes of action to restore to the corporation the
total amount of salaries he received as assistant vice president for corporate planning; and For their part, respondents, aside from questioning the propriety of the instant petition as the
same allegedly raises only questions of fact and not of law, also put in issue the purported
likewise ordering Fortunato V. de Castro and Arsenio Lopez Chua under the fourth cause of
action to restore to the corporation the salaries they each received as special assistants derivative nature of the main suit initiated by petitioner Eliodoro C. Cruz allegedly in
respectively to the president and board chairman. In case of insolvency of any or all of them, the representation of and in behalf of Filport and its stockholders.
members of the board who created their positions are subsidiarily liable.
The petition is bereft of merit.
The counter claim is dismissed.
It is axiomatic that in petitions for review on certiorari under Rule 45 of the Rules of Court, only
From the adverse decision of the trial court, herein respondents went on appeal to the CA in CA- questions of law may be raised and passed upon by the Court. Factual findings of the CA are
G.R. CV No. 73827. binding and conclusive and will not be reviewed or disturbed on appeal. 10 Of course, the rule is
not cast in stone; it admits of certain exceptions, such as when the findings of fact of the
appellate court are at variance with those of the trial court, 11 as here. For this reason, and for a
proper and complete resolution of the case, we shall delve into the records and reexamine the xxx The Board of Directors shall fix the compensation of the officers and agents of the
same. corporation. (Emphasis supplied.)

The governing body of a corporation is its board of directors. Section 23 of the Corporation Unfortunately, the bylaws of the corporation are silent as to the creation by its board of directors
Code12 explicitly provides that unless otherwise provided therein, the corporate powers of all of an executive committee. Under Section 3515 of the Corporation Code, the creation of an
corporations formed under the Code shall be exercised, all business conducted and all property executive committee must be provided for in the bylaws of the corporation.
of the corporation shall be controlled and held by a board of directors. Thus, with the exception
only of some powers expressly granted by law to stockholders (or members, in case of non-
Notwithstanding the silence of Filport’s bylaws on the matter, we cannot rule that the creation of
stock corporations), the board of directors (or trustees, in case of non-stock corporations) has
the executive committee by the board of directors is illegal or unlawful. One reason is the
the sole authority to determine policies, enter into contracts, and conduct the ordinary business
absence of a showing as to the true nature and functions of said executive committee
of the corporation within the scope of its charter, i.e., its articles of incorporation, by-laws and
considering that the "executive committee," referred to in Section 35 of the Corporation Code
relevant provisions of law. Verily, the authority of the board of directors is restricted to the
which is as powerful as the board of directors and in effect acting for the board itself, should be
management of the regular business affairs of the corporation, unless more extensive power is
distinguished from other committees which are within the competency of the board to create at
expressly conferred.
anytime and whose actions require ratification and confirmation by the board.16 Another reason
is that, ratiocinated by both the two (2) courts below, the Board of Directors has the power to
The raison d’etre behind the conferment of corporate powers on the board of directors is not lost create positions not provided for in Filport’s bylaws since the board is the corporation’s
on the Court. Indeed, the concentration in the board of the powers of control of corporate governing body, clearly upholding the power of its board to exercise its prerogatives in managing
business and of appointment of corporate officers and managers is necessary for efficiency in the business affairs of the corporation.
any large organization. Stockholders are too numerous, scattered and unfamiliar with the
business of a corporation to conduct its business directly. And so the plan of corporate
As well, it may not be amiss to point out that, as testified to and admitted by petitioner Cruz
organization is for the stockholders to choose the directors who shall control and supervise the
himself, it was during his incumbency as Filport president that the executive committee in
conduct of corporate business.13
question was created, and that he was even the one who moved for the creation of the positions
of the AVPs for Operations, Finance and Administration. By his acquiescence and/or ratification
In the present case, the board’s creation of the positions of Assistant Vice Presidents for of the creation of the aforesaid offices, Cruz is virtually precluded from suing to declare such
Corporate Planning, Operations, Finance and Administration, and those of the Special acts of the board as invalid or illegal. And it makes no difference that he sues in behalf of himself
Assistants to the President and the Board Chairman, was in accordance with the regular and of the other stockholders. Indeed, as his voice was not heard in protest when he was still
business operations of Filport as it is authorized to do so by the corporation’s by-laws, pursuant Filport’s president, raising a hue and cry only now leads to the inevitable conclusion that he did
to the Corporation Code. so out of spite and resentment for his non-reelection as president of the corporation.

The election of officers of a corporation is provided for under Section 25 of the Code which With regard to the increased emoluments of the Board Chairman, Vice-President, Treasurer and
reads: Assistant General Manager which are supposedly disproportionate to the volume and nature of
their work, the Court, after a judicious scrutiny of the increase vis-à-vis the value of the services
rendered to the corporation by the officers concerned, agrees with the findings of both the trial
Sec. 25. Corporate officers, quorum. – Immediately after their election, the directors of a
and appellate courts as to the reasonableness and fairness thereof.
corporation must formally organize by the election of a president, who shall be a director, a
treasurer who may or may not be a director, a secretary who shall be a resident and citizen of
the Philippines, and such other officers as may be provided for in the by-laws. (Emphasis Continuing, petitioners contend that the CA did not appreciate their evidence as to the alleged
supplied.) acts of mismanagement by the then incumbent board. A perusal of the records, however,
reveals that petitioners merely relied on the testimony of Cruz in support of their bold claim of
mismanagement. To the mind of the Court, Cruz’ testimony on the matter of mismanagement is
In turn, the amended Bylaws of Filport14 provides the following:
bereft of any foundation. As it were, his testimony consists merely of insinuations of alleged
wrongdoings on the part of the board. Without more, petitioners’ posture of mismanagement
Officers of the corporation, as provided for by the by-laws, shall be elected by the board of must fall and with it goes their prayer to hold the respondents liable therefor.
directors at their first meeting after the election of Directors. xxx
But even assuming, in gratia argumenti, that there was mismanagement resulting to corporate
The officers of the corporation shall be a Chairman of the Board, President, a Vice-President, a damages and/or business losses, still the respondents may not be held liable in the absence, as
Secretary, a Treasurer, a General Manager and such other officers as the Board of Directors here, of a showing of bad faith in doing the acts complained of.
may from time to time provide, and these officers shall be elected to hold office until their
successors are elected and qualified. (Emphasis supplied.)
If the cause of the losses is merely error in business judgment, not amounting to bad faith or
negligence, directors and/or officers are not liable.17 For them to be held accountable, the
Likewise, the fixing of the corresponding remuneration for the positions in question is provided mismanagement and the resulting losses on account thereof are not the only matters to be
for in the same by-laws of the corporation, viz: proven; it is likewise necessary to show that the directors and/or officers acted in bad faith and
with malice in doing the assailed acts. Bad faith does not simply connote bad judgment or
negligence; it imports a dishonest purpose or some moral obliquity and conscious doing of a
wrong, a breach of a known duty through some motive or interest or ill-will partaking of the
nature of fraud.18 We have searched the records and nowhere do we find a "dishonest purpose" So it is that in Philippine Stock Exchange, Inc. v. CA,21 the Court unequivocally held:
or "some moral obliquity," or "conscious doing of a wrong" on the part of the respondents that
"partakes of the nature of fraud."
Questions of policy or of management are left solely to the honest decision of the board as the
business manager of the corporation, and the court is without authority to substitute its judgment
We thus extend concurrence to the following findings of the CA, affirmatory of those of the trial for that of the board, and as long as it acts in good faith and in the exercise of honest judgment
court: in the interest of the corporation, its orders are not reviewable by the courts.

xxx As a matter of fact, it was during the term of appellee Cruz, as president and director, that In a last-ditch attempt to salvage their cause, petitioners assert that the CA went beyond the
the executive committee was created. What is more, it was appellee himself who moved for the issues raised in the court of origin when it ruled on the absence of receipt of actual payment of
creation of the positions of assistant vice presidents for operations, for finance, and for the salaries/emoluments pertaining to the positions of Assistant Vice-President for Corporate
administration. He should not be heard to complain thereafter for similar corporate acts. Planning, Special Assistant to the Board Chairman and Special Assistant to the President.
Petitioners insist that the issue of nonpayment was never raised by the respondents before the
trial court, as in fact, the latter allegedly admitted the same in their Answer With Counterclaim.
The increase in the salaries of the board chairman, president, treasurer, and assistant general
manager are indeed reasonable enough in view of the responsibilities assigned to them, and the
special knowledge required, to be able to effectively discharge their respective functions and We are not persuaded.
duties.
By claiming that Filport suffered damages because the directors appointed to the assailed
Surely, factual findings of trial courts, especially when affirmed by the CA, are binding and positions are not doing anything to deserve their compensation, petitioners are saddled with the
conclusive on this Court. burden of proving that salaries were actually paid. Since the trial court, in effect, found that the
petitioners successfully proved payment of the salaries when it directed the reimbursements of
the same, respondents necessarily have to raise the issue on appeal. And the CA rightly
There is, however, a factual matter over which the CA and the trial court parted ways. We refer
resolved the issue when it found that no evidence of actual payment of the salaries in question
to the accommodation angle.
was actually adduced. Respondents’ alleged admission of the fact of payment cannot be inferred
from a reading of the pertinent portions of the parties’ respective initiatory pleadings.
The trial court was with petitioner Cruz in saying that the creation of the positions of the three (3) Respondents’ allegations in their Answer With Counterclaim that the officers corresponding to
AVPs for Corporate Planning, Special Assistant to the President and Special Assistant to the the positions created "performed the work called for in their positions" or "deserve their
Board Chairman, each with a salary of ₱13,050.00 a month, was merely for accommodation compensation," cannot be interpreted to mean that they were "actually paid" such
purposes considering that Filport is not a big corporation requiring multiple executive positions. compensation. Directly put, the averment that "one deserves one’s compensation" does not
Hence, the trial court’s order for said officers to return the amounts they received as necessarily carry the implication that "such compensation was actually remitted or received."
compensation. And because payment was not duly proven, there is no evidentiary or factual basis for the trial
court to direct respondents to make reimbursements thereof to the corporation.
On the other hand, the CA took issue with the trial court and ruled that Cruz’s accommodation
theory is not based on facts and without any evidentiary substantiation. This brings us to the respondents’ claim that the case filed by the petitioners before the SEC,
which eventually landed in RTC-Davao City as Civil Case No. 28,552-2001, is not a derivative
suit, as maintained by the petitioners.
We concur with the line of the appellate court. For truly, aside from Cruz’s bare and self-serving
testimony, no other evidence was presented to show the fact of "accommodation." By itself, the
testimony of Cruz is not enough to support his claim that accommodation was the underlying We sustain the petitioners.
factor behind the creation of the aforementioned three (3) positions.
Under the Corporation Code, where a corporation is an injured party, its power to sue is lodged
It is elementary in procedural law that bare allegations do not constitute evidence adequate to with its board of directors or trustees. But an individual stockholder may be permitted to institute
support a conclusion. It is basic in the rule of evidence that he who alleges a fact bears the a derivative suit in behalf of the corporation in order to protect or vindicate corporate rights
burden of proving it by the quantum of proof required. Bare allegations, unsubstantiated by whenever the officials of the corporation refuse to sue, or when a demand upon them to file the
evidence, are not equivalent to proof under the Rules of Court. 19 The party having the burden of necessary action would be futile because they are the ones to be sued, or because they hold
proof must establish his case by a preponderance of evidence. 20 control of the corporation.22 In such actions, the corporation is the real party-in-interest while the
suing stockholder, in behalf of the corporation, is only a nominal party.23
Besides, the determination of the necessity for additional offices and/or positions in a corporation
is a management prerogative which courts are not wont to review in the absence of any proof Here, the action below is principally for damages resulting from alleged mismanagement of the
that such prerogative was exercised in bad faith or with malice.1awphi1.nét affairs of Filport by its directors/officers, it being alleged that the acts of mismanagement are
detrimental to the interests of Filport. Thus, the injury complained of primarily pertains to the
corporation so that the suit for relief should be by the corporation. However, since the ones to be
Indeed, it would be an improper judicial intrusion into the internal affairs of Filport were the Court sued are the directors/officers of the corporation itself, a stockholder, like petitioner Cruz, may
to determine the propriety or impropriety of the creation of offices therein and the grant of salary
validly institute a "derivative suit" to vindicate the alleged corporate injury, in which case Cruz is
increases to officers thereof. Such are corporate and/or business decisions which only the
corporation’s Board of Directors can determine.
only a nominal party while Filport is the real party-in-interest. For sure, in the prayer portion of
petitioners’ petition before the SEC, the reliefs prayed were asked to be made in favor of Filport.

Besides, the requisites before a derivative suit can be filed by a stockholder are present in this
case, to wit:

a) the party bringing suit should be a shareholder as of the time of the act or
transaction complained of, the number of his shares not being material;

b) he has tried to exhaust intra-corporate remedies, i.e., has made a demand on the
board of directors for the appropriate relief but the latter has failed or refused to heed
his plea; and

c) the cause of action actually devolves on the corporation, the wrongdoing or harm
having been, or being caused to the corporation and not to the particular stockholder
bringing the suit.24

Indisputably, petitioner Cruz (1) is a stockholder of Filport; (2) he sought without success to have
its board of directors remedy what he perceived as wrong when he wrote a letter requesting the
board to do the necessary action in his complaint; and (3) the alleged wrong was in truth a
wrong against the stockholders of the corporation generally, and not against Cruz or Minterbro,
in particular. In the end, it is Filport, not Cruz which directly stands to benefit from the suit. And
while it is true that the complaining stockholder must show to the satisfaction of the court that he
has exhausted all the means within his reach to attain within the corporation itself the redress for
his grievances, or actions in conformity to his wishes, nonetheless, where the corporation is
under the complete control of the principal defendants, as here, there is no necessity of making
a demand upon the directors. The reason is obvious: a demand upon the board to institute an
action and prosecute the same effectively would have been useless and an exercise in futility. In
fine, we rule and so hold that the petition filed with the SEC at the instance of Cruz, which
ultimately found its way to the RTC of Davao City as Civil Case No. 28,552-2001, is a derivative
suit of which Cruz has the necessary legal standing to institute.

WHEREFORE, the petition is DENIED and the challenged decision of the CA is AFFIRMED in
all respects.

No pronouncement as to costs.

SO ORDERED.
G.R. No. 125778 June 10, 2003 net worth shortfall thus amounted to P13,244,225.00 after adding the net worth deficiency of
P1,244,225.00 to the Minimum Guaranteed Net Worth of P12,000,000.00.
INTER-ASIA INVESTMENTS INDUSTRIES, INC., Petitioner,
vs. The adjusted contract price, therefore, amounted to P6,225,775.00 which is the difference
COURT OF APPEALS and ASIA INDUSTRIES, INC., Respondents. between the contract price of P19,500,000.00 and the shortfall in the guaranteed net worth of
P13,224,225.00. Private respondent having already paid petitioner P12,000,000.00, it was
entitled to a refund of P5,744,225.00.
DECISION

Petitioner thereafter proposed, by letter13 of January 24, 1980, signed by its president, that
CARPIO-MORALES, J.:
private respondent’s claim for refund be reduced to P4,093,993.00, it promising to pay the cost
of the Northern Cotabato Industries, Inc. (NOCOSII) superstructures in the amount of
The present petition for review on certiorari assails the Court of Appeals Decision1 of January 25, P759,570.00. To the proposal respondent agreed. Petitioner, however, weiched on its promise.
1996 and Resolution2 of July 11, 1996. Petitioner’s total liability thus stood at P4,853,503.00 (P4,093,993.00 plus
P759,570.00)14exclusive of interest.15
The material facts of the case are as follows:
On April 5, 1983, private respondent filed a complaint16 against petitioner with the Regional Trial
Court of Makati, one of two causes of action of which was for the recovery of above-said amount
On September 1, 1978, Inter-Asia Industries, Inc. (petitioner), by a Stock Purchase of P4,853,503.0017 plus interest.
Agreement3 (the Agreement), sold to Asia Industries, Inc. (private respondent) for and in
consideration of the sum of P19,500,000.00 all its right, title and interest in and to all the
outstanding shares of stock of FARMACOR, INC. (FARMACOR). 4 The Agreement was signed Denying private respondent’s claim, petitioner countered that private respondent failed to pay
by Leonides P. Gonzales and Jesus J. Vergara, presidents of petitioner and private respondent, the balance of the purchase price and accordingly set up a counterclaim.
respectively.5
Finding for private respondent, the trial court rendered on November 27, 1991 a Decision, 18 the
Under paragraph 7 of the Agreement, petitioner as seller made warranties and representations dispositive portion of which reads:
among which were "(iv.) [t]he audited financial statements of FARMACOR at and for the year
ended December 31, 1977... and the audited financial statements of FARMACOR as of
WHEREFORE, judgment is rendered in favor of plaintiff and against defendant (a) ordering the
September 30, 1978 being prepared by S[ycip,] G[orres,] V[elayo and Co.]... fairly present or will
latter to pay to the former the sum of P4,853,503.0019 plus interest thereon at the legal rate from
present the financial position of FARMACOR and the results of its operations as of said
the filing of the complaint until fully paid, the sum of P30,000.00 as attorney’s fees and the costs
respective dates; said financial statements show or will show all liabilities and commitments of
of suit; and (b) dismissing the counterclaim.
FARMACOR, direct or contingent, as of said respective dates . . ."; and "(v.) [t]he Minimum
Guaranteed Net Worth of FARMACOR as of September 30, 1978 shall be Twelve Million Pesos
(P12,000,000.00)."6 SO ORDERED.

The Agreement was later amended with respect to the "Closing Date," originally set up at 10:00 On appeal to the Court of Appeals, petitioner raised the following errors:
a.m. of September 30, 1978, which was moved to October 31, 1978, and to the mode of
payment of the purchase price.7
THE TRIAL COURT ERRED IN HOLDING THE DEFENDANT LIABLE UNDER THE
FIRST CAUSE OF ACTION PLEADED BY THE PLAINTIFF.
The Agreement, as amended, provided that pending submission by SGV of FARMACOR’s
audited financial statements as of October 31, 1978, private respondent may retain the sum of
THE TRIAL COURT ERRED IN AWARDING ATTORNEY’S FEES AND IN
P7,500,000.00 out of the stipulated purchase price of P19,500,000.00; that from this retained
DISMISSING THE COUNTERCLAIM.
amount of P7,500,000.00, private respondent may deduct any shortfall on the Minimum
Guaranteed Net Worth of P12,000,000.00;8 and that if the amount retained is not sufficient to
make up for the deficiency in the Minimum Guaranteed Net Worth, petitioner shall pay the THE TRIAL COURT ERRED IN RENDERING JUDGMENT IN FAVOR OF THE
difference within 5 days from date of receipt of the audited financial statements.9 PLAINTIFF, THE ALLEGED BREACH OF WARRANTIES AND REPRESENTATION
NOT HAVING BEEN SHOWN, MUCH LESS ESTABLISHED BY THE PLAINTIFF. 20
Respondent paid petitioner a total amount of P 12,000,000.00: P5,000,000.00 upon the signing
of the Agreement, and P7,000,000.00 on November 2, 1978. 10 By Decision of January 25, 1996, the Court of Appeals affirmed the trial court’s decision.
Petitioner’s motion for reconsideration of the decision having been denied by the Court of
Appeals by Resolution of July 11, 1996, the present petition for review on certiorari was filed,
From the STATEMENT OF INCOME AND DEFICIT attached to the financial report 11 dated
assigning the following errors:
November 28, 1978 submitted by SGV, it appears that FARMACOR had, for the ten months
ended October 31, 1978, a deficit of P11,244,225.00.12 Since the stockholder’s equity amounted
to P10,000,000.00, FARMACOR had a net worth deficiency of P1,244,225.00. The guaranteed I
THE RESPONDENT COURT ERRED IN NOT HOLDING THAT THE LETTER OF THE authority of such individuals to bind the corporation is generally derived from law,
PRESIDENT OF THE PETITIONER IS NOT BINDING ON THE PETITIONER corporate bylaws or authorization from the board, either expressly or impliedly by habit,
BEING ULTRA VIRES. custom or acquiescence in the general course of business, viz:

II A corporate officer or agent may represent and bind the corporation in transactions with third
persons to the extent that [the] authority to do so has been conferred upon him, and this includes
powers as, in the usual course of the particular business, are incidental to, or may be implied
THE LETTER CAN NOT BE AN ADMISSION AND WAIVER OF THE PETITIONER AS A
from, the powers intentionally conferred, powers added by custom and usage, as usually
CORPORATION.
pertaining to the particular officer or agent, and such apparent powers as the corporation has
caused person dealing with the officer or agent to believe that it has conferred.
III
xxx
THE RESPONDENT COURT ERRED IN NOT DECLARING THAT THERE IS NO BREACH OF
WARRANTIES AND REPRESENTATION AS ALLEGED BY THE PRIVATE RESPONDENT.
[A]pparent authority is derived not merely from practice. Its existence may be ascertained
through (1) the general manner in which the corporation holds out an officer or agent as having
IV the power to act or, in other words the apparent authority to act in general, with which it clothes
him; or (2) the acquiescence in his acts of a particular nature, with actual or constructive
knowledge thereof, within or beyond the scope of his ordinary powers.
THE RESPONDENT COURT ERRED IN ORDERING THE PETITIONER TO PAY It requires presentation of evidence of similar act(s) executed either in its favor or
ATTORNEY’S FEES AND IN SUSTAINING THE DISMISSAL OF THE COUNTERCLAIM. 1 8 in favor of other parties. It is not the quantity of similar acts which establishes
(Underscoring in the original) apparent authority, but the vesting of
a corporate officer with power to bind the corporation.
Petitioner argues that the January 24, 1980 letter-proposal (for the reduction of private
respondent’s claim for refund upon petitioner’s promise to pay the cost of NOCOSII x x x (Emphasis and underscoring supplied)
superstructures in the amount of P759,570.00) which was signed by its president has no legal
force and effect against it as it was not authorized by its board of directors, it citing the
Corporation Law which provides that unless the act of the president is authorized by the board of As correctly argued by private respondent, an officer of a corporation who is authorized to
directors, the same is not binding on it. purchase the stock of another corporation has the implied power to perform all other obligations
arising therefrom, such as payment of the shares of stock. By allowing its president to sign the
Agreement on its behalf, petitioner clothed him with apparent capacity to perform all acts which
This Court is not persuaded. are expressly, impliedly and inherently stated therein.21

The January 24, 1980 letter signed by petitioner’s president is valid and binding. The case
Petitioner further argues that when the Agreement was executed on September 1, 1978, its
of People’s Aircargo and Warehousing Co., Inc. v. Court of Appeals19 instructs: financial statements were extensively examined and accepted as correct by private respondent,
hence, it cannot later be disproved "by resorting to some scheme such as future financial
The general rule is that, in the absence of authority from the board of directors, no auditing;"22 and that it should not be bound by the SGV Report because it is self-serving and
person, not even its officers, can validly bind a corporation. A corporation is a juridical biased, SGV having been hired solely by private respondent, and the alleged shortfall of
person, separate and distinct from its stockholders and members, "having x x x powers, FARMACOR occurred only after the execution of the Agreement.
attributes and properties expressly authorized by law or incident to its existence."
This Court is not persuaded either.
Being a juridical entity, a corporation may act through its board of directors, which exercises
almost all corporate powers, lays down all corporate business policies and is responsible for the The pertinent provisions of the Agreement read:
efficiency of management, as provided in Section 23 of the Corporation Code of the Philippines:

7. Warranties and Representations - (a) SELLER warrants and represents as follows:


SEC. 23. The Board of Directors or Trustees. - Unless otherwise provided in this Code, the
corporate powers of all corporations formed under this Code shall be exercised, all business
conducted and all property of such corporations controlled and held by the board of directors or xxx
trustees x x x.
(iv) The audited financial statements of FARMACOR as at and for the year ended December 31,
Under this provision, the power and responsibility to decide whether the corporation should enter 1977 and
into a contract that will bind the corporation is lodged in the board, subject to the articles of the audited financial statements of FARMACOR as at September 30, 1978 being prepared
incorporation, bylaws, or relevant provisions of law. However, just as a natural person may by SGV pursuant toparagraph 6(b) fairly present or will present the financial position of F
authorize another to do certain acts for and on his behalf, the board of directors may ARMACOR and the results of its operations as of said respective dates; said financial
validly delegate some of its functions and powers to officers, committees or agents.The statements show or will show all liabilities and commitments of FARMACOR, direct or
contingent, as of said respective dates; and the receivables set forth in said financial
statements are fully due and collectible, free and clear of any set-offs, defenses, claims and
other impediments to their collectibility.

(v) The Minimum Guaranteed Net Worth of FARMACOR as of September 30, 1978 shall be
Twelve Million Pesos (P12,000,000.00), Philippine Currency.1âwphi1

x x x (Underscoring in the original; emphasis supplied)23

True, private respondent accepted as correct the financial statements submitted to it when the
Agreement was executed on September 1, 1978. But petitioner expressly warranted that the
SGV Reports "fairly present or will present the financial position of FARMACOR." By such
warranty, petitioner is estopped from claiming that the SGV Reports are self-serving and
biased.1âwphi1

As to the claim that the shortfall occurred after the execution of the Agreement, the declaration
of Emmanuel de Asis, supervisor in the Accounting Division of SGV and head of the team which
conducted the auditing of FARMACOR, that the period covered by the audit was from January to
October 1978 shows that the period beforethe Agreement was entered into (on September 1,
1978) was covered.24

As to petitioner’s assigned error on the award of attorney’s fees which, it argues, is bereft of
factual, legal and equitable justification, this Court finds the same well-taken.

On the matter of attorney’s fees, it is an accepted doctrine that the award thereof as an item of
damages is the exception rather than the rule, and counsel’s fees are not to be awarded every
time a party wins a suit. The powerof the court to award
attorney’s fees under Article 2208 of the Civil Code demands
factual, legal and equitable justification, without which the award is
a conclusion without a premise, its basis being improperly left
to speculation and conjecture. In all events, the court must explicitly state in the text of
the decision, and not only in the decretal portion thereof, the legal reason for the award of
attorney’s fees.25

x x x (Emphasis and underscoring supplied; citations omitted)

WHEREFORE, the instant petition is PARTLY GRANTED. The assailed decision of the Court of
Appeals affirming that of the trial court is modified in that the award of attorney’s fees in favor of
private respondent is deleted. The decision is affirmed in other respects.

SO ORDERED.
G.R. No. 126006 January 29, 2004 For his part, petitioner Tan admitted that he contracted the loans from the respondent Bank in
his personal capacity. The parties, however, agreed that the loans were to be paid from the
proceeds of petitioner Tan’s shares of common stocks in the Lapulapu Industries Corporation, a
LAPULAPU FOUNDATION, INC. and ELIAS Q. TAN, Petitioners,
real estate firm. The loans were covered by promissory notes which were automatically
vs.
renewable ("rolled-over") every year at an amount including unpaid interests, until such time as
COURT OF APPEALS (Seventeenth Division) and ALLIED BANKING CORP., Respondents.
petitioner Tan was able to pay the same from the proceeds of his aforesaid shares.

DECISION
According to petitioner Tan, the respondent Bank’s employee required him to affix two
signatures on every promissory note, assuring him that the loan documents would be filled out in
CALLEJO, SR., J.: accordance with their agreement. However, after he signed and delivered the loan documents to
the respondent Bank, these were filled out in a manner not in accord with their agreement, such
that the petitioner Foundation was included as party thereto. Further, prior to its filing of the
Before the Court is the petition for review on certiorari filed by the Lapulapu Foundation, Inc. and complaint, the respondent Bank made no demand on him.
Elias Q. Tan seeking to reverse and set aside the Decision1 dated June 26, 1996 of the Court of
Appeals (CA) in CA-G.R. CV No. 37162 ordering the petitioners, jointly and solidarily, to pay the
respondent Allied Banking Corporation the amount of ₱493,566.61 plus interests and other After due trial, the court a quo rendered judgment the dispositive portion of which reads:
charges. Likewise, sought to be reversed and set aside is the appellate court’s Resolution dated
August 19, 1996 denying the petitioners’ motion for reconsideration.
WHEREFORE, in view of the foregoing evidences [sic], arguments and considerations, this court
hereby finds the preponderance of evidence in favor of the plaintiff and hereby renders judgment
The case stemmed from the following facts: as follows:

Sometime in 1977, petitioner Elias Q. Tan, then President of the co-petitioner Lapulapu "1. Requiring the defendants Elias Q. Tan and Lapulapu Foundation, Inc. [the
Foundation, Inc., obtained four loans from the respondent Allied Banking Corporation covered by petitioners herein] to pay jointly and solidarily to the plaintiff Allied Banking Corporation
four promissory notes in the amounts of ₱100,000 each. The details of the promissory notes are [the respondent herein] the amount of ₱493,566.61 as principal obligation for the four
as follows: promissory notes, including all other charges included in the same, with interest at
14% per annum, computed from January 24, 1979, until the same are fully paid, plus
2% service charges and 1% monthly penalty charges.
P/N No. Date of P/N Maturity Date Amount as of 1/23/79
"2. Requiring the defendants Elias Q. Tan and Lapulapu Foundation, Inc., to pay
BD No. 504 Nov. 7, 1977 Feb. 5, 1978 ₱123,377.76 jointly and solidarily, attorney’s fees in the equivalent amount of 25% of the total
amount due from the defendants on the promissory notes, including all charges;
BD No. 621 Nov. 28, 1977 Mar. 28, 1978 ₱123,411.10

BD No. 716 Dec. 12, 1977 Apr. 11, 1978 ₱122,322.21 "3. Requiring the defendants Elias Q. Tan and Lapulapu Foundation, Inc., to pay
jointly and solidarily litigation expenses of ₱1,000.00 plus costs of the suit." 3
BD No. 839 Jan. 5, 1978 May 5, 1978 ₱120,455.54 2

On appeal, the CA affirmed with modification the judgment of the court a quo by deleting the
award of attorney’s fees in favor of the respondent Bank for being without basis.
As of January 23, 1979, the entire obligation amounted to ₱493,566.61 and despite demands
made on them by the respondent Bank, the petitioners failed to pay the same. The respondent
Bank was constrained to file with the Regional Trial Court of Cebu City, Branch 15, a complaint The appellate court disbelieved petitioner Tan’s claim that the loans were his personal loans as
seeking payment by the petitioners, jointly and solidarily, of the sum of ₱493,566.61 the promissory notes evidencing them showed upon their faces that these were obligations of
representing their loan obligation, exclusive of interests, penalty charges, attorney’s fees and the petitioner Foundation, as contracted by petitioner Tan himself in his "official and personal
costs. character." Applying the parol evidence rule, the CA likewise rejected petitioner Tan’s assertion
that there was an unwritten agreement between him and the respondent Bank that he would pay
the loans from the proceeds of his shares of stocks in the Lapulapu Industries Corp.
In its answer to the complaint, the petitioner Foundation denied incurring indebtedness from the
respondent Bank alleging that the loans were obtained by petitioner Tan in his personal
capacity, for his own use and benefit and on the strength of the personal information he Further, the CA found that demand had been made by the respondent Bank on the petitioners
furnished the respondent Bank. The petitioner Foundation maintained that it never authorized prior to the filing of the complaint a quo. It noted that the two letters of demand dated January 3,
petitioner Tan to co-sign in his capacity as its President any promissory note and that the 19794 and January 30, 19795 asking settlement of the obligation were sent by the respondent
respondent Bank fully knew that the loans contracted were made in petitioner Tan’s personal Bank. These were received by the petitioners as shown by the registry return cards6 presented
capacity and for his own use and that the petitioner Foundation never benefited, directly or during trial in the court a quo.
indirectly, therefrom. The petitioner Foundation then interposed a cross-claim against petitioner
Tan alleging that he, having exceeded his authority, should be solely liable for said loans, and a
Finally, like the court a quo, the CA applied the doctrine of piercing the veil of corporate entity in
counterclaim against the respondent Bank for damages and attorney’s fees.
holding the petitioners jointly and solidarily liable. The evidence showed that petitioner Tan had
represented himself as the President of the petitioner Foundation, opened savings and current Having established that the loans had become due and demandable, the Court shall now
accounts in its behalf, and signed the loan documents for and in behalf of the latter. The CA, resolve the issue of whether the CA correctly held the petitioners jointly and solidarily liable
likewise, found that the petitioner Foundation had allowed petitioner Tan to act as though he had therefor.
the authority to contract the loans in its behalf. On the other hand, petitioner Tan could not
escape liability as he had used the petitioner Foundation for his benefit.
In disclaiming any liability for the loans, the petitioner Foundation maintains that these were
contracted by petitioner Tan in his personal capacity and that it did not benefit therefrom. On the
Aggrieved, the petitioners now come to the Court alleging that: other hand, while admitting that the loans were his personal obligation, petitioner Tan avers that
he had an unwritten agreement with the respondent Bank that these loans would be renewed on
a year-to-year basis and paid from the proceeds of his shares of stock in the Lapulapu Industries
I. THE COURT OF APPEALS GRAVELY ERRED IN HOLDING THAT THE LOANS
Corp.
SUBJECT MATTER OF THE INSTANT PETITION ARE ALREADY DUE AND
DEMANDABLE DESPITE ABSENCE OF PRIOR DEMAND.
These contentions are untenable.
II. THE COURT OF APPEALS GRAVELY ERRED IN APPLYING THE PAROL
EVIDENCE RULE AND THE DOCTRINE OF PIERCING THE VEIL OF CORPORATE The Court particularly finds as incredulous petitioner Tan’s allegation that he was made to sign
ENTITY AS BASIS FOR ADJUDGING JOINT AND SOLIDARY LIABILITY ON THE blank loan documents and that the phrase "IN MY OFFICIAL/PERSONAL CAPACITY" was
PART OF PETITIONERS ELIAS Q. TAN AND LAPULAPU FOUNDATION, INC.7 superimposed by the respondent Bank’s employee despite petitioner Tan’s protestation. The
Court is hard pressed to believe that a businessman of petitioner Tan’s stature could have been
so careless as to sign blank loan documents.
The petitioners assail the appellate court’s finding that the loans had become due and
demandable in view of the two demand letters sent to them by the respondent Bank. The
petitioners insist that there was no prior demand as they vigorously deny receiving those letters. In contrast, as found by the CA, the promissory notes11 clearly showed upon their faces that they
According to petitioner Tan, the signatures on the registry return cards were not his. are the obligation of the petitioner Foundation, as contracted by petitioner Tan "in his official and
personal capacity."12 Moreover, the application for credit accommodation,13 the signature cards
of the two accounts in the name of petitioner Foundation,14 as well as New Current Account
The petitioners’ denial of receipt of the demand letters was rightfully given scant consideration
Record,15 all accompanying the promissory notes, were signed by petitioner Tan for and in the
by the CA as it held:
name of the petitioner Foundation.16 These documentary evidence unequivocally and
categorically establish that the loans were solidarily contracted by the petitioner Foundation and
Exhibits "R" and "S" are two letters of demand, respectively dated January 3, 1979 and January petitioner Tan.
30, 1979, asking settlement of the obligations covered by the promissory notes. The first letter
was written by Ben Tio Peng Seng, Vice-President of the bank, and addressed to Lapulapu
As a corollary, the parol evidence rule likewise constrains this Court to reject petitioner Tan’s
Foundation, Inc., attention of Mr. Elias Q. Tan, President, while the second was a final demand
claim regarding the purported unwritten agreement between him and the respondent Bank on
written by the appellee’s counsel, addressed to both defendants-appellants, and giving them five
the payment of the obligation. Section 9, Rule 130 of the of the Revised Rules of Court provides
(5) days from receipt within which to settle or judicial action would be instituted against them.
that "[w]hen the terms of an agreement have been reduced to writing, it is to be considered as
Both letters were duly received by the defendants, as shown by the registry return cards, marked
containing all the terms agreed upon and there can be, between the parties and their
as Exhibits "R-2" and "S-1," respectively. The allegation of Tan that he does not know who
successors-in-interest, no evidence of such terms other than the contents of the written
signed the said registry return receipts merits scant consideration, for there is no showing that
agreement."17
the addresses thereon were wrong. Hence, the disputable presumption "that a letter duly
directed and mailed was received in the regular course of mail" (per par. V, Section 3, Rule 131
of the Revised Rules on Evidence) still holds.8 In this case, the promissory notes are the law between the petitioners and the respondent Bank.
These promissory notes contained maturity dates as follows: February 5, 1978, March 28, 1978,
April 11, 1978 and May 5, 1978, respectively. That these notes were to be paid on these dates is
There is no dispute that the promissory notes had already matured. However, the petitioners
clear and explicit. Nowhere was it stated therein that they would be renewed on a year-to-year
insist that the loans had not become due and demandable as they deny receipt of the
basis or "rolled-over" annually until paid from the proceeds of petitioner Tan’s shares in the
respondent Bank’s demand letters. When presented the registry return cards during the trial,
Lapulapu Industries Corp. Accordingly, this purported unwritten agreement could not be made to
petitioner Tan claimed that he did not recognize the signatures thereon. The petitioners’
vary or contradict the terms and conditions in the promissory notes.
allegation and denial are self-serving. They cannot prevail over the registry return cards which
constitute documentary evidence and which enjoy the presumption that, absent clear and
convincing evidence to the contrary, these were regularly issued by the postal officials in the Evidence of a prior or contemporaneous verbal agreement is generally not admissible to vary,
performance of their official duty and that they acted in good faith. 9 Further, as the CA correctly contradict or defeat the operation of a valid contract.18 While parol evidence is admissible to
opined, mails are presumed to have been properly delivered and received by the addressee "in explain the meaning of written contracts, it cannot serve the purpose of incorporating into the
the regular course of the mail."10 As the CA noted, there is no showing that the addresses on the contract additional contemporaneous conditions which are not mentioned at all in writing, unless
registry return cards were wrong. It is the petitioners’ burden to overcome the presumptions by there has been fraud or mistake.19 No such allegation had been made by the petitioners in this
sufficient evidence, and other than their barefaced denial, the petitioners failed to support their case.
claim that they did not receive the demand letters; therefore, no prior demand was made on
them by the respondent Bank.
Finally, the appellate court did not err in holding the petitioners jointly and solidarily liable as it
applied the doctrine of piercing the veil of corporate entity. The petitioner Foundation asserts that
it has a personality separate and distinct from that of its President, petitioner Tan, and that it
cannot be held solidarily liable for the loans of the latter.1âwphi1

The Court agrees with the CA that the petitioners cannot hide behind the corporate veil under
the following circumstances:

The evidence shows that Tan has been representing himself as the President of Lapulapu
Foundation, Inc. He opened a savings account and a current account in the names of the
corporation, and signed the application form as well as the necessary specimen signature cards
(Exhibits "A," "B" and "C") twice, for himself and for the foundation. He submitted a notarized
Secretary’s Certificate (Exhibit "G") from the corporation, attesting that he has been authorized,
inter alia, to sign for and in behalf of the Lapulapu Foundation any and all checks, drafts or other
orders with respect to the bank; to transact business with the Bank, negotiate loans,
agreements, obligations, promissory notes and other commercial documents; and to initially
obtain a loan for ₱100,000.00 from any bank (Exhibits "G-1" and "G-2"). Under these
circumstances, the defendant corporation is liable for the transactions entered into by Tan on its
behalf.20

Per its Secretary’s Certificate, the petitioner Foundation had given its President, petitioner Tan,
ostensible and apparent authority to inter alia deal with the respondent Bank. Accordingly, the
petitioner Foundation is estopped from questioning petitioner Tan’s authority to obtain the
subject loans from the respondent Bank. It is a familiar doctrine that if a corporation knowingly
permits one of its officers, or any other agent, to act within the scope of an apparent authority, it
holds him out to the public as possessing the power to do those acts; and thus, the corporation
will, as against anyone who has in good faith dealt with it through such agent, be estopped from
denying the agent’s authority.21

In fine, there is no cogent reason to deviate from the CA’s ruling that the petitioners are jointly
and solidarily liable for the loans contracted with the respondent Bank.

WHEREFORE, premises considered, the petition is DENIED and the Decision dated June 26,
1996 and Resolution dated August 19, 1996 of the Court of Appeals in CA-G.R. CV No. 37162
are AFFIRMED in toto.

SO ORDERED.
G.R. No.176897 December 11, 2013 Advance Paper presented the checks to the drawee bank but these were dishonored either for
"insufficiency of funds" or "account closed." Despite repeated demands, however, Arma Traders
failed to settle its account with Advance Paper.16
ADVANCE PAPER CORPORATION and GEORGE HAW, in his capacity as President of
Advance Paper Corporation, Petitioners,
vs. On December 29, 1994, the petitioners filed a complaint17 for collection of sum of money with
ARMA TRADERS CORPORATION, MANUEL TING, CHENG GUI and BENJAMIN application for preliminary attachment against Arma Traders, Tan, Uy, Ting, Gui, and Ng.
NG, Respondents.
Claims of the petitioners
x-------------------------------------------------x
The petitioners claimed that the respondents fraudulently issued the postdated checks as
ANTONIO TAN and UY SENG KEE WILLY, Respondents. payment for the purchases and loan transactions knowing that they did not have sufficient funds
with the drawee banks.18
DECISION
To prove the purchases on credit, the petitioners presented the summary of the transactions
and their corresponding sales invoices as their documentary evidence.19
BRION, J.:

During the trial, Haw also testified that within one or two weeks upon delivery of the paper
Before us is a Petition for Review1 seeking to set aside the Decision of the Court of
products, Arma Traders paid the purchases in the form of postdated checks. Thus, he personally
Appeals (CA) in CA-G.R. CV No. 71499 dated March 31, 2006 and the Resolution dated March
collected these checks on Saturdays and upon receiving the checks, he surrendered to Arma
7, 2007.2 The Decision reversed and set aside the ruling of the Regional Trial Court (RTC) of
Traders the original of the sales invoices while he retained the duplicate of the invoices.20
Manila, Branch 18 in Civil Case No. 94-72526 which ordered Arma Traders Corporation (Arma
Traders) to pay Advance Paper Corporation (Advance Paper) the sum of ₱15,321,798.25 with
interest, and ₱1,500,000.00 for attorney’s fees, plus the cost of the suit. 3 To prove the loan transactions, the petitioners presented the copies of the checks 21 which
Advance Paper issued in favor of Arma Traders. The petitioners also filed a
manifestation22 dated June 14, 1995, submitting a bank statement from Metrobank EDSA
Factual Antecedents
Kalookan Branch. This was to show that Advance Paper’s credit line with Metrobank has been
transferred to the account of Arma Traders as payee from October 1994 to December 1994.
Petitioner Advance Paper is a domestic corporation engaged in the business of producing,
printing, manufacturing, distributing and selling of various paper products. 4 Petitioner George
Moreover, Haw testified to prove the loan transactions. When asked why he considered
Haw (Haw) is the President while his wife, Connie Haw, is the General Manager. 5
extending the loans without any collateral and loan agreement or promissory note, and only on
the basis of the issuance of the postdated checks, he answered that it was because he trusted
Respondent Arma Traders is also a domestic corporation engaged in the wholesale and Arma Traders since it had been their customer for a long time and that none of the previous
distribution of school and office supplies, and novelty products. 6 Respondent Antonio Tan (Tan) checks ever bounced.23
was formerly the President while respondent Uy Seng Kee Willy (Uy) is the Treasurer of Arma
Traders.7 They represented Arma Traders when dealing with its supplier, Advance Paper, for
Claims of the respondents
about 14 years.8

The respondents argued that the purchases on credit were spurious, simulated and fraudulent
On the other hand, respondents Manuel Ting, Cheng Gui and Benjamin Ng worked for Arma
since there was no delivery of the ₱7,000,000.00 worth of notebooks and other paper
Traders as Vice-President, General Manager and Corporate Secretary, respectively.9
products.24

On various dates from September to December 1994, Arma Traders purchased on credit
During the trial, Ng testified that Arma Traders did not purchase notebooks and other paper
notebooks and other paper products amounting to ₱7,533,001.49 from Advance Paper. 10
products from September to December 1994. He claimed that during this period, Arma Traders
concentrated on Christmas items, not school and office supplies. He also narrated that upon
Upon the representation of Tan and Uy, Arma Traders also obtained three loans from Advance learning about the complaint filed by the petitioners, he immediately looked for Arma Traders’
Paper in November 1994 in the amounts of ₱3,380,171.82, ₱1,000,000.00, and ₱3,408,623.94 records and found no receipts involving the purchases of notebooks and other paper products
or a total of ₱7,788,796.76.11 Arma Traders needed the loan to settle its obligations to other from Advance Paper.25
suppliers because its own collectibles did not arrive on time.12 Because of its good business
relations with Arma Traders, Advance Paper extended the loans. 13
As to the loan transactions, the respondents countered that these were the personal
obligations of Tan and Uy to Advance Paper. These loans were never intended to benefit the
As payment for the purchases on credit and the loan transactions, Arma Traders issued 82 respondents.
postdated checks14payable to cash or to Advance Paper. Tan and Uy were Arma Traders’
authorized bank signatories who signed and issued these checks which had the aggregate
amount of ₱15,130,636.87.15
The respondents also claimed that the loan transactions were ultra vires because the board of The RTC Ruling
directors of Arma Traders did not issue a board resolution authorizing Tan and Uy to obtain the
loans from Advance Paper. They claimed that the borrowing of money must be done only with
On June 18, 2001, the RTC ruled that the purchases on credit and loans were sufficiently proven
the prior approval of the board of directors because without the approval, the corporate officers
by the petitioners. Hence, the RTC ordered Arma Traders to pay Advance Paper the sum of
are acting in excess of their authority or ultra vires. When the acts of the corporate officers
₱15,321,798.25 with interest, and ₱1,500,000.00 for attorney’s fees, plus the cost of the suit.
are ultra vires, the corporation is not liable for whatever acts that these officers committed in
excess of their authority. Further, the respondents claimed that Advance Paper failed to verify
Tan and Uy’s authority to transact business with them. Hence, Advance Paper should suffer the The RTC held that the respondents failed to present hard, admissible and credible evidence to
consequences.26 prove that the sale invoices were forged or fictitious, and that the loan transactions were
personal obligations of Tan and Uy. Nonetheless, the RTC dismissed the complaint against Tan,
Uy, Ting, Gui and Ng due to the lack of evidence showing that they bound themselves, either
The respondents accused Tan and Uy for conspiring with the petitioners to defraud Arma
jointly or solidarily, with Arma Traders for the payment of its account.38
Traders through a series of transactions known as rediscounting of postdated checks. In
rediscounting, the respondents explained that Tan and Uy would issue Arma Traders’ postdated
checks to the petitioners in exchange for cash, discounted by as much as 7% to 10% depending Arma Traders appealed the RTC decision to the CA.
on how long were the terms of repayment. The rediscounted percentage represented the
interest or profit earned by the petitioners in these transactions. 27
The CA Ruling

Tan did not file his Answer and was eventually declared in default.
The CA held that the petitioners failed to prove by preponderance of evidence the existence of
the purchases on credit and loans based on the following grounds:
On the other hand, Uy filed his Answer28 dated January 20, 1995 but was subsequently declared
in default upon his failure to appear during the pre-trial. In his Answer, he admitted that Arma
Traders together with its corporate officers have been transacting business with Advance First, Arma Traders was not liable for the loan in the absence of a board resolution authorizing
Tan and Uy to obtain the loan from Advance Paper.39 The CA acknowledged that Tan and Uy
Paper.29 He claimed that he and Tan have been authorized by the board of directors for the past
13 years to issue checks in behalf of Arma Traders to pay its obligations with Advance were Arma Traders’ authorized bank signatories. However, the CA explained that this is not
Paper.30 Furthermore, he admitted that Arma Traders’ checks were issued to pay its sufficient because the authority to sign the checks is different from the required authority to
contractual obligations with Advance Paper.31 However, according to him, Advance Paper contract a loan.40
was informed beforehand that Arma Traders’ checks were funded out of the ₱20,000,000.00
worth of collectibles coming from the provinces. Unfortunately, the expected collectibles did not Second, the CA also held that the petitioners presented incompetent and inadmissible evidence
materialize for unknown reasons.32 to prove the purchases on credit since the sales invoices were hearsay. 41 The CA pointed out
that Haw’s testimony as to the identification of the sales invoices was not an exception to the
Ng filed his Answer33 and claimed that the management of Arma Traders was left entirely to Tan hearsay rule because there was no showing that the secretaries who prepared the sales
and Uy. Thus, he never participated in the company’s daily transactions. 34 invoices are already dead or unable to testify as required by the Rules of Court.42 Further, the
CA noted that the secretaries were not identified or presented in court. 43

Atty. Ernest S. Ang, Jr. (Atty. Ang), Arma Traders’ Vice-President for Legal Affairs and Credit
and Collection, testified that he investigated the transactions involving Tan and Uy and Third, the CA ruling heavily relied on Ng’s Appellant’s Brief44 which made the detailed
discovered that they were financing their own business using Arma Traders’ resources. He also description of the "badges of fraud." The CA averred that the petitioners failed to satisfactorily
rebut the badges of fraud45 which include the inconsistencies in:
accused Haw for conniving with Tan and Uy in fraudulently making Arma Traders liable for their
personal debts. He based this conclusion from the following: First, basic human experience and
common sense tell us that a lender will not agree to extend additional loan to another person (1) "Exhibit E-26," a postdated check, which was allegedly issued in favor of Advance
who already owes a substantial sum from the lender – in this case, petitioner Advance Paper but turned out to be a check payable to Top Line, Advance Paper’s sister
Paper. Second, there was no other document proving the existence of the loan other than the company;46
postdated checks. Third, the total of the purchase and loan transactions vis-à-vis the total
amount of the postdated checks did not tally. Fourth, he found out that the certified true copy of
Advance Paper’s report with the Securities and Exchange Commission (SEC report) did not (2) "Sale Invoice No. 8946," an evidence to prove the existence of the purchases on
reflect the ₱15,000,000.00 collectibles it had with Arma Traders. 35 credit, whose photocopy failed to reflect the amount stated in the duplicate
copy,47 and;

Atty. Ang also testified that he already filed several cases of estafa and qualified theft 36 against
Tan and Uy and that several warrants of arrest had been issued against them. (3) The SEC report of Advance Paper for the year ended 1994 reflected its account
receivables amounting to ₱219,705.19 only – an amount far from the claimed
₱15,321,798.25 receivables from Arma Traders.48
In their pre-trial brief,37 the respondents named Sharow Ong, the secretary of Tan and Uy, to
testify on how Tan and Uy conspired with the petitioners to defraud Arma Traders. However, the
respondents did not present her on the witness stand. Hence, the CA set aside the RTC’s order for Arma Traders to pay Advance Paper the sum of
₱15,321,798.25, ₱1,500,000.00 for attorney’s fees, plus cost of suit.49 It affirmed the RTC
decision dismissing the complaint against respondents Tan, Uy, Ting, Gui and Ng. 50 The CA
also directed the petitioners to solidarily pay each of the respondents their counterclaims of cannot escape their liability by simply pointing the SEC report because the petitioners
₱250,000.00 as moral damages, ₱250,000.00 as exemplary damages, and ₱250,000.00 as have established their cause of action – that the purchases on credit and loan
attorney’s fees.51 transactions took place, the respondents issued the dishonored checks to cover their
debts, and they refused to settle their obligation with Advance Paper.63
The Petition
The Case for the Respondents
The petitioners raise the following arguments.
The respondents argue that the Petition for Review should be dismissed summarily because of
the following procedural grounds: first, for failure to comply with A.M. No. 02-8-13-
First, Arma Traders led the petitioners to believe that Tan and Uy had the authority to obtain
SC;64 and second, the CA decision is already final and executory since the petitioners filed their
loans since the respondents left the active and sole management of the company to Tan and Uy
Motion for Reconsideration out of time. They explain that under the rules of the CA, if the last
since 1984. In fact, Ng testified that Arma Traders’ stockholders and board of directors never
day for filing of any pleading falls on a Saturday not a holiday, the same must be filed on said
conducted a meeting from 1984 to 1995. Therefore, if the respondents’ position will be
Saturday, as the Docket and Receiving Section of the CA is open on a Saturday. 65
sustained, they will have the absurd power to question all the business transactions of Arma
Traders.52 Citing Lipat v. Pacific Banking Corporation,53 the petitioners said that if a corporation
knowingly permits one of its officers or any other agent to act within the scope of an apparent The respondents argue that while as a general rule, a corporation is estopped from denying the
authority, it holds him out to the public as possessing the power to do those acts; thus, the authority of its agents which it allowed to deal with the general public; this is only true if the
corporation will, as against anyone who has in good faith dealt with it through such agent, be person dealing with the agent dealt in good faith.66 In the present case, the respondents claim
estopped from denying the agent’s authority. that the petitioners are in bad faith because the petitioners connived with Tan and Uy to make
Arma Traders liable for the non-existent deliveries of notebooks and other paper
products.67 They also insist that the sales invoices are manufactured evidence. 68
Second, the petitioners argue that Haw’s testimony is not hearsay. They emphasize that Haw
has personal knowledge of the assailed purchases and loan transactions because he dealt with
the customers, and supervised and directed the preparation of the sales invoices and the As to the loans, the respondents aver that these were Tan and Uy’s personal obligations with
deliveries of the goods.54 Moreover, the petitioners stress that the respondents never objected to Advance Paper.69Moreover, while the three cashier’s checks were deposited in the account of
the admissibility of the sales invoices on the ground that they were hearsay.55 Arma Traders, it is likewise true that Tan and Uy issued Arma Traders’ checks in favor of
Advance Paper. All these checks are evidence of Tan, Uy and Haw’s systematic conspiracy to
siphon Arma Traders corporate funds.70
Third, the petitioners dispute the CA’s findings on the existence of the badges of fraud. The
petitioners countered:
The respondents also seek to discredit Haw’s testimony on the basis of the following. First, his
testimony as regards the sales invoices is hearsay because he did not personally prepare these
(1) The discrepancies between the figures in the 15 out of the 96 photocopies and
documentary evidence.71 Second, Haw suspiciously never had any written authority from his
duplicate originals of the sales invoices amounting to ₱4,624.80 – an insignificant
own Board of Directors to lend money. Third, the respondents also questioned why Advance
amount compared to the total purchases of ₱7,533,001.49 – may have been
Paper granted the ₱7,000,000.00 loan without requiring Arma Traders to present any collateral
caused by the failure to put the carbon paper.56 Besides, the remaining 81 sales
or guarantees.72
invoices are uncontroverted. The petitioners also raise the point that this
discrepancy is a nonissue because the duplicate originals were surrendered in the
RTC.57 The Issues

(2) The respondents misled Haw during the cross-examination and took his answer The main procedural and substantive issues are:
out of context.58 The petitioners argue that this maneuver is insufficient to discredit
Haw’s entire testimony.59
I. Whether the petition for review should be dismissed for failure to comply with A.M.
No. 02-8-13-SC.
(3) Arma Traders should be faulted for indicating Top Line as the payee in Exhibit E-
26 or PBC check no. 091014. Moreover, Exhibit E-26 does not refer to PBC check no.
II. Whether the petition for review should be dismissed on the ground of failure to file
091014 but to PBC check no. 091032 payable to the order of cash. 60
the motion for reconsideration with the CA on time.

(4) The discrepancy in the total amount of the checks which is ₱15,130,363.87 as
III. Whether Arma Traders is liable to pay the loans applying the doctrine of apparent
against the total obligation of ₱15,321,798.25 does not necessarily prove that the
authority.
transactions are spurious.61

IV. Whether the petitioners proved Arma Traders’ liability on the purchases on credit
(5) The difference in Advance Paper’s accounts receivables in the SEC report and in
by preponderance of evidence.
Arma Traders’ obligation with Advance Paper was based on non-existent evidence
because Exhibit 294-NG does not pertain to any balance sheet.62 Moreover, the term
"accounts receivable" is not synonymous with "cause of action." The respondents
The Court's Ruling pertaining to the particular officer or agent, and such apparent powers as the corporation has
caused person dealing with the officer or agent to believe that it has conferred.
We grant the petition.
[A]pparent authority is derived not merely from practice. Its existence may be ascertained
through (1) the general manner in which the corporation holds out an officer or agent as having
The procedural issues.
the power to act or, in other words the apparent authority to act in general, with which it clothes
him; or (2) the acquiescence in his acts of a particular nature, with actual or constructive
First, the respondents correctly cited A.M. No. 02-8-13-SC dated February 19, 2008 which refer knowledge thereof, within or beyond the scope of his ordinary powers. It requires
to the amendment of the 2004 Rules on Notarial Practice. It deleted the Community Tax presentation of evidence of similar act(s) executed either in its favor or in favor of other
Certificate among the accepted proof of identity of the affiant because of its inherent unreliability. parties. It is not the quantity of similar acts which establishes apparent authority, but the
The petitioners violated this when they used Community Tax Certificate No. 05730869 in their vesting of a corporate officer with the power to bind the corporation. [emphases and
Petition for Review.73 Nevertheless, the defective jurat in the Verification/Certification of Non- underscores ours]
Forum Shopping is not a fatal defect because it is only a formal, not a jurisdictional, requirement
that the Court may waive.74 Furthermore, we cannot simply ignore the millions of pesos at stake
In People’s Aircargo and Warehousing Co., Inc. v. Court of Appeals,79 we ruled that the doctrine
in this case. To do so might cause grave injustice to a party, a situation that this Court intends to
of apparent authority is applied when the petitioner, through its president Antonio Punsalan Jr.,
avoid.
entered into the First Contract without first securing board approval. Despite such lack of board
approval, petitioner did not object to or repudiate said contract, thus "clothing" its president with
Second, no less than the CA itself waived the rules on the period to file the motion for the power to bind the corporation.
reconsideration. A review of the CA Resolution75 dated March 7, 2007, reveals that the
petitioners’ Motion for Reconsideration was denied because the allegations were a mere rehash
"Inasmuch as a corporate president is often given general supervision and control over
of what the petitioners earlier argued – not because the motion for reconsideration was filed out
corporate operations, the strict rule that said officer has no inherent power to act for the
of time.
corporation is slowly giving way to the realization that such officer has certain limited powers in
the transaction of the usual and ordinary business of the corporation." 80 "In the absence of a
The substantive issues. charter or bylaw provision to the contrary, the president is presumed to have the
authority to act within the domain of the general objectives of its business and within the
scope of his or her usual duties."81
Arma Traders is liable to pay the
loans on the basis of the doctrine of
apparent authority. In the present petition, we do not agree with the CA’s findings that Arma Traders is not liable to
pay the loans due to the lack of board resolution authorizing Tan and Uy to obtain the loans. To
begin with, Arma Traders’ Articles of Incorporation82 provides that the corporation may borrow
The doctrine of apparent authority provides that a corporation will be estopped from denying the or raise money to meet the financial requirements of its business by the issuance of bonds,
agent’s authority if it knowingly permits one of its officers or any other agent to act within the promissory notes and other evidence of indebtedness. Likewise, it states that Tan and Uy are
scope of an apparent authority, and it holds him out to the public as possessing the power to do not just ordinary corporate officers and authorized bank signatories because they are also Arma
those acts.76 The doctrine of apparent authority does not apply if the principal did not commit any Traders’ incorporators along with respondents Ng and Ting, and Pedro Chao. Furthermore, the
acts or conduct which a third party knew and relied upon in good faith as a result of the exercise respondents, through Ng who is Arma Traders’ corporate secretary, incorporator, stockholder
of reasonable prudence. Moreover, the agent’s acts or conduct must have produced a change of and director, testified that the sole management of Arma Traders was left to Tan and Uy and
position to the third party’s detriment.77
that he and the other officers never dealt with the business and management of Arma
Traders for 14 years. He also confirmed that since 1984 up to the filing of the complaint
In Inter-Asia Investment Industries v. Court of Appeals,78 we explained: against Arma Traders, its stockholders and board of directors never had its meeting.83

Under this provision [referring to Sec. 23 of the Corporation Code], the power and responsibility Thus, Arma Traders bestowed upon Tan and Uy broad powers by allowing them to transact with
to decide whether the corporation should enter into a contract that will bind the corporation is third persons without the necessary written authority from its non-performing board of directors.
lodged in the board, subject to the articles of incorporation, bylaws, or relevant provisions of Arma Traders failed to take precautions to prevent its own corporate officers from abusing their
law. However, just as a natural person who may authorize another to do certain acts for powers. Because of its own laxity in its business dealings, Arma Traders is now estopped from
and on his behalf, the board of directors may validly delegate some of its functions and denying Tan and Uy’s authority to obtain loan from Advance Paper.
powers to officers, committees or agents. The authority of such individuals to bind the
corporation is generally derived from law, corporate bylaws or authorization from the
We also reject the respondents’ claim that Advance Paper, through Haw, connived with Tan and
board, either expressly or impliedly by habit, custom or acquiescence in the general
Uy. The records do not contain any evidence to prove that the loan transactions were personal
course of business, viz.:
to Tan and Uy. A different conclusion might have been inferred had the cashier’s checks been
issued in favor of Tan and Uy, and had the postdated checks in favor of Advance Paper been
A corporate officer or agent may represent and bind the corporation in transactions with third either Tan and/or Uy’s, or had the respondents presented convincing evidence to show how Tan
persons to the extent that [the] authority to do so has been conferred upon him, and this includes and Uy conspired with the petitioners to defraud Arma Traders. 84 We note that the respondents
powers as, in the usual course of the particular business, are incidental to, or may be implied initially intended to present Sharow Ong, the secretary of Tan and Uy, to testify on how Advance
from, the powers intentionally conferred, powers added by custom and usage, as usually
Paper connived with Tan and Uy. As mentioned, the respondents failed to present her on the Additionally, the issue of credibility of witnesses is to be resolved primarily by the trial court
witness stand. because it is in the better position to assess the credibility of witnesses as it heard the
testimonies and observed the deportment and manner of testifying of the witnesses.
Accordingly, its findings are entitled to great respect and will not be disturbed on appeal in the
The respondents failed to object to
absence of any showing that the trial court overlooked, misunderstood, or misapplied some facts
the admissibility of the sales invoices
or circumstances of weight and substance which would have affected the result of the case. 94
on the ground that they are hearsay

In the present case, the RTC judge took into consideration the substance and the manner by
The rule is that failure to object to the offered evidence renders it admissible, and the court
which Haw answered each propounded questions to him in the witness stand. Hence, the minor
cannot, on its own, disregard such evidence.85 When a party desires the court to reject the
inconsistencies in Haw’s testimony notwithstanding, the RTC held that the respondents claim
evidence offered, it must so state in the form of a timely objection and it cannot raise the
that the purchase and loan transactions were spurious is "not worthy of serious consideration."
objection to the evidence for the first time on appeal. Because of a party’s failure to timely object,
Besides, the respondents failed to convince us that the RTC judge overlooked, misunderstood,
the evidence becomes part of the evidence in the case. Thereafter, all the parties are considered
or misapplied some facts or circumstances of weight and substance which would have affected
bound by any outcome arising from the offer of evidence properly presented. 86
the result of the case.

In Heirs of Policronio M. Ureta, Sr. v. Heirs of Liberato M. Ureta,87 however, we held:


On the other hand, we agree with the petitioners that the discrepancies in the photocopy of the
sales invoices and its duplicate copy have been sufficiently explained. Besides, this is already a
[H]earsay evidence whether objected to or not cannot be given credence for having no probative non-issue since the duplicate copies were surrendered in the RTC. 95 Furthermore, the fact that
value.1âwphi1 This principle, however, has been relaxed in cases where, in addition to the the value of Arma Traders' checks does not tally with the total amount of their obligation with
failure to object to the admissibility of the subject evidence, there were other pieces of Advance Paper is not inconsistent with the existence of the purchases and loan transactions.
evidence presented or there were other circumstances prevailing to support the fact in
issue. (emphasis and underscore ours; citation omitted)
As against the case and the evidence Advance Paper presented, the respondents relied on the
core theory of an alleged conspiracy between Tan, Uy and Haw to defraud Arma Traders.
We agree with the respondents that with respect to the identification of the sales invoices, Haw’s However, the records are bereft of supporting evidence to prove the alleged conspiracy. Instead,
testimony was hearsay because he was not present during its preparation88 and the secretaries the respondents simply dwelled on the minor inconsistencies from the petitioners' evidence that
who prepared them were not presented to identify them in court. Further, these sales invoices do the respondents appear to have magnified. From these perspectives, the preponderance of
not fall within the exceptions to the hearsay rule even under the "entries in the course of evidence thus lies heavily in the petitioners' favor as the RTC found. For this reason, we find the
business" because the petitioners failed to show that the entrant was deceased or was unable to petition meritorious.
testify.89
WHEREFORE, premises considered, we GRANT the petition. The decision dated March 31,
But even though the sales invoices are hearsay, nonetheless, they form part of the records of 2006 and the resolution dated March 7, 2007 of the Court of Appeals in CA-G.R. CV No. 71499
the case for the respondents’ failure to object as to the admissibility of the sales invoices on the are REVERSED and SET ASIDE. The Regional Trial Court decision in Civil Case No. 94-72526
ground that they are hearsay.90Based on the records, the respondents through Ng objected to dated June 18, 2001 is REINSTATED. No costs.
the offer "for the purpose [to] which they are being offered" only – not on the ground that they
were hearsay.91
SO ORDERED.

The petitioners have proven their


claims for the unpaid purchases on
credit by preponderance of evidence.

We are not convinced by the respondents’ argument that the purchases are spurious because
no less than Uy admitted that all the checks issued were in payments of the contractual
obligations of the Arma Traders with Advance Paper.92 Moreover, there are other pieces of
evidence to prove the existence of the purchases other than the sales invoices themselves. For
one, Arma Traders’ postdated checks evince the existence of the purchases on credit.
Moreover, Haw testified that within one or two weeks, Arma Traders paid the purchases in the
form of postdated checks. He personally collected these checks on Saturdays and upon
receiving the checks, he surrendered to Arma Traders the original of the sales invoices while he
retained the duplicate of the invoices.93

The respondents attempted to impugn the credibility of Haw by pointing to the inconsistencies
they can find from the transcript of stenographic notes. However, we are not persuaded that
these inconsistencies are sufficiently pervasive to affect the totality of evidence showing the
general relationship between Advance Paper and Arma Traders.
G.R. No. 51765 March 3, 1997 certificates. Private respondents attached to their complaint, a letter-demand dated January 5,
1979 which, significantly, was not formally offered in evidence.
REPUBLIC PLANTERS BANK, petitioner,
vs. Petitioner filed a Motion to Dismiss3 private respondents' Complaint on the following grounds: (1)
HON. ENRIQUE A. AGANA, SR., as Presiding Judge, Court of First Instance of Rizal, that the trial court had no jurisdiction over the subject-matter of the action; (2) that the action was
Branch XXVIII, Pasay City, ROBES-FRANCISCO REALTY & DEVELOPMENT unenforceable under substantive law; and (3) that the action was barred by the statute of
CORPORATION and ADALIA F. ROBES, respondents. limitations and/or laches.

HERMOSISIMA, JR., J.: Petitioner's Motion to Dismiss was denied by the trial court in an Order dated March 16,
1979.4 Petitioner then filed its Answer on May 2, 1979. 5 Thereafter, the trial court gave the
parties ten (10) days from July 30, 1979 to submit their respective memoranda after the
This is a petition for certiorari seeking the annulment of the Decision1 of the then Court of First
submission of which the case would be deemed submitted for resolution.6
Instance of Rizal2 for having been rendered in grave abuse of discretion. Private respondents
Robes-Francisco Realty and Development Corporation (hereafter, "the Corporation") and Adalia
F. Robes filed in the court a quo, an action for specific performance to compel petitioner to On September 7, 1979, the trial court rendered the herein assailed decision in favor of private
redeem 800 preferred shares of stock with a face value of P8,000.00 and to pay 1% quarterly respondents. In ordering petitioner to pay private respondents the face value of the stock
interest thereon as quarterly dividend owing them under the terms and conditions of the certificates as redemption price, plus 1% quarterly interest thereon until full payment, the trial
certificates of stock. court ruled:

The court a quo rendered judgment in favor of private respondents; hence, this instant petition. There being no issue of fact raised by either of the parties who filed their
respective memoranda delineating their respective contentions, a judgment
on the pleadings, conformably with an earlier order of the Court, appears to
Herein parties debate only legal issues, no issues of fact having been raised by them in the
be in order.
court a quo. For ready reference, however, the following narration of pertinent transactions and
events is in order:
From a further perusal of the pleadings, it appears that the provision of the
stock certificates in question to the effect that the plaintiffs shall have the
On September 18, 1961, private respondent Corporation secured a loan from petitioner in the
right to receive a quarterly dividend of One Per Centum(1%), cumulative
amount of P120,000.00. As part of the proceeds of the loan, preferred shares of stocks were
and participating, clearly and unequivocably [sic] indicates that the same are
issued to private respondent Corporation, through its officers then, private respondent Adalia F.
"interest bearing stocks" which are stocks issued by a corporation under an
Robes and one Carlos F. Robes. In other words, instead of giving the legal tender totaling to the
agreement to pay a certain rate of interest thereon (5 Thompson, Sec.
full amount of the loan, which is P120,000.00, petitioner lent such amount partially in the form of
3439). As such, plaintiffs become entitled to the payment thereof as a
money and partially in the form of stock certificates numbered 3204 and 3205, each for 400
matter of right without necessity of a prior declaration of dividend.
shares with a par value of P10.00 per share, or for P4,000.00 each, for a total of P8,000.00. Said
stock certificates were in the name of private respondent Adalia F. Robes and Carlos F. Robes,
who subsequently, however, endorsed his shares in favor of Adalia F. Robes. On the question of the redemption by the defendant of said preferred shares
of stock, the very wordings of the terms and conditions in said stock
certificates clearly allows the same.
Said certificates of stock bear the following terms and conditions:

To allow the herein defendant not to redeem said preferred shares of stock
The Preferred Stock shall have the following rights, preferences,
and/or pay the interest due thereon despite the clear import of said
qualifications and limitations, to wit:
provisions by the mere invocation of alleged Central Bank Circulars
prohibiting the same is tantamount to an impairment of the obligation of
1. Of the right to receive a quarterly dividend of One Per Centum (1%), contracts enshrined in no less than the fundamental law itself.
cumulative and participating.
Moreover, the herein defendant is considered in estoppel from taking shelter
xxx xxx xxx behind a General Banking Act provision to the effect that it cannot buy its
own shares of stocks considering that the very terms and conditions in said
stock certificates allowing their redemption are its own handiwork.
2. That such preferred shares may be redeemed, by the system of drawing
lots, at any time after two (2) years from the date of issue at the option of
the Corporation. . . . As to the claim by the defendant that plaintiffs' cause of action is barred by
prescription, suffice it to state that the running of the prescriptive period was
considered interrupted by the written extrajudicial demands made by the
On January 31, 1979, private respondents proceeded against petitioner and filed a Complaint plaintiffs from the defendant.7
anchored on private respondents' alleged rights to collect dividends under the preferred shares
in question and to have petitioner redeem the same under the terms and conditions of the stock
Aggrieved by the decision of the trial court, petitioner elevated the case before us essentially on the case may be. Preferences granted to preferred stockholders, moreover, do not give them a
pure questions of law. Petitioner's statement of the issues that it submits for us to adjudicate lien upon the property of the corporation nor make them creditors of the corporation, the right of
upon, is as follows: the former being always subordinate to the latter. Dividends are thus payable only when there
are profits earned by the corporation and as a general rule, even if there are existing profits, the
board of directors has the discretion to determine whether or not dividends are to be
A. RESPONDENT JUDGE COMMITTED A GRAVE ABUSE OF
declared. 15 Shareholders, both common and preferred, are considered risk takers who invest
DISCRETION AMOUNTING TO LACK OR EXCESS OF JURISDICTION IN
capital in the business and who can look only to what is left after corporate debts and liabilities
ORDERING PETITIONER TO PAY RESPONDENT ADALIA F. ROBES
are fully paid. 16
THE AMOUNT OF P8213.69 AS INTERESTS FROM 1961 TO 1979 ON
HER PREFERRED SHARES.
Redeemable shares, on the other hand, are shares usually preferred, which by their terms are
redeemable at a fixed date, or at the option of either issuing corporation, or the stockholder, or
B. RESPONDENT JUDGE COMMITTED A GRAVE ABUSE OF
both at a certain redemption price.17 A redemption by the corporation of its stock is, in a sense, a
DISCRETION AMOUNTING TO LACK OR EXCESS OF JURISDICTION IN
repurchase of it for cancellation. 18 The present Code allows redemption of shares even if there
ORDERING PETITIONER TO REDEEM RESPONDENT ADALIA F.
are no unrestricted retained earnings on the books of the corporation. This is a new provision
ROBES' PREFERRED SHARES FOR P8,000.00.
which in effect qualifies the general rule that the corporation cannot purchase its own shares
except out of current retained earnings. 19 However, while redeemable shares may be redeemed
C. RESPONDENT JUDGE COMMITTED A GRAVE ABUSE OF regardless of the existence of unrestricted retained earnings, this is subject to the condition that
DISCRETION AMOUNTING TO LACK OR EXCESS OF JURISDICTION IN the corporation has, after such redemption, assets in its books to cover debts and liabilities
DISREGARDING THE ORDER OF THE CENTRAL BANK TO PETITIONER inclusive of capital stock. Redemption, therefore, may not be made where the corporation is
TO DESIST FROM REDEEMING ITS PREFERRED SHARES AND FROM insolvent or if such redemption will cause insolvency or inability of the corporation to meet its
PAYING DIVIDENDS THEREON . . . . debts as they mature. 20

D. THE TRIAL COURT ERRED IN NOT HOLDING THAT THE We come now to the merits of the case. The petitioner argues that it cannot be compelled to
COMPLAINT DOES NOT STATE A CAUSE OF ACTION. redeem the preferred shares issued to the private respondent. We agree. Respondent judge, in
ruling that petitioner must redeem the shares in question, stated that:
E. THE TRIAL COURT ERRED IN NOT HOLDING THAT THE CLAIM OF
RESPONDENT ADALIA F. ROBES IS BARRED BY PRESCRIPTION OR On the question of the redemption by the defendant of said preferred shares
LACHES. 8 of stock, the very wordings of the terms and conditions in said stock
certificates clearly allows the same. 21
The petition is meritorious.
What respondent judge failed to recognize was that while the stock certificate does
allow redemption, the option to do so was clearly vested in the petitioner bank. The
Before passing upon the merits of this petition, it may be pertinent to provide an overview on the redemption therefore is clearly the type known as "optional". Thus, except as
nature of preferred shares and the redemption thereof, considering that these issues lie at the otherwise provided in the stock certificate, the redemption rests entirely with the
heart of the dispute. corporation and the stockholder is without right to either compel or refuse the
redemption of its stock. 22Furthermore, the terms and conditions set forth therein use
A preferred share of stock, on one hand, is one which entitles the holder thereof to certain the word "may". It is a settled doctrine in statutory construction that the word "may"
preferences over the holders of common stock. The preferences are designed to induce persons denotes discretion, and cannot be construed as having a mandatory effect. We fail to
to subscribe for shares of a corporation.9 Preferred shares take a multiplicity of forms. The most see how respondent judge can ignore what, in his words, are the "very wordings of the
common forms may be classified into two: (1) preferred shares as to assets; and (2) preferred terms and conditions in said stock certificates" and construe what is clearly a mere
shares as to dividends. The former is a share which gives the holder thereof preference in the option to be his legal basis for compelling the petitioner to redeem the shares in
distribution of the assets of the corporation in case of liquidation; 10 the latter is a share the question.
holder of which is entitled to receive dividends on said share to the extent agreed upon before
any dividends at all are paid to the holders of common stock. 11 There is no guaranty, however, The redemption of said shares cannot be allowed. As pointed out by the petitioner, the Central
that the share will receive any dividends. Under the old Corporation Law in force at the time the Bank made a finding that said petitioner has been suffering from chronic reserve
contract between the petitioner and the private respondents was entered into, it was provided
deficiency, 23 and that such finding resulted in a directive, issued on January 31, 1973 by then
that "no corporation shall make or declare any dividend except from the surplus profits arising Gov. G.S. Licaros of the Central Bank, to the President and Acting Chairman of the Board of the
from its business, or distribute its capital stock or property other than actual profits among its petitioner bank prohibiting the latter from redeeming any preferred share, on the ground that said
members or stockholders until after the payment of its debts and the termination of its existence
redemption would reduce the assets of the Bank to the prejudice of its depositors and
by limitation or lawful dissolution." 12 Similarly, the present Corporation Code 13 provides that the creditors. 24Redemption of preferred shares was prohibited for a just and valid reason. The
board of directors of a stock corporation may declare dividends only out of unrestricted retained directive issued by the Central Bank Governor was obviously meant to preserve the status quo,
earnings. 14 The Code, in Section 43, adopting the change made in accounting terminology,
and to prevent the financial ruin of a banking institution that would have resulted in adverse
substituted the phrase "unrestricted retained earnings," which may be a more precise term, in repercussions, not only to its depositors and creditors, but also to the banking industry as a
place of "surplus profits arising from its business" in the former law. Thus, the declaration of whole. The directive, in limiting the exercise of a right granted by law to a corporate entity, may
dividends is dependent upon the availability of surplus profit or unrestricted retained earnings, as
thus be considered as an exercise of police power. The respondent judge insists that the
directive constitutes an impairment of the obligation of contracts. It has, however, been settled SO ORDERED.
that the Constitutional guaranty of non-impairment of obligations of contract is limited by the
exercise of the police power of the state, the reason being that public welfare is superior to
private rights. 25

The respondent judge also stated that since the stock certificate granted the private respondents
the right to receive a quarterly dividend of One Per Centum (1%) cumulative and participating, it
"clearly and unequivocably (sic) indicates that the same are "interest bearing stocks" or stocks
issued by a corporation under an agreement to pay a certain rate of interest thereon. As such,
plaintiffs (private respondents herein) become entitled to the payment thereof as a matter of right
without necessity of a prior declaration of dividend." 26 There is no legal basis for this
observation. Both Sec. 16 of the Corporation Law and Sec. 43 of the present Corporation Code
prohibit the issuance of any stock dividend without the approval of stockholders, representing
not less than two-thirds (2/3) of the outstanding capital stock at a regular or special meeting duly
called for the purpose. These provisions underscore the fact that payment of dividends to a
stockholder is not a matter of right but a matter of consensus. Furthermore, "interest bearing
stocks", on which the corporation agrees absolutely to pay interest before dividends are paid to
common stockholders, is legal only when construed as requiring payment of interest as
dividends from net earnings or surplus only. 27 Clearly, the respondent judge, in compelling the
petitioner to redeem the shares in question and to pay the corresponding dividends, committed
grave abuse of discretion amounting to lack or excess of jurisdiction in ignoring both the terms
and conditions specified in the stock certificate, as well as the clear mandate of the law.

Anent the issue of prescription, this Court so holds that the claim of private respondent is already
barred by prescription as well as laches. Art. 1144 of the New Civil Code provides that a right of
action that is founded upon a written contract prescribes in ten (10) years. The letter-demand
made by the private respondents to the petitioner was made only on January 5, 1979, or almost
eighteen years after receipt of the written contract in the form of the stock certificate. As noted
earlier, this letter-demand, significantly, was not formally offered in evidence, nor were any other
evidence of demand presented. Therefore, we conclude that the only time the private
respondents saw it fit to assert their rights, if any, to the preferred shares of stock, was after the
lapse of almost eighteen years. The same clearly indicates that the right of the private
respondents to any relief under the law has already prescribed. Moreover, the claim of the
private respondents is also barred by laches. Laches has been defined as the failure or neglect,
for an unreasonable length of time, to do that which by exercising due diligence could or should
have been done earlier; it is negligence or omission to assert a right within a reasonable time,
warranting a presumption that the party entitled to assert it either has abandoned it or declined
to assert it. 28

Considering that the terms and conditions set forth in the stock certificate clearly indicate that
redemption of the preferred shares may be made at any time after the lapse of two years from
the date of issue, private respondents should have taken it upon themselves, after the lapse of
the said period, to inquire from the petitioner the reason why the said shares have not been
redeemed. As it is, not only two years had lapsed, as agreed upon, but an additional sixteen
years passed before the private respondents saw it fit to demand their right. The petitioner, at
the time it issued said preferred shares to the private respondents in 1961, could not have
known that it would be suffering from chronic reserve deficiency twelve years later. Had the
private respondents been vigilant in asserting their rights, the redemption could have been
effected at a time when the petitioner bank was not suffering from any financial crisis.

WHEREFORE, the instant petition, being impressed with merit, is hereby GRANTED. The
challenged decision of respondent judge is set aside and the complaint against the petitioner is
dismissed.

Costs against the private respondents.


G.R. No. 183278 April 24, 2009 Book. The Republic further prayed for the issuance of an order for PTIC to account for all cash
and stock dividends declared and/or issued by PLDT in favor of PTIC from 1986 up to the
present including compounded interests appurtenant thereto.
IMELDA O. COJUANGCO, PRIME HOLDINGS, INC., AND THE ESTATE OF RAMON U.
COJUANGCOPetitioners,
vs. By Resolution dated December 14, 2006, the Sandiganbayan granted the Motion for the
SANDIGANBAYAN, REPUBLIC OF THE PHILIPPINES, AND THE SHERIFF OF Issuance of a Writ of Execution with respect to the reconveyance of the shares, but denied the
SANDIGANBAYAN,Respondents. prayer for accounting of dividends.

DECISION On Motion for Reconsideration of the Republic, the Sandiganbayan, by the first assailed
Resolution dated November 7, 2007, directed PTIC to deliver the cash and stock dividends
pertaining to the 111,415 shares, including compounded interests, ratiocinating that the same
CARPIO MORALES, J.:
were covered by this Court’s Decision in G.R. No. 153459, since the Republic was therein
adjudged the owner of the shares and, therefore, entitled to the fruits thereof.
The present petition is one for Certiorari.
The Cojuangcos (hereafter petitioners) moved to reconsider the November 7, 2007
Petitioners Imelda O. Cojuangco, Prime Holdings, Inc., and the Estate of Ramon Cojuangco Sandiganbayan Resolution, alleging that this Court’s Decision in G.R. No. 153459 did not
assail via certiorari the Resolutions dated November 7, 20071 and June 13, 20082 of the include a disposition of the dividends and interests accruing to the shares adjudicated in favor of
Sandiganbayan in Civil Case No. 0002, Republic of the Philippines v. Ferdinand Marcos, et. al. the Republic.

A brief recital of the antecedent facts is in order. By the other challenged Resolution dated June 13, 2008, the Sandiganbayan partly granted
petitioners’ Motion for Reconsideration by including legal interests, but not compounding the
same, from the accounting and remittance to the Republic. The Sandiganbayan thereupon
On July 16, 1987, respondent Republic of the Philippines (Republic) filed before the
issued a Writ of Execution,4 hence, spawned the present petition for certiorari.
Sandiganbayan a "Complaint for Reconveyance, Reversion, Accounting, Restitution and
Damages," docketed as Civil Case 0002, praying for the recovery of alleged ill-gotten wealth
from the late President Marcos and former First Lady Imelda Marcos and their cronies, including From the myriad assignments of error proffered by petitioners, the pivotal issues for the Court’s
some 2.4 million shares of stock in the Philippine Long Distance Telephone Company (PLDT). resolution are: (1) whether the Sandiganbayan gravely abused its discretion in ordering the
accounting, delivery, and remittance to the Republic of the stock, cash, and property dividends
pertaining to the 111,415 PTIC shares of Prime Holdings, this Court’s Decision in G.R. No.
The complaint, which was later amended to implead herein petitioners Ramon and Imelda
153459 not having even discussed the same; and (2) whether the Republic, having transferred
Cojuangco (the Cojuangcos), alleged that the Marcoses’ ill-gotten wealth included shares in the the shares to a third party, is entitled to the dividends, interests, and earnings thereof.
PLDT covered by shares of stock in the Philippine Telecommunications Investment Corporation
(PTIC), registered in the name of Prime Holdings, Inc. (Prime Holdings).
Petitioners insist on a literal reading of the dispositive portion of this Court’s Decision in G.R. No.
153459 as excluding the dividends, interests, and earnings accruing to the shares of stock from
The Sandiganbayan dismissed the complaint with respect to the recovery of the PLDT shares, being accounted for and remitted.
hence, the Republic appealed to this Court, docketed as G.R. No. 153459, which appeal was
later consolidated with pending cases of similar import – G.R. Nos. 149802, 150320, and
150367. The term "dividend" in its technical sense and ordinary acceptation is that part or portion of the
profits of the enterprise which the corporation, by its governing agents, sets apart for ratable
division among the holders of the capital stock.5 It is a payment to the stockholders of a
By Decision3 dated January 20, 2006, this Court, in G.R. No. 153459, ruled in favor of the corporation as a return upon their investment,6 and the right thereto is an incident of ownership
Republic, declaring it to be the owner of 111,415 PTIC shares registered in the name of Prime
of stock.7
Holdings. The dispositive portion of the Decision reads:

This Court, in directing the reconveyance to the Republic of the 111,415 shares of PLDT stock
WHEREFORE, the petition of the Republic of the Philippines in G.R. No. 153459
owned by PTIC in the name of Prime Holdings, declared the Republic as the owner of said
is GRANTED to the extent that it prays for the reconveyance to the Republic of 111,415 PTIC
shares and, necessarily, the dividends and interests accruing thereto.
shares registered in the name of PHI. The petitions in G.R. Nos. 149802, 150320, 150367, and
153207 are DENIED for lack of merit.
Ownership is a relation in law by virtue of which a thing pertaining to one person is completely
subjected to his will in everything not prohibited by law or the concurrence with the rights of
SO ORDERED.
another. Its traditional elements or attributes include jus utendi or the right to receive from the
thing what it produces.81avvphi1
The Decision became final and executory on October 26, 2006, hence, the Republic filed on
November 20, 2006 with the Sandiganbayan a Motion for the Issuance of a Writ of Execution, Contrary to petitioners’ contention, while the general rule is that the portion of a decision that
praying for the cancellation of the 111,415 shares/certificates of stock registered in the name of
becomes the subject of execution is that ordained or decreed in the dispositive part thereof,
Prime Holdings and the annotation of the change of ownership on PTIC’s Stock and Transfer
there are recognized exceptions to this rule, viz: (a).where there is ambiguity or uncertainty, the
body of the opinion may be referred to for purposes of construing the judgment, because the
dispositive part of a decision must find support from the decision’s ratio decidendi;and (b).where
extensive and explicit discussion and settlement of the issue is found in the body of the
decision.9

In G.R. No. 153459, although the inclusion of the dividends, interests, and earnings of the
111,415 PTIC shares as belonging to the Republic was not mentioned in the dispositive portion
of the Court’s Decision, it is clear from its body that what was being adjudicated in favor of the
Republic was the whole block of shares and the fruits thereof, said shares having been found to
be part of the Marcoses’ ill-gotten wealth, and therefore, public money.

It would be absurd to award the shares to the Republic as their owner and not include the
dividends and interests accruing thereto. An owner who cannot exercise the "juses" or attributes
of ownership -- the right to possess, to use and enjoy, to abuse or consume, to accessories, to
dispose or alienate, to recover or vindicate, and to the fruits - is a crippled owner.10

Respecting petitioners’ argument that the Republic has yielded its right to the fruits of the shares
when it sold them to Metro Pacific Assets Holdings, Inc., (Metro Pacific), the same does not lie.

Dividends are payable to the stockholders of record as of the date of the declaration of
dividends or holders of record on a certain future date, as the case may be, unless the parties
have agreed otherwise.11 And a transfer of shares which is not recorded in the books of the
corporation is valid only as between the parties, hence, the transferor has the right to dividends
as against the corporation without notice of transfer but it serves as trustee of the real owner of
the dividends, subject to the contract between the transferor and transferee as to who is entitled
to receive the dividends.12

It is thus clear that the Republic is entitled to the dividends accruing from the subject 111,415
shares since 1986 when they were sequestered up to the time they were transferred to Metro
Pacific via the Sale and Purchase Agreement of February 28, 2007; 13 and that the Republic has
since the latter date been serving as trustee of those dividends for the Metro Pacific up to the
present, subject to the terms and conditions of the said agreement they entered into.

WHEREFORE, the petition is DENIED. The challenged Resolutions dated November 7, 2007
and June 13, 2008 of the Sandiganbayan in Civil Case No. 0002 are, in light of the foregoing,
AFFIRMED.

SO ORDERED.
G.R. No. 117897 May 14, 1997 these two contending groups, came out with a Decision in SEC Case No. 2687 declaring the
election of both the Carpizo Group and the Abbas Group as IDP board members to be null and
void. The dispositive portion of the SEC Decision reads:
ISLAMIC DIRECTORATE OF THE PHILIPPINES, MANUEL F. PEREA and SECURITIES &
EXCHANGE COMMISSION, petitioners,
vs. WHEREFORE, judgment is hereby rendered declaring the elections of both
COURT OF APPEALS and IGLESIA NI CRISTO, respondents. the petitioners 7 and respondents 8 as null and void for being violative of the
Articles of Incorporation of petitioner corporation. With the nullification of the
election of the respondents, the approved by-laws which they certified to
HERMOSISIMA, JR., J.:
this Commission as members of the Board of Trustees must necessarily be
likewise declared null and void. However, before any election of the
The subject of this petition for review is the Decision of the public respondent Court of members of the Board of Trustees could be conducted, there must be an
Appeals, 1 dated October 28, 1994, setting aside the portion of the Decision of the Securities approved by-laws to govern the internal government of the association
and Exchange Commission (SEC, for short) in SEC Case No. 4012 which declared null and void including the conduct of election. And since the election of both petitioners
the sale of two (2) parcels of land in Quezon City covered by the Deed of Absolute Sale entered and respondents have been declared null and void, a vacuum is created as
into by and between private respondent Iglesia Ni Cristo (INC, for short) and the Islamic to who should adopt the by-laws and certify its adoption. To remedy this
Directorate of the Philippines, Inc., Carpizo Group, (IDP, for short). unfortunate situation that the association has found itself in, the members of
the petitioning corporation are hereby authorized to prepare and adopt their
by-laws for submission to the Commission. Once approved, an election of
The following facts appear of record. the members of the Board of Trustees shall immediately be called pursuant
to the approved by-laws.
Petitioner IDP-Tamano Group alleges that sometime in 1971, Islamic leaders of all Muslim major
tribal groups in the Philippines headed by Dean Cesar Adib Majul organized and incorporated SO ORDERED. 9
the ISLAMIC DIRECTORATE OF THE PHILIPPINES (IDP), the primary purpose of which is to
establish an Islamic Center in Quezon City for the construction of a "Mosque (prayer place),
Madrasah (Arabic School), and other religious infrastructures" so as to facilitate the effective Neither group, however, took the necessary steps prescribed by the SEC in its October 3, 1986
practice of Islamic faith in the area. 2 Decision, and, thus, no valid election of the members of the Board of Trustees of IDP was ever
called. Although the Carpizo Group 10attempted to submit a set of by-laws, the SEC found that,
aside from Engineer Farouk Carpizo and Atty. Musib Buat, those who prepared and adopted the
Towards this end, that is, in the same year, the Libyan government donated money to the IDP to by-laws were not bona fide members of the IDP, thus rendering the adoption of the by-laws
purchase land at Culiat, Tandang Sora, Quezon City, to be used as a Center for the Islamic
likewise null and void.
populace. The land, with an area of 49,652 square meters, was covered by two titles: Transfer
Certificate of Title Nos. RT-26520 (176616) 3 and RT-26521 (170567), 4 both registered in the
name of IDP. On April 20, 1989, without having been properly elected as new members of the Board of
Trustee of IDP, the Carpizo Group caused to be signed an alleged Board Resolution 11 of the
IDP, authorizing the sale of the subject two parcels of land to the private respondent INC for a
It appears that in 1971, the Board of Trustees of the IDP was composed of the following per consideration of P22,343,400.00, which sale was evidenced by a Deed of Absolute Sale 12 dated
Article 6 of its Articles of Incorporation: April 20, 1989.

Senator Mamintal Tamano 5


On May 30, 1991, the petitioner 1971 IDP Board of Trustees headed by former Senator
Congressman Ali Dimaporo Mamintal Tamano, or the Tamano Group, filed a petition before the SEC, docketed as SEC
Congressman Salipada Pendatun Case No. 4012, seeking to declare null and void the Deed of Absolute Sale signed by the
Dean Cesar Adib Majul
Carpizo Group and the INC since the group of Engineer Carpizo was not the legitimate Board of
Sultan Harun Al-Rashid Lucman Trustees of the IDP.
Delegate Ahmad Alonto
Commissioner Datu Mama Sinsuat
Mayor Aminkadra Abubakar 6 Meanwhile, private respondent INC, pursuant to the Deed of Absolute Sale executed in its favor,
filed an action for Specific Performance with Damages against the vendor, Carpizo Group,
before Branch 81 of the Regional Trial Court of Quezon City, docketed as Civil Case No. Q-90-
According to the petitioner, in 1972, after the purchase of the land by the Libyan government in
6937, to compel said group to clear the property of squatters and deliver complete and full
the name of IDP, Martial Law was declared by the late President Ferdinand Marcos. Most of the physical possession thereof to INC. Likewise, INC filed a motion in the same case to compel one
members of the 1971 Board of Trustees like Senators Mamintal Tamano, Salipada Pendatun, Mrs. Leticia P. Ligon to produce and surrender to the Register of Deeds of Quezon City the
Ahmad Alonto, and Congressman Al-Rashid Lucman flew to the Middle East to escape political
owner's duplicate copy of TCT Nos. RT-26521 and RT-26520 covering the aforementioned two
persecution. parcels of land, so that the sale in INC's favor may be registered and new titles issued in the
name of INC. Mrs. Ligon was alleged to be the mortgagee of the two parcels of land executed in
Thereafter, two Muslim groups sprung, the Carpizo Group, headed by Engineer Farouk Carpizo, her favor by certain Abdulrahman R.T. Linzag and Rowaida Busran-Sampaco claimed to be in
and the Abbas Group, led by Mrs. Zorayda Tamano and Atty. Firdaussi Abbas. Both groups behalf of the Carpizo Group.
claimed to be the legitimate IDP. Significantly, on October 3, 1986, the SEC, in a suit between
The IDP-Tamano Group, on June 11, 1991, sought to intervene in Civil Case No. Q-90-6937 On April 6, 1992, the above Order was amended by Judge Reyes directing Ligon "to deliver the
averring, inter alia: owner's duplicate copies of TCT Nos. RT-26521 (170567) and RT-26520 (176616) to
the Register of Deeds of Quezon City for the purposes stated in the Order of March 2, 1992." 19
xxx xxx xxx
Mortgagee Ligon went to the Court of Appeals, thru a petition for certiorari, docketed as CA-G.R
No. SP-27973, assailing the foregoing Orders of Judge Reyes. The appellate court dismissed
2. That the Intervenor has filed a case before the Securities and Exchange
her petition on October 28, 1992. 20
Commission (SEC) against Mr. Farouk Carpizo, et. al., who, through false
schemes and machinations, succeeded in executing the Deed of Sale
between the IDP and the Iglesia Ni Kristo (plaintiff in the instant case) and Undaunted, Ligon filed a petition for review before the Supreme Court which was docketed as
which Deed of Sale is the subject of the case at bar; G.R. No. 107751.

3. That the said case before the SEC is docketed as Case No. 04012, the In the meantime, the SEC, on July 5, 1993, finally came out with a Decision in SEC Case No.
main issue of which is whether or not the aforesaid Deed of Sale between 4012 in this wise:
IDP and the Iglesia ni Kristo is null and void, hence, Intervenor's legal
interest in the instant case. A copy of the said case is hereto attached as
1. Declaring the by-laws submitted by the respondents 21 as unauthorized,
Annex "A";
and hence, null and void.

4. That, furthermore, Intervenor herein is the duly constituted body which


2. Declaring the sale of the two (2) parcels of land in Quezon City covered
can lawfully and legally represent the Islamic Directorate of the Philippines;
by the Deed of Absolute Sale entered into by Iglesia ni Kristo and the
Islamic Directorate of the Philippines, Inc. 22 null and void;
xxx xxx xxx 13
23
3. Declaring the election of the Board of Directors, of the corporation from
Private respondent INC opposed the motion arguing, inter alia, that the issue sought to be 1986 to 1991 as null and void;
litigated by way of intervention is an intra-corporate dispute which falls under the jurisdiction of
the SEC. 14
4. Declaring the acceptance of the respondents, except Farouk Carpizo and
Musnib Buat, as members of the IDP null and void.
Judge Celia Lipana-Reyes of Branch 81, Regional Trial Court of Quezon City, denied petitioner's
motion to intervene on the ground of lack of juridical personality of the IDP-Tamano Group and
No pronouncement as to cost.
that the issues being raised by way of intervention are intra-corporate in nature, jurisdiction
thereto properly pertaining to the SEC. 15
SO ORDERED. 24
Apprised of the pendency of SEC Case No. 4012 involving the controverted status of the IDP-
Carpizo Group but without waiting for the outcome of said case, Judge Reyes, on September 12, Private respondent INC filed a Motion for Intervention, dated September 7, 1993, in SEC Case
1991, rendered Partial Judgment in Civil Case No. Q-90-6937 ordering the IDP-Carpizo Group No. 4012, but the same was denied on account of the fact that the decision of the case had
to comply with its obligation under the Deed of Sale of clearing the subject lots of squatters and become final and executory, no appeal having been taken therefrom. 25
of delivering the actual possession thereof to INC. 16
INC elevated SEC Case No. 4012 to the public respondent Court of Appeals by way of a special
Thereupon, Judge Reyes in another Order, dated March 2, 1992, pertaining also to Civil Case civil action for certiorari, docketed as CA-G.R SP No. 33295. On October 28, 1994, the court a
No. Q-90-6937, treated INC as the rightful owner of the real properties and disposed as follows: quo promulgated a Decision in CA-G.R. SP No. 33295 granting INC's petition. The portion of the
SEC Decision in SEC Case No. 4012 which declared the sale of the two (2) lots in question to
INC as void was ordered set aside by the Court of Appeals.
WHEREFORE, Leticia P. Ligon is hereby ordered to produce and/or
surrender to plaintiff 17 the owner's copy of RT-26521 (170567) and RT-
26520 (176616) in open court for the registration of the Deed of Absolute Thus, the IDP-Tamano Group brought the instant petition for review, dated December 21, 1994,
Sale in the latter's name and the annotation of the mortgage executed in her submitting that the Court of Appeals gravely erred in:
favor by herein defendant Islamic Directorate of the Philippines on the new
transfer certificate of title to be issued to plaintiff.
1) Not upholding the jurisdiction of the SEC to declare the nullity of the sale;

SO ORDERED. 18
2) Encouraging multiplicity of suits; and

26
3) Not applying the principles of estoppel and laches.
While the above petition was pending, however, the Supreme Court rendered judgment in G.R. intervenor. 28 It was never originally a principal party thereto. It must be noted that intervention is
No. 107751 on the petition filed by Mrs. Leticia P. Ligon. The Decision, dated June 1, 1995, not an independent action, but is merely collateral, accessory, or ancillary to the principal action.
denied the Ligon petition and affirmed the October 28, 1992 Decision of the Court of Appeals in It is just an interlocutory proceeding dependent on or subsidiary to the case between the original
CA-G.R. No. SP-27973 which sustained the Order of Judge Reyes compelling mortgagee Ligon parties. 29 Indeed, the IDP-Tamano Group cannot be considered a principal party in G.R. No.
to surrender the owner's duplicate copies of TCT Nos. RT-26521 (170567) and RT-26520 107751 for purposes of applying the principle of res judicata since the contrary goes against the
(176616) to the Register of Deeds of Quezon City so that the Deed of Absolute Sale in INC's true import of the action of intervention as a mere subsidiary proceeding without an independent
favor may be properly registered. life apart from the principal action as well as the intrinsic character of the intervenor as a mere
subordinate party in the main case whose right may be said to be only in aid of the right of the
original party. 30 It is only in the present case, actually, where the IDP-Tamano Group became a
Before we rule upon the main issue posited in this petition, we would like to point out that our
principal party, as petitioner, with the Iglesia Ni Cristo, as private respondent. Clearly, there is no
disposition in G.R. No. 107751 entitled, "Ligon v. Court of Appeals," promulgated on June 1,
identity of parties in both cases.
1995, in no wise constitutes res judicata such that the petition under consideration would be
barred if it were the ease. Quite the contrary, the requisites or res judicata do not obtain in the
case at bench. In this connection, although it is true that Civil Case No. Q-90-6937, which gave rise to G.R. No.
107751, was entitled, "Iglesia Ni Kristo, Plaintiff v. Islamic Directorate of the Philippines,
Defendant," 31 the IDP can not be considered essentially a formal party thereto for the simple
Section 49, Rule 39 of the Revised Rules of Court lays down the dual aspects of res judicata in
reason that it was not duly represented by a legitimate Board of Trustees in that case. As a
actions in personam, to wit:
necessary consequence, Civil Case No. Q-90-6937, a case for Specific Performance with
Damages, a mere action in personam, did not become final and executory insofar as the true
Effect of judgment. — The effect of a judgment or final order rendered by a IDP is concerned since petitioner corporation, for want of legitimate representation, was
court or judge of the Philippines, having jurisdiction to pronounce the effectively deprived of its day in court in said case. Res inter alios judicatae nullum allis
judgment or order, may be as follows: praejudicium faciunt. Matters adjudged in a cause do not prejudice those who were not parties to
it. 32 Elsewise put, no person (natural or juridical) shall be affected by a proceeding to which he
is a stranger. 33
xxx xxx xxx

Granting arguendo, that IDP may be considered a principal party in Ligon, res judicata as a "bar
(b) In other cases the judgment or order is, with respect to the matter
by former judgment" will still not set in on the ground that the cause of action in the two cases
directly adjudged or as to any other matter that could have been raised in are different. The cause of action in G.R. No. 107751 is the surrender of the owner's duplicate
relation thereto, conclusive between the parties and their successors in copy of the transfer certificates of title to the rightful possessor thereof, whereas the cause of
interest by title subsequent to the commencement of the action or special action in the present case is the validity of the Carpizo Group-INC Deed of Absolute Sale.
proceeding, litigating for the same thing and under the same title and in the
same capacity;
Res Judicata in the form of "conclusiveness of judgment" cannot likewise apply for the reason
that any mention at all in Ligon as to the validity of the disputed Carpizo Board-INC sale may
(c) In any other litigation between the same parties or their successors in only be deemed incidental to the resolution of the primary issue posed in said case which is:
interest, that only is deemed to have been adjudged in a former judgment Who between Ligon and INC has the better right of possession over the owner's duplicate copy
which appears upon its face to have been so adjudged, or which was of the TCTs covering the IDP property? G.R. No. 107751 cannot be considered determinative
actually and necessarily included therein or necessary thereto. and conclusive on the matter of the validity of the sale for this particular issue was not the
principal thrust of Ligon. To rule otherwise would be to cause grave and irreparable injustice to
Section 49(b) enunciates the first concept of res judicata known as "bar by prior judgment," IDP which never gave its consent to the sale, thru a legitimate Board of Trustees.
whereas, Section 49(c) is referred to as "conclusiveness of judgment."
In any case, while it is true that the principle of res judicata is a fundamental component of our
There is "bar by former judgment" when, between the first case where the judgment was judicial system, it should be disregarded if its rigid application would involve the sacrifice of
rendered, and the second case where such judgment is invoked, there is identity of parties, justice to technicality. 34
subject matter and cause of action. When the three identities are present, the judgment on the
merits rendered in the first constitutes an absolute bar to the subsequent action. But where The main question though in this petition is: Did the Court of Appeals commit reversible error in
between the first case wherein judgment is rendered and the second case wherein such setting aside that portion of the SEC's Decision in SEC Case No. 4012 which declared the sale
judgment is invoked, there is only identity of parties but there is no identity of cause of action, the
of two (2) parcels of land in Quezon City between the IDP-Carpizo Group and private
judgment is conclusive in the second case, only as to those matters actually and directly respondent INC null and void?
controverted and determined, and not as to matters merely involved therein. This is what is
termed "conclusiveness of judgment." 27
We rule in the affirmative.
Neither of these concepts of res judicata find relevant application in the case at bench. While
there may be identity of subject matter (IDP property) in both cases, there is no identity of There can be no question as to the authority of the SEC to pass upon the issue as to who
parties. The principal parties in G.R. No. 107751 were mortgagee Leticia P. Ligon, as petitioner, among the different contending groups is the legitimate Board of Trustees of the IDP since this is
and the Iglesia Ni Cristo, as private respondent. The IDP, as represented by the 1971 Board of a matter properly falling within the original and exclusive jurisdiction of the SEC by virtue of
Trustees or the Tamano Group, was only made an ancillary party in G.R. No. 107751 as Sections 3 and 5(c) of Presidential Decree No. 902-A:
Sec. 3. The Commission shall have absolute jurisdiction, supervision and the existence of a contract, and where it is wanting, the contract is non-existent. 38 In
control over all corporations, partnership or associations, who are the this case, the IDP, owner of the subject parcels of land, never gave its consent, thru a
grantees of primary franchises and/or a license or permit issued by the legitimate Board of Trustees, to the disputed Deed of Absolute Sale executed in favor
government to operate in the Philippines . . . . of INC. This is, therefore, a case not only of vitiated consent, but one where consent
on the part of one of the supposed contracting parties is totally wanting. Ineluctably,
the subject sale is void and produces no effect whatsoever.
xxx xxx xxx

The Carpizo Group-INC sale is further deemed null and void ab initio because of the Carpizo
Sec. 5. In addition to the regulatory and adjudicative functions of the
Group's failure to comply with Section 40 of the Corporation Code pertaining to the disposition of
Securities and Exchange Commission over corporations, partnerships and
all or substantially all assets of the corporation:
other forms of associations registered with it as expressly granted under
existing laws and decrees, it shall have original and exclusive jurisdiction to
hear and decide cases involving: Sec. 40. Sale or other disposition of assets. — Subject to the provisions of
existing laws on illegal combinations and monopolies, a corporation may, by
a majority vote of its board of directors or trustees, sell, lease, exchange,
xxx xxx xxx
mortgage, pledge or otherwise dispose of all or substantially all of its
property and assets, including its goodwill, upon terms and conditions and
c) Controversies in the selection or appointment of directors, trustees, for such consideration, which may be money, stocks, bonds or other
officers, or managers of such corporations, partnerships or associations. . . . instruments for the payment of money or other property or consideration, as
. its board of directors or trustees may deem expedient, when authorized by
the vote of the stockholders representing at least two-thirds (2/3) of the
outstanding capital stock; or in case of non-stock corporation, by the vote of
If the SEC can declare who is the legitimate IDP Board, then by parity of reasoning, it at least two-thirds (2/3) of the members, in a stockholders' or members'
can also declare who is not the legitimate IDP Board. This is precisely what the SEC
meeting duly called for the purpose. Written notice of the proposed action
did in SEC Case No. 4012 when it adjudged the election of the Carpizo Group to the and of the time and place of the meeting shall be addressed to each
IDP Board of Trustees to be null and stockholder or member at his place of residence as shown on the books of
void. 35 By this ruling, the SEC in effect made the unequivocal finding that the IDP-
the corporation and deposited to the addressee in the post office with
Carpizo Group is a bogus Board of Trustees. Consequently, the Carpizo Group is postage prepaid, or served personally: Provided, That any dissenting
bereft of any authority whatsoever to bind IDP in any kind of transaction including the stockholder may exercise his appraisal right under the conditions provided
sale or disposition of ID property. in this Code.

It must be noted that SEC Case No. 4012 is not the first case wherein the SEC had the A sale or other disposition shall be deemed to cover substantially all the
opportunity to pass upon the status of the Carpizo Group. As far back as October 3, 1986, the corporate property and assets if thereby the corporation would be rendered
SEC, in Case No. 2687, 36 in a suit between the Carpizo Group and the Abbas Group, already incapable of continuing the business or accomplishing the purpose for which
declared the election of the Carpizo Group (as well as the Abbas Group) to the IDP Board as it was incorporated.
null and void for being violative of the Articles of Incorporation. 37 Nothing thus becomes more
settled than that the IDP-Carpizo Group with whom private respondent INC contracted is a fake
Board. xxx xxx xxx

Premises considered, all acts carried out by the Carpizo Board, particularly the sale of the The Tandang Sora property, it appears from the records, constitutes the only property of the
Tandang Sora property, allegedly in the name of the IDP, have to be struck down for having IDP. Hence, its sale to a third-party is a sale or disposition of all the corporate property and
been done without the consent of the IDP thru a legitimate Board of Trustees. Article 1318 of the assets of IDP falling squarely within the contemplation of the foregoing section. For the sale to
New Civil Code lays down the essential requisites of contracts: be valid, the majority vote of the legitimate Board of Trustees, concurred in by the vote of at least
2/3 of the bona fide members of the corporation should have been obtained. These twin
requirements were not met as the Carpizo Group which voted to sell the Tandang Sora property
There is no contract unless the following requisites concur: was a fake Board of Trustees, and those whose names and signatures were affixed by the
Carpizo Group together with the sham Board Resolution authorizing the negotiation for the sale
(1) Consent of the contracting parties; were, from all indications, not bona fide members of the IDP as they were made to appear to be.
Apparently, there are only fifteen (15) official members of the petitioner corporation including the
eight (8) members of the Board of Trustees. 39
(2) Object certain which is the subject matter of the contract;

All told, the disputed Deed of Absolute Sale executed by the fake Carpizo Board and private
(3) Cause of the obligation which is established. respondent INC was intrinsically void ab initio.

All these elements must be present to constitute a valid contract. For, where even one Private respondent INC nevertheless questions the authority of the SEC to nullify the sale for
is absent, the contract is void. As succinctly put by Tolentino, consent is essential for being made outside of its jurisdiction, the same not being an intra-corporate dispute.
The resolution of the question as to whether or not the SEC had jurisdiction to declare the
subject sale null and void is rendered moot and academic by the inherent nullity of the highly
dubious sale due to lack of consent of the IDP, owner of the subject property. No end of
substantial justice will be served if we reverse the SEC's conclusion on the matter, and remand
the case to the regular courts for further litigation over an issue which is already determinable
based on what we have in the records.

It is unfortunate that private respondent INC opposed the motion for intervention filed by the
1971 Board of Trustees in Civil Case. No. Q-90-6937, a case for Specific Performance with
Damages between INC and the Carpizo Group on the subject Deed of Absolute Sale. The
legitimate IDP Board could have been granted ample opportunity before the regional trial court
to shed light on the true status of the Carpizo Board and settled the matter as to the validity of
the sale then and there. But INC, wanting to acquire the property at all costs and threatened by
the participation of the legitimate IDP Board in the civil suit, argued for the denial of the motion
averring, inter alia, that the issue sought to be litigated by the movant is intra-corporate in nature
and outside the jurisdiction of the regional trial court. 40 As a result, the motion for intervention
was denied. When the Decision in SEC Case No. 4012 came out nullifying the sale, INC came
forward, this time, quibbling over the issue that it is the regional trial court, and not the SEC,
which has jurisdiction to rule on the validity of the sale. INC is here trifling with the courts. We
cannot put a premium on this clever legal maneuverings of private respondent which, if
countenanced, would result in a failure of justice.

Furthermore, the Court observes that the INC bought the questioned property from the Carpizo
Group without even seeing the owner's duplicate copy of the titles covering the property. This is
very strange considering that the subject lot is a large piece of real property in Quezon City
worth millions, and that under the Torrens System of Registration, the minimum requirement for
one to be a good faith buyer for value is that the vendee at least sees the owner's duplicate copy
of the title and relies upon the same. 41 The private respondent, presumably knowledgeable on
the aforesaid workings of the Torrens System, did not take heed of this and nevertheless went
through with the sale with undue haste. The unexplained eagerness of INC to buy this valuable
piece of land in Quezon City without even being presented with the owner's copy of the titles
casts very serious doubt on the rightfulness of its position as vendee in the transaction.

WHEREFORE, the petition is GRANTED. The Decision of the public respondent Court of
Appeals dated October 28, 1994 in CA-G.R. SP No. 33295 is SET ASIDE. The Decision of the
Securities and Exchange Commission dated July 5, 1993 in SEC Case No. 4012 is
REINSTATED. The Register of Deeds of Quezon City is hereby ordered to cancel the
registration of the Deed of Absolute Sale in the name of respondent Iglesia Ni Cristo, if one has
already been made. If new titles have been issued in the name of Iglesia Ni Cristo, the Register
of Deeds is hereby ordered to cancel the same, and issue new ones in the name of petitioner
Islamic Directorate of the Philippines. Petitioner corporation is ordered to return to private
respondent whatever amount has been initially paid by INC as consideration for the property
with legal interest, if the same was actually received by IDP. Otherwise, INC may run after
Engineer Farouk Carpizo and his group for the amount of money paid.

SO ORDERED.
G.R. No. 182331 April 18, 2012 DECISION

MA. CORINA C. JIAO, RODEN B. LOPEZ, FRANCISCO L. DIMAYUGA, NORMA G. DEL REYES, J.:
VALLE, MACARIO G. MARASIGAN, LANIE MARIA B. PASANA, NILO M. DE CASTRO,
ANGELITO M. BALITAAN, CESAR L. RICO, CRISPIN S. CONSTANTINO, GLENDA S.
Nature of the Case
CORPUZ, LEONILA C. TUAZON, ALFREDO S. DAZA, LORNA R. CRUZ, MARIA M.
AMBOJIA, NOEMI M. JAPOR, ANGELITO V. DANAN, GLORIA M. SALAZAR, JOHN V.
VIGILIA, ROEL D. ROBINO, WILLIAM L. ENDAYA, TERESITA M. ROMAN, ARTURO M. Before this Court is a Petition for Review on Certiorari under Rule 45 of the Rules of Court
SABALLE, AUGUSTO N. RIGOR, ALLAN O. OLANO, RODOLFO T. CABATU, NICANOR R. wherein the petitioners assail the Resolutions dated November 7, 20071 and March 26,
BRAVO, EDUARDO M. ALCANTARA, FELIPE F. OCAMPO, ELPIDIO C. ADALIA, RENATO 2008,2 respectively, of the Court of Appeals (CA) in CA-G.R. SP No. 101065.
M. CRUZ, JOSE C. PEREZ, JR., FERNANDO V. MAPILE, ROMEO R. PATRICIO,
FERNANDO N. RONGAVILLA, FERMIN A. COBRADOR, ANTONIO O. BOSTRE, RALPH M.
MICHAELSON, CRISTINA G. MANIO, EDIGARDO M. BAUTISTA, CYNTHIA C. SANIEL, Antecedent Facts
PRISCILLA F. DAVID, MACARIO V. ARNEDO, NORLITO V. HERNANDEZ, ALFREDO G.
BUENAVENTURA, JOSE R. CASTRONUEVO, OLDERICO M. AGORILLA, CESAR M. The petitioners were regular employees of the Philippine Banking Corporation (Philbank), each
PEREZ, RONALD M. GENER, EMMANUEL G. QUILAO, BENJAMIN C. CUBA, EDGARDO S. with at least ten years of service in the company.3 Pursuant to its Memorandum dated August
MEDRANO, GODOFREDO D. PATENA, VIRGILIO G. ILAGAN, MYRNA C. LEGASPI, 28, 1970, Philbank established a Gratuity Pay Plan (Old Plan) for its employees. The Old Plan
ELIZABETH P. REYES, ANTONIO A. TALON, ROMEO P. CRUZ, ELEANOR T. TAN, provided:
FERDINAND G. PINAUIN, MA. OLIVETTE A. NAKPIL, GILBERT NOVIEM A. COLUMNA,
ARTHUR L. ABELLA, BENJAMIN L. ENRIQUEZ, ANTONINO P. QUEVEDO, ADFEL
GEORGE MONTEMAYOR, RAMON S. VELASCO, WILFREDO M. HALILI, ANTONIO M. 1. Any employee who has reached the compulsory retirement age of 60 years, or who wishes to
LUMANGLAS, ANDREW M. MAGNO, SONNY S. ESTANISLAO, RODOLFO S. ALABASTRO, retire or resign prior to the attainment of such age or who is separated from service by reason of
MICAH B. MARALIT, LINA M. QUEBRAL, REBECCA R. NARCISO, RONILO T. TOLENTINO, death, sickness or other causes beyond his/her control shall for himself or thru his/her heirs file
RUPERTO B. LETAN, JR., MEDARDO A. VASQUEZ, VALENTINA A. SANTIAGO, RODELO with the personnel office an application for the payment of benefits under the plan[.] 4
S. DIAZ, JOHN O. CORDIAL, EDWIN J. ANDAYA, RODRIGO M. MOJADO, GERMAN L.
ESTRADA, BENJAMIN B. DADUYA, MARLYN A. MUNOZ, MARIVIC M. DIONISIO, CESAR Section 1 laid down the benefits to which the employee would be entitled, to wit:
M. FLORES, JACINTO T. GUINTO, JR., BELEN C. SALAVERRIA, EVELYN M. ANZURES,
GLORIA D. ABELLA, LILIAN V. BUNUAN, MA. CONCEPCION G. UBIADAS, ROLANDO I.
CAMPOSANO, MONICO R. GOREMBALEM, ELADIO M. VICENCIO, AMORSOLO B. Section 1
BELTRAN, LEOPOLDO B. JUAREZ, NEPHTALI V. SALAZAR, SANGGUNI P. ROQUE, ROY
O. SAPANGHILA, MELVIN A. DEVEZA, CARMENCITA D. ABELLA, PRIMITIVO S. AGUAS, Benefits
JOSE MA. ANTONIO I. BUGAY, HILARIO P. DE GUZMAN, WILLIAM C. VENTIGAN, NOEL L.
AMA, ROMEO G. USON, RAOUL E. VELASCO, FLORENCIO B. PAGSALIGAN, RUBEN C.
CRUZ, ANGELA D. CUSTODIO, NOEL C. CABEROY, GUILLERMO V. GAVINO, JR., 1.1 The gratuity pay of an employee shall be an amount equivalent to one-month
GAUDENCIO P. BESA, AIDA M. PADILLA, ROWENA M. BAUYON, HENRY C. EPISCOPE, salary for every year of credited service, computed on the basis of last salary
ALVIN T. PATRIARCA, EUSTAQUIO C. AQUINO, JR., VALENTINO T. ARELLANO, received.
REYNALDO J. AUSTRIA, BAYANI A. CUNANAN, EFREN T. JOSE, EDUARDO P. LORIA,
REYNALDO M. PORTILLO, ARMANDO B. DUPAYA, SESINANDO S. GOMEZ, BRICCIO B. 1.2 An employee with credited service of 10 years or more, shall be entitled to and
GAFFUD III, DANILO N. PALO, MARIO F. SOLANO, MARIANITO B. GOOT and ELSA S. paid the full amount of the gratuity pay, but in no case shall the gratuity pay exceed
TANGO, ZENAIDA N. GARIN, RUBY L. TEJADA, JOEL B. GARCIA, MA. RUBY L. JIMENEA, the equivalent of 24 months, or two years, salary.5
ARLENE L. MADLANGBAYAN, ROCELY P. MARASIGAN, MA. ROSARIO H. RIVERA,
OSCAR G. BARACHINA, EDITA M. REMO, ROBERTO P. ENDAYA, ALELI B. ALANO,
FRANCISCO T. MENEZ, CAMILO N. CARILLO, ROSEMARIE A. DOMINGO, LYNDON D. On March 8, 1991, Philbank implemented a new Gratuity Pay Plan (New Gratuity Plan). 6 In
ENOROBA, MERLY H. JAVELLANA, HERNES M. MANDABON, LUZ G. ONG, GILBERTO B. particular, the New Gratuity Plan stated thus:
PICO, CRISPIN A. TAMAYO, RICARDO C. VERNAIZ, RENATO V. SACRAMENTO,
CLODUALDO O. GOMEZ, MARINEL O. ALPINO, ELY P. RAMOS, NICANOR E. REYES, x x x An Employee who is involuntarily separated from the service by reason of death, sickness
JR.,Petitioners, or physical disability, or for any authorized cause under the law such as redundancy, or other
vs. causes not due to his own fault, misconduct or voluntary resignation, shall be entitled to either
NATIONAL LABOR RELATIONS COMMISSION, GLOBAL BUSINESS BANK, INC., one hundred percent (100%) of his accrued gratuity benefit or the actual benefit due him under
CORPORATE OFFICERS OF GLOBAL BANK: ROBIN KING, HENRY M. SUN, BENJAMIN G. the Plan, whichever is greater.7
CHUA, JR., JOVENCIO F. CINCO, EDWARD S. GO, MARY VY TY, TAKANORI NAKANO,
JOHN K.C. NG, FLORENCIO T. MALLARE, EDMUND/EDDIE GAISANO, FRANCISCO
SEBASTIAN, SAMUEL S. YAP, ALFRED VY TY, GEN TOMII, CHARLES WAI-BUN CHEUNG In February 2000, Philbank merged with Global Business Bank, Inc. (Globalbank), with the
and METROPOLITAN BANK AND TRUST COMPANY, Respondents. former as the surviving corporation and the latter as the absorbed corporation, but the bank
operated under the name Global Business Bank, Inc. As a result of the merger, complainants’
respective positions became redundant. A Special Separation Program (SSP) was implemented
and the petitioners were granted a separation package equivalent to one and a half month’s pay
(or 150% of one month’s salary) for every year of service based on their current salary. Before gratuity plan after finding that its enactment was not attended by bad faith or any design to
the petitioners could avail of this program, they were required to sign two documents, namely, an defraud complainants. Thus, the New Gratuity Plan must be deemed to have superseded the
Acceptance Letter and a Release, Waiver, Quitclaim (quitclaim). 8 Old Plan.19 The LA also ruled that the minimum amount due to the petitioners under the New
Gratuity Plan, in relation to Article 283 of the Labor Code was one month’s pay for every year of
service. Thus, anything over that amount was discretionary.
As their positions were included in the redundancy declaration, the petitioners availed of the
SSP, signed acceptance letters and executed quitclaims in Globalbank’s favor9 in consideration
of their receipt of separation pay equivalent to 150% of their monthly salaries for every year of As to the validity of the quitclaim, the LA held that the issue has been rendered moot.
service. Nonetheless, the LA upheld the petitioners’ undertaking under their respective quitclaims,
considering the amount involved is not unconscionable, and that their supposed lack of complete
understanding did not mean that they were coerced or deceived into executing the same. 20
In August 2002, respondent Metropolitan Bank and Trust Company (Metrobank) acquired the
assets and liabilities of Globalbank through a Deed of Assignment of Assets and Assumption of
Liabilities.10 The LA also absolved Metrobank from liability. The LA found that the petitioners had already
been separated from Globalbank when Metrobank took over the former’s banking operations.
Moreover, the liabilities that Metrobank assumed were limited to those arising from banking
Subsequently, the petitioners filed separate complaints for non-payment of separation pay with
operations and excluded those pertaining to Globalbank’s employees or to claims of previous
prayer for damages and attorney’s fees before the National Labor Relations Commission
employees.21
(NLRC).11

The NLRC’s Decision


The petitioners asserted that, under the Old Plan, they were entitled to an additional 50% of their
gratuity pay on top of 150% of one month’s salary for every year of service they had already
received. They insisted that 100% of the 150% rightfully belongs to them as their separation pay. Aggrieved, the petitioners appealed to the NLRC. In a decision22 dated August 15, 2007, the
Thus, the remaining 50% was only half of the gratuity pay that they are entitled to under the Old NLRC dismissed the appeal and affirmed the LA’s decision.
Plan. They argued that even if the New Gratuity Plan were to be followed, the computation
would be the same, since Section 10.1 of the New Gratuity Plan provided that:
The NLRC held that the petitioners did not acquire a vested right to Philbank’s gratuity plans
since, at the outset, it was made clear that these plans would not perpetuate into eternity. It also
10.1 Employees who have attained a regular status as of March 8, 1991 who are covered by the noted that, under the SSP, the employee to be separated due to redundancy would be receiving
Old Gratuity Plan and are now covered by this Plan shall be entitled to which is the higher more than the rate in the old plan and higher than the legal rate for the separated employees.
benefit between the two Plans. Double recovery from both plans is not allowed. 12
The petitioners elevated the case to the CA via a Petition for Certiorari under Rule 65.
The petitioners further argued that the quitclaims they signed should not bar them from claiming
their full entitlement under the law. They also claimed that they were defrauded into signing the
The CA’s Decision
same without full knowledge of its legal implications.13

In the first of the assailed CA resolutions, the CA ruled that the petition was dismissible outright
On the other hand, Globalbank asserted that the SSP should prevail and the petitioners were no
for failure of the petitioners to file a motion for reconsideration of the decision under review
longer entitled to the additional 50% gratuity pay which was already paid, the same having been
before resorting to certiorari. Further, the CA held that the case did not fall under any of the
included in the computation of their separation pay. It maintained further that the waivers
recognized exceptions to the rule on motions for reconsideration.23
executed by the petitioners should be held binding, since these were executed in good faith and
with the latter’s full knowledge and understanding.14
The petitioners then moved for the reconsideration, which was denied in the second assailed
Resolution, noting the absence of an explanation for their failure to file a motion for
Meanwhile, Metrobank denied any liability, citing the absence of an employment relationship
reconsideration of the assailed NLRC decision in their petition for certiorari. 24
with the petitioners. It argued that its acquisition of the assets and liabilities of Globalbank did not
include the latter’s obligation to its employees. Moreover, Metrobank pointed out that the
petitioners’ employment with Globalbank had already been severed before it took over the The Issues
latter’s banking operations.15
The petitioners are now before this Court raising the following errors supposedly committed by
The Labor Arbiter’s Decision the CA:

On August 30, 2004, the Labor Arbiter (LA) promulgated a decision16 dismissing the 1. In dismissing the petition for failure to file a motion for reconsideration before filing a
complaint.17 The LA ruled that the petitioners were not entitled to the additional 50% in gratuity petition under Rule 65 as it blatantly ignored the application of the recent
pay that they were asking for.18 jurisprudence on labor law.

The LA held that the 150% rate used by Globalbank could legally cover both the separation pay 2. In dismissing the petition without taking into consideration the meritorious grounds
and the gratuity pay of complainants. The LA upheld the right of the employer to enact a new laid down by [the] petitioners by categorically outlining the grave abuse of discretion
amounting to lack or excess of jurisdiction committed by [the] NLRC in affirming the The petitioners claim that it was error for the CA to have dismissed their petition on the sole
decision of the Labor Arbiter, to wit: basis thereof. According to the petitioners, they had opted not to file a motion for reconsideration
as the issues that will be raised therein are those that the NLRC had already passed upon. The
petitioners likewise invoke the liberal application of procedural rules.
2.a. In holding that [the] petitioners "did not acquire a vested right under the
PHILBANK gratuity plan."
To begin with, the petitioners do not have the discretion or prerogative to determine the propriety
of complying with procedural rules. This Court had repeatedly emphasized in various cases
2.b. In holding that "the bank had abandoned the old plan" (referring to the
involving the tedious attempts of litigants to relieve themselves of the consequences of their
old Gratuity Pay Plan) and replaced it with a Special Separation Program
neglect to follow a simple procedural requirement for perfecting a petition for certiorari that he
under which [the] petitioners "would be receiving more than the rate in the
who seeks a writ of certiorari must apply for it only in the manner and strictly in accordance with
old plan and higher than the legal rate for redundant employees."
the provisions of the law and the Rules. The petitioners may not arrogate to themselves the
determination of whether a motion for reconsideration is necessary or not. To dispense with the
2.c. In holding that the benefits under the Special Separation Program requirement of filing a motion for reconsideration, the petitioners must show a concrete,
legally replaced not only the gratuity pay plan to which [the] petitioners were compelling, and valid reason for doing so.26
entitled under the old and new Gratuity Pay Plans but also all other benefits
including separation pay under the law.
As the CA correctly noted, the petitioners did not bother to explain their omission and only did so
in their motion for reconsideration of the dismissal of their petition. Aside from the fact that such
2.d. In not holding that when [the] petitioners were separated due to belated effort will not resurrect their application for a writ of certiorari, the reason proffered by the
redundancy they were entitled per provision of Article 283 of the Labor Code petitioners does not fall under any of the recognized instances when the filing of a motion for
to separation pay equivalent to one month pay for every year of service. reconsideration may be dispensed with. Whimsical and arbitrary deviations from the rules cannot
be condoned in the guise of a plea for a liberal interpretation thereof. We cannot respond with
alacrity to every claim of injustice and bend the rules to placate vociferous protestors crying and
2.e. In holding that [the] petitioners are bound under the Acceptance x x x
claiming to be victims of a wrong.27
and Release, Waiver and Quitclaim x x x that they had executed and
[cannot] question the same, hence they [cannot] claim benefits in addition to
those they had received from the bank. We now rule on the substantive issues.

2.f. In not holding that respondent METROBANK is the parent corporation of The petitioners’ receipt of separation pay equivalent to their one and a half months salary for
GLOBALBANK and the latter is the subsidiary, hence METROBANK is every year of service as provided in the SSP and the New Gratuity Plan more than sufficiently
liable for the payment of the employment benefits of [the] petitioners as it complies with the Labor Code, which only requires the payment of separation pay at the rate of
had acquired all the assets of GLOBALBANK. one month salary for every year of service.

2.g. In not holding that the Assignment of Assets and Liabilities x x x The petitioners do not question the legality of their separation from the service or the basis for
executed by GLOBALBANK and METROBANK is a scheme to defraud [the] holding their positions redundant. What they raise is their entitlement to gratuity pay, as provided
petitioners of the employment benefits due them upon separation from in the Old Plan, in addition to what they received under the SSP. According to the petitioners,
service. they are entitled to separation pay at a rate of one month salary for every year of service under
the Labor Code and gratuity pay at a rate of one month salary for every year of service whether
under the Old Plan or the New Gratuity Plan. Since what they received as separation pay was
2.h. In not holding that [the] respondents are liable to [the] petitioners for equivalent to only 150% or one and one-half of their monthly salaries for every year of service,
moral, exemplary and temperate damages because [the] respondents are the respondents are still liable to pay them the deficiency equivalent to one-half of their monthly
guilty of deceit and fraud in not paying [the] petitioners the full amount of
salary for every year of service.
their employment benefits.25

We disagree.
The Court’s Decision

The New Gratuity Plan has


The Petition has no merit, hence, must be denied. repealed the Old Plan.

The petitioners’ unexplained failure to move for the reconsideration of the NLRC’s resolution It is clear from the provisions of Section 8 of the New Gratuity Plan that the Old Plan has been
before applying for a writ of certiorari in the CA is reason enough to deny such application. revoked or superseded. Thus:

We shall first discuss the procedural issue raised by the petitioners: whether the CA erred in SECTION 8
dismissing their petition due to their failure to file a motion for reconsideration of the NLRC’s INTEGRATION OF SOCIAL LEGISLATION, CONTRACTS, ETC.
adverse resolution.
8.1 This Plan is not intended to duplicate or cause the double payment of similar or analogous A direct reference to the New Gratuity Plan reveals the contrary. The above-quoted Section 8 of
benefits provided for under existing labor and social security laws. Accordingly, benefits under the New Gratuity Plan expressly states that "the benefits under this Plan shall be deemed
this Plan shall be deemed integrated with and in lieu of (i) statutory benefits under the New integrated with and in lieu of (i) statutory benefits under the New Labor Code and Social Security
Labor Code and Social Security Laws, as now or hereafter amended[;] and (ii) analogous Laws, as now or hereafter amended" and that "[t]his Plan is not intended to duplicate or cause
benefits granted under present or future collective bargaining agreements, and other employee the double payment of similar or analogous benefits provided for under existing labor and
benefit plans providing analogous benefits which may be imposed by future legislations. In the security laws."
event the benefits due under the Plan are less than those due and demandable under the
provisions of the New Labor Code and/or present or future Collective Bargaining Agreements
Article 283 of the Labor Code30 provides only the required minimum amount of separation pay,
and/or future plans of similar nature imposed by law, the Fund shall respond for the difference. 28
which employees dismissed for any of the authorized causes are entitled to receive. Employers,
therefore, have the right to create plans, providing for separation pay in an amount over and
Globalbank’s right to replace the Old Plan and the New Gratuity Plan is within legal bounds as above what is imposed by Article 283. There is nothing therein that prohibits employers and
the terms thereof are in accordance with the provisions of the Labor Code and complies with the employees from contracting on the terms of employment, or from entering into agreements on
minimum requirements thereof. Contrary to the petitioners’ claim, they had no vested right over employee benefits, so long as they do not violate the Labor Code or any other law, and are not
the benefits under the Old Plan considering that none of the events contemplated thereunder contrary to morals, good customs, public order, or public policy. 31 As this Court held in a case:
occurred prior to the repeal thereof by the adoption of the New Gratuity Plan. Such right accrues
only upon their separation from service for causes contemplated under the Old Plan and the
[E]ntitlement to benefits consequent thereto are not limited to those provided by said provision of
petitioners can only avail the benefits under the plan that is effective at the time of their
law. Otherwise, the provisions of collective bargaining agreements, individual employment
dismissal. In this case, when the merger and the redundancy program were implemented, what
contracts, and voluntary retirement plans of companies would be rendered inutile if we were to
was in effect were the New Gratuity Plan and the SSP; the petitioners cannot, thus, insist on the
limit the award of monetary benefits to an employee only to those provided by statute. x x x.32
provisions of the Old Plan which is no longer existent.

Previously, the Court adopted the CA’s ruling, upholding the validity of a similar provision in a
The SSP did not revoke or supersede the New Gratuity Plan.
company’s retirement plan:

On the other hand, the issuance of the SSP did not result to the repeal of the New Gratuity Plan.
[T]here is no further doubt that the payment of separation pay is a requirement of the law, i.e.[,]
As the following provision of the SSP shows, the terms of the New Gratuity Plan had been
the Labor Code, which is a social legislation. The clear intent of Article XI, section 6 [of the
expressly incorporated in the SSP and should, thus, be implemented alongside the SSP:
Retirement Plan] is to input the effects of social legislation in the circulation of Retirement
benefits due to retiring employees x x x. The Retirement Plan itself clearly sets forth the intention
II. Separation Pay Package of the parties to entitle employees only to whatever is greater between the Retirement Benefits
then due and that which the law requires to be given by way of separation pay. To give way to
complainant’s demands would be to totally ignore the contractual obligations of the parties in the
Affected employees are entitled to the following tax free:
Retirement Plan, and to distort the clear intent of the parties as expressed in the terms and
conditions contained in such plan. x x x.33(emphasis supplied)
a. Gratuity Benefits which they are entitled to under the respective retirement plans. The bank
shall give a premium by rounding up the benefit to an equivalent of 1.5 months salary per every
Consequently, if the petitioners were allowed to receive separation pay from both the Labor
year of service based on their salary as of separation date.29 (emphasis supplied)
Code, on the one hand, and the New Gratuity Plan and the SSP, on the other, they would
receive double compensation for the same cause (i.e., separation from the service due to
The SSP was not intended to supersede the New Gratuity Plan. On the contrary, the SSP was redundancy) even if such is contrary to the provisions of the New Gratuity Plan. The petitioners’
issued to make the benefits under the New Gratuity Plan available to employees whose claim of being shortchanged is certainly unfounded. They have recognized the validity of the
positions had become redundant because of the merger between Philbank and Globalbank, SSP and the New Gratuity Plan as evidenced by the acceptance letters and quitclaims they
subject to compliance with certain requirements such as age and length of service, and to executed; and the benefits they received under the SSP and the New Gratuity Plan are more
improve such benefits by increasing or rounding it up to an amount equivalent to the affected than what is required by the Labor Code.
employees’ one and a half monthly salary for every year of service. In other words, the benefits
to which the redundated employees are entitled to, including the petitioners, are the benefits
In the absence of proof that any of the vices of consent are present, the petitioners’ acceptance
under the New Gratuity Plan, albeit increased by the SSP.
letters and quitclaims are valid; thus, barring them from claiming additional separation pay.

Considering that the New Gratuity Plan still stands and has not been revoked by the SSP, does
The Court now comes to the issue on the validity of the acceptance letters and quitclaims that
this mean that the petitioners can claim the benefits thereunder in addition to or on top of what is
the petitioners executed, which they claim do not preclude them from asking for the benefits
required under the Article 283 of the Labor Code?
rightfully due them under the law.

For as long as the minimum requirements of the Labor Code are met, it is within the
It is true that quitclaims executed by employees are often frowned upon as contrary to public
management prerogatives of employers to come up with separation packages that will be given
policy.34 Hence, deeds of release or quitclaims cannot bar employees from demanding benefits
in lieu of what is provided under the Labor Code.
to which they are legally entitled or from contesting the legality of their dismissal. The
acceptance of those benefits would not amount to estoppel. 35
However, the Court, in other cases, has upheld quitclaims if found to comply with the following What the petitioners desire, the Court cannot do. This fiction of corporate entity can only be
requisites: (1) the employee executes a deed of quitclaim voluntarily; (2) there is no fraud or disregarded in cases when it is used to defeat public convenience, justify wrong, protect fraud,
deceit on the part of any of the parties; (3) the consideration of the quitclaim is credible and or defend crime. Moreover, to justify the disregard of the separate juridical personality of a
reasonable; and (4) the contract is not contrary to law, public order, public policy, morals or good corporation, the wrongdoing must be clearly and convincingly established. 41
customs or prejudicial to a third person with a right recognized by law. 36
In the instant case, none of these circumstances is present such as to warrant piercing the veil
In this case, there is no allegation of fraud or deceit employed by the respondents in making the of corporate fiction and treating Globalbank and Metrobank as one.1âwphi1
petitioners sign the acceptance letters and quitclaims. Neither was there any claim of force or
duress exerted upon the petitioners to compel them to sign the acceptance letters and
Lastly, the petitioners’ prayer for the award of damages must be denied for lack of legal basis.
quitclaims. Likewise, the consideration is credible and reasonable since the petitioners are
getting more than the amount required under the law. Thus, the acceptance letters and
quitclaims executed by the petitioners are valid and binding. In sum, the New Gratuity Plan and SSP are valid and must be given effect, inasmuch as their
provisions are not contrary to law; and, indeed, grant benefits that meet the minimum amount
required by the Labor Code. The petitioners have voluntarily sought such benefits and upon their
Considering that the petitioners have already waived their right to file an action for any of their
receipt thereof, executed quitclaims in Globalbank’s favor. The petitioners cannot, upon a mere
claims in relation to their employment with Globalbank, the question of whether Metrobank can
change of mind, seek to invalidate such quitclaims and renege on their undertaking thereunder,
be held liable for these claims is now academic. However, in order to put to rest any doubt in the
which, to begin with, is supported by a substantial consideration and which they had knowingly
petitioners’ minds as to Metrobank’s liabilities, we shall proceed to discuss this issue.
assumed and imposed upon themselves.

We hold that Metrobank cannot be held liable for the petitioners’ claims.
WHEREFORE, the foregoing premises considered, the petition is DENIED. The assailed
Resolutions dated November 7, 2007 and March 26, 2008, respectively, of the Court of Appeals
As a rule, a corporation that purchases the assets of another will not be liable for the debts of the in CA-G.R. SP No. 101065 are AFFIRMED.
selling corporation, provided the former acted in good faith and paid adequate consideration for
such assets, except when any of the following circumstances is present: (1) where the purchaser
SO ORDERED.
expressly or impliedly agrees to assume the debts; (2) where the transaction amounts to a
consolidation or merger of the corporations; (3) where the purchasing corporation is merely a
continuation of the selling corporation; and (4) where the selling corporation fraudulently enters
into the transaction to escape liability for those debts.37

Under the Deed of Assignments of Assets and Assumption of Liabilities 38 between Globalbank
and Metrobank, the latter accepted the former’s assets in exchange for assuming its liabilities.
The liabilities that Metrobank assumed, which were clearly set out in Annex "A" of the
instrument, are: deposit liabilities; interbank loans payable; bills payable; manager’s checks and
demand drafts outstanding; accrued taxes, interest and other expenses; and deferred credits
and other liabilities.39

Based on this enumeration, the liabilities that Metrobank assumed can be characterized as
those pertaining to Globalbank’s banking operations. They do not include Globalbank’s liabilities
to pay separation pay to its former employees. This must be so because it is understood that the
same liabilities ended when the petitioners were paid the amounts embodied in their respective
acceptance letters and quitclaims. Hence, this obligation could not have been passed on to
Metrobank.

The petitioners insist that Metrobank is liable because it is the "parent" company of Globalbank
and that majority of the latter’s board of directors are also members of the former’s board of
directors.

While the petitioners’ allegations are true, one fact cannot be ignored – that Globalbank has a
separate and distinct juridical personality. The petitioners’ own evidence – Global Business
Holdings, Inc.’s General Information Sheet40filed with the Securities and Exchange Commission
– bears this out.

Even then, the petitioners would want this Court to pierce the veil of corporate identity in order to
hold Metrobank liable for their claims.
G.R. No. L-20850 November 29, 1965 29, 1958) in which said judgment was rendered. Moreover, appellee purchased the shares of
stock of Insular Farms as the highest bidder at an auction sale held at the instance of a bank to
which said shares had been pledged as security for an obligation of Insular Farms in favor of
THE EDWARD J. NELL COMPANY, petitioner,
said bank. It has, also, been established that the appellee had paid P285,126.99 for said shares
vs.
of stock, apart from the sum of P10,000.00 it, likewise, paid for the other assets of Insular
PACIFIC FARMS, INC., respondent.
Farms.

CONCEPCION, J.:
Neither is it claimed that these transactions have resulted in the consolidation or merger of the
Insular Farms and appellee herein. On the contrary, appellant's theory to the effect that appellee
Appeal by certiorari, taken by Edward J. Nell Co. — hereinafter referred to as appellant — from is an alter ego of the Insular Farms negates such consolidation or merger, for a corporation
a decision of the Court of Appeals. cannot be its own alter ego.

On October 9, 1958, appellant secured in Civil Case No. 58579 of the Municipal Court of Manila It is urged, however, that said P10,000.00 paid by appellee for other assets of Insular Farms is a
against Insular Farms, Inc. — hereinafter referred to as Insular Farms a judgment for the sum of grossly inadequate price, because, appellant now claims, said assets were worth around
P1,853.80 — representing the unpaid balance of the price of a pump sold by appellant to Insular P285,126.99, and that, consequently, the sale must be considered fraudulent. However, the sale
Farms — with interest on said sum, plus P125.00 as attorney's fees and P84.00 as costs. A writ was submitted to and approved by the Securities and Exchange Commission. It must be
of execution, issued after the judgment had become final, was, on August 14, 1959, returned presumed, therefore, that the price paid was fair and reasonable. Moreover, the only issue
unsatisfied, stating that Insular Farms had no leviable property. Soon thereafter, or on November raised in the court of origin was whether or not appellee is an alter ego of Insular Farms. The
13, 1959, appellant filed with said court the present action against Pacific Farms, Inc. — question of whether the aforementioned sale of assets for P10,000.00 was fraudulent or not, had
hereinafter referred to as appellee — for the collection of the judgment aforementioned, upon the not been put in issue in said court. Hence, it may, not be raised on appeal.
theory that appellee is the alter ego of Insular Farms, which appellee has denied. In due course,
the municipal court rendered judgment dismissing appellant's complaint. Appellant appealed,
Being a mere consequence of the first assignment of error, which is thus clearly untenable,
with the same result, to the court of first instance and, subsequently, to the Court of Appeals.
appellant's second assignment of error needs no discussion.
Hence this appeal by certiorari, upon the ground that the Court of Appeals had erred: (1) in not
holding the appellee liable for said unpaid obligation of the Insular Farms; and (2) in not granting
attorney's fees to appellant. WHEREFORE, the decision appealed from is hereby affirmed, with costs against the appellant.
It is so ordered.
With respect to the first ground, it should be noted that appellant's complaint in the municipal
court was anchored upon the theory that appellee is an alter ego of Insular Farms, because the
former had purchased all or substantially all of the shares of stock, as well as the real and
personal properties of the latter, including the pumping equipment sold by appellant to Insular
Farms. The record shows that, on March 21, 1958, appellee purchased 1,000 shares of stock of
Insular Farms for P285,126.99; that, thereupon, appellee sold said shares of stock to certain
individuals, who forthwith reorganized said corporation; and that the board of directors thereof,
as reorganized, then caused its assets, including its leasehold rights over a public land in
Bolinao, Pangasinan, to be sold to herein appellee for P10,000.00. We agree with the Court of
Appeals that these facts do not prove that the appellee is an alter ego of Insular Farms, or is
liable for its debts. The rule is set forth in Fletcher Cyclopedia Corporations, Vol. 15, Sec. 7122,
pp. 160-161, as follows:

Generally where one corporation sells or otherwise transfers all of its assets to
another corporation, the latter is not liable for the debts and liabilities of the transferor,
except: (1) where the purchaser expressly or impliedly agrees to assume such debts;
(2) where the transaction amounts to a consolidation or merger of the corporations; (3)
where the purchasing corporation is merely a continuation of the selling corporation;
and (4) where the transaction is entered into fraudulently in order to escape liability for
such debts.

In the case at bar, there is neither proof nor allegation that appellee had expressly or impliedly
agreed to assume the debt of Insular Farms in favor of appellant herein, or that the appellee is a
continuation of Insular Farms, or that the sale of either the shares of stock or the assets of
Insular Farms to the appellee has been entered into fraudulently, in order to escape liability for
the debt of the Insular Farms in favor of appellant herein. In fact, these sales took place (March,
1958) not only over six (6) months before the rendition of the judgment (October 9, 1958) sought
to be collected in the present action, but, also, over a month before the filing of the case (May

S-ar putea să vă placă și